Está en la página 1de 778

NUTRICION INFANTIL

Multiple Choice
Identify the choice that best completes the statement or answers the question.

____ 1. La etiología de la anemia ferropénica incluye, señale la incorrecta:


a. Absorción insuficiente c. Aumento de requerimientos
b. Pérdidas aumentadas d. Hemolisis
____ 2. las manifestaciones neuromusculares de la anemia ferropénica incluye:
a. Sensación de mareo y vértigo c. Cefalea y visión borrosa.
b. Cansancio precoz d. Todas las anteriores

Multiple Response
Identify one or more choices that best complete the statement or answer the question.

____ 3. Que datos podemos encontrar en el hemograma, en la anemia ferropénica?


a. Recuento de reticulocitos bajo c. Recuento de plaquetas bajo
b. Recuento de leucocitos bajo d. Hemoglobina y hematocrito: disminuidos
____ 4. La dosificación de sulfato ferroso en el tratamiento de la anemia es de:
a. 3 a 6 mg/kg/día c. 10 mg/kg/día
b. 1 a 2 mg/kg/día d. 30 mg/kg/día
____ 5. La absorción del hierro disminuye con la administración de: Señale la incorrecta
a. Bloqueadores H2 c. Multivitamínicos con sales de calcio
b. Leche d. ácido ascórbico
ESPECIALIDAD QUIRURGICA
ONCOLOGIA

MULTIPLE CHOICE

1. Ca DE CUELLO UTERINO: PatologÍa Oncológica de mayor relevancia epidemiológica


en estos últimos 50 años. Las campañas masivas de prevención primaria y secundaria
están cumpliendo su objetivo: Hay disminución de la incidencia de Ca Infiltrante y
tenemos mas Diagnósticos de lesiones Preinvasivas. PREGUNTA: INFORME
CITOLOGICO LIE DE ALTO GRADO: CUAL ES SU CONDUCTA:
a. Repetir el Papanicolaou.
b. Conducta expectante porque No es cáncer.
c. Colposcopía mas Biopsia.
d. Histerectomia Simple.

ANS: C

La Colposcopia y Biopsia son métodos Diagnósticos de Confirmación.

2.
CARCINOGENESIS: SON LAS MUTACIONES DE GENES (protooncogenes-genes
supresores-genes reparadores del ADN) QUE CONTROLAN LA PROLIFERACION
CELULAR QUE SE PRODUCE EN FORMA ACUMULATIVA Y SECUENCIAL
HASTA LA APARICION DE LA NEOPLASIA MALIGNA. PREGUNTA: VIAS
DE LA CARCINOGENESIS.

a. Importante conocimiento del ciclo celular.


b. Iniciación, Promoción, Progresión.
c. Mitosis, Apoptosis, Metastasis.
d. Mitosis, Apoptosis. Necrosis.

ANS: B
La Carcinogénesis las vías estas descritas: Iniciación, Promoción, Progresión.

3.
LA QUIMIOTERAPIA FORMA PARTE DEL TRATAMIENTO MULTIDISCIPLINARIO
ONCOLOGICO QUIMIOTERAPIA ADYUVANTE ES: EXCEPTO
a. Después del tratamiento locoregional con Cirugía.
b. Controla la enfermedad micrometastásica.
c. De preferencia la Perfusión en por vía venosa.
d. Es el primer tratamiento locoregional.

ANS: D
Quimioterapia adyuvante tratamiento que se indica despues de tratamiento con cirugia o
radioterapia.

4.

Paciente de 55 años de edad: TUMOR de 4 cts cuadrante supero externo de mama


derecha. axila negativa. (biopsia trucut) Histopatológico: Carcinoma Ductal
infiltrante moderadamente diferenciado. INMUNOHISTOQUIMICA: Receptor estrógeno
Positivo 50%. Ki 67 Positivo 100%. Her 2 Neu Positivo: +. EXAMENES DE
EXTENSION NEGATIVOS. PREGUNTA el T.N.M. es:

a. T1- N0. M0
b. T2. N0. M0.
c. T3. N0. M0.
d. T4. N1. M0

ANS: B

En el Tumor de Mama el T2 es el tumor que tiene un tamaño de mas de 2


centímetros y menos de 5 centímetros.

5. CANCER GASTRICO: problema de salud pública. En el Ecuador en el sexo


masculino segundo lugar de incidencia después de Ca de Prostata con una tasa
estandarizada de 21.6 x 100.000, en el sexo femenino el cuarto lugar en incidencia. Y
como causa de muerte primer lugar. Todo es correcto EXCEPTO.
a. Gastritis atrófica con metaplasia intestinal es un factor predisponente.
b. Intestinal y difuso es la clasificación según Lauren.
c. Histológicamente la presentación de células en anillo de sello no es de mal
pronóstico
d. La endoscopía es un método diagnóstico.

ANS: C
Histologicamente las celulas en anillo de sello siempre es de mal pronostico

6. Cáncer de TIROIDES: Paciente de sexo femenino, 38 años de edad en un chequeo


de rutina un ECO DE TIROIDES reporta nodulo de 0,8 cts en lóbulo derecho.
TIRADS 4b. CUAL ES SU CONDUCTA DIAGNÓSTICA.
a. P.A.A.F.
b. Dosificacion de Tiroglobulina.
c. Resonancia Magnetica de cuello.
d. Laringoscopia.
e. Todas
ANS: A
La Conducta Diagnóstica de acuerdo a las guías Oncológicas es la Punción con
aguja fina P.A.A.F que confirma el Diagnóstico.

7. Paciente 65 años de edad. presenta SINTOMAS Irritativos-Obstructivos de la


micción evolusion 1 año. PSA TOTAL: 40 ng/ml. Biopsia prostática. Histopatológico:
adenocarcinoma acinar. GLEASON 8 (5+3). TAC s/c torax abdomen pelvis.
NEGATIVA para mts. GAMMAGRAFIA osea POSITIVA en huesos iliacos, lumbar
IV-V. GUIAS ONCOLOGICA DE TRATAMIENTO.

a. PROSTATECTOMIA RADICAL GUIADO POR ROBOT.


b. ORQUIECTOMIA BILATERAL.
c. RADIOTERAPIA CON LA TECNICA DE IMRT
d. BRAQUITERAPIA.
e. NINGUNA POR QUE ES ESTADIO IV
ANS: B
De acuerdo a esta Historia Clínica es un Estadio IV y las Guías Oncológicas en este
estadio es Orquiectomia Bilateral.

8.

CANCER DE PIEL: Cual de estas definiciones NO es CORRECTA.

a. Ca basocelular, ca escamocelular son tipos de cáncer de piel


b. Radiación Ultravioleta no es un factor de Riesgo.
c. Importante en el Ecuador por su situación geográfica.
d. .Melanoma Maligno: Incidencia baja, Mortalidad alta.

ANS: B
La radiación ultravioleta es el principal factor de riesgo para cáncer de ñpiel.

9. MARCADORES TUMORALES de CANCER GERMINAL DE OVARIO Y


TESTICULO . SEÑALE LA REPUESTA CORRECTA
a. HCGB CA 125, AFP.
b. DHL, B2 MICROGOBULINA, CA 15-3
c. AFP, HCGB, DHL.
d. HCGB, PSA, TIROGLOBULINA
ANS: C
AFP, HCGB, DHL

10. Usted forma parte del equipo de Salud del: PUESTO DE SALUD DE CHOCAVI,
que se encuentra en la Parroquia San Isidro del Cantón Guano de la Provincia del
Chimborazo. Esta Comunidad tiene 2.324 habitantes (ultima actualización del INEC).
LE SOLICITAN QUE DE UNA CHARLA de CANCER DE CERVIX ENFOCADA EN
PREVENCIÓN PRIMARIA Y PREVENCION SECUNDARIA. Cual de estas
definiciones no es CORRECTA:
a. Importancia de la Histerectomia Radical.
b. Educación a la Comunidad de HPV, Patrones de Conducta sexual.
c. Importancia del Papanicolaou y Colposcopia.
d. Importancia de la Vacuna Bivalente y Pentavalente.

ANS: A
La Histerectomía Radical pertenece a la Prevención terciaria.

11. CA DE CUELLO UTERO. Paciente con Pap-Test: NIC III. Estudio Histopatologico de
Colposcopia NIC III. Realizan CONIZACION Histopatologico INVASION AL ESTROMA
PROFUNDIDAD de SEIS milimetros
PREGUNTA estadio de este caso.
a. ESTADIO 1A1 c. ESTADIO 1B1
b. ESTADIO 1A2 d. ESTADIO IIA
ANS: C

12. Mujer de 20 años. presenta Tumoración de 2 centimetros de diametros en cuadrante supero


externo de mama derecha, No dolorosa, consistencia firme, superficie lisa, ovoidea, bien
deliimitada, muy movil. no piel de naranja, no retracción de piel
PREGUNTA DIAGNOSTICA DE PRESUNCION.
a. Mastopatia Fibro quistica
b. Quiste de Mama
c. Cáncer
d. Fibroadenoma
ANS: D

13. Paciente de 47 años, posterior a confirmacion histopatologica y examenes de extension:


realizan MASTECTOMIA RADICAL MODIFICADA + LINFADENECTOMIA NIVEL I-II
Histopatologico de pieza quirurgica CARCINOMA DUCTAL INFILTRANTE, tamaño 2.5 cts
de diametro, 2 ganglios POSITIVOS PARA MALIGNIDAD DE 15 resecados RECEPTORES
DE ESTROGENO, PROGESTERONA Y HER2 NEU NEGATIVOS.
PREGUNTA CUAL ES LA CONDUCTA MAS APROPIADA seguir tras la MASTECTOMIA.
a. OBSERVACION Y CONTROLES PERIODICOS
b. RADIOTERAPIA SOBRE LECHO MAMARIO
c. QUIMIOTERAPIA ADYUVANTE
d. HORMONOTERAPIA ADYUVANTE
ANS: C
14. DE LAS SIGUINTES DEFINICIONES DE ONCOLOGIA. SEÑALE CUAL DEFINICION
NO ES CORRECTA
a. LAS ESCALAS QUE MIDEN EL ESTADO FUNCIONAL DEL PACIENTE SON
E.C.O.G; KARNOFSKY
b. FACTOR PRONOSTICO PROPORCIONA INFORMACION AL MOMENTO DEL
DIAGNOSTICO
c. SUPERVIVENCIA GLOBAL: TIEMPO QUE TRASCURRE DESDE EL INICIO DE
UN TRATAMIENTO ONCOLOGICO HASTA EL FINAL DEL TRATAMIENTO
ONCOLOGICO
d. SE ESTIMA QUE EL 5 AL 10% DE LOS CANCERES SON HEREDITARIOS
ANS: C
DEFINIMOS COMO SUPERVIVENCIA GLOBAL AL TIEMPO QUE TRASCURRE DESDE
EL INICIO DEL PRIMER TRATAMIENTO ONCOLOGICO HASTA EL MOMENTO QUE
LA PACIENTE FALLECE

15. EN PATOLOGIAS ONCOLOGICAS EL CONOCIMIENTO DE LAS VIAS DE


METASTASIS ES IMPORTANTE EN LA HISTORIA NATURAL DEL TUMOR, SEÑALE LA
RESPUESTA CORRECTA.
a. CONTINUIDAD, LINFATICA, c. INICIACION, PROGRESION,
HEMATICA, TRANSCELOMICA PROMOCION
b. INICIACION, PROMOCION, d. DESPRENDIMIENTO,
PROGRESION INVASION,PENETRACION
ANS: A
EN LAS METASTASIS LAS VIAS DE DISEMINACION SON: CONTINUIDAD,
LINFATICA, HEMATICA Y TRANSCELOMICA
Neonatologia

Multiple Choice
Identify the choice that best completes the statement or answers the question.

____ 1. Con respecto a la intubación endotraqueal en el recién nacido. ¿Cuándo se debe considerar la inserción de
un tubo endotraqueal en el neonato? Seleccione una.

a. Después de completar los pasos iniciales, c. Después de calentar, secar, estimular,


si el bebe no está respirando o se posicionar vía aérea, succionar y si la
encuentra jadeando y si la frecuencia frecuencia cardiaca es inferior a 100 lpm.
cardiaca es inferior a 60 lpm.
b. Cuando la frecuencia cardiaca del bebe d. Cuando la frecuencia cardiaca permanece
sigue siendo inferior a 100 lpm y no por debajo de los 60 lpm a pesar de al
aumenta después de la VPP con mascarilla menos 30 segundos de VPP que inflan los
facial o laríngea. pulmones.
____ 2. Se presenta una paciente en labor de parto, con 39 semanas de gestación, se prepara el equipo necesario
para la atención. ¿Cuáles son las acciones iniciales para el cuidado del recién nacido?
a. Posicionar al recién nacido en el pecho de c. Evaluar el estado general de la madre
la madre para contacto piel con piel
b. Calentar, secar, estimular, reposicionar vía d. Considerar RCP al recién nacido a los 15
aérea, y succionar secreciones si es segundos tras el parto
necesario
____ 3. Neonato con Frecuencia Cardiaca de <60 lpm es asegurado una ventilación efectiva con tubo
endotraqueal, se coloca monitor cardiaco y decidimos iniciar compresiones torácicas, colocamos los
pulgares sobre el esternón, en el centro, justo debajo de una línea imaginaria que conecta los pezones del
bebé. rodeamos el torso con ambas manos. apoyamos la espalda con los dedos. En qué momento
detenemos las compresiones torácicas:
a. Después de 60 segundos de compresiones c. Si la frecuencia cardíaca es de 60 lpm o
torácicas y ventilación, detenga menos, suspenda las compresiones y
brevemente las compresiones y controle la reanude la VPP de 30 a 60 respiraciones
frecuencia cardíaca. por minuto. Cuando se logre una señal
confiable del oxímetro de pulso, no es
necesario ajustar la concentración de
oxígeno para cumplir con las pautas de
saturación de oxígeno objetivo
b. Si la frecuencia cardíaca del bebé sigue d. Si se ha cumplido una 60 compresiones
siendo inferior a 60 lpm a pesar de 90 por minuto y la tasa de respiración es de
segundos de ventilación efectiva y 45 a 60 respiraciones por minuto.
compresiones torácicas coordinadas de
alta calidad, no está indicada la
administración de epinefrina y talvez se
necesita un acceso vascular.
____ 4. Una mujer entra en labor de parto a las 32 semanas de gestación, es atendida en el hospital, encontrandose
ella en buenas condiciones pero al ser su niño un bebé prematuro se induce a reanimación neonatal en la
que se administrará oxígeno para compensar su saturación y alcanzar los valores normales de ésta. ¿Qué
porcentaje de oxígeno necesará administrarse y con que frecuencia se pasara el mismo?
a. Para la reanimación inicial de este recién c. Para la reanimación inicial de los recién
nacido de 32 semanas de gestación, se nacidos con menos de 35 semanas de
debería ajustar el mezclador de oxígeno al gestación, ajustar el mezclador a un
21 % a 8 l/minuto oxígeno del 21 % al 30 %. a 10 l/minuto
b. Para la reanimación inicial del recién d. Para la reanimación inicial de los recién
nacidos con 32 semanas o menos de nacidos con 32 semanas de gestación, es
gestación, es necesario ajustar el recomendable ajustar el mezclador a un
mezclador a un oxígeno al 15 % a 10 oxígeno al 35 % a 1 l/minuto
l/minuto
____ 5. Un recién nacido ha estado recibiendo ventilación con mascarilla, pero no mejora. A pesar de realizar los
primeros 5 pasos correctivos de ventilación, la frecuencia cardíaca es de 75 lpm, no aumenta y hay un
movimiento torácico deficiente. ¿Qué se debe hacer?
a. Debe iniciarse inmediatamente las c. Debe insertarse inmediatamente una vía
compresiones torácicas, a nivel del aérea alternativa, como un tubo
esternón. endotraqueal o una máscara laríngea.
b. Debe verificar la frecuencia cardiaca d. Debe insertarse después de 10 minutos
durante 5 minutos, si no mejora se procede una vía aérea alternativa, como un tubo
a insertar la vía aérea alternativa. endotraqueal o una máscara laríngea.
____ 6. Los pasos correctivos de ventilación son una serie de ajustes que se realiza si la frecuencia cardíaca del
bebé no mejora y el pecho no se mueve después de comenzar con la VPP. ¿Cuáles son las razones más
probables de que una ventilación con mascarilla sea ineficaz?
a. Fugas alrededor de la mascarilla, c. El bebe está ejerciendo demasiada presión
obstrucción de las vías respiratorias, en sus pulmones
ventilación insuficiente.
b. Los bebés nacidos a término pueden d. Es común la fuga de aire entre la mejilla y
requerir una presión de inflado más alta el puente de la nariz
durante las primeras respiraciones para
inflar sus pulmones
____ 7. Una mujer llega en trabajo de parto a las 39 semanas de gestación. El trabajo de parto progresa
rápidamente y el proveedor de obstetricia llama a su equipo de reanimación para asistir al parto vaginal.
¿Cuáles son las cuatro preguntas prenatales que se debe realizar al proveedor de obstetricia para evaluar
los factores de riesgo perinatal?
a. ¿Cuál es la edad gestacional esperada? c. ¿Cuál es nuestro plan de manejo del
¿Está claro el líquido amniótico? ¿Existen cordón umbilical? ¿Existen factores de
factores de riesgo adicionales? ¿Tiene riesgo adicionales? ¿Está el bebé
buen tono muscular? respirando o llorando? ¿Parece que el bebé
está a término?
b. ¿Parece que el bebé está a término?, d. ¿Cuál es la edad gestacional esperada?
¿Tiene buen tono muscular? ¿Está el bebé ¿Está claro el líquido amniótico? ¿Existen
respirando o llorando? ¿Está claro el factores de riesgo adicionales? ¿Cuál es
líquido amniótico? nuestro plan de manejo del cordón
umbilical?
____ 8. El indicador más importante de una ventilación con presión positiva (VPP) exitosa es el aumento de la
frecuencia cardíaca. Cuando se comienza la VPP, un asistente controlará la respuesta del ritmo cardíaco
del bebé. Una vez que comienza la VPP, un asistente debe aplicar un sensor de oxímetro de pulso para
evaluar continuamente la saturación de oxígeno del bebé. ¿Qué se debe hacer si la frecuencia cardiaca del
bebé no aumenta después de los primeros 15 segundos de aplicar VPP?
a. Pregúntele a su asistente si el tórax se está c. Pregúntele a su asistente si el tórax se está
moviendo. Si el tórax se mueve, continúe moviendo. Si el Tórax se mueve, continúe
con la VPP durante 50 segundos. con la VPP mientras supervisa su técnica
de ventilación. Verificará la respuesta del
bebé nuevamente después de 30 segundos
de VPP.
b. Pregúntele a su asistente si el tórax se está d. Pregúntele a su asistente si el tórax se está
moviendo. Si el tórax NO se mueve, moviendo. Si el tórax NO se mueve,
administre epinefrina IV cada 3-5 considere un posible neumotórax.
minutos.
____ 9. Cuando la frecuencia cardiaca desciende a -100 lpm y la ventilación a presión positiva no ha sido exitosa,
se iniciará en el neonato la ventilación por tubo endotraqueal. Tras la inserción del tubo endotraqueal es
necesario confirmar si está ubicado en la tráquea.
Mencione los métodos oportunos para confirmar la ubicación del tubo endotraqueal:
a. Detector de CO2 y O2 c. Monitores eléctricos
b. Detector de CO2 colorimétrico y d. Pulsioximetro.
capnógrafo.
____ 10. Para las compresiones torácicas se necesita aplicar presión en el tercio inferior del esternón, colocando los
pulgares sobre el esternón justo debajo de una línea imaginaria que conecta los pezones del bebé. Sus
dedos pulgares deben colocarse en el centro del esternón, ya sea uno al lado del otro o uno sobre el otro.
Utilizando sus pulgares ¿a que presión debe deprimir el esternón para una correcta compresión torácica al
bebe?
a. Aproximadamente un tercio del diámetro c. Aproximadamente un cuarto del diámetro
anteroposterior (AP) del tórax, y luego posterior (P) del tórax, y luego libere la
libere la presión para dejar que el corazón presión para dejar que el corazón vuelva a
vuelva a llenarse. llenarse.
b. Aproximadamente un tercio del diámetro d. Aproximadamente un tercio superior del
antero inferior (AI) del tórax, y luego tórax, y luego libere la presión para dejar
libere la presión para dejar que el corazón que el corazón vuelva a llenarse.
vuelva a llenarse.
____ 11. Después del nacimiento, todos los recién nacidos deben someterse a una evaluación rápida para
determinar si pueden permanecer con su madre para continuar con la transición o si deben ser trasladados
a un calentador radiante para una evaluación adicional. Esta evaluación inicial puede ocurrir durante el
intervalo entre el nacimiento y el pinzamiento del cordón umbilical. Rápidamente hará 3 preguntas.
¿Cuáles son las 3 preguntas de evaluación rápida que determinan qué recién nacidos deben llevarse al
calentador radiante para los pasos iniciales?
a. ¿Es el bebé a término? ¿El bebé tiene buen c. ¿El bebé tiene buen tono? ¿El bebé está
tono? ¿El bebé está respirando o llorando? llorando? ¿El bebé está cianótico?

b. ¿El bebé está respirando o llorando? ¿El d. ¿El bebé está activo? ¿Es el bebé a
bebé está cianótico? ¿Es el bebé a término? ¿El bebé está cianótico?
término?
____ 12. Cuando se proporciona CPAP o VPP con una máscara, el gas ingresa en el esófago y estómago del
neonato. Este gas que se filtra hacia el estómago puede interferir con el proceso de ventilación ¿Qué
procedimiento se debe considerar para evitar esto si un recién nacido requiere de CPAP o VPP con
máscara durante más de varios minutos?
a. Colocar una sonda orogástrica y dejarla c. Colocar un oxímetro en la muñeca del
tapada para que no se filtre gas hacia el neonato para verificar una correcta
esófago y estómago ventilación con el ascenso de la saturación
de oxígeno
b. Proceder a realizar intubación d. Colocar una sonda orogástrica y dejarla
endotraqueal para asegurar la ventilación sin tapar para que actúe como vía de salida
para el estómago.
____ 13. En caso de que se esté dando ventilación por presión positiva, pero no se observa expansibilidad de tórax
del bebé, y la frecuencia cardiaca sigue por debajo de los 100 latidos por minuto; lo que se debe realizar
son pasos correctivos de ventilación, entre los que se encuentran, excepto:
a. Ajuste de la mascarilla: Vuelva a aplicar c. Colocar vía aérea alternativa: Inserte una
la mascarilla y levante la mandíbula hacia máscara laríngea o un tubo endotraqueal.
adelante. Considere el agarre de 2 manos.
b. Aumentar la presión: Aumentar en 5-10 d. Succión de secreciones (nariz y boca):
cm H2O incrementos hasta la presión Utilice una pera de goma o un catéter de
máxima recomendada. • Máx. 60 cm de succión.
H2O término • Max 50 cm H2O
prematuro.
____ 14. Si un recién nacido requiere CPAP o VPP durante más de varios minutos, se considera colocar una sonda
orogástrica y dejarla destapada para que actúe como ventilación del estómago. ¿Cuál es la medición
correcta para insertar una sonda orogástrica?
a. Mida la distancia desde el puente de la c. Medir la distancia entre la nariz o los
nariz hasta el lóbulo de la oreja y desde el labios y el lóbulo de la oreja y luego entre
lóbulo de la oreja hasta un punto a medio la oreja y el estómago, de tal manera que
camino entre la apófisis xifoides y el el último orificio se encuentre a la altura
ombligo. del apéndice xifoides.
b. El cardias está generalmente d. Sostenga el dispositivo a lo largo del tubo
aproximadamente 40 cm desde la boca de las vías respiratorias luego deslice el
del recién nacido. dispositivo hacia abajo y hacia atrás,
siguiendo el contorno del paladar, con un
empujón continuo pero suave hasta que
sienta una resistencia definitiva.
____ 15. El algoritmo Programa de Reanimación Neonatal describe los pasos que seguirá para evaluar y resucitar a
un recién nacido, ¿cuáles son los 5 bloques que comprenden este algoritmo?
a. Evaluación rápida, via aérea, respiración, c. Vía aérea, respiración, circulación, déficit
circulación, droga neurológico, control de hemorragias
b. Vía aérea, respiración, circulación, déficit d. Evaluación rápida, via aérea, respiración,
neurológico, ambiente circulación, desfibrilación
Reactivos Ginecología y Obstetricia Anabela Criollo

MULTIPLE CHOICE

1. Puérpera de 23 años, con 3 horas posparto, sangrado transvaginal en


moderada cantidad acompañada de coágulos. Al examen físico palidez
generalizada, TA: 100/50 mmHg, FC: 90X, abdomen blando, depresible, poco
doloroso en hipogastrio, útero contraído con AFU: O+2. Tacto vaginal:
cérvix posterior, dilatado 3cm, se extraen coágulos.
¿Del caso anterior indique cuál considera que sería el diagnóstico?

A____ Hemorragia posparto inmediata.


B____ Retención de restos placentarios.

C____ Hemorragia posparto tardía.


D____ Atonía uterina.

a.
A, B
b.
A, D
c.
B, C
d.
C, D
e.
B, D

ANS: A

2. Gestante de 19 años, nulípara, cursa con embarazo de 16,4 semanas,


asintomática. Es citada a consulta en subcentro de salud porque en examen de
control le diagnosticaron bacteriuria asintomática.
Del caso anterior diga cuál es la opción correcta.

a.
No requiere tratamiento farmacológico ya que no tiene síntomas.
b.
El tratamiento de elección es con fosfomicina 3 g VO dosis única.
c. No se requiere resultado de urocultivo para iniciar el tratamiento porque
demora 72 horas.
d.
Es necesario el urocultivo para iniciar antibiótico terapia.

ANS: D
3. Puérpera de 31 años con 2 horas posparto, con diagnóstico de preeclampsia,
está bajo su cuidado clínico con sulfato de magnesio por lo que debe examinarla
y determinar si tiene signos de intoxicación por sulfato de magnesio. Marque la
respuesta correcta.

a.
Convulsiones y poliuria
b.
Hiperreflexia
c.
Hiporeflexia y Bradipnea
d.
Edema de pulmón

ANS: C

4. Gestante de 36 años, antecedentes de hipertensión gestacional en el embarazo


anterior hace 12 años, con HO: G4, P4, A0, C0, cursa con embarazo de 10.6
semana por FUM. Usted la valora en su primer control prenatal, decide indicar
profilaxis de preeclampsia. ¿En qué semanas aparece con más frecuencia la
preeclampsia?

a.
En las primeras 10 semanas
b.
En el parto
c.
Después de las 20 semanas
d.
Al terminar el embarazo

ANS: C

5. Mujer de 24 años, antecedentes de EPI a repetición con amenorrea de 8,5


semanas. Acude a emergencia con dolor abdominal tipo cólico localizado en fosa
iliaca derecha e irradiado a hipogastrio de 10/10 en la escala de EVA. Al tacto
vaginal: cérvix posterior, puntiforme cerrado, se palpa masa anexial derecha muy
dolorosa, escaso sangrado achocolatado. ECO transvaginal: útero de 7x4x5 cm,
con línea endometrial de 20 mm, no hay saco gestacional. ¿Cuál considera que
sería el diagnóstico?
a.
Amenaza de aborto.
b.
Aborto completo.
c.
Embarazo ectópico.
d.
Enfermedad trofoblástica gestacional.

ANS: C
6. Paciente de 26 años, antecedentes de hipertensión gestacional en el embarazo
anterior. Cursa con embarazo de 37.6 semana por FUM, constata dos cifras de
TA elevada de 145/90 mmHg y la proteinuria en tirilla positiva ++, asintomática.
Usted diagnostica preeclampsia e inicia tratamiento preventivo de eclampsia con
sulfato de magnesio. ¿En cuánto tiempo se pasa la dosis de impregnación de
sulfato de magnesio.

a.
5 minutos
b.
20 minutos
c.
30 minutos
d.
40 minutos

ANS: B
7. Paciente de 16 años; G1, P1, A0, C0, parto en domicilio atendido por partera,
sangrado de más o menos 1.5 litros. Presenta dolor intenso en abdomen que no
cede, al examen físico: paciente confundida, con signos vitales: TA: 85/48 mmHg,
FC: 122x, FR: 28X, Temperatura: 36,4; SatO2: 85%. Presenta signos de shock
hipovolémico por hemorragia posparto. Calcule el índice de shock y clasifique el
grado de este.
A. 0,7

B. Grado 2.
C. 2.0
D. Grado 4.

E. 1.4
F. Grado 3.
G. 0,4

H. Grado 1.
a.
A, B
b.
C, D
c.
E. F
d.
G, H

ANS: C
8. Gestante de 29 años, antecedentes de hipertensión gestacional en el embarazo
anterior, HO: G4, P3, A1, C0, cursa con gravidez de 35.6 semana. Usted la valora
y constata cifras de TA de 145/100 mmHg, refiere cefalea de gran intensidad que
no cede con analgésicos, epigastralgia, náuseas y dos vómitos, proteinuria en
tirilla es positiva ++, diagnostica preeclampsia con signos de gravedad y
sospecha un síndrome de Hellp por lo que solicita exámenes de laboratorio.
¿Según la clasificación de Mississippi a qué clase corresponde la presencia de
50.000 a 100.000 plaquetas en el síndrome de hellp?

a.
Clase I
b.
Clase II
c.
Clases III
d.
Incompleto

ANS: A
9. Paciente de 27 años, amenorrea de 9,3 semanas. Acude a emergencia por
presentar dolor pélvico, tiene HCG positiva, en ECO transvaginal: saco
gestacional cuyo tamaño corresponde a 7 semanas y ausencia de latido cardiaco.
Tacto vaginal: cérvix posterior, largo, cerrado.
¿Del caso anterior indique cuál considera que sería el diagnóstico y la conducta a
seguir?
A____ Aborto incompleto.
B____ Aborto terapéutico.
C____ Aborto diferido.
D____ Misoprostol: 600 mcg VV.

E____ Misoprostol: 200 mcg VV cada 6-12 horas hasta 4 dosis, si con la primera
dosis no se logra actividad uterina se aumenta a 400 mcg, dosis máxima en un
día 1600 mcg.

F___ Misoprostol: 800 mcg VV.


G_____AMEU.
H_____LIU:
I_____ Alta después de administrar el misoprostol.

a.
A, E, I
b.
B, F, H
c.
C, F, G
d.
B, E, G
e.
C, D, H

ANS: C
10. Paciente de 34 años, sin antecedentes de importancia, cursa con embarazo de
37,6 semana por FUM. Se constata dos cifras de TA: 160/90 mmHg, proteinuria
en tirilla es positiva ++, esta asintomática. Usted la valora y diagnostica
preeclampsia e iniciar tratamiento preventivo de eclampsia y administra
antihipertensivo. Seleccione la opción correcta acerca del tratamiento
farmacológico oral en los trastornos hipertensivos del embarazo en nuestro
medio.

a.
Nimodipino
b.
Labetalol
c.
Nifedipina
d.
Hidralazina

ANS: C
11. Paciente de 19 años, antecedentes de salud anterior. Se atiende parto céfalo
vaginal con alumbramiento tipo Duncan, sangrado de 1000 ml. Ahora presenta
dolor intenso en abdomen que no cede, al examen físico: paciente consciente,
orientada en las 3 esferas, con signos vitales: TA: 100/65 mmHg, FC: 100x, FR:
28X, Temperatura: 36,4; SatO2: 95%. Por características clínicas se considera
que paciente está con shock hipovolémico por hemorragia posparto. Seleccione
el planteamiento que considere correcto acerca de la hemorragia posparto.
Calcule el índice de shock y clasifique el grado de este.
A. 0,7

B. Grado 2.
C. 1.0
D. Grado 1.

E. 1.4
F. Grado 3.
G. 2.0

H. Grado 4.
a.
A, B
b.
C, D
c.
E, F
d.
G, H

ANS: B
12. Paciente nulípara de 27 años, antecedentes de salud anterior. Refiere 2 abortos
espontáneos anteriores ocurridos a las 18 y 20.5 semanas respectivamente.
¿Cuál es la primera causa que usted debería descartar?

a. Malformaciones uterinas
b. Cicatrices uterinas
c. Incompetencia cervical
d. Endometriosis

ANS: C

13. Gestante de 39 años, con embarazo de 18,3 semanas por FUM, TA: 155/100
mmHg en dos tomas consecutivas separadas por 20 minutos, proteinuria en tirilla
negativa, asintomática. ¿Cuál es su probable diagnóstico?
a.
Hipertensión gestacional.
b.
Preeclampsia con signos de gravedad.
c.
Hipertensión crónica.
d.
Preeclampsia sin signos de gravedad.

ANS: C
14. Paciente de 36 años, multípara, cursa embarazo de 34 semanas de gestación,
presenta sangrado genital rojo oscuro como manchas, acompañada de dolor
abdominal intenso, cefalea y escotomas. Al examen físico: TA:150/95 mmHg,
útero duro y leñoso, especuloscopia: salida escasa de sangre oscura sin cambios
cervicales; frecuencia cardiaca fetal 142 Latidos por minuto. ¿Cuál es el
diagnóstico probable?

a. Amenaza de aborto
b. Desgarro vaginal
c. Placenta previa
d. Desprendimiento prematuro de la placenta normo inserta
e. Rotura uterina
ANS: D
15. Paciente de 27 años, cursa embarazo de 14,6 semanas, con un IMC de 30
kg/m2. Refiere ligera polifagia y polifagia, trae resultado de exámenes, glucosa:
108 mg/dL, al examen físico constata altura uterina a nivel de ombligo y edema
marcado en miembros inferiores. ¿Cuál sería el diagnóstico probable?

a.
Diabetes mellitus tipo 1 insulino dependiente
b.
Diabetes gestacional
c.
Diabetes pregestacional
d.
Todas las anteriores

ANS: B
REACTIVOS DE PSIQUIATRIA

Multiple Choice
Identify the choice that best completes the statement or answers the question.

____ 1. 1-En consulta vemos un paciente de 22 años femenina que acude en compañia de su tia materna quien
refiere que repite mucho las cosas,que no entiende el lenguaje de su sobrina A la entrevista observamos
que no existe una idea directriz a pesar de que fragmentos del mismo resultan comprensibles. Esta
alteración del lenguaje-pensamiento, se observa en la esquizofrenia, y en psicopatología se conoce como:
a. Disociación del pensamiento d. Desorganización del pensamiento.

b. Fuga de ideas e. Bloqueo del pensamiento


c. Lenguaje perseverante.

____ 2. En la sala de Medicina interna del HPGR se encuentra hospitalizado un adulto masculino de 30 años, con
diagnostico de Hipertension Arterial en estudio y que hace un mes se le hizo diagnostico de Esquizofrenia
paranoide y que actualamente lleva tratamiento ambulatorio con haloperidol 5 mg vo cada 12 horas . Se
nos envia interconsulta ya que presenta intranquilidad y no puede permanecer mucho tiempo sentado ni
acostado, camina de un lado al otro y cuando esta de pie , balancea las piernas , refiere sensación
subjetiva de inquietud. El cuadro anteriormente descrito corresponderia a el siguiente trastorno inducido
por fármacos:

a. Parkinsonismo. c. Distonía Aguda


b. Síndrome Neuroléptico Maligno d. Acatisia.

____ 3. A la Emergencia acude paciente de 18 años, sin antecedentes personales de enfermedades psiquiatricas ni
organicas, que refiere que haber ingerido 30 tabletas de las que no precisa nombre con el proposito de
quitarse la vida . A la entrevista observamos llanto facil ,labilidad emocional, inhibición psicomotriz y
enlentecimiento del pensamiento, refiere dificultad en la atencion y concentración , dificultades
academicas ,sensacion que le quieren hacer daño y hablan mal de ella y que le persiguen , principalmente
una compañera del colegio que vive a una cuadra de su casa y eso le genera gran angustia.Su mama que
le acompaña refiere que dede hace mas o menos tres semanas presenta aislamiento social,tristeza ,
anhedonia , tristeza marcada , sentimientos de culpa, pensamientos negativos, hipersomnia diurna y
empeoramiento matutino de los síntomas, desesperanza, perdida del apetito y perdida de peso. . Señale el
diagnóstico:

a. Esquizofrenia. c. Trastorno depresivo con síntomas


psicóticos
b. Trastorno de ideas delirantes persistentes d. Trastorno distimico.
____ 4. Paciente femenina de 50 años sin antecedentes psiquiátricos que es hospitalizada en el servicio de
ginecologia para estudio de metrorragia y a los 3 días de su ingreso es informada que presenta neoplasia
uterina con metástasis. Se interconsulta a psiquiatría porque 24 horas después muestra tristeza y llanto
frecuente, refiere ideas de muerte y presenta insomnio. En la evaluación psiquiátrica no se recoge ningún
antecedente psiquiátrico y la exploración detecta elevada ansiedad y desesperanza en relación con las
consecuencias de su enfermedad neoplásica. El diagnóstico más probable es:

a. Tastorno esquizoafectivo d. Trastorno de estres postraumatoico


b. Distimia e. Reaccion normal frente al estres
c. Trastorno de ansiedad generalizada

____ 5. Es referida a nuestra consulta paciente adolescente de 16 años acude a la consulta en compañia de su
hermana de 26 quien esta a su cuidado ya que sus padres trabajan en otra provincia y solo estan con ella
un fin de semana al mes desde que tenia 13 años .A la entrevista refiere que lleva de cerca de un año
sintiéndose más cansada, con poco apetito y dificultades para concentrarse en los estudios, cuando se le
pregunta comenta también que sale menos con las amigas y se muestra pesimista respecto de su futuro. El
diagnóstico más probable seria:

a. Anorexia d. Trastorno de ansiedad

b. Distimia e. Agarofobia

c. Depresión mayor

____ 6. Paciente masculino de de 23 años de edad, que refiere que desde que comenzo la pandemia,siente mucha
ansiedad al tocar objetos, como picaportes de puertas, o que le rocen en transportes públicos por miedo a
contaminarse. Sabe que es absurdo, pero mantiene una actitud continuada de vigilancia, realiza conductas
de lavado de manos repetidamente y progresivamente ha ido restringiendo sus salidas para evitar las
numerosas situaciones que considera de potencial riesgo de contaminación. ¿Cuál seria el diagnostico en
este paciente?

a. Idea delirante de contaminación. d. Temores hipocondriacos.


b. Fobia especifica. e. Ideas sobrevaloradas de contaminación
c. Obsesiones de contaminación.
____ 7. Vemos en sala de Medicina interna un adulto mayor de 73 años previamente con antecedentes de salud,
que acude a consulta ya que presenta cuadro de varias horas de evolución de alteración de conciencia y de
las funciones mentales superiores con tendencia a la apatía y a la somnolencia. Tiene trastornos de la
percepción con algunas alucinaciones visuales y auditivas . A su familia lo que más le extraña es que el
cuadro sea muy fluctuante, pues se pasa de estar casi dormido a agitarse y vociferar, y a ratos parece estar
lucido. ¿Cual seria el diagnostico posible en este caso?
a. Trastorno histérico de la personalidad. d. Inicio de demencia.

b. Síndrome Confusional agudo e. Angiopatia amiloide


c. Ictus en territorio de la arteria cerebral
media derecha.
____ 8. Es ingresado en observacion de Emergencia un paciente de 70 años, jubilado, que se encuentra
acompañado por uno de sus hijos; quien refiere que que el día anterior, de forma bastante brusca,
comenzó a decir cosas raras, a no responder a lo que le preguntaban y a mostrarse confuso incluso
respecto a su propio nombre. Efectivamente en la exploración parece no entender lo que se le pregunta, no
recuerda nada de lo que le ha pasado y no sabe ni el día, ni el lugar en el que está. El diagnostico seria:

a. Psicosis breve. d. Esquizofrenia

b. Delirium. e. Esquizofrenia

c. Alzheimer
____ 9. Paciente de 30 años que acude por segunda vez en la semana a la urgencia por cortes superficiales,
autoinflingidos, en ambos brazos, teniendo como causa a parente discusion con la pareja . En la entrevista
refiere tener antecedentes varios intentos autoliticos siempre secundaios a problemas con su pareja ,tien
conductas irritables frecuentes, esr impulsivo , consume marihuana y alcohol de manera abusiva, es
inestable en las relaciones , señale el diagnóstico más probable.

a. Fase maniaca de un trastorno afectivo d. Trastorno de ansiedad generalizada


bipolar.
b. Trastorno de personalidad limite. e. Síndrome de Cotard.

c. Trastorno de personalidad esquizotípico

____ 10. Es referido del centro de salud paciente masculino de 38 años, soltero, que vive en
zona rural y realiza actitividades agricolas con un hermano con quien vive actualmente,
tercer grado de escolaridad, repitio varios grados ya que no entendia las materias.A los
6 meses de nacido presento un cuadro de meningitis bactriana con hipertermia severa
y convulsiones por lo que estuvo hospitalizado durante 2 meses.. Luego de su alta
comenzo con dificultades en el deasrrollo psicomotos,como importante retraso para
caminar y hablar. Su vocabulario es de aproximadamente un niño de 6 años.
Desempeña su trabajo adecuadamente, cuando se le orienta que hacer.
a. Retraso menal d. Distimia
b. Esquizofrenia e. Demencia
c. Epilepsia
____ 11. A continuación, exponemos referencias sintomáticas que pueden obtenerse tanto del
propio paciente como de otras fuentes de información. El objetivo es que usted valore
para identificar el trastorno psicopatológico y las ubique en el acápite correspondiente
del examen psiquiátrico.

b-El paciente refiere ¨Oigo claramente dentro de mi cabeza una voz femenina que me
ordena ofender a mis vecinos¨.

a. Pseudoalucinacion d. Ilusion

b. Despersonalizacion e. Desrealizacion
c. Alucinacion

____ 12. Identifique dentro de las alteraciones del contenido del pensamiento el trastorno
psicopatológico que presenta este paciente.
-Paciente AMP , masculino de 38 años , de ocupación ingeniero que acude a consulta
consciente, orientado globalmente que viene remitido del subcentro de salud y
acompañado por la esposa ya que desde hace una semana se encuentra con gran
preocupación después de haber tenido una insignificante discusión de trabajo con uno
de sus compañeros. Su esposa en la entrevista refiere ¨Aunque todos le hemos
señalado que debe olvidar el incidente, desde que discutió con su compañero de oficina
se recrimina constantemente por su comportamiento, ha llegado a decir que jamás lo
perdonaremos y que su solución es trasladarse de trabajo. Nadie comprende su
reacción pues el incidente fue algo muy banal.´

a. Idea Fija d. Idea sobrevalorada

b. Idea Fóbica e. Idea delirante

c. Idea obsesiva

____ 13. El paciente refiere en la consulta ¨Cuando pasé por el grupo me di cuenta que se
estaban burlando de mí y les fui a pedir cuenta,ellos se excusaron, pero yo sé que
realmente se habían burlado¨. El objetivo es que usted identifique el trastorno
psicopatológico y las ubique en el acápite correspondiente del examen psiquiátrico.

a. Idea delirante de daño d. Idea delirante de grandeza


b. Idea delirante de referencia e. Idea delirante de influencia
c. Idea delirante de persecusion
____ 14.
169.Un paciente de 40 años presenta, desde hace 2meses, una tendencia que no puede
resistir a realizar actos que le resultan desagradables. Se le ocurren pensamientos, percibe
imágenes y siente impulsos reiterados y molestos que, aún reconociendo como absurdos, no
puede tampoco evitar. Durante el último mes se siente triste y abatido, sin ganas de hacer
nada. Esporádicamente presenta episodios angustiosos de gran intensidad. ¿Cuál sería la
hipótesis diagnóstica más probable según la CIE-10?:

a. Trastorno mixto ansioso-depresivo d. Trastorno obsesivo-compulsivo.

b. Trastorno depresivo con síntomas e. Ataques de pánico.


obsesivos.

c. Trastorno esquizo-afectivo.

____ 15.
Ante un paciente que bruscamente y por segunda vez en un un mes ha comenzado con un
conjunto de síntomas constituido por: sensación de dificultad respiratoria, de
ahogo (disnea)o de paro respiratorio, mareo, sensación de muerte inminente, angustia,
desesperación, sudoraciones, temblores, palpitaciones o taquicardia y náuseas o malestar
abdominal. ¿Cuál es el diagnóstico más probable?:

a. Trastornos de ansiedad generalizada. d. Trastorno de pánico.

b. Trastorno mixto ansioso-depresivo. e. Trastorno de adaptación.

c. Trastorno fóbico.
EVALUACIÓN DE ENDOCRINOLOGÍA
7mo SEMESTRE C

TRUE/FALSE

1. Mencione VERDADERO o FALSO sobre las funciones del Sistema Endocrino, mantener el
equilibrio hidroelectrolítico, la homeostasis del organismo y el metabolismo de los hidratos de
carbono, lípidos y proteínas, así como favorecer a la reproducción y junto con el sistema
inmunológico proteger el organismo contra enfermedades.

ANS: T
las funciones del Sistema Endocrino son mantener el equilibrio hidroelectrolítico, la homeostasis
del organismo y el metabolismo de los hidratos de carbono, lípidos y proteínas, así como favorecer
a la reproducción y junto con el sistema inmunológico proteger el organismo contra enfermedades.

PTS: 1 DIF: MEDIA


REF: Guyton & Hall. Tratado de fisiología médica Ed.14
OBJ: Comprender la fisiología del sistema endocrino para identificar las alteraciones y
controlarlas.
TOP: ENDOCRINOLOGÍA Y FISIOLOGÍA KEY: sistema endocrino
NOT: Dra. Katherine Maldonado

2. SEÑALE VERDADERO O FALSO SEGUN CORRESPONDA.


La Diabetes insípida se refiere a la patología pancreática caracterizada por aumento de la glucosa
por resistencia a la insulina.

ANS: F
Trastorno del metabolismo de la sal y el agua caracterizado por sed intensa y mucha orina.
La diabetes insípida ocurre cuando el cuerpo no puede regular el manejo de los líquidos. La
enfermedad es ocasionada por una anomalía hormonal y no está relacionada con la diabetes.

PTS: 1 DIF: MEDIA REF: Harrison F. Principios de Medicina Interna 19


Ed.
OBJ: Reconocer las patologías endocrinas para poder tratarlas de forma adecuada.
TOP: ENDOCRINOLOGIA KEY: patologia endocrina
NOT: DRA. KATHERINE MALDONADO

3. SEÑALE VERDADERO O FALSO


La patologia gonadal en la que existe infertilidad, disminución de la líbido, cambios en los
caracteres sexuales y alteraciones menstruales, es el hipogonadismo.

ANS: T
VERDADERO

PTS: 1 DIF: MEDIA REF: Harrison F. Principios de Medicina Interna 19


Ed.
OBJ: Reconocer las patologías endocrinas para poder tratarlas de forma adecuada.
TOP: ENDOCRINOLOGIA KEY: patologia endocrina
NOT: DRA. KATHERINE MALDONADO

MULTIPLE CHOICE
1. De las siguientes opciones escoja la respuesta correcta. En relación al Sistema Endocrino cuales
son las hormonas que secreta la Hipófisis.
a. Hormona del crecimiento c. Cortisol
b. Insulina d. Tiroxina
ANS: A PTS: 1 DIF: BAJA
REF: Guyton & Hall. Tratado de fisiología médica Ed.14
OBJ: Conocer la fisiologia del Sistema Endocrino para identificar sus alteraciones.
TOP: ENDOCRINOLOGIA KEY: Sistema endocrino
NOT: Dra. Katherine Maldonado

2. Las patologías de la Hipófisis pueden deberse al incremento o disminución de su función, al


referirse a la hiperfunción, señale la respuesta incorrecta de las siguientes opciones:
a. ADENOMAS c. CRANEOFARINGEOMA
b. PROLACTINOMAS d. GLUCAGONOMA
ANS: D
La hipófisis es la glándula maestra del sistema endocrino, rige todo el organismo de manera directa
o indirecta; sin embargo, puede verse afectada por patologías intrínsecas a la hipófisis o por tejidos
vecinos, dando como resultado alteraciones endocrinológicas que no siempre son fáciles de
determinar. La glándula hipófisis puede manifestar insuficiencia, misma que se muestra también a
nivel multiglandular, así como estados de hipersecreción, ya sea por adenomas o hiperplasias, con
producción de algún tipo de hormona, también puede presentar sobrecrecimiento, con compresión
de estructuras vecinas como Craneofaringeomas y Tumores de la Bolsa de Rathke.

PTS: 1 DIF: MEDIA REF: Harrison Principios de Medicina Interna 19 Ed.


OBJ: Reconocer las patologias endocrinas para poder tratarlas de forma adecuada.
TOP: ENDOCRINOLOGIA KEY: patologia endocrina
NOT: Katherine Maldonado

3. La Acromegalia y el Gigantismo son patologías hipofisiarias dependientes de la hipersecreción de


la Hormona de Crecimiento, de las siguientes opciones señale la correcta
a. El exceso de hormona de crecimiento en d. El tratamiento de elección es quirúrgico
al adulto se denomina acromegalia
b. La Medición de la hormona del e. Todas las anteriores.
crecimiento se debe hacer con una prueba
de Tolerancia oral a la glucosa
c. Las manifestaciones pueden tardar de 6 a
8 años
ANS: E
La acromegalia en una enfermedad causada por la producción anormal de hormona del
crecimiento en momentos en los que los huesos ya han tenido su cierre epifisiario, es decir,
se presenta en la poblacin adulta de ambos sexos. Cuando esta anormalidad se evidencia
en poblacin peditrica o que no ha terminado su crecimiento, se denomina gigantismo.
Las manifestaciones clínicas demoran en aparecer de 6 a 8 años.
Tratamiento quirúrgico: idealmente, el tratamiento para la acromegalia es la reseccin
quirrgica del adenoma con preservacin de la funcin hipofisiaria. El abordaje quirúrgico ms
aceptado es la va transesfenoidal.

PTS: 1 DIF: MEDIA


REF: De Pablos P.L. Epidemiología. En: Webb S. Acromegalia.
OBJ: Reconocer las patologías endocrinas para poder tratarlas de forma adecuada.
TOP: ENDOCRINOLOGIA KEY: patologia endocrina
NOT: DRA. KATHERINE MALDONADO

4. Los adenomas hipofisiarios pueden establecer incremento de la produccion de TSH, lo que se


traduce a nivel de la Glandula Tiroides en: Señale la correcta
a. Hipotiroidismo primario c. Hipotiroidismo terciario
b. Hipotiroidismo secundario d. Panhipopituitarismo
ANS: B
El hipotiroidismo se desarrolla a cualquier edad pero es más frecuente en adultos mayores, donde
se manifiesta de manera sutil y puede ser difícil de reconocer. El hipotiroidismo puede ser
Primario: causado por un trastorno en la tiroides
Secundario: causado por un trastorno en el hipotálamo o la hipófisis

PTS: 1 DIF: ALTA


REF: De Pablos P.L. Epidemiología. En: Webb S. Acromegalia. Madrid: Editorial Acción
Médica; 1998. OBJ: Reconocer las patologías endocrinas para poder tratarlas de forma
adecuada.
TOP: ENDOCRINOLOGIA KEY: patología endocrina
NOT: DRA KATHERINE MALDONADO

5. La tirotoxicosis se define como el exceso de hormonas tiroideas y no es sinonimo de


hipertiroidismo, que es el resultado de un exceso de funcion tiroidea. No obstante, las principales
causas de la tirotoxicosis son el hipertiroidismo causado por:
a. Enfermedad de Graves c. Adenomas toxicos
b. Bocio multinodular toxico d. Todas las anteriorres

ANS: D
La tirotoxicosis se defi ne como el exceso de hormonas tiroideas y no es sinonimo de
hipertiroidismo, que es el resultado de un exceso de funcion tiroidea. No obstante, las principales
causas de la tirotoxicosis son el hipertiroidismo causado por la enfermedad de Graves, bocio
multinodular toxico y adenomas toxicos.

PTS: 1 DIF: ALTA REF: Harrison. F Principios de Medicina Interna 19


Ed.
OBJ: Reconocer las patologías endocrinas para poder tratarlas de forma adecuada.
TOP: ENDOCRINOLOGIA KEY: patologia endocrina
NOT: DRA. KATHERINE MALDONADO

6. La corteza suprarrenal produce tres clases principales de hormonas corticosteroides:


a. HORMONAS ADRENÉRGICAS d. HORMONA CORTICOTROPA
b. MINERALOCORTICOIDES e. TODAS LAS ANTERIORES
c. GLUCOCORTICOIDES f. SOLO B Y C
ANS: F PTS: 1 DIF: MEDIA
REF: Harrison F. Principios de Medicina Interna 19 Ed.
OBJ: Reconocer las patologías endocrinas para poder tratarlas de forma adecuada.
TOP: ENDOCRINOLOGIA KEY: patologia endocrina
NOT: DRA,KATHERINE MALDONADO

YES/NO
1. En la Enfermedad de Graves la presentacion clinica, depende de la gravedad de la tirotoxicosis, la
duracion de la enfermedad, la susceptibilidad individual al exceso de hormona tiroidea y la edad
del paciente.

ANS: Y
SI
Los signos y los sintomas abarcan las caracteristicas comunes a cualquier causa de tirotoxicosis y
las especificas de la enfermedad de Graves. La presentacion clinica depende de la gravedad de la
tirotoxicosis, la duracion de la enfermedad, la susceptibilidad individual al exceso de hormona
tiroidea y la edad del paciente.

PTS: 1 DIF: ALTA REF: Harrison F. Principios de Medicina Interna 19


Ed.
OBJ: Reconocer las patologías endocrinas para poder tratarlas de forma adecuada.
TOP: ENDOCRINOLOGIA KEY: patologia endocrina
NOT: DRA. KATHERINE MALDONADO

COMPLETION

1. Manifestaciones.................... son: edema facial; mixedema; cabello seco, escaso y grueso; piel
seca, gruesa, escamosa y áspera; carotenemia, particularmente notable en las palmas y plantas
(causada por el depósito de caroteno en las capas epidérmicas ricas en lípidos); macroglosia debido
a la acumulación de la sustancia fundamental proteinácea en la lengua

ANS: Manifestaciones dermatológicas: edema facial; mixedema; cabello seco, escaso y grueso;
piel seca, gruesa, escamosa y áspera; carotenemia, particularmente notable en las palmas y plantas
(causada por el depósito de caroteno en las capas epidérmicas ricas en lípidos); macroglosia debido
a la acumulación de la sustancia fundamental proteinácea en la lengua

PTS: 1 DIF: ALTA


REF: Harrison F. Principios de Meidicina Interna 19 Ed.
OBJ: Reconocer las patologías endocrinas para poder tratarlas de forma adecuada.
TOP: ENDOCRINOLOGIA KEY: patologia endocrina
NOT: DRA. KATHERINE MALDONADO

2. Sindrome de Cushing refleja una constelación de caracteristicas clinicas que son resultado de la
............. crónica al exceso de glucocorticoides de cualquier causa.

ANS: Sindrome de Cushing refleja una constelacion de caracteristicas clinicas que son resultado
de la exposicion cronica al exceso de glucocorticoides de cualquier causa.

PTS: 1 DIF: ALTA REF: Harrison F. Principios de Medicina Interna 19


Ed.
OBJ: Reconocer las patologías endocrinas para poder tratarlas de forma adecuada.
TOP: ENDOCRINOLOGIA KEY: patologia endocrina
NOT: DRA KATHERINE MALDONADO

MATCHING

En relación a las Neoplasias de tiroides relacione las 1. Neoplasias Benignas y 2. Malignas.


a. Adenomas de celulas epiteliales e. Adenoma macrofolicular
foliculares
b. Carcinoma folicular f. Carcinomas papilares
c. Linfoma g. Adenoma microfolicular
d. Adenoma trabecular h. Cancer medular
1. 1
2. 2
3. 2
4. 1
5. 1
6. 2
7. 1
8. 2

1. ANS: A PTS: 1 DIF: ALTA


REF: Harrison F. Principios de Medicina Interna 19 Ed.
OBJ: Reconocer las patologías endocrinas para poder tratarlas de forma adecuada.
TOP: ENDOCRINOLOGIA KEY: patologia endocrina
NOT: DRA. KATHERINE MALDONADO
2. ANS: B PTS: 1
3. ANS: C PTS: 1
4. ANS: D PTS: 1
5. ANS: E PTS: 1
6. ANS: F PTS: 1
7. ANS: G PTS: 1
8. ANS: H PTS: 1

SHORT ANSWER

1. La Diabetes Mellitus constituye una serie de alteraciones que derivan en hiperglicemia, de su


control o no depende la aparición de complicaciones. Para establecer el diagnóstico mencione los
criterios de diagnóstico.

ANS:
CRITERIOS DIAGNOSTICOS DE DIABETES
Sintomas de diabetes mas concentracion de glucemia al azar ? .11.1 mmol/L
(200 mg/100 mL) a o bien
Glucosa plasmatica en ayunas .7.0 mmol/L (126 mg/100 mL)b o bien
Hemoglobina A1c ? .6.5%c o bien
Glucosa plasmatica a las 2 h ? .11.1 mmol/L (200 mg/100 mL) durante una
prueba oral de tolerancia a la glucosad

PTS: 1 DIF: ALTA REF: Harrison Medicina Interna 9 Ed.


OBJ: Reconocer las patologías endocrinas para poder tratarlas de forma adecuada.
TOP: ENDOCRINOLOGIA MSC: patologia endocrina
NOT: DRA. KATHERINE MALDONADO
EVALUACION DE GASTROENTEROLOGIA

True/False
Indicate whether the statement is true or false.

____ 1. Del siguiente enunciado señale si es VERDADERO O FALSO


La concepción actual de ENFERMEDAD POR REFLUJO GASTROESOFÁGICO es que comprende una
familia de padecimientos que tienen en comun su origen en la disfagia el cual desencadena sintomas
problemáticos o manifestaciones potenciales esofagicas y extraesofagicas
ANS: F
____ 2. Del siguiente enunciado señale si es VERDADERO O FALSO.
La hepatitis viral es una enfermedad infecciosa que afecta al hígado. Hasta el momento, han sido
reconocidos y caracterizados minuciosamente en humanos cinco diferentes virus de la hepatitis: el virus
de la hepatitis A(VHA), el virus de la hepatitis B (VHB), el virus de lahepatitis C (VHC), el virus de la
hepatitis D (VHD) y el virusde la hepatitis E (VHE).
Los cuales son transmitidos todos por via parenteral.
ANS: F

Multiple Choice
Identify the choice that best completes the statement or answers the question.

____ 3. El tubo digestivo se extiende desde la boca hasta el ano y esta compuesto por varios organos con
diferentes funciones. Los organos se separan por medio de esfinteres engrosados y especializados con
control independiente, que ayudan a la division del intestino en compartimientos. En relacion a los
enfinteres existentes en el esofago, señale la opción incorrecta.
a. ESFINTER ESOFAGICO INFERIOR c. ESFINTER PILÓRICO
b. ESFINTER ESOFAGICO SUPERIOR d. NINGUNO DE LOS ANTERIORES
____ 4. La deglución es un acto más complejo, que incluye al depósito de alimentos en la boca y su paso hasta
que el bolo alimentario por el esofago llegue al estomago y siga su trayectoria.En relación a la fisiología
de la deglución, señale la respuesta correcta de las fases de la deglución

a. FASE ORAL c. FASE ESOFÁGICA


b. FASE FARINGEA d. TODAS LAS ANTERIORES
____ 5. El tratamiento no farmacológico de la Enfermedad por Reflujo Gastroesofágico, requiere la realización de
varias medidas. Escoja la opción correcta.
a. Evitar los alimentos que aumentan la c. Adoptar conductas que lleven al
presión del esfínter esofágico inferior reflujo y la pirosis

b. Evitar los alimentos ácidos que son d. Todas las anteriores


esencialmente irritantes

____ 6. En relación a la función del intestino delgado y el colon, además de la digestión y absorción de los
nutrientes, el epitelio intestinal tiene muchas otras funciones: Señale la opción correcta en relación a las
mismas:

a. Sintesis y secreción de proteinas d. solo a y b


b. Defensa inmunitaria e. Todas las anteriores
c. Formacion de bolo alimenticio f. Ninguna de las anteriores
____ 7. Más de 30 años después de su descubrimiento, la infección por H. pylori es la causa principal de varias
enfermedades, de las siguientes opciones escoja la respuesta incorrecta:
a. gastritis crónica c. linfoma MALT gástrico
b. úlcera péptica d. adenocarcinoma hepático
____ 8. Señale la opcion correcta
El estreñimiento crónico (EC) es una condición muy frecuente que afecta a la población general y que
impacta la calidad de vida de los sujetos que la padecen, se define como:
a. Disminución en la frecuencia de las d. Todas las anteriores
evacuaciones
b. Aumento en la consistencia de las heces e. Ninguna de las anteriores
c. Dificultad para expulsar las heces

Multiple Response
Identify one or more choices that best complete the statement or answer the question.

____ 9. La pancreatitis aguda consiste en una inflamación aguda, de inicio repentino, de más o menos intensidad,
en la que, tras el episodio, se recupera la forma y la función de la glándula por completo. De las siguientes
opciones señale las principales causas de este padecimiento:

a. Colelitiasis c. Consumo excesivo de alcohol


b. Hipertrigliceridemia d. Alteraciones de la tiroides

Completion
Complete each statement.

10. La Disfagia constituye la sensación subjetiva de dificultad para el paso del bolo alimenticio desde la
faringe hasta el estómago, en relación a este tema, escriba la clasificación de la Disfagia.
ANS: La disfagia se clasifica en disfagia orofaringea y esofágica

11. La Enfermedad Acido péptica constituye una serie de alteraciones caracterizadas


por:....................................................................................................................................................................
..........................................................................................................................................................................
..........................................................................
ANS: La Enfermedad Acido péptica constituye una serie de alteraciones caracterizadas por: dolor
epigastrico urente, exacerbado por el ayuno y que mejora con la alimentaciónn

12. De la Etiologia de la Enfermedad Acido Peptica mencione las causas de Enfermedad acido peptica.

.........................................
.........................................
.........................................
ANS:
Infección por Helicobacter Pilory
Consumo de AINES
Idiopáticas
13. De los factores de riesgo para desarrollar Cáncer Colorrectal coloque 5 de ellos:
1. ...............................................................
2. ...............................................................
3. ...............................................................
4. ...............................................................
5. ...............................................................
ANS:
De los factores de riesgo para desarrollar Cáncer Colorrectal coloque 5 de ellos:
Antecedentes familiares de cáncer colorrectal en un pariente de primer grado.
Antecedentes personales de adenomas colorrectales, cáncer colorrectal o cáncer de ovario.
Afecciones hereditarias, como la poliposis adenomatosa familiar (PAF) y el síndrome de Lynch (cáncer
de colon sin poliposis hereditario [HNPCC]).
Antecedentes personales de colitis ulcerosa crónica o colitis de Crohn duraderas.
Consumo excesivo de alcohol.
Tabaquismo.
Raza u origen étnico: afroamericano.
Obesidad

Matching

Enlace segun corresponda el producto de la secrecion de las celulas del Estómago.


1. Pepsinogeno 2. Gastrina. 3. Mucosa. 4.Acido clorhídrico
a. Celulas principales
b. Celulas parietales
c. Celulas G
d. Celulas mucosas
____ 14. 1a 2c 3d 4b

Short Answer

15. Al siguiente enunciado responda brevemente SI O NO,argumente su respuesta


Los trastornos de la absorcion representan una amplia variedad de procesos con multiples causas y
diversas manifestaciones clinicas. Casi todos estos problemas clinicos se relacionan con decremento de la
absorcion intestinal de uno o mas nutrientes de la dieta, y con frecuencia se los denomina sndrome de
malabsorcion (absorcion defi ciente). Segun este criterio el Sindrome de Malaabsorción se considerariá
como un diagnóstico?
ANS:
NO, El termino malabsorción no es el mas correcto, ya que representa un estado fisiopatologico, pero no
proporciona una explicacion etiologica del problema subyacente y por tanto no debe considerarse un
diagnostico final adecuado
DERMATOLOGIA

____ 1.
Cambio de color de la piel sin relieve ni cambio de textura, por lo que no es palpable, hablamos de (escoja
la respuesta correcta)
a. Pápula
b. Macula

c. Habón

d. Escama
e. A y b
f. Ninguna

____ 2.
Lesión donde encontramos eritema, vesiculación mínima, existe humedad al tacto, no hay exudación,
hablamos de: (escoja la respuesta correcta)
a. Estado agudo de la piel
b. Estado subagudo de la piel

c. Estado Sobreagudo de la piel

d. Estado Seco

____ 3.
Cómo confirmaría el diagnóstico de una Dermatofitosis (escoja la respuesta correcta)
a. Mediante un examen microscópico directo de las escamas del centro de la placa
b. Mediante un examen microscópico directo de las escamas de la periferia de la placa

c. Mediante serología específica para dermatofitos

d. Mediante un Citodiagnóstico de Tzank


e. Mediante una Inmunofluorescencia Indirecta

____ 4.
Niña de 11 años de edad, con antecedentes personales de rinitis y familiares de asma bronquial ( padre y
hermana mayor ), que acudió a la consulta por lesiones muy pruriginosas que le impedían conciliar el
sueño , que se originaron en los pliegues de flexión y el cuello hace años , que evolucionaban a brotes de
distinta intensidad y que a lo largo de los últimos meses se extendieron al resto del tegumento. Al
examen físico la paciente presentaba numerosas lesiones eritematosas, exudativas y algunas costras
entremezcladas con lesiones de morfología lineal como consecuencia del intenso rascamiento. Cual es el
diagnostico más probable
a. Dermatitis Seborreica
b. Impétigo
c. Dermatitis atópica

d. Escabiosis
e. Forma infantil de la Dermatitis IgA lineal

____ 5.
Son signos característicos de la lesión Psoriásica todos los siguientes, excepto:
a. Halo de Woronoff
b. Signo del roció sangrante

c. Estrías de Wickham

d. Membrana de Duncan Dulckley


e. Signo de la mancha de cera

____ 6. Las lesiones elementales del liquen plano son:


a. Vesículas
b. Habones

c. Pápulas de superficie lisa , poligonales

d. Pápulas umbilicadas
e. Maculas hipocrómicas

____ 7. Varón de 35 años con antecedentes de hipotiroidismo autoinmune que consulta por la aparición
espontánea y la extensión progresiva de máculas acrómicas, asintomáticas y con distribución simétrica
en ambos codos, las rodillas, los tobillos y los dedos de las manos, por varios meses de evolución.
a. Esclerosis tuberosa
b. Vitíligo

c. Hipopigmentación postinflamatoria

d. Nevus acrómico
e. Albinismo

____ 8. El Melasma :
a. Es intensamente pruriginoso
b. Responde a la corticoterapia tópica

c. Se pigmenta más tras la exposición solar

d. Se localiza habitualmente en el dorso de las manos


e. Desaparece al dejar los anovulatorios

____ 9. La alteración primaria en la patogenia del acne es :


a. Un trastorno endocrinológico
b. Una infección
c. Una hiperproducción sebácea

d. Un trastorno de la queratinización folicular


e. Un trastorno inmunitario

____ 10. Ante un paciente de 67 años que consulta por una pápula de aspecto brillante, perlada, asintomática,
localizada en la punta nasal, ¿Qué deberíamos sospechar?
a. Queratosis actínica
b. Carcinoma Escamoso

c. Leucoplasia

d. Carcinoma Basocelular
e. Queratoacantoma

____ 11. Cual de las siguientes es una manifestación clínica de la dermatitis seborreica del lactante
a. Costra láctea
b. Palmas hiperlineales

c. Muguet

d. Pitiriasis versicolor
e. Todas
Endocrinología
Answer Section

MULTIPLE CHOICE

1. ANS: B
Paciente femenina de 38 años de edad, con antecedente de hiperprolactinemia, es
referida a consulta externa de endocrinología por cambios en sus rasgos faciales,
caracterizados por prognatismo, abombamiento frontal y nariz grande, además de
aumento de tamaño de los pies y manos, tras realizarse todos los estudios pertinentes,
se indicó estudio histopatológico que informó adenomas productor de hormona de
crecimiento. Debe plantearse que podría haber otras manifestaciones clínicas, como:

a) Es incorrecta porque son síntomas sugerentes de hipertiroidismo


b) Es correcta porque en la acromegalia surge visceromegalias que pueden
generalizarse.
c) Es incorrecta porque son síntomas sugerentes de síndrome de Cushing.

d) Es incorrecta porque constituye la triada clásica del Feocromocitoma.

PTS: 1 DIF: Baja


REF: HARRISON Tinsley: Principios de medicina interna, 19ª Edición McGraw-Hill Interamericana de
España 2016. Pag. 2269 - 2271
OBJ: Establece un diagnóstico clínico relacionado con los trastornos de la hipófisis y el hipotálamo.
TOP: Medicina Interna - Endocrinología KEY: Acromegalia, prognatismo.
NOT: Autora: Rebeca Silvestre Ramos
2. ANS: C
Paciente masculino de 42 años de edad, acude por bocio, refiere dificultad para
concentrarse en el trabajo, aumento de peso no relacionado con mayor ingesta, ya que
además presentó pérdida de apetito, en la última semana se asocia párpados
edematosos y edema pretibial sin fóvea. De acuerdo a su sospecha diagnóstica se
puede afirmar que:
a) Es incorrecta porque estos pueden sugerir hipertiroidismo secundario.

b) Es incorrecta porque constituyen signos clínicos de oftalmopatía tiroidea.


c) Es correcta porque en el hipotiroidismo primario la TSH se encuentra elevada debido
a un mecanismo de retroalimentación.
d) Es incorrecta porque constituyen el diagnóstico diferencial de Feocromocitoma.

PTS: 1 DIF: Media


REF: HARRISON Tinsley: Principios de medicina interna, 19ª Edición McGraw-Hill Interamericana de
España 2016. Pag. 2289 - 2292.
OBJ: Identifica los trastornos de la glándula tiroidea y paratiroidea.
TOP: Medicina Interna - Endocrinología KEY: Bocio, proptosis.
NOT: Autora: Rebeca Silvestre Ramos
3. ANS: C
Paciente femenina de 52 años de edad, con sobrepeso, acude por presentar poliuria,
polidipsia y pérdida de peso de 6 meses de evolución, en la última semana se asocia
debilidad y visión borrosa, su sospecha diagnostica es diabetes mellitus tipo 2, por lo
que podría esperarse los siguientes resultados de laboratorio:

a) Es incorrecta porque hay aumento de hemoglobina glicosilada


b) Es incorrecta porque la glucosa debe ser superior a 200mg/dl.
c) Es correcta porque ambas constituyen los criterios diagnósticos de diabetes mellitus
tipo 2.

d) Es incorrecta porque la intolerancia a la glucosa excluye el diagnóstico de diabetes


mellitus tipo 2.

PTS: 1 DIF: Media


REF: HARRISON Tinsley: Principios de medicina interna, 19ª Edición McGraw-Hill Interamericana de
España 2016. Pag. 2399 - 2407.
OBJ: Establece factores de riesgo para el desarrollo de Síndrome Metabólico y diabetes mellitus, para
favorecer la intervención oportuna mediante la prevención y promoción.
TOP: Medicina Interna - Endocrinología KEY: Diabetes, glucosa
NOT: Autora: Rebeca Silvestre Ramos
4. ANS: D
Paciente femenina de 46 años de edad, es referida por obesidad, refiere haber seguido
diferentes dietas, además de ejercicios de moderada intensidad en los últimos 6 meses,
sin lograr disminución de peso, al examen físico se constata presión arterial
146/92mmHg, piel delgada, rubicundez facial, cara redonda, giba de búfalo, estrías
violáceas, se solicita medir ACTH y se encuentra disminuida; el diagnóstico probable
es:
a) Es incorrecta porque predominan síntomas asociados a hiperandrogenismo, los
descritos son típicos del síndrome de Cushing.
b) Es incorrecta, porque sólo coincide con la Hipertensión Arterial, sin embargo esta
es paroxística, el resto de las manifestaciones son típicas del Síndrome de
Cushing.
c) Es incorrecta porque en la Enfermedad de Cushing la ACTH se encuentra
normal o elevada.

d) Es correcta, porque son manifestaciones del exceso de glucocorticoides y en


ocasiones de mineralocorticoides.

PTS: 1 DIF: Media


REF: HARRISON Tinsley: Principios de medicina interna, 19ª Edición McGraw-Hill Interamericana de
España 2016. Pag. 2399 - 2407.
OBJ: Identifica, relaciona y aplica el conocimiento de las enfermedades de las Glándulas Suprarrenales y
gónadas. TOP: Medicina Interna - Endocrinología KEY: Suprarrenal, Cushing
NOT: Autora: Rebeca Silvestre Ramos
5. ANS: B
Paciente femenina de 49 años de edad, en postoperatorio inmediato de tiroidectomía
total por carcinoma papilar de tiroides, que empieza con espasmos musculares y
gesticulación facial, al examen físico presenta positividad del signo de Trousseau, para
corroborar su diagnóstico se solicitaría con prioridad:
a) Es incorrecta porque se solicita cuando de plantea Osteoporosis, no es un
examen que se use con prioridad en estos casos.

b) Es correcta, ya que la medición de calcio será la prueba inicial para corroborar


hipocalcemia y la parathormona confirmará que el mismo es secundario a
hipoparatiroidismo.

c) Es incorrecta porque los mismos no son prioritarios, estos pueden solicitarse con
posterioridad.
d) Es incorrecta, ya que estas son de prioridad cuando se sospecha enfermedades
cardíacas.

PTS: 1 DIF: Media


REF: HARRISON Tinsley: Principios de medicina interna, 19ª Edición McGraw-Hill Interamericana de
España 2016. Pag. 2482 - 2495.
OBJ: Identifica los trastornos de la glándula tiroidea y paratiroidea.
TOP: Medicina interna - Endocrinología KEY: Calcio, vitamanina D.
NOT: Autora: Rebeca Silvestre Ramos.
6. ANS: D
Paciente masculino de 57 años de edad con antecedentes de radiación de cabeza y
cuello, acude por aumento de volumen unilateral de cuello, se realiza ecografía de
tiroides y reporta nódulo tiroideo sospechoso de malignidad, con TSH, T4 libre y T3
libre en rango de la normalidad, su conducta de elección debe ser solicitar:
a) Es incorrecto, porque la gammagrafía de tiroides se solicita cuando contamos
con perfil tiroideo compatible con nódulo tiroideo hiperduncionante o tóxico, en
ese caso T4 libre y T3 libre se encuentran elevados.

b) Es incorrecto, porque no es de elección para confirmar cáncer de tiroides,


c) Es incorrecto, porque la tiroglobulina se solicita para seguimiento postoperatorio
de cáncer de tiroides.
d) Es correcto, porque describirá las características histopatológicas para confirmar
el diagnóstico.

PTS: 1 DIF: Baja


REF: HARRISON Tinsley: Principios de medicina interna, 19ª Edición McGraw-Hill Interamericana de
España 2016. Pag. 2482 - 2495.
OBJ: Identifica los trastornos de la glándula tiroidea y paratiroidea.
TOP: Medicina Interna - Endocrinología. KEY: Nódulo.
NOT: Autora: Rebeca Silvestre Ramos
7. ANS: D
Paciente masculino de 32 años de edad, que se encuentra en terapia intensiva por
Traumatismo Craneoencefálico Severo, Glasgow 8 puntos, presenta poliuria de 6.500ml
en las últimas 24 horas, con sodio 145mEq/L, osmolaridad normal, el diagnóstico más
probable sería:

a) Es incorrecta, porque en el SIADH hay hiponatremia euvolémica con


hipoosmolalidad.
b) Es incorrecta, ya que por el antecedente de traumatismo craneoencefálico es
más planteable la Diabetes insípida hipofisiaria o central.
c) Es incorrecta, porque la polidipsia dipsógena se caracteriza por sed inapropiada,
un paciente con Glasgow de 8 puntos no tolera vía oral.

d) Es correcta, ya que una de las causas de Diabetes insípida hipofisiaria o central


es el traumatismo craneoencefálico y se caracteriza por poliuria importante.

PTS: 1 DIF: Media


REF: HARRISON Tinsley: Principios de medicina interna, 19ª Edición McGraw-Hill Interamericana de
España 2016. Pag. 2275 - 2280.
OBJ: Establece un diagnóstico clínico relacionado con los trastornos de la hipófisis y el hipotálamo.
TOP: Medicina Interna - Endocrinología. KEY: Diabetes insípida.
NOT: Autora: Rebeca Silvestre Ramos
8. ANS: B
Dentro de los criterios de Síndrome metabólico, según ATP III, se incluyen:
a) Es incorrecta, porque el perímetros abdominal es mayor a 102cm en mujeres y
88cm en hombres, la hipercolesterolemia no está dentro de los criterios.

b) Es correcta, porque los niveles de colesterol no son parte del Síndrome


Metabólico.
c) Es incorrecta, porque Colesterol HDL alto y perímetro abdominal mayor a 102 en
mujeres.
d) Es incorrecta, porque los valores de HDL están bajos.

PTS: 1 DIF: Media


REF: HARRISON Tinsley: Principios de medicina interna, 19ª Edición McGraw-Hill Interamericana de
España 2016. Pag. 2449 - 2454
OBJ: Establece factores de riesgo para el desarrollo de Síndrome Metabólico y diabetes mellitus, para
favorecer la intervención oportuna mediante la prevención y promoción.
TOP: Medicina Interna - Endocrinología KEY: Síndrome metabólico.
NOT: Autora: Rebeca Silvestre Ramos
9. ANS: B
Paciente masculino de 32 años de edad, que es referido por bocio nodular, refiere
disminución de peso, irritabilidad y deposiciones diarreicas, al examen físico se
constata: frecuencia cardíaca: 108 latidos por minuto, proptosis, piel caliente, temblor
fino en manos, se diagnosticó hipertiroidismo primario. Los exámenes complementarios
que sustentaron al diagnóstico serían:
a) Es incorrecto, porque en el hipertiroidismo primario la TSH se encuentra baja.

b) Es correcto, ya que el perfil tiroideo se correlaciona con hipertiroidismo


primario y la gammagrafía corrobora nódulo hiperfuncionante o tóxico.
c) Es incorrecto, porque en el hipertiroidismo primario la TSH se encuentra baja.
d) Es incorrecto, porque la T4 se encuentra baja.

PTS: 1 DIF: Media


REF: HARRISON Tinsley: Principios de medicina interna, 19ª Edición McGraw-Hill Interamericana de
España 2016. Pag. 2293 - 2299
OBJ: Identifica los trastornos de la glándula tiroidea y paratiroidea.
TOP: Medicina Interna - Endocrinología. KEY: hipertiroidismo
NOT: Autora: Rebeca Silvestre Ramos
10. ANS: D
Las siguientes manifestaciones son parte de las complicaciones microvasculares de la
diabetes mellitus:

a) Es incorrecta, porque las Coronariopatías son complicaciones macrovasculares.


b) Es incorrecta, porque la enfermedad cerebrovascular es una complicación
macrovascular.

c) Es incorrecta, porque la Arteriopatía periférica, es una complicación


macrovascular.
d) Es correcta, porque las tres son complicaciones microvasculares.

PTS: 1 DIF: Media


REF: HARRISON Tinsley: Principios de medicina interna, 19ª Edición McGraw-Hill Interamericana de
España 2016. Pag. 2293 - 2299
OBJ: Establece factores de riesgo para el desarrollo de Síndrome Metabólico y diabetes mellitus, para
favorecer la intervención oportuna mediante la prevención y promoción.
TOP: Medicina Interna - Endocrinología. KEY: Diabetes.
NOT: Autora: Rebeca Silvestre Ramos
11. ANS: A
Acerca del tratamiento farmacológico de la Diabetes Mellitus, la siguiente afirmación se
considera correcta:
a) Es correcta porque las biguanidas disminuyen producción hepática de glucosa y
mejoran ligeramente la utilización periférica de ese azúcar.

a) Es incorrecta. La metformina no es un secretagogo de insulina, es parte de las


biguanidas.
b) Es incorrecta. Los inhibibidores de la enzima dipeptidil-peptidasa IV, actúan
prolongando la acción de GLP-1 endógena y los inhibidores del cotransportador
sodio-glucosa son los que reducen la glucemia por inhibición selectiva de este
cotransportador sodio-glucosa, que se expresa casi de manera exclusiva en el
túbulo contorneado proximal de los riñones.

c) Las insulinas glargina y detemir, son insulinas de acción prolongada.

PTS: 1 DIF: Alta


REF: HARRISON Tinsley: Principios de medicina interna, 19ª Edición McGraw-Hill Interamericana de
España 2016. Pag. 2407 - 2422
OBJ: Establece factores de riesgo para el desarrollo de Síndrome Metabólico y diabetes mellitus, para
favorecer la intervención oportuna mediante la prevención y promoción.
TOP: Medicina Interna - Endocrinología KEY: Biguanidas NOT: Autor: Rebeca Silvestre Ramos
12. ANS: D
Acerca de la fisiología de la hipófisis, la siguiente afirmación se considera correcta:
a) Es incorrecta. En la neurohipófisis no se sintetizan hormonas, sólo se almacenan
y liberan hormonas como la vasopresina u hormona antidiurética y oxicitocina,
además la prolactina se sintetiza en la adenohipófisis.
b) Es incorrecta. La hipófisisis tiene regulación por sistemas neuroendocrinos como
el hipotálamo.
a) Es incorrecta. Las hormonas hipofisarias raras veces provocan respuestas
específicas en otros tejidos glandulares periféricos.
b) Es correcta. La adenohipófisis produce seis hormonas importantes: hormona del
crecimiento (GH) hormona adrenocorticotropica (ACTH), hormona luteinizante
(LH), hormona foliculoestimulante (FSH) hormona estimulante de la tiroides
(TSH) y prolactina.

PTS: 1 DIF: Baja


REF: HARRISON Tinsley: Principios de medicina interna, 19ª Edición McGraw-Hill Interamericana de
España 2016. Pag. 2255 - 2261.
OBJ: Establece un diagnóstico clínico relacionado con los trastornos de la hipófisis y el hipotálamo.
TOP: Medicina Interna - Endocrinología KEY: Adenohipófisis
NOT: Autor: Rebeca Silvestre Ramos
13. ANS: B
El hipotiroidismo es una enfermedad con relevante prevalencia en las consultas de
endocrinología, acerca del mismo, ¿cuál planteamiento se considera correcto?:
a) Es incorrecto. La punción del talón como tamizaje se realiza a todo recién
nacido, independientemente si hay o no manifestaciones clínicas de la
enfermedad.
b) Es correcto. En áreas en las que hay suficiente yodo, la enfermedad
autoinmunitaria (tiroiditis de Hashimoto) es causa frecuente de hipotiroidismo.
c) Es incorrecto. Las manifestaciones clínicas del hipotiroidismo no incluyen
disminución de peso y sudoración profusa.
d) Es incorrecto. Una vez diagnosticado el hipotiroidismo, el tratamiento consiste en
levotiroxina, las tionamidas son para tratar casos con hipertiroidismo.

PTS: 1 DIF: Media


REF: HARRISON Tinsley: Principios de medicina interna, 19ª Edición McGraw-Hill Interamericana de
España 2016. Pag. 2289 - 2293.
OBJ: Identifica los trastornos de la glándula tiroidea y paratiroidea.
TOP: Medicina Interna - Endocrinología KEY: Hipotiroidismo
NOT: Autora: Rebeca Silvestre Ramos
14. ANS: A
La tirotoxicosis se define como el exceso de hormonas tiroideas, acerca de esta
patología, ¿Cuál enunciado se considera correcto?
a) Es correcto. Una de las principales causas de la tirotoxicosis es la enfermedad
de Graves.
b) Es incorrecto. La manifestación cardiovascular más frecuente es la taquicardia
sinusal.
c) Es incorrecto. La levotiroxina no es un antitiroideo.
d) Es incorrecto. Los beta bloqueadores son útiles para el control de síntomas
adrenérgicos.

PTS: 1 DIF: Media


REF: HARRISON Tinsley: Principios de medicina interna, 19ª Edición McGraw-Hill Interamericana de
España 2016. Pag. 2293 - 2307.
OBJ: Identifica los trastornos de la glándula tiroidea y paratiroidea.
TOP: Medicina Interna - Endocrinología KEY: Tirotoxicosis
NOT: Autora: Rebeca Silvestre Ramos
15. ANS: C
A su consulta acude paciente masculino de 52 años de edad, con diagnóstico de
diabetes mellitus tipo 2 desde hace tres meses, dentro de los objetivos terapeúticos
¿Cuál enunciado es el correcto?
a) Es incorrecta. La glucosa en ayunas debe estar entre 80 – 130mg/dl.
b) Es incorrecta. La hemoglobina glicosilada debe ser menor a 7% y la glucosa
posprandial menor a 180mg/dl.
c) Es correcta. La lipoproteína de baja densidad debe ser menor a 100mg/dl, la
hemoglobina glicosilada menor a 7%, los triglicéridos menor a 150mg/dl.
d) Es incorrecta. La lipoproteína de baja densidad debe estar menos a 100mg/dl.

PTS: 1 DIF: Media


REF: HARRISON Tinsley: Principios de medicina interna, 19ª Edición McGraw-Hill Interamericana de
España 2016. Pag. 2407 - 2422.
OBJ: Establece factores de riesgo para el desarrollo de Síndrome Metabólico y diabetes mellitus, para
favorecer la intervención oportuna mediante la prevención y promoción.
TOP: Medicina Interna II - Endocrinología KEY: Diabetes.
NOT: Autora: Rebeca Silvestre Ramos
OFTALMOLOGIA

Opción Múltiple
Elige la opción que complete mejor el enunciado o que responda mejor a la pregunta.

____ 1. SEÑALE LO CORRECTO SOBRE MIOPIA


a. MIOPIA ES UNA AMETROPIA EN c. MIOPIA ES UNA AMETROPIA EN
QUE LOS RAYOS DE LUZ TIENEN SU QUE LOS RAYOS DE LUZ TIENEN SU
PUNTO FOCAL POR DELANTE DE LA PUNTO FOCAL POR DETRAS DE LA
RETINA RETINA
b. MIOPIA ES UNA AMETROPIA EN d. NINGUNA DE LAS ANTERIORES
QUE LOS RAYOS DE LUZ
PRESENTAN VARIOS PUNTOS
FOCALES UNO POR DELANTE Y
OTRO POR DETRAS DE LA RETINA
____ 2. SEÑALE LO CORRECTO. SOBRE CATARATA.
a. CATARATA ES UNA c. CATARATA ES CUALQUIER
OPACIFICACION DE LA CORNEA OPASIFICACION DEL CRISTALINO
b. CATARATA ES UNA d. NINGUNA DE LAS ANTERIORES
DEGENERACION FIBROSA DE LA
CONJUNTIVA
____ 3. SOBRE ANATOMIA SEÑALE LO CORRECTO
a. LA CAPA MEDIA DEL OJO ESTA c. LA CAPA MEDIA DEL OJO ESTA
FORMADA POR IRIS, CUERPO FORMADA POR IRIS, CUERPO
CILIAR Y COROIDES CILIAR Y RETINA
b. LA CAPA MEDIA DEL OJO ESTA d. LA CAPA MEDIA DEL OJO ESTA
FORMADA POR CRISTALINO, FORMADA POR VITREO, CUERPO
CUERPO CILIAR Y COROIDES CILIAR Y COROIDES

____ 4. SEÑALE LO CORRECTO SOBRE ANATOMIA


a. EL SEXTO PAR CRANEAL INERVA c. EL SEXTO PAR CRANEAL INERVA
AL RECTO EXTERNO AL OBLICUO MAYOR
b. EL SEXTO PAR INERVA AL d. EL SEXTO PAR CRANEAL INERVA
OBLICUO MENOR AL RECTO INTERNO
____ 5. SEÑALE LO CORRECTO SOBRE HIPERMETROPIA
a. LA HIPERMETROPIA SE c. LA HIPERMETROPIA SE
CARACTERIZA POR MALA VISION CARACTERIZA POR MALA VISION
DE CERCA, Y SE CORRIGA CON DE LEJOS, Y SE CORRIGA CON
LENTES CONCAVAS LENTES CONCAVAS
b. LA HIPERMETROPIA SE d. LA HIPERMETROPIA SE
CARACTERIZA POR MALA VISION CARACTERIZA POR MALA VISION
DE CERCA, Y SE CORRIGA CON DE LEJOS, Y SE CORRIGA CON
LENTES CONVEXAS LENTES COVEXAS
____ 6. SEÑALE LO CORRECTO SOBRE QUERATOCONO
a. EL QUERATOCONO ES UNA c. EL QUERATOCONO ES UNA
ENFERMEDAD CARACTERIZA POR ENFERMEDAD CARACTERIZA POR
ADELGAZAMIENTO DEL ESTROMA ENGROSAMIENTO DEL ESTROMA
CORNEAL QUE PUEDE PROVOCAR CORNEAL QUE PUEDE PROVOCAR
EN LOS PEORES CASOS UN EN LOS PEORES CASOS UN
TRANSPLANTE CORNEAL TRANSPLANTE CORNEAL
b. EL QUERATOCONO ES UNA d. EL QUERATOCONO ES UNA
ENFERMEDAD CARACTERIZA POR ENFERMEDAD CARACTERIZA POR
ADELGAZAMIENTO DEL ESTROMA ADELGAZAMIENTO DEL ESTROMA
CONJUNTIVAL QUE PUEDE ESCLERAL QUE PUEDE PROVOCAR
PROVOCAR EN LOS PEORES CASOS EN LOS PEORES CASOS UN
UN TRANSPLANTE CORNEAL TRANSPLANTE CORNEAL
____ 7. SEÑALE LOC ORRECTO SOBRE GLAUCOMA
a. EL GLAUCOMA ES UNA PATOLOGIA c. EL GLAUCOMA ES UNA PATOLOGIA
CARACTERIZADA POR DANO DEL CARACTERIZADA POR DANO DEL
NERVIO OPTICO QUE SE NERVIO TRIGEMINO QUE SE
MANIFIESTA CON PERDIDA DEL MANIFIESTA CON PERDIDA DEL
CAMPO VISUAL PUEDE O NO ESTAR CAMPO VISUAL PUEDE O NO ESTAR
ACOMPAÑADO DE AUMENTO DE LA ACOMPAÑADO DE AUMENTO DE LA
PRESION INTRAOCULAR PRESION INTRAOCULAR
b. EL GLAUCOMA ES UNA PATOLOGIA d. EL GLAUCOMA ES UNA PATOLOGIA
CARACTERIZADA POR DANO DEL CARACTERIZADA POR DANO DEL
NERVIO OPTICO QUE SE NERVIO PATETICO QUE SE
MANIFIESTA CON PERDIDA DEL MANIFIESTA CON PERDIDA DEL
CAMPO VISUAL PUEDE SIEMPRE CAMPO VISUAL PUEDE O NO ESTAR
ACOMPAÑADO DE AUMENTO DE LA ACOMPAÑADO DE AUMENTO DE LA
PRESION INTRAOCULAR PRESION INTRAOCULAR
____ 8. SEÑALE LO CORRECTO SOBRE CONJUNTIVITIS
a. EL GERMEN MAS c. EL GERMEN MAS
FRECUENTEMENTE ASOCIADO A FRECUENTEMENTE ASOCIADO A
CONJUNTIVITIS BACTERIANA CONJUNTIVITIS BACTERIANA
HIPERAGUDA ES NEISSERIA HIPERAGUDA ES ESTAFILOCO
GONORRAE AUREUS
b. EL GERMEN MAS d. EL GERMEN MAS
FRECUENTEMENTE ASOCIADO A FRECUENTEMENTE ASOCIADO A
CONJUNTIVITIS BACTERIANA CONJUNTIVITIS BACTERIANA
HIPERAGUDA ES HAEMOPHILUS HIPERAGUDA ES MORAXELLA
INFLUENZAE CATARRALIS
____ 9. SEÑALE LO CORRECTO SOBRE CELULITIS
a. LA CELULITIS ORBITARIA SE c. LA CELULITIS PRE SEPTAL SE
CARACTERIZA POR PRESENTAR: CARACTERIZA POR PRESENTAR:
OFTALMOPLEJIA DOLOROSA, OFTALMOPLEJIA DOLOROSA,
DISMINUCION DE LA AGUDEZA DISMINUCION DE LA AGUDEZA
VISUAL, ALTERACION DEL REFLEJO VISUAL, ALTERACION DEL REFLEJO
PUPILAR Y PROPTOSIS PUPILAR Y PROPTOSIS
b. LA CELULITIS ORBITARIA SE d. NINGUNA DE LAS ANTERIORES
CARACTERIZA POR PRESENTAR:
OFTALMOPLEJIA DOLOROSA,
FIEBRE, HIPERTENSION ARTERIAL,
ALTERACION DEL REFLEJO
PUPILAR Y PROPTOSIS
Respuesta Múltiple
Identifica una o más respuestas que completen los enunciados o que respondan mejor a la pregunta.

____ 10. SEÑALE LO CORRECTO SOBRE PTERIGION


a. PTERIGION ES UNA c. PTERIGION ES UNA
DEGENERACION FIBROVASCULAR DEGENERACION FIBROVASCULAR
DE LA CONJUNTIVA BULBAR QUE DE LA CONJUNTIVA LIMAR QUE
PUEDE INVADIR CORNEA PUEDE INVADIR CORNEA
b. PTERIGION ES UNA d. NINGUNA DE LAS ANTERIORES
DEGENERACION FIBROVASCULAR
DE LA CONJUNTIVA TARSAL QUE
PUEDE INVADIR CORNEA
Pediatria

Opción Múltiple
Elige la opción que complete mejor el enunciado o que responda mejor a la pregunta.

____ 1.
Diagnósticos diferenciales de hernias pediátricas:

a. Hidrocele, quiste del cordón, c. Hidrocele, pancreatitis, testículo no


neumonía, varicocele descendido, varicocele
b. Hidrocele, quiste del cordón, testículo d. Hidrocele, quiste del cordón,
no descendido, varicocele colelitiasis, varicocele

____ 2. Signo de maltzer


a. Descompresión brusca del abdomen en c. Dolor durante la rotación interna de la
el punto de McBurney cadera
b. La tos puede acentuar dolor d. Dolor intenso en fosa iliaca derecha
por la compresión del punto de
McBurney al mismo tiempo que se
levanta el miembro inferior derecho
____ 3.
Apéndice gangrenada o perforada: grado III-IV:
a. Ceftriaxona: IV. 50-75mg/kg/d x7 c. Ceftriaxona: IV. 50-75mg/kg/d x7
hasta 14 días, Metronidazol: IV. hasta 14 días, Metronidazol: IV.
30mg/kg/d c/8horasx7 hasta 14 días 100mg/kg/d c/8horasx7 hasta 14 días
b. H Ceftriaxona: IV. 30mg/kg/d x7 hasta d. Ceftriaxona: IV. 30mg/kg/d x7 hasta
14 días, Metronidazol: IV. 60mg/kg/d 14 días, Metronidazol: IV. 50-
c/8horasx7 hasta 14 días 75mg/kg/d c/8horasx7 hasta 14 días
____ 4.
Apendicitis: ecografía. INCORRECTA

a. Imagen tubular no compresible c. Diámetro transverso del apéndice


menor a 6 mm
b. Apendicolito d. Hiperecogenicidad de grasa peri
apendicular
____ 5. Tratamiento hormonal de la criptorquidia
a. Hormona gonadotropina coriónica, c. Hormona gonadotropina coriónica,
betaBHCG 5000 U. 3 por semana por betaBHCG 1500 U. 6 por semana por
3 semanas 3 semanas
b. Hormona gonadotropina coriónica, d. Hormona gonadotropina coriónica,
betaBHCG 1500 U. 3 por semana por betaBHCG 1500 U. 3 por semana por
6 semanas 3 semanas
____ 6. Criptorquidia: anatomía patológica. INCORRECTA
a. Disminución de las células germinales c. Degeneración tubular aumentada
de Sertoli
b. Aumento de espermatogonias d. Aumento del tejido intersticial

____ 7. Señale la Correcta. Hernia Epigástrica


a. Es la protrusión del peritoneo sin la c. Es la protrusión del peritoneo y grasa
presencia de grasa peritoneal, que y/o grasa peritoneal, que puede
puede localizarse en cualquier parte localizarse en cualquier parte entre el
entre el apéndice xifoides y el apéndice xifoides y el ombligo. Su
ombligo. Su tratamiento es quirúrgico tratamiento es quirúrgico
b. Es la protrusión del peritoneo y grasa d. Es la protrusión del peritoneo sin
y/o grasa peritoneal, que puede grasa peritoneal, que puede localizarse
localizarse en cualquier parte entre el en cualquier parte entre el apéndice
apéndice xifoides y el ombligo. Su xifoides y el ombligo. Su tratamiento
tratamiento es conservador es conservador
____ 8. Señale la respuesta correcta en relación al concepto de infección del tracto urinario
a. Presencia de bacterias en el torrente c. Son infecciones que ocurren por la
sanguíneo. Puede producirse ingesta de quistes, huevos o larvas,
espontáneamente, durante la infección siendo los helmintos aquellos parásitos
de determinados tejidos, por el uso de que tienen entre 1 mm y 1 metro o
sondas gastrointestinales o catéteres incluso más, clasificándose en
venosos, o después de procedimientos nemátodos o gusanos cilíndricos
odontológicos, digestivos, la curación
de una herida u otras maniobras
b. Se define por la presencia de bacterias d. infección del parénquima renal y del
en la orina (bacteriuria), que al tejido pielocalicial. Fiebre, vomito,
cultivarlas crecen en número afectación del estado general, dolor
significativo asociado a sintomatología lumbar
compatible
____ 9. Cuando se recomienda realizar quimioprofilaxis en las infecciones del tracto urinario:
a. Menor de 2 años con ITU febril, hasta c. cuando hay cambios significativos en
completar estudio de imágenes y ITU los niveles de pH de los genitales, un
recurrente desequilibrio en el nivel de acidez o
una bajada de defensas
b. pacientes que presentan hemorragia d. Prurito vaginal o perianal
digestiva y casos seleccionados de especialmente en la noche
pacientes con bajo nivel de proteínas
en líquido ascítico y/o aumento de
bilirrubina en suero
____ 10.
Señale la respuesta correcta: en que consiste la prueba Trendelenburg

a. Se debe hacer con el niño en decúbito c. Colocando a los niños en decúbito


supino, relajado y flexionando las supino, la asimetría de los pliegues
caderas y rodillas 90 grados, fijado el glúteos y de los muslos o en las niñas,
muslo con el dedo pulgar, mientras el la desviación de la vulva aparece en un
segundo y tercer dedo fijan el relieve 12% de los casos y se mantiene hasta
del trocánter el tercer mes
b. El niño se coloca en decúbito supino d. Con el niño en bipedestación, al
con las caderas en abducción a 45 pedirle que se mantenga apoyado sobre
grados. Mientras una cadera fija la el lado afectado usando un apoyo extra
pelvis y la otra se moviliza, (ej. mano). Después debe elevar la
suavemente en aducción y abducción, pierna no afectada doblando la rodilla.
tratando de deslizarla sobre el borde Luego, provocando la inclinación de la
acetabular, pretendiendo luxarle pelvis, cuando normalmente se eleva la
nalga del lado que no se apoya
____ 11. Respecto al tratamiento de la displasia congénita de cadera. Todas son correctas excepto:
a. Tratamiento de 0-6 meses: En esta c. Tratamiento de 1-4 años: El
etapa las caderas que muestran sólo tratamiento a esta edad es la tracción-
inestabilidad tienen buen pronóstico, abducción por tres semanas o la
mantener la cadera en una posición tenotomía del músculo psoas y
estable colocándola en flexión de 90° y miotomía de los abductores, ya que la
en abducción. De esta forma se deformidad progresiva requiere de
consigue reducir el defecto de la tratamiento quirúrgico
cadera
b. Tratamiento de 6 meses a 1 año: Estos d. Tratamiento después de los 5 años: se
niños se deben someter a tracción utiliza el arnés de Paulik, y en caso de
durante tres semanas para disminuir la no conseguir la reducción a las 2-3
retracción de las partes blandas. En semanas, efectuaremos tracción
caso de presentar una tensión excesiva continúa con abducción progresiva,
pueden requerir de una tenotomía de reducción bajo anestesia general y
aductores yeso.
____ 12. Seleccione la respuesta correcta: El diagnostico Gold-Standart para la enfermedad por
reflujo gastroesofágico:
a. Radiología con Bario: La fluoroscopia c. PH-metria de 24 horas: Se han de
con bario, presenta mayor utilidad para utilizar microelectrodos de pH de
identificar cuadros causantes de cristal o de antimonio monocristalinos,
vómitos “estructurales”, congénitos o previamente calibrados con el de
adquiridos, como membranas, referencia externa que requiere pasta
estenosis, o mal rotaciones conductora para unirlo a piel, y que
gastrointestinales, hernia hiatal, etc irán conectados a una unidad de
almacenamiento de datos de memoria
b. Ecografia: Es un procedimiento inocuo d. Gamagrafia: Es una técnica rápida y
y fiable con una sensibilidad que no invasiva que ofrece algunas
supera el 65%. Una ventaja, es que no ventajas respecto al esofagograma
recibe radiación y nos puede descartar común con bario, en cuanto a que
obstrucciones distales (estenosis permite la vigilancia constante durante
hipertrófica de píloro, membranas todo el tiempo que dura su práctica con
antrales o duodenales, etc.) menor radiación
____ 13. Señale la respuesta correcta: En cuanto al tratamiento por enfermedad de reflujo
gastroesofágico cual es el tratamiento más utilizado y recomendado:
a. Tratamiento postural: A partir de la c. Bloqueantes de los receptores H2 de la
aparición de trabajos que demostraban histamina: Cimetidina, ranitidina,
correlación directa entre muerte súbita nizatidina y famotidina han
del lactante (MSL) y la posición prona demostrado su efecto beneficioso
con inclinación de 30 grados anti disminuyendo la sintomatología por
Trendelenburg RGE y la curación en esofagitis
b. Inhibidores de la bomba de protones d. Antiacidos: Hidróxido de aluminio-
(IBP): Disminuyen la secreción de hidroxido de magnecio-sucralfato-
ácido por medio de la inhibición de la alginato sódico. Actúan neutralizando
bomba Na+-K+ AT-Pasa, situada en la la acidez gástrica, mejorando la
membrana apical de la célula parietal presión del esfínter esofágico inferior
gástrica (EEI) y depurando la acidez esofágica
____ 14. Señale la respuesta correcta: Las bacterias patógenas más frecuentemente aisladas en casos
de vulvovaginitis bacteriana especifica:
a. El Streptococo pyogenes y H. c. Clostridium y peptoestreptococos
influenzae
b. Estafilococos Epidermidis y d. Bacteroides y Fusobacterium
Corynebacterium
____ 15. Señale la respuesta correcta: El tratamiento para la vulvovaginits por cándida:
a. Se recomienda Ceftriaxone, como c. Mebendazol 100MG una sola dosis
tratamiento inicial para todas las independiente de la edad. O
edades Albendazol: 400 mg y repetir en 2
semanas o pamoato de
pirantel:10mg/kg (dosis máxima 1gr)
dosis única y repetir en 2 semanas
b. El tratamiento consistirá en la d. Ninguna es correcta
aplicación tópica de pomadas u óvulos
de derivados imidazólicos
(clotrimazol, fluconazol) o nistatina
reactivos
pediatria

TRUE/FALSE

1. El diazepam intravenoso es el tratamiento de elección en convulsiones verdadero o falso

ANS: F PTS: 1 DIF: baja


REF: Martínez,(2017) El niño que convulsiona , pagina 363
OBJ: diferenciar el uso de benzodiacepinas en un caso de urgencia

2. Las convulsiones febriles se dan de los 3 meses a los 5 años, verdadero o falso

ANS: T PTS: 1 DIF: baja


REF: Martinez (2017), el niño que convusiona

MULTIPLE CHOICE

1. Ema de 1año 3 meses de edad, acude por presentar deposicones semiblandas por 6 ocasiones, sin
moco y no de mal olor, se acompaña de vomitos, que diagnóstico le daría a la madre y en que
tiempo va a ceder aproximadamente el cuadro?
a. Diarrea bacteriana, en 3 dias c. Diarrea parasitaria en 1 semana
b. Diarrea viral de 5 a 7 dias d. Diarrea por hongos 2 dias
ANS: B PTS: 1 DIF: media
REF: Schwartz (2015),Manual de Pediatria clinica, diarreas. pg 346.
OBJ: diferenciar los tipos de diarrea TOP: Enfermedad Diarreica

2. En la deshidratación por diarreas agudas en niños, en cual de los compartimentos, es donde


empieza la perdida de liquidos
a. intracelular c. extracelular
b. intersticial d. intracelular y extracelular
ANS: C PTS: 1 DIF: media
REF: Garcia Herrero,(2020), Deshidratación aguda, pag 215
OBJ: analizar la fisiopatogenia de la deshidratación, para poder tratarla
TOP: Deshidratacion

3. Paciente de 4 años de edad que acude por presenta diarreas por 10 ocasiones líquidas, madre le
administra aguas caseras, pero nota que su hijo se encuentra somnoliento y no ha realizado
realizado diures por mas de 12 horas que haría usted?
a. Enviar a casa con sales de rehidratacion c. Adminustrar por vía intravenosa cloruro
oral de sodio 0,9% 360 ml
b. Administrar cloruro de sodio al 0.9% 100 d. Adminustrar por vía oral 360 ml de sales
ml y dar suero oral de rehidratacion
ANS: C PTS: 1 DIF: media
REF: Garcia Herro (2020), Deshidratación aguda, Protocolos diagnosticos de pediatría 215
OBJ: diferenciar los casos que requiera tratamiento hospitalario

4. Paciente de 2 años de edad que desde hace 5 dias presenta alza termica, que se acompaña de tos
que despierta la noche desde hace 24 horas nota que respira mas rápido y la tos se escucha
productiva. Cuál es posible diagnóstico
a. Neumonia adquirida en la comunidad c. Faringo amigdalitis aguda
b. Crisis asmática d. sinusitis aguda
ANS: A PTS: 1 DIF: media
REF: Guia de práctica clínica, Neumonia adquirida en la comunidad 2015
OBJ: identificar signos clínicos de neumonía

5. Seleccione la respuesta correcta.


a. En la deshidratacion moderada la perdida c. la pérdida del 5 al 9% es en la
es del 10% deshidratación moderada
b. En la deshidratación leve hay sed intensa d. En la deshidratación grave no se altera la
diuresis
ANS: C PTS: 1 DIF: media
REF: Garcia ,(2020), Deshidratación aguda, Protocolos de deshidratación 219
OBJ: diferenciación del grado de deshidratación

6. En el servicio de emergencia es traido un niños de 6 años de edad con movimientos involuntarios


en brazos y piernas, con sialorrea, se evidencia acrocianosis distal, y desvió de la mirada, la madre
entre llantos refiere que desde hace 3 días presenta alza térmica, secundario a infección por mano
pie boca, cuál sería el posible diagnóstico

a. Convulsión febril c. Meningitis


b. Epilepsia d. Encefalitis
ANS: D PTS: 1 DIF: Media
REF: Martinez(2017), el niño que convulsiona

7.
Paciente de 7 años de edad que es traída hoy jueves, por su madre por presenta dolor
abdominal y que se acompaña de fiebre hasta 39 grados desde hace 3 dias, además presenta
vómitos, y dolor retro ocular, madre niega la ingesta de alguna sustancia, administra
ibuprofeno cada 8 horas con lo que mejora la fiebre pero se intensifica dolor abdominal, único
antecedente viaje por las vacaciones el día sábado previo. Cuál es el posible diagnóstico?

a. Gastroenteritis aguda c. Amigdalitis


b. abdomen agudo inflamatorio d. dengue
ANS: D PTS: 1 DIF: Media
REF: Guia de manejo del dengue 2020 OBJ: identificar los sintomas de dengue

8. Para diagnostica infección de vias urinarias en un elemental de orina que parámetros nos sirve
a. Leuco esterasas y nitritos c. Leucocitos y pH
b. Piocitos y bacterias d. densidad y bacterias
ANS: A PTS: 1 DIF: Media
REF: Gonzalez (2015), Infeccion de vias urinarias en la infancia, Protocolos diagnósticos en
pediatria OBJ: Interpretación de resultados de examenes de orina

MULTIPLE RESPONSE

1. Cálcule la hidratación de mantenimiento según Holliday Segar para un paciente de 5 años de edad
a. 60 ml/hora c. 52 ml/hora
b. 54 ml/hora d. 56 ml/hora
ANS: D PTS: 1 DIF: alta
REF: Carranza,(2016),Rehidratación en el paciente pediatrico,pg 40
OBJ: Cálculo en paciente pediatrico

2. Paciente de 2 años que presenta diarreas 4 diarias con moco y las 2 últimas con sangre
macroscópica, alza térmica no cuantificada, madre refiere que hace 24 horas paciente con sed
intensa y llanto sin lágrimas. Posible diagnóstico y grado de diferenciación

a. diarrea bacteriana c. deshidratación leve


b. diarrea viral d. deshidratación moderada
ANS: A, D PTS: 1 DIF: Media
REF: Garcia,(2020) Deshidratación aguda, pag 217 OBJ: Análisis de casos
clinicos
Reactivos Ginecología y Obstetricia

Multiple Choice
Identify the choice that best completes the statement or answers the question.

____ 1. GESTANTE DE 24 SEMANAS ACUDE A LA CONSULTA PORQUE A SU HIJO DE 5 AÑOS HACE 5 DÍAS
LE DIAGNOSTICARON VARICELA. LA PACIENTE NO RECUERDA SI PADECIÓ LA ENFERMEDAD EN
LA INFANCIA, PERO SI SABE QUE NO FUE VACUNADA Y ESTÁ MUY PREOCUPADA POR LA
POSIBLE AFECCIÓN FETAL. QUE ACTUACIÓN SERIA LA CORRECTA?.

a. ADMINISTRAR VACUNA ESPECIFICA d. ADMINISTRAR ACICLOVIR A DOSIS DE


800MG VO 5 VECES AL DÍA POR 5- - 7
DÍAS.

b. SOLICITAR CUANTIFICACIÓN DE IGG, Y e. ADMINISTRAR GAMAIGLOBULINA


SI FUERA ESPECIFICO ADMINISTRAR ESPECIFICA Y TRANQUILIZAR A LA
GAMAGLOBULINA ESPECIFICA. PACIENTE INFORMÁNDOLE DE LA
AUSENCIA DE RIESGOS.

c. TRANQUILIZAR A LA PACIENTE
INFORMÁNDOLE DE LA AUSENCIA DE
RIESGOS FETALES

____ 2. PACIENTE DE 28 AÑOS EN LAS 42.3 SG Y ANTECEDENTES DE OTRO PARTO A TÉRMINO. EL


EMBARAZO HA TRANSCURRIDO NORMALMENTE. AL EXAMEN FISICO VARIEDAD DE
PRESENTACION: FETO EN OIIA, NST REACTIVO Y CERVIX FAVORABLE BISHOP 9. CUÁL DE LAS
SIGUIENTES CONDUCTAS ES LA MAS FAVORABLE?

a. ESPERAR HASTA QUE DESENCADENE d. CONDUCIR EL TRABAJO DE PARTO


TRABAJO DE PARTO..
C.- ADMINISTRAR CORTICOIDES A LA
MADRE E INDUCIR EL PARTO PASADAS
LAS 48 HORAS

b. CONTROLES CADA 48 HORAS. e. HACER CESÁREA ELECTIVA.

c. ADMINISTRAR CORTICOIDES A LA
MADRE E INDUCIR EL PARTO PASADAS
LAS 48 HORAS

____ 3. PACIENTE EN PERIODO EXPULSIVO DE 38 SEMANAS DE GESTACIÓN, PRESENTACIÓN EN TERCER


PLANO DE HODGE, POSICIÓN OIIT Y APARICIÓN DE UNA BRADICARDIA FETAL DE 100 - 110 LAT X
MIN CONSTATÁNDOSE LA EXISTENCIA DE UNA ACIDOSIS RESPIRATORIA FETAL: PH DE CUERO
CABELLUDO 7.18. CUÁL SERÍA LA ACTITUD CORRECTA?

a. REALIZACIÓN DE UNA CESÁREA d. COLOCAR EL PULSOXIMETRO PARA


PROGRAMADA CONSTATAR LA SATURACIÓN DE
OXIGENO MATERNO.

b. UTILIZACIÓN DE UNA VENTOSA - e. APLICACIÓN DE OXIGENO A LA MADRE


VACUUM. PARA REALIZACIÓN DE PH EN 10
MINUTOS.
c. UTILIZACIÓN DE UN FÓRCEPS DE
SALIDA

____ 4. GESTANTE DE 8 SEMANAS QUE ACUDE A CONSULTA PARA REALIZAR ECOGRAFÍA EN LA QUE UD.
OBSERVA UNA GESTACIÓN INTRAUTERINA CON EMBRIÓN ÚNICO CON LCC DE 16MM (ACORDE
CON 8 SEMANAS) SIN LATIDO CARDIACO SEÑALE LO CORRECTO:

a. GESTACIÓN MAL DATADA. LA LCC d. DIAGNOSTICO DE FETO ACARDIO


NOS INDICA EL TIEMPO DE GESTACIÓN
EN EL PRIMER TRIMESTRE.
ÚNICAMENTE CORREGIMOS LA FECHA
PROBABLE DE PARTO.
b. AMENAZA DE ABORTO, RECOMIENDA e. DIAGNOSTICO DE ABORTO DIFERIDO =
REPOSO Y GESTÁGENOS. LEGRADO

c. DETERMINACIÓN DE BHCG
PLASMÁTICA CADA 48 H.

____ 5.
GESTANTE DE 38 SEMANAS QUE COMIENZA BRUSCAMENTE CON METRORRAGIA OBSCURA Y EN
POCA CANTIDAD, DOLOR ABDOMINAL INTENSO, DIFUSO, MAL ESTADO GENERAL Y SUFRIMIENTO
FETAL. CUÁL ES LA COMPLICACIÓN MAS PROBABLE EN ELLA?.

a. DESPRENDIMIENTO PREMATURO DE d. NECROSIS DE UN MIOMA.


PLACENTA
b. PLACENTA PREVIA e. TORSIÓN DE UN TUMOR OVÁRICO.

c. INSERCIÓN VELAMENTOSA DE
CORDÓN UMBILICAL.

____ 6. GESTANTE DE 26 SEMANAS QUE PRESENTA DESDE HACE 5 DÍAS: MALESTAR GENERAL, NAUSEA,
CEFALEA, EDEMA Y LIGERO DOLOR EN HIPOCONDRIO DERECHO. TA 140/90 PROTEINURIA 3g/24H
EN EXÁMENES HB 12G/DL, BILIRRUBINA TOTAL 1.4 MG/DL, LDH 670 UI/I, TGO 182UI/I, PLAQUETAS
DE 80000MM3. LO MAS PROBABLE ES QUE SE TRATE DE UN EMBARAZO COMPLICADO POR?.

a. EMBOLISMO DE LÍQUIDO AMNIÓTICO d. ISOINMUNIZACIÓN ERITROCITARIA


CON TRANSFUSIÓN FETO – MATERNA
b. MUERTE FETAL CON PASO DE e. E.- PREECLAMPSIA SEVERA + SD HELLP
SUSTANCIAS TRONBOPLÁSTICAS A LA
CIRCULACIÓN MATERNA.
c. DESPRENDIMIENTO PREMATURO DE
PLACENTA SUPERIOR A 50%

____ 7. CUAL ES EL MÉTODO DE DETECCIÓN (SCREENING) DE DIABETES GESTACIONAL QUE SE


RECOMIENDA EN TODAS LAS EMBARAZADAS?

a. DETERMINACIÓN DE GLUCOSA EN d. TEST DE TOLERANCIA ORAL A LA


ORINA GLUCOSA CON 100G DE SOBRECARGA
b. DETERMINACIÓN DE GLICEMIA EN e. TEST DE TOLERANCIA ORAL A LA
AYUNAS GLUCOSA CON 75G DE SOBRECARGA
c. DETERMINACIÓN DE GLICEMIA EN
AYUNASEN DÍAS DIFERENTES

____ 8. CUAL DE LAS SITUACIONES CLINICAS ES UNA CONTRAINDICACION ABSOLUTA PARA LA


ANTICONCEPCION HORMONAL?

a. HIPERTENSION ARTERIAL BIEN d. MUJER MENOR DE 35 AÑOS Y


CONTROLADA. FUMADORA
b. HISTORIA PERSONAL DE e. INFECCION URINARIA.
TROMBOEMBOLISMO PREVIO.
c. DIABETES
____ 9.
CUAL DE LOS SIGUIENTES CARACTERISTICAS NO ES FRECUENTE EN LA VAGINOSIS BACTERIANA

a. PH VAGINAL MENOR A 4.5. d. AUSENCIA DE ERITEMA VULVAR


b. LEUCORREA FLUIDA Y HOMOGENEA e. AUSENCIA DE PRURITO VULVAR
c. PRESENCIA DE CELULAS CLAVE.

____ 10. CUAL DE LAS SIGUIENTES AFIRMACIONES ES FALSA EN RELACIÓN A LA TRANSMISIÓN VERTICAL
AL FETO/RN POR PARTE DE UNA GESTANTE CON INFECCIÓN DE VIH?

a. AUMENTA LA POSIBILIDAD DE d. CON EL TRATAMIENTO


MALFORMACIÓN FETAL. ANTIRRETROVIRAL DURANTE EL
EMBARAZO Y SOBRE TODO DURANTE
EL PARTO
b. TIENE LUGAR SOBRE TODO EN EL e. EL TRATAMIENTO CON ZIDOVUDINA
PARTO. NO HA DEMOSTRADO EFECTOS
ADVERSOS EN EL RECIÉN NACIDO
HASTA LOS 5 AÑOS
c. LA LACTANCIA AUMENTA EL RIESGO
DE TRANSMISIÓN ENTRE UN 10% A
20%.

____ 11. PRIMIGESTA DE 37 SEMANAS INGRESA POR RPM DE 24H DE EVOLUCIÓN NO HAY TRABAJO DE
PARTO NI SIGNOS DE INFECCIÓN AMNIÓTICA. EL TEST DE BISHOP ES DE 4 PUNTOS, EL FETO
ESTA EN CEFÁLICA Y NO HAY SIGNOS DE COMPROMISO FETAL. CUÁL ES LA CONDUCTA
OBSTÉTRICA INDICADA?

a. CESÁREA d. ADMINISTRAR ANTIBIÓTICOS Y


ESPERAR EL COMIENZO ESPONTANEO
DEL PARTO.
b. ADMINISTRAR ANTIBIÓTICOS E e. ESPERAR EL COMIENZO ESPONTANEO
INDUCIR EL PARTO CON SIN TRATAMIENTO O HACER CESÁREA
PROSTAGLANDINAS. CON SIGNOS DE INFECCIÓN
AMNIÓTICA.
c. ADMINISTRAR ANTIBIÓTICOS,
CORTICOIDES Y CONDUCIR EL PARTO
CON OXITOCINA48H DESPUÉS

____ 12. MUJER DE 25 AÑOS CONSULTA POR RETRASO MENSTRUAL DE 3 SEMANAS Y


SANGRADO TRANSVAGINAL DE 24 HORAS, ESTADO GENERAL BUENO. BHCG: 1000
mUI/ml, ECOGRAFIA TRANSVAGINAL ENDOMETRIO DE 12 mm, EN OVARIO DERECHO
IMAGEN QUE APARENTRA CUERPO LUTEO , NO HAY LIQUIDO LIBRE EN FONDO DE
SACO DE DOUGLAS. ¿CUAL ES LA INDICACIÓN CORRECTA?
a. repertir seriadamente cada 48 horas la d. diagnostico de embarazo ectópico
BHCG y la ecografia
b. reposo absoluto y repetir la ecografia en 2 e. laparotomia o laparoscopia
- 3 semenas
c. legrado uterino
____ 13. INFECCION POR ESTREPTOCOCO DEL GRUPO B EN RN PUEDE PREVENIRSE
ADMINISTRANDO ANTIBIOTICOTERAPIA PROFILACTICA A LAS EMBARAZADAS
PORTADORAS O CON FACTORES DE RIESGO ¿CUAL SERIA LA ACTITUD INCORRECTA
a. la profilaxis se realiza durante el parto d. la prueba se solicitara entre la 35 y 37
semanas o ante la eminecia del parto
b. en las pacientes alergicas a la ampicilina e. la incidencia de infeccion neonatal es
se puede utilizar la eritromicina mayor entre RN prematuros que a termino
c. la via mas frecuente de infección es
transplacentarea
____ 14. COMO CLASIFICA USTED A UNA GESTACION QUE ANTES DEL EMBARAZO REFIERE
TENER TENSIONES ARTERIALES NORMALES, QUE EN LA SEMANA 8 SE LE DETECTA
TENSION ARTERIAL 140/90 Y QUE EN LA SEMANA 18 TIENE UNA ENSION ARTERIAL DE
170/110 SIN EDEMA Y INDICE QUE PROTEINURIA/CREATINURIA DE 100 mg/mmol
a. preeclampsia d. hipertensión crónica más preeclampsia
sobreañadida
b. preeclampsia grave e. hipertensión gestacional
c. hipertensión crónica
____ 15. PUERPERA DE 42 AÑOS CON PARTO VAGINAL HACE 12 HORAS EN LA QUE SE
SOSPECHA DE DIAGNOSTICO DE SD HELLP. ¿INDIQUE QUE LITERAL NO
CORRESPONDE?
a. plaquetas de 75000 d. hemoglobina: 7.2 g/dl
b. bilirrubina indirecta de 3.7 mg/dl e. TGO de 520 UI/L
c. TA: 170/110 mmHg
____ 16. SEGUNDIGESTA CON ANTECEDENTE DE PARTO ANTERIOR A LAS 35 SEMANAS, QUE
CONSULTA EN LA SEMANA 32 POR PERCIBIR CONTRACCIONES UTERINAS, EN LA
EXPLORACION SE COMPRUEBA DILATACION CERVICAL DE 1 CM BORRADO 30% CON
UNA LONGITUD CERVICAL DE 1.5 CM POR ECOGRAFIA, EL MONITOREO FETAL
REVELA UNA FCF: 140 LPM, Y UNA CONTRACCIÓN UTERINA DE CADA 5 MINUTOS.
¿CUAL DE LAS SIGUIENTES AFIRMACIONES ES CORRECTA?
a. se trata de un APP establecida que obliga c. se aconseja la administración de
a realizar tocolisis y maduración cervical corticoides para maduración pulmonar
fetal unicamente por el antecedente del
parto pretermino
b. la longitud cervical, inferior a 2.5 cm d. se trata de una APP incierta requiere una
supone un elevado valor predictivo nueva valoracion clinica (AU, FCF Y TV)
negatico para parto pretermino en 1 - 2 horas

____ 17. GESTANTE A TERMINO QUE INGRESA EN TRABAJO DE PARTO EN FASE LATENTE, A
LA VALORACION SE DETECTA PEQUEÑAS VESICULAS DE HERPES SIMPLE
RESIDIVANTE. AÑOS ANTES TUVO UN PRIMOINFECCIÓN DE HERPES GENITAL Y
VARIOS BROTES DE HERPES RESIDIVANTE. ¿CUAL ES LA CONDUCTA A SEGUIR?
a. cesarea d. tratar inmediatamente las lesiones con Ac.
tricloro-acetico, para inactivar el virus y
permitir el parto vaginal
b. parto normal y tratar al RN con aciclovir e. parto normal y aislar al RN de su madre
c. no es necesaria una conducta especial ya
que el herpes residivante no tiene riesgo
para el RN
____ 18. LAS HEMORRAGIAS ANORMALES VAGINALES, FRECUENTES EN EL PRIMER AÑO
TRAS LA MENARQUIA, QUE SE CONOCEN COMO HEMORRAGIAS UTERINAS
DISFUNCIONALES SE DEBEN A:
a. hemorragias de la ovulacion y no ameritan d. trastornos de la coagulacion y ameritan
tratamiento tratamiento
b. ovario poliquistico y amerita tratamiento e. enfermedad inflamatoria pelvica que
amerita tratamiento
c. ciclos anovulatorios y no ameritan
tratamiento
____ 19. MUJER DE 35 AÑOS CON 31 SEMANAS DE EMBARAZO CONSULTA POR UNA
SECRECION VAGINAL DE MAL OLOR, “PESCADO PODRIDO”, NO TIENE PRURITO
GENITAL, A LA VALORACIÓN SE VIDENCIA ABUNDANTE SECRECION BLANCO
GRISACEA QUE NO SE ADHIERE A LAS PAREDES VAGINALES. AL MEZCLAR ESTA
MUESTRA CON HIROXIDO DE POTASIO AL 10% DESPRENDE UN OLOR INTENSO
SUIGENERIS. ¿CUAL DE LAS SIGUIENTES ES EL TRATEMIENTO DE ELECCIÓN?
a. amoxicilina + ac. clavulánico VO o IV c. fluconazol por VO
b. clotrimazol por via intravaginal o VO d. metronizadol por via intravaginal o VO
____ 20. PACIENTE EN SU PRIMER TRIMIESTE DE GESTACION CON HEMOGLOBINA DE 11
CATALOGADA CON ANEMIA LEVE USTED DECIDE INICIAR EL APORTE DE HIERRO.
a. Se trata una anemia cronica y no necesita d. Inicia 200 mg de hierro elemental diarios
de aporte de hierro
b. Considera utilizar hierro sacarosa IV e. Sulfato ferroso 500 mg VO diarios
c. A pesar de que la paciente esta
asintomatica por su etapa de embarazo
considera transfusión
Name: ________________________ Class: ___________________ Date: __________ ID: A

5 componente salud mental

Multiple Choice
Identify the choice that best completes the statement or answers the question.

____ 1. Identifique dentro de las alteraciones del contenido del pensamiento el trastorno
psicopatológico que presenta este paciente.
-Paciente AMP , masculino de 38 años , de ocupación ingeniero que acude a consulta
consciente, orientado globalmente que viene remitido del subcentro de salud y
acompañado por la esposa ya que desde hace una semana se encuentra con gran
preocupación después de haber tenido una insignificante discusión de trabajo con uno
de sus compañeros. Su esposa en la entrevista refiere ¨Aunque todos le hemos
señalado que debe olvidar el incidente, desde que discutió con su compañero de
oficina se recrimina constantemente por su comportamiento, ha llegado a decir que
jamás lo perdonaremos y que su solución es trasladarse de trabajo. Nadie comprende
su reacción pues el incidente fue algo muy banal.´

a. Idea Fija c. Idea Fobica


b. Idea Obsesiva d. Idea Sobrevalorada

____ 2. - Al realizar la entrevista al paciente OHF masculino de 33 años de edad con escolaridad
superior de profesión profesor este refiere ¨Oigo claramente dentro de mi cabeza una
voz femenina que me ordena ofender a mis vecinos¨.
El objetivo es que usted identifique el trastorno psicopatológico presenta este paciente.

a. Alucinaciones c. Ilusiones
b. Pseudoalucinaciones d. Alucinosis

____ 3. En la entrevista psicológica a un paciente en consulta este recuerda con nitidez el lugar
donde ocurrió el accidente automovilístico e incluso, que participo en la conducción de
un herido grave, pero no recuerda que dicho herido fue su hijo, que falleció
posteriormente.

a. Amnesia lacunar c. Hipomnesia


b. Hipermnesia d. Paramnesia

____ 4. Paciente MDP de 25 años de edad, femenina, soltera, desocupada que viene referida del
subcentro de salud y que acude acompañada de su madre. En la entrevista ¨Decía oír
voces que la llamaban desde la ventana y que le indicaban lo que debía hacer¨.
Identifique que trastorno psicopatológico presenta esta paciente.

a. Ilusiones c. Alucinaciones
b. Pseudoalucinaciones d. Despersonalizacion

1
Name: ________________________ ID: A

____ 5. Paciente OGH de 36 años que viene referido del subcentro de salud. De profesión
albañil, casado con 2 hijos que desde hace algún tiempo su esposa nota que dice
cosas sin sentido . En la entrevista el paciente refiere ¨Yo sé que cuando venga la
mañana del rio, tendrá más frio¨

a. Prolijidad c. Retardo del pensamiento


b. Aceleracion del pensamiento d. Disgregacion

____ 6. A continuación, exponemos una serie de referencias sintomáticas que pueden obtenerse
tanto del propio paciente como de otras fuentes de información. El objetivo es que
usted valore para identificar el trastorno psicopatológico
El paciente refiere ¨Estaba de visita en la casa de Rosa, la esposa de mi nieto que
estudio en Estados Unidos, cuando llego mi vecina con la noticia del accidente y
después de esa impresión empezaron los temblores. ¨

a. Retardo del pensamiento c. Incoherencia


b. Prolijidad d. Disgregacion

____ 7. El paciente refiere ¨Doctor no puedo quitarme de la mente sus palabras cuando ella me
dijo que ya no me quería. Esto se ha convertido en una tortura y no puedo evitar ese
recuerdo¨.

a. Idea sobrevalorada c. Idea Fija


b. Idea delirante d. Idea Hipocondriaca

2
Name: ________________________ ID: A

____ 8. En el caso clínico que exponemos a continuación diga qué tipo de esquizofrenia
presenta este paciente.
Paciente RMG, 30 años, masculino, leptosómico, casado, arquitecto con antecedentes
familiares psiquiátricos dados por un hermano y la madre con ingresos hospitalarios y
tratamiento de terapia electroconvulsiva. Desde hace unos dos meses se ha visto
distraído en su trabajo, prácticamente no habla con nadie y ha descuidado su aseo y
vestuario; parece como si nada le interesara. En días pasados le comunico a su jefe en
forma descompuesta que él tenía derecho a vivir su vida en privado y que no permitía
que hicieran públicos sus pensamientos a través de la televisión. La víspera del ingreso
todos se alarmaron cuando fue al trabajo con un casco protector de motociclista
refiriendo que lo tenía forrado de plomo por dentro. En la consulta vemos un paciente
con barba de varios días, que rechaza la entrevista con el argumento que no está
enfermo. Dice que todo cuanto le pasa es fruto de los aparatos que controlan sus ideas
y le imponen pensamientos extraños ¨que escucha dentro de su cabeza en forma de
ecos¨. Sabe que todo cuanto hace es ajeno a su voluntad y no puede pensar cosas
privadas porque después las difunden por la televisión. Esto lo expresa sonriendo
inadecuadamente. Los familiares refieren que antes de enfermarse era un joven
sociable, amistoso y de magnifico carácter. No se evidencian factores ambientales
significativos.

a. Esquizofrenia Paranoide c. Esquizofrenia Residual


b. Esquizofrenia Hebefrenica d. Esquizofrenia Indiferenciada

____ 9. - Paciente AMJ femenina, 17 años, leptosómica soltera, secretaria, segundo curso de
bachillerato, su madre falleció en un hospital psiquiátrico y tiene dos ríos con historia de
ingresos por trastornos mentales. Ella siempre fue una joven seria, estudiosa y
apegada a la familia. Sin embargo, desde los 17 años se presentó un cambio
importante en su comportamiento, comenzó a no interesarse por la escuela y, por
último, dejo de asistir, refiriendo que iba a trabajar. Por esos días comenzó a llegar
tarde al hogar contrariamente a su costumbre, en varias ocasiones durmió fuera y
expresaba que estaba con unas amigas. La familia la notaba descuidada en su aseo,
aunque muy interesada por maquillarse y vestirse en forma algo provocativa. Comenzó
a tener muchos enamorados y hablaba de ellos en forma despectiva, a veces con
palabras obscenas.
Hace unos meses ha estado muy rara, se ríe por gusto, a veces habla cosas sin
sentido, abandono el trabajo porque sus compañeros eran¨ muy pesados¨.
Últimamente hace bromas de mal gusto, y la manifestación que decidió a su familia a
traerla al psiquiatra es que insinuó sexualmente a su hermano. En la entrevista vemos
a una paciente exageradamente maquillada, en actitud seductora y mantiene una risa
inadecuada y pueril que intensifica después de bromas de mala gusto. Doctor, su nariz
es más grande que la de un elefante¨Su comunicación es muy desorganizada, a veces
difícilmente comprensible.

a. Esquizofrenia Paranoide c. Esquizofrenia residual


b. esquizofrenia Hebefrenica d. Esquizofrenia Indiferenciada

3
Name: ________________________ ID: A

____ 10. Paciente ABD, de 40 años, leptosómico, 17 años, soltero, estudiante de bachillerato,
con referencia de varios familiares con ingresos psiquiátricos. Desde hace unos 3
meses se le comenzó a notar muy pensativo y alejado de sus amistades y familiares
por momentos parecía que estaba en otro mundo ¨dejó de preocuparse por su aseo y
vestuario y frecuentemente se le observaba mirando por las persianas de la casa. Se
armó de un machete que guardaba debajo de la almohada y decía que había que
dormir con un ojo abierto. Los familiares lo han visto hablando solo, como si contestara
a personas imaginarias y ha dicho a su familia que por momentos se nota los brazos
más gruesos. Se ha negado a comer en la casa porque piensa que su madre lo quiere
envenenar, la familia aprecia que nada le interesa ni lo hace reaccionar.
En días pasados falleció su abuela y durante el sepelio se le veía sonriendo y
conversando solo. Siempre fue un joven solitario y reservado, pero ahora estas
características se han exagerado. No se evidencias contingencias ambientales
actuales.

a. Esquizofernia Residual c. Esquizofrenia Paranoide


b. Esquizofernia Hebefrenica d. Esquizofrenia Indiferenciada

____ 11. Paciente RGB de 46 años, desocupada, con historia familiar de cuadros de excitación
repetidos en el padre, por lo que ha estado ingresado, 2 tíos maternos con intentos
suicidas y 1 abuelo con cuadros depresivos a repetición. Acude a consulta
acompañada de su hija, quien nos refiere que en el último mes ha notado a su madre
muy triste, alejada de todas sus amistades, desinteresada en las tareas del hogar, y
descuidada en su aseo y arreglo personal. Se le ha visto frecuentemente llorando y
refiere que tiene una mala enfermedad, ha dicho que preferiría estar muerta; en la
noche se duerme muy temprano, pero a las 3:00 am. Se despierta y ya no puede
conciliar el sueño.
Dice que ha sido una mujer mala y que merece lo que está pasando. Señala que es un
estorbo para su familia y reporta que escucha a sus vecinas en conversaciones en las
que se alegran de su padecimiento. Refiere escuchar voces desconocidas que le dicen
¨matate¨, prácticamente no come y ha pedido 20 libras de peso. En la entrevista vemos
a una paciente con notable tristeza que habla y se mueve muy pausadamente,
comunica que ha perdido la capacidad para disfrutar y ve su futuro con pesimismo.

a. Trastorno bipolar episodio actual c. Trastorno bipolar episodio actual


hipomaniaco maniaco sin sintomas psicoticos
b. Trastorno bipolar episodio depresivo d. Trastorno Bipolar episodio actual
grave con sintomas psicoticos depresivo leve o moderado

4
Name: ________________________ ID: A

____ 12. Paciente ADU de 32 años, soltero, trabaja con su padre en la agricultura, segundo
grado de escolaridad, repitió primero y segundo grado de escolaridad. Nació de un
parto institucional eutócico, a los 6 meses presento un cuadro de hipertermia severa y
convulsiones que requirió tratamiento institucional por 2 meses y fue diagnosticada una
meningitis bacteriana. Luego de su alta se apreció importante retraso para caminar y
hablar, presentó incontinencia vesical hasta los 20 años. Su vocabulario es de
aproximadamente un niño de 6 años. Desempeña su trabajo adecuadamente, cuando
se le orienta que hacer.

a. Retraso Mental Severo c. Retraso Mental Moderado


b. Retraso Mental Leve d. Retraso Mental Profundo

5
ID: A

5 componente salud mental


Answer Section

MULTIPLE CHOICE

1. ANS: D PTS: 1 DIF: Alta


REF: Psicotalogia general.Deshaies Gabriel Editorial. Kapelusz
OBJ: Explica la normalidad y patologia de los trastornos psicopatologicos de los procesos mentales
superiores. TOP: Psicologia- Procesos Mentales Superiores. Normalidad y Patologia
2. ANS: B PTS: 1 DIF: Alta
REF: Psicopatologia General.Deshaies Gabriel. Editorial Kapelusz
OBJ: Explica la normalidad y la patologia de los trastornos psicopatologicos de los procesos mentales
superiores TOP: Psicologia- Procesos Mentales Superiores
3. ANS: A PTS: 1 DIF: Alta
REF: Psicopatologia General. Deshaies Gabriel. Editorial Kapelusz
OBJ: Explica la normalidad y patologia de los trastornos psicopatologicos de los procesos mentales
superiores TOP: Psicologia. Procesos mentales superiores. Normalidad y patologia
4. ANS: C PTS: 1 DIF: Alta
REF: Psicopatologia General.Deshaies Gabriel. Editorial Kapelusz
OBJ: Explica la normalidad y patologia de los trastornos psicopatologicos de los procesos mentales
superiores TOP: Psicologia- Procesos mentales superiores. Normalidad y patologia
5. ANS: D PTS: 1 DIF: Alta
REF: Psicopatologia General.Deshaies Gabriel, editorial Kapelusz
OBJ: Explica la normalidad y patologia de los trastornos psicopatologicos de los procesos mentales
superiores TOP: Psicologia-Procesos mentales superiores. Normalidad y Patologia
6. ANS: B PTS: 1 DIF: Alta
REF: Psicopatologia General. Deshaiez Gabriel. Editorial Kapelusz
OBJ: Explica la normalidad y patologia de lostrastornos psicopatologiavcos de los procesos mentales
superiores TOP: Psicologia- Procesos Mentales superiores. Normalida y patologia
7. ANS: C PTS: 1 DIF: Alta
REF: Psicopatologia General. Deshaies Gabriel, Editorial Kapelusz
OBJ: Explica la normalida y patologia de los trastornos psicopatologicos de los procesos mentales
superiores
TOP: Psicologia. Procesos mentales superipores , Normalidad y patologia
8. ANS: A PTS: 1 DIF: Alta
REF: Kaplan y Sadock. Manual de bolsillo de Psiquiatria Clinica. 5ta Edicion España.,Lippincott
Willians and Wilkins, Wolters Kluwer Health 2015
OBJ: Diagnostica temapranamente da tratamiento oportuno y adecuado para su nivel de complejidad,
tanto farmacologico y psicologico basico
TOP: Psiaquiatria II- Psicosis organicas o somaticas y funcionales o psicogenas
9. ANS: B PTS: 1 DIF: Alta
REF: Kaplan y sadock. Manual de bolsillo de Psiquiatria . 5ta edicion España, Lippincott Willians and
Wilkinins, woltwers Kluwer Health 2015
OBJ: Diagnostica tempranamente, da tratamiento oportuno y adecuado para su nivel de complejidad,
tanto farmacologico psicologico basico
TOP: PsiquiatriaII- Psicosis organicas o somaticas funcionales o psicogenas

1
ID: A

10. ANS: C PTS: 1 DIF: Alta


REF: Kaplan y Sadock. Masnual de bolsillo de Psiquiatria Clinica. 5ta edicion España Lippincott
Willians and Wilkins, Wolters Kluwer Health 2015
OBJ: Diagnosticar tempranamente, da tratamiento oportuno y adecuado para su nivel de complejidad,
tanto faramacologico y psicologico basico
TOP: Psiquiatria II - Psicosis organicas o somaticas y funcionales o psicogenas
11. ANS: B PTS: 1 DIF: Alta
REF: Clasificacion Internacional de Enfermedades 10 version CIE-10
OBJ: Diagnostica tempranamente da tratamiento oportuno y adecuado para su nivel de complejidad,
tanto farmacologico y psicologico basico
TOP: Psiquiatria II Psicosis organicas o somaticas y funcionales o psicogenas
12. ANS: C PTS: 1 DIF: Alta
REF: Clasificacion Internacional de Enfermedades
OBJ: Diagnostica tempranamente , da tratamiento oportuno y adecuado para su nivel de complejiodad,
tanto farmacologico y psicologico basico TOP: Psiquiatria II - Trastoornos del desarrollo

2
5 componente salud mental [Answer Strip] ID: A

D
_____ 5. A
_____ 8. C 10.
_____ C 12.
_____

D
_____ 1.

B
_____ 6.

C
_____ 7.
B 11.
_____
B
_____ 2.

B
_____ 9.

A
_____ 3.

C
_____ 4.
GINECOLOGÍA Y OBSTETRICIA
ABORTO

MULTIPLE CHOICE

1.
Paciente de 25 años que acude a urgencias por presentar amenorrea, sangrado uterino y dolor
tipo cólico en hipogastrio de magnitud variable, es realizada eco transvaginal cuyo resultado
es gestación de 12 semanas con vitalidad fetal. Al examen físico volumen uterino acorde con
amenorrea y sin dilatación cervical evidente al TV. De acuerdo a los datos clínicos usted
diagnostica:

a. Amenaza de aborto c. Aborto incompleto

b. Aborto inevitable d. Aborto completo

ANS: A
Los datos clínicos: amenorrea secundaria, prueba de embarazo positiva (B-HCG cuantitativa),
presencia de vitalidad fetal, sangrado uterino de magnitud variable, dolor tipo cólico en
hipogastrio de magnitud variable y volumen uterino acorde con amenorrea sin dilatación cervical
evidente, nos lleva al diagnóstico de amenaza de aborto.

PTS: 1 DIF: MEDIA


REF: GPC / MSP Diagnóstico y tratamiento del aborto espontáneo, incompleto, diferido y
recurrente. PAG: 16
OBJ: DIAGNOSTICAR HEMORRAGIAS DEL PRIMER TRIMESTRE
TOP: HEMORRAGIAS DEL RPIMER TRIMESTRE

2.
Paciente de 25 años que acude a urgencias por presentar amenorrea, sangrado uterino
abundante y dolor tipo cólico en hipogastrio que ha ido incrementándose desde hace 4 horas.
Al examen físico volumen uterino menor que amenorrea y al TV se evidencia dilatación
cervical. De acuerdo a los datos clínicos usted diagnostica:

a. Amenaza de aborto c. Aborto incompleto

b. Aborto inevitable d. Aborto completo

ANS: B
Los datos clínicos de aumento progresivo del dolor, de las contracciones o de la hemorragia con
modificaciones cervicales o ruptura de membranas, nos lleva al diagnóstico de aborto inevitable

PTS: 1 DIF: MEDIA


REF: GPC / MSP Diagnóstico y tratamiento del aborto espontáneo, incompleto, diferido y
recurrente. PAG: 15 - 16
OBJ: DIAGNOSTICAR Y TRATAR HEMORRAGIAS DEL PRIMER TRIMESTRE
TOP: HEMORRAGIAS DEL PRIMER TRMESTRE

1
3.
Paciente de 25 años que acude a urgencias por presentar amenorrea de 12 semanas. Al examen
físico volumen uterino menor que amenorrea, es realizada eco transvaginal se evidencia
ausencia de vitalidad fetal, al TV no hay modificaciones cervicales. De acuerdo a los datos
clínicos usted diagnostica:

a. Amenaza de aborto c. Aborto incompleto

b. Aborto inevitable d. Aborto diferido

ANS: D
Los datos clínicos de retención en la cavidad uterina de un embrión o feto muerto (incluyendo el
embarazo anembrionado), volumen uterino menor que por amenorrea, ausencia de vitalidad fetal
en ecosonografia y sin modificaciones cervicales al TV, diagnosticamos de aborto diferido

PTS: 1 DIF: ALTA


REF: GPC / MSP Diagnóstico y tratamiento del aborto espontáneo, incompleto, diferido y
recurrente. PAG: 15 - 16
OBJ: DIAGNOSTICAR Y TRATA HEMORRAGIAS DEL PRIMER TRIMESTRE
TOP: HEMORRAGIAS DEL PRIMER TRIMESTRE

4.
Entre los factores de riesgo para que una paciente presente aborto recurrente, señale el literal
correcto:

a. Si existe un aborto previo se incrementa c. Pacientes diagnosticadas de lupus


la probabilidad de recurrencia en un 5 % eritematoso sistémico

b. Los anticuerpos antifosfolipidos y los d. Todas son verdaderas


anticuerpos anticardiolipinas están
asociados a aborto recurrente

ANS: D
La evidencia ha demostrado que cuando existe antecedentes de aborto previo se incrementa la
probabilidad de un 5 %, al igual que en aquellas pacientes con anticuerpos antifosfolipidos,
anticuerpos anticardiolipinas y lupus eritematoso sistémico están asociados a aborto recurrente

PTS: 1 DIF: ALTA


REF: GPC / MSP Diagnóstico y tratamiento del aborto espontáneo, incompleto, diferido y
recurrente. PAG: 19
OBJ: DIAGNOSTICAR Y TRATAR HEMORRAGIAS DEL PRIMER TRIMESTRE
TOP: HEMORRAGIAS DEL PRIMER TRIMESTRE

2
GINECOLOGÍA Y OBSTETRICIA
ALIMENTACIÓN Y NUTRICIÓN DURANTE EL EMBARAZO Y LA
LACTANCIA

MULTIPLE CHOICE

1.
Paciente de 30 semanas gestación acude a su cuarto control prenatal, usted se percata que no ha
existido una ganancia de peso adecuada, y le explica que los recién nacidos (RN) de madres con
aumento de peso insuficiente son más propensos a: señale el literal correcto

a. Prematuros, pequeños para la edad c. Posibles consecuencias adicionales a largo


gestacional y repercute en el desarrollo plazo sobre funciones endócrinas y
cognitivo del niño cardiovasculares
b. están predispuestos a la obesidad y a d. Todo es verdadero
problemas metabólicos durante su vida
ANS: D
Los recién nacidos (RN) de madres con aumento de peso insuficiente son más propensos a ser
prematuros y pequeños para la edad gestacional. También están predispuestos a la obesidad y a
problemas metabólicos durante su vida. La exposición fetal a factores limitantes del crecimiento,
de forma intensa o prolongada, puede no solo resultar en un sustancial retardo del crecimiento
prenatal, sino también en un persistente retardo o reducción del crecimiento posnatal, con posibles
consecuencias adicionales a largo plazo sobre funciones endócrinas, metabólicas y
cardiovasculares. La deficiencia de micronutrientes afecta al crecimiento intrauterino, que
repercute en el retardo en talla, el desarrollo cognitivo del niño, el cual está determinado por
condiciones sociales y económicas como la pobreza de la madre.

PTS: 1 DIF: MEDIA


REF: GPC /MSP Alimentación y nutrición de la mujer gestante y de la madre en periodo de
lactancia. Pág.: 14 OBJ: VALORAR ESTADO NUTRICIONAL EN EL EMBARAZO
TOP: NUTRICION EN EL EMBARAZO

1
Name: ________________________ Class: ___________________ Date: __________ ID: A

Bioética

Multiple Choice
Identify the choice that best completes the statement or answers the question.

____ 1. Señale qué criterio se toma en cuenta en el documento de Directrices Anticipadas


(testamento vital) de un enfermo terminal:
a. De sustitución c. De discriminación
b. Estigmatización d. De paternalismo

____ 2. Seleccione la opción que contiene un elemento fundamental del principio de autonomía y
que corresponde al consentimiento informado:
a. La información es opcional c. Influencia sobre el sujeto
b. Comprensión del problema d. Debe haber obligatoriedad

____ 3. Señale uno de los principios bioéticos en cuidados paleativos:


a. Principio desconcentrado. c. Principio de la inviolabilidad de la
vida humana.
b. Principio Participativo. d. Principio de universalidad.
____ 4. El diagnóstico de muerte por cese de las funciones cardiorrespiratorias de forma
irreversible, se establece en la comprobación de forma inequívoca de:
a. Ausencia de latido cardíaco y de c. La irreversibilidad del cese
respiración espontánea durante cardiorrespiratorio se deberá
un período no inferior a 5 minutos, constatar tras maniobras de
tras adecuado período de reanimación cardiopulmonar
maniobras de reanimación avanzada no menor a 8 minutos
b. En casos de temperatura corporal d. Ausencia trazado
inferior a 28°C se deberá electrocardiográfico, de ausencia
recalentar el cuerpo antes de de respiración espontánea,
poder establecer la irreversibil¡dad ambas cosas durante un periodo
de la parada no inferior a 10 minutos
____ 5. Paciente de 38 años, G4 P4 A0 C 0, presenta carcinoma de mama izquierda por invasión
ganglionar recibe radioterapia y quimioterapia local y regional, se demuestra que el
tumor es estrógeno dependiente. A la 7ma semana del tratamiento se demuestra que
la paciente está embarazada. La paciente y su pareja solicitan un aborto. ¿Cuál es la
decisión que debe tomar el médico respetando los pilares de la bioética?
a. Esperar resultados de punción c. Referir a otra institución
(LA)
b. Practicar el aborto d. Suspender los tratamientos

1
Name: ________________________ ID: A

____ 6. Relacione los conceptos con sus características:


CONCEPTOS
1) ÉTICA
2) MORAL
CARACTERÍSTICAS
a ) Conjunto de normas que vienen del interior.
b ) Vinculada estrechamente a las costumbres.
c ) Tiene una base de reflexión personal.
d ) Hace referencia a las normas en una sociedad.
a. 1ab, 2cd c. 1ad,2bc
b. 1bc, 2ad d. 1ac, 2bd

____ 7. En el caso de donantes vivos se debe poner especial atención en:


a. Velar para que exista una c. Considerar éticamente correcto
proporción razonable entre el un trasplante para mejorar
riesgo para el donante y el sustancialmente la calidad de vida
beneficio para el receptor. de una persona
b. Actuar siguiendo un protocolo d. Asegurar que el proceso de
consensuado con la familia y el información sea claro y que haya
comité de ética asistencial del mediado la familia por la presión
centro emocional
____ 8. ¿El incumplimiento de qué principio de la bioética está penado por la ley?
a. Beneficencia c. No maleficencia
b. Justicia d. Autonomía

____ 9. De los siguientes enunciados, seleccione la opción que contenga los deberes
primordiales del médico según el Código de Deontología Médica
1) Respetar el cuidado de la salud de la comunidad.
2) Atender sin discriminación alguna.
3) Respetar la dignidad de la persona.
4) Renunciar a ser dominador de sus semejantes.
5) Atender con la misma diligencia a todos los pacientes.
6) Respetar el cuidado de la salud del individuo.

a. 1, 3, 6. c. 1, 4, 6.
b. 2, 5, 6. d. 3, 4, 5.

____ 10. Seleccione la opción que contiene un elemento fundamental del principio de autonomía y
que corresponde al consentimiento informado
a. La información es opcional c. Comprensión del problema
b. Influencia sobre el sujeto d. Debe haber obligatoriedad

2
Name: ________________________ ID: A

____ 11. El diagnóstico de muerte por cese de las funciones cardiorrespiratorias de forma
irreversible, se establece en la comprobación de forma inequívoca de
a. Ausencia trazado c. La irreversibilidad del cese
electrocardiográfico, de ausencia cardiorrespiratorio se deberá
de respiración espontánea, constatar tras maniobras de
ambas cosas durante un periodo reanimación cardiopulmonar
no inferior a 10 minutos avanzada no menor a 8 minutos
b. Ausencia de latido cardíaco y de d. En casos de temperatura corporal
respiración espontánea durante inferior a 28°C se deberá
un período no inferior a 5 minutos, recalentar el cuerpo antes de
tras adecuado período de poder establecer la irreversibilidad
maniobras de reanimación de la parada
____ 12. Señale uno de los principios bioéticos en cuidados paleativos
a. Principio de universalidad c. Principio desconcentrado
b. Principio Participativo d. Principio de la inviolabilidad de la
vida humana
____ 13. En el caso de donantes vivos se debe poner especial atención en
a. Asegurar que el proceso de c. Considerar éticamente correcto
información sea claro y que haya un trasplante para mejorar
mediado la familia por la presión sustancialmente la calidad de vida
emocional de una persona
b. Actuar siguiendo un protocolo d. Velar para que exista una
consensuado con la familia y el proporción razonable entre el
comité de ética asistencial del riesgo para el donante y el
centro beneficio para el receptor
____ 14. Señale qué criterio se toma en cuenta en el documento de Directrices Anticipadas
(testamento vital) de un enfermo terminal
a. De paternalismo c. De sustitución
b. De discriminación d. Estigmatización
____ 15. ¿El incumplimiento de qué principio de la bioética está penado por la ley?
a. No maleficencia c. Beneficencia
b. Justicia d. Autonomía

____ 16. Paciente de 38 años, G4 P4 AO C 0, presenta carcinoma de mama izquierda por


invasión ganglionar recibe radioterapia y quimioterapia local y regional, se demuestra
que el tumor es estrógeno dependiente. A la 7ma semana del tratamiento se demuestra
que la paciente está embarazada. La paciente y su pareja solicitan un aborto. ¿Cuál es
la decisión que debe tomar el médico respetando los pilares de la bioética?
a. Suspender los tratamientos c. Referir a otra institución
b. Esperar resultados de punción d. Practicar el aborto
(LA)

3
Name: ________________________ ID: A

____ 17. De los siguientes enunciados, seleccione la opción que contenga los deberes
primordiales del médico según el Código de Deontología Médica:
1) Respetar el cuidado de la salud de la comunidad.
2) Atender sin discriminación alguna.
3) Respetar la dignidad de la persona.
4) Renunciar a ser dominador de sus semejantes.
5) Atender con la misma diligencia a todos los pacientes.
6) Respetar el cuidado de la salud del individuo.
a. 1, 3, 6 c. 1, 4, 6
b. 2, 5, 6 d. 3, 4, 5

____ 18. Relacione los conceptos con sus características:


CONCEPTOS
1) ÉTICA
2) MORAL
CARACTERÍSTICAS
a ) Conjunto de normas que vienen del interior.
b ) Vinculada estrechamente a las costumbres.
c ) Tiene una base de reflexión personal.
d ) Hace referencia a las normas en una sociedad.
a. 1ab, 2cd c. 1ad, 2bc
b. 1ac, 2bd d. 1bc, 2ad

____ 19. El consentimiento informado debe reunir los siguientes requisitos, EXCEPTO
a. Información comprensible c. Capacidad de decisión
b. Voluntariedad de decisión d. Tecnicismos médicos

____ 20. Paciente Testigo de Jehová, natural de Cuenca, mayor de edad, se encuentra ingresado
en un hospital. El paciente manifiesta libre y voluntariamente su negativa a aceptar una
transfusión de sangre entera o componentes sanguíneos, para una cirugía que tendrá
en pocas horas, aun si los médicos la consideraran necesarias para preservar su vida y
salud. Nos informa que no quiere sufrir los efectos perjudiciales y letales de la
transfusión de sangre por los riesgos que conlleva. Además, presenta su DPA
(Declaración previa de voluntad para la atención médica), debidamente completada.
Sin embargo, no se opone a otros procedimientos médicos alternativos al uso de la
sangre. Por la situación crítica en la que se encuentra, los médicos recomiendan una
transfusión pese a no encontrarse en una situación de urgencia y emergencia. ¿Qué
decisión debe tomar el médico-cirujano encargado de la intervención quirúrgica del
paciente?
a. Respetar la decisión del paciente c. Solicitar la presencia de un
en todo momento, aun si está en familiar, que no sea Testigo de
desacuerdo con ella Jehová, para que decida por el
paciente
b. Transfundir sangre, pues el d. Convencer al paciente que acepte
derecho a la vida es más una transfusión de sangre, al ser
importante que el derecho a la la recomendación médica más
libertad religiosa viable

4
Name: ________________________ ID: A

____ 21. Mientras usted termina la consulta de una paciente diagnosticada con una Enfermedad
de Transmisión Sexual ingresa al consultorio uno de sus colegas. Al siguiente día su
colega le comenta que la paciente que usted atendió es la esposa de uno de sus
amigos, y la menciona que te gustaría saber su diagnóstico para poder ayudar a su
amigo. ¿En este caso cuál debe ser la respuesta a su colega?
a. "Me parece que tienes un c. "No me parece adecuado y
comportamiento poco ético y profesional que conversemos
profesional de tu parte en sobre este tema"
preguntarme esto"
b. "Entiendo de tu interés en ayudar, d. "La próxima vez que venga mi
busquemos un lugar privado para paciente puedo pedir su
conversar" autorización para esto"
____ 22. El consentimiento informado procede de uno de los cuatro principios de la bioética, ¿cuál
es este principio?
a. Justicia c. No maleficencia
b. Beneficencia d. Autonomía
____ 23. Feto de 24 semanas de edad gestacional con base a ecografía realizada a las 20
semanas. Madre inicia actividad uterina y no es posible inhibirla, por lo que el parto es
inminente. Los padres indican al médico que dado el mal pronóstico del recién nacido y
siendo ellos los tutores legales del neonato, prefieren que no se lo reanime. ¿Cuál de
las siguientes acciones debería ser la conducta ética del médico?
a. Reanimar al recién nacido con c. No reanimar al recién nacido
todo el rigor de las conforme a lo solicitado por los
recomendaciones médicas pues padres, puesto que ellos son los
el deber del facultativo es ante tutores legales del neonato que
todo salvaguardar el derecho a la no puede decidir por sí mismo
vida de los individuos
b. Informar a los padres que se d. Indicar a los padres que esa es
tratará de respetar su voluntad una decisión técnica que compete
pero que la conducta médica a los médicos y que los padres en
dependerá de la condición este caso solo deben preocuparse
objetiva en la que nazca el recién por asumir su rol de progenitores
nacido
____ 24. El Consentimiento Informado realizado por un paciente de forma involuntaria, no es
validado desde el punto de vista legal ni ético. Según la President's Commissionfor the
Study of Ethlcal Problems in Medicine and Biomedical and Behavioral Research, la
limitación de la libertad puede presentarse de tres formas posibles, EXCEPTO:
a. La Información c. La Manipulación
b. La Coacción d. La Persuasión

5
Name: ________________________ ID: A

____ 25. Indique, ¿En qué caso debe mantenerse el principio de confidencialidad de los datos o
secreto profesional?
a. Un paciente adulto mayor con c. Un paciente con diagnóstico de
diagnóstico de cáncer en estado VIH positivo, no desea ser
terminal y con poco tiempo de tratado. Tiene su pareja estable.
sobrevida. El paciente solicita no Solicita mantener la
informar a sus familiares confidencialidad de los resultados
b. En una cirugía por obstrucción d. Una mujer de 14 años, presenta
intestinal, un cirujano encuentra sangrado vaginal. En la vagina se
varios paquetes tubulares con observa la presencia de pastillas
contenido de un polvo hexagonales. Ella solicita no
blanquecino en el intestino informar a sus padres

____ 26. La ausencia de uno de los siguientes signos es imprescindible para el diagnóstico de
muerte cerebral. Señale el que corresponda:
a. Signo de Babinski c. Exoftalmia
b. Reflejos osteotendinosos d. Reflejo corneal

____ 27. Se vive una situación de guerra que implica conflicto con riesgo de muerte para las
personas inmiscuidas en el mismo. El médico presencia un accidente que implica un
riesgo vital para quien lo sufrió. ¿Cuál de las conductas siguientes es la más ética para
el médico?
a. No aplicar los primeros auxilios c. Aplicar en el paciente los primeros
puesto que hacerlo supone un auxilios, independientemente del
riesgo para la vida del galeno y su riesgo que ello supone para la
vida es la prioridad sobre la del seguridad del galeno
paciente
b. Avisar del accidente al personal d. Solicitar a la autoridad militar más
de salud que trabaja en esa área cercana, el envío de un equipo de
(911), pero no intervenir rescate para que el paciente sea
directamente pues la vida del trasladado a una unidad de salud
médico está en riesgo
____ 28. Indique, ¿En qué caso debe mantenerse el principio de confidencialidad de los datos o
secreto profesional?
a. Un paciente con diagnóstico de c. En una cirugía por obstrucción
VIH positivo, no desea ser intestinal, un cirujano encuentra
tratado. Tiene su pareja estable. varios paquetes tubulares con
Solicita mantener la contenido de un polvo
confidencialidad de los resultados blanquecino en el intestino
b. Una mujer de 14 años, presenta d. Un paciente adulto mayor con
sangrado vaginal. En la vagina se diagnóstico de cáncer en estado
observa la presencia de pastillas terminal y con poco tiempo de
hexagonales. Ella solicita no sobrevida. El paciente solicita no
informar a sus padres informar a sus familiares
____ 29. La ausencia de uno de los siguientes signos es imprescindibe para el diagnóstico de la
muerte cerebral. Señale el que corresponda:
a. Reflejo corneal c. Signo de Babinski
b. Reflejos osteotendinosos d. Exoftalmla

6
Name: ________________________ ID: A

____ 30. El Consentimiento Informado realizado por un paciente de forma involuntaria, no es


validado desde el punto de vista legal ni ético. Según la President's Commlssion for the
Study of Ethical Problems in Medicine and Biomedical and Behavioral Research, la
limitación de la libertad puede presentarse de tres formas posibles, EXCEPTO:
a. La Información c. La Manipulación
b. La Coacción d. La Persuasión

____ 31. Paciente Testigo de Jehová, natural de Cuenca, mayor de edad, se encuentra ingresado
en un hospital. El [paciente manifiesta libre y voluntariamente su negativa a aceptar
una transfusión de sangre entera o componentes sanguíneos, para una cirugía que
tendrá en pocas horas, aun si los médicos la consideraran necesarias para preservar
su vida y salud. Nos informa que no quiere sufrir los efectos perjudiciales y letales de la
transfusión de sangre por los riesgos que conlleva. Además, presenta su DPA
(Declaración previa de voluntad para la atención médica}, debidamente completada.
Sin embargo, no se opone a otros procedimiento médicos alternativos al uso de la
sangre. Por la situación critica en la que se encuentra, los médicos recomiendan una
transfusión pesa a no encontrarse en una situación de urgencia y emergencia. ¿Que
decisión debe tomar el médico-cirujano encargado de la intervención quirúrgica del
paciente?
a. Solicitar la presencia de un c. Respetar la decisión del paciente
familiar, que no sea Testigo de en todo momento, aun si está en
Jehová, para que decida por el desacuerdo con ella.
paciente
b. Convencer al paciente que acepte d. Transfundir sangre, pues el
una tranfusión de sangre, al ser la derecho a la vida es más
recomendación médica más importante que el derecho a la
viable. libertad religiosa
____ 32. Mientras usted termina la consulta de una paciente diagnosticada con una Enfermedad
de Transmisión Sexual ingresa al consultorio uno de sus colegas. Al siguiente día su
colega le comenta que la paciente que usted atendió es la esposa de uno de sus
amigos, y le menciona que le gustaría saber su diagnóstico para poder ayudar a su
amigo. ¿En este caso cuál debe ser la respuesta a su colega?
a. "No me parece adecuado y c. "Entiendo de tu interés en ayudar,
profesional que conversemos busquemos un lugar privado para
sobre este tema" conversar"
b. "La próxima vez que venga mi d. "Me parece que tienes un
paciente puedo pedir su comportamiento poco ético y
autorización para esto" profesional de tu parte en
preguntarme esto"
____ 33. El consentimiento informado procede de uno de los cuatro principios de la bioética, ¿cual
es este principio?
a. Autonomía c. Beneficencia
b. Justicia d. No maleficencia
____ 34. El consentimiento Informado debe reunir los siguientes requisitos, EXCEPTO:
a. Capacidad de decisión c. Información comprensible
b. Voluntariedad de decisión d. Tecnicismos médicos

7
Name: ________________________ ID: A

____ 35. Feto de 24 semanas de edad gestacional con base a ecografía realizada a las 20
semanas, Madre inicia actividad uterina y no es posible inhibirla, por lo que el parto es
inminente. Los padres indican al médico que dado el mal pronóstico del recién nacido y
siendo ellos los tutores legales del neonato, prefieren que no se lo reanime. ¿Cuál de
las siguientes acciones debería ser la conducta ética del médico?
a. Reanimar al recién nacido con c. Indicar a los padres que esa es
todo el rigor de las una decisión técnica que compete
recomendaciones médicas pues a los médicos y que los padres en
el deber del facultativo es ante este caso solo deben preocuparse
todo salvaguardar el derecho a la por asumir su rol de progenitores
vida de los individuos
b. No reanimar al recién nacido d. Informar a los padres que se
conforme a lo solicitado por los tratará de respetar su voluntad
padres, puesto que ellos son los pero que la conducta médica
tutores legales del neonato que dependerá de la condición
no puede decidir por sí mismo objetiva en la que nazca el recién
nacido
____ 36. Se vive una situación de guerra que implica conflicto con riesgo de muerte para las
personas inmiscuidas en el mismo. El médico presencia un accidente que Implica un
riesgo vital para quien lo sufrió. ¿Cuál de las conductas siguientes es la más ética para
el médico?
a. Aplicar en el paciente los primeros c. No aplicar los primeros auxilios
auxilios, independientemente del puesto que hacerlo supone un
riesgo que ello supone para la riesgo para la vida del galeno y su
seguridad del galeno vida es la prioridad sobre la del
paciente
b. Solicitar a la autoridad militar más d. Avisar del accidente al personal
cercana, el envío de un equipo de de salud que trabaja en esa área
rescate para que el paciente sea (911), pero no intervenir
trasladado a una unidad de salud directamente pues la vida del
médico está en riesgo
____ 37. Usted va a empezar a realizar una investigación observacional analítica para buscar la
relación que existe entre la obesidad y los videos juegos en estudiantes de secundaria,
mayores de 13 años y menores de 18 años. Para la expresión de voluntariedad y
ejercicio de autonomía. ¿Qué documentos deben firmar los participantes?
a. Consentimiento informado del c. Consentimiento informado firmado
participante y de sus padres o por los padres y la carta de
representante legal aprobación del director escolar
b. Consentimiento informado firmado d. Carta de aprobación del
por el participante y la carta de director(a) del colegio
aprobación del director escolar considerando que es un estudio
observacional analítico
____ 38. De los siguientes casos, seleccione aquel que requiere ¡a firma del consentimiento
informado
a. Situaciones de emergencia c. Papanicolaou para despistaje de
lesiones pre-invasoras
b. Tratamientos exigidos por la ley d. Procedimientos de reproducción
como en pandemias asistida

8
Name: ________________________ ID: A

____ 39. En relación con el secreto profesional, confidencialidad y privacidad en adolescentes,


se deben tomar en cuenta las siguientes consideraciones. EXCEPTO:
a. Para la prestación de servicios de c. Los adolescentes deben ser
salud a los/las adolescentes informados sobre políticas de
siempre es necesaria la presencia confidencialidad del servicio al
de uno de los padres o el que asisten, incluyendo las
representante legal. circunstancias en las que puede
ser necesario romper la
confidencialidad
b. Los profesionales de la salud d. La protección de la
deben garantizar a los/las confidencialidad es un
adolescentes la confidencialidad componente esencial de la
de la información que les ha sido atención de salud para
revelada en la consulta adolescentes, ya que es
compatible con el desarrollo
progresivo de su madurez y su
autonomía
____ 40. Paciente femenina con enfermedad catastrófica, solicita a su médico le ayude a terminar
con su vida, este le administra una sustancia específica a sabiendas que causará la
muerte de la paciente. Según el Código Integral Penal del Ecuador. ¿Qué tipo de delito
comete el médico?
a. Abandono de personal c. Femicidio
b. Sicariato d. Asesinato

____ 41. La constancia por escrito del consentimiento informado es obligatorio en las siguientes
situaciones. EXCEPTO:
a. Transfusiones de sangre en casos c. Intervenciones quirúrgicas
de emergencia o urgencia
b. Procedimientos de reproducción d. Examen radiológico con anestesia
asistida y/o medios de contraste
____ 42. Paciente ole 7 años es diagnosticado con faringoamigdalitis aguda, el médico prescribe
un antibiótico intramuscular para su infección. La madre solicita un antibiótico por vía
oral para su hijo, pero el médico desecha esa posibilidad, argumentando que es mejor
para el niño recibir el tratamiento en un solo pinchazo. Escoja el modelo de relación
médico-paciente correspondiente a este caso clínico
a. Paternalista c. Informativo
b. Interpretativo d. Autonomista

____ 43. ¿En qué caso el consentimiento informado debe ser escrito y notariado?
a. Exámenes radiológicos que c. Donante vivo y trasplante de
impliquen anestesia y/o medios órganos
de contraste
b. Procedimientos endoscópicos y d. Tratamientos de radioterapia y
biopsias quimioterapia

9
Name: ________________________ ID: A

____ 44. Paciente acude con su hijo de 12 meses a vacunación en el centro de salud. Le
corresponden vacunas de sarampión, rubéola, parotiditis (SRP) y fiebre amarilla. La
paciente indica a la enfermera que le administre solamente la vacuna de SRP. ¿Cuál es
la conducta a seguir en esta situación?
a. Se le solicita que firme el c. Se le sugiere que acuda a un
consentimiento informado donde centro privado de vacunación
rechaza el procedimiento y no se para que le apliquen vacuna
le aplica ninguna vacuna porque contra la fiebre amarilla y que
el esquema debe completarse firme el consentimiento informado
estrictamente que rechaza el procedimiento
b. Brindar información sobre los d. Se le brinda información sobre los
beneficios y riesgos de la vacuna beneficios y riesgos de la vacuna
de fiebre amarilla, y firmar el de fiebre amarilla y se le solicita
consentimiento informado donde que regrese otro día una vez que
rechaza el procedimiento y se reflexione sobre la información
aplica sólo SRP brindada
____ 45. En relación con el secreto profesional, todos los enunciados son verdaderos. EXCEPTO:
a. No hay obligación de los c. El secreto médico es la categoría
profesionales de la salud de que se asigna a toda información
establecimientos públicos o que es revelada por un/a
privados, de denunciar un usuario/a al profesional que le
presunto delito cuando el brinda la atención de salud
conocimiento de los hechos esté
amparado por el secreto
profesional
b. Existen casos donde la ruptura d. Bajo cualquier circunstancia es
del secreto profesional o el obligación de los profesionales de
denunciar, no causan daño al la salud de establecimientos
paciente, sino lo protegen de públicos o privados presentar una
situaciones de vulnerabilidad e denuncia, al conocer la comisión
indefensión de un presunto delito
____ 46. Los siguientes enunciados corresponden a los factores causales de la distanasia
(obstinación terapéutica), EXCEPTO:
a. Convencimiento de algunos c. Adopción de medidas
médicos de que la vida biológica terapéuticas que contemplan más
es un bien por el que luchar, al los aspectos científicos de la
margen de consideraciones sobre enfermedad que al enfermo
la calidad de esa vida afectado de un proceso
irreversible
b. Deseo del paciente crítico por d. Desconocer el derecho del
eliminar su propia existencia o de paciente o de sus representantes
sus representantes legales, para legales, a rechazar el inicio o
evitar la prolongación del continuación de tratamientos
sufrimiento del paciente médicos que prolonguen el
sufrimiento del enfermo crítico o
su agonía.

10
Name: ________________________ ID: A

____ 47. Paciente de 9 años con síndrome de Laron que ha sido seleccionado para entrar en un
ensayo clínico multicéntrico para probar un nuevo IGF-1 recombinante humano. El niño
y sus padres firman el consentimiento informado. ¿Qué principio bioético está
ejerciendo el paciente?
a. Justicia c. Beneficencia
b. Autonomía d. Privacidad

____ 48. Señale en qué caso se respeta la objeción de conciencia del médico:
a. Padecimientos graves c. Petición expresa y reiterada del
insoportables paciente
b. Enfermedad clínica irreversible e d. Suicidio médicamente asistido
incurable
____ 49. Complete la siguiente frase: en relación con los principios bioéticos, el respeto a las
decisiones anticipadas ó directrices previas de un paciente incompetente en la fase
final de la vida promueve la........
a. No maleficencia c. Beneficencia
b. Justicia d. Autonomía

____ 50. Señale en qué circunstancia la firma de aceptación del consentimiento informado debe
ser notarizada
a. Procedimientos diagnósticos c. Terapias nuevas de baja
complejidad
b. Trasplante de órganos d. Intervenciones quirúrgicas
____ 51. Las personas que tienen derecho a solicitar copia certificada de la Historia Clínica en un
establecimiento de salud público o privado son los siguientes, EXCEPTO:
a. Los representantes legales o c. Los usuarios de los servicios del
apoderado(a) para menores de establecimiento de salud público o
edad privado
b. Las instituciones que representan d. Familiares de segundo y tercer
las autoridades judiciales grado de consanguinidad para
fallecidos
____ 52. Señale qué principio de la bioética permite al paciente rechazar un tratamiento:
a. Justicia c. Beneficencia
b. Autodeterminación d. No maledicencia

____ 53. Paciente de 68 años, diagnosticado con diabetes se hospitaliza por descompensación de
valores glicémicos. Señale la opción que refleja un modelo autonomista en la relación
clínica de este paciente
a. Paciente debe seguir c. Establecer con el paciente los
estrictamente indicaciones objetivos que se quieren lograr y
médicas aún cuando tiene que él quiera lograr
opiniones contrarias
b. El paciente menciona que dejará d. El médico toma todas las
que el médico tome las decisiones decisiones porque el paciente no
por él. sabe nada de medicina

11
Name: ________________________ ID: A

____ 54. Relacione las siguientes categorías y los procesos en relación con el consentimiento
informado (Cl):
1. Niños/as mayores de 12 años y adolescentes.
2. Padres menores de edad emancipados.
3. Menor de edad de padres menores de edad no emancipados.
4. Disputa entre representantes legales de menores de edad.
a) El profesional de la salud, basándose en la valoración clínica, actuará en
consideración al interés superior y beneficio del paciente.
b) Uno de los abuelos del menor, bajo cuya patria potestad viva, podrá suscribir el
consentimiento informado.
c) El consentimiento podrá ser suscrito por los mismos, tanto para intervenciones
clínicas en ellos o en sus hijos.
d) Informar al menor verbalmente el procedimiento médico que se le va a realizar,
utilizando términos sencillos, claros y con calidez. Solicitar además consentimiento
informado escrito de los padres o
representantes legales en caso de requerirse.
a. 1a, 2c, 3b, 4d. c. 1d, 2b, 3c, 4a.
b. 1d, 2c, 3b, 4a. d. 1a, 2c, 3d, 4b.

12
ID: A

Bioética
Answer Section

MULTIPLE CHOICE

1. ANS: A PTS: 1
2. ANS: B PTS: 1
3. ANS: C PTS: 1
4. ANS: A PTS: 1
5. ANS: B PTS: 1
6. ANS: D PTS: 1
7. ANS: A PTS: 1
8. ANS: C PTS: 1
9. ANS: A PTS: 1
10. ANS: C PTS: 1
11. ANS: B PTS: 1
12. ANS: D PTS: 1
13. ANS: D PTS: 1
14. ANS: C PTS: 1
15. ANS: A PTS: 1
16. ANS: D PTS: 1
17. ANS: A PTS: 1
18. ANS: B PTS: 1
19. ANS: D PTS: 1
20. ANS: A PTS: 1
21. ANS: C PTS: 1
22. ANS: D PTS: 1
23. ANS: B PTS: 1
24. ANS: A PTS: 1
25. ANS: A PTS: 1
26. ANS: D PTS: 1
27. ANS: B PTS: 1
28. ANS: D PTS: 1
29. ANS: A PTS: 1
30. ANS: A PTS: 1
31. ANS: C PTS: 1
32. ANS: A PTS: 1
33. ANS: A PTS: 1
34. ANS: D PTS: 1
35. ANS: D PTS: 1
36. ANS: D PTS: 1
37. ANS: A PTS: 1
38. ANS: D PTS: 1
39. ANS: A PTS: 1
40. ANS: D PTS: 1
41. ANS: A PTS: 1
42. ANS: A PTS: 1
43. ANS: C PTS: 1

1
ID: A

44. ANS: B PTS: 1


45. ANS: D PTS: 1
46. ANS: B PTS: 1
47. ANS: B PTS: 1
48. ANS: D PTS: 1
49. ANS: D PTS: 1
50. ANS: B PTS: 1
51. ANS: D PTS: 1
52. ANS: B PTS: 1
53. ANS: C PTS: 1
54. ANS: B PTS: 1

2
Bioética [Answer Strip] ID: A

D
_____ 6. B 11.
_____ A 17.
_____ C 21.
_____

A
_____ 1.

B
_____ 2.
B 18.
_____
A
_____ 7.
D 12.
_____ D 22.
_____

C
_____ 3.

D 13.
_____ B 23.
_____

A
_____ 4.

C
_____ 8.
D 19.
_____

A
_____ 9.
A 20.
_____
C 14.
_____

A 15.
_____
B
_____ 5.

D 16.
_____ A 24.
_____

C 10.
_____
Bioética [Answer Strip] ID: A

A 25.
_____ A 30.
_____ D 35.
_____ A 39.
_____ B 44.
_____

C 31.
_____

D 26.
_____

D 45.
_____
B 27.
_____ D 36.
_____ D 40.
_____

A 41.
_____

A 32.
_____

A 42.
_____
D 28.
_____ A 37.
_____ B 46.
_____

C 43.
_____

A 33.
_____

D 38.
_____
A 29.
_____ D 34.
_____
Bioética [Answer Strip] ID: A

B 47.
_____ B 54.
_____

D 48.
_____

D 49.
_____

B 50.
_____

D 51.
_____

B 52.
_____

C 53.
_____
Name: ________________________ Class: ___________________ Date: __________ ID: A

CIRUGÍA GENERAL

Multiple Choice
Identify the choice that best completes the statement or answers the question.

____ 1. De las siguientes alternativas, en una paciente mujer de 70 años de edad, con dolor abdominal cólico
localizado en hipocondrio derecho, signos de deterioro físico, falta de apetito, baja de peso; cual sería una
contraindicación absoluta para colecistectomía laparoscópica
a. Cirugía abdominal previa
b. Peritonitis difusa
c. Sospecha de carcinoma de vesícula biliar
d. Sospecha de coledocolitiasis

____ 2. Paciente mujer de 48 años, quien presenta dolor en hipocondrio derecho, ictericia, se tiene como sospecha
obstrucción de la vía biliar por litiasis, se decide realizar ultrasonografía, la cual tiene una capacidad para
establecer el diagnóstico de coledocolitasis en un:
a. 100%
b. 50%
c. 90%
d. 28%

____ 3. Paciente varón de 70 años de edad quien presenta signos de colangitis, perso sin fiebre; no se puede
descartar el diagnóstico indicado, ya que la fiebre en casos de colangitis aparece en un:
a. 98%
b. 90%
c. 70%
d. 60%

____ 4. En el caso de una paciente mujer de 55 años de edad con posibilidad diagnóstica de pancreatitis de etiología
biliar, se decide manejar la evolución de la enfermedad basados en la escala de APACHE II, ya que es:
a. Un sistema de puntuación fisiológico que trata de estimar la gravedad de la enfermedad
b. Es específico para pancreatitis aguda
c. Debe esperar 48 horas de evolución de la enfermedad para aplicarlo
d. Perso no es útil luego de intervención quirúrgica por la pancratitis

____ 5. En un apaciente varón de 72 años, con diagnóstico de pancreatitis aguda grave de etiología biliar, en quien
se discute el tratamiento a seguir referente a una posible necrosis pancreática infectada, señale el criterio
correcto:
a. La mortalidad es de casi el 100% sin interveción
b. Con terapia quirúrgica adecuada la mortalidad es del 50%
c. En la mayoría de pacientes se evidencia radiológicamente la infección de la necrosis
d. No es útil la PAF guiada por TAC para el diagnóstico de necrosis infectada

____ 6. En el caso de un paciente varón de 46 años, con antecedente de laparotomía hace 4 años por peritonitis
secundaria a apendicitis complicada, quien acude con una masa reductible en la herida quirúrgica, de 2
años de evolución, que progresivamente incrementa de tamaño y produce dolor, ante la posibilidad
diagnóstica de hernia incisional, señale cual de los siguientes criterios es verdadero:
a. La infección de una hedida de laparotomía no influye en la aparición de hernia incisional
b. 2 al 11% de los pacientes sometidos a laparotomía desarrollan hernia incisional
c. 20 al 40% de los pacientes sometidos a laparotomía desarrollan hernia inciosnal
d. El sitio de la incisión y el método de cierre en la laparotomía inicial no influye en el
aparecimiento posterior de hernia incisional

1
Name: ________________________ ID: A

____ 7. Un paciente con hemorroides que sobresalen y sangran con la defecación y que debe reducirlas
manualmente, que tipo de hemorroides tiene:
a. interna grado l
b. mixtas
c. internas grado lll
d. internas grado lV

____ 8. En un paciente con proctorragia y que ha sido diagnosticado por rectosigmoideoscopía de hemorroides
grado ll, cual sería el procedimiento a seguir:
a. hemorroidectomía con técnica abierta
b. hemorroidectomía con técnica submucosa
c. cambio de dieta más cirugía
d. Ligadura con banda elástica

____ 9. En un paciente varón de 12 años con cuadro de dolor abdominal de 10 horas de evolución, iniciado en
epigastrio, con McBurney positivo, cual sería su primera posibilidad diagnóstica:
a. intusucepcción
b. adenitis mesentérica
c. apendicitis aguda
d. colitis

____ 10. En un paciente de 15 años con cuadro de dolor abdominal de 20 horas, que a la ecosonografía presenta:
engrosamiento de la pared apendicular, con pérdida de su compresibilidad, aumento de la ecogenicidad
de la grasa circunvencina, más liíquido pericecal loculado. Cual sería su diagnóstico
a. Apendicitis aguda
b. Peritonitis
c. Adenitis mesentérica
d. Plastrón apendicular

2
ID: A

CIRUGÍA GENERAL
Answer Section

MULTIPLE CHOICE

1. ANS: C
La sospecha de enfermedad maligna de la vesícula biliar obliga a que se realice resección abierta
estándar, para una remoción adecuada adecuada del tumor

PTS: 3 DIF: alta


REF: Nagle, Alexander; Colecistectomía (abierta y laparoscópica) en Maingot Operaciones
Abdominales; cap 32, pag 849 OBJ: Conocer indicaciones de colecistectomía
TOP: Vía biliar y páncreas KEY: colecistectomía, laparoscopía, cáncer vesicular
NOT: Dr. Vinicio Moreno Rueda
2. ANS: B
La capacidad de la ultrasonografía transcutánea para establecer e diagnóstico de coledocolitiasis es solo
de 50%, varía de 30 a 90%

PTS: 3 DIF: alta


REF: McFadden, David; Coledocolitiasis y colangitis; en Maingot Operaciones Abdominales, capítulo
33, pag 866 OBJ: Conocer el rendimiento de los métodos de diagnóstico en patología biliar
TOP: vía biliar y páncreas KEY: coledocolitiasis, ecosonografía
NOT: Dr. Vinicio Moreno Rueda
3. ANS: B
En el momento de la presentación de los pacientes con colangitis, la fiebre se la constata en el 90% de los
enfermos, el dolor abdominal en el 70% y la ictericia en el 60%

PTS: 3 DIF: alta


REF: McFadden, David; Coledocolitiasis y colangitis; en Maingot Operaciones Abdominales, cap 33,
pag 880 OBJ: conocimiento de la presntación clínica de la colangitis
TOP: patología de la vía biliar y páncreas KEY: colangitis, fiebre
NOT: Dr. Vinicio Moreno Rueda
4. ANS: A
El APACHE II es un sistema de puntuación fisiológico que intenta estimar la gravedad de la enfermedad
con base en la cuantificación del grado de anormalidad de múltiples variables fisiológicas, no es
específico para pancreatitis, y se lo usa desde el incio de la artención al pacientes, día a día, siendo muy
útil luego de la intervenciones quirúrgicas que se pudieren necesitar

PTS: 3 DIF: 3
REF: Clancy, Thomas, Manejo de la pancreatitis aguda; en maingot Operaciones Abdomnales, cap 36,
pag 941 OBJ: Conocimiento de escalas de valoración y manjo de la pancreatitis
TOP: patología de de vía biliar y pancreas KEY: APACHE II, pancratitis aguda
NOT: Dr. Vinicio Moreno Rueda

1
ID: A

5. ANS: A
En casos de necrosis infectada en pancratitis aguda grave sin tratamiento quirúrgico la mortalidad se
acerca al 100%, la cual disminuye al 15% con una intervención adecuada, pocos casos son diagnosticado
con radiología ante la sospecha de necrosis pancreatitica y puede ser por la presencia de neumoperitoneo
y finalmente el diagnóstico adecuado de infección de la necrosis se lo relaiza con PAF guiada por TAC

PTS: 3 DIF: 3
REF: Clancy, Thomas; Manejo de la pancratitis aguda; en Maingot Operaciones abdominales, cap 36,
pag 947 OBJ: conocimiento de patología pancreatitca TOP: Enfermedad de vía
biliar y páncreas
KEY: pancratitis aguda grave, necrosectomía, TAC, punsión con aguja fina
NOT: Dr. Vinicio Moreno Rueda
6. ANS: B
En 2 al 11% de los pacientes en quines se efectúa laparotomía se forman hernias incisinales, con notorio
incremento si la herida quirúrgica se infectó, o si el enfermo es fumador, diabético no controlado,
malnutrido, usa esteroides, es obeso, es de edad avanzada o sufren incrementos de la presión
traabdominal por cirrosis, estreñimiento, embarazo, etc.

PTS: 3 DIF: alta


REF: Amirlak, Bardia; Reparación de hernias; en Maingot Operaciones Abdominales, cap 46, pag 1139
OBJ: Conocimiento de la fisiopatología de la eventración y su manejo
TOP: hernias de pared abdominal KEY: hernia incisional, eventración
NOT: Dr. Vinicio Moreno Rueda
7. ANS: C
Las hemorroides internas grado lll son las que sobresalen y sangran con la depocisión, por lo que el
paciente las reduce digitalmente

PTS: 3 DIF: alta


REF: Lowney, Jennifer, Trastornos benignos del anorrecto; en Maingot Operaciones abdominales, cap
24, pag 677 OBJ: Conocer la clasificación de la patología hemorroidal
TOP: Patología Proctológica KEY: hemorroide NOT: Dr. Vinicio Moreno Rueda
8. ANS: D
Este procedimiento está indicado y tiene buenos resultados en hemorroides internas grado l, ll, lll; en
externas o interna grado lV no se lo debe realizar.

PTS: 3 DIF: alta


REF: Lowney Jennifer, Trastornos benignos del anorrecto; en Maingot Operaciones Abdominales, cap
24, pag 679 OBJ: Conocimiento del manejo de la patología proctológica más común
TOP: Patología Procotlógica KEY: hemorroides, ligadura, banda elástica
NOT: Dr. Vinicio Moreno Rueda
9. ANS: C
A pesar de que los cuadros clínicos tengan cierta similitud, el grupo etareo en el varón en el que más se
diagnostica apendicitis aguda está entre los 10 y 14 años (27,6 por 10.000 habitantes).

PTS: 3 DIF: alta


REF: Smink, Douglas; Apéndice y apendicectomía; en Maingot Operaciones Abdominales, cap 21, pag
590
OBJ: Disgnosticar apendicitis aguda TOP: Abdomen agudo inflamatorio
KEY: apendicitis aguda, dolor abdominal NOT: Dr. Vinicio Moreno Rueda

2
ID: A

10. ANS: A
Por el tiempo de evolución y el hallzago ecosonográfico, debemos pensar inicialmente en una apendicitis
aguda y tratarla como tal

PTS: 3 DIF: alta


REF: Smink Douglas, apéndice y apendicectomía; en Maingot Operaciones Abdominales, cap 21, pag
594
OBJ: Diagnóstico y tratamiento adecuado de la apendicitis aguda
TOP: abdomen agudo inflamatorio KEY: apendicitis aguda, ecosonografía
NOT: Dr. Vinicio Moreno Rueda

3
CIRUGÍA GENERAL [Answer Strip] ID: A

C
_____ 7.

C
_____ 1.
D
_____ 8.

B
_____ 2. C
_____ 9.

A 10.
_____
B
_____ 3.

A
_____ 4.

A
_____ 5.

B
_____ 6.
Name: ________________________ Class: ___________________ Date: __________ ID: A

Cirugía

Multiple Choice
Identify the choice that best completes the statement or answers the question.

____ 1. Paciente de 54 años con fractura distal de radio izquierdo extraarticular tratada mediante
inmovilización con yeso. A los 2 días presenta: aumento de perímetro del antebrazo
izquierdo, parestesias, edema, palidez distal e intenso dolor que aumenta con el
estiramiento pasivo de la mano izquierda ¿Cuál es el diagnóstico?
a. Trombosis venosa profunda. c. Desplazamiento de la fractura
b. Síndrome compartimental agudo d. Celulitis del antebrazo izquierdo

____ 2. La letra D del acrónimo ABCDE de la revisión primaria en trauma corresponde a:


a. Valoración neurológica c. Valoración de vía aérea
b. Valoración circulatoria d. Valoración respiratoria

____ 3. Paciente con traumatismo abdominal cerrado hemodinámicamente inestable ¿Cuál de


los siguientes métodos diagnósticos es útil en la valoración inicial?
a. FAST (ecografía abdominal c. Radiografía simple de abdomen.
dirigida para traumatismos)
b. Tomografía helicoidal. d. DPA (aspiración peritoneal
diagnóstica)
____ 4. Las siguientes son indicaciones de referencia al oftalmólogo ante un caso de conjuntivitis
aguda, EXCEPTO:
a. Pérdida de la visión c. Epífora o lagrimeo continuo
b. Inmunosupresión d. Episodios recurrentes
____ 5. Indique la complicación más frecuente de úlcera péptica
a. Mallory Weis c. Perforación
b. Sangrado d. Obstrucción intestinal

____ 6. Paciente de 10 años con antecedente de criptorquidia derecha. Acude al hospital por
dolor en el testículo derecho de inicio súbito sin traumatismo previo. La exploración
física muestra edema con asimetría escrotal, testículo derecho en posición alta y el
reflejo cremastérico está ausente. En la ecografía Doppler hay disminución del flujo
sanguíneo intratesticular derecho. ¿Cuál es la conducta terapéutica indicada en este
paciente?
a. Alta domiciliaria con c. Diclofenaco intramuscular y
antiinflamatorios controles
b. Intervención quirúrgica inmediata d. Observación hospitalaria con
y analgésicos analgesia
____ 7. Paciente de 40 años, acude a la emergencia por presentar disuria, polaquiuria, nicturia,
se le realiza PSA el cual reporta en 2.5 y PSA libre 30%, en elemental de orina
hematíes de 5 a 6, en ecografía se reporta una próstata de 40 gramos. Indique el
diagnóstico de este paciente:
a. Cistitis aguda c. Cáncer de Próstata
b. Prostatitis aguda d. Hiperplasia prostética benigna

1
Name: ________________________ ID: A

____ 8. ¿Cuál es el examen complementario para el diagnóstico precoz de displasia congénita


de cadera en un niño menor de tres meses?
a. Ecografía c. Tomografía
b. Electromiografía d. Radiografía

____ 9. Paciente de 35 años, bebe alcohol y fuma una cajetilla de tabacos al día desde hace 2
años de forma ininterrumpida, presenta odinofagia unilateral derecha que ha
progresado a disfagia, otalgia del mismo lado, de tres meses de evolución. Al examen
físico se aprecia aumento unilateral del tamaño de la amígdala palatina ¿Cuál es el
diagnóstico más probable de este paciente?
a. Tumor amigdalino c. Tonsilitis exudativa
b. Angina de Vincent d. Absceso periamigdalino

____ 10. Paciente de 75 años con antecedente de radioterapia por cáncer de próstata, refiere
visión borrosa desde hace 10 meses para objetos lejanos y ha dejado de necesitar los
lentes para visión de cerca, no tiene dolor ni enrojecimiento ocular, presenta en fondo
de ojo alteración del reflejo rojo bilateral. ¿Cuál es el diagnóstico de este paciente?
a. Degeneración macular senil. c. Neuritis óptica isquémica.
b. Glaucoma crónico simple. d. Catarata.

____ 11. Hombre de 38 años, sin antecedentes de interés, que realiza viaje a la costa. Acude a
emergencias por dolor repentino de gran intensidad en flanco derecho, irradiado a ingle
derecha y genitales, acompañado de náuseas y vómitos. ¿Cuál es el diagnóstico de
este paciente?
a. Píelonefritis aguda c. Litiasis en la vejiga
b. Litiasis en pelvis renal d. Litiasis en uréter distal

____ 12. ¿A qué grado de luxación acromio clavicular corresponde la disrupción de los ligamentos
acromioclaviculares con indemnidad de los coracoclaviculares?
a. Grado II c. Grado III
b. Grado I d. Grado IV

____ 13. Paciente de 45 años, refiere otorrea purulenta e hipoacusia unilateral de conducción, sin
otalgia, de tres meses de evolución. ¿Cuál es el diagnóstico?
a. Mastoiditis c. Otitis media aguda
b. Otitis externa maligna d. Otitis medía crónica

____ 14. Paciente de 43 años, asintomático y con el antecedente de un hermano fallecido por
cáncer de próstata. En un control rutinario de su empresa se encuentra PSA (Antígeno
Prostático Específico) de 5.6 ng/ml, con un cociente de PSA libre/PSA total del 9%. El
tacto rectal muestra aumento de consistencia en el lóbulo derecho prostático. Indique el
método diagnóstico ideal en este paciente:
a. Gammagrafía ósea c. Resonancia magnética abdominal
y pélvica
b. Tomografía computada de d. Ecografía transrectal y biopsia
abdomen y pelvis prostática
____ 15. ¿Cuál es el diagnóstico de un paciente que tras sufrir un accidente de tránsito presenta
hipotensión severa refractaria a líquidos, bradicardia y déficit sensitivo?
a. Choque séptico c. Choque hipovolémico
b. Choque medular d. Choque cardiogénico

2
Name: ________________________ ID: A

____ 16. Indique la conducta más eficaz para el vértigo postural paroxístico benigno:
a. Sulpiride intravenoso c. Maniobra de Dix-Hallpike
b. Maniobras liberadoras d. Cinarizina oral

____ 17. Paciente de 25 años que acude a urgencias por odinofagia y fiebre de 40°C de 3 días de
evolución, desde hace 6 horas presenta dificultad para deglutir, babeo, voz gangosa y
trismus. A la exploración se objetiva abombamiento de una amígdala palatina con
desplazamiento de la úvula hacia el lado sano. Indique el diagnóstico:
a. Absceso periamigdalino c. Mononucleosis infecciosa
b. Angina de Vincent d. Angina de Ludwing

____ 18. La radiografía simple en la valoración de displasia del desarrollo de caderas está
indicada en la siguiente edad:
a. 4 a 6 meses c. 2 o 3 meses
b. 0 a 28 días d. 1 a 2 meses
____ 19. Paciente de 22 años sin antecedentes de interés, refiere que desde hace 4 días presenta
odinofagia y fiebre de 39°C. Desde hace 48 horas manifiesta dificultad para deglutir,
babeo, voz gangosa y trismus. A la exploración se objetiva abombamiento e intenso
eritema de la amígdala palatina derecha y desplazamiento de la úvula hacia el lado
opuesto. ¿Cuál es el tratamiento antimicrobiano empírico de elección?
a. Clindamicina por vía intravenosa c. Piperacilina más tazobactam
intravenoso
b. Amoxicilina más ácido clavulánico d. Azitromicina por vía oral
intravenoso
____ 20. Paciente de 80 años colecistectomizada hace 2 años, acude a emergencias por dolor
agudo en el cuadrante superior derecho, coluria, acolia, náuseas e ictericia, con
dilatación de la vía biliar mayor de 6 mm. Indique el método diagnóstico ideal en este
paciente:
a. Radiografía del tracto c. Ultrasonido complemento con
gastrointestinal inferior baritada Doppler
b. Colangiopancreatografía d. Tomografía computarizada de
retrógada endoscópica abdomen con contraste
____ 21. Señale el signo de exploración abdominal que corresponde a inflamación peritoneal de
causa no hemorrágica:
a. Signo de Charcot c. Signo de Kehr
b. Signo de Cullen d. Signo de Grey Turner
____ 22. Paciente de 40 años, acude a la emergencia por presentar disuria, polaquiuria, nicturia,
se le realiza PSA el cual reporta en 2.5 y PSA libre 30%, en elemental de orina
hematíes de 5 a 6, en ecografía se reporta una próstata de 40 gramos. Indique el
diagnóstico de este paciente:
a. Hiperplasia prostética benigna c. Prostatitis aguda
b. Cáncer de Próstata d. Cistitis aguda

____ 23. La radiografía simple en la valoración de displasia del desarrollo de caderas está
indicada en la siguiente edad:
a. 1 a 2 meses c. 2 o 3 meses
b. 4 a 6 meses d. 0 a 28 días

3
Name: ________________________ ID: A

____ 24. Paciente de 75 años con antecedente de radioterapia por cáncer de próstata, refiere
visión borrosa desde hace 10 meses para objetos lejanos y ha dejado de necesitar los
lentes para visión de cerca, no tiene dolor ni enrojecimiento ocular, presenta en fondo
de ojo alteración del reflejo rojo bilateral. ¿Cuál es el diagnóstico de este paciente?
a. Degeneración macular senil c. Catarata
b. Neuritis óptica isquémica d. Glaucoma crónico simple

____ 25. Paciente con traumatismo abdominal cerrado hemodinámicamente inestable ¿Cuál de
los siguientes métodos diagnósticos es útil en la valoración inicial?
a. DPA (aspiración peritoneal c. Radiografía simple de abdomen
diagnóstica)
b. Tomografía helicoidal d. FAST (ecografía abdominal
dirigida para traumatismos)
____ 26. Paciente de 80 años colecistectomizada hace 2 años, acude a emergencias por dolor
agudo en el cuadrante superior derecho, coluria, acolia, náuseas e ictericia, con
dilatación de la vía biliar mayor de 6 mm. Indique el método diagnóstico ideal en este
paciente:
a. Ultrasonido complemento con c. Radiografía del tracto
Doppler gastrointestinal inferior baritada
b. Colangiopancreatografía d. Tomografía computarizada de
retrógada endoscópica abdomen con contraste
____ 27. Paciente de 43 años, asintomático y con el antecedente de un hermano fallecido por
cáncer de próstata. En un control rutinario de su empresa se encuentra PSA (Antígeno
Prostático Específico) de 5.6 ng/ml, con un cociente de PSA libre/PSA total del 9%. El
tacto rectal muestra aumento de consistencia en el lóbulo derecho prostático. Indique el
método diagnóstico ideal en este paciente:
a. Ecografía transrectal y biopsia c. Tomografía computada de
prostética abdomen y pelvis
b. Gammagrafía ósea d. Resonancia magnética abdominal
y pélvica
____ 28. ¿Cuál es el examen complementario para el diagnóstico precoz de displasia congénita
de cadera en un niño menor de tres meses?
a. Electromiografía c. Ecografía
b. Radiografía d. Tomografía

____ 29. Paciente de 45 años, refiere otorrea purulenta e hipoacusia unilateral de conducción, sin
otalgia, de tres meses de evolución. ¿Cuál es el diagnóstico?
a. Otitis media crónica c. Otitis externa maligna
b. Otitis media aguda d. Mastoiditis

____ 30. Señale el signo de exploración abdominal que corresponde a inflamación peritoneal de
causa no hemorrágica:
a. Signo de Cullen c. Signo de Kehr
b. Signo de Charcot d. Signo de Grey Turner
____ 31. Indique la conducta más eficaz para el vértigo postural paroxístico benigno:
a. Maniobras liberadoras c. Maniobra de Dix-HalIpike
b. Sulpiride intravenoso d. Cinarizina oral

4
Name: ________________________ ID: A

____ 32. Hombre de 38 años, sin antecedentes de interés, que realiza viaje a la costa. Acude a
emergencias por dolor repentino de gran intensidad en flanco derecho, irradiado a ingle
derecha y genitales, acompañado de náuseas y vómitos. ¿Cuál es el diagnóstico de
este paciente?
a. Pielonefritis aguda c. Litiasis en uréter distal
b. Litiasis en pelvis renal d. Litiasis en la vejiga

____ 33. Paciente de 25 años que acude a urgencias por odinofagia y fiebre de 40°C de 3 días de
evolución, desde hace 6 horas presenta dificultad para deglutir, babeo, voz gangosa y
trismus. A la exploración se objetiva abombamiento de una amígdala palatina con
desplazamiento de la úvula hacia el lado sano. Indique el diagnóstico:
a. Mononucleosis infecciosa c. Angina de Vincent
b. Absceso periamigdalino d. Angina de Ludwing

____ 34. La letra D del acrónimo ABCDE de la revisión primaria en trauma corresponde a:
a. Valoración circulatoria c. Valoración neurológica
b. Valoración respiratoria d. Valoración de vía aérea

____ 35. Paciente de 54 años con fractura distal de radio izquierdo extraarticular tratada mediante
inmovilización con yeso. A los 2 días presenta: aumento de perímetro del antebrazo
izquierdo, parestesias, edema, palidez distal e intenso dolor que aumenta con el
estiramiento pasivo de la mano izquierda ¿Cuál es el diagnóstico?
a. Trombosis venosa profunda c. Celulitis del antebrazo izquierdo
b. Síndrome compartimental agudo d. Desplazamiento de la fractura

____ 36. Paciente de 35 años, bebe alcohol y fuma una cajetilla de tabacos al día desde hace 2
años de forma ininterrumpida, presenta odinofagia unilateral derecha que ha
progresado a disfagia, otalgia del mismo lado, de tres meses de evolución. Al examen
físico se aprecia aumento unilateral del tamaño de la amígdala palatina ¿Cuál es el
diagnóstico más probable de este paciente?
a. Angina de Vincent c. Tumor amigdalino
b. Absceso periamigdalino d. Tonsilitis exudativa

____ 37. ¿Cuál es el diagnóstico de un paciente que tras sufrir un accidente de tránsito presenta
hipotensión severa refractaria a líquidos, bradicardia y déficit sensitivo?
a. Choque séptico c. Choque cardiogénico
b. Choque hipovolémico d. Choque medular

____ 38. Las siguientes son indicaciones de referencia al oftalmólogo ante un caso de conjuntivitis
aguda, EXCEPTO:
a. Pérdida de la visión c. Episodios recurrentes
b. Epífora o lagrimeo continuo d. Inmunosupresión

5
Name: ________________________ ID: A

____ 39. Paciente de 10 años con antecedente de criptorquidia derecha. Acude al hospital por
dolor en el testículo derecho de inicio súbito sin traumatismo previo. La exploración
física muestra edema con asimetría escrotal, testículo derecho en posición alta y el
reflejo cremastérico está ausente. En la ecografía Doppler hay disminución del flujo
sanguíneo intratesticular derecho. ¿Cuál es la conducta terapéutica indicada en este
paciente?
a. Diclofenaco intramuscular y c. Observación hospitalaria con
controles analgesia
b. Intervención quirúrgica inmediata d. Alta domiciliaria con
y analgésicos antiinflamatorios
____ 40. ¿A qué grado de luxación acromio clavicular corresponde la disrupción de los ligamentos
acromioclaviculares con indemnidad de los coracoclaviculares?
a. Grado III c. Grado II
b. Grado I d. Grado IV

____ 41. Paciente de 22 años sin antecedentes de interés, refiere que desde hace 4 días presenta
odinofagia y fiebre de 39°C. Desde hace 48 horas manifiesta dificultad para deglutir,
babeo, voz gangosa y trismus. A la exploración se objetiva abombamiento e intenso
eritema de la amígdala palatina derecha y desplazamiento de la úvula hacia el lado
opuesto. ¿Cuál es el tratamiento antimicrobiano empírico de elección?
a. Clindamicina por vía intravenosa c. Piperacilina más tazobactam
intravenoso
b. Amoxicilina más ácido clavulánico d. Azitromicina por vía oral
intravenoso
____ 42. Indique la complicación más frecuente de úlcera péptica.
a. Mallory Weis c. Sangrado
b. Perforación d. Obstrucción intestinal

____ 43. Seleccione la prueba diagnóstica más importante para confirmar cáncer en un paciente
con nódulo solitario tiroideo:
a. Ecografía tiroidea c. Gammagrafía tiroidea
b. BAAF (Biopsia por aspiración con d. Dosificación de T3, T4 y TSH
aguja fina)
____ 44. Se presenta un paciente masculino de 35 años, fumador y bebedor habitual, con
antecedentes de úlcera duodenal asociada a una infección por Helicobacter pylori, para
lo cual lleva tratamiento médico. El paciente aqueja intenso dolor abdominal de
comienzo brusco "como puñalada en la boca del estómago" una hora antes, el cual se
fue haciendo difuso y más intenso, con irradiación al hombro izquierdo. Al examen
físico se constata ansiedad, sudoración, taquicardia y un abdomen muy contracturado y
doloroso al palpar y percutir, con borramiento de la matidez hepática. Identifique cual
sería la complicación de la enfermedad ulcerosa péptica que presenta el paciente:
a. Peritonitis por perforación c. Sangrado digestivo alto
b. Malignización de úlcera benigna d. Obstrucción de la salida gástrica

6
Name: ________________________ ID: A

____ 45. Paciente de 35 años, con antecedentes de haber sido intervenido quirúrgicamente por
una peritonitis secundaria a apendicitis aguda durante su infancia, se presenta con un
cuadro clínico de dolor abdominal en forma de cólico intenso que alivia con vómitos
biliosos y de contenido intestinal oscuro y fétido. Refiere tener sed y que hace más de
24 horas no defeca, ni expulsa gases por el recto. Al examen físico se comprueban las
mucosas y lengua secas, distensión abdominal asimétrica, con una cicatriz quirúrgica
media infraumbilical y un patrón acelerado de los ruidos hidroaéreos a la auscultación.
Se solicitó un Rx de abdomen simple en bipedestación el cuaO se muestra en la
Imagen. ¿Cuál considera usted es el diagnóstico de este paciente?

a. Úlcera gastroduodenal perforada c. Íleo paralítico


b. Oclusión mecánica intestinal d. Gastroenterocolitis aguda
____ 46. Para confirmar el diagnóstico de abdomen agudo, ¿cuál de las siguientes pruebas de
laboratorio se debe solicitar?
a. Proteína C Reactiva c. Procalcitonina
b. Recuento de glóbulos blancos y d. Velocidad eritorsedimentación
fórmula diferencial

7
Name: ________________________ ID: A

____ 47. Seleccione la prueba más adecuada para confirmar el diagnóstico positivo de cáncer
gástrico:
a. Endoscopia gastrointestinal c. Ecografía gástrica endoscópica
superior con biopsia
b. Tomografía axial computarizada d. Estudio radiológico baritado
abdominal simple y contrastada gastrointestinal superior con doble
contraste
____ 48. Paciente femenina de 44 años, obesa, que se presentó con dolor abdominal muy
intenso, de comienzo súbito después de una comida grasosa, acompañado de
náuseas y vómitos, localizado en el cuadrante superior derecho, con irradiación a la
región interescapular. La paciente refiere que había tenido este dolor en otras
ocasiones, pero que este es más intenso y no se alivia a pesar de que le han
administrado analgésicos en las últimas seis horas. Al examen del abdomen hay
hlpersensibllidad y resistencia en el cuadrante superior derecho y a la palpación
profunda del hipocondrio derecho se constata una masa redondeada que al comprimir
profundamente produce intenso dolor que detiene la inspiración de la paciente. Se
solicitó un conteo de células blancas que arrojó 11800 leucocitos/mm3 y una ecografía
abdominal que se muestra en la imagen. Seleccione el diagnóstico adecuado en esta
paciente:

a. Coledocolitiasis c. Colangitis ascendente


b. Colelitiasis sintomática d. Colecistitis aguda

8
Name: ________________________ ID: A

____ 49. Paciente femenina, obesa, que se presentó con dolor muy intenso, de comienzo súbito
después de una comida grasosa, acompañado de náuseas y vómitos, localizado en el
cuadrante superior derecho, con irradiación a la región interescapular. La paciente
refiere que había tenido este dolor en otras ocasiones. El examen físico y las pruebas
de laboratorio fueron normales, por lo que se administró 75 mg de diclofenaco sódico
IM, logrando alivio del dolor. Se solicitó ecografía abdominal que se muestra en la
imagen. Seleccione el diagnóstico adecuado en esta paciente.

a. Coledocolitiasis c. Coledocolitiasis
b. Colecistitis aguda d. Colelitiasis sintomática
____ 50. En un paciente con traumatismo abdominal cerrado, con inestabilidad hemodinámica,
usted, diagnostica, estabiliza y refiere a un centro de segundo o tercer nivel, las
pruebas óptimas para realizar una evaluación rápida y definir la conducta definitiva son:
a. Resonancia magnética nuclear c. TAC simple y contrastada
b. Radiología simple y contrastada d. Lavado peritoneal diagnóstico y
Ecografía FAST
____ 51. Todos los siguientes son factores que incrementan el riesgo de cáncer colorectal,
EXCEPTO:
a. Dieta alta en fibra vegetal y ácido c. Antecedentes familiares
oleico
b. Dieta alta en grasas saturadas o d. El envejecimiento
poliinsaturadas

9
Name: ________________________ ID: A

____ 52. Todas las siguientes complicaciones de las hernias de la pared abdominal son
indicaciones de tratamiento quirúrgico urgente, EXCEPTO:
a. Hernia por deslizamiento con c. Hernia encarcelada con signos
signos de obstrucción intestinal obstrucción intestinal
b. Hernia con signos de d. Hernia encarcelada sin signos
estrangulación obstrucción intestinal
____ 53. De las siguientes combinaciones de síntomas y signos, identifique cuál se corresponde
mejor con el cuadro clínico del abdomen agudo de causa quirúrgica:
a. Dolor abdominal, fiebre y diarreas c. Dolor abdominal, náuseas y
acuosas vómitos
b. Dolor abdominal, fiebre y defensa d. Dolor abdominal, fiebre y disuria
involuntaria
____ 54. Durante un partido de fútbol, Jacinto sufre una lesión en su tobillo derecho por lo que
acude a emergencia. Además de presentar dolor en la región de los maléolos se
aprecia dolor a la palpación sobre la tibia posterior. Las radiografías no muestran lesión
ósea. El diagnóstico es esguince de tobillo. Para el tratamiento inicial de este paciente,
además de reposo, ¿cuáles serían las tres acciones más adecuadas?
1. Elevación del miembro.
2. Aplicación de calor local.
3. Aplicación de masaje.
4. Aplicación de hielo local.
5. Colocación de yeso.
6. Vendaje elástico.

a. 2, 3, 5 c. 3, 4, 6
b. 1,4,5 d. 1, 4, 6
____ 55. Se presenta paciente masculino de 35 años, fumador y bebedor habitual, con
antecedentes de úlcera duodenal asociada a una infección por Helicobacter pylori, para
lo cual ha recibido tratamiento médico. El paciente aqueja náuseas, mareos, debilidad,
sudoraciones, "erizamientos" y sed, después de haber tenido dos deposiciones
amplias, fétidas y muy oscuras, "como alquitrán". Al examen físico se encuentra
mucosas secas y pálidas, FC 112 latidos por minuto, TA 90/65 mmHg, y el abdomen no
aporta datos relevantes. Seleccione la secuencia correcta de acciones en el manejo de
este paciente después de la anamnesis y el examen físico:
1. Referir para endoscopia diagnostica-terapéutica.
2. Fluido-terapia con cristaloides, coloides y hemoderivados.
3. Lavado-aspirado nasogástrico + tacto rectal.
4. Colocar dos catéteres venosos periféricos gruesos.
5. Si fuese necesario luego de endoscopia, cirugía de emergencia.
6. Extraer muestras sanguíneas para hematología, química y tipificación.

a. 3-4-2-6-1-5 c. 4-6-2-3-1-5
b. 3-1-4-2-6-5 d. 6-3-4-2-1-5

10
Name: ________________________ ID: A

____ 56. Un paciente de 35 años, fumador y consumidor habitual de bebidas alcohólicas, se


presenta en emergencias con un dolor abdominal agudo e intenso, de comienzo brusco
en epigastrio y rápida irradiación a todo el abdomen y hombro derecho. Al examen
físico se comprueba sudoración, FC 112 latidos x minuto, marcada contractura
abdominal y timpanismo abdominal con pérdida de la matidez hepática ¿Cuál sería su
principal sospecha diagnóstica?
a. Apendicitis aguda c. Úlcera gastroduodenal perforada
b. Obstrucción mecánica intestinal d. Colecistitis aguda

____ 57. Se trata de una paciente femenina de 42 años con historia de trastornos dispépticos que,
tras un cuadro de ictericia transitoria que cursó con aumento de bilirrubina a predominio
de la conjugada, fosfatasa alcalina y transaminasas séricas, se le realizó una ecografía
que demostró una vesícula biliar de tamaño normal, sin imágenes de litiasis en su
interior, por lo que se indicó una colangiopancreatografía retrógrada endoscopios
(CPRE), la cual se muestra en la imagen. Indique cual sería el diagnóstico adecuado en
esta paciente:

a. Colecistitis aguda c. Colangitis ascendente


b. Colelitiasis sintomática d. Coledocolitiasis
____ 58. Todos los siguientes son factores que incrementan el riesgo de cáncer gástrico,
EXCEPTO:
a. Consumo de ácido acetil salicílico, c. Infección por Helicobacter pylori
vitamina C, frutas, verduras y
alcohol
b. Antecedentes familiares, poliposis d. Dieta rica en nitratos, sal, grasas,
familiar, cáncer colorectal consumo de tabaco
hereditario
____ 59. Complete el siguiente enunciado: Las tres complicaciones más comunes de la
enfermedad ulcero péptica en orden decreciente de frecuencia son:
a. Obstrucción, perforación y c. Sangrado, perforación y
sangrado obstrucción
b. Obstrucción, sangrado y d. Perforación, sangrado y
perforación obstrucción

11
Name: ________________________ ID: A

____ 60.
Complete el siguiente enunciado (espacios en blanco), con relación a la enfermedad
diverticular del colon:
El término _____________se refiere a la existencia de divertículos sin inflamación. El
término ______________alude a la inflamación e infección relacionadas con
divertículos. Casi todos los del colon son divertículos _____________en los que se
herniaron la mucosa y la muscular de la mucosa a través de la pared del colon. Al
parecer se tratan de divertículos por____________que resultan de una presión
intralurninal___________.
a. Diverticulitis - Diverticulosis - c. Diverticulosis - Diverticulitis -
Falsos - Pulsión - Disminuida Falsos - Pulsión - Elevada
b. Diverticulosis - d. Diverticulitis - Diverticulosis -
Diverticulitis-Verdaderos -Tracción Verdaderos - Tracción -
- Elevada Disminuida
____ 61. Paciente masculino de 34 años es valorado en el centro de trauma por sufrir accidente
automovilístico hace 5 minutos. Al examen físico presenta: frecuencia respiratoria
28/min, temperatura 36° C, pulso 110/min, tensión arterial 60/40 mmHg, palidez
generalizada, abdomen: blando, poco doloroso. No hay signos de focallzación. En
radiografía se observa mediastino ensanchado. Selaccions el diagnóstico probable y su
acción prioritaria considerando la distribución trimodal de la mortalidad por trauma.
a. Neumotorax a tensión que c. Neumotorax abierto y colocación
requiere toracotomía exploradora de tubo de tórax
b. Ruptura de grandes vasos que d. Contusión pulmonar que requiere
requiere exploración toracotomía exploradora
____ 62. Todas las afirmaciones sobre el cáncer de tiroides son correctas, EXCEPTO:
a. Ocurren alrededor de tres veces c. La exposición a radiaciones y las
más en los hombres mujeres que dietas bajas en yodo incrementan
en las mujeres el riesgo
b. La poliposis adenomatosa familiar d. Los antecedentes en familiares de
incrementa el riesgo de carcinoma primer orden incrementan en
papilar riesgo
____ 63. ¿Cuál sería el objetivo principal en la revisión primaria de un trauma de tórax precoz?
a. Corregir y prevenir el dolor c. Corregir y prevenir la acidosis
metabólica
b. Corregir y prevenir la hipoxia d. Corregir y prevenir la hemorragia
____ 64. Walter de 22 años, presenta desde hace 6 horas un malestar abdominal vago, seguido
por leve náusea, anorexia e indigestión. El dolor es persistente y continuo, pero no es
intenso, en ocasiones hay calambres abdominales leves, al momento se localiza en
cuadrante inferior derecho. Al examen físico presenta: FR 16/min, FC 80/min, TA
110/75 mmHg, T 38° C. Existe ligera hípersensibilidad en cuadrante inferior derecho del
abdomen a la palpación con un dedo e hipersensibilidad de rebote en el mismo sitio.
Además, presenta hipersensibilidad al esfuerzo de la tos. Al momento no cuenta con
resultados de laboratorio, ni exámenes de gabinete y el hospital más cercano se
encuentra a 6 horas en vehículo particular. Según el diagnóstico probable, ¿Qué
antibiótico utiliza para una profilaxis prequirúrgica?
a. Gentamicina c. Amikacina
b. Tetraciclina d. Cefazolina

12
Name: ________________________ ID: A

____ 65. Durante un partido de fútbol, Jacinto sufre una lesión en su tobillo derecho por lo que
acude a emergencia. Además de presentar dolor en la región de los maléolos se
aprecia dolor a la palpación sobre la tibia posterior. Las radiografías no muestran lesión
ósea. El diagnóstico es esguince de tobillo. Para el tratamiento inicial de este paciente,
además de reposo, ¿cuáles serían las tres acciones más adecuadas?
1. Elevación del miembro.
2. Aplicación de calor local.
3. Aplicación de masaje.
4. Aplicación de hielo local.
5. Colocación de yeso.
6. Vendaje elástico.
a. 2, 3, 5 c. 1, 4, 5
b. 3, 4, 6 d. 1, 4, 6

____ 66. Se trata de una paciente femenina de 42 años con historia de trastornos dispépticos que,
tras un cuadro de ictericia transitoria que cursó con aumento de bilirrubina a predominio
de la conjugada, fosfatasa alcalina y transaminasas séricas, se le realisó una ecografía
que demostró una vesícula biliar de tamaño normal, sin imágenes de litiasis en su
interior, por lo que se indicó una colangiopancreatografía retrógrada endoscópica
(CPRE), la cual se muestra en la imagen. Indique cual sería el diagnóstico adecuado en
esta paciente:

a. Colangitis ascendente c. Colecistitis aguda


b. Coledocolitlasis d. Colelitiasis sintomática

13
Name: ________________________ ID: A

____ 67. Todos los siguientes son factores que incrementan el riesgo de cáncer gástrico,
EXCEPTO:
a.Dieta rica en nitratos, sal, grasas, c. Consumo de ácido acetil salicílico,
consumo de tabaco vitamina C, frutas, verduras y
alcohol
b. Antecedentes familiares, poliposís d. Infección por Helicobacter pylori
familiar, cáncer colorectal
hereditario
____ 68. Paciente masculino de 34 años es valorado en el centro de trauma por sufrir accidente
automovilístico hace 5 minutos. Al examen físico presenta: frecuencia respiratoria
28/min, temperatura 36° C, pulso 110/min, tensión arterial 60/40 mmHg, palidez
generalizada, abdomen: blando, poco doloroso. No hay signos de focalización. En
radiografía se observa mediastino ensanchado. Seleccione el diagnóstico probable y su
acción prioritaria considerando la distribución trimodal de la mortalidad por trauma
a. Neumotorax abierto y colocación c. Ruptura de grandes vasos que
de tubo de tórax requiere exploración
b. Neumotorax a tensión que d. Contusión pulmonar que requiere
requiere toracotomía exploradora toracotomía exploradora
____ 69. Todas las afirmaciones sobre el cáncer de tiroides son correctas, EXCEPTO:
a. Los antecedentes en familiares de c. La exposición a radiaciones y las
primer orden Incrementan en dietas bajas en yodo incrementan
riesgo el riesgo
b. Ocurren alrededor de tres veces d. La políposis adenomatosa familiar
más en los hombres mujeres que incrementa el riesgo de carcinoma
en las mujeres papilar

14
Name: ________________________ ID: A

____ 70. Paciente femenina, obesa, que se presentó con dolor muy intenso, de comienzo súbito
después de una comida grasosa, acompañado de náuseas y vómitos, localizado en el
cuadrante superior derecho, con Irradiación a la región interescapular. La paciente
refiere que había tenido este dolor en otras ocasiones. El examen físico y las pruebas
de laboratorio fueron normales, por lo que se administró 75 mg de diclofenaco sódico
IM, logrando alivio del dolor. Se solicitó ecografía abdominal que se muestra en la
imagen. Seleccione el diagnóstico adecuado en esta paciente.

a. Colelitiasis sintomática c. Coledocolitiasis


b. Coledocolitiasis d. Colecistitis aguda
____ 71. Seleccione la prueba diagnóstica más importante para confirmar cáncer en un paciente
con nódulo solitario tiroideo:
a. Gammagrafía tiroidea c. Dosificación de T3, T4 y TSH
b. Ecografía tiroidea d. BAAF (Biopsia por aspiración con
aguja fina)
____ 72. En un paciente con traumatismo abdominal cerrado, con inestabilidad hemodinámica,
usted, diagnostica, estabiliza y refiere a un centro de segundo o tercer nivel, las
pruebas óptimas para realizar una evaluación rápida y definir la conducta definitiva son:
a. Lavado peritoneal diagnóstico y c. Resonancia magnética nuclear
Ecografía FAST
b. Radiología simple y contrastada d. TAC simple y contrastada

15
Name: ________________________ ID: A

____ 73. Todas las siguientes complicaciones de las hernias de la pared abdominal son
indicaciones de tratamiento quirúrgico urgente, EXCEPTO:
a. Hernia encarcelada sin signos c. Hernia encarcelada con signos
obstrucción intestinal obstrucción intestinal
b. Hernia por deslizamiento con d. Hernia con signos de
signos de obstrucción intestinal estrangulación
____ 74. Para confirmar el diagnóstico de abdomen agudo, ¿cuál de las siguientes pruebas de
laboratorio se debe solicitar?
a. Velocidad eritorsedimentación c. Recuento de glóbulos blancos y
fórmula diferencial
b. Procalcitonina d. Proteína C Reactiva
____ 75. Complete el siguiente enunciado: Las tres complicaciones más comunes de la
enfermedad ulcero-péptica en orden decreciente de frecuencia son
a. Obstrucción, perforación y c. Perforación, sangrado y
sangrado obstrucción
b. Obstrucción, sangrado y d. Sangrado, perforación y
perforación obstrucción
____ 76. Seleccione la prueba más adecuada para confirmar el diagnóstico positivo de cáncer
gástrico:
a. Tomografía axial computarizada c. Endoscopia gastrointestinal
abdominal simple y contrastada superior con biopsia
b. Estudio radiológico baritado d. Ecografía gástrica endoscópica
gastrointestinal superior con doble
contraste
____ 77. ¿Cuál sería el objetivo principal en la revisión primaria de un trauma de tórax precoz?
a. Corregir y prevenir la hipoxia c. Corregir y prevenir el dolor
b. Corregir y prevenir la acídosis d. Corregir y prevenir la hemorragia
metabólica
____ 78. Todos los siguientes son factores que incrementan el riesgo de cáncer colorectal,
EXCEPTO:
a. Dieta alta en fibra vegetal y ácido c. Antecedentes familiares
oleico
b. El envejecimiento d. Dieta alta en grasas saturadas o
poliinsaturadas
____ 79. Un paciente de 35 años, fumador y consumidor habitual de bebidas alcohólicas, se
presenta en emergencias con un dolor abdominal agudo e Intenso, de comienzo brusco
en epigastrio y rápida irradiación a todo el abdomen y hombro derecho. Al examen
físico se comprueba sudoración, FC 112 latidos x minuto, marcada contractura
abdominal y timpanismo abdominal con pérdida de la matidez hepática ¿Cuál sería su
principal sospecha diagnóstica?
a. Obstrucción mecánica intestinal c. Úlcera gastroduodenal perforada
b. Apendicitis aguda d. Colecistitis aguda

16
Name: ________________________ ID: A

____ 80. Paciente de 35 años, con antecedentes de haber sido intervenido quirúrgicamente por
una peritonitis secundaria a apendicitis aguda durante su infancia, se presenta con un
cuadro clínico de dolor abdominal en forma de cólico intenso que alivia con vómitos
biliosos y de contenido intestinal oscuro y fétido. Refiere tener sed y que hace más de
24 horas no defeca, ni expulsa gases por el recto. Al examen físico se comprueban las
mucosas y lengua secas, distensión abdominal asimétrica, con una cicatriz quirúrgica
media infraumbilical y un patrón acelerado de los ruidos hidroaéreos a la auscultación.
Se solicitó un Rx de abdomen simple en bipedestación el cuaO se muestra en la
Imagen. ¿Cuál considera usted es el diagnóstico de este paciente?

a. Oclusión mecánica Intestinal c. Úlcera gastroduodenal perforada


b. Gastroenterocolitis aguda d. Ileo paralítico

17
Name: ________________________ ID: A

____ 81.
Complete el siguiente enunciado (espacios en blanco), con relación a la enfermedad
diverticular del colon:
El término _____________se refiere a la existencia de divertículos sin inflamación. El
término ______________alude a la inflamación e infección relacionadas con
divertículos. Casi todos los del colon son divertículos _____________en los que se
herniaron la mucosa y la muscular de la mucosa a través de la pared del colon. Al
parecer se tratan de divertículos por____________que resultan de una presión
intralurninal___________.
a. Diverticulitis - Diverticulosis - c. Diverticulosis - Diverticulitis -
Falsos - Pulsión - Disminuida Falsos - Pulsión - Elevada
b. Diverticulosis - d. Diverticulitls - Diverticulosis -
Diverticulitls-Verdaderos - Verdaderos - Tracción -
Tracción - Elevada Disminuida
____ 82. Se presenta un paciente masculino de 35 años, fumador y bebedor habitual, con
antecedentes de úlcera duodenal asociada a una infección por Helicobacter pylori, para
lo cual lleva tratamiento médico. El paciente aqueja intenso dolor abdominal de
comienzo brusco "como puñalada en la boca del estómago" una hora antes, el cual se
fue haciendo difuso y más intenso, con irradiación al hombro izquierdo. Al examen
físico se constata ansiedad, sudoración, taquicardia y un abdomen muy contracturado y
doloroso al palpar y percutir, con borramiento de la matidez hepática. Identifique cual
sería la complicación de la enfermedad ulcerosa péptica que presenta el paciente:
a. Peritonitis por perforación c. Obstrucción de la salida gástrica
b. Malignización de úlcera benigna d. Sangrado digestivo alto

____ 83. De las siguientes combinaciones de síntomas y signos, identifique cuál se corresponde
mejor con el cuadro clínico del abdomen agudo de causa quirúrgica:
a. Dolor abdominal, fiebre y diarreas c. Dolor abdominal, náuseas y
acuosas vómitos
b. Dolor abdominal, fiebre y defensa d. Dolor abdominal, fiebre y disuria
involuntaria
____ 84. Se presenta paciente masculino de 38 años, fumador y bebedor habitual, con
antecedentes de úlcera duodenal asociada a una infección por Helicobacter pylori, para
lo cual ha recibido tratamiento médico. El paciente aqueja náuseas, mareos, debilidad,
sudoraciones, "erizsmientos" y sed, después de haber tenido dos deposiciones
amplias, fétidas y muy oscuras, "como alquitrán". Al examen físico se encuentra
mucosas secas y pálidas, FC 112 latidos por minuto, TA 90/65 mmHg, y el abdomen no
aporta datos relevantes. Seleccione la secuencia correcta de acciones en el manejo de
este paciente después de la anamnesis y el examen físico:
1. Referir para endoscopla diagnóstica-terapéutlca.
2. Fluido-terapia con cristaloides, coloides y hemoderivados.
3. Lavado-aspirado nasogástrico + tacto rectal.
4. Colocar dos catéteres venosos periféricos gruesos.
5. SI fuese necesario luego de endoscopia, cirugía de emergencia.
Extraer muestras sanguíneas para hematología, química y tipificación.
a. 3-1-4-2-6-5 c. 6-3-4-2-1-5
b. 4-6-2-3-1-5 d. 3-4-2-6-1-5

18
Name: ________________________ ID: A

____ 85. Paciente femenina de 44 años, obesa, que se presentó con dolor abdominal muy
intenso, de comienzo súbito después de una comida grasosa, acompañado de náuseas
y vómitos, localizado en el cuadrante superior derecho, con irradiación a la región
interescapular. La paciente refiere que había tenido este dolor en otras ocasiones, pero
que este es más intenso y no se alivia a pesar de que le han administrado analgésicos
en las últimas seis horas. Al examen del abdomen hay hipersensibilidad y resistencia
en el cuadrante superior derecho y a la palpación profunda del hipocondrio derecho se
constata una masa redondeada que al comprimir profundamente produce intenso dolor
que detiene la inspiración de la paciente. Se solicitó un conteo de células blancas que
arrojó 11800 leücocitos/mm3 y una ecografía abdominal que se muestra en la imagen.
Seleccione el diagnóstico adecuado en esta paciente:

a. Colelitiasis sintomática c. Coledocolitiasis


b. Colecistitis aguda d. Colangitis ascendente
____ 86. Walter de 22 años, presenta desde hace 6 horas un malestar abdominal vago, seguido
por leve náusea, anorexia e indigestión. El dolor es persistente y continuo, pero no es
intenso, en ocasiones hay calambres abdominales leves, al momento se localiza en
cuadrante inferior derecho. Al examen físico presenta: FR 16/min, FC 80/min, TA
110/7S mmHg, T 38° C. Existe ligera hipersensibilidad en cuadrante inferior derecho
del abdomen a la palpación con un dedo e hipersensibilidad de rebote en el mismo
sitio. Además presenta hipersensibilidad al esfuerzo de la tos. Al momento no cuenta
con resultados de laboratorio, ni exámenes de gabinete y el hospital más cercano se
encuentra a 6 horas en vehículo particular. Según el diagnóstico probable, ¿qué
antibiótico utiliza para una profilaxis prequirúrgica?
a. Gentamicina c. Cefazolina
b. Tetraciclina d. Amikacina

19
Name: ________________________ ID: A

____ 87. Llega un paciente con una herida penetrante por arma de fuego en el flanco derecho,
se acompaña de hipotensión y es evidente una evisceración intestinal. ¿Cuál es el
manejo apropiado en este paciente?
a. Laparotomía exploratoria c. Tomografía axial computarizada
abdominal
b. Ecografía Focalizada en Trauma d. Cierre de herida
(FAST)
____ 88. ¿Cuál es la indicación para referir a un paciente a un centro especializado para la
atención de quemados?
a. Quemaduras de espesor parcial c. Quemadura de segundo grado en
con un compromiso de 5% TBSA cualquier edad
b. Quemaduras en pacientes sin d. Quemaduras químicas, de
comorbilidades cualquier mecanismo de acción
____ 89. Paciente masculino de 60 años con antecedentes de estreñimiento y obesidad acude por
cuadro de dolor abdominal en el cuadrante inferior izquierdo que se acompaña de
astenia y alza térmica. Usted solicita una Nomografía de abdomen en la que se
evidencia una diverticulitis del colon sigmoideo asociado a un absceso menor a 2 cm.
El tratamiento es:
a. Drenaje percutáneo c. Laparotomía
b. Antibioticoterapia d. Laparoscopia y drenaje de
absceso
____ 90. Seleccione los parámetros radiográficos evaluados para diagnosticar displasia del
desarrollo ele cadera.
1. Acetabular.
2. Barlow.
3. Perkins.
4. Hilgenreiner.
5. Shenton.
6. Ortolarti.
a. 1, 3, 4, 5 c. 1, 2, 4, 5
b. 2, 3, 5, 6 d. 1, 3, 5, 6
____ 91. La regla de los 9 puede utilizarse como una referencia rápida para calcular en un
paciente quemado, el área de superficie corporal comprometida. ¿Cuál es el porcentaje
comprometido en un paciente con una quemadura de todo el tronco anterior y el
miembro superior derecho?
a. 27% c. 36%
b. 18% d. 19%

20
Name: ________________________ ID: A

____ 92. Señale la afirmación correcta relacionada a los signos y síntomas de la apendicitis
aguda:
a. La falta de apetito temporal es un c. El signo de Rovsing es el dolor
síntoma poco frecuente en que se presenta en el cuadrante
cuadros apendiculares inferior derecho al ejercer presión
en el cuadrante inferior izquierdo
b. El dolor durante el examen rectal d. El signo de Blumberg es positivo
es característico en los apéndices cuando hay dolor en fosa ilíaca
inflamados de localización izquierda al comprimir fosa ilíaca
paracecal ascendente derecha
____ 93. La herida por una colecistecíomía laparoscópica sin complicaciones se considera:
a. Limpia c. Contaminada
b. Sucia d. Limpia contaminada

____ 94. De las siguientes opciones. ¿Cuál es una complicación metafoólica de la pancreatitis
aguda?
a. Hipercalcemia c. Hiponatremia
b. Hipotrigliceridemia d. Hiperglucemia
____ 95. En revisión secundaria, indique las Sesiones potencialmente fatales en trauma de tórax
1. Hemotórax masivo.
2. Neumotorax simple.
3. Lesión cardiaca cerrada.
4. Taponamiento cardiaco.
5. Lesión cerrada de esófago.
a. 1,2,3 c. 2,3,4
b. 1,3,5 d. 2,3,5
____ 96. Paciente varón de 65 años, bebedor habitual, que se presenta con intenso dolor
epigástrico después de una comida abundante, describe al dolor como “puñalada”, con
irradiación a la espalda y que se alivia cuando se inclina hacia adelante. Refiere
náusea y vómito abundantes. Al examen físico presenta FC: 112 latidos por minuto, TA:
90/60 mmHg, temperatura: 38,8 grados C; el abdomen se observa distendido, con
defensa involuntaria y dolor al palpar todo el hemiabdomen superior, los ruidos
hidroaéreos están disminuidos. En los exámenes de laboratorio se encuentra
hemoconcentración, hiperglucemia, hiperazoemia e hiperamilasemia. Se realizó
ecografía que no dio información útil por la presencia de abundantes gases intestinales.
Se solicitó TAC que se muestra en la imagen. ¿Cuál es el diagnóstico a plantear en
este paciente?
a. Pancreatitis aguda edematosa c. Apendicitis aguda complicada
leve
b. Úlcera péptica gastroduodenal d. Pancreatitis aguda necrosante
perforada grave
____ 97. De los siguientes criterios. ¿Cuál está asociado con la presencia de una pancreatitis
aguda grave?
a. Escala de APACHE II mayor a 8 c. Escala de RAMSON de 1
b. Escala de BISAP de 1 d. Presencia de 1 criterio de Ranson

21
Name: ________________________ ID: A

____ 98. Las siguientes son indicaciones de laparotomía en (pacientes con lesión penetrante de
abdomen. EXCEPTO:
a. Signos de irritación peritoneal c. Signos de penetración de la fascia
b. Alteración hemodinámica d. Distención abdominal
____ 99. Seleccione la respuesta correcta sobre profilaxis antibiótica:
a. La profilaxis antibiótica c. La profilaxis antibiótica se utiliza
administrada correctamente sobre todo para tratar la infección
previene infecciones del sitio quirúrgico
nosocomiales
b. La profilaxis antibiótica está d. La profilaxis antibiótica
indicada para la mayoría de las administrada incorrectamente es
cirugías sucias ineficaz e incluso perjudicial
____ 100. Síntomas y signos de neumotorax a tensión. EXCEPTO:
a. Desviación traqueal c. Ruidos cardiacos apagados
b. Dificultad respiratoria d. Dolor torácico

____ 101. La colangitis aguda se caracteriza por una triada clásica (Triada de Charcot). Indique sus
componentes:
a. Ictericia.
b. Alteración mental.
c. Dolor.
d. Fiebre.
e. Choque séptico.
a. a, c, e c. c, d, e
b. b, c, d d. a, c, d
____ 102. ¿Cuál de los siguientes signos Geográficos corresponde a colecistitis aguda?
a. Presencia de litos al interior de la c. Pared vesicular de 2 mm de
vesícula diámetro
b. Murphy ecográfico d. Vesícula biliar con un volumen de
30 ml
____ 103. Indique. ¿Cuál es el componente de la solución salina?
a. Solución salina 0.9% contiene 109 c. Solución salina al 0.9% contiene:
mEq/L de cloro 4 mEq/L de sodio
b. Solución salina 0.9% tiene un pH d. Solución salina al 0.9% contiene:
de 8 154 mEq/L de sodio
____ 104. Indique la afirmación correcta en relación con el manejo de conjuntivitis.
a. El uso de corticoides puede c. La conjuntivitis bacteriana leve
empeorar una infección por debe ser tratada con antibióticos
herpes virus pues no se autolimita
b. La conjuntivitis por molusco d. La conjuntivitis viral responde a
contagioso, responde los agentes antibacterianos
adecuadamente a antivirales

22
Name: ________________________ ID: A

____ 105. Paciente que presenta un accidente de tránsito, es llevado a la sala de emergencia con
disnea, dolor torácico, taquicardia y ausencia de ruidos respiratorios en hemitórax
derecho, usted sospecha de un neumotorax a tensión. ¿Cuál es el sitio en el que se
debe colocar la aguja de descompresión en esta patología?
a. Sexto espacio intercostal, línea c. Cuarto espacio intercostal, línea
axilar anterior del hemitórax medio clavicular del hemitórax
afectado afectado
b. Segundo espacio intercostal, línea d. Segundo espacio intercostal, línea
medio clavicular del hemitórax axilar anterior del hemitórax
afectado afectado
____ 106. ¿Cómo se denomina la maniobra de valoración de displasia del desarrollo de cadera, en
la cual la cabeza femoral dislocada se reduce al acetábulo?
a. Galeazzi c. Ortolani
b. Barlow d. Allis

____ 107. En la clasificación TNM para cáncer de próstata un estadio T3b se refiere a:
a. El tumor es un hallazgo c. El tumor afecta a más de la mitad
histológico fortuito en más del 5% de un lóbulo, pero no a los dos
del tejido resecado lóbulos
b. Extensión extracapsular incluida d. El tumor invade una o ambas
la afectación microscópica del vesículas seminales
cuello de la vejiga
____ 108. Las siguientes son anomalías de pared abdominal, EXCEPTO:
a. Onfalocele c. Hernia umbilical
b. Hematoma de la vaina del recto d. Quiste epiploico

____ 109. Indique, ¿Cuándo un lavado peritoneal diagnóstico (LPD) es positivo?


a. Más de 100.000 eritrocitos por c. Extracción de 5 cc de sangre
mm3 inmediatamente a la punción
b. Retorno mayor del 30% del líquido d. Un lavado negativo excluye
infundido en la cavidad abdominal cualquier tipo de lesión abdominal
____ 110. Señale el procedimiento representativo que se relaciona con cirugía sucia (clase IV).
a. Diverticulitis perforada c. Enterotomía durante la
obstrucción intestinal
b. Traumatismo abdominal d. Reparación de hernia
penetrante
____ 111. En pacientes con trauma, en la valoración primaria, en la sección de circulación.
Identifique la primera lesión más importante que pone en riesgo la vida:
a. Fracturas pélvicas c. Taponamiento cardíaco
b. Hemotórax masivo d. Hemoperitoneo masivo

____ 112. Las siguientes son características de la triada de Beck, EXCEPTO:


a. Elevación de la presión venosa c. Disminución de la presión arterial
b. Hiperresonancia a la percusión d. Ruidos cardiacos apagados

23
Name: ________________________ ID: A

____ 113. Niño de 8 años, circuncidado, con antecedente de testículo en ascensor, refiere dolor
súbito a nivel del testículo izquierdo con aumento de volumen en esa misma área. El
examen físico muestra asimetría escrotal y reflejo cremasteriano lento. ¿Cuál es la
sospecha diagnóstica y qué prueba la confirma?
a. Orquiepididimitís - Ecografía c. Parafimosis - Ecografía simple
simple
b. Priapismo -Tomografía d. Torsión testicular - Ecografía
computarizada doppler
____ 114. Identifique cuál de los siguientes signos clínicos nos permite realizar una diferenciación
entre una conjuntivitis gonocócica y no gonocócica:
a. Inyección conjuntival c. Ojo rojo
b. Presencia de úlcera corneal d. Secreción purulenta o
mucopurulenta
____ 115. Son criterios de CENTOR para diagnóstico de amigdalitis, EXCEPTO:
a. Fiebre c. Presencia de tos
b. Exudado amigdalino d. Adenopatías cervicales anteriores
dolorosas
____ 116. La siguiente es causa benigna de elevación del PSA (Antígeno prostático Especifico),
EXCEPTO:
a.Retención urinaria c. Trauma perineal
b.Epididimitls d. Prostatitis aguda
____ 117. Señale que examen determina una extensión extracapsular (local) de un cáncer de
próstata:
a. Uretrocistoscopia c. Ecografía transrectal
b. Gammagrafía ósea d. Biopsia de próstata
____ 118. Paciente adulto con fiebre, dolor por primera ocasión, sensibilidad dolorosa a la
palpación y defensa muscular en el cuadrante superior derecho. En los exámenes
realizados se encuentran leucocitosis y ligeras elevaciones de la fosfatasa alcalina,
bilirrubinas y transaminasas. Señale el diagnóstico:
a. Colecistitis crónica c. Colangitis aguda
b. Coledocolitiasis d. Colecistitis aguda

____ 119. Son síntomas y signos de Rinosinusitis aguda, EXCEPTO:


a. Dolor facial c. Epistaxis recurrente
b. Descarga nasal purulenta d. Hipersensibilidad senos
paranasales

24
Name: ________________________ ID: A

____ 120. Una mujer de 42 años fue expulsada de su vehículo durante una colisión vehicular.
Camino al departamento de emergencia personal paramédico reporta: frecuencia
cardiaca de 130/min, presión arterial de 90/45 mmHg y frecuencia respiratoria de
34/min. Presenta una herida en extremidad inferior derecha a la que se realiza presión
directa para controlar el sangrado. Al momento la paciente se encuentra ansiosa y
confusa. Seleccione el grado de hemorragia según pérdida sanguínea estimada y elija
el tipo de restitución de líquidos.
a. Hemorragia grado III, necesita c. Hemorragia grado IV, necesita
cristaloides y sangre cristaloides y sangre
b. Hemorragia grado II, necesita d. Hemorragia grado III, necesita
cristaloides cristaloides
____ 121. Causa de Abdomen agudo no quirúrgico, EXCEPTO:
a. Peritonitis primaria c. Plastrón Apendicular con SIRS
b. Crisis Cetodiabéticas d. Pancreatitis aguda

____ 122. Señale el antibiótico de primera elección usado en la profilaxis de la Colecistectomía


a. Cefuroxima c. Ampicilina sulbactam
b. Cefazolina d. Cefazolina más metronidazol

____ 123. ¿Cuál es el manejo inicial de un varón con retención aguda de orina causada por
hematuria con coágulos?
a. Realizar cistoscopia con dilatador c. Colocar un catéter de tres vías
uretral
b. Colocar una sonda vesical Foley d. Colocar una sonda suprapúbica
____ 124. Señale un signo que represente déficit agudo de volumen extracelular:
a. Aumento de peso c. Hiperazoemia
b. Soplo d. Edema intestinal

____ 125. Señale la indicación de referencia de un paciente quemado a un centro especializado:


a. Quemadura de glúteo c. Quemadura de segundo grado
b. Quemadura de tercer grado d. Quemadura de espesor parcial
del 5 % de TBSA
____ 126. Paciente masculino de 33 años con trabajo extenuante que le ocasiona estrés, con
antecedentes de enfermedad ulcerosa péptica, presenta vómitos de contenido no
bilioso y alcalosis metabólica intensa con hipopotasemia. Indique a que complicación
de la enfermedad ulcerosa péptica se refiere el enunciado.
a. Úlcera péptica perforada c. Obstrucción pilórica
b. Síndrome de Zollinger - Ellison d. Gastritis crónica

____ 127. Identifique la etapa de choque hemorrágico en la que se encuentra un paciente


traumatizado, que al examen físico presenta una FC: 125 Ipm, presión arterial
90/60mmHg, FR: 35 rpm, diuresis: 10cc/H, en la evaluación del sistema neurológico
presenta confusión, con una pérdida hemática de 1500 a 2000 cc de sangre.
a. Clase I c. Clase IV
b. Clase II d. Clase III

____ 128. Señale un factor de riesgo de infecciones de sitio quirúrgico que depende del paciente.
a. Necrosis del tejido c. Operación reciente
b. Profilaxis inadecuada d. Hipotermia, hipoxia

25
ID: A

Cirugía
Answer Section

MULTIPLE CHOICE

1. ANS: B PTS: 1
2. ANS: A PTS: 1
3. ANS: A PTS: 1
4. ANS: C PTS: 1
5. ANS: B PTS: 1
6. ANS: B PTS: 1
7. ANS: D PTS: 1
8. ANS: A PTS: 1
9. ANS: A PTS: 1
10. ANS: D PTS: 1
11. ANS: D PTS: 1
12. ANS: A PTS: 1
13. ANS: D PTS: 1
14. ANS: D PTS: 1
15. ANS: B PTS: 1
16. ANS: B PTS: 1
17. ANS: A PTS: 1
18. ANS: A PTS: 1
19. ANS: B PTS: 1
20. ANS: B PTS: 1
21. ANS: A PTS: 1
22. ANS: A PTS: 1
23. ANS: B PTS: 1
24. ANS: C PTS: 1
25. ANS: D PTS: 1
26. ANS: B PTS: 1
27. ANS: A PTS: 1
28. ANS: C PTS: 1
29. ANS: A PTS: 1
30. ANS: B PTS: 1
31. ANS: A PTS: 1
32. ANS: C PTS: 1
33. ANS: B PTS: 1
34. ANS: C PTS: 1
35. ANS: B PTS: 1
36. ANS: C PTS: 1
37. ANS: D PTS: 1
38. ANS: B PTS: 1
39. ANS: B PTS: 1
40. ANS: C PTS: 1
41. ANS: B PTS: 1
42. ANS: C PTS: 1
43. ANS: B PTS: 1

1
ID: A

44. ANS: A PTS: 1


45. ANS: B PTS: 1
46. ANS: B PTS: 1
47. ANS: A PTS: 1
48. ANS: D PTS: 1
49. ANS: D PTS: 1
50. ANS: D PTS: 1
51. ANS: A PTS: 1
52. ANS: D PTS: 1
53. ANS: B PTS: 1
54. ANS: D PTS: 1
55. ANS: C PTS: 1
56. ANS: C PTS: 1
57. ANS: D PTS: 1
58. ANS: A PTS: 1
59. ANS: C PTS: 1
60. ANS: C PTS: 1
61. ANS: B PTS: 1
62. ANS: A PTS: 1
63. ANS: B PTS: 1
64. ANS: D PTS: 1
65. ANS: D PTS: 1
66. ANS: D PTS: 1
67. ANS: C PTS: 1
68. ANS: C PTS: 1
69. ANS: B PTS: 1
70. ANS: A PTS: 1
71. ANS: D PTS: 1
72. ANS: A PTS: 1
73. ANS: A PTS: 1
74. ANS: C PTS: 1
75. ANS: D PTS: 1
76. ANS: C PTS: 1
77. ANS: A PTS: 1
78. ANS: A PTS: 1
79. ANS: C PTS: 1
80. ANS: A PTS: 1
81. ANS: C PTS: 1
82. ANS: A PTS: 1
83. ANS: B PTS: 1
84. ANS: B PTS: 1
85. ANS: B PTS: 1
86. ANS: C PTS: 1
87. ANS: A PTS: 1
88. ANS: D PTS: 1
89. ANS: B PTS: 1
90. ANS: A PTS: 1
91. ANS: A PTS: 1

2
ID: A

92. ANS: C PTS: 1


93. ANS: D PTS: 1
94. ANS: D PTS: 1
95. ANS: D PTS: 1
96. ANS: D PTS: 1
97. ANS: A PTS: 1
98. ANS: D PTS: 1
99. ANS: D PTS: 1
100. ANS: C PTS: 1
101. ANS: D PTS: 1
102. ANS: B PTS: 1
103. ANS: D PTS: 1
104. ANS: A PTS: 1
105. ANS: B PTS: 1
106. ANS: C PTS: 1
107. ANS: D PTS: 1
108. ANS: D PTS: 1
109. ANS: A PTS: 1
110. ANS: A PTS: 1
111. ANS: B PTS: 1
112. ANS: B PTS: 1
113. ANS: D PTS: 1
114. ANS: B PTS: 1
115. ANS: C PTS: 1
116. ANS: B PTS: 1
117. ANS: C PTS: 1
118. ANS: D PTS: 1
119. ANS: C PTS: 1
120. ANS: A PTS: 1
121. ANS: C PTS: 1
122. ANS: C PTS: 1
123. ANS: C PTS: 1
124. ANS: C PTS: 1
125. ANS: B PTS: 1
126. ANS: C PTS: 1
127. ANS: D PTS: 1
128. ANS: C PTS: 1

3
Cirugía [Answer Strip] ID: A

A
_____ 8. B 16.
_____ C 24.
_____ C 32.
_____

A 17.
_____
A
_____ 9.
B
_____ 1.
D 25.
_____ B 33.
_____

A 18.
_____

A
_____ 2. D 10.
_____
B 26.
_____ C 34.
_____

B 19.
_____
A
_____ 3.
B 35.
_____

D 11.
_____

C
_____ 4. A 27.
_____

C 36.
_____
B 20.
_____
A 12.
_____
B
_____ 5.

B
_____ 6. D 13.
_____
C 28.
_____ D 37.
_____

A 21.
_____

D 14.
_____
A 29.
_____ B 38.
_____

A 22.
_____

D
_____ 7. B 30.
_____

B 15.
_____ B 23.
_____
A 31.
_____
Cirugía [Answer Strip] ID: A

B 39.
_____ B 45.
_____ A 47.
_____ D 49.
_____ D 52.
_____

B 53.
_____
D 48.
_____

C 40.
_____

D 54.
_____

B 41.
_____

C 42.
_____

C 55.
_____
B 43.
_____

D 50.
_____

A 44.
_____

B 46.
_____
A 51.
_____
Cirugía [Answer Strip] ID: A

C 56.
_____ C 60.
_____ D 65.
_____ C 67.
_____ A 70.
_____

D 57.
_____ C 68.
_____

D 66.
_____
B 61.
_____

B 69.
_____

A 62.
_____

D 71.
_____
B 63.
_____

A 58.
_____

D 64.
_____
A 72.
_____

C 59.
_____
Cirugía [Answer Strip] ID: A

A 73.
_____ A 80.
_____ C 81.
_____ B 85.
_____ A 87.
_____

C 74.
_____
D 88.
_____

D 75.
_____

A 82.
_____ B 89.
_____

C 76.
_____

A 90.
_____

B 83.
_____
A 77.
_____

C 86.
_____
A 78.
_____
B 84.
_____

A 91.
_____

C 79.
_____
Cirugía [Answer Strip] ID: A

C 92.
_____ D 98.
_____ B
_____105. D
_____113. A
_____120.

D 99.
_____

B
_____114.

D 93.
_____ C
_____106.
C
_____121.

C
_____100.
D 94.
_____ C
_____115.
D
_____107. C
_____122.

D
_____101.

D 95.
_____ B
_____116. C
_____123.

D
_____108.
C
_____117.
C
_____124.

B
_____102. A
_____109.
D 96.
_____
D
_____118. B
_____125.

D
_____103. A
_____110.
C
_____126.

C
_____119.

A
_____104. B
_____111.

D
_____127.

B
_____112.
A 97.
_____

C
_____128.
Name: ________________________ Class: ___________________ Date: __________ ID: A

COMP CIRUGIA

Multiple Choice
Identify the choice that best completes the statement or answers the question.

____ 1. 1.Paciente de 30 años con cuadro abdominal agudo, con antecedentes personales de asma desde hace 2
años, y va a ser sometida a una intervención quirúrgica, para realizarle una apendicectomia de
emergencia, cual de las siguientes drogas inductoras anestésicas usted usaría para este caso
a. Propofol c. Tiopental sódico
b. Midazolam d. Ketamina

____ 2. Paciente Femenina de 68 años, con diagnóstico de Artrosis de cadera derecha, que está programada para
una intervención quirúrgica de Artroplastia Total de Cadera, y que se encuentra en tratamiento clínico
con varias drogas, cual de la lista deberá ser suspendida antes de su procedimiento quirúrgico?
a. Ketorolaco c. Aspirina
b. Metamizol d. Losartán

____ 3. Paciente que en el momento de realizar una laringoscopía para proceder a una intubación endotraqueal, se
producen dos eventos hemodinámicos importantes, elija cuáles son:
a. Bradicardia, hipotensión c. Bradicardia, hipertensión
b. Taquicardia, hipotensión d. taquicardia, hipertensión

____ 4. Paciente masculino de 45 años, que al momento de la intubación endotraqueal, se presenta una alteración
hemodinámica de Taquicardia e Hipertensión arterial y que para atenuar este suceso, debemos
administrar previamente las siguientes drogas anestésicas.
a. Clonidina+fentanil c. Lidocaina+fentanil
b. Losartan+lidocaina d. Lidocaina+clonidina

____ 5. Escriba correctamente la dosis ideal de la Atropina que se usa para una administración previa a una técnica
anestésica regional, previo a un procedimiento quirúrgico de una Artroscopía
a. 0,01-0,02 mg/kg. c. 0,01-0,04 mg/kg.
b. 0,01-0,03 mg/kg. d. 0,01-0,05 mg/kg.

____ 6. Paciente masculino de 68 años de edad, que está programado para una cirugía de hernia hiatal importante,
va a ser administrado una técnica anestésica general y de los gases anestésicos usados para dicho
procedimiento, cuál es el más dañino para el Paciente.
a. Sevorane c. Oxido nitroso
b. Halotane d. Oxígeno

____ 7. Paciente que va a ser sometido a un procedimiento quirúrgico de una Drenaje de hematoma subdural, está
planificado para recibir una técnica anestésica General, y para la misma, necesitamos administrar un
relajante muscular, para tener una buena estabilidad hemodinámica del paciente, cuál cree Usted que es el
relajante ideal para este acto quirúrgico.
a. Norcuron c. Quelicin
b. Cisatracurio d. Esmeron

____ 8. De las siguientes drogas utilizadas para obtener una excelente relajación muscular en un paciente que va a
ser sometido a una intervención quirúrgica de Laparatomía exploratoria, idique Usted cuál es la dosis
ideal del relajante muscular Bromuro de Rocuronio.
a. 0,3-0,6 mg/kg c. 0,4-0,7 mg/kg
b. 0,4-0,6 mg/kg d. 0,2-0,5 mg/kg

1
Name: ________________________ ID: A

____ 9. De la lista de las siguientes drogas de inducción anestésica, para administrar una técnica de anestesia
general, indique cuál es la más idonea e inofensiva, para un Paciente masculino de 2 años de edad, con
diagnóstico de Hidrocefalia.
a. Tiopental sódico c. Midazolam
b. Propofol d. Ketamina

____ 10. El Diprivan o propofol actualmente es un excelente inductor anestésico, sobre todo convinado con fentanil,
es una asociación fantástica, especialmente para realizar procedimientos de corta duración, como son los
Legrados uterinos, RMN, TC, Litotrixias, etc.¿cuál es la dosis ideal de este medicamente?
a. 1-3 mg/kg c. 1-5 mg/kg
b. 1-4 mg/kg d. 1-2 mg/kg

____ 11. El inductor anestésico, siendo uno de los más antiguos y utilizados en el mundo entero, no puede ser
utilizado en todas las técnicas anestésicas posibles, ya que tiene sus indicaciones exclusivas para cierto
tipo de pacientes que van a ser intervenidos quirúrgicamente, señale cuál de la siguiente lista de personas
que adolecen de enfermedades sobreañadidas, se lo puede administrar.
a. Diabéticos c. Cardiópatas
b. Tubercolosos d. Hipertensos

____ 12. Siendo la Lidocaina simple un excelente anestésico y analgésico local, tiene que ser administrado con
mucha delicadeza, razón por la cual debemos conocer la dosis exacta y correcta de la misma, indique
cuál es la dosis correcta?
a. 1-1,5 mg/kg c. 1-1,8 mg/kg
b. 1-2 mg/kg d. 1-3 mg/kg

____ 13. paciente masculino de 35 años, con diagnóstico de artrosis de rodilla derecha, va ser sometido a una
artroscopía, está planificado para administrar una técnica anestésica de bloqueo regional, indique de la
siguiente lista, cual es la única y absoluta contraindicación para no realizar esta técnica?
a. Tuberculosis c. Negativa del paciente
b. Infección d. Taquicardia

____ 14. De la siguiente lista de medicamentos o drogas anestésicas, que son prescritas en la Visita Preanestésica, 8
horas previa a la intervención quirúrgica, cuál es la más importante o más utilizada.
a. Midazolam c. Ketamina
b. Tiopental sódico d. Propofol

____ 15. De las siguientes patologías que pondremos a consideración, cuál es la más connotada que nos va a
presentar inconvenientes para poder realizar una correcta intubación endotraqueal o como comunmente
se la conoce como intubación difícil.
a. Cuello corto c. Macroglosia
b. Incisivos prominentes d. Limitación articulación tempomaxilar

____ 16. De la lista de los siguientes gases anestésicos medicinales, usados la práctica de la anestesiología, señale
cuales son los gases más usados y más importantes:
a. CO2, O2, N2O c. CO2. N2, O2
b. O2, N2O, AIRE d. CO2, AIRE, N2

____ 17. En un paciente que va a ser sometido a una intervención quirúrgica por una colecistectomía convencional y
en el momento que el Anestesiólogo va a realizar la laringoscopía, se presenta una complicación llamada
Laringospasmo, cuáles de las siguientes tratamientos es el más ideal para tratar esta complicación.
a. O2 al 100%+Aire+fentanil 2-3 ug/kg c. O2 al 100%+fentanil 2-3 ug/kg
b. O2 al 100%+Aire+lidocaina 1-1,5 mg/kg d. O2 al 100%+lidocaina 1-1,5 mg/kg

2
Name: ________________________ ID: A

____ 18. Paciente mujer de 55 años con diagnóstico de cáncer de ovario es preparada para una intervención
quirúrgica de Protocolo de ovario y colocado el respectivo monitoreo necesario e indispensable en la
paciente, decimos que el uso transoperatorio del electrocardiograma en forma regular, nos ayuda a
detectar las siguientes disrritmias, como las siguientes, señale las más importantes de este caso.
a. Disritmias, Isquemia Miocárdica, c. Disritmias, Isquemia Miocárdica,
Anormalidades de la Conducción e Anormalidades de la Conducción e Hipoxia
Hipertensión arterial. cerebral.
b. Disritmias, Isquemia Miocárdica, d. Disritmias, Isquemia Miocárdica,
Anormalidades de la Conducción, y Anormalidades de la Conducción e
perturbaciones Electrolíticas. Hipotensión arterial.

____ 19. Paciente femenina que está programada para realizarle una cirugía de Toracotomía, es indispensable el
monitereo complero, más la colocación de una Cateterización de la Arteria Pulmonar, la misma que está
Contraindicada en las siguientes patologías, indique cuáles son las correctas:
a. Síndrome de Wolff Parkinson White, BCRDHH, c. Síndrome de Wolff Parkinson White, BIRDHH,
Malformación de Ebstein. Malformación de Ebstein.
b. Síndrome de Wolff Parkinson White, BCRIHH, d. Síndrome de Wolff Parkinson White, BIRIHH,
Malformación de Ebstein. Malformación de Ebstein.

____ 20. Paciente masculino de 70 años de edad, está programado para realizarse un Procedimiento Quirúrgico de
Prostatectomía Convencional, para lo cual es necesario la colocación de un Cateter urinario, sobre todo
en personas las siguientes patologías:
a. ICC, IR, IAM c. ICC, IR, HTA
b. ICC, IR, BCRDHH d. ICC, IR, Enfermedad Hepática Avanzada

____ 21. Paciente de 40 años varón, con ictericia, fiebre y dolor abdominal cólico localizado en hipocondrio derecho
de 72 horas de evolución, que en la ecosonografía presenta una vía biliar no dilatada en las porciones
intra ni esxtrahepáticas, con antecedentes de problemas de colitis; su diagnóstico será:

a. Ca periampular
b. Colangitis esclerosante
c. Coledocolitiasis
d. Cirroisis hepática

____ 22. Paciente mujer de 35 años, con dolor cólico en hipocondrio derecho de 4 días de evolución, ha sido
sometida a colecistectomía y en los hallazgos se reporta vesícula grande con paredes infamadas, cálculo
impactado en el cístico, llena de material mucoide; el posible diagnóstico es:
a. Empiema vesicular
b. Colecistitis crónica
c. Coledocolitiasis
d. Hidropesía vesicular

____ 23. Paciente varón de 40 años con diagnóstico de pancreatitis aguda, a quien se le somete a TAC dinámica y se
reporta lo siguiente: Agrandamiento difuso del páncreas, aspecto heterogéneo de la glándula y de la
grasa peripancreática, con dilatación del conducto de Wirsung y pequeñas colecciones extrapancreáticas.
Señalar que tipo de pancreatitis según la escala tomográfica de Balthazar presenta:
a. A
b. B
c. C
d. D

3
Name: ________________________ ID: A

____ 24. Paciente varón 35 años con diagnóstico de pancreatitis aguda severa de etiología biliar, con 20 días de
evolución, cuyo APACHE ll al momento es de 14, con una TAC que reporta según la escala de Balthazar
una pancreatitis tipo E, con un índice de severidad tomográfico de 7 y cuyo estado general se deteriora
progresivamente, en su opinión el tratamiento a seguir es
a. ERCP
b. Rotación del esquema antibiótico
c. Colecistectomía urgente
d. Necrosectomía

____ 25. Paciente varón de 42 años de edad, quien acude con un sangrado digestivo alto de instauración rápida, se le
somete a una endoscopia digestiva alta que reporta un sangrado activo tipo Forrest B, se señala que al
parecer tiene un sangrado mayor a 1000cc, con un hematocrito real de 28%, más criterios de shock
hipovolémico. Como se debe proceder
a. Incrementar volumen de líquido y sangre
b. Incrementar la dosis de los bloqueadores de bomba de protones
c. Cirugía
d. Nueva endoscopía

____ 26. Paciente de 80 años con episodios de diarrea alternados con estreñimiento, dolor en cuadrante inferior
izquierdo abdominal, fiebre, vómito, eventual proctorragia y que al examen físico se palpa una masa
dolorosa en fosa ilíaca izquierda, nos hace pensar en cuál de las siguientes posibilidades diagnósticas:

a. Enfermedad hemorroidal
b. Colon irritable
c. Diverticulitis colónica
d. Vólvulo de sigma

____ 27. Paciente mujer de 40 años de edad, con antecedente de ascitis secundario a insuficiencia cardíaca
congestiva; con un cuadro de dolor abdominal de 3 días de evolución, nausea, alza térmica, al examen
abdomen doloroso con signos de irritación peritoneal; la posibilidad diagnóstica y tratamiento serán,
señale lo correcto:
a. Peritonitis primaria y resolución quirúrgica urgente
b. Peritonitis primaria y tratamiento antibiótico empírico de inicio
c. Peritonitis secundaria y tratamiento quirúrgico urgente
d. Peritonitis terciaria y tratamiento antibiótico empírico de inicio

____ 28. Paciente varón de 35 años, policía sufre varios impactos de proyectil a nivel abdominal, llegando a
emergencias con 1 hora de evolución, shocado, hipotérmico. Ingresa a cirugía en forma emergente
encontrando, hemoperitoneo de 1000cc, trauma esplénico, trauma hepático, múltiples perforaciones
intestinales y colónicas. Cuál sería la cirugía a realizar
a. Reparación hepática, esplénica, resección intestinal y rafia colónica
b. Drenaje de hemoperitoneo, reparación hepática, rafia esplénica, rafia intestinal y
colostomía temporal
c. Drenaje de hemoperitoneo, sutura de todas las lesiones
d. Cirugía de control de daños

4
Name: ________________________ ID: A

____ 29. Varón de 55 años, campesino, sin antecedentes quirúrgicos, con informe de distensión abdominal,
constipación; que desde hace 24 horas presenta dolor abdominal cólico intenso, distensión abdominal
muy marcada, sin tránsito intestinal, ni expulsión de flatos; señala que en ocasiones anteriores ha
presentado episodios similares, pero que se resolvían solos y en menos tiempo, hace 1 hora un vómito
tipo fecaloide; al examen físico se tiene intenso timpanismo y mayor dolor con resistencia voluntaria en
fosa ilíaca izquierda., además signos de irritación peritoneal. Que posibilidad diagnóstica a su criterio
sería la correcta:

a. Obstrucción intestinal por brida adherencial


b. Obstrucción intestinal por vólvulo de sigma
c. Diverticulits colónica complicada
d. Síndrome pilórico

____ 30. Paciente varón de 23 años de edad, que presenta un peritonitis secundario a apendicitis aguda complicada,
ha sido intervenido en dos ocasiones, siendo sometido a dos laparotomías, al momento el abdomen
distendido, está con una faja para evitar dehiscencia de sutura, dificultad respiratoria, y signos de
hipoperfusión; se le ha midiendo la presión intrabadominal, obteniendo un valor de 26 mmHg, con este
valor favor indicar que tipo de hipertensión abdominal tiene y cuál sería el tratamiento a seguir
a. grado 1 y observación
b. Grado ll y descompresión quirúrgica
c. Grado lll y posible descompresión abdominal quirúrgica
d. Grado lV y descompresión quirúrgica

____ 31. Paciente mujer de 32 años de edad, quien sufre una agresión con arma blanca hace 4 horas, es sometida a
laparotomía encontrando biliperitoneo de 300cc, secundario a lesión de la pared de la vesícula biliar.
Según los datos proporcionados la intervención quirúrgica realizada es una:
a. Cirugía limpia
b. Cirugía límpia contaminada
c. Cirugía contaminada
d. Cirugía sucia

____ 32. Paciente varón, que sufre un trauma abdominal abierto y es sometido a lavado peritoneal diagnóstico,
presentando el siguiente resultado: Contaje de glóbulos rojos de 11.000 por mm cúbico, señale lo
correcto:

a. Es un resultado positivo
b. Es un resultado negativo
c. Es un resultado intermedio
d. No se debe realizar lavado peritoneal diagnóstico en trauma abdominal abierto

____ 33. Paciente mujer de 44 años, quien presenta dolor cólico en HD de 7 horas de evolución, iniciado luego de la
ingesta de comida grasa, cuadro similares en otras ocasiones, Murphy positivo; se le solicita un eco de
hígado y vía biliar el cual reporta: pared de vesícula de 2mm, diámetro transverso de 2cm, presencia de
múltiples cálculos pequeños. Indicar cuál es el diagnóstico ecosonográfico:

a. Colelitasis
b. Colecistitis aguda
c. Hidrocolecisto
d. Piocolecisto

5
Name: ________________________ ID: A

____ 34. Paciente de 28 años con un cuadro de evolución de 72 horas de evolución de dolor abdominal, localizado
en FID, ha sido diagnosticado de peritonitis secundario a apendicitis complicada. Al ser peritonitis
secundaria se debe iniciar con una antibióticoterapia empírica, que esque será el más adecuada:

a. ampicilina, gentamicina, tetraciclina


b. ciprofloxacina, cloranfenicol
c. ampicilina más sulbactam
d. ceftriaxone y metronidazol

____ 35. Paciente varón de 31 años que acude por presentar masa reductible en ingle derecha, de 4 meses de
evolución, no dolorosa, al examen fisco se palpa masa superior que aparece en el orificio profundo y baja
aparentemente por el interior del conducto inguinal, su diagnóstico será:
a. Hernia inguinal directa
b. Hernia inguinal indirecta
c. Hernia crural
d. Hernia mixta

____ 36. ¿A qué tipo de abdomen Agudo pertenece la peritonitis?


a. Abdomen Traumatico
b. Abdomen Inflamatorio
c. Abdomen Obstructivo
d. Abdomen Hemorragico

____ 37.
¿Cuáles son las causas que producen una perforación tífica?

a. Colecistitis

b. Colitis ulcerativa

c. Apendicitis

d. Todas las anteriores

6
Name: ________________________ ID: A

____ 38.
Mujer de 75 años de edad con HTA controlada, hipercolesterolemia, que 2 años antes el
inicio del cuadro actual fue estudiada por presentar un cuadro de diarreas y se
realizó. Enema Opaco, que no mostró alteraciones y posteriormente, colonoscopia:
hasta ciego no se ven alteraciones de la mucosa. En la actualidad acude al Servicio
de Urgencias con un cuadro de dolor abdominal, más intenso en hemiabdomen
izqdo. Malestar general, sudoración y emisión de sanare roía tranca gar ano. En la
exploración física destaca el abdomen doloroso, con leve distensión y el tacto rectal
muestra restos hemáticos. Presenta Hto. 36%, Hb 11,7 g/dl, VCM 78 fl, urea 55,
creatinina 1,1 mg/dl. Se realiza colonoscopía. ¿Cuál considera que es el diagnóstico
más probable?
a. Cancer de Colon
b. Colitis Ulcerosa
c. Colitis Actìnica
d. Colitis Isquemica

____ 39. Un varón de 50 años, cardiópata conocido, en fibrilación auricular crónica acude a servicio
de emergencia refiriendo dolor centro abdominal intenso y continuo, irradiado a epigastrio,
y de comienzo brusco hace unas 2 horas. En la exploración, el paciente está estable y con
sensación de mucho dolor abdominal, aunque el abdomen aparece blando y depresible sin
signos de irritación peritoneal. La exploración radiológica simple de abdomen es normal.
Señale, entre las siguientes, la afirmación correcta
a. La localización y características del dolor permiten descartar una isquemia
miocárdica.

b. La exploración abdominal normal permite descartar un abdomen agudo quirúrgico


c. Se debe realizar arteriografía mesentérica urgente para descartar una embolia
mesentérica

d. Lo más probable es que se trate de un dolor abdominal inespecífico y sin


consecuencias adversas.

____ 40.
Un paciente de 72 años, con fibrilación auricular crónica, acude al servicio de Urgencia
por un cuadro brusco de intenso dolor abdominal en región periumbilical progresiva. En
la arteriográfica selectica se observa una obstrucción redondeada de la arteria mesentérica
superior distal a la salida de la arteria cólica media. El tratamiento fundamental será:

a. Quirúrgico: embolectomía y/o resección del intestino no variable.

b. Quirúrgico: derivación mesentérica safena sin resección intestinal.

c. Medico: bolos de vasopresina por vía sistémica.

d. Medico: perfusión continua de glucagón por vía arterial.

7
Name: ________________________ ID: A

____ 41.
Una mujer de 85 años ingreso con insuficiencia ingreso con insuficiencia cardiaca y
diarrea sanguinolenta. La exploración abdominal no mostraba signos de irritación
peritoneal y había ruidos. Un enema opaco mostro imágenes “en huella digital” y la
colonoscopia, colitis en colon izquierdo respetando la mucosa rectal. ¿Cuál sería, entre
las siguientes, la actuación correcta?
a. Realizar artereografia

b. Practicar hemicolectomía izquierda


c. Prescribir dieta absoluta, tratamiento de insuficiencia cardiaca y observación
d. Instaurar anticoagulación con heparina
____ 42.
Varón de 70 años, con enfermedad pulmonar obstructiva crónica con restricción modera,
insuficiencia renal crónica, con creatinina sérica de 3ml/dl, infarto de miocardio antiguo y
aneurisma de aorta abdominal de 7cm de diámetro, asintomático. ¿Cuál es la actitud
correcta?

a. Resección quirúrgica del aneurisma e interposición de injerto aórtico.

b. Controles con Tomografía Axial Computarizada anuales.


c. Mediación antiagregante plaquetaria por alto riesgo quirúrgico.

d. Anticoagulación con dicumarinicos y controles con Resonancia Magnética


Nuclear
____ 43. A sala de urgencia llega un politraumatizado múltiple con costillas derechas fracturadas, que
se presenta en coma medialmente reactivo con discreta anisocoria pupilar, hipotensión
arterial muy grave, grave compromiso respiratorio con murmullo vesicular inaulidable en
hemitórax derecho y abdomen contracturado a la palpación. Indique, entre los siguientes,
cual es los procedimientos asistenciales menos prioritarios
a. Radiografia de torax

b. Intubación traqueal
c. Puncion – lavado intraperitoneal
d. Tomografía computalizada craneal
____ 44. Discreta distensión abdominal y matidez en flancos y el Hto, que era prácticamente normal al
ingreso, disminuye a 30 % a las tres horas. En la Rx de tórax se objetiva fractura de las
costillas 10-11 izquierdas. La causa más probable de la anodización es este paciente es
a. Traumatismo renal con hemorragia retroperitoneal

b. Rotura de hígado con hemoperitoneo


c. Rotura de bazo con hemoperitoneo
d. Traumatismo pancreático con pancreatitis traumática

8
Name: ________________________ ID: A

____ 45.
Indique en que tipos de heridas se utiliza el Hilo de Poliamida y en que cirugía es
recomendable.

a. Heridas complicadas, cirugía.


b. Heridas Simples y complicadas.
c. Heridas Simples y en cirugía plástica.
d. Cirugia

____ 46. El ácido poliglicolico cuales son sus propiedades e indicaciones


a. Monofilamento, Sintético, gastroenterología.
b. Heridas Simples y en cirugía plástica.
c. Negro Alta Fuerza tensil, Inerte, dura de 14 a 21 días.
d. Multifilamento trenzado, fuerza tensil dura de 14 a 21 días, cirugía general,
gastroenterología, urología, etc.

____ 47.
Como se realiza la Desinfección del contorno de la herida.

a. Antiinflamatorios, analgésicos.
b. Tocando los bordes de la herida
c. Antisépticos de uso tópico, de preferencia no irritantes, Iodopovidona o alcohol.
d. Zona menos sensible al dolor.

____ 48. Que líneas se utiliza para realizar la diéresis de la piel para realizar las suturas.
a. Líneas de klefla.
b. Líneas de langer
c. Puntos anatómicos
d. Tejido celular subcutáneo.

9
Name: ________________________ ID: A

____ 49.
Con respecto a la profilaxis antibiótica en cirugía responda cual de los siguientes
enunciados es falso.

a. Debe utilizarse un fármaco que actúe sobre los microorganismos mas


comunes en el sitio quirúrgico.
b. Debe administrarse 12 horas antes de la intervención quirúrgica
c. El antibiótico debe repetirse durante operaciones prolongadas según la
semivida del fármaco para asegurar las concentraciones adecuadas en los
tejidos.

d. en la profilaxis habitual, el régimen antibiótico no debe continuarse más de 24


h después de la intervención quirúrgica.

____ 50.
Son signos de infección en el sitio quirúrgico, EXCEPTO.

a. Edema.
b. Eritema.
c. Hipotermia.
d. Dolor.

____ 51.
Dentro de los factores del paciente que promueven una infección en el sitio quirúrgico
cual de los siguientes no corresponde.

a. Edad, obesidad, tabaquismo


b. Preparación del sitio quirúrgico deficiente.
c. Anemia, desnutrición.
d. Patología de base.

10
Name: ________________________ ID: A

____ 52.
Las infecciones del sitio quirúrgico se clasifican en. EXCEPTO:

a. aInfecciones insicionales.
b. Infecciones de órgano/ espacio.
c. Limpias, limpias-contaminadas, contaminadas y sucias.
d. Infecciones de piel y tejido subcutáneo e infecciones profundas.

____ 53.
La peritonitis primaria o espontánea se produce por invasión bacteria via:

a. Hematogena o linfática
b. Por perforación de víscera hueca
c. Perforación por úlcera
d. Ninguna

____ 54. Con respecto a la herida limpia/contaminada señale el enunciado falso.


a. Comprenden aquellas en las cuales se abre una víscera hueca, como las vías
respiratorias, digestivas o genitourinarias.
b. Ejemplos de estas son: Diverticulitis perforada, infecciones necrosantes de tejido
blando.
c. El cierre de estas heridas es por primera intención
d. Ejemplos de estas heridas son: Colecistectomía, cirugía electiva de tubo digestivo.

____ 55.
Sobre cicatrización patológica: señale lo correcto
a. Entre las principales encontramos: Cicatriz Hipertrófica, cicatriz dolorosa,
Ulceras cicatriciales rebeldes.
b. Cicatriz Hipertrófica: surge por un desarrollo excesivo de miofibroblastos,
que conlleva a una tendencia a la retracción y tiene importante
repercusiones funcionales. Suele diferenciarse del queloide en que si
sobrepasa los límites de la cicatriz.
c. Queloide: cicatriz exuberante por exceso de colágeno, que rebasa los límites
de la piel sana. Se considera un tumor benigno. Son más frecuente en la
región pree esternal y espalda.
d. Ulceras cicatriciales rebeldes: se ocasionan cuando la proliferación del colágeno
estrangula la formación de yemas vasculares, produciendo isquemia.

11
Name: ________________________ ID: A

____ 56.
Fisiología de la cicatrización: señale lo incorrecto
a. Fase inflamatoria
b. Epitelización. Fase celular o de neo formación vascular
c. Fase proliferativa y de síntesis de ácido hialuronico
d. Fase de remodelado
____ 57.
Señale lo correcto sobre los aspectos que afectan a la cicatrización de heridas
a. Existen dos aspectos: Sistémicos y Locales
b. Sistémicos: edad, nutrición, traumatismos, enf. Metabólicas,
inmunodepresión, trastornos del tejido conjuntivos, tabaquismo.
c. Locales: lesión mecánica, infección, edema, isquemia/necrosis de tejido,
agentes tópicos, radiación ionizante, tensión de oxigeno baja, cuerpos extraños.
d. Todas son correctas.
____ 58.
Características de Queloides y Cicatrices Hipertróficas: señale lo correcto
a. Queloide: incidencia rara, predisposición genética, no hay regresión
b. Cicatriz hipertrófica: incidencia frecuente, sin predilección por grupos
étnicos,
c. Cicatriz hipertrófica: sin predisposición genética, regresión frecuente y
espontanea
d. Todo es correcto

____ 59. Sobre la clasificación de los materiales de sutura según su origen ponga la respuesta correcta
a. Simples y compuestos
b. Inorgánicos, orgánicos, sintéticos

c. acero inoxidable, agrafes, catgut.

d. polipropileno, polietileno, poligaláctico


____ 60. Señale las complicaciones de una hemorroidectomía.
a. Retención urinaria
b. Retención fecal
c. Hemorragia masiva
d. Todas
e. Ninguna

12
Name: ________________________ ID: A

____ 61. Referente al tratamiento del absceso perianal, señale lo falso:


a. Drenaje bajo anestesia local en el consultorio, clínica o departamento de
urgencias.
b. En abscesos grandes y complicados es indispensable el drenaje en el quirófano
c. Hacer una incisión cutánea y se extirpa un disco de piel para prevenir el cierre
prematuro.
d. Los abscesos simples se drenan a través de una incisión de la piel suprayacente.
____ 62.
Señale lo falso referente al quiste pilonidal
a. Consiste en un seno o absceso que contiene pelo y aparece en la hendidura
interglútea.
b. La hendidura interglútea ocasiona aspiración que introduce pelos al interior de
fosos en la línea media cuando el sujeto se sienta.
c. Estos abscesos suelen ser muy profundos, por lo que el procedimiento se
realiza en el quirófano.
d. Debe cortarse y drenarse el absceso agudo.
____ 63.
Referente al procedimiento correcto para el quiste pilonidal señale lo verdadero
a. Consiste en destechar el trayecto, raspar la base y marsupializar la herida.
b. Mantener limpia la herida y sin pelo hasta que cicatrice por completo.
c. Practicar una incisión lateral pequeña y extirpar el foso.
d. Todas
e. Ninguna
____ 64.
Referente a una fistula anal, señale lo incorrecto:
a. Casi todas las fístulas son de origen criptoglandular
b. Las fístulas se clasifican de acuerdo con su relación con el complejo del
esfínter anal y las opciones terapéuticas se basan en estas clasificaciones:
fístula interesfinteriana, fístula Transesfinteriana, fístula
supraesfinteriana, fístula extraesfinteriana.
c. El objetivo del tratamiento de la fístula anal es erradicar la infección sin
sacrificar la continencia.
d. Los trayectos fistulosos circundan tramos variables del complejo esfinteriano,
por lo tanto no es considerable un tratamiento quirúrgico.

13
Name: ________________________ ID: A

____ 65.
Sobre el tema: tratamiento de las heridas: señale lo correcto
a. Cierre secundario o por primera intención, cierre por segunda intención y,
cierre por tercera intención o sutura primaria diferida.
b. Por primera intención: sutura inmediata de la herida. En heridas con
mínima contaminación bacteriana, hemorragia controlable, y sin tejido necrótico
ni cuerpos extraños.
c. Cierre por segunda intención: se sutura la herida y no se espera que
cicatrice espontáneamente. Indicada en: heridas muy contaminadas,
cuando el tratamiento se ha demorado más de 6 -8 horas, cuando hay
trayectos muy irregulares, mordeduras humanas y de animales con
tratamiento antibiótico

d. Ninguna es correcta.

____ 66.
Señale la opción correcta acerca de la pancreatitis aguda y la frecuencia de
presentación:

a. Ileo reflejo 90%


b. Dolor Abdominal 50%
c. Hemorragia 12%
d. Vomitos 50%

____ 67. Aun paciente de 70 años, colecistectomizado, con ictericia de 48 horas, de evolución, con bilirrubina total
de 8 mg/dl y bilirrubina directa de 6 mg/dl. fosfatasa alcanila 620 U/L, fiebre de 39 C y leucocitosis
mayor de 20000 con desviación a la izquierda. Se le relaiza eco abdominal que da como resultado
coledocolitiasis. El tratamiento inicial debe ser:
a. Reposición hidroelectrolítica y antibioticoterapia unicamente, posponiendo cualquier
otro proceder a la desaparición de los síntomas y signos de la infección
b. Reposición hidroelectrolítica y antibioticoterapia y laparotomía urgente
c. Reposición hidroelectrolítica y antibioticoterapia y corticoides
d. Reposición hidroelectrolítica y antibioticoterapia y esfinterotomía más drenaje bililar
más colangiografía retrógrada endoscópica

14
Name: ________________________ ID: A

____ 68.
Hombre de 69 años de edad con antecedentes de diabetes mellitus y consumo
moderado de alcohol desde hacía años, que acude a Urgencias por dolor abdominal de
tipo cólico de 7 días de evolución en hemiabdomen superior, más localizado en
hipocondrio derecho. Asocia prurito generalizado que le dificulta el sueño y orinas
oscuras en los últimos dos días. En la exploración física destacaba TA 130/61, FC 102
lpm, Tª 36,8ºC. El abdomen estaba blando, depresible, doloroso a la palpación
profunda y sensación de masa en el hipocondrio derecho. Los análisis de sangre
muestran proteína C reactiva 12.0 mg/dL, glucosa 115 mg/dL, urea 45 mg/dL,
creatinina 0.72 mg/dL, bilirrubina total 8.45 mg/dL, electrolitos normales, ALT (GPT
plasma) 45 U/L, GGT 112 U/L, LDH 110 U/L, Lipasa 16 U/L. Leucocitos 9.3 10
E3/μl, hematíes 4.08 10 E6/μl, hemoglobina 12.3 g/dl, hematocrito 35,9%, VCM 87.9
fl, recuento de plaquetas 217 10 E3/μl ( N 69.3%, L 26.2%, M 4.05%, Eo 0.0%, Bo
0.0%). Se efectuó ecografía abdominal que se muestra en la imagen dilatación del
colédoco, con positividad de coledocolitiasis.
¿Cuál es la actitud más recomendable en este momento?

a. Colecistectomía, Antibioterapia y medidas sintomáticas


b. Colangiografía retrógrada endoscópica con papilotomía
c. Colecistectomía diferida
d. Ecoendoscopia para valorar el páncreas
____ 69. Paciente colecistectomizada hace 6 años por colelitiasis. desde hace 6 meses presenta dolor cólico en flanco
derecho a temporadas. En la analítica hay valores de colestacis moderada. La ecografía abdominal
suguiere coledocolitiasis. ¿Cuál entre las siguientes, es la prueba indicada para realizar, en primer lugar,
de este caso?
a. Tomografía computarizada con contraste abdominal IV
b. Colangiografía I.V
c. Colangiografía retrógrada endoscópica
d. Colangiografía isotópica

____ 70. Los càlculos en el colédoco pueden ser asintomáticos o presentar:


a. Còlico biliar
b. Ictericia Obstructiva
c. Colangitis ascendente y pancreatitis
d. Todas las anteriores

____ 71.
Que maniobra se puede realizar al paciente con una hernia de la pared abdominal al
momento de explorar señale la correcta

a. Valsalva
b. Mac Burney
c. Murphy
d. Ninguna de las anteriores

15
Name: ________________________ ID: A

____ 72.
Las hernias umbilicales adquiridas se cierran de forma espontánea alrededor de los.
Señale la correcta

a. 5 años de edad
b. 10 años de edad
c. 12 años de edad
d. Ninguna de las anteriores

____ 73.
Señale cuales son los factores que pueden contribuir a una eventración o hernia
postoperatoria que dé lugar a su formación

a. Obesidad + cicatrización de heridas


b. Diabetes + malos hábitos de alimentación
c. Esfuerzo físico + mala técnica quirúrgica
d. Ninguna de las anteriores

____ 74.
Con respecto a las hernias de la Pared Abdominal señale la opción correcta.

a. En las Hernias ventrales no siempre están afectadas la aponeurosis y


músculos de la pared abdominal
b. Estas pueden ser congénitas y no adquiridas
c. El hallazgo más común es una masa o un abultamiento
d. Todas son correctas

____ 75.
Con respecto a la utilización de las mallas en cirugía señale la opción correcta

a. No son absorbibles
b. Son absorbibles y se degradan
c. Se eliminan desde los tejidos
d. Pierden su integridad estructural más rápido en los tejidos
e. A y B son correctas

16
Name: ________________________ ID: A

____ 76.
Las infecciones del sitio quirúrgico se clasifican en. EXCEPTO:

a. Infecciones insicionales.
b. Infecciones de órgano/ espacio.
c. Limpias, limpias-contaminadas, contaminadas y sucias.
d. Infecciones de piel y tejido subcutáneo e infecciones profundas.

____ 77. La peritonitis primaria o espontánea se produce por invasión bacteria vía:
a. Hematogena o linfática
b. Por perforación de víscera hueca
c. Perforación por úlcera
d. Ninguna
____ 78.
Peritonitis tuberculosa (tuberculosis peritoneal), Etiología:

a. Reactivación de un foco peritoneal secundario a una diseminación hematógena


b. Reactivación de un foco peritoneal secundario a una rotura de una adenopatía
mesentérica afecta
c. Más frecuente en mujeres
d. Todas las anteriores
____ 79.
Síntomas subdiafragmaticos por vecindad en la peritonitis secundaria:

a. Disnea
b. Tos
c. Dolor en hombro izquierdo o derecho e hipo por irritación diafragmática.
d. Todas las anteriores
____ 80. En la peritonitis secundaria extrahospitalara cuál es el tratamiento de elección.

a. Ceftriaxona o cefotaxima + metronidazol.


b. Imipenen o meropenem
c. Piperzacilia + tazobactam
d. El tratamiento de elección en peritonitis secundaria extrahospitalaria es
Ceftriaxona o cefotaxima + metronidazol.

17
Name: ________________________ ID: A

____ 81. Se define fístula enterocutánea a:


a. Una cavidad de líquido infectado y pus localizada dentro de la cavidad abdominal, puede
haber más de una
b. La comunicación anormal entre el aparato gastrointestinal y la piel, con salida del
contenido intestinal a través de la misma
c. La protrusión de cualquier órgano o tejido fuera de la cavidad del cuerpo en que está
alojado normalmente
d. Una respuesta sistémica a un proceso infeccioso localizado

____ 82. Las complicaciones principales de los pacientes con fístulas son:

a. Desequilibrio hidroelectrolítico
b. Desnutrición
c. Sepsis
d. Todas son correctas

____ 83. Según la etiología de las fistulas enterocutáneas las fistulas postoperatorias son causales de:
a. Isquemia intestinal
b. Enfermedad de Crohn
c. Operaciones por cáncer
d. Apendicitis
____ 84. Paciente varón de 69 años intervenido quirúrgicamente hace 13 meses por hernia inguinal derecha
estrangulada, con compromiso de asas intestinales, como consecuencia portador de ileostomía, hace 13
días acude al servicio para intervención quirúrgica de restitución de tránsito intestinal, donde le realizan
anastomosis latero lateral ileontransversa isoperistaltica, afrontada en dos planos. Actualmente ingresa al
servicio por presentar malestar general, asociado a secreción de material fecaloide por sitio operatorio.
Cúal es su diagnóstico?

a. Hernia estrangulada
b. Fístula biliocutánea
c. Fascitis necrosante
d. Fístula enterocutánea

____ 85. Cuál es la complicación más temida en los pacientes con fistulas enterocutáneas.
a. Formación de adherencias
b. Deshidratación
c. Sepsis
d. Todas las anteriores

18
Name: ________________________ ID: A

____ 86.
Respecto a los canales de calcio: señale lo incorrecto
a. A los canales de calcio también se les denomina canales lento y necesitan 10 a 20
veces más tiempo para su activación que los canales de sodio.
b. Cuando hay un déficit de iones calcio la fibra e hace muy excitable
c. Para que se produzca tetania muscular es necesario una disminución del 30% del
ion calcio
d. Los iones Ca+ parecen unirse a la superficie externa de la molécula de la proteína
del canal sodio

____ 87.
Todos los signos y síntoma de hiponatremia excepto:
a. Taquicardia
b. Hipertensión arterial
c. Convulsiones
d. Diarrea acuosa

____ 88.
Todo son signos y síntoma de hipernatremia excepto:
a. Taquicardia
b. Hipotensión
c. Bradicardia
d. Delirio
____ 89.
Los síntomas predominantes de la hiperpotasemia son excepto:

a. Gastrointestinales
b. Neuromusculares
c. Cardiovasculares
d. Respiratorios
____ 90. ¿Cuál no es una causa de hipopotasemia?
a. Ingestión insuficiente
b. Excreción renal disminuida
c. Diuréticos
d. Diarreas

19
Name: ________________________ ID: A

____ 91.
La nutrición parenteral total determina acciones como:

a. Mayor expresión del interferón gamma en el epitelio intestinal


b. Perdida de la función de barrera epitelial intestinal
c. Presenta complicaciones sépticas con alimentación entera
d. Solo a y b
____ 92. Las causas de la desnutrición en un paciente hospitalizado son, señale la respuesta incorrecta
a. infecciones agudas o cronicas
b. ayunos repetidos (paciente bien investigados pero ma nutridos )
c. cicatrizacion deficiente de heridas
d. uno de infusiones glucosadas (hipocaloricas)

____ 93. En qué período de tiempo se da el ayuno prolongado?


a. de 14 a 16 horas sin ingerir alimentos.
b. de 24 a 48 horas sin ingerir alimentos
c. más de 72 horas.
d. más de 96 horas

____ 94. ¿La nutrición enteral es?


a. Modalidad de terapia nutricional en la cual se administran soluciones de elementos
nutritivos en el torrente circulatorio
b. Es una técnica de soporte nutricional mediante la cual se aportan nutrientes de forma
directa al aparato digestivo, por vía oral mediante fórmulas líquidas químicamente
definidas o en los diversos tramos del tubo digestivo con sondas específicas.
c. Todas las anteriores
d. Ninguna de las anteriores

____ 95. ¿La necesidad basal de potasio es de?


a. 90mEg
b. 5 mEg
c. 20mEg
d. Ninguna de las anteriores

20
Name: ________________________ ID: A

____ 96. Paciente de 11 años llega al servicio de emergencia por presentar, hematuria macroscópica y dolor en
epigastrio que se irradia hacia fosa iliaca derecha, con síntomas acompañantes: disuria, fiebre máxima de
38,5°C en las últimas 24 horas, presenta 3 vómitos desde el inicio del cuadro, no presenta diarreas y la
diuresis se encuentra conservada. ¿Al examen físico que signo especifico desea encontrar?
a. Signo de Mc Burney
b. Signo de Murphy
c. A y B son correctas
d. Ninguna de las anteriores.

____ 97. Paciente de 33 años a emergencia por presentar dolor en hipocondrio derecho con intensidad 8/10, refiere
haber ingerido comida copiosa, acompañado de nausea, vomito e hipertermia. Al examen físico: dolor a
la descompresión y presencia de signo de Murphy. ¿Qué espera encontrar en los exámenes de
laboratorio?
a. Leucocitos y neutrofilia
b. Leucocitosis entre 12000 y 15000
c. Bilirrubinas hasta niveles de 4mg/dl
d. Todas las anteriores son correctas.

____ 98. ¿Cuál es el examen Gold estándar para identificar la pancreatitis crónica?
a. Ecografía abdominal
b. Rx de abdomen
c. Resonancia magnética de abdomen
d. CPRE o colangiopancreatografia retrograda endoscópica

____ 99.
Cuando se presenta peritonitis terciaria señale la correcta.
a. En pacientes postoperados con una peritonitis secundaria que no
responden a tratamientoy que presentan fallo multiorganico o sepsis.
b. Aparece tras una complicación intraabdominal o tras contaminación
quirúrgica o trauma.
c. Esta relaciona con un foco abdominal o perforación del tubo digestivo.
d. Ninguna
____ 100.
El absceso intrabdominal está vinculado con ciertas patologías señale la correcta.
a. Infección de sitio quirúrgico
b. Sigmoitis aguda
c. Ulcera perforada
d. Todas las anteriores

21
Name: ________________________ ID: A

____ 101.
Cuáles son los factores predisponentes para un absceso intraabdominal señale la
incorrecta.
a. Liberación de microorganismos en la cavidad peritoneal
b. Sinergismo bacteriano
c. Ausencia de hemoglobina y pigmentos biliares
d. Obstrucción
____ 102. Los abscesos abdominales según su clasificación por la localización intraperitoneal tenemos.
Señale la incorrecta.

a. Visceral.
b. De los espacios anatómicos existentes.
c. Del espacio de un órgano extirpado.
d. Diafragmaticos
____ 103.
Entre las complicaciones de un absceso intraabdominal tenemos las siguientes . Señale
la incorrecta.

a. Reaparición del absceso


b. Ruptura del absceso
c. Diseminación de la infección
d. Cáncer
____ 104. La hernia inguinal incarcerada se manifiesta:
a. Bulto doloroso en la región inguinal y cuadro obstructivo
b. Bulto doloroso, caliente, eritematoso o azulado
c. Se acompaña de leucocitosis, fiebre y signos de sepsis
d. Cuadro obstructivo, eritematoso o azulado
____ 105. La hernia inguinal estrangulada se debe:
a. Intentar reducir bajo sedación suave
b. Debe ser intervenida sin intentar reducirla
c. No se debe intentar reducir por el riesgo que conlleva reintroducir un segmento
intestinal con compromiso vascular
d. B y C son correctas

22
Name: ________________________ ID: A

____ 106. La hernia directa:


a. Protruye a través del suelo del canal inguinal a nivel del triángulo de Hesselbach,
no pasan a través del orificio profundo y no se localizan por dentro de las fibras
del cremáster, sino por detrás.
b. No tiene relación con el conducto inguinal, su riesgo de incarceración y
estrangulación es más elevado que en cualquier otra hernia
c. Sale de la cavidad abdominal por el anillo profundo, surge lateralmente a la arteria
epigástrica y al ligamento de Hesselbach
d. Acompaña a las estructuras del cordón inguinal por dentro de las fibras del
músculo cremáster, pudiendo salir por el orificio externo hasta el escroto.
____ 107. En la hernioplastia como procedimiento quirúrgico en hernias inguinales se utiliza con
mayor frecuencia las técnicas:
a. Técnica de Lichtenstein
b. Técnica de Rutkow
c. Técnica original de Bassini
d. A y B son correctas
____ 108.
Paciente de 22 años refiere que hace 3 horas recibe puñalada con arma blanca en
hipocondrio derecho, al cuadro clínico se acompaña de hemorragia que no sede,
hipotensión, bradicardia, bradipnea y con Glasgow 12, cuál sería el primer examen que se
le realizaría al paciente.

a. EMO

b. Tomografía computarizada

c. Biometría hemática

d. Rx de tórax

____ 109. Que es el traumatismo abierto y cuáles son sus características:

a. Se caracteriza por no presentar solución de continuidad en la pared abdominal. El


agente que lo produce es de superficie roma o plana, tipo barra de timón, puño,
etc.
b. Es cuando existe solución de continuidad en la pared abdominal, producida por
elementos cortantes o transfixiones, como en las heridas por arma blanca o heridas por
arma de fuego

c. Se caracteriza por no presentar una cicatriz a nivel abdominal

d. Se caracteriza por no presentar solución una herniación a nivel abdominal.

23
Name: ________________________ ID: A

____ 110.
Mujer de 54 años que, desde hace 6 horas, presenta vómitos intensos y dolor abdominal.
En la exploración clínica se aprecia tumoración umbilical dolorosa y en la radiología
simple, dilatación de asas de intestino delgado. Entre los siguientes, el diagnóstico más
probable es:

a. Tumor de intestino delgado

b. Invaginación intestinal

c. Hernia estrangulada

d. Estenosis Pilórica

____ 111.
Paciente de 75 años, sin antecedentes clínicos de interés, que consulta por un episodio de
hematoquecia reciente. La colonoscopia total demuestra un pólipo único o pediculado, de
unos 25 mm de diámetro, en sigma con una ulceración en su cúspide. ¿Qué actitud de las
siguientes es más adecuada?

a. Seguimiento cada 6 meses e intervención quirúrgica, si aumenta de tamaño

b. Indicar intervención quirúrgica.

c. Biopsia endoscópica del pólipo y decidir según resultado.

d. Realizar polipectomía endoscópica y estudio anatomopatológico del pólipo


____ 112. Ante un paciente que presenta dolor abdominal de tipo cólico, vómitos, distención abdominal
e incapacidad para evacuar gases y heces, hay que pensar como primer diagnóstico en:
a. Diverticulitis aguda c. Colecistitis
b. Obstrucción intestinal d. Apendicitis aguda

24
Name: ________________________ ID: A

____ 113.
Enfermo de 50 años que acude al servicio de urgencias por dolor abdominal difuso y
progresivo, distensión, borborigmos y vómitos ocasionales en las últimas 48 horas.
Apendicetomía a los 14 años. En la exploración hay fiebre (38.5°C) distensión abdominal
con ruidos intestinales aumentados y ocasionalmente en “espita”, sensibilidad a la
palpitación abdominal sin defensa ni signo de rebote. En la analítica hay leucocitosis con
neutrofilia; NA 133 mEq/L; CO#H de 14 mEq/L. En la placa simple de abdomen hecha en
bipedestación hay dilatación de las asas del delgado con niveles y edema de la pared, sin
prácticamente gas en el colon, ¿Cuál sería la conducta a seguir?

a. Aspiración nasogastrica más antibioticos c. Laparotomía urgente


de amplio espectro
b. Enema opaco para descartar volvulo de d. Colonoscopia descompresiva
sigma

____ 114.
Paciente masculino de 74 años de edad que presenta dolor abdominal de tipo cólico,
diarrea, distensión abdominal e incapacidad para evacuar gases y heces, hay que pensar
como primer diagnóstico en la radiología simple se observa un dilatación colónica con
una imagen de “asa en omega” o en “grano de café”

a. Volvulo del sigma c. Síndrome de Colon Irritable

b. Pseudoobstrucciónn del colon d. Obstrucción intestinal

____ 115.
Una mujer de 54 años inicia estudio por molestias abdominales. El escáner descubre una
lesión quística de 8 cm en el ovario derecho y dos nódulos abdominales que parecen
depender del peritoneo. El Ca-125 es normal. ¿Cuál de estas actitudes le parece más
acertada?

a. Laparotomía exploradora para c. Se trata de un quiste diseminado, la


diagnosticar y tratar un probable paciente debe recibir quimioterapia
cáncer de ovario sistémica.

b. Puede tratarse de un quiste lúteo, d. Vigilancia, repitiendo la exploración


convendría realizar un tratamiento de escáner en tres meses
prueba con anovulatorios y reevaluar
en tres meses.

25
Name: ________________________ ID: A

____ 116.
Paciente que presenta amenorrea de 7 semanas, asintomática, sin evidencia, mediante
ecografía vaginal, de útero ocupado. Se determina la fracción b de la gonadotropina
coriónica humana, obteniéndose una cifra de 2500 mU/ml. Ante estos datos, en primer
lugar, habrá que pensar en:

a. Gestación de evolución normal c. Gestación ectópica de evolución


correspondiente a la amenorrea asintomática.
b. Gestación normal con menor tiempo d. Aborto precoz completo, con
de evolución del correspondiente a su expulsión total de restos ovulares
amenorrea. intrauterinos.
____ 117.
Paciente de 30 años, nuligesta, con última regla hace 20 días, que acude a la consulta por
presentar esterilidad y dismenorrea importante. En las exploraciones que se realizan,
destaca tumoración anexial en la ecografía y elevación moderada de Ca-125 sérico. ¿Cuál
es el diagnóstico de presunción?

a. Embarazo ectópico c. Mioma uterino


b. Cáncer de ovario d. Endometriosis
____ 118. Paciente de 20 años que acude a consulta por dolor abdominal localizado a nivel de fosa iliaca
izquierda, de intensidad 6/10, de tipo sordo, desde hace aproximadamente 2 semanas, el
dolor se acompaña además nauseas que llegan al vómito por dos ocasiones en el último día.
Paciente es diagnosticado con enfermedad inflamatoria pélvica leve. ¿Cuál es el tratamiento
antibiótico ambulatorio de elección que usted prescribiría?

a. Clindamicina y gentamicina. c. Ceftriaxona y doxiciclina.


b. Metronidazol. d. Azitromicina.

26
Name: ________________________ ID: A

____ 119.

6. Adolescente de 15 años de edad, nulípara, acude con dolor abdominal bajo, de inicio
súbito, de 60 minutos de evolución, sin vómito, evacuaciones normales, sin molestias
urinarias ni flujo genital. Al examen físico: paciente muy quejumbrosa, afebril, con
palidez de piel y mucosas, diaforesis, sin agitación psicomotora, abdomen blando,
depresible, sensible en la fosa iliaca derecha, signo de Blumberg ausente, sin masas
palpables. Con un espéculo se observó el cuello uterino sano, sin metrorragia. Al
tacto vaginal se apreció el útero en retroversoflexión, sin dolor a la movilización
cervical, anexos sin masas palpables. La ecografía transvaginal mostró el útero en
retroversoflexión. En relación con el fondo uterino y hacia la izquierda, se observó
una imagen bien delimitada, ecogénica, de 8 x 9 cm de diámetro. Se observó una
representación ecográfica del pedículo vascular rotado, un vaso arterial central
rodeado por un vaso venoso, lo que daba la imagen de remolino, se perdía el sentido
paralelo en que corrían los vasos arteriales y venosos, compatible con el signo del
remolino. ¿En qué diagnóstico pensaría usted?

a. Torsión ovárica c. Enfermedad inflamatoria pélvica


b. Embarazo ectópico d. Ovario poliquístico
____ 120.
Paciente de 87 años con antecedentes de bronquitis crónica e insuficiencia cardiaca ha sido
diagnosticado de colecistitis aguda litiásica tras 4 días de hospitalización en tratamiento con
dieta absoluta, sueroterapia, piperacilina/tazobactam, el paciente continua con fiebre, dolor
abdominal persistente y leucocitosis. La actitud más adecuada en estos momentos sería:

a. Tratamiento quirúrgico c. Sustituir la piperacilina/tazobactam


colecistectomía urgente. por metronidazol + cefotaxima.
b. Drenaje biliar mediante d. Añadir al tratamiento un
colecistostomia percutánea. aminoglucósido como gentamicina.
____ 121.
Paciente de 52 años con datos clínico- biológicos de ictericia obstructiva de evolución
intermitente y sangre oculta en heces positiva, con dieta idónea. De los diagnósticos que se
reseñan. ¿Cuál es el más probable?:

a. Carcinoma pancreático c. Colecistitis crónica.


b. Litiasis vesicular. d. Ampuloma.

27
Name: ________________________ ID: A

____ 122.
El procedimiento inicial de elección para investigar una posible obstrucción de las vías
biliares es:
a. Ecografía hepatobiliar. c. Colangio resonancia magnética
b. Colangiopancreatografía retrógrada d. Tomografía computarizada.
endoscópica(CPRE).
____ 123.
¿Cuales son los factores que predisponen para el desarrollo de colelitiasis?

a. Obesidad, embarazo, factores c. Esferocitosis hereditaria, talasemia,


dietéticos, cirrosis. diabetes.
b. Enfermedad de Crohn, resección ileal d. Todas las anteriores
terminal
____ 124.
Un tumor de la bifurcación de los conductos biliares principales (t. de Klatskin), se
asocia con los siguientes hechos, EXCEPTO:
a. Ictericia indolora c. Acolia
b. Prurito d. Distensión vesicular
____ 125.
La colecistectomía laparoscópica tiene como ventaja sobre la colecistectomía por
laparotomía:
a. La reducción de la estancia c. No se asocia a lesiones de la vía
hospitalaria. biliar.
b. Permite conservar la vesícula. d. Es una técnica exenta de mortalidad.
____ 126.
¿Cuál de las siguientes sustancias forma parte de la secreción biliar?:
a. Carboxipeptidasa. c. Elastasa
b. Tripsina d. Lecitina

28
Name: ________________________ ID: A

____ 127.
Al leer el resultado de una ecografía abdominal, que usted ordenó por infecciones
urinarias de repetición, comprueba que la mujer de 67 años que tiene sentada en su
consulta tiene cálculos en la vesícula biliar, sin otros hallazgos significativos. La paciente
niega haber presentado cólicos o ninguna otra complicación relacionada con la colelitiasis.
Su única queja digestiva es la distensión abdominal postprandial. ¿Cuál de las siguientes
recomendaciones le daría usted?

a. La distención postprandial indica que c. Debe ser sometida a una


debe ser sometida a una colecistectomía abierta y no
colecistectomía de urgencia. laparoscópica para prevenir un cáncer
de vesícula.

b. No está indicado el tratamiento d. La colecistectomía profiláctica esta


quirúrgico porque se trata de una indicada por tratarse de una mujer
colelitiasis asintomática.

____ 128.
De las cuatro afirmaciones que se hacen sobre la colangitis aguda, una de ellas es
FALSA:

a. La presentación clásica se caracteriza c. Es obligatoria la hospitalización


por fiebre alta, ictericia y dolor reposición hidroelectrolítica, soporte
abdominal. general y terapéutica antibiótica

b. Una causa frecuente de colangitis es d. La endoscopia precoz permite el


la estenosis maligna del colédoco diagnóstico por colangiografía y el
drenaje de la vía biliar

29
Name: ________________________ ID: A

____ 129. Paciente de29 años de edad, tratado por una colitis ulcerosa desde hace dos años, y sin
episodios de descompensación en los últimos 6 meses. Refiere un dolor sordo en
hipocondrio derecho y astenia en el último mes. La exploración física no demuestra
ninguna alteración significativa, y en los estudios complementarios se observa una
bilirrubinemia total de 1.2 mg/ dl (bilirrubina directa 0.7 mg/dl), aspartato
aminotransferasa 89 UI/I (N=40 UI/I), fosfatasa alcalina 1 124 UI/I, (N= 320 UI/I), gamma
glutamiltransferasa 345 UI/I, albuminemia 38g/l y tasa de protrombina 100%. Negatividad
de los anticuerpos mitocondriales. Señale cuál de las siguientes enfermedades padece con
mayor probabilidad.

a. Fase inicial de una cirrosis biliar c. Metástasis hepática de un


primaria adenocarcinoma de colon

b. Hepatitis autoinmune d. Colangitis esclerosante primaria

____ 130.
Señale la respuesta correcta con respecto al diagnóstico de colangitis aguda.

a. Puede ser silenciosos y con frecuencia c. Cursa con falla orgánica, riesgo de
se descubren de manera incidental. progresión a una colangitis grave sin
Pueden provocar obstrucción, drenaje biliar temprano
completa o incompleta

b. Fiebre, dolor en el epigastrio o el d. Hipersensibilidad epigástrica o en el


cuadrante superior derecho e ictericia. cuadrante superior derecho e ictericia
se conocen como la tríada de Charcot ligeras.

30
Name: ________________________ ID: A

____ 131.
En la colangitis aguda es común encontrar en los exámenes de laboratorio lo siguiente:

a. Leucocitosis, hiperbilirrubinemia y c. En el 30% de los casos hay


aumento de la fosfatasa alcalina y de hipergammaglobulinemia y en el
las transaminasas 40-50% hay un aumento de la IgM.

b. Presencia de cálculos en la vesícula d. Fosfatasa alcalina e IgM elevadas.


biliar y demostrará conductos
dilatados y tal vez señala el sitio de
obstrucción.

____ 132.
Señale lo correcto con respecto al tratamiento de colangitis:

a. Se recomienda colecistectomía c. El tratamiento inicial de la colangitis


laparoscópica electiva incluye antibióticos y reanimación
con líquidos por vía intravenosa, es
necesario drenar el conducto biliar
obstruido tan pronto se estabilice el
paciente.
b. Utilización de quelantes de las sales d. El tratamiento específico de la
biliares, administración de vitaminas enfermedad se han ensayado diversos
liposolubles. fármacos como: ursodiol, colchicina,
metotrexate, azatioprina o
ciclosporina. Los corticoides y la
D-penicilina no son eficaces.

31
Name: ________________________ ID: A

____ 133.
Señale lo correcto:

a. La pancreatitis aguda es una c. Muy pocos episodios de pancreatitis


enfermedad inflamatoria del aguda se inician con dolor intenso,
páncreas que no se acompaña de por lo general después de una comida
fibrosis de la glándula, o muy poca. abundante

b. Un 50% de los casos depende de dos d. El dolor característico de la


factores: cálculos de vías biliares y pancreatitis aguda se describe como
alcoholismo, mientras que el restante “puñalada” o “transfictivo” hacia la
50% lo constituyen una enfermedad espalda y se incrementa cuando se
idiopática o una diversidad de causas, inclina el paciente hacia adelante.
entre ellas traumatismo, operación,
fármacos, herencia, infección y
toxinas.

____ 134. De la anatomofisiologia de las fosas nasales y senos perinasales, seleccione la respuesta correcta.

a. En la pared externa de las fosas nasales esta constituida por los cornetes y los meatos

b. La pared superior de las fosas nasales esta constituidas por los huesos propios de la
nariz, la espina nasal del frontal, la lamina cribosa y el cuerpo del esfenoides

c. La pared inferior o suelo se separa de las fosas nasales de la cavidad oral

d. Los senos etmoidales posteriores drenan en el meato medio

____ 135. Identifique las complicaciones extracraneales de la otitis media

a. Mastoiditis aguda

b. Meningitis bacteranea

c. Tromboflebitis septica del seno lateral y del golfo de la yugular

d. Laberintitis purulenta

32
Name: ________________________ ID: A

____ 136. El tumor maligno más frecuente de la glándula submaxilar es:

a. Carcinoma mucoepidermoide

b. Crcinoma ex-adenoma pleomorfo

c. Cilindroma o carcinoma adenoide quistico

d. Citoadenolinfoma papilomatoso

____ 137. Ante un paciente con otitis externa disfusa, cual es el tratamiento inicial

a. Cirugía

b. Antibioticos mas analgésicos, ambos vía endovenosa

c. Antibióticos tópicos

d. Limpieza del conducto auditivo externo, antibioticoterapia tópica más analgésicos orales

____ 138. Dentro de los carcinomas de cabeza y cuello, que tipo de tumor se relaciona de forma más evidente
con el virus del papiloma humano.

a. Carcinoma epidermoide de Laringe

b. Carcinoma epidermoide de orofaringe

c. Carcinoma epidermoide de hipofaringe

d. Adenocarcinoma nasosinusal

____ 139. Niño de 5 años de edad que no pasó las pruebas de cribado auditivo al nacimiento y que presenta
potenciales evocados auditivos del tronco cerebral que determinan una hipoacusia bilateral leve
moderada - grave en el oído externo, cual es las actitud mas correcta a seguir en el paciente.

a. Realizar audiometría en el plazo de 6 meses para confirmar diagnóstico

b. Cirugía para adaptar implante coclear en el oído derecho

c. Cirugía para adaptar un implante coclear en el oído izquierdo

d. Adaptación de audoprótesis bilateral y rehabilitación logopédica

33
Name: ________________________ ID: A

____ 140. Paciente de 45 años de edad en el 15avo día postransplante de progenitores hematopoyéticos con
neutropenioa absoluta, plaquetas de 15000/ul y hemoglobina de 7g/dl que presenta un cuadro
clínico de dolor ocular con edema preorbitario con discreta secresión nasal sanguinolienta, cual es
el diagnóstico de presunción.

a. Hematoma preorbitario

b. Mucormicosis

c. Sinusitis aguda bacteriana

d. Reacción alércica posiblemente a la medicación

____ 141. Colocando un diapasón que está vibrando frente al conducto auditivo del oído que queremos
explorar y apoyandolo después en la apófisis masotides podemos de modo sencillo y en la cosnulta
distinguir entre sordera nerviosa alteración en la cóclea o nervio auditivo y sordera de conducción
transtorno en el sistema de transmisión tímpano oscilar, cual de estas afirmaciones es correcta para
un paciente que presenta sordera de conducción.

a. La percepción del sonido es igual por vía aérea que por vía ósea

b. La percepción del sonido es mejor por vía ósea que por vía aérea

c. La percepción del sonido es mejor por vía aérea que por vía ósea

d. La percepción del sonido es inditinguible tanto por vía aérea como por vía ósea

____ 142. La otitis media aguda no suele complicarse, no obstante cuando lo hace es con más frecuencia con
una:

a. Petrositis

b. Meningitis

c. Mastoiditis

d. Osteomielitis

____ 143. El síndrome de Ramsay Hunt se caracteriza por parálisi facial acompañada de:

a. Otitis media colesteatomatosa

b. Parálisis Yatrogénica

c. Parálisis por traumatismo craneal

d. Herpes zóster

34
ID: A

COMP CIRUGIA
Answer Section

MULTIPLE CHOICE

1. ANS: B PTS: 1 DIF: Alta


2. ANS: C PTS: 1 DIF: Alta
3. ANS: D PTS: 1 DIF: Alta
4. ANS: C PTS: 1 DIF: Alta
5. ANS: A PTS: 1 DIF: Baja
6. ANS: B PTS: 1 DIF: Alta
7. ANS: D PTS: 1 DIF: Baja
8. ANS: A PTS: 1 DIF: Baja
9. ANS: C PTS: 1 DIF: Alta
10. ANS: D PTS: 1 DIF: Alta
11. ANS: C PTS: 1 DIF: Alta
12. ANS: A PTS: 1 DIF: Alta
13. ANS: C PTS: 1 DIF: Baja
14. ANS: A PTS: 1 DIF: Alta
15. ANS: C PTS: 1 DIF: Alta
16. ANS: B PTS: 1 DIF: Alta
17. ANS: D PTS: 1 DIF: Alta
18. ANS: B PTS: 1 DIF: Alta
19. ANS: B PTS: 1 DIF: Alta
20. ANS: D PTS: 1 DIF: Alta
21. ANS: B
La colangitis esclerosante produce una obliteración inflamatoria y fibrótica progresiva de las vías biliares
intra y extrahepática, se asocia a colitis ulcerativa. La respuesta es B

PTS: 3 DIF: alta


REF: Brunicardi, F; Capítulo 31: Vesícula biliar y sistema biliar extrahepático, Schwartz.: Principios de
Cirugía, 8va edición, McGraw-Hill ISBN 97897010533737, 2006.
OBJ: Tratar al paciente quirúrgico con giagnóstico de coledocolitiasis y colangitis
TOP: Vesícula, vía biliar, páncreas KEY: colangitis, esclerosante
NOT: Dr. Vinicio Moreno Rueda
22. ANS: D
En casos de colecistitis aguda no resuelta en forma inmediata la vesícula se llena de material mucoide y
se distiende hasta alcanzar gran tamaño, este material proviene del epitelio de la vesícula y se denomina
hidropesía vesicular o hidrocolecisto. La respuesta es D.

PTS: 3 DIF: alta


REF: Cazorla, G; Colelitiasis y Colecistitis aguda: Urgencias quirúrgica 2da edición, Indugraf, ISBN
9789942035042, 2011.
OBJ: Tratar al paciente quirúrgico con diagnóstico de patología de vesícula biliar
TOP: Vesícula, vía biliar, páncreas KEY: colecistitis, empiema
NOT: Dr. Vinicio Moreno Rueda

1
ID: A

23. ANS: C
Pancreatitis tipo C según la escala de Balthazar está constituido por el grado B de la misma escala más el
aspecto heterogéneo de la grasa peripancreatica lo que implica compromiso de la misma. Representa 2
puntos para el índice de severidad tomográfica, entonces las respuesta es C

PTS: 3 DIF: alta


REF: Camacho, I; Pancreatitis aguda, en Urgencias Quirúrgicas, 2da edición, Indugraf, ISBN
9789942035042, 2011.
OBJ: Tratar al paciente quirúrgico con diagnóstico de pancreatitis
TOP: Vesícula, v{ia biliar y p{ancreas KEY: TAC, Balthazar
NOT: Dr. Vinicio Moreno Rueda
24. ANS: D
El manejo adecuado de una pancreatitis se basa en los criterios de los consensos de Atlanta, en el
APACHE ll, dentro de los cuales se recomienda la cirugía en forma tardía, siendo una de sus indicaciones
el deterioro progresivo de paciente a pesar del buen tratamiento instaurado; recomendándose la
necrosectomía laparoscópica o por técnica abierta de ULM. La respuesta es D

PTS: 3 DIF: alta


REF: Uretsky, G; Chaildhood Pancreatitis, Am Fam Physican, 1999; 59: 2507-12
OBJ: Tratar al paciente quirúrgico con diagnóstico de pancreatitits
TOP: Vesícula, v{ia biliar, páncreas KEY: pancreatitis, APACHE II
NOT: Dr. Vinicio Moreno Rueda
25. ANS: C
Al tener un sangrado activo, un paciente en shock hipovolémico, un diagnóstico endoscópico que señala
sangrado activo en sábana, se someter a cirugía, manteniendo como es lógico el resto de componentes del
tratamiento como coadyuvantes, son indicaciones quirúrgicas en este caso hemorragia de más de 1000 cc,
hcto inferior a 28, necesidad de haber recibido 1500cc de paquetes globulares, shock persistente,
ancianos, grupo sanguíneo raro y hemorragia recurrente. La respuesta es C

PTS: 3 DIF: alta


REF: Hoskins, S; duodenal ulcer healing by eradication of H pylori withaut antiacid treatment:
randomized controlled trial. Lancet 2004. 343, 508-510.
OBJ: Tratar al paciente quirúrgico con diagnóstico de abdomen agudo
TOP: Abdomen agudo KEY: sangrado digestivo, endoscopia, Forrest
NOT: Dr. Vinicio Moreno Rueda
26. ANS: C
Dentro de la presentación clínica de la enfermedad diverticular del colon sintomática complicada se
encuentra la diverticulitis que puede cursar con plastrón, absceso, o complicarse con perforación,
hemorragia digestiva baja, fístulas u obstrucción colónica; todo lo anterior en un paciente anciano. La
respuesta es C

PTS: 3 DIF: alta


REF: Belén, E; Laparoscopyc surgery in colorectal disease anales, vol 28, suplemento 3; 2005.
OBJ: Tratar al paciente quirúrgico con diagnóstico de abdomen agudo
TOP: Abdomen agudo KEY: diverticulitis, abdomen agudo
NOT: Dr. Vinicio Moreno Rueda

2
ID: A

27. ANS: B
Luego de hacer el diagnóstico de peritonitis primaria en base a parámetros clínico, estudio de líquido
ascítico es mandataria una rápida y adecuada antibioticoterapia endovenosa, recomendado ampicilina y
aminoglucósidos. La respuesta es B

PTS: 3 DIF: alta


REF: Palacios, J; Peritonitis; Urgencias Quirúrgicas 2da edición, Indugraf, ISBN 9789942035042, 2011
OBJ: Tratar al paciente quirúrgico con diagnóstico de infección intra abdominal y sus complicaciones
TOP: Infección intra-abdominal KEY: peritonitis, antibiótico
NOT: Dr. Vinicio Moreno Rueda
28. ANS: D
Para prevenir la hipotermia, acidosis y coagulopatía en el paciente grave con trauma abdominal; se debe
realizar la cirugía de control de daños, la cual consta de tres tiempos: de los cuales el primero y el último
son quirúrgicos y el intermedio se lo realiza en UCI. La respuesta es D.

PTS: 3 DIF: alta


REF: Rotondo, M; Damage control: an approach for improved survival in exsanguinating abdominal
injury. Journal of trauma. 1993; 35. 375-383.
OBJ: Tratar al paciente quirúrgico con diagnóstico de abdomen agudo
TOP: Abdomen agudo KEY: Trauma abdominal, control de daños
NOT: Dr. Vinicio Moreno Rueda
29. ANS: B
Patología en países en vías de desarrollo, más frecuente sobre los 50 años y en hombres, presenta un
cuadro de oclusión intestinal baja de ahí la gran distensión y el vómito tardío. Frecuente en nuestro
medio. La respuesta es B

PTS: 3 DIF: alta


REF: Moreno V; Vólvulo de Sigma: Urgencias Quirúrgicas 2da edición, Indugraf, ISBN
9789942035042, 2011. OBJ: Tratar al paciente con diagnóstico de abdomen agudo
TOP: Abdomen agudo KEY: obstrucción intestinal, vólvulo de sigma
NOT: Dr. Vinicio Moreno Rueda
30. ANS: D
Los grados de presión intrabadominal medido a través de la vejiga son de 4 grados, el grado lll va de 26 a
35mmHg, siendo indicado reanimar en forma agresiva al paciente y una descompresión quirúrgica del
abdomen. La respuesta es D

PTS: 3 DIF: alta


REF: Platell, C; The omentum. Wordl J Gastroenterology 2000; 6: 169-176.
OBJ: Tratar al paciente quirúrgico con diagn´sotico de abdomen agudo
TOP: Abdomen agudo KEY: PIA, hipertensión abdominal
NOT: Dr. Vinicio Moreno Rueda
31. ANS: C
La cirugía contaminada se ve en los casos de heridas traumáticas menores a 6 horas de evolución,
apertura del tracto biliar o genitourinario, en presencia de bilis u orina, contaminación importante por el
contenido del tubo digestivo, ruptura de la asepsia importante o intervención en presencia de inflamación
aguda sin pus. La respuesta es C

PTS: 3 DIF: alta


REF: Bather, GR: Direct observations of surgical wound infections al a comprehensive center. Arch
Surg 2003; 130:1042-1047.
OBJ: Tratar al paciente quirúrgico con diagnóstico con inpección del sitio operatorio
TOP: infección intra-abdominal KEY: herida quirúrgica
NOT: Dr. Vinicio Moreno Rueda

3
ID: A

32. ANS: A
El Lavado peritoneal es un examen muy útil para el diagnóstico del trauma abdominal, incluso en el de
tipo abierto, en el que un contaje superior a los 10.000 glóbulos rojos por mm cúbico es indicativo de
positividad e indica que posiblemente se debe someter a cirugía al paciente. La respuesta es A

PTS: 3 DIF: alta


REF: Puylaert, JB; us of acute GI tract conditions. Eur. Radiol. 2001; 11:1167-77.
OBJ: Tratar al paciente quirúrgico con diagnóstico de abdomen agudo
TOP: Andomen agudo KEY: lavado peritoneal diagnóstico
NOT: Dr. Vinicio Moreno Rueda
33. ANS: A
En el eco no se evidencia engrosamiento de la pared vesicular, el diámetro transverso está en el tamaño
considerado normal, lo que se evidencia es la presencia de cálculos. Es una paciente no emergente que
debe ser aliviada su molestia y programada por consulta externa la colecistectomía. La respuesta es A

PTS: 3 DIF: alta


REF: Pérez, B; Abdomen Agudo: Urgencias Quirúrgicas 2da edición, Indugraf, ISBN 9789942035042,
2011. OBJ: Tratar al paciente quirúrgico con diagnóstico de patología de vesícula biliar
TOP: Vesícula, vía biliar y páncreas KEY: ecosonografía de vesícula
NOT: Dr. Vinicio Moreno Rueda
34. ANS: D
El mejor esquema seguro y sin resistencia alta es el dado por las cefalosporinas de tercera generación por
su efecto contra bacteria gram negativas tipo E. coli más metronidazol con su cobertura para anaerobios
con el B frágilis. L respuesta es D

PTS: 3 DIF: alta


REF: Molly, A; Amebic liver absces infectous disease clinics of North america Vol 14 Num 3 set 2000.
OBJ: Tratar al paciente quirúrgico con diagnóstico de infección intra-abdominal y sus complicaciones
TOP: Infección intra-abdominal KEY: peritonitis, antibiótico
NOT: Dr. Vinicio Moreno Rueda
35. ANS: B
La hernia inguinal indirecta aparece por el orificio inguinal profundo, a menudo aumentado de tamaño y
baja junto con los elementos del cordón espermático dentro de su saco herniario dentro del conducto
inguinal. La respuesta es B

PTS: 3 DIF: alta


REF: Wantz, G; Hernias de la Pared Abdominal: Princios de Cirugía, 6ta edición, cap. 34, Ed.
Interamericana - McGraw-Hill.
OBJ: Tratar al pacente quirúrgico diagnosticado con hernia de la pared abdominal
TOP: Hernias de la pared abdominal KEY: hernia inguinal
NOT: Dr. Vnicio MOreno Rueda
36. ANS: B
Es una inflamación o irritación del peritoneo, el tejido delgado que recubre la pared interna del
abdomen y la mayor parte de los órganos abdominales

PTS: 3 DIF: ALTA


REF: SINCHINARO. (2017). CIRUGIA GENERAL. Argentina: surgeryforum.
OBJ: Reconocer los principales conceptos y patologias relacionados a la cirugia
TOP: CIRUGIA KEY: Abdomen Agudo NOT: DR. VICTOR QUISIGUIÑA

4
ID: A

37. ANS: D
Por qué todas estas enfermedades pueden ser una causa de una perforación tífica debido a su
etiología y su forma de presentación.

PTS: 3 DIF: ALTA


REF: Gasman, O. A. (2014). Urgencias Quirurgicas. Quito: Biblioteca de salud publica.
OBJ: Reconocer los principales conceptos y patologias relacionadas a la cirugia
TOP: CIRUGIA KEY: Abdomen Agudo NOT: DR. VICTOR QUISIGUIÑA
38. ANS: D
La forma de presentación de la CI es heterogénea y varía en función de la causa y la extensión de
la oclusión vascular, el tiempo que el colon permanece privado del flujo vascular, el grado de
circulación colateral y las comorbilidades asociadas. En su forma más típica, el paciente presenta
un cuadro de dolor abdominal cólico, a menudo referido al hemiabdomen izquierdo, de
intensidad moderada, seguido de urgencia por la defecación y por la emisión de heces mezcladas
con sangre de color rojo brillante u oscura en las 12-24 h siguientes a la presentación del dolor

PTS: 3 DIF: ALTA REF: Dr. Carlos Téllez. (2014). Aparato digestivo. AMIR,
186.
OBJ: Reconocer los principales conceptos y patologias relacionadas a la cirugia
TOP: CIRUGIA KEY: ABDOMEN AGUDO INFLAMATORIO
NOT: DR.VICTOR QUISIGUIÑA
39. ANS: C
La sospecha de IMA es indicación de arteriografía mesenterio, que es el estudio de diagnóstico
definitivo, y debe realizarse proyecciones anteroposterior y lateral (para visualizar el origen del
tronco celiaco de la AMS)

PTS: 3 DIF: ALTA


REF: Soto S. (2004 - 2005). Isquemi mesenterica cronica o angina intestinal. Manual CTO digestivo,
69.
OBJ: Reconocer los principales conceptos y patologías relacionadas a la cirugía
TOP: CIRUGIA KEY: Isquemia Mesenterica NOT: DR. VICTOR QUISIGUIÑA
40. ANS: A
El único tratamiento definitivo es la revascularización quirúrgica mediante by-pass con injerto de
dacrón o vena safena o endarterectomía

PTS: 3 DIF: ALTA


REF: Soto S. (2004 - 2005). Enfermedades vasculares intestinales . Manual CTO Digestivo, 289.
OBJ: Reconocer los principales conceptos y patologías relacionadas a la cirugía
TOP: CIRUGIA GENERAL KEY: ENFERMEDADES VASCULARES
NOT: DR. VICTOR QUISIGUIÑA
41. ANS: C
En la colitis isquémica transitoria la más frecuente, los síntomas son leves y se controlan bien con
medicas conservadoras

PTS: 3 DIF: ALTA


REF: Soto S. (2004 - 2005). Enfermedades vasculares intestinales . Manual CTO Digestivo, 289.
OBJ: Reconocer los principales conceptos y patologías relacionadas a la cirugía
TOP: CIRUGIA KEY: ABDOMEN VASCULAR NOT: DR. VICTOR QUISIGUIÑA

5
ID: A

42. ANS: A
Los aneurismas son dilataciones localizadas y permanentes de un vaso de por lo menos 1.5 veces
el diámetro normal del mismo. La aorta es el vaso más frecuente implicado, siendo el 80% de los
aneurismas aórticos de localización infrarrenal. La evolución natural es hacia el crecimiento,
siendo la ruptura la complicación más grave y muchas veces mortal. La cirugía es la única actitud
terapéutica definitiva

PTS: 3 DIF: ALTA REF: Cirugia Vascular . (2005). MIR , 94.


OBJ: Reconocer los principales conceptos y patologías relacionadas a la cirugía
TOP: CIRUGIA KEY: ABDOMEN VASCULAR NOT: DR. VICTOR QUISIGUIÑA
43. ANS: D
La tomografía computarizada craneal es la menos indicada porque las lesiones del paciente son
más a nivel de tórax y abdomen, otro dato importante es que este no tiene ningún trauma a nivel
craneal

PTS: 3 DIF: ALTA REF: SOTO. S. (2005). HERNIAS. CTO.


OBJ: Reconocer los principales conceptos y patologías relacionadas a la cirugía.
TOP: CIRUGIA KEY: ABDOMEN TRAUMATICO NOT: DR.VICTOR QUISIGUIÑA
44. ANS: C
La rotura espontánea es rara en el bazo normal. Ésta suele afectar a bazos patológicos,
habiéndose descrito por traumatismos mínimos e incluso espontáneos en distintas entidades,
sobre todo aquellas que cursan con esplenomegalia

PTS: 3 DIF: ALTA REF: SOTO.S. (2005). OBSTRUCCIÓN INTESTINAL.


CTO.
OBJ: Reconocer los principales conceptos y patologías relacionadas a la cirugía
TOP: CIRUGIA KEY: ABDOMEN OBSTRUCTIVO NOT: DR. VICTOR QUISIGUIÑA
45. ANS: C
Para suturar las Heridas Simples de la Piel, se puede utilizar Hilo de Poliamida. (NYLON /
DAFILON M.R.). Es recomendable en cirugía plástica y reparadora como en oftalmológica. Es
IRREABSORBIBLE, Monofilamento, Sintético, Azul / Negro Alta Fuerza tensil, Inerte.

PTS: 3 DIF: ALTA REF: (MARIO, MANUAL DE CIRUGIA, 2004)


OBJ: Reconocer los principales conceptos y principales patologías relacionados a la cirugia
TOP: Cirugia NOT: Dr. Victor Quisiguiña
46. ANS: D
ACIDO POLIGLICOLICO. (DEXON / SAFIL M. R.) PROPIEDADES: Multifilamento
trenzado, fuerza tensil dura de 14 a 21 días. REABSORBIBLE: degradación por hidrólisis entre
60 y 90 días. INDICACIONES: Cirugía General, gastroenterología. Urología, ginecología,
oftalmología cirugía plástica y pediatría. Se esteriliza en Óxido de etileno.

PTS: 3 DIF: ALTA REF: (MARIO, MANUAL DE CIRUGIA, 2004)


OBJ: Reconocer los principales conceptos y patologias relacionadas ala cirugia
TOP: Cirugia NOT: Dr. Victor Quisiguiña

6
ID: A

47. ANS: C
La desinfeccion se realiza con antisépticos de uso tópico, de preferencia no irritantes, por
ejemplo, Iodopovidona, DG6 o alcohol aplicándolo con una gasita estéril suavemente sobre piel
sana, en el contorno de la herida (que el antiséptico no haga contacto con los bordes de la lesión).
En esta zona se hará la primera punción. No tocar los bordes, es la zona más sensible al dolor.

PTS: 3 DIF: ALTA REF: (MARIO, MANUAL DE CIRUGIA, 2004)


OBJ: Reconocer los principales conceptos y patologias relacionadas a la cirugia
TOP: Cirugia NOT: Dr. Victor Quisiguiña
48. ANS: B
LINEAS DE LANGER: Estas líneas fueron diseñadas por el autor en 1861 con el objetivo de
mejorar la estética de la cicatriz. Los estudios lo realizó en cadáveres, obviamente sin tener en
cuenta la dinámica muscular como lo hizo Kraissl en 1951. Estas líneas aseguran una buena
aposición de los bordes de la herida cuando éstas son superficiales (dérmicas) no cuando llegan la
tics.

PTS: 3 DIF: ALTA REF: (MARIO, MANUAL DE CIRUGIA, 2004)


OBJ: Reconocer los principales conceptos y patologia relacionadas a la cirugia
TOP: Cirugia NOT: Dr Victor Quisiguiña
49. ANS: B
En la profilaxis antibiotica se debe administrar de 30 minutos a 1 hora previa a la insicion quirurgica,

PTS: 3 DIF: ALTA


REF: Principios de Cirugía de Schwartz 10ma edición capitulo 6 (Infecciones
OBJ: Reconocer los principales conceptos y patologias relacionadas a la cirugia
TOP: Cirugia NOT: Dr. Victor Quisiguiña
50. ANS: C PTS: 3 DIF: ALTA
REF: Digestivo y Cirugía General AMIR tercera edición, pagina 88.
OBJ: Reconocerlos principales conceptos y patologias relacionadas a la cirugia
TOP: CIRUGIA KEY: Infeccion sitio quirurgico NOT: Dr. Victor Quisiguiña
51. ANS: B PTS: 3 DIF: ALTA
REF: Principios de Cirugía de Schwartz 10ma edición capitulo 6 (Infecciones
OBJ: Reconocerlos principales conceptos y patologias relacionadas a la cirugia
TOP: CIRUGIA KEY: Infeccion sitio quirurgico NOT: Dr. Victor Quisiguiña
52. ANS: C PTS: 3 DIF: ALTA
REF: Principios de Cirugía de Schwartz 10ma edición capítulo 6 (Infecciones
OBJ: Reconocerlos principales conceptos y patologias relacionadas a la cirugia
TOP: CIRUGIA KEY: Infecciones sitio quirurgico NOT: Dr. Victor Quisiguiña
53. ANS: A

La peritonitis primaria o espontánea se produce por invasión bacteria via Hematogena y linfática

PTS: 3 DIF: ALTA REF: CTO Séptima edición /Medicina y cirugía / Pag 291
OBJ: Reconocerlos principales conceptos y patologias relacionadas a la cirugia
TOP: CIRUGIA KEY: Peritonitis NOT: Dr. Victor Quisiguiña
54. ANS: B PTS: 3 DIF: ALTA
REF: Principios de Cirugía de Schwartz 10ma edición capitulo 6 (Infecciones
OBJ: Reconocerlos principales conceptos y patologias relacionadas a la cirugia
TOP: CIRUGIA KEY: Heridas limpias NOT: Dr. Victor Quisiguiña

7
ID: A

55. ANS: C
Queloide: cicatriz exuberante por exceso de colágeno, que rebasa los límites de la piel sana. Se
considera un tumor benigno. Son más frecuente en la región pree esternal y espalda, así como en
sujetos de raza negra.

PTS: 3 DIF: Alta REF: (Maria Rodriguez, 2014)


OBJ: Reconcer los principales conceptos y patologias relacionadas a la cirugia
TOP: Cirugia KEY: Cicatrización NOT: Dr. Victor Quisiguiña
56. ANS: C
La reparación de las heridas presenta una serie de cambios químicos, morfológicos y físicos que
dan como resultado la formación del tejido cicatricial: Fase inflamatoria, Epitelización, Fase
celular o de neo formación vascular, Fase proliferativa y de síntesis de colágeno, Fase de
remodelado

PTS: 3 DIF: Alta REF: (Maria Rodriguez, 2014)


OBJ: Reconocer los principales conceptos y patologias relacionadas con la cirugia
TOP: Cirugia KEY: cicatrización NOT: Dr. Victor Quisiguiña
57. ANS: D
Factores que afectan a la cicatrización de heridas:
Sistémicos
 Edad, nutrición, traumatismos, enf. Metabólicas, inmunodepresión, trastornos del
tejido conjuntivos, tabaquismo.
Locales
 lesión mecánica, infección, edema, isquemia/necrosis de tejido, agentes tópicos,
radiación ionizante, tensión de oxigeno baja, cuerpos extraños.

PTS: 3 DIF: Alta REF: (Charles Brunicardi, 2015)


OBJ: Reconocer los principales conceptos y patologias relacionadas a la cirugia
TOP: Cirugia KEY: cicatrización NOT: Dr. Victor Quisiguiña
58. ANS: D
Queloide: incidencia rara, predisposición genética, no hay regresión, contractura rara.
Cicatriz hipertrófica: incidencia frecuente, sin predilección por grupos étnicos, sin predisposición
genética, regresión frecuente y espontánea, contractura frecuente.

PTS: 3 DIF: Alto REF: (Charles Brunicardi, 2015)


OBJ: Reconocer los principales conceptos y palabras relacionadas a la cirugia
TOP: Cirugia KEY: Queloides y Cicatrices NOT: Dr. Victor Quisiguiña

8
ID: A

59. ANS: B
Desde el punto de vista didáctico los materiales de sutura se clasifican atendiendo su origen o
algunas propiedades específicas.
Así tenemos materiales
- Inorgánicos; acero inoxidable, agrafes.
- Orgánicos; sedas, catgut, lino, algodón.
- Sintéticos; nylon, polipropileno, polietileno, poligaláctico, lino.

PTS: 3 DIF: Alta REF: (MARIO, MANUAL DE CIRUGIA, 2004)


OBJ: reconocer los principales conceptos y palabras relacionadas a la cirugia
TOP: Cirugia KEY: materiales de sutura NOT: Dr. Victor Quisiguiña
60. ANS: D
Una complicación frecuente después de la hemorroidectomía es la retención urinaria y ocurre en
10 a 50% de los casos. El dolor también puede ocasionar retención fecal. También es posible que
después de la hemorroidectomía suceda un sangrado masivo.

PTS: 3 DIF: alta


REF: SCHWARTZ. (2015). Principios de Cirugía. Mexico: Mc Graw Hill .
OBJ: reconocer los principales conceptos y palabras relacionadas a la cirugia
TOP: cirugia KEY: hemorroidectomía NOT: Dr. Victor Quisiguiña
61. ANS: D
Los abscesos isquiorrectales simples son los que se drenan a través de una incisión de la piel
suprayacente

PTS: 3 DIF: alto


REF: SCHWARTZ. (2015). Principios de Cirugía. Mexico: Mc Graw Hill .
OBJ: reconocer los principales conceptos y patologias relacionados a la cirugia
TOP: cirugia KEY: absceso perianal NOT: Dr. Victor Quisiguiña
62. ANS: C
Estos abscesos suelen ser muy superficiales, por lo que con frecuencia es posible practicar este
procedimiento en el consultorio, la clínica o el departamento de urgencias bajo anestesia local.

PTS: 3 DIF: alta


REF: SCHWARTZ. (2015). Principios de Cirugía. Mexico: Mc Graw Hill .
OBJ: reconocer los principales conceptos y patologias relacionados a la cirigia
TOP: cirugia KEY: quiste pilonidal NOT: Dr. Victor Quisiguiña.
63. ANS: D
Se han propuesto varios procedimientos para tratar un seno pilonidal crónico. El método más
simple consiste en destechar el trayecto, raspar la base y marsupializar la herida. A continuación,
debe conservarse limpia la herida y sin pelo hasta que cicatrice por completo. Otra opción
consiste en practicar una incisión lateral pequeña y extirpar
el foso.

PTS: 3 DIF: alta


REF: SCHWARTZ. (2015). Principios de Cirugía. Mexico: Mc Graw Hill .
OBJ: reconocer los principales conceptos y patologias relacionasos a la cirugia
TOP: cirugia KEY: quiste pilonidal NOT: Dr. Victor Quisiguiña

9
ID: A

64. ANS: D
Los trayectos fistulosos circundan tramos variables del complejo esfinteriano, por lo tanto el
tratamiento quirúrgico depende de la localización de las aberturas interna y externa y el trayecto
de la fístula.

PTS: 3 DIF: alta


REF: SCHWARTZ. (2015). Principios de Cirugía. Mexico: Mc Graw Hill .
OBJ: reconocer los principales conceptos y patologias relacionadas a la cirugia
TOP: cirugia KEY: fistula anal NOT: Dr. Victor Quisiguiña
65. ANS: B
Por primera intención: sutura inmediata de la herida. En heridas con mínima contaminación
bacteriana, hemorragia controlable, y sin tejido necrótico ni cuerpos extraños

PTS: 3 DIF: Alta REF: (Maria Rodriguez, 2014)


OBJ: reconocer los principales conceptos y patologias relacionadas a la cirugia
TOP: Cirugia KEY: tratamiento de las heridas NOT: Dr. Victor Quisiguiña
66. ANS: D
Principales manifestaciones clínicas de la pancreatitis aguda y su frecuencia: dolor abdominal
90%, Vómitos 50%, Ileo reflejo 30%, Estado de shock 30%, Disnea 10%, Oliguria o anuria
10-15%, Signos neurológicos 5%, signos de infección 4% y Hemorragia 3%.

PTS: 3 DIF: alto REF: (LEVY, 2015)


OBJ: Reconocer los principales conceptos y patologías relacionadas a la cirugía
TOP: CIRUGIA KEY: vómito NOT: DR. Victor Quisiguiña
67. ANS: D
La coledocolitiasis debe tratarse con CPRE, reservándose la cirugía a los fallos de esta La cpre permitirá
realizar una conlangiografía retrógada endoscópica que permitirá realizar una colangiografía, la
extracción de cálculos y esfinterotomía endoscópica

PTS: 3 DIF: Alto


REF: ORTEGA, M. R. (2014). CTO 9na Edición manuela de Medicina y Cirugía. Mexico.
OBJ: Reconocer los principales conceptos y patologías relacionadas a la cirugía
TOP: Cirugía KEY: coledocolitiasis NOT: DR. Victor Quisiguiña
68. ANS: A
El tratamiento de la coledocolitiasis consiste en la extracción de cálculos y colecistectomía. Si el
paciente tiene una historia de ictericia obstructiva o pancreatitis, alteración de las pruebas
hepáticas o un cálculo coledociano con dilatación de la vía biliar, es preferible realizar una
CPRE, con esfinterotomía y extracción de cálculos y posteriormente una colecistectomía
laparoscópica. También puede explorarse la vía biliar durante la colecistectomía y realizar la
CPRE tras la colecistectomía, si es necesario. En caso de que no se hayan extraído todos los
cálculos (empedramientos coledocianos o litiasis intrahepáticas) puede ser preciso realizar una
derivación biliodigestiva.

PTS: 3 DIF: Alta


REF: FARRERAS, V., & ROZMAN, C. (2016). MEDICINA INTERNA. BARCELONA: ELSEVIER.
OBJ: Reconocer los principales conceptos y patologías relacionadas con cirugía
TOP: Cirugía KEY: Colicistectomía NOT: DR. Victor Quisiguiña

10
ID: A

69. ANS: C
En el caso de coledocolitiasis, tras iniciar un tratamiento médico con antibióticos y fluidoterapia, si la
evolución es favorable se debe realizar una colangiografía retrógrada endoscópica retrógrada con
esfinterotomía para extraer la litiasos con una cestilla endoscópica

PTS: 3 DIF: Alta REF: ARREOS, M. (2012). AMIR. MADRID: MARBAN.


OBJ: Reconocer los principales conceptos y patologías relacionadas con cirugía
TOP: Cirugía KEY: litiasis NOT: DR. Victor Quisiguiña
70. ANS: D
Los cálculos del colédoco pueden cursar asintomáticos o presentar Còlico biliar, ictericia bstructiva,
colangitis ascendente y pancreatitis. Otras complicaciones menos común es la cirrosis biliar secundaria.
Ante un cuadro de colangitis o ictericia en un paciente colecistectomizado, hay que pensar siempre en
coledocolitiasis residual.

PTS: 3 DIF: Alto


REF: ORTEGA, M. R. (2014). CTO 9na Edición manuela de Medicina y Cirugía. Mexico.
OBJ: Reconocer los principales conceptos y patologías relacionadas con cirugía
TOP: Cirugía KEY: Coledocolitiasis NOT: DR. Victor Quisiguiña
71. ANS: A
El hallazgo más común es una masa o abultamiento en la pared anterior del abdomen que
aumenta de tamaño con la maniobra de Valsalva.

PTS: 3 DIF: ALTA REF: (SCHAWARTZ, 2010)


OBJ: Reconocer los principales conceptos y patologias relacionadas a la cirugia
TOP: CIRUGIA KEY: Hernia abdominal NOT: Dr. Victor Quisiguiña
72. ANS: D
Casi todas las hernias umbilicales congénitas se cierran de manera espontánea alrededor de los
cinco años de edad.

PTS: 3 DIF: ALTA REF: (SCHAWARTZ, 2010)


OBJ: Reconocer los principales conceptos y patologias relacionadas a la cirugia
TOP: CIRUGIA KEY: Hernias umbilicales NOT: Dr. Victor Quisiguiña
73. ANS: A
Factores que pueden contribuir son obesidad, defectos primarios de cicatrización de heridas,
práctica previa de múltiples métodos, hernias posquirúrgicas previas y errores técnicos durante la
reparación

PTS: 3 DIF: ALTA REF: (SCHAWARTZ, 2010)


OBJ: Reconocer los principales conceptos y patologias relacionadas a la cirugia
TOP: CIRUGIA KEY: Hernia postoperatoria NOT: Dr. Victor Quisiguiña

11
ID: A

74. ANS: C
Las hernias de la pared anterior del abdomen, o hernias ventrales, representan defectos de la
aponeurosis y los músculos de la pared abdominal a través de los cuales puede salir contenido
intraabdominal o preperitoneal. Las hernias ventrales pueden ser congénitas o adquiridas. Estas
últimas se forman por un deterioro lento de la estructura de las aponeurosis musculares o se
desarrollan por fracaso de la cicatrización de una incisión en la pared anterior del abdomen
(hernia incisional). El hallazgo más común es una masa o abultamiento en la pared anterior del
abdomen que aumenta de tamaño con la maniobra de Valsalva

PTS: 3 DIF: ALTA REF: (SCHAWARTZ, 2010)


OBJ: Reconocer los principales conceptos y patologias relacionadas a la cirugia
TOP: CIRUGIA KEY: Hernias abdominales NOT: Dr. Victor Quisiguiña
75. ANS: E
Las mallas absorbibles se degradan y eliminan desde los tejidos y pierden poco a poco la
integridad estructural necesaria para el soporte de los tejidos.

PTS: 3 DIF: ALTA REF: (SCHAWARTZ, 2010)


OBJ: Reconocer los principales conceptos y patologias relacionadas a la cirugia
TOP: CIRUGIA KEY: Mallas NOT: Dr. Victor Quisiguiña
76. ANS: C
Las infecciones del sitio quirúrgico se clasifican en infecciones insicionales (Infecciones de piel y
tejido subcutáneo e infecciones profundas) e infecciones de órgano/ espacio.

PTS: 3 DIF: ALTA


REF: Principios de Cirugía de Schwartz 10ma edición capítulo 6 (Infecciones
OBJ: RECONOCER LOS PRINCIPALES CONCEPTOS Y PALABRAS RELACIONADAS A ESTA
PATOLOGÍA TOP: CIRUGÍA KEY: INFECCIÓN
NOT: DR. VICTOR QUISIGUIÑA
77. ANS: A
La peritonitis primaria o espontánea se produce por invasión bacteria via Hematogena y linfática

PTS: 3 DIF: ALTA REF: CTO Séptima edición /Medicina y cirugía / Pag 291
OBJ: RECONOCER LOS PRINCIPALES CONCEPTOS Y PALABRAS RELACIONADAS A ESTA
PATOLOGÍA TOP: CIRUGÍA KEY: PERITONITIS
NOT: DR. VICTOR QUISIGUIÑA
78. ANS: D
La causa más frecuente es la reactivación de un foco peritoneal (secundario a una diseminación
hematógena o por rotura de una adenopatía mesentérica afecta). Más frecuente en mujeres. Es la
tercera causa de ascitis en nuestro medio, tras la cirrótica y la neoplásica.

PTS: 3 DIF: ALTA REF: (AMIR, digestivo y cirugía general, 3 edición)


OBJ: RECONOCER LOS PRINCIPALES CONCEPTOS Y PALABRAS RELACIONADAS A ESTA
PATOLOGÍA TOP: CIRUGÍA KEY: PERITONITIS TUBERCULOSA
NOT: DR. VICTOR QUISIGUIÑA

12
ID: A

79. ANS: D
Sintomas Subdiafragmáticos: disnea, tos, dolor en hombro izquierdo o derecho e hipo por
irritación diafragmática.

PTS: 3 DIF: ALTA REF: (AMIR, digestivo y cirugía general, 3 edición)


OBJ: RECONOCER LOS PRINCIPALES CONCEPTOS Y PALABRAS RELACIONADAS A ESTA
PATOLOGÍA TOP: CIRUGÍA KEY: PERITONITIS
NOT: DR. VICTOR QUISIGUIÑA
80. ANS: A
Debido a que la flora bacteriana que afecta es mixta anaerobia y aerobia; los grampositivos son
caracteristicos de la perforación gástrica, gramnegativos y anaerobios de perforación apendicular
y del colon.

PTS: 3 DIF: ALTA


REF: Manual CTO de Medicina y Cirugía 8.a edición Digestivo y cirugía pag 97
OBJ: RECONOCER LOS PRINCIPALES CONCEPTOS Y PALABRAS RELACIONADAS A ESTA
PATOLOGÍA TOP: CIRUGIA KEY: PERITONITIS SECUNDARIA
NOT: DR. VICTOR QUISIGUIÑA
81. ANS: B
Se define fístula enterocutánea a la comunicación anormal entre el aparato gastrointestinal y la
piel, con salida del contenido intestinal a través de la misma por un periodo mayor de 24 horas.

PTS: 3 DIF: Alta REF: (Martínez-Ordaz, 2002)


OBJ: Reconocer los principales conceptos y patologías relacionados a la cirugía
TOP: CIRUGÍA KEY: Fístula Enterocutánea NOT: Dr. Victor Quisiguiña
82. ANS: D
Las tres complicaciones principales de los pacientes con fistulas son desequilibrio
hidroelectrolítico, desnutrición y sepsis, las cuales están en relación con la
localización de la fistula, el gasto, las características bioquímicas y electrolíticas de la
descarga y la condición patológica subyacente.

PTS: 3 DIF: Alta REF: (Martínez-Ordaz, 2002)


OBJ: Reconocer los principales conceptos y palabras relacionadas a la cirugía
TOP: CIRUGÍA KEY: complicaciones de fístulas NOT: Dr. Victor Quisiguiña
83. ANS: B
En la tabla 39.1 se reflejan las principales etiologías implicadas en la génesis de las fistulas
enterocutáneas.

PTS: 3 DIF: Alta REF: (PARICIO)


OBJ: Reconocer los principales conceptos y palabras relacionadas a la cirugía
TOP: CIRUGÍA KEY: fistulas enterocutaneas, fistulas postoperatorias
NOT: Dr. Victor Quisiguiña
84. ANS: D PTS: 3 DIF: Alta REF: (Joseph, 2012)
OBJ: Reconocer los principales conceptos y palabras relacionadas a la cirugía
TOP: CIRUGÍA KEY: hernia inguinal estrangulada NOT: Dr. Victor Quisiguiña

13
ID: A

85. ANS: C
La sepsis es la complicación más temida en los pacientes con fistulas enterocutáneas.

PTS: 3 DIF: Alta REF: (Martínez-Ordaz, 2002)


OBJ: Reconocer los principales conceptos y palabras relacionadas a la cirugía
TOP: CIRUGÍA KEY: fistulas enterocutáneas NOT: Dr. Victor Quisiguiña
86. ANS: C
Hay abundantes canales de calcio tanto en el músculo cardíaco como el músculo liso. De hecho,
en algunos tipos de músculo liso apenas hay canales rápidos de sodio, de modo que los
potenciales de acción están producidos casi totalmente por la activación de los canales lentos de
calcio.

PTS: 3 DIF: Alto


REF: GUYRON, Arthur; Hall, Jhon: Tratado de fisiología medica 11° edición 2006; Elsevier España
OBJ: reconocer los principales conceptos y patologias relacionadas con la cirugia
TOP: cirugia KEY: canales de calcio NOT: Dr. Victor Quisiguiña
87. ANS: A
El nivel bajo de sodio sérico se produce cuando hay exceso de agua extracelular respecto
del sodio, por lo que puede producir aumento del gasto cardíaco.

PTS: 3 DIF: Alto


REF: SCHWARTZ. Principios de cirugía Mc Graw-Hill. Interamericana. Editores S.A. Séptima edición.
México 2003 OBJ: reconocer las principales conceptos y patologias relacionadas a la cirugia
TOP: Cirugia KEY: Hiponatremia NOT: Dr. Victor Quisiguiña
88. ANS: C
Los síntomas de hipernatremia son raros, a menos que la concentración sérica de sodio sea mayor
de 160 meq/L. Como los síntomas se deben a la hiperosmolaridad, predominan los efectos en el
sistema nervioso central.

PTS: 3 DIF: Alto


REF: SCHWARTZ. Principios de cirugía Mc Graw-Hill. Interamericana. Editores S.A. Séptima edición.
México 2003. pág. 35
OBJ: reconocer los principales conceptos y patologias relacionadas con la cirugia
TOP: Cirugia KEY: Hipernatremia NOT: Dr. Victor Quisiguiña
89. ANS: D
Los síntomas predominantes de la hiperpotasemia son gastrointestinales, neuromusculares y
cardiovasculares

PTS: 3 DIF: Alto


REF: SCHWARTZ. Principios de cirugía Mc Graw-Hill. Interamericana. Editores S.A. Séptima edición.
México 2003. pág. 36
OBJ: Reconocer las principales conceptos y patologias relacionadas con la cirugia
TOP: Cirugia KEY: hiperpotasemia NOT: Dr. Victor Quisiguiña

14
ID: A

90. ANS: B
Las causas de hipopotasemia incluyen ingestión insuficiente, excreción renal excesiva
(hiperaldosteronismo, medicamentos como diuréticos que incrementan la excreción de potasio, o
fármacos como la penicilina que promueven la pérdida tubular renal de potasio), pérdida en
secreciones gastrointestinales (pérdida directa de potasio en heces o pérdida renal de potasio por
vómito o gasto nasogástrico alto) o desviaciones intracelulares (como en la alcalosis metabólica o
el tratamiento con insulina).

PTS: 3 DIF: Alto


REF: SCHWARTZ. Principios de cirugía Mc Graw-Hill. Interamericana. Editores S.A. Séptima edición.
México 2003. pág. 36
OBJ: reconocer los principales conceptos y patologias relacionadas con la cirugia
TOP: Cirugia KEY: hipopotasemia NOT: Dr. Victor Quisiguiña
91. ANS: D
Es la que intenta satisfacer todas las necesidades calórico-proteicas, además de las
hidroelectrolíticas, de todo tipo de pacientes, incluidos los que padecen una agresión grave y una
desnutrición severa.

PTS: 3 DIF: Alto REF: Bosco Mera.2006. AFEME. Ecuador


OBJ: Reconocer los principales conceptos y patologias relacionadas a la cirugia
TOP: Cirugia KEY: Nutricion Parenteral NOT: Dr. Victor Quisiguiña
92. ANS: C
en las actualidad concoermos que la desnutricion hospitalaria nos lleva a mala cicaatrizacion de las
heridas quirurgicas, alteracion de los mecanismos inmunologicos de defensa, aumento de infecciones,
aumento de estancia hospitalaria, mayor mortalidad, lo que incide en aumento en costos hospitalatios

PTS: 3 DIF: Alto REF: ROCABRUNA, R. (13 de marzo de 2009). Scielo.


OBJ: renococer las principales conceptos y patologias relacionadas en la cirugia
TOP: Cirugia KEY: desnutricion NOT: Dr. Victor Quisiguiña
93. ANS: C
Ayuno: Acto de abstenerse total o parcialmente de comer o beber, a veces por un período de tiempo. Este
puede dividirse en:
• Simple: de 14 a 16 horas sin ingerir alimentos.
• Basal: de 24 a 48 horas sin ingerir alimentos.
• Prolongado: más de 72 horas.
• Inanición: más de 96 horas

PTS: 3 DIF: Alto REF: ROCABRUNA, R. (13 de marzo de 2009). Scielo.


OBJ: reconocer los principales conceptos y patologias relacionadas con la cirugia
TOP: Cirugia KEY: tiempo de ayuno NOT: Dr.Victor Quisiguiña

15
ID: A

94. ANS: B
La nutrición parenteral es la modalidad de terapia nutricional en la cual se administran soluciones de
elementos nutritivos en el torrente circulatorio.

PTS: 3 DIF: Alto


REF: MARTINEZ, S. (05 de noviembre de 2012). Nutricion en cirugia.
OBJ: reconocer los principales conceptos y patologias relacionadas en la cirugia
TOP: Cirugia KEY: la nutricion enteral NOT: Dr. Victor Quisiguiña
95. ANS: D
La necesidad basal de potasio es de 80mEg

PTS: 3 DIF: Alto REF: ROCABRUNA, R. (13 de marzo de 2009). Scielo.


OBJ: reconocer los principales conceptos y patologias relacionadas en cirugia
TOP: Cirugia KEY: necesidad basal NOT: Dr. Victor Quisiguiña
96. ANS: A
En las apendicitis el signo de Mc Burney es uno de los signos específicos e importante para poder
diagnosticar en la mediante el examen físico que es una apendicitis

PTS: 3 DIF: Alto REF: (Gasman, 2014)


OBJ: reconocer los principales conceptos y patologias relacionadas en la cirugia
TOP: Cirugia KEY: caso clinico NOT: Dr. Victor Quisiguiña
97. ANS: B
Se va a encontrar los leucocitos mayores a 12000 por presentar la infección y esta hace que sistema de
defensa se altere

PTS: 3 DIF: Alto REF: (Gasman, 2014)


OBJ: reconocer los principales coneptos y patolgogias relaciondas con la cirugia
TOP: Cirugia KEY: Caso clinico NOT: Dr. Victor Quisiguiña
98. ANS: D
Es un procedimiento para examinar los conductos biliares y se realiza a través de un endoscopio. Los
conductos biliares son las vías que llevan la bilis desde el hígado hasta la vesícula y el intestino delgado.
La CPRE se usa para tratar cálculos, tumores o áreas estrechas de los conductos biliares

PTS: 3 DIF: Alto REF: (SINCHINARO, 2017)


OBJ: reconocer los principales conceptos y patologias relacionadas con la cirugia
TOP: Cirugia KEY: Examen Gold estandar de pancreatitis
NOT: Dr. Victor Quisiguiña
99. ANS: A
En pacientes postoperados con una peritonitis secundaria que no responden a tratamientoy que
presentan fallo multiorganico o sepsis. Por perforación de una visera hueca.

PTS: 3 DIF: alta


REF: Manual CTO de Medicina y Cirugía 8.a edición Digestivo y cirugía pag 97
OBJ: reconocer los principales conceptos y patologias relacionados a la cirugia
TOP: cirugia KEY: peritonitis terciaria NOT: Dr. Victor Quisiguiña

16
ID: A

100. ANS: D
Los abscesos intrabdominales son una forma de peritonitis localizada que en su evolución pueden
ser locales vinculados a sigmoiditis aguda, apendicitis, colecistitis ulcera perforada, infeccion del
sitio quirurgico etc

PTS: 3 DIF: alta REF: (Galindo, 2009)


OBJ: reconoser los principales conseptos y patologias relacionadas a la cirugia
TOP: cirugia KEY: absceso intrabdominal NOT: Dr.Victor Quisiguiña
101. ANS: C
Hemoglobina y pigmentos biliares en cavidad peritoneal aumenta la proliferación bacteriana y los
pigmentos bacterianos disminuyen la tensión superficial del peritoneo.

PTS: 3 DIF: alta REF: (Suarez, 2004)


OBJ: reconoser los principales conseptos y patologias relacionadas a la cirugia
TOP: cirugia KEY: absceso intraabdominal NOT: Dr.Victor Quisiguiña
102. ANS: D
Según su localización
a) Intraperitoneal
- Visceral: hepático, esplénico, vesícula biliar, ovario, trompas y útero.
- De los espacios anatómicos existentes (supramesocólicos: subfrénico, subhepático y trascavidad
de los epiplones e inframesocólicos: interasas
- Del espacio de un órgano extirpado.
b) Retroperitoneal
- Visceral: riñón, páncreas.
- De los espacios anatómicos preexistentes
- Del espacio de un órgano extirpado.

PTS: 3 DIF: alta REF: (Garcia, 2014)


OBJ: reconoser los proncipales conseptos y patologias relacionadas a la cirugia
TOP: cirugia KEY: intraperitoneal NOT: Dr.Victor Quisiguiña
103. ANS: D
Las complicaciones comprenden:

 Reaparición del absceso

 Ruptura de un absceso

 Diseminación de la infección al torrente sanguíneo

 Infección generalizada en el abdomen

PTS: 3 DIF: alta REF: (Sanchez, 2012)


OBJ: reconoser los proncipales conseptos y patologias relacionadas a la cirugia
TOP: cirugia KEY: absceso intraabdominal NOT: Dr. Victor Quisiguiña

17
ID: A

104. ANS: A
La hernia inguinal incarcerada se manifiesta con bulto doloroso en la región inguinal y cuadro
obstructivo; se puede intentar la reducción de una hernia inguinal incarcerada bajo sedación
suave (diazepam IM o IV), pero nunca de una hernia crural, pues puede poner en riesgo el asa
intestinal.

PTS: 3 DIF: ALTO REF: (Rodríguez & Merino, 2014)


OBJ: RECONOCER LOS PRINCIPALES CONCEPTOS Y PALABRAS RELACIONADAS A ESTA
PATOLOGÍA TOP: CIRUGÍA KEY: HERNIA INGUINAL
NOT: DR. VICTOR QUISUGUIÑA
105. ANS: D
La hernia estrangulada se manifiesta con bulto doloroso, caliente, eritematoso o azulado, que
puede crepitar, en la región inguinal y obstrucción intestinal; puede acompañarse de fiebre,
leucocitosis y signos de sepsis.
Toda hernia estrangulada por el riesgo que conlleva reintroducir un segmento intestinal con
compromiso vascular, debe ser intervenida sin intentar reducirla.

PTS: 3 DIF: ALTO REF: (Rodríguez & Merino, 2014)


OBJ: RECONOCER LOS PRINCIPALES CONCEPTOS Y PALABRAS RELACIONADAS A ESTA
PATOLOGÍA TOP: CIRUGÍA KEY: HERNIA ESTRANGULADA
NOT: DR. VICTOR QUISIGUIÑA
106. ANS: A
La hernia directa protruye a través del suelo del canal inguinal a nivel del triángulo de
Hesselbach, que está formado por la fascia transversalis reforzada por fibras aponeuróticas del
músculo transverso del abdomen. Así pues, estas hernias no pasan a través del orificio profundo y
no se localizan por dentro de las fibras del cremáster, sino por detrás.

PTS: 3 DIF: ALTA REF: (Rodríguez & Merino, 2014)


OBJ: RECONOCER LOS PRINCIPALES CONCEPTOS Y PALABRAS RELACIONADAS A ESTA
PATOLOGÍA TOP: CIRUGÍA KEY: HERNIA DIRECTA
NOT: DR, VICTOR QUISIGUIÑA
107. ANS: D
Hernioplastia (reparación protésica): reparación de la hernia con material sintético. Actualmente
se realizan con mayor frecuencia, dados los excelentes resultados obtenidos (técnicas de
Lichtenstein, Rutkow, entre otros).

PTS: 3 DIF: ALTA REF: (Rodríguez & Merino, 2014)


OBJ: RECONOCER LOS PRINCIPALES CONCEPTOS Y PALABRAS RELACIONADAS A ESTA
PATOLOGÍA TOP: CIRUGÍA KEY: HERNIOPLASTIA
NOT: DR. VICTOR QUISIGUIÑA

18
ID: A

108. ANS: C
La biometría hemática y química sanguínea nos sirve para saber la cantidad tanto de hemoglobina
que tiene el paciente y el caso de esta estar baja reponer para compensar al paciente al igual que
electrolitos

PTS: 3 DIF: ALTA REF: SOTO. S. (2005). HERNIAS. CTO.


OBJ: Reconocer los principales conceptos y patologías relacionadas a la cirugia
TOP: CIRUGIA GENERAL KEY: Hernia NOT: DR. VICTOR QUISIGUIÑA
109. ANS: B
En general el traumatismo abierto es producido por elementos cortantes que dañan la continuidad
del tejido, a diferencia del cerrado que es lo contrario

PTS: 3 DIF: ALTA REF: SOTO. S. (2005). HERNIAS. CTO


OBJ: Reconocer los principales conceptos y patologias relacionados con la cirugía
TOP: CIRUGIA KEY: Hernias NOT: DR. VICTOR QUISIGUIÑA
110. ANS: C
Son aquellas que presentan un riesgo de necrosis por compromiso vascular e isquemia de la
víscera herniada. Normalmente el trastorno del riego acompaña a una incarceración pero no
siempre es así, como ocurre en la hernia de Richter (sólo se estrangula un sector limitado de la
porción antimesentérica) y en la hernia de Littre (la luz intestinal está permeable porque existe
una estrangulación del divertículo de Meckel). Es la complicación más grave de una hernia
pudiendo llegar a acaba con la vida del paciente

PTS: 3 DIF: ALTA REF: SOTO. S. (2005). HERNIAS. CTO.


OBJ: Reconocer los principales conceptos y patologías relacionadas con cirugia
TOP: CIRUGIA KEY: Hernias NOT: DR. VICTOR QUISIGUIÑA
111. ANS: D
Un pólipo es una protrusión macroscópica de la pared intestinal.

Polipectomía y seguimiento endoscópica: Benignos y malignízados intramucosos. Los pólipos


adenomatosos necesitan de más de cinco años de crecimiento antes de alcanzar expresión clínica
por lo que no es necesario repetir las colonoscopias antes de tres años después de la polipectomía.
Hay lesiones sincrónicas y metacronicas lo que obliga a una primera colonoscopia total tras
extirpación de un pólipo.

Si se ha extraído un adenoma avanzado se recomienda efectuar una colonoscopia a los 3 años,


por el contrario, si tan sola se detectaron l ó2 adenomas no avanzados el intervalo entre
exploraciones puede ser entre 5- 10 años.

PTS: 3 DIF: ALTA REF: SOTO.S. (2004). POLIPOS . CTO.


OBJ: Reconocer los conceptos y patologías relacionadas a la cirugia
TOP: CIRUGIA KEY: Polipos NOT: DR. VICTOR QUISIGUIÑA

19
ID: A

112. ANS: B
Ocurre obstrucción intestinal cuando hay interferencia en la progresión normal del contenido
intestinal. Se denomina obstrucción mecánica cuando existe una verdadera barrera física que
obstruye la luz intestinal. La localización más frecuente es el intestino delgado. Se caracteriza por
olor abdominal, vómitos (fecaloideos si es distal), distensión abdominal (mayor cuanto más distal
sea la obstrucción), hiperperistaltismo con ruidos metálicos.

PTS: 3 DIF: Alta REF: (SOTO.S, 2005)


OBJ: Reconocer los principales conceptos y patologías relacionadas a la cirugía
TOP: Abdomen agudo obstructivo KEY: Obstrucción intestinal
NOT: Dr. Victor Quisiguiña
113. ANS: C
El 90% de las obstrucciones de ID se resuelven con aNG y reposición hidroelectrolítica. Es
necesaria la intervención quirúrgica cuando sospechamos estrangulación, el dolor y la fiebre
aumentan, y si no se resuelve en un plazo de 3 – 5 días.

Laparotomía indica sección o incisión del flanco, vacío o de las partes blandas colocadas por
debajo de las costillas. Laparotomía exploradora es pues la incisión pura y simple de las paredes
del abdomen con el objeto de llegar a formular un diagnóstico

PTS: 3 DIF: Alta REF: (SOTO.S, 2005)


OBJ: Reconocer los principales conceptos y patologías relacionados a la cirugía
TOP: Abdomen Agudo obstructivo KEY: Laparotomía NOT: Dr. Víctor Quisiguiña
114. ANS: A
El vólvulo de sigma es la localización más frecuente. La torsión se produce en sentido
antihorario. Se manifiesta por dolor abdominal, ausencia de flatos y distensión abdominal
asimétrica. En la radiografía de abdomen se aprecia una importante dilatación colónica con una
imagen “asa en omega” o en “grano de café

PTS: 3 DIF: ALTA REF: (SOTO; 2005)


OBJ: Reconocer los principales conceptos y patologías relacionados a la cirugía
TOP: Abdomen Agudo Obstructivo KEY: Dilatación Colonica
NOT: VICTOR QUISIGUIÑA
115. ANS: A
La clasificación del cáncer de ovario se hace mediante laparotomía exploradora, por lo que la
cirugía se realizará a todas las mujeres. En la mayoría de los casos, se encontrarán tumores en
estadios avanzados, ya que la vía de diseminación es la implantación directa por siembra
peritoneal de células tumorales

PTS: 3 DIF: Alta REF: (SOTO,2005)


OBJ: Reconocer los principales conceptos y patologías relacionados a la cirugía
TOP: ABDOMEN GINECOLOGICO KEY: Lesión quística
NOT: Víctor Quisiguiña

20
ID: A

116. ANS: C
El diagnóstico de embarazo ectópico se realiza fundamentalmente mediante ecografía, al
visualizar una cavidad uterina vacía y una imagen anexial sugestiva/diagnóstica de gestación. En
caso de duda, es posible ayudarse con la determinación de HCG. En el embarazo ectópico
presentará niveles más bajos, y además aumentará más lentamente que en el embarazo normal

PTS: 3 DIF: ALTA REF: (SOTO;S, 2005)


OBJ: Reconocer los principales conceptos y patologías relacionados a la cirugía
TOP: ABDOMEN GINECOLOGICO KEY: QUISTE NOT: VICTOR QUISIGUIÑA
117. ANS: D
El dolor es el síntoma más frecuente y característico. Generalmente se manifiesta en forma de
dismenorrea progresiva que no cede con la toma de antiinflamatorios ni con anticonceptivos
hormonales, otros síntomas que pueden aparecer son alteraciones menstruales rectorragia, disuria

PTS: 3 DIF: ALTA REF: (SOTO.S, 2005)


OBJ: Reconocer los principales conceptos y patologías relacionados a la cirugía
TOP: ABDOMEN GINECOLOGICO KEY: Dismenorrea importante
NOT: VICTOR QUISIGUIÑA
118. ANS: C
En la enfermedad inflamatoria pélvica, el tratamiento de elección en fases precoces para prevenir
secuelas, combina de 2 ó 3 antibióticos de amplio espectro, así pues, ceftriaxona y doxiciclina.

PTS: 3 DIF: ALTA REF: (SOTO.S, 2005).


OBJ: Reconocer los principales conceptos y patologías relacionados a la cirugía
TOP: ABDOMEN GINECOLOGICO KEY: ENFERMEDAD PELVICA INFLAMATORIA
NOT: VICTOR QUISIGUIÑA
119. ANS: A
Las manifestaciones clínicas de la torsión ovárica son dolor intenso de inicio súbito localizado en
el hemiabdomen inferior, generalmente lateralizado con signos de irritación peritoneal, con o sin
masa palpable. Náusea y vómito, junto con alza térmica leve, a la ecografía se visualiza el signo
de remolino que se describe inicialmente en la rotación del mesenterio en el vólvulo intestinal y
luego se adapta a la torsión anexial, representa del pedículo vascular rotado, un vaso arterial
central rodeado por un vaso venoso.

PTS: 3 DIF: ALTA REF: (SOTO.S, 2005)


OBJ: Reconocer los principales conceptos y patologías relacionados a la cirugía
TOP: ABDOMEN GINECOLOGICO KEY: Sensible en la fosa iliaca derecha
NOT: VICTOR QUISIGUIÑA
120. ANS: B
Tenemos un paciente de edad avanzada y comorbilidades importantes que pueden limitar desde
un punto de vista anestésico la cirugía abdominal por la alta mortalidad durante la cirugía. Tras la
ausencia de mejoría con medidas de resucitación y antibioticoterapia, la medida mas segura de
lograr una posible mejoría del paciente es descomprimir la vesícula mediante una colecistostomia
guiada radiológicamente por ecografía o fluoroscopia

PTS: 3 DIF: ALTA REF: (AMIR, 2011)


OBJ: Reconocer los principales conceptos y patologías relacionados a la cirugía
TOP: Colecistitis KEY: Colecistitis NOT: VICTOR QUISIGUIÑA

21
ID: A

121. ANS: D
El cuadro descrito es el clásico de un Ampuloma. Tenga en cuenta que el dato clave para el
diagnostico diferencial es la sangre en heces, que en un principio no estaría presente en el resto de
las opciones. También se habla de heces plateadas, debido a las melenas acolicas que puede
provocar el Ampuloma.

PTS: 3 DIF: ALTA REF: (AMIR, 2010)


OBJ: Reconocer los principales conceptos y patologías relacionados a la cirugía
TOP: ABDOMEN OBSTRUCTIVO KEY: SANGRE OCULTA EN HECES
NOT: VICTOR QUISIGUIÑA
122. ANS: A
La ecografía es la primera prueba diagnostica ante un cuadro de posible obstrucción de las vías
biliares, y según los resultados de la misma se valoraría CPRE (cuando se puede hacer
terapéutica, ya que es invasiva) o colangio RMN.

PTS: 3 DIF: ALTA REF: (MIR, 2014)


OBJ: Reconocer los principales conceptos y patologías relacionados a la cirugía
TOP: CIRUGIA KEY: OBSTRUCCION NOT: DR. VICTOR QUISIGUIÑA
123. ANS: D
Los cálculos de colesterol se producen por aumento del colesterol en la bilis que forman grumos
en la vesícula, algunos factores predisponen a la formación del cálculo: el mal vaciamiento de la
vesícula, obesidad, sexo femenino, edad avanzada, embarazo, algunas enfermedades hepáticas,
rápida pérdida de peso, dietas ricas en grasa.

PTS: 3 DIF: ALTA REF: (HospitalEugenioEspejo, 2014)


TOP: CIRUGIA KEY: COLELITIASIS NOT: DR. VICTOR QUISIGUIÑA
124. ANS: D
Se trata de una pregunta de fácil contestación si se piensa en causas anatómicas para explicar la
respuesta. El conducto cístico desemboca en el colédoco una vez que los conductos hepáticos
principales se han fusionado por lo que un tumor que se origina por encima de la desembocadura
del conducto cístico no puede ocasionar distensión vesical por colestasis obstructiva.

PTS: 3 DIF: ALTA REF: (AMIR, 2011)


OBJ: Reconocer los principales conceptos y patologías relacionados a la cirugía
TOP: CIRUGIA KEY: VIA BILIAR NOT: DR. VICTOR QUISIGUIÑA
125. ANS: A
Como se conoce, las principales ventajas del abordaje laparoscópico son la reducción de la
estancia postoperatoria, la reducción del dolor postoperatorio, y una mejor recuperación funcional
postquirúrgica.

PTS: 3 DIF: ALTO REF: (AMIR, 2011)


OBJ: Reconocer los principales conceptos y patologías relacionados a la cirugía
TOP: CIRUGIA KEY: COLECISTECTOMIA NOT: DR. VICTOR QUISIGUIÑA
126. ANS: D
La lecitina es un fosfolípido presente en la secreción biliar que sirve para solubilizar los ácidos
biliares. El resto de opciones son enzimas presentes en la secreción pancreática.

PTS: 3 DIF: ALTO REF: (AMIR, 2011)


OBJ: Reconocer los principales conceptos y patologías relacionados a la cirugía
TOP: CIRUGIA KEY: BILIS NOT: DR. VICTOR QUISIGUIÑA

22
ID: A

127. ANS: B
Si bien la colelitiasis sintomática es siempre tributaria de tratamiento, la conducta que ha de
seguir cuando es asintomática debe ser todo lo conservadora posible. Numerosos estudios han
demostrado que pacientes con colelitiasis asintomática siguen en general un curso benigno sin
tratamiento y que el riesgo de desarrollar complicaciones es pequeña y no supera el de la cirugía
de modo que en principio no se debería intervenir al paciente de la pregunta. Debemos recordar
ciertas excepciones en donde aun siendo asintomática debemos intervenir al paciente calculo
grandes(>2,5cm) anomalías congénitas, diabéticos, y los pacientes con calcificación de la pared
de la vesícula biliar porque se asocia con mayor riesgo de carcinoma.

PTS: 3 DIF: ALTA REF: (AMIR, AMIR TEST, 2016)


OBJ: Reconocer los principales conceptos y patologías relacionados a la cirugía
TOP: CIRUGÌA KEY: CASO NOT: DR. VICTOR QUISIGUIÑA
128. ANS: B
La colangitis presenta la denominada triada de Charcot (fiebre, ictericia y dolor abdominal). Cada
vez se utiliza más la vía endoscópica para el diagnóstico y tratamiento de la patología biliar. La
opción dos es la Falsa, el cáncer de colédoco no se considera una causa frecuente de colangitis

PTS: 3 DIF: ALTA REF: (ARREOS.M; AMIR.ESPAÑA: MARBAN, 2012)


OBJ: Reconocer los principales conceptos y patologías relacionados a la cirugía
TOP: CIRUGÌA KEY: COLANGITIS NOT: DR. VICTOR QUISIGUIÑA
129. ANS: D
Los hallazgos de laboratorio revelan un aumento de la fosfatasa alcalina, en general con valores
que superan 3 veces a los normales. Una elevación de la fosfatasa alcalina en un paciente con
enfermedad inflamatoria intestinal puede sugerir el diagnóstico de colangitis esclerosante, pero
no es un requisito indispensable, ya que un 8.5% de los pacientes la fosfatasa alcalina es normal
en el momento del diagnóstico de la enfermedad hepática. También se observa un aumento
moderado de las transaminasas. En el 60% de los casos, la bilirrubinemia es normal en el
momento del diagnóstico, así como la albuminemia y la tasa de protrombina. Puede observarse
un aumento de las inmunoglobulinas en el 61% de los casos, preferen- temente de la IgG. La IgM
está aumentada en menos ocasiones (20-45%). Los anticuerpos antimitocondriales son negativos
y, desde el punto de vista inmunológico, el dato más característico es la detección

PTS: 3 DIF: ALTA REF: (DIGESTIVO-MANUAL CTO 7a EDICIÓN, 2010)


OBJ: Reconocer los principales conceptos y patologías relacionados a la cirugía
TOP: CIRUGIA KEY: CASO NOT: DR. VICTOR QUISIGUIÑA
130. ANS: B
La presentación habitual incluye fiebre, dolor en el epigastrio o el cuadrante superior derecho e
ictericia. Estos síntomas comunes, que se conocen como la tríada de Charcot, se identifican en
casi dos tercios de los enfermos.

PTS: 3 DIF: ALTA


REF: (SCHWARTZ PRINCIPIOS DE CIRUGIIA 10A EDICION, 2010)
OBJ: Reconocer los principales conceptos y patologías relacionados a la cirugía
TOP: CIRUGÌA KEY: COLÀNGITIS AGUDA NOT: DR. VICTOR QUISIGUIÑA

23
ID: A

131. ANS: A
Son comunes leucocitosis, hiperbilirrubinemia y aumento de la fosfatasa alcalina y de las
transaminasas; cuando se presentan sugieren el diagnóstico clínico de colangitis. Es útil una
ecografía, ya que revela la presencia de cálculos en la vesícula biliar y demostrará conductos
dilatados y tal vez señala el sitio de obstrucción; sin embargo, rara vez dilucida exactamente la
causa.

PTS: 3 DIF: ALTA


REF: (SCHWARTZ PRINCIPIOS DE CIRUGIIA 10A EDICION, 2010)
OBJ: Reconocer los principales conceptos y patologías relacionados a la cirugía
TOP: CIRUGÌA KEY: COLANGITIS NOT: DR. VICTOR QUISIGUIÑA
132. ANS: C
El tratamiento inicial de la colangitis incluye antibióticos y reanimación con líquidos por vía
intravenosa. Estos enfermos tal vez ameriten vigilancia en la unidad de cuidados intensivos y
apoyo vasopresor. Casi todos los enfermos responden a estas medidas. No obstante, es necesario
drenar el conducto biliar obstruido tan pronto se estabilice el paciente.

PTS: 3 DIF: ALTA


REF: (SCHWARTZ PRINCIPIOS DE CIRUGIIA 10A EDICION, 2010)
OBJ: Reconocer los principales conceptos y patologías relacionados a la cirugía
TOP: CIRUGIA KEY: CCOLANGITIS NOT: DR. VICTOR QUISIGUIÑA
133. ANS: A
La pancreatitis aguda es una enfermedad inflamatoria del páncreas que no se acompaña de
fibrosis de la glándula, o muy poca.

PTS: 3 DIF: ALTA REF: (SCWARTZ, XXX)


OBJ: Reconocer los principales conceptos y patologías relacionados a la cirugía
TOP: CIRUGÌA KEY: APENDICITIS NOT: DR. VICTOR QUISIGUIÑA
134. ANS: B
La pared superior de las fosas nasales esta constituidas por los huesos propios de la nariz, la espina nasal
del frontal, la lamina cribosa y el cuerpo del esfenoides forman la anatomofisiologia de las fosas nasales
y senos perinasales

PTS: 1 DIF: Alta REF: Otorrinolaringología. secretos Jafek Bruce W Elsevier


OBJ: Diagnostica las enfermedades de la nariz y los senos paranasales. - Operaciones Cognitivas -
Resolución de errores - Planificacion de acciones y correccion de errores
LOC: Dr. Rolando Perez TOP: NARIZ Y SENOS PARANASALES
135. ANS: A
La mastoiditis aguda es una de las principales complicaciones de la otitis media

PTS: 1 DIF: Alta REF: Otorrinolaringología. secretos Jafek Bruce W Elsevier


OBJ: Diagnostica las enfermedades de oído- | Operaciones Cognitivas - Resolución de errores -
Planificacion de acciones y correccion de errores LOC: Dr. Rolando Perez
TOP: OIDO

24
ID: A

136. ANS: C
El carcinoma mucoepidermoide es el tumor maligno mas frecuente en la glandula parotida, sin embargo
cuando se trata de la glandula submaxilar el carcinoma adenoide quistico presenta uno en cada dos
tumores malignos

PTS: 1 DIF: Alta


REF: Escajadillo JR. Oídos, Nariz, Garganta y Cirugía de Cabeza y Cuello. Editorial: El Manual
Moderno. 2da ed; 2002
OBJ: Diagnostica las enfermedades de las glándulas salivales y las que se originan en el cuello -
Operaciones Cognitivas - Resolución de errores - Planificacion de acciones y correccion de errores
LOC: Dr. Rolando Perez TOP: GLANDULAS SALIVALES Y CUELLO
137. ANS: D
Según La guía clínica de la academia americana de Otorrinolaringología hay una fuerte recomendación
para el tratamiento del dolor y la inflamación, tambein hacen referencia el tratamiento inicial de la otitis
externas no complicadas que debe ser con atibióticos tópicos así como inciar por la limpieza del conduto
auditivo externo

PTS: 1 DIF: Alta


REF: Escajadillo JR. Oídos, Nariz, Garganta y Cirugía de Cabeza y Cuello. Editorial: El Manual
Moderno. 2da ed; 2002
OBJ: Diagnostica las enfermedades de oído - Operaciones Cognitivas - Resolución de errores -
Planificacion de acciones y correccion de errores LOC: Dr. Rolando Perez
TOP: OIDO
138. ANS: B
El VPH se contagia muchas veces a través de secreciones por lo que el lugar donde las secreciones
tienden hacer mas contacto es la orofaringe, se debe añadir que los tumores de orofaringe con positividad
para VPH se presentan en pacientes jovenes con ausencia de otros factores de reisgo y tienden mejor
pronóstico

PTS: 1 DIF: Alta


REF: Escajadillo JR. Oídos, Nariz, Garganta y Cirugía de Cabeza y Cuello. Editorial: El Manual
Moderno. 2da ed; 2002
OBJ: Diagnostica las enfermedades de las glándulas salivales y las que se originan en el cuello -
Operaciones Cognitivas - Resolución de errores - Planificacion de acciones y correccion de errores
LOC: Dr. Rolando Perez TOP: GLANDULAS SALIVALES Y CUELLO
139. ANS: D
El objetivo de cribado universal es el diagnóstico y tratamiento lo más pronto posible trartando de
conservar tanto antómica como funcionalmente la mayor cantidad de estructuras y adaptar al paciente
para que este se desemvuelva con la mayor normalidad posible en la sociedad.

PTS: 1 DIF: Alta


REF: Escajadillo JR. Oídos, Nariz, Garganta y Cirugía de Cabeza y Cuello. Editorial: El Manual
Moderno. 2da ed; 2002
OBJ: Diagnostica las enfermedades de oído - Operaciones Cognitivas - Resolución de errores -
Planificacion de acciones y correccion de errores LOC: Dr. Rolando Perez
TOP: OIDO

25
ID: A

140. ANS: B
Se describen tres tipos de risnosinusitis invasivas, la invasiva crónica por Aspergillus Fumigatus, propia
de pacientes inmunodeprimidos que se desarrolla en más de tres meses, la invasiva subaguda
granulomatosa, por Aspergillus Flavus propia de pacientes inmunocompetentes, y la invasiva aguda por
Mucor muy rápida que es la que orienta a la pregunta.

PTS: 1 DIF: ALta


REF: Escajadillo JR. Oídos, Nariz, Garganta y Cirugía de Cabeza y Cuello. Editorial: El Manual
Moderno. 2da ed; 2002
OBJ: Diagnostica las enfermedades de la nariz y los senos paranasales- Operaciones Cognitivas -
Resolución de errores - Planificacion de acciones y correccion de errores
LOC: Dr. Rolando Perez TOP: NARIZ Y SENOS PARANASALES
141. ANS: B
Para un paciente que presenta sordera de conducción. la afirmación correcta es que La percepción del
sonido es mejor por vía ósea que por vía aérea

PTS: 1 DIF: Alta


REF: Escajadillo JR. Oídos, Nariz, Garganta y Cirugía de Cabeza y Cuello. Editorial: El Manual
Moderno. 2da ed; 2002 | Otorrinolaringología. secretos Jafek Bruce W Elsevier
OBJ: Diagnostica las enfermedades de oído - Operaciones Cognitivas - Resolución de errores -
Planificacion de acciones y correccion de errores LOC: Dr. Roalndo Perez
TOP: OIDO
142. ANS: C
La complicación mas frecuente en caso de otitis media es la mastoiditis

PTS: 1 DIF: Alta


REF: Otorrinolaringología. secretos Jafek Bruce W Elsevier | Escajadillo JR. Oídos, Nariz, Garganta y
Cirugía de Cabeza y Cuello. Editorial: El Manual Moderno. 2da ed; 2002
OBJ: Diagnostica las enfermedades de oído - Operaciones Cognitivas - Resolución de errores -
Planificacion de acciones y correccion de errores LOC: Dr. Rolando Perez
TOP: OIDO
143. ANS: D
El virus varicela zóster que cusa el síndromde Ramsay Hunt es el mismo que causa la varicela y el herpes
zóster, en las personas con este virus infecta al nervio facial cercano al oído interno lo cual lleva a que se
presente irritación e inflamación del mismo causando parálisis facial

PTS: 1 DIF: Alta


REF: Otorrinolaringología. secretos Jafek Bruce W Elsevier | Escajadillo JR. Oídos, Nariz, Garganta y
Cirugía de Cabeza y Cuello. Editorial: El Manual Moderno. 2da ed; 2002
OBJ: Diagnostica las enfermedades de oído - Operaciones Cognitivas - Resolución de errores -
Planificacion de acciones y correccion de errores LOC: Dr. Rolando Perez
TOP: OIDO

26
COMP CIRUGIA [Answer Strip] ID: A

C
_____ 9. B 18.
_____ D 24.
_____ B 29.
_____

B
_____ 1. D 10.
_____

C 25.
_____
B 19.
_____

C
_____ 2. C 11.
_____

D 30.
_____

D 20.
_____
D
_____ 3. C 26.
_____
A 12.
_____

C
_____ 4. B 21.
_____
C 31.
_____
C 13.
_____

B 27.
_____
A
_____ 5.

A 14.
_____
A 32.
_____
D 22.
_____
B
_____ 6.

C 15.
_____
D 28.
_____

D
_____ 7.
C 23.
_____
B 16.
_____ A 33.
_____

A
_____ 8. D 17.
_____
COMP CIRUGIA [Answer Strip] ID: A

D 34.
_____ D 38.
_____ C 41.
_____ C 45.
_____ B 49.
_____

B 35.
_____

D 46.
_____
A 42.
_____

B 36.
_____
C 39.
_____

C 50.
_____

C 47.
_____

D 37.
_____
D 43.
_____

B 51.
_____

A 40.
_____ B 48.
_____

C 44.
_____
COMP CIRUGIA [Answer Strip] ID: A

C 52.
_____ C 56.
_____ D 61.
_____ B 65.
_____ A 68.
_____

D 57.
_____
C 62.
_____
A 53.
_____

D 66.
_____

D 58.
_____
D 63.
_____
B 54.
_____
C 69.
_____

D 64.
_____
D 67.
_____
D 70.
_____
B 59.
_____
C 55.
_____

A 71.
_____

D 60.
_____
COMP CIRUGIA [Answer Strip] ID: A

D 72.
_____ C 76.
_____ B 81.
_____ C 86.
_____ D 91.
_____

C 92.
_____
D 82.
_____

A 77.
_____
A 73.
_____
A 87.
_____
C 93.
_____
B 83.
_____

D 78.
_____

D 84.
_____
C 88.
_____ B 94.
_____

C 74.
_____

D 79.
_____

D 89.
_____

C 85.
_____
D 95.
_____

A 80.
_____
E 75.
_____
B 90.
_____
COMP CIRUGIA [Answer Strip] ID: A

A 96.
_____ C
_____101. A
_____106. C
_____110. C
_____113.

D
_____102.

B 97.
_____
D
_____107.

D
_____111.

D
_____103. A
_____114.

C
_____108.

D 98.
_____

A
_____104.

B
_____112.
A
_____115.
A 99.
_____

D
_____105.

B
_____109.

D
_____100.
COMP CIRUGIA [Answer Strip] ID: A

C
_____116. A
_____119. A
_____122. B
_____127. D
_____129.

D
_____123.

D
_____117.

D
_____124.

B
_____130.

A
_____125. B
_____128.
C
_____118. B
_____120.

D
_____126.

D
_____121.
COMP CIRUGIA [Answer Strip] ID: A

A
_____131. A
_____133. C
_____136. B
_____140.

D
_____137.

B
_____141.

B
_____138.
C
_____132.

C
_____142.
B
_____134.

D
_____139.

D
_____143.

A
_____135.
Name: ________________________ Class: ___________________ Date: __________ ID: A

COMP GINECOLOGIA

Multiple Choice
Identify the choice that best completes the statement or answers the question.

____ 1. PCTE DE 41 AÑOS QUE ACUDE A PRIMER CONTROL DE EMBARAZO, FUM 18/09/2017; COMO
ANTECEDENTE PERSONAL REFEIRE DIABETES MELLITUS INSULINODEPENDIENTE +
PREECLAMPSIA EN EMBARAZO ANTERIOR. ESCOJA LA INDICACION MAS ADECUADA
PARA ESTA PCTE SEGUN GPC MSP ECUADOR.

a. ACIDO FOLICO 1MG VO CD + c. ACIDO FOLICO 5M + CALCIO


CALCIO 1.5-2MG CD + AC 1.5-2MG CD + AC
ACETILSALICILICO 100 MG CD ACETILSALICILICO 100MG CD

b. AC FOLICO 5MG d. CALCIO 1.5-2MG CD + AC


ACETILSALICILICO 100MG CD

____ 2. PCTE DE 3O AÑOS ACUDE A SU PRIMER CONTROL PRENATAL. SE REALIZA HISTORIA


CLNICA: REFEIRE FUM DEL 25/05/2017 . CASADA; OCUPACION OBRERA EN EMPRESA DE
CULTIVO DE FLORES; REFIERE CARGA HORARIA DE 10 HORAS DIARIAS PARADA EN SU
MAYOR PARTE DE TIEMPO.
ADEMAS REFEIRE PRESENTAR EN SU EMBARAZO ANTERIOR PARTO CEFALO-VAGINAL A
LAS 36SEMANAS DE GESTACION.
AL EX FISICO: PESO 62KG ; TALLA 157CM
SEGUN GUIA DE CONTROL PRENATAL. EMITA SU DIAGNOSTICO.
AGREGUE HOJA DE CURVAS DE NUTRICION.

a. EMBARAZO DE 25 SEMANAS c. EMBARAZO DE 25SEMANAS


EMBARZO DE MUY ALTO RIESGO EMBARAZO DE ALTO RIESGO
SOBREPESO PESO NORMAL
b. EMBARAZO DE 24.4 SEMANAS d. EMBARAZO DE 24.4 SEMANAS
EMBAARZO DE ALTO EMBARAZO DE MUY ALTO RIESGO
RIESGO PESO NORMAL
PESO NORMAL

____ 3. PACIENTE DE 25AÑOS QUE ACUDE A LA EMERGENCIA CON EMBARZO DE 35 SEMANAS DE


GESTACION MAS SANGRADO GENITAL DE MODERADA CANTIDAD ACOMPAÑADO DE
COAGULOS .
SIGNOS VITALES : PULSO 100X´; PA 160/95; SAT 92; FR 24; T° 37; PROTEINURIA POSITIVA 3
CRUCES ; ESTADO DE CONCIENCIA ESTUPOROSA
INDIQUE CUAL ES EL SCORE MAMA DE LA PCTE ( ANOTAR ) Y CADA CUANTO USTED
HARIA LOS CONTROLES
a. SCORE CONTROL CADA 4H c. SCORE CONTROL CADA
30MIN
b. SCORE CONTROL CADA H d. SCORE 0 ALTA A LA CASA

1
Name: ________________________ ID: A

____ 4. PCTE DE 19 AÑOS PRIMIGESTA, ALERGICA A LA PENICILINA. QUE CURSA CON


EMBARAZO DE 33 SEMANAS DE GESTACION. ACUDE POR PRESENTAR DOLOR
ABDOMINAL TIPO CONTRACCION 3/10/45”; TACTO VAGINAL DILATADO 4 ; BORRADO 80%
; MENBRANAS ROTA DE 8H DE EVOLUCION. POR LO QUE LA INGRESAN CON
DIAGNOSTICO DE TRABAJO DE PARTO PREMATURO MAS RUPTURA DE MENBRANAS .

SEGÚN GPC QUE CONDUCTA REALIZARIA ? .

a. INGRESO c. INGRESO
MADURACION PULMONAR MADURACION PULMONAR
FETAL FETAL
TRATAMIENTO PARA SGB CON TRATAMIENTO PARA SGB CON
PENICILINA AMPICILINA

b. INGRESO d. INGRESO
MADURACION PULMONAR MADURACION PULMONAR
FETAL FETAL
TRATAMIENTO PARA SGB CON TRATAMIENTO PARA SGB CON
CEFAZOLINA CLINDAMICINA

____ 5. PCTE DE 35AÑOS PRIMIGESTA CON GRUPO SANGUINEO O RH NEGATIVO, ACUDE A LA


CONSULTA CON EMBARAZO DE 14SEMANAS DE GESTACION, EN LA PRIMERA CONSULTA
SE SOLICITA COMBS INDIRECTO, EL MISMO QUE ES NEGATIVO. SEGÚN GPC ¿A QUE
SEMANA SE DEBE COLOCAR PROFILAXIS ANTI-D SI NO HAY ANTECEDENTES DE
SANGRADO EN EL RESTO DEL EMBARAZO ? .
a. 35-37 c. 28-30
b. EN EL MOMENTO DEL TRABAJO DE d. 20-24
PARTO

____ 6. PCTE DE 22AÑOS SECUNDIGESTA COLOMBIANA , CON EMBARAZO DE 12SEMANAS DE


GESTACION, ACUDE A LA SEGUNDA CONSULTA, CON RESULTADOS DE EXAMENES, DE
LOS CUALES SE DESTACA GLICEMIA DE 91mg/dL RESTO DE EXAMENES DENTRO DE
PARAMETROS NORMALES. SEGÚN GPC ¿QUE DIAGNOSTICARIA Y CUAL SERIA EL
CONTROL A SEGUIR? .

a. DIABETES GESTACIONAL c. EXAMEN NORMAL SE SOLICITARA


PRUEBA DE CONTROL (PTGO) A
LAS 26-28 SEMANAS
b. EXAMEN NORMAL, SE SOLICITARA d. DIABETES PREEXISTENTE
PRUEBA DE CONTROL (PTGO) A
LAS 24-28 SEMANAS

2
Name: ________________________ ID: A

____ 7. GESTANTE DE 24 SEMANAS ACUDE A LA CONSULTA PORQUE A SU HIJO DE 5 AÑOS HACE 5 DÍAS
LE DIAGNOSTICARON VARICELA. LA PACIENTE NO RECUERDA SI PADECIÓ LA ENFERMEDAD EN
LA INFANCIA, PERO SI SABE QUE NO FUE VACUNADA Y ESTÁ MUY PREOCUPADA POR LA POSIBLE
AFECCIÓN FETAL. QUE ACTUACIÓN SERIA LA CORRECTA?.

a. ADMINISTRAR VACUNA ESPECIFICA d. ADMINISTRAR ACICLOVIR A DOSIS DE


800MG VO 5 VECES AL DÍA POR 5- - 7
DÍAS.

b. SOLICITAR CUANTIFICACIÓN DE IGG, e. ADMINISTRAR GAMAIGLOBULINA


Y SI FUERA ESPECIFICO ADMINISTRAR ESPECIFICA Y TRANQUILIZAR A LA
GAMAGLOBULINA ESPECIFICA. PACIENTE INFORMÁNDOLE DE LA
AUSENCIA DE RIESGOS.

c. TRANQUILIZAR A LA PACIENTE
INFORMÁNDOLE DE LA AUSENCIA DE
RIESGOS FETALES

____ 8. PACIENTE DE 28 AÑOS EN LAS 42.3 SG Y ANTECEDENTES DE OTRO PARTO A TÉRMINO. EL


EMBARAZO HA TRANSCURRIDO NORMALMENTE. AL EXAMEN FISICO VARIEDAD DE
PRESENTACION: FETO EN OIIA, NST REACTIVO Y CERVIX FAVORABLE BISHOP 9. CUÁL DE LAS
SIGUIENTES CONDUCTAS ES LA MAS FAVORABLE?

a. ESPERAR HASTA QUE DESENCADENE d. CONDUCIR EL TRABAJO DE PARTO


TRABAJO DE PARTO..
C.- ADMINISTRAR CORTICOIDES A LA
MADRE E INDUCIR EL PARTO
PASADAS LAS 48 HORAS

b. CONTROLES CADA 48 HORAS. e. HACER CESÁREA ELECTIVA.

c. ADMINISTRAR CORTICOIDES A LA
MADRE E INDUCIR EL PARTO
PASADAS LAS 48 HORAS

____ 9. PACIENTE EN PERIODO EXPULSIVO DE 38 SEMANAS DE GESTACIÓN, PRESENTACIÓN EN TERCER


PLANO DE HODGE, POSICIÓN OIIT Y APARICIÓN DE UNA BRADICARDIA FETAL DE 100 - 110 LAT X
MIN CONSTATÁNDOSE LA EXISTENCIA DE UNA ACIDOSIS RESPIRATORIA FETAL: PH DE CUERO
CABELLUDO 7.18. CUÁL SERÍA LA ACTITUD CORRECTA?

a. REALIZACIÓN DE UNA CESÁREA d. COLOCAR EL PULSOXIMETRO PARA


PROGRAMADA CONSTATAR LA SATURACIÓN DE
OXIGENO MATERNO.

b. UTILIZACIÓN DE UNA VENTOSA - e. APLICACIÓN DE OXIGENO A LA


VACUUM. MADRE PARA REALIZACIÓN DE PH EN
10 MINUTOS.

c. UTILIZACIÓN DE UN FÓRCEPS DE
SALIDA

3
Name: ________________________ ID: A

____ 10. GESTANTE DE 8 SEMANAS QUE ACUDE A CONSULTA PARA REALIZAR ECOGRAFÍA EN LA QUE UD.
OBSERVA UNA GESTACIÓN INTRAUTERINA CON EMBRIÓN ÚNICO CON LCC DE 16MM (ACORDE
CON 8 SEMANAS) SIN LATIDO CARDIACO SEÑALE LO CORRECTO:

a. GESTACIÓN MAL DATADA. LA LCC d. DIAGNOSTICO DE FETO ACARDIO


NOS INDICA EL TIEMPO DE
GESTACIÓN EN EL PRIMER
TRIMESTRE. ÚNICAMENTE
CORREGIMOS LA FECHA PROBABLE
DE PARTO.
b. AMENAZA DE ABORTO, RECOMIENDA e. DIAGNOSTICO DE ABORTO DIFERIDO
REPOSO Y GESTÁGENOS. = LEGRADO

c. DETERMINACIÓN DE BHCG
PLASMÁTICA CADA 48 H.

____ 11.
GESTANTE DE 38 SEMANAS QUE COMIENZA BRUSCAMENTE CON METRORRAGIA OBSCURA Y EN
POCA CANTIDAD, DOLOR ABDOMINAL INTENSO, DIFUSO, MAL ESTADO GENERAL Y SUFRIMIENTO
FETAL. CUÁL ES LA COMPLICACIÓN MAS PROBABLE EN ELLA?.

a. DESPRENDIMIENTO PREMATURO DE d. NECROSIS DE UN MIOMA.


PLACENTA
b. PLACENTA PREVIA e. TORSIÓN DE UN TUMOR OVÁRICO.

c. INSERCIÓN VELAMENTOSA DE
CORDÓN UMBILICAL.

____ 12. GESTANTE DE 26 SEMANAS QUE PRESENTA DESDE HACE 5 DÍAS: MALESTAR GENERAL, NAUSEA,
CEFALEA, EDEMA Y LIGERO DOLOR EN HIPOCONDRIO DERECHO. TA 140/90 PROTEINURIA 3g/24H
EN EXÁMENES HB 12G/DL, BILIRRUBINA TOTAL 1.4 MG/DL, LDH 670 UI/I, TGO 182UI/I, PLAQUETAS
DE 80000MM3. LO MAS PROBABLE ES QUE SE TRATE DE UN EMBARAZO COMPLICADO POR?.

a. EMBOLISMO DE LÍQUIDO AMNIÓTICO d. ISOINMUNIZACIÓN ERITROCITARIA


CON TRANSFUSIÓN FETO – MATERNA
b. MUERTE FETAL CON PASO DE e. E.- PREECLAMPSIA SEVERA + SD
SUSTANCIAS TRONBOPLÁSTICAS A LA HELLP
CIRCULACIÓN MATERNA.
c. DESPRENDIMIENTO PREMATURO DE
PLACENTA SUPERIOR A 50%

____ 13. CUAL ES EL MÉTODO DE DETECCIÓN (SCREENING) DE DIABETES GESTACIONAL QUE SE


RECOMIENDA EN TODAS LAS EMBARAZADAS?

a. DETERMINACIÓN DE GLUCOSA EN d. TEST DE TOLERANCIA ORAL A LA


ORINA GLUCOSA CON 100G DE SOBRECARGA
b. DETERMINACIÓN DE GLICEMIA EN e. TEST DE TOLERANCIA ORAL A LA
AYUNAS GLUCOSA CON 75G DE SOBRECARGA
c. DETERMINACIÓN DE GLICEMIA EN
AYUNASEN DÍAS DIFERENTES

4
Name: ________________________ ID: A

____ 14. CUAL DE LAS SITUACIONES CLINICAS ES UNA CONTRAINDICACION ABSOLUTA PARA LA
ANTICONCEPCION HORMONAL?

a. HIPERTENSION ARTERIAL BIEN d. MUJER MENOR DE 35 AÑOS Y


CONTROLADA. FUMADORA
b. HISTORIA PERSONAL DE e. INFECCION URINARIA.
TROMBOEMBOLISMO PREVIO.
c. DIABETES

____ 15.
CUAL DE LOS SIGUIENTES CARACTERISTICAS NO ES FRECUENTE EN LA VAGINOSIS BACTERIANA

a. PH VAGINAL MENOR A 4.5. d. AUSENCIA DE ERITEMA VULVAR


b. LEUCORREA FLUIDA Y HOMOGENEA e. AUSENCIA DE PRURITO VULVAR

c. PRESENCIA DE CELULAS CLAVE.

____ 16. CUAL DE LAS SIGUIENTES AFIRMACIONES ES FALSA EN RELACIÓN A LA TRANSMISIÓN VERTICAL
AL FETO/RN POR PARTE DE UNA GESTANTE CON INFECCIÓN DE VIH?

a. AUMENTA LA POSIBILIDAD DE d. CON EL TRATAMIENTO


MALFORMACIÓN FETAL. ANTIRRETROVIRAL DURANTE EL
EMBARAZO Y SOBRE TODO DURANTE
EL PARTO
b. TIENE LUGAR SOBRE TODO EN EL e. EL TRATAMIENTO CON ZIDOVUDINA
PARTO. NO HA DEMOSTRADO EFECTOS
ADVERSOS EN EL RECIÉN NACIDO
HASTA LOS 5 AÑOS
c. LA LACTANCIA AUMENTA EL RIESGO
DE TRANSMISIÓN ENTRE UN 10% A
20%.

____ 17. PRIMIGESTA DE 37 SEMANAS INGRESA POR RPM DE 24H DE EVOLUCIÓN NO HAY TRABAJO DE
PARTO NI SIGNOS DE INFECCIÓN AMNIÓTICA. EL TEST DE BISHOP ES DE 4 PUNTOS, EL FETO
ESTA EN CEFÁLICA Y NO HAY SIGNOS DE COMPROMISO FETAL. CUÁL ES LA CONDUCTA
OBSTÉTRICA INDICADA?

a. CESÁREA d. ADMINISTRAR ANTIBIÓTICOS Y


ESPERAR EL COMIENZO ESPONTANEO
DEL PARTO.
b. ADMINISTRAR ANTIBIÓTICOS E e. ESPERAR EL COMIENZO ESPONTANEO
INDUCIR EL PARTO CON SIN TRATAMIENTO O HACER CESÁREA
PROSTAGLANDINAS. CON SIGNOS DE INFECCIÓN
AMNIÓTICA.
c. ADMINISTRAR ANTIBIÓTICOS,
CORTICOIDES Y CONDUCIR EL PARTO
CON OXITOCINA48H DESPUÉS

5
Name: ________________________ ID: A

____ 18. MUJER DE 25 AÑOS CONSULTA POR RETRASO MENSTRUAL DE 3 SEMANAS Y


SANGRADO TRANSVAGINAL DE 24 HORAS, ESTADO GENERAL BUENO. BHCG: 1000
mUI/ml, ECOGRAFIA TRANSVAGINAL ENDOMETRIO DE 12 mm, EN OVARIO DERECHO
IMAGEN QUE APARENTRA CUERPO LUTEO , NO HAY LIQUIDO LIBRE EN FONDO DE
SACO DE DOUGLAS. ¿CUAL ES LA INDICACIÓN CORRECTA?
a. repertir seriadamente cada 48 horas la d. diagnostico de embarazo ectópico
BHCG y la ecografia
b. reposo absoluto y repetir la ecografia en e. laparotomia o laparoscopia
2 - 3 semenas
c. legrado uterino

____ 19. INFECCION POR ESTREPTOCOCO DEL GRUPO B EN RN PUEDE PREVENIRSE


ADMINISTRANDO ANTIBIOTICOTERAPIA PROFILACTICA A LAS EMBARAZADAS
PORTADORAS O CON FACTORES DE RIESGO ¿CUAL SERIA LA ACTITUD INCORRECTA
a. la profilaxis se realiza durante el parto d. la prueba se solicitara entre la 35 y 37
semanas o ante la eminecia del parto
b. en las pacientes alergicas a la ampicilina e. la incidencia de infeccion neonatal es
se puede utilizar la eritromicina mayor entre RN prematuros que a
termino
c. la via mas frecuente de infección es
transplacentarea

____ 20. COMO CLASIFICA USTED A UNA GESTACION QUE ANTES DEL EMBARAZO REFIERE
TENER TENSIONES ARTERIALES NORMALES, QUE EN LA SEMANA 8 SE LE DETECTA
TENSION ARTERIAL 140/90 Y QUE EN LA SEMANA 18 TIENE UNA ENSION ARTERIAL DE
170/110 SIN EDEMA Y INDICE QUE PROTEINURIA/CREATINURIA DE 100 mg/mmol
a. preeclampsia d. hipertensión crónica más preeclampsia
sobreañadida
b. preeclampsia grave e. hipertensión gestacional
c. hipertensión crónica

____ 21. PUERPERA DE 42 AÑOS CON PARTO VAGINAL HACE 12 HORAS EN LA QUE SE
SOSPECHA DE DIAGNOSTICO DE SD HELLP. ¿INDIQUE QUE LITERAL NO
CORRESPONDE?
a. plaquetas de 75000 d. hemoglobina: 7.2 g/dl
b. bilirrubina indirecta de 3.7 mg/dl e. TGO de 520 UI/L
c. TA: 170/110 mmHg

____ 22. SEGUNDIGESTA CON ANTECEDENTE DE PARTO ANTERIOR A LAS 35 SEMANAS, QUE
CONSULTA EN LA SEMANA 32 POR PERCIBIR CONTRACCIONES UTERINAS, EN LA
EXPLORACION SE COMPRUEBA DILATACION CERVICAL DE 1 CM BORRADO 30% CON
UNA LONGITUD CERVICAL DE 1.5 CM POR ECOGRAFIA, EL MONITOREO FETAL
REVELA UNA FCF: 140 LPM, Y UNA CONTRACCIÓN UTERINA DE CADA 5 MINUTOS.
¿CUAL DE LAS SIGUIENTES AFIRMACIONES ES CORRECTA?
a. se trata de un APP establecida que obliga c. se aconseja la administración de
a realizar tocolisis y maduración cervical corticoides para maduración pulmonar
fetal unicamente por el antecedente del
parto pretermino
b. la longitud cervical, inferior a 2.5 cm d. se trata de una APP incierta requiere una
supone un elevado valor predictivo nueva valoracion clinica (AU, FCF Y
negatico para parto pretermino TV) en 1 - 2 horas

6
Name: ________________________ ID: A

____ 23. GESTANTE A TERMINO QUE INGRESA EN TRABAJO DE PARTO EN FASE LATENTE, A
LA VALORACION SE DETECTA PEQUEÑAS VESICULAS DE HERPES SIMPLE
RESIDIVANTE. AÑOS ANTES TUVO UN PRIMOINFECCIÓN DE HERPES GENITAL Y
VARIOS BROTES DE HERPES RESIDIVANTE. ¿CUAL ES LA CONDUCTA A SEGUIR?
a. cesarea d. tratar inmediatamente las lesiones con
Ac. tricloro-acetico, para inactivar el
virus y permitir el parto vaginal
b. parto normal y tratar al RN con aciclovir e. parto normal y aislar al RN de su madre
c. no es necesaria una conducta especial ya
que el herpes residivante no tiene riesgo
para el RN

____ 24. LAS HEMORRAGIAS ANORMALES VAGINALES, FRECUENTES EN EL PRIMER AÑO TRAS
LA MENARQUIA, QUE SE CONOCEN COMO HEMORRAGIAS UTERINAS
DISFUNCIONALES SE DEBEN A:
a. hemorragias de la ovulacion y no d. trastornos de la coagulacion y ameritan
ameritan tratamiento tratamiento
b. ovario poliquistico y amerita tratamiento e. enfermedad inflamatoria pelvica que
amerita tratamiento
c. ciclos anovulatorios y no ameritan
tratamiento

____ 25. MUJER DE 35 AÑOS CON 31 SEMANAS DE EMBARAZO CONSULTA POR UNA SECRECION
VAGINAL DE MAL OLOR, “PESCADO PODRIDO”, NO TIENE PRURITO GENITAL, A LA
VALORACIÓN SE VIDENCIA ABUNDANTE SECRECION BLANCO GRISACEA QUE NO SE
ADHIERE A LAS PAREDES VAGINALES. AL MEZCLAR ESTA MUESTRA CON HIROXIDO
DE POTASIO AL 10% DESPRENDE UN OLOR INTENSO SUIGENERIS. ¿CUAL DE LAS
SIGUIENTES ES EL TRATEMIENTO DE ELECCIÓN?
a. amoxicilina + ac. clavulánico VO o IV c. fluconazol por VO
b. clotrimazol por via intravaginal o VO d. metronizadol por via intravaginal o VO

____ 26. PACIENTE EN SU PRIMER TRIMIESTE DE GESTACION CON HEMOGLOBINA DE 11


CATALOGADA CON ANEMIA LEVE USTED DECIDE INICIAR EL APORTE DE HIERRO.
a. Se trata una anemia cronica y no necesita d. Inicia 200 mg de hierro elemental diarios
de aporte de hierro
b. Considera utilizar hierro sacarosa IV e. Sulfato ferroso 500 mg VO diarios
c. A pesar de que la paciente esta
asintomatica por su etapa de embarazo
considera transfusión

____ 27. Seleccione la respuesta correcta entre los planteamientos que se relacionan a continuación
sobre Preeclampsia.

a. El tratamiento con sulfato de magnesio c. El sulfato de magnesio es el


se administra solamente en la medicamento anticonvulsivante de
preeclampsia leve. elección en la preeclampsia.

b. La preeclampsia severa se caracteriza


solamente por presentar TA: mayor o
igual a 160/110 mmHg.

7
Name: ________________________ ID: A

____ 28. Seleccione el orden correcto en que se realiza el manejo activo del tercer periodo del parto
(MATEP). a) Masaje uterino, b) Tracción y contratracción controlada del cordón,
c)Adminidtración de 10 UI de oxitocina IM dentro del primer minuto, , d)Verificar que no
haya otro bebé, .

a. d, c, b, a, c. a,c,d,b

b. d,a,c,b d. d,c,a,b

____ 29. Seleccione la opción correcta acerca del tratamiento farmcológico en los trastornos
hipertensivos del embarazo en nuestro medio .

a. Nimodipino c. Nifedipino

b. Labetalol d. Hidralazina

____ 30. Seleccione la opción en la que se refleja el tratamiento correcto de la Ruptura prematura de
membranas.

1. Ampicilina 2 gramos IM dosis inicial, seguido de 1 gramo cada 6 horas + Eritromicina 250
miligramos vía oral cada 6 horas por 48 horas.

2. Ampicilina 2 gramos IM dosis inicial, seguido de 1 gramo cada 6 horas + Eritromicina 250
miligramos vía oral cada 8 horas por 48 horas.

3. Ampicilina 2 gramos IV dosis inicial, seguido de 1 gramo IV cada 6 horas + Eritromicina


250 miligramos vía oral cada 6 horas por 48 horas.

4. Ampicilina 2 gramos IV dosis inicial, seguido de 1 gramo IV cada 6 horas + Eritromicina


250 miligramos vía oral cada 8 horas por 48 horas.

5. Amoxacilina 500 miligramos vía oral cada 6 horas + Eritromicina 250 miligramos vía oral
cada 8 por 5 días.

6. Amoxacilina 500 miligramos vía oral cada 8 horas + Eritromicina 250 miligramos vía oral
cada 6 horas por 5 días.

a. 1,6 c. 3,6

b. 3,5 d. 4,5

8
Name: ________________________ ID: A

____ 31. Seleccione la opción que muestra los planteamientos verdaderos Con relación a la clínica del
parto. Responda V o F, según corresponda.
A___ Una gestante esta en trabajo de parto cuando se constata dilatación cervical de 5cm

B___ La duración de la contracción debe ser de 20-25 segundos.

C___ El cuarto tiempo o movimiento cardinal del parto es aquel donde se produce el
desprendimiento de la cabeza y encajamiento de los hombros.

D___ En el segundo tiempo del mecanismo de parto en cefálico es cuando ocurre el


encajamiento de la cabeza en la excavación pélvica.

E___ El periodo de alumbramiento comienza con la salida de la cabeza del feto

a. B,C,E c. A,C,E

b. A,C,D d. C,D,E

____ 32. Seleccione el planteamiento que considere correcto acerca de la hemorragia posparto.

a. La HPP es la pérdida de sangre que c. La HPP primaria es aquella que se


supera los 500 ml en un parto vaginal y produce entre las 24 horas y seis
que supera los 1.000 ml en un parto por semanas posparto.
cesárea.

b. La HPP secundaria es aquella que se


produce entre las 12 y 24 horas
posparto.

9
Name: ________________________ ID: A

____ 33. Seleccione la opción que muestra los planteamientos correctos acerca de la clasificación de
los trastornos hipertensivos..

1. En la Hipertensión crónica encontramos una TAS = 140 mmHg y/o TAD = 90 mmHg
presente antes del embarazo, o después de las 20 semanas de gestación, o que persiste
después de las 12 semanas del posparto y ausencia de proteinuria (excepto si hay lesión
renal).

2. En la Eclampsia se produce el desarrollo de convulsiones tónico - clónicas generalizadas


y/o coma en mujeres con preeclampsia durante el embarazo, parto o puerperio, no
atribuible a otras patologías o condiciones neurológicas.

3. La Hipertensión gestacional se caracteriza por presentar TAS = 140 mmHg y/o TAD = 90
mmHg, presente antes de las 20 semanas de gestación y ausencia de proteinuria.

4. La Preeclampsia con signos de gravedad (grave), se caracteriza por presentar TAS = 160
mmHg y/o TAD = 110 mmHg y/o uno o más criterios de gravedad y/o afectación de órgano
blanco.

5. La Preeclampsia sin signos de gravedad (leve) se caracteriza por presentar TAS = 140 mm
Hg y menor (<) 160 mmHg y/o TAD = 90 mmHg y < 110 mmHg más proteinuria y sin
criterios de gravedad ni afectación de órgano blanco.

a. 1, 2, 5 c. 2, 3, 4

b. 2, 4,5

____ 34. Seleccione el orden de frecuencia correcto de las causas de HPP.

a- Tono: atonía uterina.

b- Tejido: retención de placenta o coágulos.

c- Trauma: lesión cervical o vaginal, ruptura uterina.

d- Trombina: coagulopatía preexistente.

a. a, c, b,d c. b, a,c,d

b. a, b, c, d

10
Name: ________________________ ID: A

____ 35. Cuáles son los puntos de corte máximos de la Prueba de tolerancia oral a la glucosa (PTGO)
con 75 gramos de glucosa.

a. Basal: menor a 92 mg/dL c. Basal: menor a 92 mg/dL


Primera hora: menor a 183 mg/dL Primera hora: menor a 153 mg/dL
Segunda hora: menor a 150 Segunda hora: menor a 180
mg/dL mg/dL
b. Basal: menor a 92 mg/dL
Primera hora: menor a 180 mg/dL
Segunda hora: menor a 153
mg/dL

____ 36. A) Las etiología de la ruptura prematura de membranas es ------------------------------, el tratamiento con
antibióticos se inicia a ---------------------------------- y cuando la paciente acude con ruptura prematura
de membranas ovulares está
---------------------- realizarle un tacto vaginal, si niega la existencia de dolores y comprobamos la
ausencia de contracciones.

a. unifactorial – 24 horas - c. multifactorial - 12 horas - indicado -


contraindicado

b. multifactorial - 24 horas - d. multifactorial - 12 horas -


contraindicado - contraindicado -

____ 37. ELIJA LA OPCIÓN CON LAS PALABRAS QUE COMPLETAN CORRECTAMENTE LA SIGUIENTE
PROPOSICIÓN: La ------------------------------- ,es la presencia de bacterias en orina detectada
por -------------------------------------- sin síntomas típicos de infección aguda del TU, su
tratamiento de elección es con -----------------------------.

a. cistitis- elemental y microscópico de c. bacteriuria asintomática - urocultivo -


orina - cefalexina nitrofurantoína.

b. pielonefritis - urocultivo - d. cistitis- urocultivo - nitrofurantoína


nitrofurantoína

11
Name: ________________________ ID: A

la respuesta correcta es la c, ya que el sangrado que se presenta en esta patología es característico,


aparece de forma súbita, generalmente no hay afectación de la frecuencia cardiaca fetal, lo que lo
diferencia de la rotura uterina y rotura de vasa previa es que aunque el sangrado es similar, generalmente
en la rotura uterina hay antecedente de cicatriz uterina o se constata una hiperdinamia uterina.

____ 38. 12.


Paciente de 36 años, secundigesta, antecedentes de salud aparente, cursa embarazo de 34, 1
semanas por FUM. Acude a su SCS refiriendo que de forma sintió salida abundante de sangre
roja rutilante de sus “partes” mientras estaba acostada sin otros síntomas acompañantes,
movimientos fetales presentes. Al examen físico se constata signos vitales dentro de
parámetros normales, FCF: 136 lat/mit, actividad uterina negativa. NST reactivo.

a. Rotura uterina c. Placenta previa


b. Desprendimientoplacentario

____ 39. 13.Paciente de 41 años de edad, antecedentes de salud aparente, último parto hace 12 años; cursa embarazo
de 14,6 semas por ECo, FUM imprecisa. Usted la valora por primera vez y realiza el primer control
prenatal en su área de salud. Analice cuál sería la conducta adecuadacon esta paciente?.
1. Anamnesis, examen físico completo y solicitar exámenes.
2. Anamnesis, examen físico completo y transferencia.
3. Iniciar tratamiento con 1.5 gde calcio elemental dividido en 3 tomas despúes de las comidas y
Ácido aceltil salicílico 75-100 mg diario.
4. Transferencia a hospital de tercer nivel.

a. 1,4 c. 2
b. 1, 3 d. 3,4

____ 40. En una gestante que cursa embarazo de 32 semanas, se constata actividad uterina: 2/10/30 segundos que
además refiere dolor abdominal en hipogastrio de 6 horas de evolución, se le realiza tacto vaginal
encontrándose un cérvix posterior, borrado 30%, dilatado 2 cm, cefálico móvil, membranas íntegras;
FCF: 128.145 lat/mit. Señale cuál sería el tratamiento indicado en esta paciente.

a. 1. Betametasona 12 mg IM cada 24 c.
horas, dos dosis 3.Dexametasona: 6 mg IM cada 12 horas
Nifedipino: 10 mg SL cada 20 minutos por dos dosis.
por 3 dosis, luego: 20 mg SL cada 8 Nifedipino 10 mg VO cada 20 minutos
horas. por 3 dosis, luego 20 mg VO cada 6
horas.
b. 2. Betametasona 12 mg IM cada 24 d. 4. Dexametasona: 6 mg IM cada 12 horas
horas, dos dosis. por 4 dosis.
Nifedipino 10 mg VO cada 20 minutos Nifedipino: 10 mg Vo si TA mayor o
por 3 dosis, luego 20 mg VO cada 6 igual a 160 y/o 110 mmHg.
horas.

12
Name: ________________________ ID: A

____ 41. Usted valora a paciente con un embarazo de 42.3semanas calculadas por FUM, que presenta una altura
uterina de 31 cm, para decidir una conducta solicita realizar un ECO más perfil biofísico. De las
opciones siguientes seleccione la que considere correcta respecto a los parámetros que mide en perfil
bioísiíco.

a. 1. Movimientos corporales, movimientos c. 3. Movimientos fetales, movimientos


respiratorios,, reactividad cardiaca, respiratorios, tono fetal, reactividad
líquido amniótico, tono fetal. cardiaca, líquido amniótico.
b. 2. Movimientos fetales, movimientos d.
corporales, tono fetal, frecuencia 4. Movimientos corporales, movimientos
cardiaca fetal. Indice de líquido respiratorios, tono fetal, reactividad
amniótico. cardiaca. Indice de líquido amniótico.

13
ID: A

COMP GINECOLOGIA
Answer Section

MULTIPLE CHOICE

1. ANS: B
POR PRESENTAR VARIOS FACTORES DE RIESGO Q INCLUYEN LA EDAD, Y EL SER
DIABETICA INSULINODEPENDIENTE SE INDICARIA AC FOLICO 5M , AUN NO SE DA
CALCIO NI AC ACETILSALICILICO X Q AUN NO CURSA LAS 12 SEMANAS

PTS: 1 DIF: ALTA REF: GPC MSP CONTRO PRENATAL PAG 24


OBJ: Identifica los cambios normales en la fisiología de la mujer, sus repercusiones clínicas, y realiza
valoración materna y fetal con criterios científicos y técnico aplicándolo en la práctica clínica de la mujer
embarazada con responsabilidad y ética. TOP: OBSTETRICIA
KEY: KATHERINE CUBILLO
2. ANS: D
AL REVISARLA HISTORIA CLINICA, PODEMOS DARNOS CUENTA Q LA PCTE PRESENTA
FACTORES DE RIESGO BAJOS X EL FACTOR MODIFICABLE DE SU LUGAR DE TRABAJO Y
DE SU EXPOSICION A AGENTESD QUIMICOS . PERO ADEMAS PRESENTA EL
ANTECEDENTE DE AMENAZA DE PARTO PRETERMINO EN EMBARAZO ANTERIOR X LO Q
SE LA CATALOGA COMO EMBARAZO DE MUY ALTO RIESGO
POR SU FUM: CURSA CON EMB DE 24.4SG
POR EL ESTADO NUTRICIONAL SE LA CATOLOGA COMO PESO NORMAL

PTS: 3 DIF: ALTA REF: GPC - MSP


OBJ: Identifica los cambios normales en la fisiología de la mujer, sus repercusiones clínicas, y realiza
valoración materna y fetal con criterios científicos y técnico aplicándolo en la práctica clínica de la mujer
embarazada con responsabilidad y ética. TOP: OBSTETRICIA
KEY: KATHERINE CUBILLO
3. ANS: C
CON LOS SIGNOS VITALES. EL SCORE Q PRESENTA LA PCTE ES DE 10
P. 100= 0; PS 160= 3; PD 95= 2; SAT 92= 0; FR 24= 2; t 37= 0; PROT (+++)= 1; ESTUPOROSA=2
, X LO TANTO LOS CONTROLES SON CADA 30 MIN

PTS: 1 DIF: ALTA REF: GPC-MSP


OBJ: Identifica los cambios normales en la fisiología de la mujer, sus repercusiones clínicas, y realiza
valoración materna y fetal con criterios científicos y técnico aplicándolo en la práctica clínica de la mujer
embarazada con responsabilidad y ética. TOP: OBSTETRICIA
KEY: KATHERINE CUBILLO
4. ANS: B
A TODA PCTE Q ACUDE CON PARTO PRETERMINO Y RPM ES UN FACTOR DE RIESGO PARA
INFECCION DEL PRODUCTO CON SGB . YA Q AUN NO TENEMOS EL CULTIVO PARA SGB X
Q SE REALIZA ENTRE LAS 35 Y 37S ENTONCES HAY Q DAR TTO PROFILACTICO, ADEMAS
X SER ALERGICA A LA PENICILINA SE DEBE DE DAR TTO CON CEFAZOLINA O
CLINDAMICINA.
Y SE DEBE DE REALIZAR MADURACION PULMONAR FETAL X TENER MENOS DE 35
SEMANAS

PTS: 1 DIF: ALTA REF: GPC-MSP CONTROL PRENATAL PAG 22


TOP: OBSTETRICIA KEY: KATHERINE CUBILLO

1
ID: A

5. ANS: C
Se recomienda la profilaxis anti-D en dosis única entre las 28 y 30 semanas a las embarazadas con
factor Rh negativo, que no estén sensibilizadas

PTS: 1 DIF: ALTA REF: GPC-MSP CONTROL PRENATAL PAG 24


OBJ: Identifica los cambios normales en la fisiología de la mujer, sus repercusiones clínicas, y realiza
valoración materna y fetal con criterios científicos y técnico aplicándolo en la práctica clínica de la mujer
embarazada con responsabilidad y ética. TOP: OBSTETRICIA
KEY: KATHERINE CUBILLO
6. ANS: B
Se realizará una glucemia en ayunas en la consulta médica inicial, que se interpretará
de la siguiente manera:
• - Mayor a 126 mg/dL = diabetes preexistente
• - Entre 92 a 126 mg/dL = DG
• - Menor de 92 mg/dL = se realizará una PTOG de 75 g entre las semanas 24
y 28 de gestación (Ver GPC diabetes en el embarazo).

PTS: 1 DIF: ALTA REF: GPC - MSP CONTROL PRENATAL PAG 20


OBJ: Identifica los cambios normales en la fisiología de la mujer, sus repercusiones clínicas, y realiza
valoración materna y fetal con criterios científicos y técnico aplicándolo en la práctica clínica de la mujer
embarazada con responsabilidad y ética. TOP: OBSTETRICIA
KEY: KATHERINE CUBILLO
7. ANS: C
LA RESPUESTA CORRECTA ES C
A.- LA VACUNA ESPECÍFICA ESTA CONTRA INDICADA EN EL EMBARAZO.
B.- LA CUANTIFICACION DE IGG INDICA INMUNIDAD ADQUIRIDA POR INFECCION PASADA.
D.- EL ACICLOVIR ESTA CONTRAINDICADO EN EL EMBARAZO.
E.- LA ADMINISTRACION DE GAMAGLOBULINA ESPECÍFICA NO A DEMOSTRADO SU EFICACIA.

PTS: 1 DIF: Alto


REF: Obstetricia; Schwarez; ENFERMEDADES INDUCIDAS POR EL EMBARAZO O QUE LO
COMPLICAN; pag 297;año 2011
OBJ: Reconoce y trata las principales enfermedades maternas que complican el embarazo basados en
conocimientos actualizados permitiéndole manejar a las pacientes obstétricas con responsabilidad y ética.
TOP: Obstetricia. KEY: Varicely Embarazo MSC: CECILIA CASCO MANZANO
8. ANS: D
LA RESPUESTA CORRECTA ES D
A.- SE TRATA DE UN EMBARAZO CRONOLOGICAMENTE PROLONGADO.
B.- SE CONSIDERA EMBARAZO DE ALTO RIESGO, LOS CONTROLES DEBEN SER PERMENENTES.
C.- ES UN EMBARAZO PROLONGADO NO NECESITA CORTICOIDES.
E.- SI NO EXISTEN CONTRAINDICACIONES MATERNO FETALES LA PRIMERA OPCION SERA EL
PARTO VAGINAL.

PTS: 1 DIF: Alta


REF: MSP (2012) Componente Normativo Materno Neonatal. GPC Trabajo de Parto (2015)
OBJ: Comprende, aplica y trata las etapas del parto que complican el embarazo basados en
conocimientos actualizados permitiéndole manejar a las pacientes obstétricas con responsabilidad y ética.
TOP: Obstetricia KEY: Trabajo de parto NOT: CECILIA CASCO MANZANO

2
ID: A

9. ANS: C
LA RESPUESTA CORRECTA ES C
A.- SE TRATA DE UN FETO EN TERCER PLANO DE HODGE POR LO QUE SE DEBE REALIZAR PARTO
INSTRUMENTAL, LA CESAREA PROGRAMADA TIENE OTRAS INDICACIONES
B.- LA VARIEDAD DE PRESENTACION ES OIIT Y NECESITAMOS UN INTRUMENTO ROTADOR.
D.- UN FETO EN NECESITA PARTO INMEDIATO.
E.- EL OXIGENO A LA MADRE ESTA INDICADO PERO LA ESTRACCION FETAL DEBE SER
INMEDIATA.

PTS: 0 DIF: Alta


REF: MSP (2012) Componente Materno Infantil. GPC Trabajo de Parto (2015)
OBJ: Reconoce y trata las principales enfermedades maternas que complican el embarazo basados en
conocimientos actualizados permitiéndole manejar a las pacientes obstétricas con responsabilidad y ética.
TOP: Obstetricia KEY: Trabajo de Parto NOT: CECILIA CASCO MANZANO
10. ANS: E
LA RESPUESTA CORRECTA ES E
A.- UNA LONGITUD CRANEO CAUDAL ACORDE CON 8 SEMANAS Y SIN LATIDO CARDIACO
CORRESPONDE UN ABORTO DIFERIDO.
B.- AMENAZA DE ABORTO = FETO VIVO; SE RECOMIENDA REPOSO Y GESTÁGENOS.
C.- CUANDO SOSPECHAMOS EN EMBARAZO ECTOPICO SOLICITAMOS DETERMINACIÓN DE BHCG
PLASMÁTICA CADA 48 H.
D.- POR EG NO ES POSOBLE LLEGAR A ESE DIAGNOSTICO.

PTS: 1 DIF: Alta REF: MSP, (2015).GPC Aborto


OBJ: Comprende, analiza y trata las enfermedades que complican el embarazo basados en
conocimientos actualizados permitiéndole manejar a las pacientes obstétricas con responsabilidad y ética.
TOP: Obstetricia KEY: Aborto NOT: CECILIA CASCO MANZANO
11. ANS: A
LA RESPUESTA CORRECTA ES A
B.- LA HEMORRAGIA EN PLACENTA PREVIA ES INDOLORA.
C.- LA INSERCIÓN VELAMENTOSA DE CORDÓN UMBILICAL-VASA PREVIA PRESENTA
HEMORRAGIA POSTERIOR A RPM.
D.-LA CLINICA DE NECROSIS DE UN MIOMA NO INCLUYE HEMORRGIA TRANSVAGINAL.
E.- TORSIÓN DE UN TUMOR OVÁRICO NO INCLUYE HEMORRGIA TRANSVAGINAL

PTS: 1 DIF: Alta REF: MSP, (2012) Componente Normativo Materno


Neonatal.
OBJ: Comprensión conceptual, analiza y trata las enfermedades que complican el embarazo y parto
basados en conocimientos actualizados permitiéndole manejar a las pacientes obstétricas con
responsabilidad y ética.
TOP: Obstetricia KEY: Desprendimiento de Placenta Normoincerta
NOT: CECILIA CASCO MANZANO

3
ID: A

12. ANS: E
LA RESPUESTACORRECTA ES E
A.- EL EMBOLISMO DE LÍQUIDO AMNIÓTICO PROVOCA DIFICULTAD RESPIRATORIA EN MUCHOS
DE LOS CASOS FULMINANTE.
B.- VALORES DE TA ELEVADOS NO SE JUSTIFICAN EN MUERTE FETAL INTRA UTERO.
C.- EL DESPRENDIMIENTO PREMATURO DE PLACENTA PUEDE SER UNA CONSECUENCIA DE
PROBLEMAS HIPERTENSIVOS EN EL EMBARAZO.
D.- EL CUADRO CLINICO DE ISOINMUNIZACIÓN ERITROCITARIA MATERNA NO CORRESPONDE.

PTS: 1 DIF: Alta


REF: MSP; (2016). GPC Transtornos Hipertensivos del Embarazo
OBJ: Aplicación conceptual de las principales enfermedades maternas que complican el embarazo y
parto basados en conocimientos actualizados permitiéndole manejar a las pacientes obstétricas con
responsabilidad y ética. TOP: Obstetricia. KEY: Transtornos Hipertensivos del
Embarazo
NOT: CECILIA CASCO MANZANO
13. ANS: E
LA RESPUESTA CORRECTA ES E.
A.- DETERMINACIÓN DE GLUCOSA EN ORINA NO HACE EL DIAGNOSTICO
B.- DETERMINACIÓN DE GLICEMIA EN AYUNAS POR SI SOLA NO ES CONCLUYENTE.
C.- DETERMINACIÓN DE GLICEMIA EN AYUNAS EN DÍAS DIFERENTE, NO ES CONCLUYENTE.
D.-TEST DE TOLERANCIA ORAL A LA GLUCOSA CON 100G DE SOBRECARGA ES UNA PRUEBA
ALTAMENTE SENSIBLE Y ESPECÍFICA PERO NO ES EL METODO DE SCREENING.

PTS: 1 DIF: Alta REF: MSP, (2015) .GPC Diabetes y Embarazo


OBJ: Aplicación conceptual sobre la clasificaciónde las principales enfermedades maternas que
complican el embarazo basados en conocimientos actualizados permitiéndole manejar a las pacientes
obstétricas con responsabilidad y ética.. TOP: Obstetricia KEY: Diabetes y
Embarazo
NOT: CECILIA CASCO MANZANO
14. ANS: B
LA RESPUESTA CORRECTA ES B
A.- HIPERTENSION ARTERIAL BIEN CONTROLADA NO CONTRAINDICA EL USO DE
ANTICONCEPCION HORMONAL.
C.- DIABETES NO CONTRAINDICA EL USO DE ANTICONCEPCION HORMONAL.
D.- MUJER FUMADORA DE CUALQUIER EDAD ES UN FACTOR DE RIESGO QUE PUEDE
CONTRAINDICAR LA ANTICONCEPCION HORMONAL PERO NODE FORMA ABSOLUTA
E.- INFECCION URINARIA NO TIENE INCONVENIENTES CON SU USO

PTS: 1 DIF: Alto


REF: MSP, (2012). Componente Materno Infantil Guia de Manejo en Planificacion Familiar
OBJ: Comprensión y aplicación conceptual de las principales enfermedades maternas que complican el
embarazo y parto basados en conocimientos actualizados permitiéndole manejar a las pacientes
obstétricas con responsabilidad y ética. TOP: Obstetricia. KEY: Planificacion
Familiar
NOT: CECILIA CASCO MANZANO

4
ID: A

15. ANS: A
LA RESPUESTA CORRECTA ES A
B.- LA LEUCORREA EN CANDIDIASIS SUELE SER GRUMOSA Y EN TRICOMONIASIS PRESENTA
BURBUJAS.
C.- LAS CELULAS CLAVE SON CELULAS LLENAS DE COCOBACILOS GRAM NEGATIVOS PROPIOS DE
VAGINOSIS BACTERIANA.
D.- EL ERITEMA VULVAR ES PROPIO DE LA CANDIDIASIS
E.- EL PRURITO VULVAR ES PROPIO DE LA CANDIDIASIS

PTS: 1 DIF: Alto REF: MSP, (2014). Manejo de ETS.


OBJ: Comprensión conceptual, analiza y trata las enfermedades que complican el embarazo y parto
basados en conocimientos actualizados permitiéndole manejar a las pacientes obstétricas con
responsabilidad y ética.
TOP: Obstetricia. KEY: ETS NOT: CECILIA CASCO MANZANO
16. ANS: A
LA RESPUESTA CORRECTA ES A
B.- TIENE LUGAR SOBRE TODO EN EL PARTO ES CORRECTO.
C.- LA LACTANCIA ESTA CONTRAINDICADA EN ESTAS PACIENTES.
D.- EL INICIO DE LA MEDICACION ANTIRETROVIRAL DEPENDERA DE LA CARGA VIRAL Y DE LA
EDAD GESTACIONAL
E.- EL TRATAMIENTO CON ZIDOVUDINA NO HA DEMOSTRADO EFECTOS ADVERSOS EN EL RECIÉN
NACIDO HASTA LOS 5 AÑOS ES CORRECTO

PTS: 1 DIF: Alto REF: MSP, (2013) Manejo de VIH y Embarazo


OBJ: Aplicación de la terminología, comprende, analiza y trata las enfermedades que complican el
embarazo basados en conocimientos actualizados permitiéndole manejar a las pacientes obstétricas con
responsabilidad y ética. TOP: Obstetricia. KEY: VIH y Embarazo.
NOT: CECILIA CASCO MANZANO
17. ANS: B
LA RESPUESTA CORRECTA ES B
A.- CESÁREA NO PORQUE TIENE CONDICIONES OBSTETRICA PARA PARTO VAGINAL NORMAL
C.- NO ES EMBZ MENOR DE 34.6 SEMANAS EL BISHOP AUN ES DEFABORABLE Y CON DG DE RPM SE
DEBE TERMINAR EL EMBARAZO INMEDIATAMENTE
D.- EN EMBARAZO A TERMINO Y RPM SE DEBE TERMINAR EL EMBARAZO
E.- NO ESPERAR EL COMIENZO ESPONTANEO POR SER EMBZ A TERMINO Y AUN CON SG DE
INFECCION LA MEJOR VIA PARA TERMINACION ES LA VIA BAJA

PTS: 1 DIF: Alta


REF: MSP, (2014). GPC Ruptura Prematura de Membranas Pretermino
OBJ: Comprende, analiza y trata las enfermedades que complican el embarazo basados en
conocimientos actualizados permitiéndole manejar a las pacientes obstétricas con responsabilidad y
ética..
TOP: Obstetricia. KEY: Ruptura Prematura de Membranas NOT: CECILIA CASCO MANZANO

5
ID: A

18. ANS: B
LA RESPUESTA CORRECTA ES B.
A. La edad gestacional no justifica la realizacion de ecografia en 48 horas, puesto que observaremos saco
gestacional entre las 4-5 semanas y embrión entre 5-6 semanas,
C.No existe imagen intrauterina que justifique el LIU
D. La BHCG esta acorde a la edad gestacional, en ovario no existe imagen sospechosa por lo que se
descarta el embarazo ectópico
E.No existe imagen extrauterina que justifique la laparotomia o laparoscopia

PTS: 1 DIF: alta REF: Guia de Practica Clinca de Aborto 2013


OBJ: Comprende, analiza y trata las enfermedades que complican el embarazo basados en
conocimientos actualizados permitiéndole manejar a las pacientes obstétricas con responsabilidad y ética.
TOP: Obstetricia. KEY: Aborto NOT: Cecilia Margarita Casco Manzano
19. ANS: C
LA RESPUESTA CORRECTA ES C
A. la profilaxis esta indicada durante el trabajo de parto inminente
B. en todas las pacientes alergicas a la ampicilana la segunda opcion es la eritromicina
D. el isopado de area perineal y perianal se debe solicitar a partir de las 35 semanas si la prueba es
positva la paciente debe resibir profilaxis durante el trabajo de parto independientemente de su semanas
de gestación
E. la incidencia de infecion neonatal pro estreptoco betahemolitico del grupo B, es mayor es en Rn
prematuros puesto que para ellos constituye mas una causa de inicio de trabajo prematuro que una
consecuencia del mismo

PTS: 1 DIF: Alta


REF: Guia de Practica Clínica de Trabajo de Parto 2015, Guia de Practica Clinica de RPM 2015.
OBJ: Reconoce y trata las principales enfermedades maternas que complican el embarazo basados en
conocimientos actualizados permitiéndole manejar a las pacientes obstétricas con responsabilidad y ética.
TOP: Obstetricia KEY: Estreptococo NOT: Cecilia Casco Manzano
20. ANS: D
LA RESPUESTA CORRECTA ES LA D
A. el diagnostico de preeclampsia supone una EG mayor a 20 semanas a ecepción de 3 casos especiales
que no corresponden a la paciente
B. sin contar sus antecedentes previos, anterioes a las 28 semanas, corresponderia a una preeclampsia
grave
C. al existir TA elevada antes de las 20 semanas, hipertensión cronica evidenciada antes de las 20
semanas se trata de una paciente hipertensa crónica
E. se evidecnia TA alta antes de las 20 semanas lo que descarta un diagnostico de hipertensión
gestacional

PTS: 1 DIF: Alta


REF: Guia de Practica Clinca de Trastornos Hipertensivos del 2017
OBJ: Reconoce y trata las principales enfermedades maternas que complican el embarazo basados en
conocimientos actualizados permitiéndole manejar a las pacientes obstétricas con responsabilidad y ética.
TOP: Obstetricia KEY: Preeclampsia NOT: Cecilia Casco Manzano

6
ID: A

21. ANS: C
LA RESPUESTA CORRECTA ES C
A. el diagnostico de Sindrome HELLP se realiza en base a los resultados de laboratorio uno de ellos es la
plaquetopenia
B. el diagnostico de Sindrome HELLP se realiza en base a los resultados de laboratorio en donde la
bilirrubina indirecta se eleva a concecuencia de un proceso de hemolisis.
D. el proceso de hemolisis es marcado lo que se refleja como anemia-
E. el diagnostico de Sindrome HELLP se realiza en base a los resultados de laboratorio uno de ellos son
las transaminasas.

PTS: 1 DIF: Alta REF: Guia de Practica Clínica de Trastornos Hipertensivos


2017
OBJ: Reconoce y trata las principales enfermedades maternas que complican el embarazo basados en
conocimientos actualizados permitiéndole manejar a las pacientes obstétricas con responsabilidad y ética.
TOP: Obstetricia. KEY: HELLP NOT: Cecilia Casco Manzano
22. ANS: A
LA RESPUESTA CORRECTA ES A
B. toda longitud cevical inferior a 2. 5 cm es un indicador predictivo positivo para amenaza de partoo
pretermino
C. toda paciente con amenaza de parto pretermino entre las 24 y 34.6 semanas debe recibir maduracion
pulmonar fetal
D. la amenaza de parto pretermino incierta presupone AU: irregular sin cambios cervicales, que en
muchas de las ociaciones se resuelven con reposo.

PTS: 1 DIF: Alta REF: Guia de Practica Clinica de Trabajo de Parto 2015
OBJ: Reconoce y trata las principales enfermedades maternas que complican el embarazo basados en
conocimientos actualizados permitiéndole manejar a las pacientes obstétricas con responsabilidad y ética.
TOP: Obstetricia. KEY: Parto pretermino NOT: Cecilia Casco Manzano
23. ANS: A
LA RESPUESTA CORRECTA ES A
B. no se puede suponer parto si existen signos clinicos de infeccion herpetica genital activa
C. todo herpes residivante en fase activa representa un riesgo para el RN de considerarse parto normal
D. si la lesion herpetica esta activa no debe considerarse parto normal el tratemiento puede demorarse
mas alla de 7-15 días
E. no se recomiendo el parto vaginal y no tiene sentido aislar al RN de su madre.

PTS: 1 DIF: Alta REF: Guia de Practica Clinica de Parto por Cesarea 2014
OBJ: Reconoce y trata las principales enfermedades maternas que complican el embarazo basados en
conocimientos actualizados permitiéndole manejar a las pacientes obstétricas con responsabilidad y ética.
TOP: Obstetricia KEY: Herpes NOT: Cecilia Casco Manzano

7
ID: A

24. ANS: C
LA RESPUESTA CORRECTA ES LA C
A. duratne los 2 primeros años inciados la menarquia, es común la amenorrea por anovulación y no
constituye un ente patológico
B. el ovario poliquistico amerita tratamiento pero no es la causa de hemorragias uterinas anormales
unicamente en el primer año tras el inicio de la menarquia
D. es una condición bastante infrecuente con una incidencia baja inferior a 0.07%
E. un EPI no justifica hemorragias uterinas en ninguna etapa de la vida fertil.

PTS: 1 DIF: Alta


REF: Guia de Practica Clinica de Sindrome Ovario Poliquistico Española 2016
OBJ: Reconoce y trata las principales enfermedades ginecologicas basados en conocimientos
actualizados permitiéndole manejar a las pacientes con responsabilidad y ética.
TOP: Ginecologia KEY: anovulación NOT: Cecilia Casco Manzano
25. ANS: D
LA RESPUESTA CORRECTA ES D
A. El acido clavulanico durante el embarazo esta contraindicado de enterolitis necrotisante
B. No se trata por una infección micotica.
C. Esta contraindicado en el embarazo

PTS: 1 DIF: Alta


REF: Guia de Practica Clínica de Infecciones Vaginales en el Embarazo 2016
OBJ: Reconoce y trata las principales enfermedades maternas que complican el embarazo basados en
conocimientos actualizados permitiéndole manejar a las pacientes obstétricas con responsabilidad y ética.
TOP: Obstricia KEY: Infecciónes vaginales NOT: Cecilia Casco Manzano
26. ANS: D
LA RESPUESTA CORRECTA ES D
A. Toda anemia crónica debe ser manejada con aporte de hierro según valores de hemoglobina
B. EL hierro intravenoso esta indicado en anemia moderado
C. Se transfundira a pacientes obstetricas con valores de hemogloibana menores de 7 tomando en cuenta
si existe o no signos de descompensación
E. Se puede indicar sulfato ferroso durante la gestacion pero tomando en cuenta que el aporte de hierro
elemental sea de 100 -200 mg

PTS: 1 DIF: Alta REF: Guia de Practica Clinica de Anemia en el Embazaro


2016
OBJ: Reconoce y trata las principales enfermedades maternas que complican el embarazo basados en
conocimientos actualizados permitiéndole manejar a las pacientes obstétricas con responsabilidad y ética.
TOP: obstetricia KEY: Anemia NOT: Cecilia Casco Manzano
27. ANS: C
La respuesta correcta es la c. puesto que el sulfato de magnesio es el medicamento propuesto como de
primera elección en el tratamiento de las convulsiones en el embarazo.
La pregunta puede ser respondida empleando entre 1 y 3 minutos.

PTS: 1 DIF: Alto


REF: MSP (2016) Transtornos hipertensivos del embarazo. Guía de práctica clínica, segunda edición.
OBJ: Reconoce y trata las principales enfermedades maternas que complican el embarazo basados en
conocimientos actualizados permitiéndole manejar a las pacientes obstétricas con responsabilidad y ética.
TOP: Obstetricia. KEY: Preeclampsia MSC: Anabela Criollo Criollo

8
ID: A

28. ANS: A
La respuesta correcta es la a. ya que esos son los pasos subsecuentes y ordenados a seguir para evitar
complicaciones en la tercera etapa del trabajo de parto, para su solución se deben emplear entre 1 y 3
minutos.

PTS: 1 DIF: Alta REF: MSP (2012) Componente Normativo Materno


Neonatal.
OBJ: Comprende, aplica y trata las etapas del parto que complican el embarazo basados en
conocimientos actualizados permitiéndole manejar a las pacientes obstétricas con responsabilidad y ética.
TOP: Obstetricia KEY: Trabajo de parto NOT: Anabela Criollo Criollo
29. ANS: C
La respuesta corecta es la c. ya tanto el labetalol como la nifedipina se recomiendan como primera línea
por ser igual de eficaces; pero, el labetalol no consta en la lista de medicamentos escenciales, no hay en el
país. Se considera que se requerirán de 1 a 3 minutos para su respuesta.

PTS: 0 DIF: Alta


REF: MSP (2016) Transtornos hipertensivos del embarazo. Guía de práctica clínica, segunda edición.
OBJ: Reconoce y trata las principales enfermedades maternas que complican el embarazo basados en
conocimientos actualizados permitiéndole manejar a las pacientes obstétricas con responsabilidad y ética.
TOP: Obstetricia KEY: Trastornos hipertensivos del embarazo.
NOT: Anabela Criollo Criollo.
30. ANS: C
La respuesta correcta es la c. porque estos planteamientos son los recomendados como esquema de
profilaxis antibiótica de primera línea por 7 días si no aparecen complicaciones que indiquen interrupción
inmediata del embarazo.
Se consideran de 1 a 3 minutos suficientes para responder la pregunta.

PTS: 1 DIF: Alta


REF: MSP, (2015). Ruptura prematura de membranas pretérmino. Guía de práctica clínica
OBJ: Comprende, analiza y trata las enfermedades que complican el embarazo basados en
conocimientos actualizados permitiéndole manejar a las pacientes obstétricas con responsabilidad y ética.
TOP: Obstetricia KEY: Ruptura prematura de membranas NOT: Anabela Criollo criollo.
31. ANS: B
La respuesta correcta es la opción b. porque estas son los movimientos cardinales o mecanismos del parto
realizados por el bebé y descritos en la literatura.
Se considera que la respuesta puede ser ofrecida entre 1 y 3 minutos.

PTS: 1 DIF: Alta REF: MSP, (2012) Componente Normativo Materno


Neonatal.
OBJ: Comprensión conceptual, analiza y trata las enfermedades que complican el embarazo y parto
basados en conocimientos actualizados permitiéndole manejar a las pacientes obstétricas con
responsabilidad y ética.
TOP: Obstetricia KEY: Trabajo departo NOT: Anabela criollo Criollo

9
ID: A

32. ANS: A
La respuesta correcta es la a. Es la definición correcta de la pérdida sanguinea esperada y considerada
como normal.
Se considera que la respuesta puede ser dada en un tiempo entre 1 y 3 minutos.

PTS: 1 DIF: Alta


REF: MSP; (2013). Prevención, diagnóstico y tratamiento de la hemorragia posparto. Guía de práctica
clínica.
OBJ: Aplicación conceptual de las principales enfermedades maternas que complican el embarazo y
parto basados en conocimientos actualizados permitiéndole manejar a las pacientes obstétricas con
responsabilidad y ética. TOP: Obstetricia. KEY: Hemorragia posparto.
NOT: Anabela Criollo Criollo.
33. ANS: B
La respuesta correcta es la b. porque es la que recoge las características más aceptadas por la comunidad
científica acerca de la clasificación actual de los trastornos hipertensivos.
El estudiante contará con entre 1 y 3 minutos para contestarla.

PTS: 1 DIF: Alta


REF: MSP, (2016) Transtornos hipertensivos del embarazo. Guía de práctica clínica, segunda edición.
OBJ: Aplicación conceptual sobre la clasificaciónde las principales enfermedades maternas que
complican el embarazo basados en conocimientos actualizados permitiéndole manejar a las pacientes
obstétricas con responsabilidad y ética.. TOP: Obstetricia KEY: Trastornos
hipertensivos
NOT: Anabela Criollo criollo.
34. ANS: A
La opción correcta es la a. Ese es el orden de frecuencia descrito en la literatura médica y evidenciado en
la práctica.Se contará con enter 1 y 3 minutos para responder la pregunta.

PTS: 1 DIF: Alto


REF: MSP, (2013). Prevención, diagnóstico y tratamiento de la hemorragia posparto. Guía de práctica
clínica.
OBJ: Comprensión y aplicación conceptual de las principales enfermedades maternas que complican el
embarazo y parto basados en conocimientos actualizados permitiéndole manejar a las pacientes
obstétricas con responsabilidad y ética. TOP: Obstetricia. KEY: Hemorragia
posparto.
NOT: Anabela Criollo Criollo
35. ANS: B
La respuesta correcta es la b. La GPC y el ADA 2016 describen esos valores como parámetros de corte
máximo de la PTGO, uno solo de ellos alterado nos orienta al diagnóstico de Diabetes Gestacional,
Se contará con entre 1 y 3 minutos para ofrecer la respuesta

PTS: 1 DIF: Alto


REF: MSP, (2014). Diagnóstico y tratamiento de la diabetes en el embarazo (pregestacional y
gestacional). Guía de práctica clínica.
OBJ: Comprensión conceptual, analiza y trata las enfermedades que complican el embarazo y parto
basados en conocimientos actualizados permitiéndole manejar a las pacientes obstétricas con
responsabilidad y ética.
TOP: Obstetricia. KEY: Diabetes y embarazo. NOT: Anabela Criollo Criollo.

10
ID: A

36. ANS: D
La opción correcta es la d. porque es la terminología más aceptada, y el tratamiento profiláctico se inicia
a las 12 horas posteriores a la RPM confirmada. Se contará con entre 1 y 3 minutos para responder la
pregunta.

PTS: 1 DIF: Alto


REF: MSP, (2015). Ruptura prematura de membranas pretérmino. Guía de práctica clínica
OBJ: Aplicación de la terminología, comprende, analiza y trata las enfermedades que complican el
embarazo basados en conocimientos actualizados permitiéndole manejar a las pacientes obstétricas con
responsabilidad y ética. TOP: Obstetricia. KEY: Ruptura prematura de
membranas.
NOT: Anabela Criollo Criollo.
37. ANS: C
La respuesta correcta es la c. porque esta patología no tiene síntomas clínicos y el diagnóstico se hace a
traves de un urocultivo positivo que demuestre la presencia de más de 100.000 col de bacterias en orina y
el tratamiento de primera línea es con nitrofurantoina. Se contará con entre 1 y 3 minutos para dar
respuesta a la pregunta

PTS: 1 DIF: Alta


REF: MSP, (2013). Infección de vías urinarias en el embarazo. Guía de práctica clínica.
OBJ: Comprende, analiza y trata las enfermedades que complican el embarazo basados en
conocimientos actualizados permitiéndole manejar a las pacientes obstétricas con responsabilidad y
ética..
TOP: Obstetricia. KEY: Infección de vías urinarias. NOT: Anabela Criollo Criollo.
38. ANS: C
La respuesta correcta es la c, ya que el sangrado que se presenta en esta patología es característico,
aparece de forma insidiosa, generalmente no hay dolor ni afectación de la frecuencia cardiaca fetal y el
útero esta relajado, lo que lo diferencia de la rotura uterina y del desprendimiento placentario.

PTS: 1 DIF: Alta REF: MSP, (2012) Componente Normaivo materno


neonatal
OBJ: Analiza los signos y síntomas, utilizando el método clínico y realiza diagnóstico diferencial entre
las patologías que comprenden las hemorragias de la segunda mitad de la gestación.
TOP: obstetricia KEY: Hemorragias de la segunda mitad de la gestación
NOT: Anabela Criollo Criollo
39. ANS: B
La respuesta correcta es la b, ya que en la primera consulta prenatal se valora y examina a la paciente de
forma integral, se solicitan todos los exámenes y se clasifica a paciente tomando en cuenta los factores de
riesgo que presente. En este caso tiene dos factores de riesgo moderado: tener más de 40 años y un
periodo intergenésico prolongado por lo que se inicia con calcio y aspirina a las dosis indicadas desde las
12 semanas de embarazo hasta el final de este como prevención primaria de la preeclampsia.

PTS: 1 DIF: Alta


REF: MSP (2016) Trastornos hipertensivos del embarazo. Guía de práctica clínica, segunda edición.
MSP (2016) Control Prenatal. Guía de Práctica clínica.
OBJ: Analiza y a aplica el método clínico que le permite valorar de forma integral y clasificar según el
riesgo a la paciente y realizar acciones de promoción y prevención de futuras complicaciones en el curso
del embarazo. TOP: Obstetricia KEY: Control prenatal y Trastornos hipertensivos del
emabarzo
NOT: Anabela Criollo Criollo

11
ID: A

40. ANS: B
La respuesta correcta es la b, ya que la paciente tiene una amenaza de parto pretérmino y hay que iniciar
maduración pulmonar con corticoides, de elección la betametasona: 12 mg IM cada 24 horas por dos
dosis y, realizar tocólisis con nifedipina: 10 mg VO cada 20 minutos por 3 dosis, seguido de 20 mg VO
cada 6 horas.

PTS: 1 DIF: Alta REF: MSP (2012) Componente Normativo Materno


Neonatal.
OBJ: Analizar y llegar a un diagnóstico para saber cuál es la conducta adecuada a seguir.
TOP: Obstetricia. KEY: Amenaza de parto pretérmino NOT: Anabela criollo criollo.
41. ANS: D
La respuesta correcta es la d, ya que los cinco parámetros que mide el perfil biofísico son: movimiento
corporales, movimientos respiratorios, reactividad cardiaca, índice de líquido amniótico y tono fetal, y
esta indicado cuando sospechamos una restricción del crecimiento intrauterino.

PTS: 1 DIF: Alta REF: MSP (2012) Componente Normativo Materno


Neonatal.
OBJ: Conocer cuáles son los parámetros que mide el perfil biofísico para saberlo interpretar y tomar una
conducta acertada según su resultado. TOP: Obstetricia.
KEY: Restricción del crecimiento intrauterino. NOT: Anabela Criollo Criollo

12
COMP GINECOLOGIA [Answer Strip] ID: A

B
_____ 4. C
_____ 7. E 10.
_____ B 14.
_____

B
_____ 1.

A 15.
_____

A 11.
_____ A 16.
_____
D
_____ 8.
D
_____ 2.

C
_____ 5.

B
_____ 6. E 12.
_____
B 17.
_____

C
_____ 9.
C
_____ 3.

E 13.
_____
COMP GINECOLOGIA [Answer Strip] ID: A

B 18.
_____ A 23.
_____ A 28.
_____ B 31.
_____ B 33.
_____

C 29.
_____

C 19.
_____
C 24.
_____

C 30.
_____

A 32.
_____

D 20.
_____ D 25.
_____

A 34.
_____

C 21.
_____ D 26.
_____

A 22.
_____

C 27.
_____
COMP GINECOLOGIA [Answer Strip] ID: A

B 35.
_____ D 41.
_____

C 38.
_____

D 36.
_____
B 39.
_____

C 37.
_____
B 40.
_____
Name: ________________________ Class: ___________________ Date: __________ ID: A

COMP PEDIATRIA

Multiple Choice
Identify the choice that best completes the statement or answers the question.

____ 1. ¿CUAL ES EL CONCEPTO ASMA BRONQUIAL SEGUN LA OMS?

a. El asma bronquial es una enfermedad c. Los síntomas pueden sobrevenir varias


crónica que se caracteriza por ataques veces al día o a la semana, y en algunas
recurrentes de disnea y sibilancias, que personas se agravan durante la actividad
varían en severidad y frecuencia de una pasiva o por la noche.
persona a otra.

b. Se define como una enfermedad d. En individuos no susceptibles la


inflamatoria aguda de las vías aéreas inflamación produce episodios
recurrentes de disnea, sibilancias,
opresión torácica y tos

____ 2. ¿CUAL ES LA EPIDEMIOLOGIA DEL ASMA BRONQUIAL?

a. La OMS calcula que en la actualidad hay c. El asma está presente en todos los países,
335 millones de pacientes con asma dependientemente de su grado de
desarrollo.

b. El asma es la enfermedad crónica más d. Más del 80% de las muertes por asma
frecuente en los adultos tienen lugar en países de ingresos bajos y
medios-bajos

1
Name: ________________________ ID: A

____ 3. ¿CUALES SON LOS FACTORES PREDISPONENTES DEL ASMA BRONQUIAL?

a. En el 40% de los casos existen c. En el asmático existe un predominio de


antecedentes de familiares afectos de la actividad adrenérgica (hiperreactividad
diversas enfermedades alérgicas, bronquial), posiblemente por alteración
apoyando el papel de la herencia, no en de los receptores â2-adrenérgicos.
cuanto a padecer asma, sino como
predisposición alérgica.

b. En los linfocitos T cooperadores se d. En el asma el epitelio es más frágil por la


distingue la subclase Th1, productora de disrupción de esas uniones intercelulares,
interferón gamma (IFNã), que actúa con la pérdida fácil de células circulares.
frente a agentes infecciosos, y la Th2,
que produce interleucinas 4 y 5 (IL-4)
(IL-5),la IL-4 provoca la producción de
IgE por los linfocitos B específicos,
mientras que la IL-5 se comporta como
quimiotáctica para los eosinófilos.

____ 4. ¿CUALES SON LOS EFECTOS DE LOS FACTORES DESENCADENANTES INESPECIFICOS DEL
ASMA BRONQUIAL?

a. En el lactante la inmadurez inmunitaria c. El asma infantil tiene más incidencia en


desfavorece las infecciones, sobre todo las regiones de clima húmedo y, en
por virus, y el déficit selectivo de IgA, diferentes condiciones de humedad, en
más común en los pacientes asmáticos las regiones más frías

b. Son factores favorecedores los cambios d. Contaminantes ambientales tienen un


bruscos de temperatura, a través de papel evidente sobre todo en pequeñas
mecanismos inespecíficos, al motivar en poblaciones industriales, pero también
el árbol respiratorio estados de edema actúa en igual sentido la vivienda con
bronquial o reactivando procesos polvo, mala ventilación, ciertas industrias
infecciosos de vías respiratorias altas. o profesiones caseras

____ 5. ¿ CUALES SON LOS FACTORES ETIOLOGIOS ALERGENOS DEL ASMA BRONQUIAL?

a. Alérgenos por inhalación son sustancias c. La alergia a la leche se manifiesta


pulverulentas que están en suspensión en también en el aparato respiratorio como
el aire y penetran en el árbol respiratorio “síndrome de Heiner” debido a la
con los movimientos de inspiración sensibilización indirecta del árbol
respiratorio

b. En el polvo hay la presencia de enzimas d. El ácido acetilsalicílico, por un


procedentes de los ácaros, especialmente mecanismo diferente de estimulación del
el Dermatophagoides pteronyssinus, metabolismo de la ciclooxigenasa, causa
parásito habitual de las viviendas crisis de disnea, asociándose en los
mismos pacientes rinitis/sinusitis y
pólipo nasal.

2
Name: ________________________ ID: A

____ 6. ¿CUALES SON LOS TIPOS DE ASMA BRONQUIAL DEPENDIENDO DEL FACTOR
DESENCADENANTE DE LA CRISIS?

a. Aguda o crónica. c. Leve, moderada o grave.

b. Extrínseca e Intrínseca. d. Infecciosa o no infecciosa.

____ 7. ¿ EL ASMA EXTRINSECA PUEDE INICIARSE POR EXPOSICION A FACTORES COMO?

a. Un agente extrínseco o a un alergeno. c. La inhalación de sustancias químicas.

b. Ejercicio o actividad física prolongada. d. Cambios hormonales y trastornos


emocionales.

____ 8. ¿ CUALES SON LOS FACTORES DESENCADENANTES DEL ASMA INTRINSECA?

a. Ácaros del polvo casero, alérgenos de c. Ejercicio, hiperventilación, inhalación de


cucarachas. sustancias químicas.

b. Caspa de animales o ciertos hongos. d. Cierto tipo de comidas como grasas o


azucares muy concentrados.

____ 9. ¿CUÁLES SON LAS CARACTERÍSTICAS DE LA FASE TEMPRANA DEL ASMA EXTRÍNSECA?

a. Ser de remisión espontanea sin necesidad c. Presentarse a los 30-50 minutos y se debe
de soporte farmacológico. a la liberación de mediadores químicos
provenientes de células cebadas pre
sensibilizadas recubiertas con IgE.

b. Presentarse a los 10-20 minutos y se debe d. Presentarse a los 10-20 minutos y se debe
a la liberación de mediadores químicos a la liberación de mediadores químicos
provenientes de células cebadas pre provenientes de células cebadas pre
sensibilizadas recubiertas con IgM. sensibilizadas recubiertas con IgE.

____ 10. ¿A TRAVÉS DE QUÉ MECANISMOS ESTÁ PRODUCIDA EL ASMA INTRÍNSECA ORIGINADA
POR INHALACIÓN DE HUMO DE TABACO?

a. Estimulación de receptores en los c. Por el mal acondicionamiento del aire


bronquios y producción de reflejo vagal. inspirado.

b. Por el mal acondicionamiento del aire d. Por fibrosis de estructuras respiratorias.


inspirado.

____ 11. ¿CUÁL ES EL INTERVALO CLAVE PARA UN DIAGNÓSTICO CLÍNICO DEL AMA?

a. La función respiratoria. c. Disnea.

b. La tos. d. Sibilancias.

3
Name: ________________________ ID: A

____ 12. ¿CON QUE PRUEBA SE INICIA LA INVESTIGACIÓN DEL ALÉRGENO PARA UN
DIAGNOSTICO ETIOPATOGENICO?

a. Elevacion de la Inmuglobulina E. c. Examen clínico

b. Pruebas Cutaneas (prick-test) d. Aumento de eosinofilia .

____ 13. ¿MEDIANTE QUE PRUEBA SE VALORA LA PROVOCACIÓN BRONQUIAL?

a. Pruebas cutaneas. c. Espirometria

b. Rinometría. d. Prueba biologica

____ 14. ¿DESDE QUÉ EDAD ES FACTIBLE REALIZAR LA ESPIROMETRÍA?

a. 2-3 años c. 1 año.

b. 3-5 años. d. 6-7 años.

____ 15. ¿QUÉ VALORA LA ESPIROMETRIA?

a. Volumen espiratorio máximo. c. Hiperreactividad.

b. Mesoflujo espiratorio. d. Ejercicio físico.

____ 16. ¿EN QUÉ CONSISTE LA TERAPIA FARMACOLÓGICA EN LAS CRISIS ASMÁTICAS LEVES?

a. Administrar un β2-mimético por vía c. Si la crisis asmática es leve solo se debe


inhalatoria (u oral, en los casos muy tomar una conducta expectante.
leves), con preferencia salbutamol o
terbutalina.

b. Administrar β-miméticos inhalados y d. Administrar β-miméticos en nebulización


corticoides orales. continua con O2 hasta alcanzar
saturación > 95% y Metil-prednisolona
EV.

____ 17. ¿LA DOSIS INDICADA DE SALBUTAMOL, EN EL TRATAMIENTO DEL ASMA BRONQUIAL
LEVE, ES DE:?

a. 100 ug, cada 30 minutos, hasta tres c. 200 μg, cada 30 minutos, hasta tres
veces. veces, (suele ser suficiente cada 6-8
horas).

b. 100 ug, cada 30 minutos, hasta tres d. 400 ug, cada hora, (suele ser suficiente
veces, (suele ser suficiente cada 12 cada 12 horas).
horas).

4
Name: ________________________ ID: A

____ 18. EN EL TRATAMIENTO NO FARMACOLÓGICO DE LAS CRISIS ASMÁSTICAS. ¿QUÉ POSTURA


PERMITE RELAJAR LA MUSCULATURA ABDOMINAL, FIJAR LA CINTURA ESCAPULAR Y
FAVORECER LA RESPIRACIÓN DIAFRAGMÁTICA?

a. No influye la posición. c. Decúbito lateral con los brazos cruzados


y piernas flexionadas

b. Sentado con el tronco inclinado hacia d. Posición de trendelemburg


atrás.

____ 19. EN EL TRATAMIENTO NO FARMACOLÓGICO DE LAS CRISIS ASMÁSTICAS. ¿EN QUÉ CASOS
ESPECIALES SE DEBEN USAR ANTIBIÓTICOS?

a. No existe beneficio con el uso de c. Niños mayores de 6 años, por el riesgo


antibióticos. de complicación existente.

b. Lactantes y párvulos, por ser frecuente la d. Lactantes por el riesgo de neumonía


infección como desencadenante. química.

____ 20. ¿EN QUÉ CONSISTE EL TRATAMIENTO ETIOLÓGICO DEL ASMA BRONQUIAL?

a. Adopción de medidas encaminadas a c. Uso de glucocorticoides.


eliminar o reducir los alérgenos.

b. Uso de potentes antiinflamatorios. d. Adopción de Inmunoterapia.

____ 21. ¿CUÁLES SON LOS SIGNOS Y SÍNTOMAS DEL ASMA BRONQUIAL?

a. Síntomas nasales paroxísticos, hidrorrea, c. Adenopatías, fiebre, odinofagia,


estornudos, tos, crisis de disnea. mucosidad nasal.

b. Tos, fiebre, expectoración (tos d. Fiebre elevada, tos, conjuntivitis y rinitis.


productiva).

____ 22. ¿EN LA CRISIS ASMÁTICA EL PREDOMINIO NOCTURNO ESTÁ CONDICIONADO POR?

a. Una tos irritativa, continuada y poco c. Sensación de alivio u opresión en el


penosa sin la menor posibilidad de pecho, manifestada a menudo en forma
expectoración. de suspiro y voz alta.

b. La inhibición del reflejo tusígeno por el d. Respiración abdominal, dilatación de


sueño, retención de secreciones mucosas, ventanas nasales, roncus y los tirajes
disminución de la capacidad respiratoria supraesternal y subcostal (disnea).
y al acostarse, el niño se pone en
contacto con el alérgeno

5
Name: ________________________ ID: A

____ 23. ¿ENTRE UNA Y OTRA CRISIS DE ASMA BENIGNA, CUÁLES SON SÍNTOMAS QUE SE
PRESENTAN EN LA RINITIS ALÉRGICA?

a. Crisis de disnea o cefalea al practicar c. Prurito intenso que motiva que el niño se
deporte, sobre todo aquellos que le frote la nariz (“saludo alérgico”),
obligan a efectuar sprint o carreras estornudos en salvas o aislados, hidrorrea
cortas. u obstrucción nasal.

b. Rinitis a veces persiste, una vez d. Deformidades torácicas con lordosis y


aparecidas las crisis de asma. prominencia de la clavícula, con
acentuación del surco correspondiente a
la implantación del diafragma.

____ 24. ¿LOS RASGOS NERVIOSOS, NEUROVEGETATIVOS Y PSÍQUICOS PUEDEN ENCONTRARSE EN


LOS CASOS GRAVES, Y AFECTAN AL?

a. Lesiones cutáneas, en forma de habón y c. Alergia respiratoria, que se manifiesta


como lesión alérgica; el eccema del por náusea recurrente, casi siempre con
lactante que puede ser concomitante con acetonemia, dolor abdominal agudo.
la asma.

b. “Eccema-asma” suele indicar gran d. El crecimiento, en las formas graves hay


importancia de los factores endógenos), una detención importante, incluso un
frecuentes manifestaciones cardiológica retraso en el desarrollo puberal, mientras
y respiratoria. que en las formas habituales hay un
crecimiento a brotes.

____ 25. ¿LA CLASIFICACIÓN DE LA GRAVEDAD DEL ASMA SEGÚN LA VARIABILIDAD DE LA


APARICIÓN E INTENSIDAD DE LOS SÍNTOMAS ES?

a. Asma intermitente: síntomas menos de c. Asma moderada persistente: síntomas


dos veces por semana, exacerbaciones semanales, exacerbaciones que pueden
breves y síntomas nocturnos no más de afectar a la actividad y el sueño.
dos veces al mes. Síntomas nocturnos más de una vez al
mes.

b. Asma leve persistente: síntomas más de 1 d. Asma grave persistente: Síntomas


vez a la semana pero menos de una vez al semanales, exacerbaciones y Síntomas
día. Las exacerbaciones pueden afectar el diurnos frecuentes, limitación de las
sueño. actividades físicas.

____ 26. Entre los siguientes conceptos, señale el que define a una crisis asmática:

a. Un aumento de los síntomas asmáticos c. La inflamación crónica de las vías


rápidamente creciente respiratorias

b. Ataque agudo de asma con un grado d. Ninguna


de obstrucción bronquial severo

6
Name: ________________________ ID: A

____ 27. Escoja la mejor guía p0ara determinar la gravedad de la crisis asmática y la mejor respuesta la
tratamiento:
a. Aspecto físico del niño c. Antecedentes de ingreso hospitalario

b. Valoración del score clínico, peak d. Todas las anteriores


flow (PEF) y saturación de O2 (SO).

____ 28. Entre los siguientes ítems, constituye un factor de riesgo de crisis asmática grave:

a. Visitas a urgencias en los últimos seis c. Uso reciente de corticoides


meses.

b. mayor o igual a 1 hospitalizacion o d. Visitas hospitalarias dentro del último


mayor o igual a 1 visita en la sala de año
urgencias

____ 29. Entre los siguientes rangos de edades, la crisis asmática es la emergencia médica más común en:

a. niños de 1 a 4 años c. niños de 5 a 10 años

b. niños de 1 a 18 años d. Ninguna

____ 30. El paciente con crisis asmática debe ser manejado en un Servicio de Urgencias con atención
inmediata, sobre todo quienes tienen mayor riesgo de un evento catastrófico, entre los que se
encuentran:

a. Pacientes con historia de asma sin c. Antecedente de suspensión brusca


necesidad de ventilación mecánica. de glucocorticoides orales o mal
manejo de esteroides

b. Los que acudieron a urgencias para d. Ninguna


atención de un evento de asma en el
último mes

____ 31. Parte del día en que inician los síntomas:

a. Diurno c. Después de consumir algún alimento

b. Nocturno d. Solo en la tarde

____ 32. Características de la tos al inicio de una crisis asmática:

a. Tos muy seca, continuada y penosa c. Tos productiva

b. Alta posibilidad de expectoración. d. Tos que se presenta en intervalos

7
Name: ________________________ ID: A

____ 33. Signos que son característicos del periodo de estado:

a. Agitación y angustia c. Estertores musicales sibilantes


apreciados incluso sin fonendoscopio

b. Cambios de postura para buscar una d. Todas son correctas


actitud que facilite la respiración

____ 34. A la auscultación que encontramos:

a. Murmullo vesicular disminuido o c. Sibilancias en la espiración


abolido prolongada

b. Predominan los múltiples estertores d. Todas son correctas


secos y roncus

____ 35. Signos característicos de la fase de regresión:

a. Tos más blanda c. Desaparece la dificultad respiratoria

b. Esputo mucoso y pequeño, como d. Todas son correctas


“perlado”

____ 36. DENTRO DE LOS FACTORES DESENCADENANTES DE LAS CRISIS ASMATICA


ENCONTRAMOS LOS SIGUENTES:

a. Infecciones víricas c. Humo del tabaco

b. Alérgenos domésticos o laborales d. Todos los anteriores

____ 37. La hiperrespuesta bronquial dentro de la fisiopatología del asma se caracteriza por :

a. Broncoconstriccion c. Tos productiva

b. Edema d. Ninguna de las anteriores

____ 38. Las sibilancias dentro de las crisis asmáticas se presentan por:

a. Disminución del flujo inspiratorio c. Forzamiento del flujo de aire a través de


las vías respiratorias.

b. Disminución del flujo sanguíneo d. Todas son correctas


pulmonar

____ 39. Al examen físico de crisis asmatica que podemos encontrar:

a. Murmullo vesicular disminuido c. tos seca

b. espiración prolongada d. Todas son correctas

8
Name: ________________________ ID: A

____ 40. La confirmación del diagnóstico del asma se realiza con:

a. Tomografía multicorte c. Espirometria

b. Radiografía simple de torax d. Ninguna de las anteriores

____ 41. Cuál es el objetivos general del tratamiento en la crisis asmática:

a. Conseguir una broncodilatación y una c. Conseguir una adecuada técnica


oxigenación adecuada inhalatoria y control ambiental

b. Conseguir una broncoconstriccion y d. Ninguna


una oxigenación adecuada

____ 42. Cuales son los pilares fundamentales en el tratamiento de la crisis asmatica
a. Broncodilatadores y corticoides c. Oxígeno, broncodilatadores y corticoides
sistémicos sistémicos

b. Únicamente oxigeno d. Ninguna

____ 43. En las crisis moderada-graves cuales son las alteraciones que con mayor frecuencia cursa:

a. Hipoventilación alveolar c. En la perfusión

b. En la relación ventilación / perfusión e d. Ninguna


hipoventilación alveolar

____ 44. En que ocasiones es fundamental la administración de oxígeno:

a. En las crisis que cursen con SO < 93% c. En las crisis en donde no se puede
tras la administración de determinar la SO
broncodilatadores, o en aquellas
moderadas-severas si no es posible
determinar la SO

b. En todas las crisis asmáticas d. Ninguna

____ 45. Como se evaluaría la óptima administración de oxígeno

a. Mantener SO2 < 92% (PaO2 > 70) c. Mantener SO2 al 100% (PaO2 > 90)

b. Mantener SO2 > 92% (PaO2 > 70) d. Ninguna

____ 46. El grupo farmacològico de elecciòn en las crisis asmàticas es:


a. B-2 adrenérgicos de corta duración c. Anticolinérgicos

b. Glucocorticoides d. Ninguna

____ 47. Principal vía de administración de los B-2 adrenérgicos de corta duración
a. Vía oral c. Vía subcutánea y Vía intravenosa

b. Vía inhalatoria d. Vía sublingual

9
Name: ________________________ ID: A

____ 48. La técnicas de inhalación de B2 adrenérgicos de mayor elección aplicable en una urgencia pediátrica por
crisis asmática leve –moderada es:

a. Dispositivo de polvo seco c. Nebulizador

b. Inhalador presurizado con cámara d. Ninguna


espaciadora

____ 49. La vía de administración de mayor preferencia para los glucocorticoides sistémicos es:

a. Vía oral c. Vía intramuscular

b. Vía intravenosa d. Vìa sublingual

____ 50. Una de las limitaciones del nebulizador es:

a. imposibilidad de usar este método en c. flujo inspiratorio mínimo de 30 litros/


la crisis asmática grave min

b. lentitud d. Todas

____ 51. En cuanto a la disentería causada por la entamoeba histolitica indique la morfología según su ciclo de
vida.

a. Trofozoíto: es la forma activamente c. Metaquiste: tienen las mismas


móvil de la especie. características que los quistes, por
derivarse de estos durante el
proceso de desenquistamiento en la
luz del colon distal.

b. Quiste: forma infectante. Contiene de d. Trofozoíto: es la forma contaminante


2 a 6 núcleos, dependiendo de la y posee 2 núcleos.
madurez del quiste.

____ 52. En cuanto al tratamiento de la shigelosis, señale el enunciado INCORRECTO

a. La dosis de la Ciprofloxacina es de c. La dosis de la Ceftriaxona es de 50


15 mg/kg/dosis, dos veces al dia VO mg/kg/dia, 1 vez al dia IM

b. La dosis de la Azitromicina es de 10 d. La dosis del Metronidazol para


mg/kg/dia, 1 vez al dia VO neonatos es de es de 15
mg/kg/dosis, dos veces al dia VO y
es la primera elección para el
tratamiento

10
Name: ________________________ ID: A

____ 53. En cuanto al patogenia de Salmonelosis Mecanismo de invasión, señale el enunciado CORRECTO

a. Puede invadir varias líneas celulares y se c. En la pared bacteriana se encuentran


considera que puede estimular más de un unos complejos de proteína-lipoproteína,
camino de transducción de señales para denominados antígenos VW y F1, cuya
promover su entrada a las células del función es evitar la fagocitosis por parte
de los macrófagos y otras células
hospedero fagocíticas

b. No se desencadena su captación por d. Se adhiere a las células del intestino


las células M del colon grueso por medio de los plásmidos

____ 54. En cuanto a la disentería enteroinvasiva indique la secuencia de su patogenia.

a. Se multiplica, invade las células c. Invade las células epiteliales del


epiteliales del colon, se adhiere, colon, se multiplica, se adhiere,
destruye la membrana de las células, causa ulceración e inflamación
causa ulceración e inflamación

b. Invade las células epiteliales del d. Invade las células epiteliales del
colon, se multiplica, se adhiere, colon, se adhiere, destruye la
destruye la membrana de las células, membrana de las células, causa
causa ulceración e inflamación ulceración e inflamación

____ 55. En cuanto al trofozoito señale lo correcto


a. Se nutre por fagocitosis c. Reproducciòn binaria
b. Carece de citoesqueleto estructurado d. Todas

____ 56. En cuanto al quiste señale lo correcto


a. Es la forma infectante c. No tiene resistencia al jugo gastrico
b. Presenta pseudopodos d. bajamente infeccioso

____ 57. Señale lo incorrecto en cuanto a la fisiologìa y patogenia


a. Ingresa por el bolo alimentario c. Es inadherible
b. Fagocita eritrocitos d. Contine Adhesina y Cisteína

____ 58. Señale lo correcto en cuanto a la epidemiología


a. Se encuentra en areas de clima templado c. Transmisión fecal oral
b. mayor frecuencia en varones adultos d. Todas

____ 59. Encuanto a la transmisión señale larespuesta correcta


a. Transmisión fecal oral c. sus vectores son las moscas y cucarachas
b. A travez de alimentos contaminados d. Todas

____ 60. De acuerdo al ciclo de vida señale la incorrecta


a. Ingesta de quistes tetranucleados c. Unos colonizan y otros son arrastrados
por las heces
b. En el estomago se convierten en d. Ninguna de las anteriores
trofozoitos

____ 61. Según la patogenia de la Shigelosis señale lo incorrecto


a. Desencadena por las celulas I del colon c. Invaden los eritrocitos
b. son bacterias nomadas en el iicio d. Se liberan del fagosoma

11
Name: ________________________ ID: A

____ 62. Son manifestaciones clinicas de la shigelosis excepto


a. Fiebre alta c. Cefalea
b. Parestesias d. Diarrea acuosa

____ 63. En cuanto a la salmonelosis señale lo correcto


a. Posee flagelos perítricos c. Aerobios facultativos
b. Son basilos gram + d. desarrollan capsulas y esporas

____ 64. Para el tratamiento de la fiebre tifodea se ocupa excepto


a. Ciprofloxacina 15-20mg/kg/día c. Penicilina Benztinica 1200000UI/dia
b. Cefexima de 15-20mg/kg/dia d. Azitromicina 10mg/kg/dia

____ 65. Para el Dg de la Salmonelosis se realiza por:


a. Aglutinaciones febriles c. coprocultivo
b. hemocultivo d. Todas las anteriores

____ 66. Los sintomas de la salmonelosis son Excepto


a. Anorexia c. Dolor abdominal
b. Nausea d. Ninguna de las anteriores

____ 67. En cuanto las manifestaciones extraintestinales y complicaciones señalen lo correcto


a. Nutrición y metabolismo c. Síndrome hemolítico-urémico.
b. Sistema nervioso d. todas son correctas

____ 68. Las especies de shigella son señale lo correcto


a. Pestis, enterocolitica c. dysenteriae, flexneri, boydii, sonnei
b. kristensenii, pseudotuberculosis d. bercoveri, frederiksenii

____ 69. Agente etiologico de la yerseniosis en el hombre señale lo incorrecto


a. Pestis c. Pseudotuberculosis
b. Enterocolica d. Disenteriaie

____ 70. el periode de incubación de la yersiniosis es:


a. 8-10 c. 15-20
b. 3-7 d. 3-10

____ 71. Como se define a la enfermedad diarreica aguda de acuerdo a la Organización Mundial de Salud

a. Son la segunda mayor causa de muerte c. En la actualidad es probable que una


de niños menores de cinco años causa son las infecciones parasitaras

b. Ocasionan la muerte de 325 000 niños d. Se considera a una diarrea la deposición,


cada año tres o más veces al día (o con una
frecuencia mayor que la normal para la
persona) de heces sueltas o líquidas

12
Name: ________________________ ID: A

____ 72. Existe diferentes tipos clínicos de diarrea, en cuanto a la diarrea con sangre aguda es conocida como:

a. Diarrea por cólera c. Diarrea persistente

b. Diarrea disentérica d. Diarrea acuosa

____ 73. Cuales son las causas de la enfermedad diarreica aguda en niños

a. Infecciones c. De persona a persona por las manos

b. Adecuada alimentación d. Buen estado físico

____ 74. Que agente con mayor frecuencia produce la enfermedad diarreica aguda en niños

a. Trichomonas c. Rotavirus

b. Influenza d. Salmonella

____ 75. Mencione el número de casos encontrados de EDA en la provincia de Chimborazo en el año 2016 por
transmisión de agua y alimentos contaminados

a. 420.000 casos c. 450 casos

b. 48.542 casos d. 4.542 casos

____ 76. Los principales agentes infecciosos que producen EDA son:
a. Virus c. Parásitos

b. Bacterias d. Hongos

____ 77. El principal virus que produce EDA es:


a. Adenovirus c. Astrovirus

b. Rotavirus d. Rinovirus

13
Name: ________________________ ID: A

____ 78. El Mecanismo enterotòxico responsable de diarrea aguda consiste en:


a. Es el mecanismo fundamental del Vibrio c. Es el mecanismo fundamental de los
cholerae, E. Coli enterotoxígeno y parásitos, en el cual segregan
Citrobacter, en el cual segregan enterotoxinas que se adhieren a la
enterotoxinas que se adhieren a la superficie del enterocito e inhiben la
superficie del enterocito e inhiben la entrada de sodio y cloro y favorecen la
entrada de sodio y cloro y favorecen la salida de sodio y agua.
salida de sodio y agua.

b. Es el mecanismo fundamental del d. Es el mecanismo fundamental de los


Rotavirus y Adenovirus, en el cual hongos, en el cual segregan enterotoxinas
segregan enterotoxinas que se adhieren a que se adhieren a la superficie del
la superficie del enterocito e inhiben la enterocito e inhiben la entrada de sodio y
entrada de sodio y cloro y favorecen la cloro y favorecen la salida de sodio y
salida de sodio y agua. agua.

____ 79. El Mecanismo enteroinvasivo responsable de diarrea aguda consiste en:


a. Es el mecanismo clásico de bacterias c.
como Salmonella, Shigella y Yersinia,
estas producen destrucción del borde en Es el mecanismo fundamental del
cepillo de las células epiteliales del Rotavirus, produce destrucción del
intestino, para penetrar en el interior de borde en cepillo de las células epiteliales
las células, produciendo un desequilibrio del intestino, para penetrar en el interior
entre la secreción y la absorción de de las células, produciendo un
líquidos con una excesiva secreción de desequilibrio entre la secreción y la
agua y electrolitos. absorción de líquidos con una excesiva
secreción de agua y electrolitos.

b. Es el mecanismo fundamental de Giardia d. Es el mecanismo fundamental del


Lamblia, esta produce destrucción del Arvovirus, produce destrucción del
borde en cepillo de las células epiteliales borde en cepillo de las células epiteliales
del intestino, para penetrar en el interior del intestino, para penetrar en el interior
de las células, produciendo un de las células, produciendo un
desequilibrio entre la secreción y la desequilibrio entre la secreción y la
absorción de líquidos con una excesiva absorción de líquidos con una excesiva
secreción de agua y electrolitos. secreción de agua y electrolitos.

____ 80. La diarrea producida por el mecanismo enteroinvasivo se caracteriza por:


a. Las deposiciones son líquidas explosivas, c. Las deposiciones presentan leucocitos,
sin sangre. moco y podrán tener sangre si hay
afección de la submucosa.

b. Las deposiciones presentan moco y d. Las deposiciones presentan hematies,


sangre. moco y podrán tener sangre si hay
afección de la submucosa.

14
Name: ________________________ ID: A

____ 81. Los factores de riesgo de la enfermedad diarreica aguda se dividen en :

a. Ambientales y relacionadas a la c. Por etnia, edad, sexo.


malnutrición

b. Sociales y económicas d. Por toxicos

____ 82. Dentro de los factores ambientales señale cuál tiene un alto grado de riesgo de infección

a. Los cultivos de hortalizas por abuso en c. Los rayos ultravioletas


uso de pesticidas

b. El agua contaminada tanto por heces d. Los cultivos en invernaderos


humanas como por heces de animales
pueden conllevar a un alto grado de
amenaza

____ 83. Respecto a la lactancia maternal señale lo correcto

a. La lactancia materna hace que los c. La lactancia materna debe ser limitada
infantes sean más propensos a contraer
una infección diarreica aguda

b. En los alimentados a pecho, las d. La lactancia materna hace que los


infecciones entéricas son raras y cuando infantes contraigan inmonoglubulinas
ocurren, el cuadro se autolimita más que afectan al sistema inmune haciendolo
rápidamente mas debil .

____ 84. En cuanto el estado nutricional subraye lo correcto

a. El estado nutricional no influye en la c. En pacientes desnutridos la velocidad de


enfermedad diarreica aguda recuperación de la mucosa intestinal es
más lenta.

b. En la desnutrición la recuperación es más d. El estado nutricional no influye en la


rápida enfermedad diarreica aguda , ya que
pueden existir pacientes obesos que
presenten dicha etiologia

____ 85. Cuales son los signos de alarma de la enfermedad diarreica aguda

a. Deposiciones poco frecuentes y vómitos c. Aumento de apetito y en el consumo de


líquidos

b. Deposiciones muy frecuentes, vómito a d. acufenos, cefaleas, escotomas , dolor tipo


repetición, más sed de lo común, colico , parecias
evacuación con sangre, no comer o beber
normalmente

15
Name: ________________________ ID: A

____ 86. los exámenes complementarios como el examen en fresco de heces es útil para:

a. Descubrir sangre, leucocitos o parásitos, c. Investigar las características de las


presencia de grasa o restos alimentarios bacterias

b. Identificar serología de microorganismos d. Verificar la cantidad de heces del niño

____ 87. Para el diagnóstico en la anamnesis es importante revisar lo siguiente en la enfermedad actual:

a. Edad, forma de comienzo e ingesta de c. Diuresis, ingesta de infusiones o


medicamentos medicamentos, estado actual

b. Edad, forma de comienzo, tiempo de d. Exclusivamente las deposiciones


evolución, número de las deposiciones,
diuresis, ingesta de infusiones o
medicamentos, estado actual

____ 88. Para el diagnóstico en la anamnesis es importante revisar lo siguiente en los antecedentes personales del
niño/a

a. Años de lactancia, tolerancia alimentaria, c. Tiempo de lactancia, incorporación de


otros episodios de diarrea con o son semisólidos, tolerancia alimentaria,
parasitosis. historia inmunitaria, enfermedades
previas, otros episodios de diarrea con o
sin internación, parasitosis y progreso
pondoestatural

b. Enfermedades previas, incorporación de d. Exclusivo la lactancia materna


semisólidos, historia inmunitaria.

____ 89. las bases del diagnóstico son:

a. La producción continua de heces c. La causa más frecuente en lactantes


>100g/kg/día hace sospechar enfermedad sanos es el uso de antibióticos seguido de
orgánica diarrea crónica específica

b. Los mecanismos incluyen transporte d. El diagnóstico depende de una


anormal de agua y sal, pérdida de área de combinación de antecedentes,
superficie mucosa, disminución de la exploración física general y análisis de
motilidad intestinal, agentes osmóticos, heces seguidos de pruebas específicas
aumento de la permeabilidad intestinal y
estimulantes de la secreción

16
Name: ________________________ ID: A

____ 90. con que entidades se puede realizar un diagnóstico diferencial

a. Intolerancia alimentaria aguda, c. Utilización de medicamentos


enfermedad celíaca, infecciones
sistémicas, enfermedades de las vías
respiratorias altas, enfermedades
metabólicas, enfermedades quirúrgicas

b. Lactancia prolongada, dieta líquida. d. Atresias digestivas, malformaciones


respiratorias y cardiacas, alteraciones
renales y desnutriciones

____ 91. Niños que tienen deshidratación leve a moderada secundaria a gastroenteritis aguda el déficit estimado es
del 3- 8% y la reposición de los líquidos con SRO es de:

a. 20-30ml/kg. c. 30-80ml/kg.

b. 10-50ml/kg. d. 11-44ml/kg.

____ 92. En niños con deshidratación clínica, incluyendo deshidratación hipernatrémica, se recomienda:

a. Uso de soluciones de alta osmolaridad c. No dar SRO frecuentemente y en


(240-250 mOsm/l) para la rehidratación pequeñas cantidades
oral

b. Dar 50ml/kg para reponer el déficit de d. Administrar lactato ringer


líquidos durante 4h, así como los
líquidos de mantenimiento.

____ 93. En que situaciones está totalmente contraindicado la rehidratación oral:

a. Deshidratación leve c. Lucidez del estado de conciencia

b. Shock hipovolémico d. Perdidas fecales poco intensas

____ 94. En cuanto a la prevención de la enfermedad diarreica aguda se debe tomar en cuenta :

a. Higiene de los alimentos y ambiental c. Una hidratación superior a las demandas


necesarias

b. Profilaxis con antibióticos d. No dar charla de prevencion

____ 95. Se recomienda el tratamiento intravenoso en:

a. Pacientes que si toleran la vía oral c. Estado tóxico o sospecha de bacteriemia.

b. Pacientes con algún tipo de d. Neonatos y menores de tres meses con


inmunodeficiencia que cursan con fiebre.
diarrea y fiebre

17
Name: ________________________ ID: A

____ 96. Los Germenes implicados en la infeccion del Tractro urinario.

a. Proteus, Klebsiella c. Staphylococcus saprophyticus,


Enterococcus.

b. Enterobacter y Citrobacter d. Todas son correctas

____ 97. Cual es el Agente que causa ITU por diseminacion hematogena.

a. Salmonella, Mycobacterium, c. Proteus


Citomegalovirus

b. Escherichia coli d. Enterococcus

____ 98. El microorganismo implicado mas frecuente en la Infeccion del Tracto Urinario es:

a. Escherichia coli c. Staphylococcus saprophyticus

b. Staphylococcus aureus. d. Ninguna de las Anteriores

____ 99. Diferentes especies de Pseudomonas se aíslan con relativa frecuencia en niños con:

a. Malformaciones de las vías urinarias, c. Recibiendo antibióticos de amplio


espectro o a dosis elevadas

b. Portadores de sondas d. Todas son correctas

____ 100. Una de las complicaciones que se desencadena al presentar infecciones de ITU repetitivas es la
pielonefritis aguda entonces que la definimos como :

a. Compromiso bacteriano agudo del c. Compromiso bacteriano agudo de la


parénquima renal. uretra

b. Compromiso bacteriano agudo de la d. Compromiso bacteriano crònico de la


vejiga. uretra.

____ 101. Debido al Porcentaje de ITU la tasa de recurrencia neonatal se estima que es:

a. Un 50 % Y, En Edades Posteriores, c. Durante Los 2 Primeros Años de vida.


30% En Los Varones Y Un 40% En
Las Niñas.

b. 30% En Los Varones Y Un 40% En d. Un 20 % Y, En Edades Posteriores,


Las Niñas. 10% En Los Varones Y Un 40% En
Las Niña.

18
Name: ________________________ ID: A

____ 102. El riesgo de padecer una infección urinaria antes de la pubertad es de:

a. 3 a 5 % en las niñas y el 1 al 2% en c. Representa el 0,8% de todas las


los varones. infecciones, siendo más frecuente en
hombres que en mujeres.

b. Las mujeres padecen cuatro veces d. Los varones padecen cuatro veces
más ITU con una frecuencia entre los más ITU con una frecuencia entre los
3 y 4 años. 5 y 6 años.
____ 103. En la actualidad el análisis de orina permite conocer en un porcentaje de bacteriuria, siendo en las niñas lactantes del 2
al 4,5%.
a. Del 2 al 4,5% en niñas c. En edad escolar aumenta las cifras a
in 1,2 y 0,03 %

b. Del 0.8% en varones. d. Del 8 al 9,5% en niños.

____ 104. Las Formas Recurrentes Idiopaticas De ITU se presenta en:

a. De 4 a 5 años y persistiendo Durante c. Con una Frecuencia entre 2 y 10 ataques


pocos años por año.

b. Frecuentes en niñas, en especial entre d. De 2 a 6 años y persistiendo Durante


2 y 3 años de edad. pocos años.

____ 105. La Nefritis intersticial focal es producida por:

a. Klebsiella c. Staphylococcu

b. E.coli d. Streptococos

____ 106. La infección urinaria se produce como consecuencia de:

a. Interacción entre un microorganismo c. Los factores predisponentes, pero no


y su huésped de los mecanismos de invasión

b. El reconocimiento del agente d. Interacción entre huésped y medio


etiológico, pero no de las vías de ambiente
acceso al riñón y tracto urinario

____ 107. Cuál de los siguientes literales con más frecuencia aumenta el riesgo de que un niño tenga una
infección en el tracto urinario?

a. malformaciones congénitas c. vegijas inestables

b. La uretra se encuentra cerca del ano d. antecedentes herediatrios.

19
Name: ________________________ ID: A

____ 108. Porque el paso de gérmenes patógenos desde la uretra hacia la vejiga ocurre con suma facilidad en la
niña?

a. uretra es corta y ancha c. uretra es larga y flexuosa

b. uretra es larga y con esfínter más d. uretra es corta y flexuosa


amplio que en el varón

____ 109. Las bacterias pueden adherirse al urotelio y ascender hacia el riñón simpre y cuando poseen:

a. cantidad de antígeno K c. fimbrias

b. cantidad de antígeno K d. Salmonella

____ 110. Indique que vía de infección es la más frecuente para la llegad adel microorganismo al riñon

a. Hematogena c. linfatica

b. Ascendente d. Descendente

____ 111. En todo niño con sospecha clínica de ITU se debe obtener:

a. muestra de orina y urocultivo c. muestra de orina y uroanálisis

b. muestra de orina y urocultivo d. sedimento

____ 112. Cuál de estos tipos de muestra urinaria no se puede obtener en niños para el diagnóstico de UTI

a. Chorro intermedio c. Micción limpia

b. Bolsa recolectora d. Catéter uretral

____ 113. La presencia de cilindros leucocitarios es marcador de:

a. infeccion urinaria baja c. cistitis

b. infección urinaria alta d. pielonefritis

____ 114. La probabilidad de infeccion en que tipo de muestra se da en un 99%:

a. punción suprapúbica c. chorro medio

b. cateterismo vesical d. bolsa recolectora

20
Name: ________________________ ID: A

____ 115. Los objetivos del diagnóstico por imagen en niños con ITU son para:

a. Detectar lesión renal crónica c. Establecer el pronóstico a largo plazo


por el desarrollo de lesiones crónicas

b. Identificar las anomalías d. Identificar las anomalías Neurologicas


nefrourológicas

____ 116. Señale uno de los criterios clínicos que motivan ingreso hospitalario en las ITU

a. Edad mayor a 2 años c. Afectación del estado general, vomito


o deshidratación

b. ITU con foco evidente d. Ninguna de las anteriores

____ 117. Señale uno de los fármacos a utilizar en la ITU bajas

a. Salbutamol c. Fentanilo

b. Amoxicilina-Acido clavulanico d. Salmeterol

____ 118. Cuales son los pasos a seguir dentro del tratamiento higiénico físico en la ITU

a. Reposo, ingesta de liquidos, higiene c. No ingerir líquidos


de los genitales

b. Conductas vesicales retencionistas d. Hospitalizacion

____ 119. Las ITU frecuentes son producidas por

a. Intervención quirúrgica c. Anomalías congénitas

b. Vejiga inestable d. Mala higiene genital

21
Name: ________________________ ID: A

____ 120. Cual es el objetivo de la quimioprofilaxis en las ITU

a. Mantener al paciente libre de ITU con c. Utilizar una gran cantidad de


el riesgo de desarrollar daño renal antibiótico

b. Que el paciente no desarrolle ninguna d. Restringir antibioticoterapia


ITU

____ 121. Paciente masculino de 6 años de edad, acude por presentar fiebre, cefalea y odinofagia, se observa al
examen físico lengua roja, apariencia de fresa junto con exantema maculopapular en tronco y
extremidades
a. Sarampión c. Fiebre escarlatina
b. Varicela d. Rubeola

____ 122. Paciente masculino de 2 años de acude por presentar diarrea acuosa desde hace 5 días en varias ocasiones
que se acompaña de dolor abdominal y nauseas que llegan al vomito en 10 ocasiones, se realiza
coproparasitario seriado por 3 ocasiones identificando los quistes de Balantidium Coli. ¿Cuál sería el
tratamiento más recomendado para esta parasitosis?
a. Doxiciclina 30 mg/kg/día después de c. Amoxicilina más ácido clavulánico 40
cada comida VO por 7 días. mg/kg/día cada 8 horas VO por 10 días

b. Tetraciclina 6,25 a 12,5 mg/kg de peso d. Albendazol 400 mg dosis única VO


cada 6 horas VO por 7 días

____ 123. La paciente de 14 años de edad, fue atendida en el hospital demandado los días 30 de marzo y 9 de abril de
2011, por presentar crisis convulsivas y fiebre. El 14 de abril de 2011 regresó a Urgencias, estaba
inquieta, pálida, con mucosa oral seca, dolor abdominal franco en epigastrio, Mc. Burney y psoas
negativos. Se diagnosticó epilepsia y gastritis medicamentosa. En esta atención refirió coluria, por lo que
se solicitaron pruebas de funcionamiento hepático que reportaron: bilirrubina total, 8.019; bilirrubina
directa, 4.4; bilirrubina indirecta, 3.5; transaminasa glutámicooxalacética, 16,323; transaminasa
glutámicopirúvica, 8,330; fosfatasa alcalina, 303; deshidrogenasa láctica, 10,570; glucosa, 90; BUN, 20;
urea, 42; creatinina, 0.9. El 16 de abril de 2011, ingresó al Servicio de Pediatría y ese mismo día fue
egresada con diagnóstico de hepatitis probablemente A. Debido a su grave estado de salud, ingresó a otro
hospital donde se le diagnosticó hepatitis B.
a. La paciente no presenta síntomas de c. Estuvo mal guiado el diagnostico
hepatitis
b. La hepatitis es fulminante d. No es fulminante la hepatitis

____ 124. Madre acude a centro de salud refiriendo que su hijo de 2 semanas de edad presenta vómitos de contenido
gástrico de gran volumen alimentario, con aspecto de leche cortada y se da después de un tiempo variable
de alimentación, el niño presenta deshidratación y detención de su curva de crecimiento, al examen
físico se palpa masa en epigastrio.
a. Reflujo gastroesofágico c. Enfermedad del reflujo gastroesofágico
b. Hipertrófia de píloro d. Intolerancia-alergia a proteínas de
vacuno

22
Name: ________________________ ID: A

____ 125. El signo de Rovsing característico en la apendicitis aguda consiste en:


a. Dolor a la presión en el epigastrio al c. Sensibilidad de rebote pasajera en la
aplicar una presión firme y persistente pared abdominal
sobre el punto de McBurney
b. Dolor agudo que aparece al comprimir el d. Dolor en el punto de McBurney al
apéndice entre la pared abdominal y la comprimir el cuadrante inferior izquierdo
cresta iliaca del abdomen.

____ 126. Niño de 3 años masculino de raza mestiza, llega a servicio de emergencia por q refiere q aproximadamente
hace 3 semanas presenta fiebre, tos, brote fino en la piel de 15 días de evolución además de Dolor en el
costado o dolor en la espalda Micción dolorosa, Incremento en la frecuencia/urgencia urinaria, Necesidad
de orinar en la noche, Color de orina anormal o turbia en incluso ha presentado Sangre en la orina, olor
de orina fétido o y sin otros antecedentes patológicos de importancia, fuera de gripas ocasionales.
Ocasionalmente por 3 ocasiones, vómitos, náusea, al momento se encuentra en mismas condiciones
a. Sepsis neonatal tardía c. Infección de vías urinaria en niños

b. Sepsis neonatal temprana d. Síndrome de aspiración meconial

____ 127. Paciente que al examen físico presenta hipopigmentación (color de cabello y piel más claro q sus
progenitores), dermatitis y olor de su orina característico a moho. ¿A que enfermedad nos referimos?
a. Hipotiroidismo congénito c. Fenilcetonuria
b. Galactosemia d. Hiperplasia suprarrenal

____ 128. Paciente masculino, recién nacido a término adecuado para la edad gestacional, quien nace por parto
vaginal, con el antecedente de fiebre en la madre durante el periodo de postparto inmediato. Los padres
consultan a los 2 días de vida pues le notan alza térmica no cuantificada, dificultad respiratoria,
hipoactividad y rechazo a la leche materna, posteriormente presenta: fallo respiratorio que amerita
ventilación mecánica asistida por varios días.
a. Sepsis neonatal tardía c. Taquipnea transitoria del recién nacido
b. Sepsis neonatal temprana d. Síndrome de aspiración meconial

____ 129. Recién nacido de 24 horas presenta ictericia, tipo de sangre de la madre ORh (-), qué conducta tomaría.
a. Realizar Coombs directo más tipificación c. Realizar Coombs indirecto
sanguínea del Recién nacido

b. Realizar exámenes de bilirrubina y d. Ninguna


esperar que ceda el cuadro.

23
Name: ________________________ ID: A

____ 130. Recién nacido femenino nació deprimido Apgar por lo que necesitó reanimación cardiopulmonar. Se
hospitalizó y se inició manejo con líquidos endovenosos, gluconato de calcio y cuidados generales. A las
cuatro horas post natales presentó episodio de cianosis generalizada que mejoró con oxigenoterapia.
Posterior aparecieron chupeteo y convulsión multifocal, se manejó con fenobarbital (20mg/kg). Durante
su evolución clínica persisten las crisis convulsivas. El examen físico: peso 2100 g, talla: 48 cm, PC: 30,5
cm, PT: 29,5 cm, hipotonía generalizada, hiporreflexia, bajo efecto de sedación farmacológica. Glicemia:
18 mg/dl, tratada, con control de 64 mg/dl. Hb 10,4 g %, Hto 31,4%, leucogramo: 24.900 mm3.
Ionograma: Na+: 130 meq/lt, K+: 6,2 meq/lt. Rx de abdomen y líquido cefalorraquídeo normales.
a. Sufrimiento fetal c. Síndrome meconial

b. Asfixia neonatal d. Taquipnea transitoria del recién nacido

____ 131. Paciente de un año de edad que sufre caída de un metro de alto (resbalo de la cama) . el niño al momento
tuvo llanto inconsolable pero calmo con el consuelo de la madre. Con que escala se valora el estado de
conciencia.
a. Escala de Nih c. Índice de Brethel
b. Escala de Glasgow d. Escala de Rankin.

____ 132. Paciente masculino, recién nacido a término adecuado para la edad gestacional, quien nace por parto
vaginal. Madre del neonato regresa a los 15 días preocupada por que su hijo presenta una ulceración en la
región deltoidea del brazo izquierdo en donde le aplicaron una vacuna al momento de nacer. ¿Cuál es la
vacuna y las causas probables de dicha ulceración?
a. BCG-Mala técnica al aplicarla o exceso c. Pentavalente-Sistema inmunológico bajo
en la dosis administrada. del niño.
b. IPV-Mala refrigeración de la vacuna. d. BCG-Sistema inmunológico bajo del
niño.

____ 133. Paciente de 6 meses de edad acude a consulta tras presentar desde hace 4 horas náuseas que llegan al
vomito en tres ocasiones de contenido alimentario teniendo como causa aparente la ingesta de jugo
naranja hace 5 horas, a este cuadro se le acompaña irritabilidad y llanto fácil.
a. Infección por rotavirus c. Intolerancia alimenticia
b. Hipertrofia del píloro d. Síndrome diarreico agudo

____ 134. Niño con anorexia, fatigabilidad e irritable, adoptando una postura con las extremidades semiflexionadas y
abducidas, que llora al movilizarlo y tiene gingivitis hemorrágica e hiperqueratosis folicular, tendrá un
déficit de:
a. Vitamina A c. Vitamina K
b. Triptófano d. Vitamina C

24
Name: ________________________ ID: A

____ 135. Paciente masculino de 18 meses de edad llega a consulta el día 10 de enero del 2017. La madre referir
que presenta diarrea, vómito y fiebre, también menciona que su hijo duerme mucho. Es traído por la
madre quien e la consulta indica que son originarios de una comunidad rural ubicada en Chimborazo.
En la exploración física del paciente se muestra irritable, con pelo seco y quebradizo, aspecto seco de la
boca, presenta hepatomegalia y miembros inferiores con edema maleolar. Además de la diarrea y el
vómito que presenta desde hace 2 días estos posiblemente causados por una infección
a. Desnutrición infantil c. Parasitosis
b. Obesidad d. Amigdalitis

____ 136. Madre acude al subcentro de salud con la preocupación de que su hija de 1 año 7 meses de edad está muy
gordita y al control de peso y talla de la niña se obtiene un IMC de 21, como referencia tomamos la tabla
de IMC para la edad de Niñas y con ella de evidencia que una nia cuyp IMC para la edad está por encima
de 3 está obesa. ¿Cuáles serían las futuras complicaciones o patologías que la niña puede tener a futuro?
a. El síndrome metabólico c. La apnea obstructiva del sueño
b. Las complicaciones ortopédicas incluyen d. Todas
enfermedad de Blount

____ 137. Madre acude a centro de salud por que su hija de 4 año de edad hace 4 días presenta rinorrea, tos leve que
suena como a la de un perro, febrícula, no presenta babeo, mismo síntomas se empeoran por las noches.
a. Amigdalitis crónica. c. Epiglotitis.
b. Laringotraqueobronquitis. d. Traqueitis bacteriana.

____ 138. Un niño de 6 meses presenta tos persistente y fiebre. La exploración física y la radiografía de tórax sugieren
una neumonía. ¿Cuál de los siguientes microorganismos es menos probable que sea el agente causal de
esta infección?
a. Virus sincitial respiratorio c. Virus parainfluenza 1
b. Adenovirus d. Rotavirus

____ 139. Un niño de seis años fue mordido mientras jugaba con el perro de un vecino y presentó después celulitis y
supuración. El antibiótico de elección contra el microorganismo causante de la infección es:
a. Oxacilina c. Cloranfenicol
b. Tetraciclina d. Penicilina

____ 140. Paciente de un año de edad con 8kg presenta, fiebre de 38°C, diarrea, acompañado de vomito hace 3 días,
las deposiciones son liquidas amarillentas abundantes por 8 ocasiones al día. Al examen físico niño
letárgico (se mueve poco, ojos muy hundidos), mucosas muy secas sin saliva, respiración muy rápida.
a. EDA con deshidratación grave. c. EDA con deshidratación leve.
b. EDA con deshidratación moderada. d. Ninguna

25
Name: ________________________ ID: A

____ 141. Madre acude a centro de salud por que su hijo de 1 mes edad hace 4 días presenta vómitos y
regurgitaciones postprandiales el niño tiene peso y talla adecuados para la edad al examen físico no se
palpan masas en el abdomen ni otro signo o síntoma.
a. Reflujo gastroesofágico c. Estenosis hipertrófica de píloro

b. Enfermedad del reflujo gastroesofágico d. Intolerancia-alergia a proteínas de


vacuno

26
ID: A

COMP PEDIATRIA
Answer Section

MULTIPLE CHOICE

1. ANS: A
La OMS, menciona que el asma es una enfermedad crónica que se caracteriza por ataques recurrentes de
disnea y sibilancias, que varían en severidad y frecuencia de una persona a otra.

RESPUESTAS INCORRECTAS

B: Se define como una enfermedad inflamatoria crónica de las vías aéreas.

C. Los síntomas pueden sobrevenir varias veces al día o a la semana, y en algunas personas se agravan
durante la actividad física o por la noche.

D: En individuos susceptibles la inflamación produce episodios recurrentes de disnea, sibilancias,


opresión torácica y tos

PTS: 1 DIF: MEDIA REF: Cruz (2015), Nuevo tratado de pediatría, pág. 1432.
OBJ: ADQUIRIR CONOCIMIENTOS ACERCA DE ASMA BRONQUIAL
NAT: DR. ANGEL MAYACELA TOP: ASMA BRONQUIAL
KEY: DEFINICION NOT: DR. ANGEL MAYACELA
2. ANS: D
Más del 80% de las muertes por asma tienen lugar en países de ingresos bajos y medios-bajos.

RESPUESTAS INCORRECTAS

A: La OMS calcula que en la actualidad hay 235 millones de pacientes con asma.

B: El asma es la enfermedad crónica más frecuente en los niños.

C: El asma está presente en todos los países, independientemente de su grado de desarrollo

PTS: 1 DIF: MEDIA REF: Cruz (2015), Nuevo tratado de pediatría pág. 1432.
OBJ: ADQUIRIR CONOCIMIENTO DE LA EPIDEMIOLOGIA DEL ASMA BRONQUIAL
NAT: DR. ANGEL MAYACELA TOP: ASMA BRONQUIAL
KEY: EPIDEMIOLOGIA NOT: DR. ANGEL MAYACELA

1
ID: A

3. ANS: B
En los linfocitos T cooperadores (Th) se distingue la subclase Th1, productora de interferón
gamma (IFNã), que actúa frente a agentes infecciosos, y la Th2, que produce interleucinas 4 y 5
(IL-4) (IL-5). La IL-4 provoca la producción de IgE por los linfocitos B específicos, mientras que
la IL-5 se comporta como quimiotáctica para los eosinófilos

RESPUESTAS INCORRECTAS
A: En el 70% de los casos existen antecedentes de familiares afectos de diversas enfermedades alérgicas,
apoyando el papel de la herencia, no en cuanto a padecer asma, sino como predisposición alérgica.

C. En el asmático existe un predominio de la actividad colinérgica (hiperreactividad bronquial),


posiblemente por alteración de los receptores â2-adrenérgicos

D: En el asma el epitelio es más frágil por la disrupción de esas uniones intercelulares, con la pérdida
fácil de células columnares.

PTS: 1 DIF: MEDIA REF: Cruz (2015), Nuevo tratado de pediatría pág. 1433.
OBJ: ADQUIRIR CONOCIMIENTOS SOBRE FACTORES PREDISPONENTES DE LA ASMA
BRONQUIAL NAT: DR. ANGEL MAYACELA TOP: ASMA BRONQUIAL
KEY: FACTORES PREDISPONENTES NOT: DR. ANGEL MAYACELA
4. ANS: B
Además del frío, son factores favorecedores los cambios bruscos de temperatura, a través de
mecanismos inespecíficos, al motivar en el árbol respiratorio estados de edema bronquial o
reactivando procesos infecciosos de vías respiratorias altas.

RESPUESTAS INCORRECTAS
A: En el lactante la inmadurez inmunitaria favorece las infecciones, sobre todo por virus, y el déficit
selectivo de IgA, más común en los pacientes asmáticos

C. El asma infantil tiene más incidencia en las regiones de clima húmedo y, en iguales condiciones de
humedad, en las regiones más frías

D: Contaminantes ambientales tienen un papel evidente sobre todo en grandes poblaciones industriales,
pero también actúa en igual sentido la vivienda con polvo, mala ventilación, ciertas industrias o
profesiones caseras

PTS: 1 DIF: MEDIA REF: Cruz (2015), Nuevo tratado de pediatría pág. 1433.
OBJ: ADQUIRIR CONOCIMIENTOS SOBRE FACTORES INESPECIFICOS DEL ASMA
BRONQUIAL
NAT: DR. ANGEL MAYACELA TOP: ASMA BRONQUIAL
KEY: FACTORES INESPECIFICOS NOT: DR. ANGEL MAYACELA

2
ID: A

5. ANS: A
En los etolologicos los alergenos por inhalacion son sustsancias pulverulentas que estan en suspencion en
el aire y penetran en el arbol respiratorias con los movimientos de inspiración.

RESPUESTAS INCORRECTAS

B. En el polvo hay la presencia de enzimas procedentes de los ácaros, especialmente el


Dermatophagoides pteronyssinus, parásito habitual de las viviendas

C. Alergia a la leche se manifiesta también en el aparato respiratorio como “síndrome de Heiner” debido
a la sensibilización indirecta del árbol respiratorio.

D. El ácido acetilsalicílico, por un mecanismo diferente de estimulación del metabolismo de la


ciclooxigenasa, causa crisis de disnea, asociándose en los mismos pacientes rinitis/sinusitis y pólipo
nasal.

PTS: 1 DIF: MEDIA REF: Cruz (2015), Nuevo tratado de pediatría pág. 1434.
OBJ: ADQUIRIR CONOCIMIENTOS SOBRE FACTORES ETIOLOGICOS DEL ASMA
BRONQUIAL
NAT: DR. ANGEL MAYACELA TOP: ASMA BRONQUIAL
KEY: FACTORES ETIOLOGICOS NOT: DR. ANGEL MAYACELA
6. ANS: B
La literatura menciona que existen dos tipos de asma bronquial de acuerdo al factor
desencadenante de la misma, intrínseca y extrínseca.

RESPUESTAS INCORRECTAS
A: Hace referencia a tiempo de evolución y no a factor desencadenante.

C. Hace referencia a la severidad con la cual se presente dicha enfermedad y no a factor desencadenante.

D: Hace referencia a muchas otras patologías menos a asma bronquial

PTS: 1 DIF: MEDIA REF: Porth (2010), Manual de Fisiopatología.


OBJ: ADQUIRIR CONOCIMIENTOS SOBRE FACTORES DESENCADENANTES DE LA CRISIS
ASMATICA NAT: DR. ANGEL MAYACELA TOP: ASMA BRONQUIAL
KEY: FACTORES DESENCADENANTES NOT: DR. ANGEL MAYACELA

3
ID: A

7. ANS: A
El manual de fisiopatología de Porth menciona que el asma extrínseca se inicia típicamente por
una reacción de hipersensibilidad inducida por exposición a un antígeno extrínseco o a un
alérgeno.
RESPUESTAS INCORRECTAS

B: Podría ser el origen del asma de carácter intrínseco.

C. Podría ser el origen del asma de carácter intrínseco.

D: Podría ser el origen del asma de carácter intrínseco.

PTS: 1 DIF: MEDIA REF: Porth (2010), Manual de Fisiopatología.


OBJ: ADQUIRIR CONOCIMIENTOS SOBRE FACTORES DESENCADENANTES DEL ASMA
BRONQUIAL NAT: DR. ANGEL MAYACELA TOP: ASMA BRONQUIAL
KEY: FACTORES DESENCADENANTES NOT: DR. ANGEL MAYACELA
8. ANS: C
El manual de fisiopatología de Porth menciona que el asma intrínseca se inicia por ejercicio,
hiperventilación, inhalación de sustancias químicas, entre otros factores.

RESPUESTAS INCORRECTAS

A: hace referencia a factores desencadenantes de asma extrínseca.

B: hace referencia a factores desencadenantes de asma extrínseca.

D: hace referencia a factores desencadenantes de otro tipo de patologías.

PTS: 1 DIF: MEDIA REF: Porth (2010), Manual de Fisiopatología.


OBJ: ADQUIRIR CONOCIMIENTOS SOBRE FACTORES DESENCADENANTES DEL ASMA
INTRINSECA NAT: DR. ANGEL MAYACELA TOP: ASMA BRONQUIAL
KEY: FACTORES DESENCADENANTES NOT: DR. ANGEL MAYACELA
9. ANS: D
El manual de Fisiopatología de Porth menciona que el asma extrínseca en fase temprana se caracteriza
por presentarse a los 10-20 minutos y se debe a la liberación de mediadores químicos provenientes de
células cebadas pre sensibilizadas recubiertas con IgE.

RESPUESTAS INCORRECTAS

A: Para que la remisión de la crisis asmática sea completa es necesario el soporte farmacológico.

B: las células cebadas presensibilizadas están recubiertas por IgE.

C: Se presenta entre 10 a 20 minutos.

PTS: 1 DIF: MEDIA REF: Porth (2010), Manual de Fisiopatología.


OBJ: ADQUIRIR CONOCIMIENTOS CARACTERISTICAS DE LA FASE TEMPRANA DEL ASMA
EXTRINSECA NAT: DR. ANGEL MAYACELA TOP: ASMA BRONQUIAL
KEY: CARACTERISTICAS DE LA FASE TEMPRANA NOT: DR. ANGEL MAYACELA

4
ID: A

10. ANS: A
El manual de fisiopatología de Porth menciona que los irritantes inhalados como el humo del tabaco u
olores fuertes inducen broncoespasmo al estimular los receptores y producir un reflejo vagal.

RESPUESTAS INCORRECTAS

B: Hace referencia a la fisiopatología del asma intrínseca desatada por ejercicio y actividad física
excesiva.

C. Hace referencia a la fisiopatología del asma intrínseca desatada por ejercicio y actividad física
excesiva.

D: Hace referencia a la fisiopatología de otras alteraciones producidas en fumadores crónicos

PTS: 1 DIF: MEDIA REF: Porth (2010), Manual de Fisiopatología.


OBJ: ADQUIRIR CONOCIMIENTOS SOBRE LOS MECANISMOS DEL ASMA BRONQUIAL
NAT: DR. ANGEL MAYACELA TOP: ASMA BRONQUIAL
KEY: MECANISMOS DEL ASMA BRONQUIAL NOT: DR. ANGEL MAYACELA
11. ANS: A
Cuando el niño esta sintomático, la exploración clínica es fundamental y, en los intervalos, la función
respiratoria puede ser clave para el diagnóstico

RESPUESTAS INCORRECTAS

B: La tos, como equivalente asmático, puede deberse a multitud de otros procesos, por lo que no siempre
es un equivalente asmático
C: La existencia de episodios de disnea son esporádicamente por lo que no confirma un diagnóstico de
asma
D: La sibilancias en los intervalos parecen o se agravan en determinados ambientes pero no determina el
diagnostico

PTS: 1 DIF: MEDIA REF: Cruz (2014), Nuevo tratado de pediatría, pg 1437.
OBJ: ADQUIRIR CONOCIMIENTOS ACERCA DEL INTERVALO CLAVE PARA REALIZAR UN
DIAGNOSYICO CLINICO DEL ASMA BRONQUIAL NAT: DR. ANGEL MAYACELA
TOP: ASMA BRONQUIAL KEY: INTERVALO DE DIAGNOSTICO
NOT: DR. ANGEL MAYACELA
12. ANS: B
La investigación del alergeno se inicia con las pruebas cutaneas, como la puntura (prick-test) o la
inyeccion (intradermorreaccion) de extractos de alergenos. Estas pruebas orientan sobre la sensibilizacion
a determinados
alergenos.

RESPUESTAS INCORRECTAS

A: La elevacion de la IgE total serica, no siempre ocurre


C: El examen clínico puede aportar datos pero no identifica el alérgeno causal
D: Este puede estar relacionado con otros precesos infecciosos.

PTS: 1 DIF: MEDIA REF: Cruz (2014), Nuevo tratado de pediatría, pg 1438.
OBJ: ADQUIRIR CONOCIMIENTOS ACERCA DEL ETIOPATOGENIA ALERGENA DEL ASMA
BRONQUIAL NAT: DR. ANGEL MAYACELA TOP: DEL ASMA BRONQUIAL
KEY: ETIOPATOGENIA ALERGENA NOT: DR. ANGEL MAYACELA

5
ID: A

13. ANS: C
La provocación bronquial se lleva a cabo haciendo inhalar un extracto del alergeno en estudio, valorando
las variaciones de los flujos espiratorios mediante espirometria.

RESPUESTAS INCORRECTAS

A: Estas pruebas orientan sobre la sensibilizacion a determinados alérgenos en la piel y no por flujo
respiratorio.

B: Se utiliza para valorar rinitis alergica

D: Es poco usual y se trata de la degranulación de basofilos.

PTS: 1 DIF: MEDIA REF: Cruz (2014), Nuevo tratado de pediatría, pg 1438.
OBJ: ADQUIRIR CONOCIMIENTOS ACERCA DE LAS PRUEBAS DE VALORACION
BRONQUIAL DEL ASMA NAT: DR. ANGEL MAYACELA TOP: ASMA BRONQUIAL
KEY: PRUEBAS DE VALORACION BRONQUIAL NOT: DR. ANGEL MAYACELA
14. ANS: D
Diagnostico funcional la exploracion funcional espiratoria por espirometria, factible a partir de los 6-7
años de edad, permite estudiar la capacidad vital y, lo que tiene mas interes, el volumen espiratorio
máximo en el primer segundo (VEMS o FEV1) que, relacionado con la anterior, consigue obtener el
llamado indice de Tiffeneau (FEV1%).

RESPUESTAS INCORRECTAS

A: No tiene la suficiente capacidad expiratoria

B: No cumple con los valores estimados.

C: No puede realizar la prueba

PTS: 1 DIF: MEDIA REF: Cruz (2014), Nuevo tratado de pediatría, pg 1438.
OBJ: ADQUIRIR CONOCIMIENTOS SOBRE LA EDA FACTIBLE PARA REALIZAR LA
ESPIROMETRIA DEL ASMA BRONQUIAL NAT: DR. ANGEL MAYACELA
TOP: ASMA BRONQUIAL KEY: DIAGNOSTICO
NOT: DR. ANGEL MAYACELA

6
ID: A

15. ANS: A
La exploracion funcional respiratoria por espirometria, factible a partir de los 6-7 anos de edad, permite
estudiar la capacidad vital y, lo que tiene mas interes, el volumen espiratorio máximo en el primer
segundo (VEMS o FEV1) que, relacionado con la anterior, consigue obtener el llamado indice de
Tiffeneau (FEV1%.

RESPUESTAS INCORRECTAS

B: valora obstruccion de las pequenas vias aereas.

C: Es por inhalación de histamina.

D: Se realiza con la prueba de ergonometria.

PTS: 1 DIF: MEDIA REF: Cruz (2014), Nuevo tratado de pediatría, pg 1438.
OBJ: ADQUIRIR CONOCIMIENTOS SOBRE LA VALORACION DE LA ESPIROMETRIA DEL
ASMA BRONQUIAL NAT: DR. ANGEL MAYACELA TOP: ASMA BRONQUIAL
KEY: DIAGNOSTICO NOT: DR. ANGEL MAYACELA
16. ANS: A
Cuando las crisis asmáticas son leves, puede administrar un β2-mimético por vía inhalatoria (u oral, en
los casos muy leves), con preferencia salbutamol (albuterol) (200 μg) o terbutalina (500 μg), que puede
repetirse cada 30 minutos, hasta tres veces, aunque suele ser suficiente cada 6-8 horas, sin pasar de las
24-36 horas

RESPUESTA INCORRECTAS

B: Es la conducta que se debe tomar en la crisis asmática moderada.


C: Cuando las crisis asmáticas son leves, puede administrar un β2-mimético por vía inhalatoria
D: Es la conducta que se debe tomar en la crisis asmática grave.

PTS: 1 DIF: MEDIA REF: Cruz (2015), Nuevo tratado de pediatría, pg 1439.
OBJ: ADQUIRIR CONOCIMIENTOS TERAPIA FARMACOLOGICA EN LAS CRISIS ASMATICA
LEVE NAT: DR. ANGEL MAYACELA TOP: ASMA BRONQUIAL
KEY: TRATAMIENTO NOT: DR. ANGEL MAYACELA
17. ANS: C
La dosis indicada de salbutamol (albuterol), es de (200 μg) o que puede repetirse cada 30 minutos, hasta
tres veces, aunque suele ser suficiente cada 6-8 horas, sin pasar de las 24-36 horas.

RESPUESTAS INCORRECTAS

A: Es solo la mitad de la dosis indicada


B: Es solo la mitad de la dosis indicada
D: Es el doble de la dosis indicada.

PTS: 1 DIF: MEDIA REF: Cruz (2015), Nuevo tratado de pediatría, pg 1439.
OBJ: ADQUIRIR CONOCIMIENTOS TRATAMIENTO DEL ASMA BRONQUIAL LEVE
NAT: DR. ANGEL MAYACELA TOP: ASMA BRONQUIAL
KEY: TRATAMIENTO NOT: DR. ANGEL MAYACELA

7
ID: A

18. ANS: C
En el tratamiento de la crisis asmática, es fundamental mantener una postura adecuada que relaje la
musculatura abdominal, fije la cintura escapular y favorezca la respiración diafragmática. Se consigue en
decúbito lateral con los brazos cruzados y piernas flexionadas, o sentado con el tronco inclinado hacia
adelante.

RESPUESTAS INCORRECTAS

A: Es fundamental mantener una postura adecuada que relaje la musculatura abdominal, fije la cintura
escapular y favorezca la respiración diafragmática. Se consigue en decúbito lateral con los brazos
cruzados y piernas flexionadas, o sentado con el tronco inclinado hacia adelante.

B: También se recomienda que el paciente esté sentado con el tronco inclinado hacia adelante.

D: No es una posición adecuada para relajar la musculatura abdominal, fijar la cintura escapular y
favorecer la respiración diafragmática.

PTS: 1 DIF: MEDIA REF: Cruz (2015), Nuevo tratado de pediatría, pg 1440.
OBJ: ADQUIRIR CONOCIMIENTOS SOBRE TRATAMIENTO NO FARMACOLÓGICO DE LAS
CRISIS ASMÁSTICAS NAT: DR. ANGEL MAYACELA
TOP: ASMA BRONQUIAL KEY: TRATAMIENTO
NOT: DR. ANGEL MAYACELA
19. ANS: B
Los antibióticos sólo están indicados en los lactantes y párvulos, por ser frecuente la infección como
desencadenante, siempre que se sospeche o se demuestre la infección, eligiendo uno hipoalergénico
(macrólido).

RESPUESTAS INCORRECTAS

A: El uso de antibiótico si está recomendado en casos especiales.

C: El uso de antibióticos sólo está indicados en los lactantes y párvulos, por ser frecuente la infección
como desencadenante.

D: El uso de antibióticos sólo están indicados en los lactantes y párvulos, por ser frecuente la infección
como desencadenante.

PTS: 1 DIF: MEDIA REF: Cruz (2015), Nuevo tratado de pediatría, pg 1440.
OBJ: ADQUIRIR CONOCIMIENTOS SOBRE TRATAMIENTO NO FARMACOLÓGICO DE LAS
CRISIS ASMÁSTICAS NAT: DR. ANGEL MAYACELA
TOP: ASMA BRONQUIAL KEY: TRATAMIENTO
NOT: DR. ANGEL MAYACELA

8
ID: A

20. ANS: A
El tratamiento etiológico del asma bronquial incluye, la adopción de medidas encaminadas a eliminar o
reducir los alérgenos.

RESPUESTAS INCORRECTAS

B: Los antiinflamatorios son usados, como parte esencial del tratamiento farmacológico.

C: Los glucocorticoides son usados, como parte esencial del tratamiento farmacológico.

D: La inmunoterapia no forma parte del tratamiento etiológico.

PTS: 1 DIF: MEDIA REF: Cruz (2015), Nuevo tratado de pediatría, pg 1440.
OBJ: ADQUIRIR CONOCIMIENTOS SOBRE TRATAMIENTO ETIOLÓGICO DEL ASMA
BRONQUIAL NAT: DR. ANGEL MAYACELA TOP: ASMA BRONQUIAL
KEY: TRATAMIENTO NOT: DR. ANGEL MAYACELA
21. ANS: A
Cuadro clínico se establece de manera progresiva, de forma que hay síntomas nasales paroxísticos
(hidrorrea, estornudos), se acompañan de tos y ruidos respiratorios, auscultándose roncus y algunas
sibilancias.

RESPUESTAS INCORRECTAS

B: Son síntomas de neumonía estos son fundamentalmente respiratorios y de afectación de las vías aéreas
bajas.
C: En las amigdalitis víricas son más frecuentes la odinofagia, fiebre y se acompaña de síntomas
catarrales con tos escasa, conjuntivitis.
D: Es el cuadro clínico que se presenta en enfermedad infecciosa vírica aguda (sarampión)

PTS: 1 DIF: MEDIA REF: Cruz (2015), Nuevo tratado de pediatría, pág. 1435
OBJ: ADQUIRIR CONOCIMIENTOS SOBRE LOS SIGNOS Y SÍNTOMAS DEL ASMA
BRONQUIAL
NAT: DR. ANGEL MAYACELA TOP: ASMA BRONQUIAL
KEY: MANIFESTACIONES CLINICAS NOT: DR. ANGEL MAYACELA
22. ANS: B
El predominio nocturno está condicionado por la inhibición del reflejo tusígeno por el sueño, retención
de secreciones mucosas, disminución de la capacidad respiratoria con la posición horizontal.

RESPUESTAS INCORRECTAS

A: El inicio de la crisis asmática se caracteriza por una tos muy seca, continuada y penosa

C: La sensación de peso u opresión en el pecho, manifestada a menudo en forma de llanto y voz corta.
D: Respiración abdominal, dilatación de ventanas nasales, sibilancias y los tirajes supraesternal y
subcostal (disnea).

PTS: 1 DIF: MEDIA REF: Cruz (2015), Nuevo tratado de pediatría, pág. 1435.
OBJ: ADQUIRIR CONOCIMIENTOS SOBRE CRISIS ASMÁTICA EL PREDOMINIO NOCTURNO
NAT: DR. ANGEL MAYACELA TOP: ASMA BRONQUIAL
KEY: MANIFESTACIONES CLINICAS NOT: DR. ANGEL MAYACELA

9
ID: A

23. ANS: C
El predominio nocturno está condicionado por la inhibición del reflejo tusígeno por el sueño, retención
de secreciones mucosas, disminución de la capacidad respiratoria con la posición horizontal.

RESPUESTAS INCORRECTAS

A: Crisis de disnea o dolor torácico al practicar deporte, sobre todo aquellos que le obligan a efectuar
sprint o largas carreras.

B: La rinitis a veces persiste, una vez desaparecidas las crisis de asma.

C: Síntomas de rinitis alérgica, que se manifiesta por un prurito intenso que motiva que el niño se frote la
nariz (“saludo alérgico”) lo que origina un pliegue transversal de la punta de la misma, hay estornudos en
salvas o aislados, hidrorrea u obstrucción nasal.

D: deformidades torácicas con cifosis y prominencia del esternón, con acentuación del surco
correspondiente a la implantación del diafragma

PTS: 1 DIF: MEDIA REF: Cruz (2015), Nuevo tratado de pediatría, pág. 1435.
OBJ: ADQUIRIR CONOCIMIENTOS SOBRE CRISIS DE ASMA BENIGNA
NAT: DR. ANGEL MAYACELA TOP: ASMA BRONQUIAL
KEY: MANIFESTACIONES CLINICAS NOT: DR. ANGEL MAYACELA
24. ANS: D
A veces afecta el crecimiento, de modo que en las formas graves hay una detención importante, incluso
un retraso en el desarrollo puberal, mientras que en las formas habituales hay un crecimiento a brotes.

RESPUESTAS INCORRECTAS

A: Son lesiones cutáneas, en forma de urticaria y como prúrigo-estrófulo; el eccema del lactante que
puede ser concomitante con la asma.
B: “Eccema-asma” suele indicar gran importancia de los factores endógenos), frecuentes manifestaciones
digestivas y abdominales.
C: Alergia digestiva, que se manifiesta por vómitos recurrentes, casi siempre con acetonemia, dolor
abdominal agudo.

PTS: 1 DIF: MEDIA REF: Cruz (2015), Nuevo tratado de pediatría, pág. 1436.
OBJ: ADQUIRIR CONOCIMIENTOS SOBRE RASGOS NERVIOSOS, NEUROVEGETATIVOS Y
PSÍQUICOS PUEDEN ENCONTRARSE EN LOS CASOS GRAVES DEL ASMA BRONQUIAL
NAT: DR. ANGEL MAYACELA TOP: ASMA BRONQUIAL
KEY: MANIFESTACIONES CLINICAS NOT: DR. ANGEL MAYACELA

10
ID: A

25. ANS: B
El asma leve persistente: síntomas más de una vez a la semana pero menos de una vez al día. Las
exacerbaciones pueden afectar el sueño.

RESPUESTAS INCORRECTAS

A: El asma intermitente: síntomas menos de una vez por semana, exacerbaciones breves y síntomas
nocturnos no más de dos veces al mes.

C: El asma moderada persistente: síntomas diarios, exacerbaciones que pueden afectar a la actividad y el
sueño. Síntomas nocturnos más de una vez a la semana.

D: El asma grave persistente: síntomas diarios, exacerbaciones y Síntomas nocturnos frecuentes,


limitación de las actividades físicas.

PTS: 1 DIF: MEDIA REF: Cruz (2015), Nuevo tratado de pediatría, pág. 1437.
OBJ: ADQUIRIR CONOCIMIENTOS SOBRE LA CLASIFICACIÓN DE LA GRAVEDAD DEL
ASMA SEGÚN LA VARIABILIDAD DE LA APARICIÓN E INTENSIDAD DE LOS SÍNTOMAS
NAT: DR. ANGEL MAYACELA TOP: ASMA BRONQUIAL
KEY: MANIFESTACIONES CLINICAS NOT: DR. ANGEL MAYACELA
26. ANS: A
La respuesta correcta es la A porque se llama crisis de asma a un aumento de los síntomas rápidamente
creciente (minutos, horas o días), con una disminución importante del flujo aéreo.

Justificación de cada opción incorrecta:


B: Es definido como Estatus asmático.
C: Define el concepto de asma.

D: No corresponde

PTS: 1 DIF: Media


REF: Artemio Bermeo Limón, Virginia Velasco Díaz, Archivos de Medicina de Urgencia de México,
Vol. 5, Núm. 2 - Mayo-Agosto 2013. Pag 60 OBJ: Evaluar el nivel de conocimientos
TOP: Crisis Asmatica KEY: Pediatria NOT: Dr. Angel Mayacela
27. ANS: B
La respuesta correcta es la A porque los niños que prefieren estar sentados, o se encuentran agitados,
confusos, ansiosos, sudorosos, incapaces de decir una frase, o presentan llanto agudo o quejido, tienen
una crisis asmática grave y pueden estar en insuficiencia respiratoria

Justificación de cada opción incorrecta:


B: es especialmente difícil encontrar un score idóneo para el niño menor de 5 años

C: Constituye un factor de riesgo de crisis asmática grave

D: No corresponde

PTS: 1 DIF: Media REF: Jesús Sánchez Etxaniz, Urgencias de Pediatría. Pág.
52
OBJ: Evalur el nivel de conocimientos TOP: Crisis Asmàtica
KEY: Pediatria NOT: Dr. Angel Mayacela

11
ID: A

28. ANS: C
La respuesta correcta es la C porque son medicamentos más efectivos para cumplir con los objetivos del
tratamiento en el asma bronquial, y si no responde a este tratamiento se le considera de riesgo vital.

Justificación de cada opción incorrecta:

A: porque un factor de riesgo son visitas a urgencias en el mes previo

B: porque se considera factor de riesgo a mayor o igual a 2 hospitalizaciones o mayor o igual a 3 visitas a
urgencias en el año previo

D: no corresponde

PTS: 1 DIF: Media REF: Jesús Sánchez Etxaniz, Urgencias de Pediatría. Pág.
52
OBJ: Evalur el nivel de conocimientos TOP: Crisis Asmatica
KEY: Pediatria NOT: Dr. Angel Mayacela
29. ANS: A
La respuesta correcta es la A por en niños tiene una prevalencia del prevalencia sube al 29%

Justificación de cada opción incorrecta:


B: La prevalencia es del18,8% en promedio en niños de 1-18 años.

C: No se ha establecido como un rango específico.

D: No corresponde

PTS: 1 DIF: Media


REF: Juan Pablo Ruiz. Asma infantil. CCAP. Volumen 10 Número 2. Pág. 36
OBJ: Evaluar el nivel de conocimientos TOP: Crisis asmatica
KEY: Pediatria NOT: Dr. Angel Mayacela
30. ANS: B
La respuesta correcta es la B porque en estos casos se requieren dosis más altas elevando
considerablemente la tasa de efectos adversos.

Justificación de cada opción incorrecta:


A: Pacientes con historia de asma que necesitaron intubación y asistencia de ventilación mecánica

B: Los que acudieron a urgencias para atención de un evento de asma en el último año

D: no corresponde

PTS: 1 DIF: Media


REF: Ruth Saraí Aldana Vergara. Diagnóstico y tratamiento de crisis asmática infantil en la Sala de
Urgencias. Rev. De Neumología y Cirugía de Tórax. Vol. 68. Pág. S2-35
OBJ: Evaluar nivel de conocimientos TOP: Crisis Asmatica
KEY: Pediatria NOT: Dr. Angel Mayacela

12
ID: A

31. ANS: B
El predominio nocturno esta condicionado por la inhibicion del reflejo tusigeno por el sueño
RESPUESTAS INCORRECTAS
A: En el día el niño por lo general acude a la escuela y se encuentra en actividad por lo que no se
produce la inhibición del reflejo tusígeno dado por el sueño
C: Después de consumir cualquier alimento no tiene relación con el inicio de los síntomas de la crisis
asmática
D: El niño se encuentra en actividad por lo que es casi imposible que se encuentre en reposo y asi iniciar
los síntomas.

PTS: 1 DIF: Media


REF: Manuel C. Nuevo Tratado de Pediatría. 10 Edición Capitulo 16.13 pág. 1435
OBJ: Evaluar el nivel de conocimiento TOP: Crisis Asmatica
KEY: Pediatria NOT: Dr. Angel Mayacela
32. ANS: A
El inicio de la crisis asmatica se caracteriza por una tos muy seca, continuada y penosa, sin la menor
posibilidad de expectoracion, aunque se trate de un niño mayor
RESPUESTAS INCORRETAS
B: La tos característica de la crisis asmática tiene la menor posibilidad de expectoración.
C: La tos se caracteriza por ser seca sin nada de esputo
D: La tos es continua

PTS: 1 DIF: Media


REF: Manuel C. Nuevo Tratado de Pediatría. 10 Edición Capitulo 16.13 pág. 1435
OBJ: Evaluar el nivel de conocimientos TOP: Crisis Asmatica
KEY: Pediatria NOT: Dr. Angel Mayacela
33. ANS: D
En el periodo de estado existe la agitacion y angustia, los cambios de postura para buscar una actitud que
facilite a respiracion. Se comprueba la insuflacion del torax, el resalte de la musculatura auxiliar y
estertores musicales sibilantes.
RESPUESTAS INCORRECTAS
Todas son correctas

PTS: 1 DIF: Media


REF: Manuel C. Nuevo Tratado de Pediatría. 10 Edición Capitulo 16.13 pág. 1435
OBJ: Evaluar el nivel de conocimientos TOP: Crisis Asmatica
KEY: Pediatria NOT: Dr. Angel Mayacela
34. ANS: D
En la auscultacion, el murmullo vesicular propiamente dicho esta disminuido o abolido y predominan los
multiples estertores secos, roncus y sobre todo, sibilancias en la espiracion, siempre prolongada
RESPUESTAS INCORRECTAS
Todas son correctas

PTS: 1 DIF: Media


REF: Manuel C. Nuevo Tratado de Pediatría. 10 Edición Capitulo 16.13 pág. 1435
OBJ: Evaluar el nivel de conocimientos TOP: Crisis asmatica
KEY: Pediatria NOT: Dr. Angel Mayacela

13
ID: A

35. ANS: D
En la fase de regresion la tos comienza a ser mas blanda. la expectoracion pocas veces visibles en el niño,
esta caracterizada por un esputo mucoso y pequeño como perlado.
RESPUESTAS INCORRECTAS
Todas son correctas

PTS: 1 DIF: Media


REF: Manuel C. Nuevo Tratado de Pediatría. 10 Edición Capitulo 16.13 pág. 1435
OBJ: Evaluar el nivel de conocimientos TOP: Crisis asmatica
KEY: Pediatria NOT: Dr. Angel Mayacela
36. ANS: D
Los factores que pueden desencadenar estos sintomas o agravarlos son por infecciones virales, alergenos
domesticos o laborales, humo de tacabo, ejercicio y estres.
RESPUESTAS INCORRECTAS:
Todas las opciones son correctas por ser factores según ultima evidencia de desencadenantes de crisis
asmáticas.

PTS: 1 DIF: Media


REF: Sitio web de la GINA, www.ginasthma.org © 2017 Global Initiative for Asthma
OBJ: Evaluar los conociemientos de crisis asmatica TOP: CRISIS ASMATICA
KEY: Pediatria NOT: Dr. Angel Mayacela
37. ANS: A
La broncoconstriccion de la musculatura lisa bronquial, que sucede frente a multiples mediadores y
neurotransmiores, es reversible frente a farmacos broncodilatadores.
RESPUESTAS INCORRECTAS:
B: El edema en la hiperrespuesta bronquial se presenta en los cuadros de exacerbación del asma,
pero no es la característica principal.
C: La tos como respuesta a la falta de flujo espiratorio se mantiene de forma constante.
D: Niguna de las anteriores.

PTS: 1 DIF: Media


REF: Global initiative for asthma. Global strategy for asthma management and prevention. Updated
2017.
OBJ: Evaluar el nivel de conocimientos TOP: CRISIS ASMATICA
KEY: Pediatria NOT: Dr. Angel Mayacela
38. ANS: C
Las sibilancias es un sonido musical agudo producido cuando el aire es forzado a traves de las vias
respiratorias.
RESPUESTAS INCORRECTAS:
A. La disminución del flujo de aire es en el momento de la espiración
B. El flujo sanguíneo pulmonar se ve limitado por la disminución de oxigeno

PTS: 1 DIF: Media


REF: Global initiative for asthma. Global strategy for asthma management and prevention. Updated
2017.
OBJ: Evaluar los conocimientos de crisis asmatica TOP: CRISIS ASMATICA
KEY: Pediatria NOT: Dr. Angel Mayacela

14
ID: A

39. ANS: D
los signos anormales son:
disminucion de la entrada de aire o sibilancias en la auscultacion
una fase espiratoria prolongada
tos seca
signos de rinitis, conjuntivitis y sinusitis
signos de una infeccion respiratoria aguda.

RESPUESTAS INCORRECTAS:
Todas son correctas

PTS: 1 DIF: Media


REF: Asthma-symtoms and Diagnostic in Children beyond the basics by the UpToDate Terms of Use
©2017 UpToDate, Inc. OBJ: Evaluar los conociemientos de crisis asmatica
TOP: CRISIS ASMATICA KEY: PEDIATRIA
NOT: Dr. Angel Mayacela
40. ANS: C
las mediaciones de espirometria incluyen capacidad vital forzada y el volumen espiratorio forzado en un
segundo.
RESPUESTAS INCORRECTAS:
A. La tomografía mullticorte no es el gold estándar para el diagnostico de asma
B. Los parámetros radiológicos no sojn de gran utilidad para el diagnostico

PTS: 1 DIF: Media


REF: Asthma-symtoms and Diagnostic in Children beyond the basics by the UpToDate Terms of Use
©2017 UpToDate, Inc. OBJ: evaluar los conociemientos de crisis asmatica
TOP: CRISIS ASMATICA KEY: Pediatria NOT: Dr. Angel Mayacela
41. ANS: A
La respuesta correcta es la A porque ambos objetivos disminuyen al máximo el número de recaídas.

Justificación de cada opción incorrecta:


B: Al conseguir una broncoconstricción empeoraría la crisis asmática
C: La técnica inhalatoria y el control ambiental seria subsecuente a los objetivos principales

D: No corresponde

PTS: 1 DIF: Media REF: Jesús Sánchez Etxaniz, Urgencias de Pediatría. Pág.
55
OBJ: Evaluar el nivel de conocimientos TOP: Crisis Asmatica
KEY: Pediatria NOT: Dr. Angel Mayacela

15
ID: A

42. ANS: C
La respuesta correcta es la C porque en conjunto el oxígeno y los fármacos consiguen la estabilización
optima del paciente.

Justificación de cada opción incorrecta:


A: El oxígeno es fundamental y complementario a los fármacos señalados

B: El oxígeno debe ir acompañado de fármacos para conseguir la estabilización requerida

D: No corresponde

PTS: 1 DIF: Media REF: Jesús Sánchez Etxaniz, Urgencias de Pediatría. Pág.
55
OBJ: Evaluar el nivel de conocimientos TOP: Crisis Asmatica
KEY: Pediatria NOT: Dr. Angel Mayacela
43. ANS: B
La respuesta correcta es la B porque al no ser crisis leves cursan con más graves alteraciones.

Justificación de cada opción incorrecta:


A: Esta hipoventilación alveolar no se presenta sola sino mas bien en conjunto

C: Las alteraciones en la perfusión esta en relación con la ventilación

D: No corresponde

PTS: 1 DIF: Media REF: Jesús Sánchez Etxaniz, Urgencias de Pediatría. Pág.
56
OBJ: Evaluar el nivel de conocimientos TOP: Crisis Asmatica
KEY: Pediatria NOT: Dr. Angel Mayacela
44. ANS: A
La respuesta correcta es la A al tener en cuenta el porcentaje de saturación del oxígeno.

Justificación de cada opción incorrecta:


B: No en todas las crisis es fundamental tener en cuenta la SO.

C: Si es lo más conveniente pero existen otros parámetros que se suman a esta opción.

D: No corresponde

PTS: 1 DIF: Media REF: Jesús Sánchez Etxaniz, Urgencias de Pediatría. Pág.
56
OBJ: Evaluar el nivel de conocimientos TOP: Crisis Asmàtica
KEY: Pediatria NOT: Dr. Angel Mayacela

16
ID: A

45. ANS: B
La respuesta correcta es la B porque es la saturación optima del paciente para mantenerlo estable.

Justificación de cada opción incorrecta:


A: Con esta saturación se deberá reevaluar si el tratamiento farmacológico realizado hasta ese momento
es suficiente o no

C: Esta saturación es adecuada sin embargo es más de la base que se necesita para conocer la correcta saturación del
paciente.

D: No corresponde

PTS: 1 DIF: Media REF: Jesús Sánchez Etxaniz, Urgencias de Pediatría. Pág.
56
OBJ: Evaluar el nivel de conocimientos TOP: Crisis Asmatica
KEY: Pediatria NOT: Dr. Angel Mayacela
46. ANS: A
La respuesta correcta es la A porque son fármacos de alivio rápido utilizados a demanda en la crisis
asmática.

Justificación de cada opción incorrecta:


B: Los efectos farmacológicos comienzan a partir de las 2 horas de su administración
C: Medicamentos de segunda línea.

D: No corresponde

PTS: 1 DIF: Media REF: Jesús Sánchez Etxaniz, Urgencias de Pediatría. Pág.
55
OBJ: Evaluar el nivel de conocimiento con respecto al tratamiento de la crisis asmática
TOP: Crisis asmática KEY: Pediatria NOT: Dr. Angel Mayacela
47. ANS: B
La respuesta correcta es la B porque permite la rápida absorción del medicamento.

Justificación de cada opción incorrecta:


A: escasa eficacia

C. se reservan para las crisis severas con riegos de parada cardiorrespiratoria o escasa eficacia de la vía
inhalatoria

D: no corresponde

PTS: 1 DIF: Media REF: Jesús Sánchez Etxaniz, Urgencias de Pediatría. Pág.
55
OBJ: Evaluar conocimiento con respecto a las principal vía de administración de los b2 adrenérgicos
TOP: Crisis asmàtica KEY: Pediatria NOT: Dr. Angel Mayacela

17
ID: A

48. ANS: C
La respuesta correcta es la C por generar partículas aerosolizadas que son dispersadas continuamente para
alcanzar el árbol bronquial, sin demasiadas limitaciones.

Justificación de cada opción incorrecta:


A: no es el mejor método de administrara un B-2 en urgencias porque es `reciso que el niño tenga un
flujo inspiratorio mínimo de unos 30 l/min, que corresponda a una edad > 6 años
B: el tamaño de las partìculas puede variar segùn la temperatura del inhalador. Además la utilización de
la càmara encarece el tratamiento
D: No corresponde

PTS: 1 DIF: Alta REF: Jesús Sánchez Etxaniz, Urgencias de Pediatría. Pág.
56
OBJ: Evaluar el conocimiento con respecto a la técnicas de inhalación de B2 adrenérgicos en una
urgencia pediàtrica TOP: Crisis asmàtica KEY: Pediatrìa
NOT: Dr. àngel Mayacela
49. ANS: A
La respuesta correcta es la A por ser menos invasiva y más económica

Justificación de cada opción incorrecta:


B: se reserva para casos graves o cuando los niños no son capaces de retener la v.o

C: más invasiva y menos económica.

D: No corresponde

PTS: 1 DIF: Alta


REF: JA. Castro-Rodríguez. Anales de la pediatría. Vol 67.Núm.4
OBJ: Evaluar el conocimiento con respecto a la vía de administración de mayor preferencia para los
glucocorticoides sistémicos TOP: Crisis asmática
KEY: Pediatria NOT: Dr. Àngel Mayacela
50. ANS: B
La respuesta correcta es la B porque se tarda en nebulizar 10-20 minutos.

Justificación de cada opción incorrecta:

A:Limitación del inhalador presurizado con cámara espaciadora

C: Limitación del dispositivo de polvo seco

D: No corresponde

PTS: 1 DIF: Alta REF: Jesús Sánchez Etxaniz, Urgencias de Pediatría. Pág.
56
OBJ: Evaluar el conocimeinto con respecto a las limitaciones del nebulizador en el tratamiento de las
crisis asmàtica TOP: Crisis asmática KEY: Pediatria
NOT: Dr. Ángel Mayacela
51. ANS: A
La respuesta correcta es la A, puesto que la forma móvil de la entamoeba histolitica es el trofozoito.

PTS: 1

18
ID: A

52. ANS: D
La respuesta correcta es la D, ya que la dosis del Metronidazol en neonatos es de 7,5mg/kg/dosis; además
de no ser tratamiento en la Shigelosis

PTS: 1
53. ANS: A
La respuesta correcta es la A ya que salmonella invade al hospedero a través de tejido linfoide,
incluyendo las placas de Peyer ,Se adhiere apicalmente a las células epiteliales del íleon y a las células M
que debido a la ausencia del borde de cepillo así como de glicocalix, representan una puerta de entrada
ideal para las enterobacterias

PTS: 1
54. ANS: A
La respuesta correcta es la A, puesto que la patogenia de la disentería enteroinvasiva empieza con su
multiplicación, invade las células epiteliales del colon, se adhiere, destruye la membrana de las
células y por ultimo causa ulceración e inflamación

PTS: 1
55. ANS: D
La respuesta es todas ya que su morfologia es completa

PTS: 1
56. ANS: A
La respuesta correcta es la forma infectante ya que el quiste es altamente infeccioso

PTS: 1
57. ANS: C
Gracias a la adhesina y cisteina es altamente adherible a la pared intestinal

PTS: 1
58. ANS: D
Segun la epidemiologia corresponde a todas las anteriores

PTS: 1
59. ANS: D
En cuanto a la transmisión todas son correctas de acuerdo a la infección tanto de alimentos como sus
vectores

PTS: 1
60. ANS: D
Mediante el ciclo de vida todos los procesos son correctos en cuanto su desarrollo en el interior del
paciente

PTS: 1
61. ANS: A
Se desencadenan de las celulas M del colon

PTS: 1

19
ID: A

62. ANS: B
Las parestesias no constituyen una manifestacion clinica de la shigelosis

PTS: 1
63. ANS: A
los flagelos perítricos le dan gran mobilidad

PTS: 1
64. ANS: C
la Penicilina no corresponde al espectro sobre la fiebre tifoidea

PTS: 1
65. ANS: D
para el Dg de salmonelosis se utilizan todas las anteriores

PTS: 1
66. ANS: D
todas corresaponden a los sintomas de la salmonelosis

PTS: 1
67. ANS: D
coreesponden todas las anteriores a las manifestqaciones

PTS: 1
68. ANS: C
Hay cuatro especies de Shigella, responsables de shigelosis: grupo A (Shigella dysenteriae), grupo B
(Shigella flexneri), grupo C (Shigella boydii) y grupo D (Shigella sonnei).

PTS: 1
69. ANS: D
Tres especies patogenas en el hombre

PTS: 1
70. ANS: B
El periodo de incubación corresponde de 3 a 7 días

PTS: 1
71. ANS: D
La respuesta correcta es la D porque la EDA se considera a heces liquidas más de lo normal.

A: No es la definición de EDA es el número de causa que produce la diarrea


B: No es la definición de EDA es el número de casos que se presenta
C: No es la definición de EDA menciona el agente común más frecuente que causa diarrea

PTS: 1 DIF: Alta OBJ: Evaluar los conocimientos sobre EDA en pediatria
TOP: Pediatría NOT: Dr. Angel Mayacela

20
ID: A

72. ANS: B
la respuesta correcta es la B porque la diarrea con sangre aguda, también llamada diarrea disentérica o
disentería

A: La diarrea acuosa aguda, que dura varias horas o días, y comprende el cólera

C: La diarrea persistente dura 14 días o más

D: Es un tipo de diarrea viral

PTS: 1 DIF: Alta


REF: ORGANIZACIÓN MUNDIAL DE SALUD |
http://www.who.int/mediacentre/factsheets/fs330/es/
OBJ: Evaluar los conocimientos sobre EDA en pediatria TOP: Diarrea
NOT: Dr. Angel Mayacela
73. ANS: A
La respuesta correcta es la A porque la mayor causa de la Eda son las infecciones

B: Mal manejo del agua


C: Fuente de agua contaminada
D: Por malnutrición

PTS: 1 DIF: Media


REF: ORGANIZACIÓN MUNDIAL DE SALUD |
http://www.who.int/mediacentre/factsheets/fs330/es/
OBJ: Evaluar los conocimientos sobre EDA en pediatría TOP: Causa de diarrea
NOT: Dr. Angel Mayacela
74. ANS: C
El agente causal más común de diarrea es el rotavirus

Trichomonas es el agente mas comun en infecciones vaginales

Influenza es agente mas comun en el aparato respiratorio

Salmonella es el agente comun en el aparato digestivo

PTS: 1 DIF: Media


REF: GACETA EPIDEMIOLOGICA DEL MINESTERIO DE SALUD PUBLICA PROGRAMA DE
INMUNIZACIONES |
http://www.paho.org/ecu/index.php?option=com_docman&view=download&category_slug=inmunizacio
nes&alias=611-protocolo-para-la-vigilancia-epidemiologica-hospitalaria-centinela-de-diarreas-causadas-p
or-rotavirus-1&Itemid=599 OBJ: Evaluar los conocimientos sobre EDA en pediatría
TOP: Agente epidemiologico NOT: Dr. Angel Mayacela

21
ID: A

75. ANS: D
Números de casos exactos de EDA en la provincia

A: No es cifra correcta
B: No es cifra correcta
C: No es cifra correcta

PTS: 1 DIF: Media


REF: ANUARIO DE VIGILANCIA EPIDEMIOLOGICA ENFERMEDADES TRANSMITIDAS POR
AGUA Y ALIMENTOS |
http://www.paho.org/ecu/index.php?option=com_docman&view=download&category_slug=inmunizacio
nes&alias=611-protocolo-para-la-vigilancia-epidemiologica-hospitalaria-centinela-de-diarreas-causadas-p
or-rotavirus-1&Itemid=599 OBJ: Evaluar los conocimientos sobre EDA en pediatría
TOP: NUmero de casos de EDA NOT: Dr. Angel Mayacela
76. ANS: A
Los virus, principalmente especies de rotavirus, son responsables del 70 al 80% de casos de diarrea
infecciosa a nivel mundial.

B.- Las bacterias son responsables del 10 a 20% de los casos de EDA.
C.- Los parasitos son responsables < 10% de los casos de EDA.

D.- Los HONGOS son responsables < 5% de los casos de EDA.

PTS: 1 DIF: Media


REF: SEMAR: Guía de práctica clínica: Diarrea Aguda. México, pág. 2-6. (2010)
OBJ: Evaluar los conocimientos sobre EDA en pediatría TOP: Etiologìa
NOT: Dr. Ángel Mayacela
77. ANS: B
B.-Los rotavirus constituyen el principal agente etiológico productor de diarrea en la infancia, se asocian
a una forma de enfermedad más grave e infectan prácticamente a todos los niños en los 4 primeros años
de vida. Es responsable de un tercio de las hospitalizaciones por diarrea y 500 000 muertes a nivel
mundial cada año.

A.- El adenovirus es responsable de un porcentaje menor de diarrea aguda en niños y producen un cuadro
clínico más leve.
C.- El astrovirus son responsable de un porcentaje menor de diarrea aguda en niños y producen un cuadro
clínico más leve.
D.- El rinoovirus son responsable de un porcentaje menor de diarrea aguda en niños y producen un
cuadro clínico más leve.

PTS: 1 DIF: Media


REF: Organización Mundial de Gastroenterología. Diarrea Aguda. (2012)
OBJ: Evaluar los conocimientos sobre EDA en pediatría TOP: Etiologia
NOT: Dr. Ángel Mayacela

22
ID: A

78. ANS: A
Justificación y fuentes de la opción correcta: Es el mecanismo fundamental del Vibrio cholerae, E.
Coli enterotoxígeno y Citrobacter, en el cual segregan enterotoxinas que se adhieren a la superficie del
enterocito y estimulan mediadores intracelulares denominados segundos mensajeros (enterotoxina
resistente al calor: GMPc o enterotoxina termolábil:AMPc), que actúan como secretagogos, pues inhiben
la entrada de sodio y cloro en la parte superior de la vellosidad intestinal y favorecen la salida de sodio y
agua en las criptas de las vellosidades. En este tipo de diarrea las deposiciones serán líquidas explosivas,
sin sangre.

Justificación de cada opción incorrecta:


B.- No es correcta porque el mecanismo patogenico caracteristico del Rotavirus y adenovirus se
denomina Mecanismo Osmótico.
C.- No es correcta porque el mecanismo enterotoxico constituye el mecanismo fundamental del Vibrio
cholerae, E. Coli enterotoxígeno y Citrobacter.
D.- No es correcta porque los hongos no cumplen con este mecanismo.

PTS: 1 DIF: Media


REF: Cruz M. Nuevo Tratado de Pediatría. In.: Medica Panamericana S.A.; 2012. p. 1250-1255.
OBJ: Evaluar los conocimientos sobre EDA en pediatría TOP: Fisiopatologia
NOT: Dr. Ángel Mayacela
79. ANS: A
Es el mecanismo clásico de bacterias como Salmonella, Shigella y Yersinia, pues producen destrucción
del borde en cepillo de las células epiteliales del intestino, para penetrar en el interior de las células,
produciendo un desequilibrio entre la secreción y la absorción de líquidos con una excesiva secreción de
agua y electrolitos.

B.- No es correcta porque el mecanismo enterotoxico constituye el mecanismo fundamental de


Salmonella, Shigella y Yersinia.
C.- No es correcta porque el mecanismo enterotoxico constituye el mecanismo fundamental de
Salmonella, Shigella y Yersinia.
D.- No es correcta porque el mecanismo enterotoxico constituye el mecanismo fundamental de
Salmonella, Shigella y Yersinia.

PTS: 1 DIF: Media


REF: Cruz M. Nuevo Tratado de Pediatría. In.: Medica Panamericana S.A.; 2012. p. 1250-1255.
OBJ: Evaluar los conocimientos sobre EDA en pediatría TOP: Fisiopatologia
NOT: Dr. Ángel Mayacela

23
ID: A

80. ANS: C
El mecanismo enteroivasivo produce diarrea que se caracterizan por deposiciones que presentan
leucocitos, moco (respuesta inflamatoria del colon) y podrán tener sangre si hay afección de la
submucosa.

A.- No es correcta porque las deposiciones líquidas explosivas y sin sangre son caracteristicas del
mecanismo enterotóxico.
B.- No es correcta porque las deposiciones con moco y sangre son caracteristicas del mecanismo
osmótico.
D.- No es correcta porque las deposiciones presentan leucocitos, moco y podrán tener sangre si hay
afección de la submucosa.

PTS: 1 DIF: Media


REF: Cruz M. Nuevo Tratado de Pediatría. In.: Medica Panamericana S.A.; 2012. p. 1250-1255.
OBJ: Evaluar los conocimientos sobre EDA en pediatría TOP: Fisiopatologia
NOT: Dr. Ángel Mayacela
81. ANS: A
La respuesta correcta es la A por estudios realizados , a nivel mundial por la OPS

B: No es la adecuada
C: No es la adecuada
D: No es la adecuada

PTS: 1 DIF: Media


REF: http://www.ops.org.gt/ADS/PERFIL%20DE%20SITUACION%20AMBIENTAL%2 0NIÑE
Z-GUATEMALA%20Versión%203%20ab%20%2003.pdf
OBJ: Evaluar los conocimientos sobre EDA en pediatría TOP: factores de riesgo
NOT: Dr. Ángel Mayacela
82. ANS: B
El agua contaminada tanto por heces humanas como por heces de animales pueden conllevar a un alto
grado de amenaza ya que esta agua es usada tanto para los cultivos.

A:No es la correcta ya que puede ser que el uso de estos pesticidas lleven a otro tipo enfermedades
C: no es la correcta.
D: Los cultivos en invernaderos pueden ser organicos

PTS: 1 DIF: Media


REF: https://sites.google.com//sanmartinpastor.com/s720116b/enfermedad-diarreica-aguda-eda
OBJ: Evaluar los conocimientos sobre EDA en pediatría TOP: factores de riesgo
NOT: Dr. Ángel Mayacela

24
ID: A

83. ANS: B
Alimentados a pecho, las infecciones entéricas son raras y cuando ocurren, el cuadro se autolimita más
rápidamente por la inmunidad que adquiere a travez de los antigenos en la leche maternal.

A: la lactancia brinda propiedades para autoinmunitarias


C: la lactancia no debe se limitada ya que es el mejor alimento a nivel pediatrico
D:la lactancia brinda propiedades para autoinmunitarias que ayudan en el proceso infeccioso

PTS: 1 DIF: Media


REF: https://sites.google.com//sanmartinpastor.com/s720116b/enfermedad-diarreica-aguda-eda
OBJ: Evaluar los conocimientos sobre EDA en pediatría TOP: Factores de riesgo
NOT: Dr. Ángel Mayacela
84. ANS: C
En pacientes desnutridos la velocidad de recuperación de la mucosa intestinal es más lenta, puede estar
aumentada la susceptibilidad a diarreas prolongadas que deterioran aún más su estado nutricional.

A.- no es correcta ya que en niños desnutridos son mas propensos a contraer una EDA

B.- la recupercion es lenta

D si influye ya que poacientes desnutridos tienen mas a una EDA

PTS: 1 DIF: Media


REF: https://sites.google.com//sanmartinpastor.com/s720116b/enfermedad-diarreica-aguda-eda
OBJ: Evaluar los conocimientos sobre EDA en pediatría TOP: Factores de riesgo
NOT: Dr. Ángel Mayacela
85. ANS: B
por que a nivel de la flora intestinal se presenta , mala absorcion y aumento de la motilidad intestinal

A: no es la correcta ya que aumenta el número de deposiciones

C: es la incorrecta ya que disminuye el apetito

D: estos se presentarian en enfermdades a nivel nerviso

PTS: 1 DIF: Media


REF: https://sites.google.com//sanmartinpastor.com/s720116b/enfermedad-diarreica-aguda-eda
OBJ: Evaluar los conocimientos sobre EDA en pediatría TOP: Manifestaciones clinicas
NOT: Dr. Ángel Mayacela

25
ID: A

86. ANS: A
La respuesta correcta es la A ya que es la medida por la cual se puede observar las principales
características de las heces.

B.Son detectables con otros tipos de exámenes

C.Las características de las bacterias se podrán observar en un cultivo

D. No está relacionada con la pregunta

PTS: 1 DIF: Alta REF: Cruz M, Jiménez R. Nuevo tratado de Pediatría. 2014.
OBJ: Aprender como diagnósticar EDA TOP: Pediatria KEY: Diagnóstico
NOT: Dr. Angel Mayacela
87. ANS: B
La respuesta correcta es B, ya que para poder realizar una correcta historia clínica en especial la
anamnesis se debe seguir los pasos correctos y secuenciales para poder tener una buena agrupación
sindrómica.

A. Falta de enunciados para realizar la anamnesis

C. Falta de enunciados para realizar la anamnesis

D. Falta de enunciados para realizar la anamnesis

PTS: 1 DIF: Alta REF: Cruz M, Jiménez R. Nuevo tratado de Pediatría. 2014.
OBJ: Aprender diagnósticar correctamente una EDA TOP: Pediatria
KEY: Anamnesis NOT: Dr. Angel Mayacela
88. ANS: C
La respuesta correcta es C, ya que para poder realizar una correcta historia clínica en especial los
antecedentes personales del niño/a se debe seguir los pasos correctos y secuenciales para poder tener una
buena agrupación sindrómica

A. Son escasos datos para los antecedentes personales


B. Son escasos datos para los antecedentes personales
D. Son escasos datos para los antecedentes personales

PTS: 1 DIF: Alta REF: Cruz M, Jiménez R. Nuevo tratado de Pediatría. 2014.
OBJ: Aprender a diagnòsticar una EDA TOP: Pediatría KEY: antecedentes
NOT: Dr. Angel Mayacela

26
ID: A

89. ANS: D
La respuesta correcta es D, ya que para poder realizar una correcta historia clínica en especial los
antecedentes personales del niño/a se debe seguir los pasos correctos y secuenciales para poder tener una
buena agrupación sindrómica

A. La producción continua de haces >15g/kg/día hace sospechar enfermedad orgánica

B. Los mecanismos incluyen transporte anormal de agua y sal, pérdida de área de superficie mucosa,
aumento de la motilidad intestinal, agentes osmóticos, aumento de la permeabilidad intestinal y
estimulantes de la secreción

C. La causa más frecuente en lactantes sanos es el uso de antibióticos seguido de diarrea crónica
inespecífica

PTS: 1 DIF: Alta REF: Cruz M, Jiménez R. Nuevo tratado de Pediatría. 2014
OBJ: Aprender como diagnòsticar una EDA TOP: Pediatría
KEY: Diarrea NOT: Dr. Angel Mayacela
90. ANS: A
La respuesta correcta es A, ya que se debe diferenciar de estas entidades ya que en la mayoría de los
casos se puede presentar sintomatología idéntica

B. no corresponde a la pregunta
C. se puede presentar diarreas pero siempre y cuando haya intolerancia a ciertos
D. fármacos y que se puede acompañar de otras manifestaciones clínicas adicionales
las malformaciones no están relacionadas con esta entidad patológica

PTS: 1 DIF: Alta


REF: Cruz M, Jiménez R. Nuevo tratado de Pediatría. 2014. Pág. 1237
OBJ: Aprender a diferenciar a una EDA TOP: Pediatria KEY: DIferencial
NOT: Dr. Angel Mayacela
91. ANS: C
La respuesta correcta es la C una correcta hidratación en un niño con que tienen deshidratación leve a
moderada secundaria es 30-80ml/Kg

A: No es la hidratación adecuada
B: No es la hidratación adecuada
D: No es la hidratación adecuada

PTS: 1 DIF: Alta


REF: Guia de practica Clinica , Prevención, Diagnóstico y Tratamiento de la Diarrea Aguda en Niños de
Dos Meses a Cinco Años en el Primero y Segundo Nivel de Atención Pág. 5
OBJ: Aprender sobre el tratamiento TOP: Pediatria KEY: Suero Oral
NOT: Dr. Angel Mayacela

27
ID: A

92. ANS: B
La respuesta correcta es B:Un adecuado tratamiento se debe dar 50ml/kg para reponer el déficit de
líquidos durante 4h, así como los líquidos de mantenimiento.

A: en una correcta hidratación el uso de soluciones es de BAJA osmolaridad (240-250 mOsm/l) para la
rehidratación oral
C: SI se debe dar SRO frecuentemente y en pequeñas cantidades

D: No se debe administrar lactato ringer

PTS: 1 DIF: Alta


REF: Guia de practica Clinica , Prevención, Diagnóstico y Tratamiento de la Diarrea Aguda en Niños de
Dos Meses a Cinco Años en el Primero y Segundo Nivel de Atención Pág. 11
OBJ: Aprender sobre Hidratacion neonatal TOP: Pediatria
KEY: Hidratacion NOT: Dr. Angel Mayacela
93. ANS: B
La respuesta correcta es la B: en caso de shock hipovolémico está recomendado hidratación por vía
intravenosa

A: deshidratación grave
C:alteración de nivel de conciencia
D:perdidas fecales intensas

PTS: 1 DIF: Alta


REF: Diarrea aguda Enriqueta Román Riechmann1, Josefa Barrio Torres2, Mª José López Rodríguez3
pág. 16 OBJ: Tratamiento de EDA TOP: Pediatria
KEY: Shock NOT: Dr. Angel Mayacela
94. ANS: A
La respùesta correcta es A: al único método de prevención de la enfermedad diarreica aguda es una
correcta higiene de alimentos y del ambiente.

B: No es el adecuado metodo de prevencion


C:No es el adecuado metodo de prevencion
D:No es el adecuado metodo de prevencion

PTS: 1 DIF: Alta


REF: Diarrea aguda Enriqueta Román Riechmann1, Josefa Barrio Torres2, Mª José López Rodríguez3
pág. 16 OBJ: Aprender sobre la prevencion de EDA TOP: Pediatria
KEY: Prevencion NOT: Dr. Angel Mayacela
95. ANS: A
la respuesta correcta es la A: los pacientes que si toleran la vía oral
B: Pacientes con algún tipo de inmunodeficiencia que cursan con diarrea y fiebre
C: Estado tóxico o sospecha de bacteriemia.
D: Neonatos y menores de tres meses con fiebre.
En los tres casos es candidato para el tratamiento intravenoso

PTS: 1 DIF: Alta


REF: Guia de practica Clinica , Prevención, Diagnóstico y Tratamiento de la Diarrea Aguda en Niños de
Dos Meses a Cinco Años en el Primero y Segundo Nivel de Atención pág. 16
OBJ: Tratamiento de EDA TOP: Pediatria KEY: Tratamiento
NOT: Dr. Angel Mayacela

28
ID: A

96. ANS: D
La respuesta Correcta es D porque En orden descendentede frecuencia, otras bacterias
implicadas incluyen otros gérmenes gramnegativos, como Proteus, Klebsiella, Enterobacter y
Citrobacter, y gérmenes grampositivos como Staphylococcus saprophyticus, Enterococcus y,
raramente, Staphylococcus aureus.

PTS: 1 DIF: Baja REF: Nuevo tratado de Pediatria M. Cruz. 2014. 1862-18
OBJ: La siguiente pregunta es de carácter objetivo (De opción múltiple)
TOP: ETIOLOGIA DE ITU NOT: DR. ANGEL MAYACELA
97. ANS: A
La respuesta correcta es la A debido a que Algunos microorganismos son capaces de producir una
ITU a través de una diseminación hematógena como la Salmonella, Mycobacterium,
Citomegalovirus.

PTS: 1 DIF: Baja REF: Nuevo tratado de Pediatria M. Cruz. 2014. 1862-18
OBJ: la pregunta es de opcion multiple TOP: Etiologia de ITU
NOT: DR. Angel Mayacela
98. ANS: A
La respuesta correcta es A . El microorganismo implicado con más frecuencia es Escherichia
coli, responsable de más del 80% de las ITU en edad pediátrica

PTS: 1 DIF: Baja REF: Nuevo tratado de Pediatria M. Cruz. 2014. 1862
OBJ: razonamiento TOP: Etiologia de ITU
NOT: DR . ANGEL MAYACELA
99. ANS: D
La respuesta correcta es D. debido a que Diferentes especies de Pseudomonas se aíslan con relativa
frecuencia en niños con malformaciones de las vías urinarias, portadores desondas, o que están
recibiendo antibióticos de amplio espectro o a dosis elevadas.

PTS: 1 DIF: MEDIA REF: Nuevo tratado de Pediatria M. Cruz. 2014. 1862-18
OBJ: Aprendizaje TOP: Etiologia de ITU NOT: DR. ANGEL MAYACELA
100. ANS: A
La respuesta es A. Compromiso bacteriano agudo del parénquima renal porque existe una
disminución del filtrado glomerular con proteinuria e hipertensión arterial, e incluso insuficiencia
renal crónica.

PTS: 1 DIF: MEDIA REF: Nuevo tratado de Pediatria M. Cruz. 2014. 1862-18
OBJ: Razonamiento TOP: Definiciones del Tracto urinario
NOT: DR. ANGEL MAYACELA
101. ANS: B
Larespuesta correcta es B. Debido a que la tasa de recurrencia de ITU neonatal se estima que es de
un 25% y, en edades posteriores, de un 30% en los varones y un 40% en las niñas. Durante los
primerosm12 meses de vida.

PTS: 1 DIF: MEDIA REF: Nuevo tratado de Pediatria M. Cruz. 2014. 1862-18
OBJ: Razonamiento. TOP: Epidemiologia de ITU
NOT: DR.ANGEL MAYACELA

29
ID: A

102. ANS: A
La respuesta correta es A. En general, se estima que el riesgo de padecer una infección urinaria
antes de la pubertad es del 3 al 5% en las niñas, y del 1 al 2% en los varones.

PTS: 1 DIF: Media REF: Nuevo tratado de Pediatria M. Cruz. 2014. 1862-18
OBJ: APRENDIZAJE TOP: Incidencia de ITU
NOT: DR. ANGEL MAYACELA
103. ANS: A
La respuesta Correcta es A. Porque La práctica ampliamente difundida en la actualidad
del análisis de orina permite conocer en un porcentaje importante de niños asintomáticos la
existencia de bacteriuria, siendo en las niñas lactantes del 2 al 4,5%

PTS: 1 DIF: MEDIA REF: Nuevo tratado de Pediatria M. Cruz. 2014. 1865-18
OBJ: CONOCIMIENTO TOP: EPIDEMIOLOGÌA DE ITU
NOT: DR. ANGEL MAYACELA
104. ANS: B
La respuesta B debido a que se presneta con mas frecuenca en niñas, en especial entre 2 a 3 años de edad
persisitiendo duranbte varios años con una frecuencia entre 1 y 10 ataques por año.

PTS: 1 DIF: MEDIA REF: Nuevo tratado de Pediatria M. Cruz. 2014. 1865-18
OBJ: conocimiento TOP: EPDEMIOLOGIA DE ITU
NOT: DR.ANGEL MAYACELA
105. ANS: C
La Respùesta correcta es C debido que a partir de un microorganismo, generalmente Staphylococcus
que, desde un foco faríngeo o cutáneo, lega al tejido intersticial del riñón, donde produce pequeños
abscesos.

PTS: 1 DIF: MEDIA REF: Nuevo tratado de Pediatria M. Cruz. 2014. 1865-18
OBJ: Conocimiento TOP: Manifestaciones de ITU
NOT: DR. ANGEL MAYACELA
106. ANS: A
La respuesta correcta es A. Porque para que exista la ITU debe haber una interacción del
microorganismo y el huésped.

PTS: 1 DIF: MEDIA REF: Nuevo tratado de Pediatria M. Cruz. 2014. 1862-1864
OBJ: CONOCIMIENTO TOP: FISIOPATOLOGIA ITU
NOT: DR. ANGEL MAYACELA
107. ANS: B
La respuesta correcta es la B porque esto facilita la posibilidad de que los microbios entren al
tracto urinario del niño

PTS: 1 DIF: MEDIA REF: Nuevo tratado de Pediatria M. Cruz. 2014. 1862-1864
OBJ: CONOCIMIENTO TOP: FISIOPATOLGIA ITU
NOT: DR. ANGEL MAYACELA

30
ID: A

108. ANS: A
La respuesta correcta es la A porque la niña tiene la uretra corta y ancha por tal motivo los
gérmenes tienen más facilidad de llegar.

PTS: 1 DIF: MEDIA REF: Nuevo tratado de Pediatria M. Cruz. 2014. 1862-1864
OBJ: CONOCIMEINTO TOP: FISIOPATOLGIA ITU
NOT: DR. ANGEL MAYACELA
109. ANS: C
La respuesta correcta es la C. Porque las bacterias que poseen fimbrias pueden adherirse al urotelio
y ascender hacia el riñón

PTS: 1 DIF: MEDIA REF: Nuevo tratado de Pediatria M. Cruz. 2014. 1862-1864
OBJ: CONOCIMIENTO TOP: FISIPATOLOGIA ITU
NOT: DR. ANGEL MAYACELA
110. ANS: A
La respuesta correcta es la A. Porque se ha establecido claramente la ruta de entrada ascendente o
retrógrada en la mayoría de ITU.

PTS: 1 DIF: MEDIA REF: Nuevo tratado de Pediatria M. Cruz. 2014. 1862-1864
OBJ: CONOCIMIENTO TOP: FISIOPATOLOGIA ITU
NOT: DR. ANGEL MAYACELA
111. ANS: A
La respuesta correcta es A porque la muestra de orina con el fin de realizar un urianálisis y si este
es patológico aumenta las probabilidades de ITU y el urocultivo positivo la confirma

PTS: 1 DIF: MEDIA REF: Nuevo tratado de Pediatria M. Cruz. 2014. 1862-1864
OBJ: CONOCIMIENTO TOP: DIAGNOSTICO DE ITU
NOT: DR. ANGEL MAYACELA
112. ANS: D
La respuesta correcta es la D porque no es un catéter uretral y sino un catéter vesical.

PTS: 1 DIF: MEDIA REF: Nuevo tratado de Pediatria M. Cruz. 2014. 1867
OBJ: CONOCIMIENTO TOP: DIAGNOSTICO DE ITU
NOT: DR. ANGEL MAYACELA
113. ANS: B
La respuesta correcta es la B porque el elemento forme que nos determina una infección urinaria
alta son cilindros leucocitarios

PTS: 1 DIF: MEDIA REF: Nuevo tratado de Pediatria M. Cruz. 2014. 1867
OBJ: CONOCIMIENTO TOP: DIAGNOSTICO ITU
NOT: DR. ANGEL MAYACELA

31
ID: A

114. ANS: A
La respuesta correcta es A porque el tipo de muestra que nos va a dar mayor probabilidad de
infección y nos confirma es la punción suprapúbica

PTS: 1 DIF: MEDIA REF: Nuevo tratado de Pediatria M. Cruz. 2014. 1868
OBJ: CONOCIMIENTO TOP: DIAGNOSTICO ITU
NOT: DR. ANGEL MAYACELA
115. ANS: A
La respuesta correcta es la A porque nos permite detectar lesión renal aguda y no crónica

PTS: 1 DIF: MEDIA REF: Nuevo tratado de Pediatria M. Cruz. 2014. 1868
OBJ: CONOCIMIENTO TOP: DIAGNOSTICO ITU
NOT: DR. ANGEL MAYACELA
116. ANS: C
La respuesta correcta es la C debido a la afectación del estado general, vomitos o deshidratación
es uno de los criterios que motivan ingreso hospitalario en las ITU

PTS: 1 DIF: MEDIA REF: Nuevo tratado de Pediatria M. Cruz. 2014. 1862-18
OBJ: CONOCIMIENTO TOP: TRATAMIENTO ITU
NOT: DR. ANGEL MAYACELA
117. ANS: B
La respuesta correcta es la B debido a que la combinación de amoxicilina y ácido clavulánico se
usa para tratar ciertas infecciones causadas por bacterias, incluyendo infecciones en los oídos,
pulmones, senos, piel y vías urinarias

PTS: 1 DIF: MEDIA REF: Nuevo tratado de Pediatria M. Cruz. 2014. 1862-18
OBJ: CONOCIMIENTO TOP: TRATAMIENTO ITU
NOT: DR. ANGEL MAYACELA
118. ANS: A
La respuesta correcta es la A debido a que en el tratamiento higienico físico se recomienda
reposo, ingesta de liquidos, higiene de los genitales etc con el fin de controlar el crecimiento
bacteriano y evitar que afecte al riñon

PTS: 1 DIF: MEDIA REF: Nuevo tratado de Pediatria M. Cruz. 2014. 1862-18
OBJ: CONOCIMIENTO TOP: TRATAMIENTO ITU
NOT: DR. ANGEL MAYACELA
119. ANS: C
La respuesta correcta es la C debido a que deberá procederse a la corrección quirúrgica de las
anomalías congénitas o adquiridas del aparato urinario ya que es el causante de ITU recurrentes

PTS: 1 DIF: MEDIA REF: Nuevo tratado de Pediatria M. Cruz. 2014. 1862-18
OBJ: CONOCIMIENTO TOP: TRATAMIENTO ITU
NOT: DR. ANGEL MAYACELA

32
ID: A

120. ANS: A
La respuesta correcta es la A debido a que la quimioprofilaxis se emplea tras una primera ITU
con el objetivo de mantener al paciente libre de ITU en caso de que presente riesgo de daño renal.

PTS: 1 DIF: MEDIA REF: Nuevo tratado de Pediatria M. Cruz. 2014. 1862-18
OBJ: CONOCIMIENTO TOP: TRATAMIENTO ITU
NOT: DR. ANGEL MAYACELA
121. ANS: C
El presente caso se trata de Fiebre escarlatina, pues sus características patognomínicas son la lengua roja
con apariencia de fresa, con exantema macupapular en tronco y extremidades.

PTS: 3 DIF: Media


REF: PALACIOS, C., DURÁN, C., OROZCO, L., GARCÍA, M., & RUIZ, R. (20 de
SEPTIEMBRE-OCTUBRE de 2015). EXANTEMAS EN PEDIATÍA. MEDIGRAPHIC, 36(5), 412-423.
TOP: Medicina NOT: Dra. Salem Rosario
122. ANS: A
El mecanismo de acción de la doxiciclina actúa inhibiendo la síntesis proteica por unión a la subunidad
ribosomal 30S, siendo asi el mejor antiparasitario.

PTS: 3 DIF: Media


REF: Kliegman R, Jenson H, Behrman R, Stanton B. Nelson Tratado de Pediatría. 18th ed. Barcelona :
Elsevier ; 2008 TOP: Medicina NOT: Dra. Salem Rosario
123. ANS: A
Para su estudio, se estiman necesarias las siguientes precisiones: En términos de la literatura
especializada, la hepatitis B es una infección vírica del hígado que puede dar lugar tanto a un cuadro
agudo como a una enfermedad crónica. El virus se transmite entre las personas por contacto directo de
sangre a sangre, a través del semen o secreciones vaginales de una persona infectada. Los modos de
transmisión son los mismos que los del virus de la inmunodeficiencia humana (VIH), pero el virus de la
hepatitis B (VHB) es entre 50 y 100 veces más infeccioso. A diferencia del VIH, el VHB puede
sobrevivir fuera del organismo durante 7 días como mínimo, y en ese lapso puede causar infección si
penetra en el organismo de una persona no protegida por la vacuna.

PTS: 3 DIF: Media REF: AMIR. Pediatria, Marban. Año: 2011. Edición: 8va
TOP: Medicina NOT: Dra. Salem Rosario
124. ANS: B
Estamos ante una Hipertrofia del piloro por que se da en niños de 2 a 4 semanas de vida y se caracteriza
por presentar vómitos de contenido gástrico de gran volumen alimentario, con aspecto de leche cortada y
se da después de un tiempo variable de alimentación, también el niño puede presentar deshidratación y
detención de su curva de crecimiento y al examen físico se palpa masa en epigastrio

PTS: 3 DIF: Media REF: AMIR. Pediatria, Marban. Año : 2011. Edición : 8va
TOP: Medicina NOT: Dra. Salem Rosario

33
ID: A

125. ANS: D
El signo de Rovsing es la presencia de dolor en la fosa iliaca derecha al presionar la fosa iliaca izquierda.
Se debe a que al presionar en ese punto el aire del colon se mueve de manera retrograda, distendiendo el
ciego donde está el apéndice inflamado, lo que provoca un aumento del dolor a ese nivel.

PTS: 3 DIF: Media REF: AMIR. Pediatria,Marban. Año : 2011. Edición : 8va.
TOP: Medicina NOT: Dra. Salem Rosario
126. ANS: C
El presente caso se trata de una infección de las vías urinarias en un niño debido a la presencia de la
sintomatología, q incluye entre los más importantes como: dolor en el costado o dolor en la espalda,
micción dolorosa, Incremento en la frecuencia urinaria incluyendo la fiebre q es la sintomatología propia
de una IVU de niños.

PTS: 3 DIF: Media


REF: Medline Plus. Infección urinaria en niños. [En línea] ADAM, 21 de diciembre de 2017.
https://medlineplus.gov/spanish/ency/article/000505.htm. TOP: Medicina
NOT: Dra. Salem Rosario
127. ANS: C
El presente caso se trata de un paciente con fenilcetonuria que se caracteriza por presentar dermatitis, la
hipopigmentación (color del pelo o de la piel bastante más claros que los hermanos o padres), así como
un olor característico de la piel o de la orina.

PTS: 3 DIF: Media REF: AMIR. Pediatria,Marban. Año : 2011. Edición : 8va
TOP: Medicina NOT: Dra. Salem Rosario
128. ANS: B
El presente caso se trata de una Sepsis Neonatal Temprana ya que se presentó en las primeras 72 horas,
también refiere sintomatología propia de la Sepsis neonatal.

PTS: 3 DIF: Media REF: AMIR. Pediatria,Marban. Año : 2011. Edición : 8va
TOP: Medicina NOT: Dra. Salem Rosario
129. ANS: A
Es necesario saber el tipo de sangre del recién nacido en cuanto a Factor Rh y Grupo ABO en caso de ser
Rh+ se pide prueba de Coombs Directo para ver si hay la formación de Anticuerpos para el Antígeno D

PTS: 3 DIF: Media


REF: Behrman, R. E., Kliegman, R. M., & Jenson, H. B. (2004). Nelson tratado de pediatria. Madrid:
Elsevier. TOP: Medicina NOT: Dra. Salem Rosario
130. ANS: B
Estamos ante un caso de asfixia neonatal debido al que el niño Nació deprimido Apgar por lo que
necesitó reanimación cardiopulmonar. A las cuatro horas post natales presentó episodio de cianosis
generalizada que mejoró con oxigenoterapia. Convulsión multifocal, se manejó con fenobarbital
(20mg/kg). El examen físico: peso 2100 g, talla: 48 cm, PC: 30,5 cm, PT: 29,5 cm, hipotonía
generalizada, hiporreflexia,

PTS: 1 DIF: Media REF: AMIR. Pediatria,Marban. Año : 2011. Edición : 8va
TOP: Medicina NOT: Dra. Rosario Salem

34
ID: A

131. ANS: B
Escala de Glasgow, escala diseñada para evaluar de manera práctica el nivel de Estado de Alerta en los
seres humanos, cuantificación de tres parámetros: la apertura ocular, la respuesta verbal y la respuesta
motora. Dando un puntaje dado a la mejor respuesta obtenida en cada categoría

PTS: 1 DIF: Media


REF: Ignacio Manrique Martinez, Pedro Jesus Alcalá Minagorre. (FEBRERO de 2016). MANEJO DEL
TRAUMATISMO CRANEAL PEDIATRICO (11edicion ed., Vol. 4). (I. V. Valencia., Ed.) VALENCIA,
MADRID, ESPAÑA: Alfaz del Pí, Alicante. TOP: Medicina
NOT: Dra. Salem Rosario
132. ANS: A
El presente caso se trata acerca de analizar la inmunización oportuna además de las complicaciones y
causas de la misma

PTS: 1 DIF: Media


REF: CENTROS PARA EL CONTROL Y LA PREVENCIÓN DE ENFERMEDADES: CDC 2017.
DISPONIBLE EN: https://espanol.cdc.gov/enes/flu/professionals/vaccination/vaccine_safety.htm
TOP: Medicina NOT: Dra. Salem Rosario
133. ANS: C
El presente caso se trata de una intolerancia alimenticia, ya que el paciente presenta el antecedente de
ingesta de alimentos ácidos el cual no consta dentro de los parámetros de inicio de la alimentación o
ablactación en donde se menciona que los alimentos ácidos no deben ser añadidos hasta los 12 meses de
edad.

PTS: 1 DIF: Media


REF: Ministerio de Salud Pública del Ecuador, NORMAS Y PROTOCOLOS DE ALIMENTACIÓN
PARA NIÑOS Y NIÑAS MENORES DE 2 AÑOS, tomado de:
http://instituciones.msp.gob.ec/images/Documentos/nutricion/Alimentacion_nino_menor_2anios.pdf ;
http://www.salud.gob.ec/unidad-de-nutricion-guias-y-manuales/ TOP: Medicina
NOT: Dra. Salem Rosario
134. ANS: D
Es un cuadro de escorbuto por déficit de vitamina C. Es más frecuente entre los 7 meses y 2 años. Se
caracteriza por irritabilidad progresiva, sobre todo, al coger al niño. Presenta hemorragias en piel y
mucosas. Las encías se encuentran edematosas y con tendencia al sangrado

PTS: 1 DIF: Media


REF: López, Joaquín. AMIR TEST. Madrid: Marban Libros. 2012. Pag: 608.
TOP: Medicina NOT: Dra. Salem Rosario
135. ANS: A
Desnutrición infantil porque el paciente presenta peso muy bajo, baja estatura, IMC bajo

PTS: 1 DIF: Media


REF: Nelson, Tratado de pediatria, Elelvier Saunders, Año:2008, Edición:18, Páginas 126-127.
TOP: Medicina NOT: Dra. Salem Rosario

35
ID: A

136. ANS: D
El literal A es correcto ya que el síndrome metabólico se asocia a un riesgo especialmente alto de
enfermedad cardiovascular, con una prevalencia global de 4% en adolescentes y del 30% en
adolescentes con sobrepeso.
El literal B es correcto ya que la enfermedad de Blount, que se caracteriza por un sobrecrecimiento
de la vertiente medial de la metáfisis proximal de la tibia, que condiciona un arqueamiento de las
piernas y el deslizamiento de la epífisis de la cabeza femoral.
El literal C es correcto, porque la apnea obstructiva del sueño es más frecuente en niños con
sobrepeso y puede contribuir a problemas como la hipertensión, la fatiga diurna y la hipertensión
pulmonar.

PTS: 1 DIF: Media


REF: Nelson, Tratado de pediatria, Elelvier Saunders, Año: 2008, Edición:18, Página 237.
TOP: Medicina NOT: Dra. Salem Rosario
137. ANS: B
Estamos ante un caso de Laringotraqueobronquitis, caracterizado por cierto grado de rinorrea, faringitis,
tos leve y febrícula durante 1-3 días antes de la aparición de signos y síntomas claros de obstrucción de la
vía alta y que se presenta a partir de 6 meses a 3 años de edad.

PTS: 1 DIF: Media


REF: Nelson, Tratado de pediatria, Elelvier Saunders, Año:2008, Edición:18, Página 1762.
TOP: Medicina NOT: Dra. Salem Rosario
138. ANS: D
El rotavirus es un virus que tiene tropismo por la mucosa digestiva y es, por tanto, la causa más frecuente
de gastroenteritis aguda en la edad pediátrica, pero sin embargo altísimamente infrecuente en la etiología
de un neumonía.

PTS: 1 DIF: Media


REF: Steven R. Boas Tratado de Pediatría de Nelson 19 Edición Capitulo 397 pág. 1796
TOP: Medicina NOT: Dra. Salem Rosario
139. ANS: D
El antibiótico de elección es la penicilina pues tiene la mayor cobertura de gérmenes encontrados en la
saliva canina, ya que las heridas por mordedura de perro pueden ser de origen polimicrobiano.

PTS: 1 DIF: Media REF: AMIR. Pediatria,Marban. Año : 2011. Edición : 8va
TOP: Medicina NOT: Dra. Salem Rosario
140. ANS: A
Estamos ante un caso de EDA más deshidratación grave, debido que el paciente cursara estuporoso,
letárgico, sudoroso, hipotónico. Mucosas orales: muy secas sin saliva. Ojos: muy hundidos. Piel
turgencia cutánea: ++/+++ . Depresión de fontanela: muy hundida. Respiración muy rápida.

PTS: 1 DIF: Media REF: AMIR. Pediatria,Marban. Año : 2011. Edición : 8va
TOP: Medicina NOT: Dra. Salem Rosario
141. ANS: A
Estamos ante un caso de RGE leve, caracterizado por pequeños vómitos sin fuerza después de las tomas,
por lo demás el niño se encuentra asintomático con peso y talla en percentiles normales.

PTS: 1 DIF: Media REF: AMIR. Pediatria, Marban. Año : 2011. Edición : 8va
TOP: Medicina NOT: Dra. Salem Rosario

36
COMP PEDIATRIA [Answer Strip] ID: A

B
_____ 3. B
_____ 6. B 12.
_____ C 18.
_____

A
_____ 1.
A
_____ 7.
C 13.
_____

B 19.
_____

C
_____ 8. D 14.
_____

D
_____ 2. B A 15.
_____
_____ 4.
A 20.
_____

D
_____ 9.

A 16.
_____
A 21.
_____

A 10.
_____
A
_____ 5.
B 22.
_____

C 17.
_____

A 11.
_____
COMP PEDIATRIA [Answer Strip] ID: A

C 23.
_____ B 27.
_____ D 33.
_____ C 40.
_____ C 48.
_____

A 41.
_____
C 28.
_____
D 34.
_____ A 49.
_____

D 24.
_____ C 42.
_____
B 50.
_____
A 29.
_____
D 35.
_____

B 43.
_____

A 51.
_____
B 30.
_____

D 36.
_____
A 44.
_____

B 25.
_____

A 37.
_____

B 31.
_____ B 45.
_____ D 52.
_____

C 38.
_____

A 46.
_____
A 32.
_____
A 26.
_____

B 47.
_____
D 39.
_____
COMP PEDIATRIA [Answer Strip] ID: A

A 53.
_____ B 62.
_____ B 72.
_____ A 78.
_____ A 81.
_____

A 63.
_____

A 73.
_____
B 82.
_____
C 64.
_____

A 54.
_____ D 65.
_____
C 74.
_____

D 66.
_____

B 83.
_____
D 75.
_____ A 79.
_____
D 67.
_____

D 55.
_____
C 68.
_____
A 76.
_____

A 56.
_____
C 84.
_____

D 69.
_____
C 57.
_____ B 77.
_____

D 58.
_____ B 70.
_____

D 59.
_____ D 71.
_____ B 85.
_____
C 80.
_____

D 60.
_____

A 61.
_____
COMP PEDIATRIA [Answer Strip] ID: A

A 86.
_____ A 90.
_____ D 96.
_____ A
_____102. A
_____108.

A 97.
_____
B 87.
_____ C
_____109.
A
_____103.

C 91.
_____

A
_____110.
A 98.
_____
B
_____104.
C 88.
_____ B 92.
_____

A
_____111.
D 99.
_____

C
_____105.

D
_____112.
B 93.
_____
A
_____100.

D 89.
_____ A
_____106.

B
_____113.
A 94.
_____

B
_____101.

B A
_____114.
_____107.
A 95.
_____
COMP PEDIATRIA [Answer Strip] ID: A

A
_____115. A
_____120. D
_____125. B
_____130. A
_____135.

C
_____116. C
_____121.
C
_____126.
D
_____136.

B
_____131.

A
_____122.

B
_____117.

B
_____137.
A
_____132.
C
_____127.

A
_____123. D
_____138.
A
_____118.
B
_____128.

C
_____133.

D
_____139.

C
_____119.
A
_____129.
D
_____134.
A
_____140.

B
_____124.
COMP PEDIATRIA [Answer Strip] ID: A

A
_____141.
Name: ________________________ Class: ___________________ Date: __________ ID: A

REACTIVOS DE MEDICINA INTERNA: NEFROLOGÍA

Multiple Choice
Identify the choice that best completes the statement or answers the question.

____ 1. Paciente femenino 23 años refiere disuria, polaquiuria y tenesmo vesical cual maniobra utiliza para
examinar los riñones

a. Maniobra mcburney
b. Maniobra de guyon
c. Maniobra de murphy
d. Maniobra de blumberg

____ 2. Paciente hombre de 44 años de edad, presenta hipertensión arterial difìcil de controlar, la cual no esta
asociada a medicamentos ni a factor genetico, ademas al examen fisico se escucha un soplo sobre el
riñon, al monento de la auscultaciòn. Se sospecha de una estenosis de la arteria renal. ¿que examen
imagenologico se puede realizar para comfirmar el diagnostico?

a. Rx simple de abdomen
b. Eco Dopler renal
c. Ecografia renal
d. Gammagrafìa renal

____ 3. Paciente femenino de 45 años que presenta proteinuria (1.5g/24h), hematuria con cilindros eritrociticos,
piuria, hipertension arterial, retencion de liquidos e incremento de la concentracion de creatinina serica,
el cuadro se acompaña de disminucion del filtrado glomerular. ¿cual es su diagnostico?

a. Sindrome nefrítico agudo


b. Tumor Renal
c. Sindrome nefrotico
d. Nefrolitiasis

____ 4. Preescolar de 4 años, sexo femenino.Consultó en el Servicio de Urgencia por vómitos intensos,
edema leve, oliguria, se constató hematuria microscópica, creatinina plasmática de 0,33 mg/dl y
ecografía renal que fue informada normal. Se continúa en seguimiento evolucionando
posteriormente con persistencia de la hematuria y aparición de proteinuria sobre los 3.5 gr/24h,
hipertrigliceridemia, hipercolesterolemia, hipoalbuminemia, hipercoagulabilidad, hinchazón en la
cara y alrededor de los ojos y compromiso de la función renal en forma progresiva. ¿Cuales el
diagnostico?

a. Sindrome Nefrotico
b. SindromeNefritico
c. IVU
d. Insuficiencia Renal Aguda

1
Name: ________________________ ID: A

____ 5. Mujer de 68 años de edad, diagnosticada hace 12 años de Diabetes Mellitus, obesa, hipertensa. Al
examen del laboratorio presenta (filtrado glomerular estimado de 38 mL/min/l, albuminuria de 420 mg
en orina de 24 h). Ecografícamente: Riñon disminuido de tamaño. ¿Identifique cual es el Estadio de
Nefropatía Diabetica que se encuentra la paciente?

a. Estadio I
b. Estadio II
c. Estadio III
d. Estadio IV

____ 6. Un paciente de 85 años acude al servicio de salud con debilidad muscular, temblor de brazos y piernas ,
hormigueos y parestesia, insomnio y disminucion de actividad cardiaca. La presión arterial es de 90/60
mmHg Potasio (K+) 6 mEq/L Mg de 2 mg/dL Ca 9 mg/ dl. En el ECG destacan T prominente
prolongaciòn del intervalo PR perdida de la onda P y ensanchamiento del compleho QRS acompañado de
arritmias ventriculares ¿Cual es su diagnostico?
a. Hiperpotasemia c. Hipercalcemia
b. Hipermagnesemia d. Hipopotasemia

____ 7. Un paciente de 80 años con episodios de diarreas abundantes acude al hospital por astenia y debilidad
muscular severa. La presión arterial es de 150/80 mmHg Potasio (K+) 2.5 mEq/L Mg de 2 mg/dL Ca 9
mg/ dl. En el ECG destacan descenso del segmento ST con aplanamiento de la onda T y aumento de
amplitud de la onda U, Cual es su diagnostico?
a. Hiperpotasemia c. Hipercalcemia
b. Hipermagnesemia d. Hipopotasemia

____ 8. Un paciente de 54 años, con antecedentes de enfermedad renal crónica, presenta disminucion de volumen
urinario y elevación de azoados, laboratorio urea 100, creatinina 4; gasometría: pH 7,10 HCO3 10, PCO2
30, ¿Cuál es su diagnóstico?

a. acidocis respiratoria
b. acidocis metabólica
c. alcalosis respiratoria
d. alcalosis metabolica

2
Name: ________________________ ID: A

____ 9. Paciente mujer de 35 años de edad presenta dolor lumbar y es positivio a la puño percusión, ademas,
presenta fiebre de 39°C acompañado de vomitos en varias ocasiones, nausea,escalofrio, disuria,
polaquiuria, dolor suprapúbico y tenesmo vesical, ademas al realizar examenes de laboratorio de un
EMO + gram tenemos los siguientes datos: Proteínas 100mg/dl; Cetonas: ++; Hemoglobina+++;
Células: 8-10/campo; Piocitos: campo lleno/campo; Hematíes: 10-12/campo; Bacterias: +++; Cilíndros
leucocitarios: 5-6/campo; Moco: ++; Gram de Gota fresca: Bacilos gram negativos + cocos gram
positivos +. Urocultivo: Escherichia coli: 100.0000 UFC/ml. Ecografía Renal con hipertrofia de
piramides.
Elija la respuesta correcta:

a. Pielonefritis
b. Cistitis
c. Prostatitis
d. Uretritis

____ 10.
Paciente de 50 años, trabajador de una institución pública, que acude a control de rutina. En el
examen fisico presenta tensión arterial alta: de 156/95 mmHg. 83 Kg de peso y 1,72 de altura
(IMC 28). Antecedentes personales: fumador de 25 paquetes/año y consumo ocasional de
alcohol (<40 gr etanol/día). Analíticas previas con cifras de colesterol de 186 mg/dl, LDL112
mg/dl, HDL 56 mg/dl, triglicéridos 143 mg/dl, glucemia 79 mg/dl, creatinina 0.8, ¿Cùal es el
estadío de hipertensiòn arterial?

a. Normal alta
b. HTA Grado 1
c. HTA Grado 2
d. HTA Grado 3

____ 11. Paciente masculino de 70 años de edad acude al centro médico por presentar, nauseas que llegan a
vómito, sudoración, palidez y dolor agudo que se inicia a nivel de la fosa lumbar irradiado a
testiculos. Al examen físico presenta puño percusion positivo y puntos renoureterales positivos.
En el uroanálisis presenta microhematuria, oxolatos > 50 mg/día, Gram: no bacterias, pH
urinario de 6
¿Cuál de las siguientes pruebas de imagen es el más específico para confirmar el diagnóstico de
litiasis renoureteral?

a. Tomografia Computarizada c. Ecografía Abdominal


Helicoidal sin contraste
b. Radiografía simple de abdomen d. Urografía intravenosa

3
Name: ________________________ ID: A

____ 12. Paciente masculino de 56 años de edad se acerca a consulta por presentar dolor en región
lumbar, disminusion de volumen urinario, fatiga, náuseas y vómitos que han durado ya varios
días, como antecedentes presenta hipertensión arterial desde hace varios años controlada. Al
examen físico presenta puntos ureterales bajos positivos. Se le envía a realizar exámenes de
laboratorio donde se ve alterada la creatinina con un valor de 2.5. Al examen imagenológico
se puede observar un cálculo bilateral de 8,3mm en los uréteres. ¿Qué tipo de insuficiencia
renal es?

a. Insuficiencia Renal Aguda Prerrenal


b. Insuficiencia Renal Aguda Intrínseca
c. Insuficiencia Renal Aguda Postrenal
d. Insuficiencia renal crónica

____ 13. Paciente masculino 78 años pesa 85kg, TA: 120/70, con antecedentes de HTA de varios años de evolucion,
presenta los siguientes datos de laboraorio Na: 145 K: 5, urea 200, Creatinina: 6 mg/dl un aclaramiento
de creatinina CKD-EPI de 12 ml/min ¿cual es el diagnostico del paciente?

a. Enfermedad renal crónica estadio 2


b. Enfermedad renal crónica estadio 5
c. Enfermedad renal crónica estadio 3
d. Enfermedad renal crónica estadio 1

____ 14. Paciente masculino de 48 años de edad con un peso de 70 Kg, que se le acaba de realizar un
trasplante renal hace 1 semana, presenta una temperatura corporal de 39.5 ºC, una tensión arterial
de 160/100, al examen físico presenta sensibilidad y edema sobre el riñón trasplantado. Su
creatinina es de 1.7. ¿Cuál es el diagnostico? Señale la respuesta correcta.

a. Rechazo Hiperagudo
b. Rechazo Agudo Acelerado
c. Rechazo Agudo
d. Rechazo Crónico

Multiple Response
Identify one or more choices that best complete the statement or answer the question.

____ 15. Paciente con diagnóstico de diabetes mellitus, nefropatia diabetica, hipertension arterial, enfermedad renal
cronica estadio 4, al laboratorio urea 140, creatinina 2,5, aclaramiento de creatinina ckd-epi de 18 ml/
minuto, potasio de 5,5, sodio 140, ¿cual es el acceso vascular permanente mas adecuado para planificar
hemodialisis?
a. cateter yugular c. cateter femoral
b. cateter subclavio d. fistula arteriovenosa

4
ID: A

REACTIVOS DE MEDICINA INTERNA: NEFROLOGÍA


Answer Section

MULTIPLE CHOICE

1. ANS: B
A.- Es incorrecto porque la Maniobra de mcburney es para valorar apendice

B.- Es correcta la Maniobra de guyon porque sirve para valorar riñon

C.- Es incorrecta Maniobra Murphy porque sirve para valorar colelitiasis

D.- Es incorrecta Maniobra de Blumberg porque sirve para valorar irritcion peritoneal

PTS: 1 DIF: Media


REF: GUARDERAS, C. SEMIOTECNIA INTEGRADA GENERAL Y ESPECIAL. QUITO:
Guarderas, Peñafiel, Arias, Dávalos.(1995) Pg.473 OBJ: Conocer las maniobras para examinar
el riñon
TOP: Nefrología KEY: Riñon NOT: Dr. Raúl Inca Andino
2. ANS: B
A.- Es incorrecta debido a que el rx simple de abdomen, solo evalua el tamaño del riñon, y posibles
calculos en los riñones o ureteres.
B.- Es correcta. debido a que el eco doppler renal nos permite visualizar el flujo sanguineo de las arterias
a color, permitiendonos dar un posible diagnostico de estenosis de la arteria renal o estrechamiento de la
misma.
C.- Es incorrecta, debido a que la ecografia renal solo nos permite visualizar el tamaño del riñon, posibles
litiasis, quistes corticales y diferenciacion cortico-,medular
D.- Es incorrecta, debido a que la Gammagrafia renal nos sirve para medir el funcionamiento de los
riñones.

PTS: 1 DIF: Alta


REF: Pedrosa. Diagnostico por imagen . España : S.A. MCGRAW-HILL. (2015)
OBJ: Identificar que examen de imagen TOP: Nefrología KEY: Ecografia Renal
NOT: Dr. Raúl Inca Andino
3. ANS: A
A.- Es correcto, porque el cuadro clínico es para síndrome nefrítico agudo
B.- Es incorrecto, porque el cuadro clínico es diferente en tumor renal
C.- Es incorrecto, el paciente con sindrome nefrótico presenta proteinuria mayor de 3.5g/24h,
hipoalbuminemia, edema, hiperlipidemia.
D.- Es incorrecto, el paciente no tiene antecedentes de litiasis renal

PTS: 1 DIF: Alta


REF: Harrison principios de Medicina Interna 18 edicción - Nefropatía crónica año 2012 CAPÍTULO
45 cuadro 45-1 OBJ: Conocer el cuadro clinico del Sindrome Nefritico
TOP: Nefrología KEY: sindrome NOT: Dr. Raúl Inca Andino

1
ID: A

4. ANS: A
A.- Es correcta ya que el sindrome nefrótico se caracteriza por proteinuria mayor de 3.5 gr,
hipoalbuminemia, hiperlipidemia, hipercoagulabilidad.
B.- Es incorrecta ya que el sindrome nefritico se caracterizapor la tríada de hipertensión arterial, edema y
hematuria
C.- Es incorrecta ya que las infecciones urinarias se caracterisa por disuria, polaquiuria, tenesmo vesical,
alza termica.
D.- Es incorrecta porque la insuficiencia renal aguda se caracteriza por el deterioro rápido de la función
renal en horas o días, con disminución de la tasa de filtración e incapacidad para regular líquidos y
electrolitos.

PTS: 1 DIF: Alta


REF: Farreras y Rozman.Medicina interna. Barcelona : Elsevier, 2016. Vol. I. pag 829
OBJ: Conocer el cuadro clinico del Sindrome Nefrotico TOP: Nefrología
KEY: Sindrome NOT: Dr. Raúl Inca Andino
5. ANS: D

Feedback
A.- NO ES CORRECTA: En el Estadio I: existe aumento del Filtrado Glomerular y la presión de
filtración, y los riñones aumentan de tamaño
B.- NO ES CORRECTA: En el Estadio II: existe excreción renal de albumina permanece entre los valores
normales y estructuralmente existe engrosamiento de la menbrana basal glomerular. Se desarrolla entre
2-3 años siguientes del diagnostico de Diabetes Mellitus.
C.- NO ES CORRECTA: En el Estadio III: existe eliminación constante de albumina por orina en
cantidad entre 30 mg/día y 300 mg/dia. Existe un incremento leve de la TA con disminución Filtrado
Glomerular. Estructuralmente los glomérulos presentan esclerosis focal y segmentaria, aumento del
grosor de la Membrana Basal con formación de nódulos.
D.- ES CORRECTA: En el Estadio IV: se inicia entre los 10 - 15 años de evolución de la Diabetes
Mellitus. Se caracteriza con albuminuria >300 mg/dl; descenso progresivo del filtrado glomerular y
aparición de HTA.

PTS: 1 DIF: Alta


REF: Harrison principios de Medicina Interna 18 edicción - Nefropatía crónica año 2012 CAPÍTULO
149 Páginas 960 - 961 OBJ: Identifcar los estadios de la Nefropatia Diabetica
TOP: Nefrología KEY: Diabetes NOT: Dr. Raúl Inca Andino
6. ANS: A
A.- Es correcto ya que en la hiperpotasemia el potasio esta elevado y no se encuentra en los valores
normales que son 3,5 a 5,3 mEq/L , la hiperpotasemia produce trastornos de la repolarización que se
traducen en la instalación de una T prominente, prolongaciòn del intervalo PR, simétrica, de base
estrecha, visible en las derivaciones precordiales(1).
B.- Es incorrecta ya que los valores normales del magnesio es 1,7 a 2,2 mg/dl (1)
C.- Es incorrecta porque los valores de calcio son 8,5 a 10.2 mg/dl y En el ECG destacan T prominente
prolongacion del intervalo PR perdida de la onda P y ensanchamiento del compleho QRS acompañado de
arritmias ventriculares (1)
D.- Es incorrecta ya una de las principales causas de hipopotasemia es trastornos gastrointestinales como
diarrea , el valor normal de potasio en los adultos es de 3,5 a 5,3 mEq/L y en este caso estan disminuido
y las alteraciones en el ECG son descenso del segmento ST con aplanamiento de la onda T y aumento de
la amplitud de la onda U (1)

PTS: 1 DIF: Alta


REF: Harrison. Principios de Medicina Interna, (2012) 18edicion Capítulo 15:
OBJ: Conocer las alteraciones electrocardiograficas de Hiperpotasemia
TOP: Nefrología KEY: Potasio NOT: Dr. Raúl Inca Andino

2
ID: A

7. ANS: D
A.- Es incorrecto porque la hiperpotasemia produce trastornos de la repolarización que se traducen en la
instalación de una T grande, simétrica, de base estrecha, visible en las derivaciones precordiales(1).
B.- Es incorrecta ya que los valores normalesç del magnesio es 1,7 a 2,2 mg/dl (1)
C.- Es incorrecta porque los valores de calcio son 8,5 a 10.2 mg/dl y en el electrocardiograma se
observan el segmento ST acortado o ausente y la duración del intervalo QT está disminuida(1)
D.- Es correcta ya que la hipopotasemia es producido por trastornos gastrointestinales como diarrea , el
valor normal de potasio en los adultos es de 3,5 a 5,3 mEq/L y en este caso el (K+) està disminuido y las
alteraciones en el ECG son descenso del segmento ST con aplanamiento de la onda T y aumento de la
amplitud de la onda U (1)

PTS: 1 DIF: Alta


REF: Harrison. Principios de Medicina Interna, (2012) 18 edicion | .Capítulo 15:
OBJ: Conocer las alteraciones electrocardiograficas de hipopotasemia
TOP: Nefrología KEY: Potasio NOT: Dr. Raúl Inca Andino
8. ANS: B
A.- Es incorrecta porque la acidosis respiratoria se caracteriza por pH disminuido y PCO2 aumentado
B.- Es correcta porque en la acidosis metabolica se caracteriza por pH disminuido y bicarbonato
disminuido.
C.- Es incorrecta porque la alcalosis respiratoria el pH aumentado, y PCO2 disminuido
D.- Es incorrecta porque en una alcalosis metabolica el pH aumentado, bicarbonato aumentado

PTS: 1 DIF: Alta


REF: Harrison Manual de Medicina 18 edicción-Fisiología respiratoria y estudios para el diagnóstico
(2012). CAPÍTULO 137 Pagina 901.
OBJ: Conocer los valores de la gasometría en acidosis metabólica
TOP: Nefrología KEY: Bicarbonato NOT: Dr. Raúl Inca Andino
9. ANS: A
A.- Es correcta porque la pielonefritis suele comenzar de forma súbita con fiebre elevada,
escalofríos y afección del estado general. Dolor en la fosa lumbar, a veces con náuseas y vómitos.
El dolor puede irradiar al flanco, a la fosa iliaca del mismo lado o al epigastrio. La percusión de
la fosa lumbar es positivo, al laboratorio EMO con bacteriuria, piuria, leucocituria.
B.- Es incorrecta porque la cistitis se caracteriza por la aparicion de disuria, polaquiuria y miccion
urgente (sndrome miccional irritativo o sindrome cistico)
C.- Es incorrecta porque las mujeres no tienen prostata.
D.- Es incorrecta porque para el disgnostico de uretritis es una paciente joven con antecedentes
de cambio reciente de pareja sexual o esta sufriendo uretritis.

PTS: 1 DIF: Alta


REF: HARRISON, R. Principios de medicina Interna XIX Edición . Barcelona : Mac-Graw Hill
Interamericana editores. (2016) OBJ: Identificar el cuadro clinico de pielonefritis
TOP: Nefrología KEY: Vias urinarias NOT: Dr. Raúl Inca Andino

3
ID: A

10. ANS: B
a.- Es incorrecta porque la presión arterial normal alta se considera entre Presión arterial sistólica:
120-129mmHg y Presion arterial diastólica: 85-89mmHg.
b.- Es correcta porque la HTA Grado 1 se considera entre Presión arterial sistólica: 140-159mmHg y
Presion arterial diastólica: 90-99mmHg.
c.- Es incorrecta porque la HTA Grado 2 se considera entre Presión arterial sistólica: 160-179mmHg y
Presion arterial diastólica: 100-109mmHg.
d.- Es incorrecta porque la HTA Grado 3 se considera entre Presión arterial sistólica: >180mmHg y
Presion arterial diastólica: >110mmHg

PTS: 1 DIF: Alta


REF: Harrison Principios de Medicina Interna (2016) 19a Ed. Vol. 2, pag 1616
OBJ: Conocer los estadias de la Hipertensión Arterial TOP: Nefrología
KEY: Presión Arterial NOT: Dr. Raúl Inca Andino
11. ANS: A
A.- Es correcta ya que la Tomografia Computarizasa helicoidal sin contraste se considera la
mejor prueba radiológica para el diagnóstico de litiasis urinaria, ya que detecta la mayoría de las
litiasis y según su densidad, su localización y su apariencia, puede sugerir la composición del
cálculo. Además, detecta obstrucciones de la vía urinaria y es capaz de definir otros diagnósticos
diferenciales.
B.- Es incorrecta ya que la radiografía simple de abdomen sólo es útil en casos de litiasis
radiopaca, sin que se visualicen las litiasis úricas y de xantinas.
C.- Es incorrecta ya que con la ecografía abdominal es difícil de ver las litiasis pequeñas (<5mm)
o localizadas en el uréter medio.
D.- Es incorrecta ya que la urografía intravenosa ha sido sustituida por la TC helicoidal por la
mayor seguridad que aporta y por su coste/efectividad.

PTS: 1 DIF: Alta


REF: Harrison. Medicina Interna (18 ed., Vol. II). México D.F.: McGraw-Hill. (2012) Pag 2382
OBJ: Conocer los examenes de imagen para Litiasis Renal TOP: Nefrología
KEY: Imagenologia Renal NOT: Dr. Raúl Inca Andino
12. ANS: C
a) Es inconrrecta por que para ser una Insuficiencia Renal Aguda prerrenal la alteracion deberia ser
causada por un decenso de la perfución renal
b) Es incorrecta por que para ser una Insuficiencia Renal Aguda intrinseca debe haber un daño o
alteracion en las estrucuras renales propiamente dichas.
c) Es correcta por que en la Insuficiencia Renal Aguda Postrenal puede presenta calculo bilateral en los
uréteres causando una obstrucción bilateral del flujo urinario.
d) Es incorrecta por que la insuficiencia renal crónica se da por deterioro de la funcion renal en varios
meses.

PTS: 1 DIF: Alta


REF: Harrison, Principios de Medicina Interna. España: McGraw-Hill Interamericana. (2012) (págs.
268-271) OBJ: Identificar el cuadro clinico de la Insuficiencia Renal Aguda
TOP: Nefrología KEY: Riñon NOT: Dr. Raúl Inca Andino

4
ID: A

13. ANS: B
a) Es incorrecta ya que la enfermedad renal crónica estadio 2 tiene un aclaramento de creatinina que va
desde 60 a 89 ml/min
b) Es correcta ya que la enfermedad renal crónica estadio 5 tiene un aclaramento de creatinina < 15
ml/min
c) Es incorrecta ya que la enfermedad renal crónica estadio 3 tiene un aclaramento de creatinina que va
desde 30 a 59 ml/min
d) Es incorrecta ya que la enfermedad renal crónica estadio 1 tiene un aclaramento de creatinina que va
desde 90 a 100 ml/min

PTS: 1 DIF: Alta


REF: Harrison principio de medicina inerna, edición español, nuevo leon, mexico,, McGrawn Hill. |
,(2012) pagina 895 OBJ: Identificar los estadios de la Enfermedad Renal Crónica
TOP: Nefrología KEY: Riñon, Creatinina NOT: Dr. Raúl Inca Andino
14. ANS: B
A.- Es incorrecta ya que el rechazo hiperagudo se presenta inmediatamente después de la
revascularización del injerto. Es causado por la presencia de anticuerpos preformados en el
receptor y se caracteriza por la trombosis y pérdida irreversible del injerto. (2)
B.- Es correcta porque un rechazo agudo acelerado ocurre en la primera semana,
generalmente en el segundo a cuarto día, y se manifiesta por fiebre, hipertensión arterial y
edema del riñón trasplantado. Además que en la mayoría de los casos presenta una elevación
de la creatinina. (2)
C.- Es incorrecta ya que el rechazo agudo se produce durante el primer año. Muchas veces es
subclínico, pero en el 65% de los casos hay manifestaciones clínicas como es el caso de una
insuficiencia renal aguda, proteinuria, hipertensión arterial, fiebre, dolor, oliguria, anuria, etc.
(2)
D.- Es incorrecta porque un rechazo crónico se caracterizada por esclerosis glomerular,
atrofia tubular y fibrosis intersticial. Se presenta en forma lenta después de meses o años del
trasplante llevando a la pérdida del mismo. (2)

PTS: 1 DIF: Alta


REF: 1. Rozman, Farreras y. Medicina interna. Barcelona : Elsevier, 2016. Vol. I.. Sección VI pag:
685-691
OBJ: Identificarlos tipos de rechazos de trasplante Renal TOP: Nefrología
KEY: Riñon, Trasplante NOT: Dr. Raúl Inca Andino

MULTIPLE RESPONSE

15. ANS: D
a.- Es incorrecta ya que el cateter yugular para hemodoalisis pernamece por tres semanas
b.- Es incorrecta ya que el cateter subclavio para hemodialisis permace por 15 dias
c.- Es incorrecta ya que el cateter femoral para hemodialisis permanece por 7 dias
d.- Es correcta ya que la fistula arteriovenosa para hemodoalisis es de forma permanente

PTS: 1 DIF: Alta


REF: Harrison principios de Medicina Interna 18 edicción - Nefropatía crónica año 2012 CAPÍTULO
281
OBJ: Conocer el acceso vascular para hemodialisis TOP: Nefrología
KEY: Accesos vasculares NOT: Dr. Raúl Inca Andino

5
REACTIVOS DE MEDICINA INTERNA: NEFROLOGÍA [Answer Strip] ID: A

D
_____ 5. A
_____ 9. C 12.
_____

B
_____ 1.

A
_____ 6.

B 13.
_____
B
_____ 2.

D
_____ 7.
B 10.
_____

B 14.
_____

A
_____ 3. B
_____ 8.

A 11.
_____

A
_____ 4.

D 15.
_____
GINECOLOGIA Y OBSTETRICIA
DIAGNOSTICO Y TRATAMIENTO DE LA ANEMIA EN EL EMBARAZO

MODIFIED TRUE/FALSE

1. Según la causa de anemia: la anemia absoluta se defne como: Es una verdadera disminución en el
contaje de eritrocitos y tiene importancia perinatal. Involucra un aumento de la destrucción del
eritrocito, disminución del volumen corpuscular o disminución de la producción de eritrocitos.

ANS: T PTS: 1 DIF: Alta


REF: Diagnóstico y tratamiento de la anemia en el embarazo. Guía de Práctica Clínica
TOP: Anemia KEY: Anemia

MULTIPLE CHOICE

1. De acuerdo a la información publicada en la Encuesta Nacional en Salud y Nutrición


(ENSANUT-ECU 2011-2013), la prevalencia de anemia en mujeres en edad reproductiva es
a. 15 % c. 30 %
b. 20 % d. 40 %
ANS: A
De acuerdo a la información publicada en la Encuesta Nacional en Salud y Nutrición
(ENSANUT-ECU 2011-2013), la prevalencia de anemia en mujeres en edad reproductiva es
del 15% a escala nacional.

PTS: 1 DIF: Alta


REF: Diagnóstico y tratamiento de la anemia en el embarazo. Guía de Práctica Clínica 2014
TOP: Anemía KEY: Anemía

2. Los principales factores de riesgo para desarrollar anemia por deficiencia de hierro son:
a. bajo aporte de hierro, síndromes de mala c. pérdidas sanguíneas crónicas a diferentes
absorción niveles
b. períodos de vida en que las necesidades d. todo lo anterior
de hierro son
especialmente altas
ANS: D
Los principales factores de riesgo para desarrollar anemia por deficiencia
de hierro son: bajo aporte de hierro, pérdidas sanguíneas crónicas a diferentes niveles,
síndromes de mala absorción y, períodos de vida en que las necesidades de hierro son
especialmente altas.

PTS: 1 DIF: Alta


REF: Diagnóstico y tratamiento de la anemia en el embarazo. Guía de Práctica Clínica
TOP: Anemia KEY: Anemia

1
3. Según la severidad clínica, en la anemia severa la reduccón de hemoglobina es de:
a. < 7,0 g/dl c. 10,1 - 10,9 g/dl
b. 7,1- 10,0 g/dl
ANS: A
Clasificación de la anemia según la severidad clínica
Clasificación por severidad Descripción del problema por parámetros de
reducción de hemoglobina
Anemia severa < 7,0 g/dL
Anemia moderada 7,1 –10,0 g/dL
Anemia leve 10,1- 10,9 g/dL

PTS: 1 DIF: Alta


REF: Diagnóstico y tratamiento de la anemia en el embarazo. Guía de Práctica Clínica
TOP: Anemia KEY: Anemia

4. Según la severidad clínica, en la anemia moderada la reduccón de hemoglobina es de:


a. < 7,0 g/dl c. 10,1 - 10,9 g/dl
b. 7,1 - 10,0 g/dl
ANS: B
Clasificación de la anemia según la severidad clínica
Clasificación por severidad Descripción del problema por parámetros de
reducción de hemoglobina
Anemia severa < 7,0 g/dL
Anemia moderada 7,1 –10,0 g/dL
Anemia leve 10,1- 10,9 g/dL

PTS: 1 DIF: Alta


REF: Diagnóstico y tratamiento de la anemia en el embarazo. Guía de Práctica Clínica
TOP: Anemia KEY: Anemia

5. Las mujeres deben ser asesoradas sobre cómo tomar suplementos de hierro por vía oral
correctamente, señale el lieral incorrecto:
a. estómago vacío, una hora antes de las c. Otros medicamentos o antiácidos si
comidas deben tomarse a la misma hora
b. con una fuente de vitamina C (ácido
ascórbico)
ANS: C
Las mujeres deben ser asesoradas sobre cómo tomar suplementos de hierro por vía oral
correctamente. Este debería ser con estómago vacío, una hora antes de las comidas, con
una fuente de vitamina C (ácido ascórbico), tal como zumo de naranja para maximizar la
absorción. Otros medicamentos o antiácidos no deben tomarse a la misma hora

PTS: 1

2
Edwin Choca

1. Un hombre de 45 años, previamente sano, es diagnosticado de neumonía del lóbulo


inferior derecho. A la exploración física el paciente está consciente y orientado, TA
120/80 en supino que sube a 130/90 tras la bipedestación. Buen estado de hidratación.
Diuresis de 24horas: 1 litro, con balance negativo. Presenta la siguiente analítica
sanguínea: Hemoglobina 12 g/dl. Creatinina 0,6mg/dl. Calcio 8,5 mg/dl. Acido úrico
2,2 mg/dl. Sodio 127mEq/l. Potasio 3,4 mEq/l. Proteínas totales 5,8 g/dl. La actitud a
seguir ante la hiponatremia de este paciente es:

a. Restricción hídrica. c. Tratamiento nutricional intensivo.

b. Tratamiento con furosemida. d. Solución salina hipertónica


intravenosa.

ANS: A PTS: 1

2. Un paciente de 58 años, hipertenso, consulta por hipoglucemia detectada casualmente. Al


examen Fisico un aspecto facial tosco y aumento de partes acras. Ante la sospecha de
acromegalia ¿Qué prueba funcional elegiría para su diagnóstico?

a. Determinación basal de la GH c. Hipoglicemia insulínica.

b. Sobrecarga de glucosa y d. Determinación de somatomedina


determinación de IGF-I. C (IGF-I)

ANS: B PTS: 1

3. Son características del hipotiroidismo congénito, EXCEPTO:

a. La facies clásica se debe a la c. Valores bajos de T4 y altos de TSH.


acumulación de mixedema en el
tejido celular subcutáneo y lengua.
b. El hipotiroidismo congénito tratado d. Se detecta en el tamizaje neonatal
tardíamente puede permitir buena actual en el país
evolución.
ANS: B PTS: 1

1
4. En unos análisis de rutina de una mujer de 59 años, fumadora de 20 cigarrillos/día desde
hace 25 años, se detecta una hipercalcemia de 11,3 mg/dL con un fósforo de 3,4 mg/dl. NO
resultaría eficiente de entrada:

a. Determinar niveles séricos de PTH. c. Determinación de hidroxiprolinuria

b. Determinar niveles séricos de vitamina D d. Una radiografía simple de tórax

ANS: C PTS: 1

5. En un paciente diagnosticado de diabetes insípida central idiopática esperamos encontrar (antes de iniciar
tratamiento alguno):

a. Osmolalidad plasmática elevada, natremia c. Osmolalidad plasmática elevada, natremia


alta, osmolalidad urinaria aumentada. alta, osmolalidad urinaria disminuida.

b. Osmolalidad plasmática baja, natremia alta, d. Osmolalidad plasmática baja, natremia baja,
osmolalidad urinaria aumentada. osmolalidad urinaria aumentada

ANS: C PTS: 1

6. Un paciente presenta, en una analítica realizada en ayunas, una glucemia basal de 120 mg/dl. Utilizando los criterios
diagnósticos de diabetes mellitus de la ADA, usted le podría diagnosticar de:

a. La cifra es normal. No se debe realizar ninguna c. La cifra indica alteraci de la glucosa en


medida. ayunas. Si la glucemia, 2 horas despu駸 de 75
g de glucosa, es inferior a 140 mg/dl, se
diagnosticar・ de intolerancia hidrocarbonada.

b. Si la cifra en ayunas esta entre 100 y 126 d. Si se realiza una sobrecarga de glucosa y la
mg/dl, se etiquetara como alteraci de la glucemia a las 2 horas esta entre 140 y 200
glucosa en ayunas. mg/dl, se le etiquetara de diabetes.

ANS: B PTS: 1

7.
Paciente de 42 años con el antecedente de ser fumador 12 unidades diarios, acude por presentar poliuria,
polidipsia y pérdida de peso. Al examen físico febril de 39° C, escalofrió, diaforético, dolor generalizado sobre
todo lumbar. Laboratorio: Glucosa basal 280 mg/dl, EMO reporta glucosa +, bacterias +++, proteinuria + y en
el Estándar de Tórax refiere aumentado la trama parahiliar. ¿Cuál es el diagnóstico probable?

a. DM tipo 1 + IVU. c. DM tipo 1 + NAC.

b. DM tipo 2 + NAC. d. DM tipo 2 + IVU.

ANS: D PTS: 1

2
8.
Paciente de 58 años, teniendo como antecedente de haber sido diagnosticado de DM Tipo 2 y hace 20 años
tratado con insulina NPH. EF. TA: 70/40 mmHg, FC: 120 X´, FR: 28 X´ Tipo Kussmaul. Deshidratado grave,
oliguria. PH arterial: 7.2, HCO3: 15, Na+ 150, Cl 110, glucosa basal: 900 mg/dl. ¿Cuál es la complicación aguda
más probable?

a. Cetoacidosis diabética. c. Acidosis metabólica.

b. Estados hiperosmolares hiperglucémicos. d. Síndrome de secreción inadecuada de la


hormona antidiurética.

ANS: B PTS: 1

9.
Para la detención temprana de la Diabetes Mellitus tipo 2, la ADA recomienda realizar exámenes de glicemia y
HbA 1 a pacientes con. Señale lo falso:

a. Pacientes > de 35 años y cada 2 años. c. Pacientes con hipertensión.

b. Paciente con BMI > o = 25/m2. d. Pacientes con HDL < 35 mg/100 ml.

ANS: A PTS: 1

10. El tratamiento para la DM tipo 2 está contraindicado de utilizar en. Excepto.

a. Embarazo c. Hepatopatías.

b. Insuficiencia renal. d. Hipertensión

ANS: D PTS: 1

11. La Diabetes Mellitus tipo 2, se presenta mayoritariamente en:


a. Paciente con sobrepeso u obesidad c. Paciente atleta de alto rendimiento
con antecedentes familiares de DM2
b. Paciente con peso normal, con d. Paciente vegetariano, con peso normal
alimentación rica en proteínas de
origen animal.

ANS: A PTS: 1

12. En los pacientes con Diabetes Mellitus tipo 2, una vez diagnosticados, se les debe
solicitar los siguientes exámenes de despistaje de complicaciones, excepto:
a. Microalbuminuria c. Perfil lipídico

b. Fondo de ojo d. Tiempos de coagulación: TP, TTP e


INR

ANS: D PTS: 1

3
13. En el tratamiento del hipotiroidismo primario, se debe emplear la dosis de
levotiroxina necesaria para llevar a un nivel de TSH, de:
a. Rango normal (0,4 – 4 mUI/mL) c. Suprimida (< 0,1 mUI/mL)

b. Se usa dosis estándar y no depende de los d. Menor a 10 mUI/mL


niveles de

ANS: A PTS: 1

14. Los siguientes perfiles tiroideos se correlacionan con las patologías mencionadas, marque la
incorrecta

a. TSH elevada, T4L normal: c. TSH suprimida, T4L elevada:


hipotiroidismo subclínico hipertiroidismo

b. TSH elevada, T4L elevada: d. TSH suprimida, T4L elevada:


hipotiroidismo primario hipotiroidismo

ANS: D PTS: 1

15. El seguimiento de pacientes diabéticos en tratamiento solo con antidiabéticos orales, no


insulinoterapia, se realiza con:

a. Glucosa venosa mensual c. Glucemia capilar diaria

b. Hemoglobina glucosilada cada 2 – 6 d. Elemental y microscópico de orina en


meses busca de glucosuria mensual

ANS: B PTS: 1

4
UNIVERSIDAD NACIONAL DE CHIMBORAZO
ÉTICA MÉDICA :
Dr. Guillermo Gualpa Jaramillo
Date: 2015/11/04

Apellidos y nombres......................................................................................................... ID: 5TO

etica1

Verdadero/Falso
seleccione verdadero o falso

____ 1. las reflexiones del deber ser del individuo y el cómo sus acciones se orientan hacia un fin se denomina
deontología

F
____ 2. la sub división de la ética que aborda aquellas acciones que deben (y las que no deben) ser realizadas,
sin tener en cuenta si las consecuencias de su concreción pueden ser positivas o negativas se conoce
como Metaetica

____ 3. la que se ocupa de definir los rasgos o características de las acciones o actos de los individuos que le
permiten determinar el bien moral es la ética normativa

F
____ 4. Título IV Regimen Académico UNACH

Art. 76.- El alumno tendrá acceso a las clases ordinarias en un curso, sin estar matriculado en el,
para lo cual secretaria proporcionara a todos los Docentes, las listas de los alumnos con sus
respectivas números de cedula de identidad.

F
____ 5. Título IV régimen académico UNACH

El Docente NO podrá requerir la presentación del certificado de acreditación de su calidad de


estudiante universitario.

F
____ 6. La ética de los mínimos corresponde a la autonomía y beneficencia

1
Name: ________________________ ID: 5S

Multiple opción
seleccione la verdadera

____ 7. EL JURAMENTO HIPOCRATICO:

a. es un artículo de la constitución política c. es un código de ética que rige la moral


del estado del médico

b. es una norma internacional que rige la d. fue reformado por el código de Ginebra
praxis médica en 1943

____ 8. LA INFRACCIÓN AL DEBER OBJETIVO DE CUIDADO ES

a. La concurrencia de tres factores: ilegal, c. La concurrencia de tres factores,


innecesario, peligroso impericia, negligencia inexperiencia

b. La voluntaria omisión de diligencia en d. se cataloga com o homicidio culposo


calcular las consecuencias posibles y
previsibles del propio hecho

____ 9. la declaración de Ginebra: elija la verdadera

a. fue promulgada en Nuremberg en 1941 c. contiene los mismos principios éticos y


morales del juramento hipocrático

b. determina los principios del deber d. Fue aprobada en la reunión de la Haya en


objetivo de cuidado el 2006

____ 10. los valores centrales de la medicina son:

a. La deontología, los derechos humanos, c. el diagnostico, la ética y los derechos


la vocación de los pacientes

b. la compasión, competencia, autonomía d. deber objetivo de cuidado, justicia no


maleficencia

2
ID: A

etica1
Answer Section

TRUE/FALSE

1. ANS: F PTS: 1 DIF: baja


2. ANS: F PTS: 1 DIF: baja
3. ANS: T PTS: 1 DIF: baja
4. ANS: F PTS: 1 DIF: baja
5. ANS: F PTS: 1 DIF: baja
6. ANS: F PTS: 1 DIF: baja

MULTIPLE CHOICE

7. ANS: B PTS: 1 DIF: baja


8. ANS: B PTS: 1 DIF: baja
9. ANS: C PTS: 1 DIF: baja
10. ANS: B PTS: 1 DIF: baja

1
etica1 [Answer Strip] ID: A

B
_____ 7.

F
_____ 1.

B
_____ 8.

F
_____ 2.

T
_____ 3.

F
_____ 4. C
_____ 9.

F
_____ 5.

B 10.
_____

F
_____ 6.
ÉTICA MÉDICA - DR. WILSON NINA
RELACIÓN MÉDICO PACIENTE

MULTIPLE CHOICE

1. La causa final de toda ética es:


a. Honestidad
b. Solidaridad
c. Justicia
d. Dependencia
ANS: A PTS: 1

2. A qué características de la bioética pertenecen: biologia, derecho,sociologia,psicologia.


a. Carácter de novedad
b. Carácter interdisciplinar
c. Principio del derecho
d. Principio fundamnetal
ANS: B PTS: 1

3. La verdad puede ser:


a. Objetiva
b. Subjetiva
c. Especifica
d. Objetiva y subjetiva
ANS: D PTS: 1

4. Respecto de la relación médico paciente en los enunciados siguientes encontrará


características que identifican al tipo de relación paternalista, señale la respuesta correcta:
a. El enfermo espera: autonomía para decidir y respeto a sus derechos. mientras que del
médico se espera: competencia técnica, comprensión humana y aceptación de la
autonomía del enfermo.
b. El médico: se sentía obligado a cumplir los deberes que la propia profesión le marcaba.
El enfermo: Se sentía obligado a obedecer, confiar y expresar gratitud.
c. El paciente necesita aclarar sus valores y resolver conflictos. El médico es el consultor
y consejero que interpreta y aplica las preferencias del paciente.
d. El paciente está abierto a la revisión, hallazgo y desarrollo de sus valores. El médico es
el amigo o maestro que delibera conjuntamente.

ANS: B PTS: 1
OBJ: Identificar los elementos o enunciados esenciales que caracterizan a los diferentes tipos de
relación médico paciente, mediante el análisis y conocimientos previos afín de clasificar y
determinar la relación médico paciente ideal. TOP: Relación médico paciente

1
5. Respecto de la relación médico paciente en los enunciados siguientes encontrará
características que identifican al tipo de relación autonomista, señale la respuesta correcta:
a. El enfermo espera: autonomía para decidir y respeto a sus derechos. Del médico se
espera: Competencia técnica, comprensión humana y aceptación de la autonomía del
enfermo.
b. El médico: se sentía obligado a cumplir los deberes que la propia profesión le marcaba.
El enfermo: Se sentía obligado a obedecer, confiar y expresar gratitud.
c. El paciente necesita aclarar sus valores y resolver conflictos. El médico es el consultor
y consejero que interpreta y aplica las preferencias del paciente.
d. El paciente está abierto a la revisión, hallazgo y desarrollo de sus valores. El médico es
el amigo o maestro que delibera conjuntamente.

ANS: A PTS: 1
OBJ: Identificar los elementos o enunciados esenciales que caracterizan a los diferentes tipos de
relación médico paciente, mediante el análisis y conocimientos previos afín de clasificar y
determinar la relación médico paciente ideal. TOP: Relación Médico Paciente

6. Respecto de la relación médico paciente en los enunciados siguientes encontrará


características que identifican al tipo de relación o modelo interpretativa, señale la respuesta
correcta:
a. El enfermo espera: autonomía para decidir y respeto a sus derechos. Del médico se
espera: Competencia técnica, comprensión humana y aceptación de la autonomía del
enfermo.
b. El médico: se sentía obligado a cumplir los deberes que la propia profesión le marcaba.
El enfermo: Se sentía obligado a obedecer, confiar y expresar gratitud.
c. El paciente necesita aclarar sus valores y resolver conflictos. El médico es el consultor
y consejero que interpreta y aplica las preferencias del paciente.
d. El paciente está abierto a la revisión, hallazgo y desarrollo de sus valores. El médico es
el amigo o maestro que delibera conjuntamente.

ANS: C PTS: 1
OBJ: Identificar los elementos o enunciados esenciales que caracterizan a los diferentes tipos de
relación médico paciente, mediante el análisis y conocimientos previos afín de clasificar y
determinar la relación médico paciente ideal. TOP: Relación médico paciente

2
7. Respecto de la relación médico paciente en los enunciados siguientes encontrará
características que identifican al tipo de relación o modelo deliberativa, señale la respuesta
correcta:
a. El enfermo espera: autonomía para decidir y respeto a sus derechos. del médico se
espera: Competencia técnica, comprensión humana y aceptación de la autonomía del
enfermo.
b. El médico: se sentía obligado a cumplir los deberes que la propia profesión le marcaba.
el enfermo: Se sentía obligado a obedecer, confiar y expresar gratitud.
c. El paciente necesita aclarar sus valores y resolver conflictos. El médico es el consultor
y consejero que interpreta y aplica las preferencias del paciente.
d. El paciente está abierto a la revisión, hallazgo y desarrollo de sus valores. El médico es
el amigo o maestro que delibera conjuntamente.

ANS: D PTS: 1
OBJ: identificar los elementos o enunciados esenciales que caracterizan a los diferentes tipos de
relación médico paciente, mediante el análisis y conocimientos previos afín de clasificar y
determinar la relación médico paciente ideal. TOP: Relación médico paciente

8. ¿Por qué es importante la confidencialidad en el ámbito médico?


a. Los pacientes no deben revelar información íntima a los médicos para ser
diagnosticados y tratados adecuadamente.

b. Los pacientes deben confiar que sus médicos van a revelar la información a otros.

c. Revelación de información personal no puede hacer daño a los intereses de


alguien.

d. La confidencialidad médica trata de tutelar el derecho fundamental a la intimidad


personal del paciente, sobre la base del respeto a la dignidad humana, al honor y al
buen nombre del mismo.

ANS: D PTS: 1
OBJ: Analizar la importancia de la Confidencialidad Médica mediante los conocimientos previos
e identificar la respuesta correcta en relación a las diferentes situaciones que se pueden presentar.
TOP: Confidencialidad médica

3
9. ¿Qué dice la declaración de Ginebra con relación a la Confidencialidad Médica?

a. Prometo solemnemente consagrar mi vida al servicio de la Humanidad.


b. Otorgar a mis maestros el respeto y gratitud que merecen, Ejercer mi profesión a
conciencia y dignamente.
c. Guardar y respetar los secretos confiados a mí, incluso después del fallecimiento
del paciente.
d. Considerar como hermanos y hermanas a mis colegas.

e. Velar ante todo por la salud de mi paciente

ANS: C PTS: 1
OBJ: Identificar el enunciado de la declaración de Ginebra respecto de Confidencialidad Médica
mediante los conocimientos previos e identificar la respuesta correcta en relación a las diferentes
frases que se presentan. TOP: Confidencialidad médica

10. Los principios deontológicos implicados en relación con la protección de los datos
sanitarios contenidos en la historia clínica son:
a. Sobriedad, en cuanto a consignar sólo las informaciones realmente necesarias para
la atención sanitaria.

b. Transparencia en la gestión y no acceso a la historia clínica (también para el


paciente).

c. Irresponsabilidad, íntimamente ligada con la no maleficencia por las


consecuencias para el paciente de errores u olvidos en el registro o la difusión de
datos privados.

d. Protección no universal de todos los datos de todos los pacientes en todos los
centros.

ANS: A PTS: 1
OBJ: Identificar los principios deontológicos implicados en relación con la protección de los
datos sanitarios contenidos en la historia clínica mediante los conocimientos previos e identificar
la respuesta correcta en relación a las diferentes situaciones que se pueden presentar.
TOP: Confidencialidad médica

4
11. Una vez otorgado el Consentimiento Infromado, el paciente deberá comprometerse a:
a. Participar responsablemente en su atención.

b. A velar por sus derechos y valores como la vida, la libertad, la integridad, la


equidad, la dignidad, la seguridad jurídica y la salud.
c. Tomar su decisión con libertad, si es posible solicitar al médico más de una opción.
d. Actuar en congruencia con los conocimientos médicos vigentes, evitar la medicina
defensiva.

ANS: A PTS: 1
OBJ: Conocer los derechos y obligaciones tanto del médico como del paciente para poder
implementarlo en la práctica médica TOP: Consentimiento informado

12. Respecto del objetivo que se persigue con el tratamiento de cuidados paliativos al paciente que
lo requiera, señale el enunciado correcto:
a. Pretende mejorar la calidad de vida
b. Pretende mejorar la calidad de vida y prolongar la supervivencia
c. Pretende mejorar la calidad de muerte
d. Pretende mejorar la calidad de la enfermedad

ANS: A PTS: 1
OBJ: Analizar la característica que identifica a los cuidados paliativos, mediante el conocimiento
previo, afín de estar claro en la asistencia al paciente que requiera cuidados paliativos.
TOP: Cuidados paliativos

13. ¿Cuál de los siguientes enunciados corresponde al concepto de estado vegetativo?


a. Situación clínica de completa inconsciencia, acompañada de ciclos de
sueño-vigilia, con preservación completa o parcial de las funciones autonómicas
del tronco encefálico y del hipotálamo.

b. Es aquel que padece una alteración de las funciones vitales que amenaza su vida,
pero conserva posibilidades de recuperación.

c. Es la enfermedad avanzada progresiva e irreversible sin posibilidad de respuesta


al tratamiento curativo y pronóstico de vida inferior a 6 meses.

ANS: A PTS: 1

5
ÉTICA MÉDICA - DR. WILSON NINA
RELACIÓN MÉDICO PACIENTE

MULTIPLE CHOICE

1.
Dentro del consentimiento informado escrito no es necesario que conste

a. Tipo de farmacología y dosis

b. Tipo de intervención y número

c. Descripción del proceso

d. Nombres y datos del paciente

ANS: A PTS: 1

2.
De las Excepciones al Consentimiento Informado señale el literal correcto:

a. Disponer de una información suficiente.

b. Grave riesgo para la salud pública, lo que puede incluso legitimar actuaciones
sanitarias coactivas, aunque no corresponde al médico adoptarlas por su cuenta.

c. Introducir en la medicina una mentalidad más probabilística y con mayor


capacidad de hacer frente a la incertidumbre.

d. a) Encontrarse libre para decidir de acuerdo con sus propios valores.

ANS: B PTS: 1

3.
Son excepciones del consentimiento informado, excepto:

a. Incompetencia o incapacidad del enfermo.

b. Riesgo inmediato y grave para la integridad del paciente.

c. Riesgo para la salud pública

d. Aceptación autónoma de una intervención médica o la elección entre cursos


alternativos posibles.

ANS: D PTS: 1

1
4. Respecto de la relación médico paciente en los enunciados siguientes encontrará
características que identifican al tipo de relación paternalista, señale la respuesta correcta:
a. El enfermo espera: autonomía para decidir y respeto a sus derechos. mientras que del
médico se espera: competencia técnica, comprensión humana y aceptación de la
autonomía del enfermo.
b. El médico: se sentía obligado a cumplir los deberes que la propia profesión le marcaba.
El enfermo: Se sentía obligado a obedecer, confiar y expresar gratitud.
c. El paciente necesita aclarar sus valores y resolver conflictos. El médico es el consultor
y consejero que interpreta y aplica las preferencias del paciente.
d. El paciente está abierto a la revisión, hallazgo y desarrollo de sus valores. El médico es
el amigo o maestro que delibera conjuntamente.

ANS: B PTS: 1
OBJ: Identificar los elementos o enunciados esenciales que caracterizan a los diferentes tipos de
relación médico paciente, mediante el análisis y conocimientos previos afín de clasificar y
determinar la relación médico paciente ideal. TOP: Relación médico paciente

5. Respecto de la relación médico paciente en los enunciados siguientes encontrará


características que identifican al tipo de relación autonomista, señale la respuesta correcta:
a. El enfermo espera: autonomía para decidir y respeto a sus derechos. Del médico se
espera: Competencia técnica, comprensión humana y aceptación de la autonomía del
enfermo.
b. El médico: se sentía obligado a cumplir los deberes que la propia profesión le marcaba.
El enfermo: Se sentía obligado a obedecer, confiar y expresar gratitud.
c. El paciente necesita aclarar sus valores y resolver conflictos. El médico es el consultor
y consejero que interpreta y aplica las preferencias del paciente.
d. El paciente está abierto a la revisión, hallazgo y desarrollo de sus valores. El médico es
el amigo o maestro que delibera conjuntamente.

ANS: A PTS: 1
OBJ: Identificar los elementos o enunciados esenciales que caracterizan a los diferentes tipos de
relación médico paciente, mediante el análisis y conocimientos previos afín de clasificar y
determinar la relación médico paciente ideal. TOP: Relación Médico Paciente

2
6. Respecto de la relación médico paciente en los enunciados siguientes encontrará
características que identifican al tipo de relación o modelo interpretativa, señale la respuesta
correcta:
a. El enfermo espera: autonomía para decidir y respeto a sus derechos. Del médico se
espera: Competencia técnica, comprensión humana y aceptación de la autonomía del
enfermo.
b. El médico: se sentía obligado a cumplir los deberes que la propia profesión le marcaba.
El enfermo: Se sentía obligado a obedecer, confiar y expresar gratitud.
c. El paciente necesita aclarar sus valores y resolver conflictos. El médico es el consultor
y consejero que interpreta y aplica las preferencias del paciente.
d. El paciente está abierto a la revisión, hallazgo y desarrollo de sus valores. El médico es
el amigo o maestro que delibera conjuntamente.

ANS: C PTS: 1
OBJ: Identificar los elementos o enunciados esenciales que caracterizan a los diferentes tipos de
relación médico paciente, mediante el análisis y conocimientos previos afín de clasificar y
determinar la relación médico paciente ideal. TOP: Relación médico paciente

7. Respecto de la relación médico paciente en los enunciados siguientes encontrará


características que identifican al tipo de relación o modelo deliberativa, señale la respuesta
correcta:
a. El enfermo espera: autonomía para decidir y respeto a sus derechos. del médico se
espera: Competencia técnica, comprensión humana y aceptación de la autonomía del
enfermo.
b. El médico: se sentía obligado a cumplir los deberes que la propia profesión le marcaba.
el enfermo: Se sentía obligado a obedecer, confiar y expresar gratitud.
c. El paciente necesita aclarar sus valores y resolver conflictos. El médico es el consultor
y consejero que interpreta y aplica las preferencias del paciente.
d. El paciente está abierto a la revisión, hallazgo y desarrollo de sus valores. El médico es
el amigo o maestro que delibera conjuntamente.

ANS: D PTS: 1
OBJ: identificar los elementos o enunciados esenciales que caracterizan a los diferentes tipos de
relación médico paciente, mediante el análisis y conocimientos previos afín de clasificar y
determinar la relación médico paciente ideal. TOP: Relación médico paciente

3
8. ¿Por qué es importante la confidencialidad en el ámbito médico?
a. Los pacientes no deben revelar información íntima a los médicos para ser
diagnosticados y tratados adecuadamente.

b. Los pacientes deben confiar que sus médicos van a revelar la información a otros.

c. Revelación de información personal no puede hacer daño a los intereses de


alguien.

d. La confidencialidad médica trata de tutelar el derecho fundamental a la intimidad


personal del paciente, sobre la base del respeto a la dignidad humana, al honor y al
buen nombre del mismo.

ANS: D PTS: 1
OBJ: Analizar la importancia de la Confidencialidad Médica mediante los conocimientos previos
e identificar la respuesta correcta en relación a las diferentes situaciones que se pueden presentar.
TOP: Confidencialidad médica

9. ¿Qué dice la declaración de Ginebra con relación a la Confidencialidad Médica?

a. Prometo solemnemente consagrar mi vida al servicio de la Humanidad.


b. Otorgar a mis maestros el respeto y gratitud que merecen, Ejercer mi profesión a
conciencia y dignamente.
c. Guardar y respetar los secretos confiados a mí, incluso después del fallecimiento
del paciente.
d. Considerar como hermanos y hermanas a mis colegas.

e. Velar ante todo por la salud de mi paciente

ANS: C PTS: 1
OBJ: Identificar el enunciado de la declaración de Ginebra respecto de Confidencialidad Médica
mediante los conocimientos previos e identificar la respuesta correcta en relación a las diferentes
frases que se presentan. TOP: Confidencialidad médica

4
10. Los principios deontológicos implicados en relación con la protección de los datos
sanitarios contenidos en la historia clínica son:
a. Sobriedad, en cuanto a consignar sólo las informaciones realmente necesarias para
la atención sanitaria.

b. Transparencia en la gestión y no acceso a la historia clínica (también para el


paciente).

c. Irresponsabilidad, íntimamente ligada con la no maleficencia por las


consecuencias para el paciente de errores u olvidos en el registro o la difusión de
datos privados.

d. Protección no universal de todos los datos de todos los pacientes en todos los
centros.

ANS: A PTS: 1
OBJ: Identificar los principios deontológicos implicados en relación con la protección de los
datos sanitarios contenidos en la historia clínica mediante los conocimientos previos e identificar
la respuesta correcta en relación a las diferentes situaciones que se pueden presentar.
TOP: Confidencialidad médica

11. Una vez otorgado el Consentimiento Infromado, el paciente deberá comprometerse a:


a. Participar responsablemente en su atención.

b. A velar por sus derechos y valores como la vida, la libertad, la integridad, la


equidad, la dignidad, la seguridad jurídica y la salud.
c. Tomar su decisión con libertad, si es posible solicitar al médico más de una opción.
d. Actuar en congruencia con los conocimientos médicos vigentes, evitar la medicina
defensiva.

ANS: A PTS: 1
OBJ: Conocer los derechos y obligaciones tanto del médico como del paciente para poder
implementarlo en la práctica médica TOP: Consentimiento informado

12. Respecto del objetivo que se persigue con el tratamiento de cuidados paliativos al paciente que
lo requiera, señale el enunciado correcto:
a. Pretende mejorar la calidad de vida
b. Pretende mejorar la calidad de vida y prolongar la supervivencia
c. Pretende mejorar la calidad de muerte
d. Pretende mejorar la calidad de la enfermedad

ANS: A PTS: 1
OBJ: Analizar la característica que identifica a los cuidados paliativos, mediante el conocimiento
previo, afín de estar claro en la asistencia al paciente que requiera cuidados paliativos.
TOP: Cuidados paliativos

5
13. ¿Cuál de los siguientes enunciados corresponde al concepto de estado vegetativo?
a. Situación clínica de completa inconsciencia, acompañada de ciclos de
sueño-vigilia, con preservación completa o parcial de las funciones autonómicas
del tronco encefálico y del hipotálamo.

b. Es aquel que padece una alteración de las funciones vitales que amenaza su vida,
pero conserva posibilidades de recuperación.

c. Es la enfermedad avanzada progresiva e irreversible sin posibilidad de respuesta


al tratamiento curativo y pronóstico de vida inferior a 6 meses.

ANS: A PTS: 1

14. Señale el concepto de Ética.


a. Reglas o normas de conducta de carácter intrínseco desarrolladas por una persona.

b. Conjunto de costumbres, hábitos, normas, códigos de conducta reglas y


tradiciones que caracterizan a una comunidad humana.
c. Cualidades propias de cada ser humano para acondicionar el mundo de nuestras
vidas y poder vivirlas en cualquier momento.
d. Parte de la filosofía que tiene por objeto la reflexión sobre la conducta moral del
ser humano a la luz de los valores y principios.

ANS: D PTS: 1

15. Encierre en un círculo la respuesta correcta. El acto humano se define como aquel que
se hace a la luz de la razón del dominio de la voluntad propia y con conciencia y que
sólo puede ser trastocado por fuerzas ajenas a la voluntad y llevadas a cabo debido a:
a. Ignorancia, pasión, miedo y violencia.

b. Autonomía, justicia, beneficencia y no maleficenciad

c. Esperanza de vida, logro educativo y PIB en salud


d. Normas éticas jurídicas, sociales, religiosas, técnicas y personales

ANS: A PTS: 1

6
16. Señale el literal correcto con respecto a la definición de ética
a. Es una cualidad propia de cada ser humano para acondicionar el mundo de
nuestras vidas y poder vivirlas en cualquier tiempo y lugar

b. Es el conjunto de costumbres reglas y tradiciones que caracterizan a una


comunidad humana

c. Es parte de la filosofía que tiene por objeto la reflexión sobre la conducta moral
del ser humano a la luz de valores y principios y nos sugiere cómo se forma el
carácter

d. Son reglas o normas de conducta de carácter intrínseco desarrolladas por una


persona es decir son las leyes naturales que son externas a nosotros

ANS: C PTS: 1

17.
Escoja la definición correcta de confidencialidad

a. Es el deber profesional de mantener oculta la intimidad del paciente y de no


revelar los datos confidenciales de éste para fines ajenos a la propia asistencia
sanitaria

b. Es el derecho de las personas a que aquellos que hayan entrado en conocimiento


de datos íntimos suyos, no puedan revelarlos ni utilizarlos sin su autorización
expresa
c. Es el deber de las personas que conocen ciertos datos de otras de no revelarlos sin
su consentimiento o sin habilitación legal.

d. Es el ámbito en que los seres humanos gestionan libre y privadamente su mundo


de valores y todo lo que tiene que ver directa o indirectamente con ellos.

ANS: B PTS: 1

18. Señale la respuesta correcta sobre cuáles son los bienes específicos del secreto
profesional del médico
a. Hacer posible una relación terapéutica eficaz, basada en la veracidad y en la
confianza.

b. No Facilitar el registro escrito de historias clínicas fidedignas y completas.

c. Asegurar la protección de otros derechos no es necesario en el secreto medico


paciente.

d. Es necesario guardar información sobre ciertos males del paciente.

ANS: A PTS: 1

7
19. Señale la respuesta incorrecta. El secreto medico tiene algunas excepciones en
circunstancias que pueden ser justificadas como:
a. Obligación legal.

b. Cuando el paciente no tenga el consentimiento adecuado.

c. Protección de terceras personas.

d. Riesgo para la salud pública o el interés colectivo.

ANS: B PTS: 1

20.
Con relación a los límites del consentimiento informado señale el literal correcto:

a. El CI no es un derecho del paciente más si es un derecho limitado.

b. Significa que los pacientes puedan imponer al médico cualquier cosa que deseen.

c. La profesión médica y la sociedad en su conjunto pueden imponer restricciones a


la capacidad de elección de los pacientes.

d. El modelo clásico de relación médico-paciente es paternalista.

ANS: C PTS: 1

8
Name: ________________________ Class: ___________________ Date: __________ ID: A

Reactivos Ginecología y Obstetricia

Multiple Choice
Identify the choice that best completes the statement or answers the question.

____ 1. GESTANTE DE 24 SEMANAS ACUDE A LA CONSULTA PORQUE A SU HIJO DE 5 AÑOS HACE 5 DÍAS LE
DIAGNOSTICARON VARICELA. LA PACIENTE NO RECUERDA SI PADECIÓ LA ENFERMEDAD EN LA
INFANCIA, PERO SI SABE QUE NO FUE VACUNADA Y ESTÁ MUY PREOCUPADA POR LA POSIBLE
AFECCIÓN FETAL. QUE ACTUACIÓN SERIA LA CORRECTA?.

a. ADMINISTRAR VACUNA ESPECIFICA d. ADMINISTRAR ACICLOVIR A DOSIS DE


800MG VO 5 VECES AL DÍA POR 5- - 7
DÍAS.

b. SOLICITAR CUANTIFICACIÓN DE IGG, e. ADMINISTRAR GAMAIGLOBULINA


Y SI FUERA ESPECIFICO ADMINISTRAR ESPECIFICA Y TRANQUILIZAR A LA
GAMAGLOBULINA ESPECIFICA. PACIENTE INFORMÁNDOLE DE LA
AUSENCIA DE RIESGOS.

c. TRANQUILIZAR A LA PACIENTE
INFORMÁNDOLE DE LA AUSENCIA DE
RIESGOS FETALES

____ 2. PACIENTE DE 28 AÑOS EN LAS 42.3 SG Y ANTECEDENTES DE OTRO PARTO A TÉRMINO. EL


EMBARAZO HA TRANSCURRIDO NORMALMENTE. AL EXAMEN FISICO VARIEDAD DE
PRESENTACION: FETO EN OIIA, NST REACTIVO Y CERVIX FAVORABLE BISHOP 9. CUÁL DE LAS
SIGUIENTES CONDUCTAS ES LA MAS FAVORABLE?

a. ESPERAR HASTA QUE DESENCADENE d. CONDUCIR EL TRABAJO DE PARTO


TRABAJO DE PARTO..
C.- ADMINISTRAR CORTICOIDES A LA
MADRE E INDUCIR EL PARTO
PASADAS LAS 48 HORAS

b. CONTROLES CADA 48 HORAS. e. HACER CESÁREA ELECTIVA.

c. ADMINISTRAR CORTICOIDES A LA
MADRE E INDUCIR EL PARTO
PASADAS LAS 48 HORAS

1
Name: ________________________ ID: A

____ 3. PACIENTE EN PERIODO EXPULSIVO DE 38 SEMANAS DE GESTACIÓN, PRESENTACIÓN EN TERCER


PLANO DE HODGE, POSICIÓN OIIT Y APARICIÓN DE UNA BRADICARDIA FETAL DE 100 - 110 LAT X
MIN CONSTATÁNDOSE LA EXISTENCIA DE UNA ACIDOSIS RESPIRATORIA FETAL: PH DE CUERO
CABELLUDO 7.18. CUÁL SERÍA LA ACTITUD CORRECTA?

a. REALIZACIÓN DE UNA CESÁREA d. COLOCAR EL PULSOXIMETRO PARA


PROGRAMADA CONSTATAR LA SATURACIÓN DE
OXIGENO MATERNO.

b. UTILIZACIÓN DE UNA VENTOSA - e. APLICACIÓN DE OXIGENO A LA


VACUUM. MADRE PARA REALIZACIÓN DE PH EN
10 MINUTOS.

c. UTILIZACIÓN DE UN FÓRCEPS DE
SALIDA

____ 4. GESTANTE DE 8 SEMANAS QUE ACUDE A CONSULTA PARA REALIZAR ECOGRAFÍA EN LA QUE UD.
OBSERVA UNA GESTACIÓN INTRAUTERINA CON EMBRIÓN ÚNICO CON LCC DE 16MM (ACORDE
CON 8 SEMANAS) SIN LATIDO CARDIACO SEÑALE LO CORRECTO:

a. GESTACIÓN MAL DATADA. LA LCC d. DIAGNOSTICO DE FETO ACARDIO


NOS INDICA EL TIEMPO DE
GESTACIÓN EN EL PRIMER
TRIMESTRE. ÚNICAMENTE
CORREGIMOS LA FECHA PROBABLE
DE PARTO.
b. AMENAZA DE ABORTO, RECOMIENDA e. DIAGNOSTICO DE ABORTO DIFERIDO
REPOSO Y GESTÁGENOS. = LEGRADO

c. DETERMINACIÓN DE BHCG
PLASMÁTICA CADA 48 H.

____ 5.
GESTANTE DE 38 SEMANAS QUE COMIENZA BRUSCAMENTE CON METRORRAGIA OBSCURA Y EN
POCA CANTIDAD, DOLOR ABDOMINAL INTENSO, DIFUSO, MAL ESTADO GENERAL Y SUFRIMIENTO
FETAL. CUÁL ES LA COMPLICACIÓN MAS PROBABLE EN ELLA?.

a. DESPRENDIMIENTO PREMATURO DE d. NECROSIS DE UN MIOMA.


PLACENTA
b. PLACENTA PREVIA e. TORSIÓN DE UN TUMOR OVÁRICO.

c. INSERCIÓN VELAMENTOSA DE
CORDÓN UMBILICAL.

2
Name: ________________________ ID: A

____ 6. GESTANTE DE 26 SEMANAS QUE PRESENTA DESDE HACE 5 DÍAS: MALESTAR GENERAL, NAUSEA,
CEFALEA, EDEMA Y LIGERO DOLOR EN HIPOCONDRIO DERECHO. TA 140/90 PROTEINURIA 3g/24H
EN EXÁMENES HB 12G/DL, BILIRRUBINA TOTAL 1.4 MG/DL, LDH 670 UI/I, TGO 182UI/I, PLAQUETAS
DE 80000MM3. LO MAS PROBABLE ES QUE SE TRATE DE UN EMBARAZO COMPLICADO POR?.

a. EMBOLISMO DE LÍQUIDO AMNIÓTICO d. ISOINMUNIZACIÓN ERITROCITARIA


CON TRANSFUSIÓN FETO – MATERNA
b. MUERTE FETAL CON PASO DE e. E.- PREECLAMPSIA SEVERA + SD
SUSTANCIAS TRONBOPLÁSTICAS A LA HELLP
CIRCULACIÓN MATERNA.
c. DESPRENDIMIENTO PREMATURO DE
PLACENTA SUPERIOR A 50%

____ 7. CUAL ES EL MÉTODO DE DETECCIÓN (SCREENING) DE DIABETES GESTACIONAL QUE SE


RECOMIENDA EN TODAS LAS EMBARAZADAS?

a. DETERMINACIÓN DE GLUCOSA EN d. TEST DE TOLERANCIA ORAL A LA


ORINA GLUCOSA CON 100G DE SOBRECARGA
b. DETERMINACIÓN DE GLICEMIA EN e. TEST DE TOLERANCIA ORAL A LA
AYUNAS GLUCOSA CON 75G DE SOBRECARGA
c. DETERMINACIÓN DE GLICEMIA EN
AYUNASEN DÍAS DIFERENTES

____ 8. CUAL DE LAS SITUACIONES CLINICAS ES UNA CONTRAINDICACION ABSOLUTA PARA LA


ANTICONCEPCION HORMONAL?

a. HIPERTENSION ARTERIAL BIEN d. MUJER MENOR DE 35 AÑOS Y


CONTROLADA. FUMADORA
b. HISTORIA PERSONAL DE e. INFECCION URINARIA.
TROMBOEMBOLISMO PREVIO.
c. DIABETES

____ 9.
CUAL DE LOS SIGUIENTES CARACTERISTICAS NO ES FRECUENTE EN LA VAGINOSIS BACTERIANA

a. PH VAGINAL MENOR A 4.5. d. AUSENCIA DE ERITEMA VULVAR


b. LEUCORREA FLUIDA Y HOMOGENEA e. AUSENCIA DE PRURITO VULVAR

c. PRESENCIA DE CELULAS CLAVE.

____ 10. CUAL DE LAS SIGUIENTES AFIRMACIONES ES FALSA EN RELACIÓN A LA TRANSMISIÓN VERTICAL
AL FETO/RN POR PARTE DE UNA GESTANTE CON INFECCIÓN DE VIH?

a. AUMENTA LA POSIBILIDAD DE d. CON EL TRATAMIENTO


MALFORMACIÓN FETAL. ANTIRRETROVIRAL DURANTE EL
EMBARAZO Y SOBRE TODO DURANTE
EL PARTO
b. TIENE LUGAR SOBRE TODO EN EL e. EL TRATAMIENTO CON ZIDOVUDINA
PARTO. NO HA DEMOSTRADO EFECTOS
ADVERSOS EN EL RECIÉN NACIDO
HASTA LOS 5 AÑOS
c. LA LACTANCIA AUMENTA EL RIESGO
DE TRANSMISIÓN ENTRE UN 10% A
20%.

3
Name: ________________________ ID: A

____ 11. PRIMIGESTA DE 37 SEMANAS INGRESA POR RPM DE 24H DE EVOLUCIÓN NO HAY TRABAJO DE
PARTO NI SIGNOS DE INFECCIÓN AMNIÓTICA. EL TEST DE BISHOP ES DE 4 PUNTOS, EL FETO
ESTA EN CEFÁLICA Y NO HAY SIGNOS DE COMPROMISO FETAL. CUÁL ES LA CONDUCTA
OBSTÉTRICA INDICADA?

a. CESÁREA d. ADMINISTRAR ANTIBIÓTICOS Y


ESPERAR EL COMIENZO ESPONTANEO
DEL PARTO.
b. ADMINISTRAR ANTIBIÓTICOS E e. ESPERAR EL COMIENZO ESPONTANEO
INDUCIR EL PARTO CON SIN TRATAMIENTO O HACER CESÁREA
PROSTAGLANDINAS. CON SIGNOS DE INFECCIÓN
AMNIÓTICA.
c. ADMINISTRAR ANTIBIÓTICOS,
CORTICOIDES Y CONDUCIR EL PARTO
CON OXITOCINA48H DESPUÉS

____ 12. MUJER DE 25 AÑOS CONSULTA POR RETRASO MENSTRUAL DE 3 SEMANAS Y


SANGRADO TRANSVAGINAL DE 24 HORAS, ESTADO GENERAL BUENO. BHCG: 1000
mUI/ml, ECOGRAFIA TRANSVAGINAL ENDOMETRIO DE 12 mm, EN OVARIO DERECHO
IMAGEN QUE APARENTRA CUERPO LUTEO , NO HAY LIQUIDO LIBRE EN FONDO DE
SACO DE DOUGLAS. ¿CUAL ES LA INDICACIÓN CORRECTA?
a. repertir seriadamente cada 48 horas la d. diagnostico de embarazo ectópico
BHCG y la ecografia
b. reposo absoluto y repetir la ecografia en e. laparotomia o laparoscopia
2 - 3 semenas
c. legrado uterino

____ 13. INFECCION POR ESTREPTOCOCO DEL GRUPO B EN RN PUEDE PREVENIRSE


ADMINISTRANDO ANTIBIOTICOTERAPIA PROFILACTICA A LAS EMBARAZADAS
PORTADORAS O CON FACTORES DE RIESGO ¿CUAL SERIA LA ACTITUD INCORRECTA
a. la profilaxis se realiza durante el parto d. la prueba se solicitara entre la 35 y 37
semanas o ante la eminecia del parto
b. en las pacientes alergicas a la ampicilina e. la incidencia de infeccion neonatal es
se puede utilizar la eritromicina mayor entre RN prematuros que a
termino
c. la via mas frecuente de infección es
transplacentarea

____ 14. COMO CLASIFICA USTED A UNA GESTACION QUE ANTES DEL EMBARAZO REFIERE
TENER TENSIONES ARTERIALES NORMALES, QUE EN LA SEMANA 8 SE LE DETECTA
TENSION ARTERIAL 140/90 Y QUE EN LA SEMANA 18 TIENE UNA ENSION ARTERIAL DE
170/110 SIN EDEMA Y INDICE QUE PROTEINURIA/CREATINURIA DE 100 mg/mmol
a. preeclampsia d. hipertensión crónica más preeclampsia
sobreañadida
b. preeclampsia grave e. hipertensión gestacional
c. hipertensión crónica

____ 15. PUERPERA DE 42 AÑOS CON PARTO VAGINAL HACE 12 HORAS EN LA QUE SE
SOSPECHA DE DIAGNOSTICO DE SD HELLP. ¿INDIQUE QUE LITERAL NO
CORRESPONDE?
a. plaquetas de 75000 d. hemoglobina: 7.2 g/dl
b. bilirrubina indirecta de 3.7 mg/dl e. TGO de 520 UI/L
c. TA: 170/110 mmHg

4
Name: ________________________ ID: A

____ 16. SEGUNDIGESTA CON ANTECEDENTE DE PARTO ANTERIOR A LAS 35 SEMANAS, QUE
CONSULTA EN LA SEMANA 32 POR PERCIBIR CONTRACCIONES UTERINAS, EN LA
EXPLORACION SE COMPRUEBA DILATACION CERVICAL DE 1 CM BORRADO 30% CON
UNA LONGITUD CERVICAL DE 1.5 CM POR ECOGRAFIA, EL MONITOREO FETAL
REVELA UNA FCF: 140 LPM, Y UNA CONTRACCIÓN UTERINA DE CADA 5 MINUTOS.
¿CUAL DE LAS SIGUIENTES AFIRMACIONES ES CORRECTA?
a. se trata de un APP establecida que obliga c. se aconseja la administración de
a realizar tocolisis y maduración cervical corticoides para maduración pulmonar
fetal unicamente por el antecedente del
parto pretermino
b. la longitud cervical, inferior a 2.5 cm d. se trata de una APP incierta requiere una
supone un elevado valor predictivo nueva valoracion clinica (AU, FCF Y
negatico para parto pretermino TV) en 1 - 2 horas

____ 17. GESTANTE A TERMINO QUE INGRESA EN TRABAJO DE PARTO EN FASE LATENTE, A LA
VALORACION SE DETECTA PEQUEÑAS VESICULAS DE HERPES SIMPLE RESIDIVANTE.
AÑOS ANTES TUVO UN PRIMOINFECCIÓN DE HERPES GENITAL Y VARIOS BROTES DE
HERPES RESIDIVANTE. ¿CUAL ES LA CONDUCTA A SEGUIR?
a. cesarea d. tratar inmediatamente las lesiones con
Ac. tricloro-acetico, para inactivar el
virus y permitir el parto vaginal
b. parto normal y tratar al RN con aciclovir e. parto normal y aislar al RN de su madre
c. no es necesaria una conducta especial ya
que el herpes residivante no tiene riesgo
para el RN

____ 18. LAS HEMORRAGIAS ANORMALES VAGINALES, FRECUENTES EN EL PRIMER AÑO TRAS
LA MENARQUIA, QUE SE CONOCEN COMO HEMORRAGIAS UTERINAS
DISFUNCIONALES SE DEBEN A:
a. hemorragias de la ovulacion y no d. trastornos de la coagulacion y ameritan
ameritan tratamiento tratamiento
b. ovario poliquistico y amerita tratamiento e. enfermedad inflamatoria pelvica que
amerita tratamiento
c. ciclos anovulatorios y no ameritan
tratamiento

____ 19. MUJER DE 35 AÑOS CON 31 SEMANAS DE EMBARAZO CONSULTA POR UNA SECRECION
VAGINAL DE MAL OLOR, “PESCADO PODRIDO”, NO TIENE PRURITO GENITAL, A LA
VALORACIÓN SE VIDENCIA ABUNDANTE SECRECION BLANCO GRISACEA QUE NO SE
ADHIERE A LAS PAREDES VAGINALES. AL MEZCLAR ESTA MUESTRA CON HIROXIDO
DE POTASIO AL 10% DESPRENDE UN OLOR INTENSO SUIGENERIS. ¿CUAL DE LAS
SIGUIENTES ES EL TRATEMIENTO DE ELECCIÓN?
a. amoxicilina + ac. clavulánico VO o IV c. fluconazol por VO
b. clotrimazol por via intravaginal o VO d. metronizadol por via intravaginal o VO

____ 20. PACIENTE EN SU PRIMER TRIMIESTE DE GESTACION CON HEMOGLOBINA DE 11


CATALOGADA CON ANEMIA LEVE USTED DECIDE INICIAR EL APORTE DE HIERRO.
a. Se trata una anemia cronica y no necesita d. Inicia 200 mg de hierro elemental diarios
de aporte de hierro
b. Considera utilizar hierro sacarosa IV e. Sulfato ferroso 500 mg VO diarios
c. A pesar de que la paciente esta
asintomatica por su etapa de embarazo
considera transfusión

5
ID: A

Reactivos Ginecología y Obstetricia


Answer Section

MULTIPLE CHOICE

1. ANS: C
LA RESPUESTA CORRECTA ES C
A.- LA VACUNA ESPECÍFICA ESTA CONTRA INDICADA EN EL EMBARAZO.
B.- LA CUANTIFICACION DE IGG INDICA INMUNIDAD ADQUIRIDA POR INFECCION PASADA.
D.- EL ACICLOVIR ESTA CONTRAINDICADO EN EL EMBARAZO.
E.- LA ADMINISTRACION DE GAMAGLOBULINA ESPECÍFICA NO A DEMOSTRADO SU EFICACIA.

PTS: 1 DIF: Alto


REF: Obstetricia; Schwarez; ENFERMEDADES INDUCIDAS POR EL EMBARAZO O QUE LO
COMPLICAN; pag 297;año 2011
OBJ: Reconoce y trata las principales enfermedades maternas que complican el embarazo basados en
conocimientos actualizados permitiéndole manejar a las pacientes obstétricas con responsabilidad y ética.
TOP: Obstetricia. KEY: Varicely Embarazo MSC: CECILIA CASCO MANZANO
2. ANS: D
LA RESPUESTA CORRECTA ES D
A.- SE TRATA DE UN EMBARAZO CRONOLOGICAMENTE PROLONGADO.
B.- SE CONSIDERA EMBARAZO DE ALTO RIESGO, LOS CONTROLES DEBEN SER PERMENENTES.
C.- ES UN EMBARAZO PROLONGADO NO NECESITA CORTICOIDES.
E.- SI NO EXISTEN CONTRAINDICACIONES MATERNO FETALES LA PRIMERA OPCION SERA EL
PARTO VAGINAL.

PTS: 1 DIF: Alta


REF: MSP (2012) Componente Normativo Materno Neonatal. GPC Trabajo de Parto (2015)
OBJ: Comprende, aplica y trata las etapas del parto que complican el embarazo basados en
conocimientos actualizados permitiéndole manejar a las pacientes obstétricas con responsabilidad y ética.
TOP: Obstetricia KEY: Trabajo de parto NOT: CECILIA CASCO MANZANO
3. ANS: C
LA RESPUESTA CORRECTA ES C
A.- SE TRATA DE UN FETO EN TERCER PLANO DE HODGE POR LO QUE SE DEBE REALIZAR PARTO
INSTRUMENTAL, LA CESAREA PROGRAMADA TIENE OTRAS INDICACIONES
B.- LA VARIEDAD DE PRESENTACION ES OIIT Y NECESITAMOS UN INTRUMENTO ROTADOR.
D.- UN FETO EN NECESITA PARTO INMEDIATO.
E.- EL OXIGENO A LA MADRE ESTA INDICADO PERO LA ESTRACCION FETAL DEBE SER
INMEDIATA.

PTS: 0 DIF: Alta


REF: MSP (2012) Componente Materno Infantil. GPC Trabajo de Parto (2015)
OBJ: Reconoce y trata las principales enfermedades maternas que complican el embarazo basados en
conocimientos actualizados permitiéndole manejar a las pacientes obstétricas con responsabilidad y ética.
TOP: Obstetricia KEY: Trabajo de Parto NOT: CECILIA CASCO MANZANO

1
ID: A

4. ANS: E
LA RESPUESTA CORRECTA ES E
A.- UNA LONGITUD CRANEO CAUDAL ACORDE CON 8 SEMANAS Y SIN LATIDO CARDIACO
CORRESPONDE UN ABORTO DIFERIDO.
B.- AMENAZA DE ABORTO = FETO VIVO; SE RECOMIENDA REPOSO Y GESTÁGENOS.
C.- CUANDO SOSPECHAMOS EN EMBARAZO ECTOPICO SOLICITAMOS DETERMINACIÓN DE BHCG
PLASMÁTICA CADA 48 H.
D.- POR EG NO ES POSOBLE LLEGAR A ESE DIAGNOSTICO.

PTS: 1 DIF: Alta REF: MSP, (2015).GPC Aborto


OBJ: Comprende, analiza y trata las enfermedades que complican el embarazo basados en
conocimientos actualizados permitiéndole manejar a las pacientes obstétricas con responsabilidad y ética.
TOP: Obstetricia KEY: Aborto NOT: CECILIA CASCO MANZANO
5. ANS: A
LA RESPUESTA CORRECTA ES A
B.- LA HEMORRAGIA EN PLACENTA PREVIA ES INDOLORA.
C.- LA INSERCIÓN VELAMENTOSA DE CORDÓN UMBILICAL-VASA PREVIA PRESENTA
HEMORRAGIA POSTERIOR A RPM.
D.-LA CLINICA DE NECROSIS DE UN MIOMA NO INCLUYE HEMORRGIA TRANSVAGINAL.
E.- TORSIÓN DE UN TUMOR OVÁRICO NO INCLUYE HEMORRGIA TRANSVAGINAL

PTS: 1 DIF: Alta REF: MSP, (2012) Componente Normativo Materno


Neonatal.
OBJ: Comprensión conceptual, analiza y trata las enfermedades que complican el embarazo y parto
basados en conocimientos actualizados permitiéndole manejar a las pacientes obstétricas con
responsabilidad y ética.
TOP: Obstetricia KEY: Desprendimiento de Placenta Normoincerta
NOT: CECILIA CASCO MANZANO
6. ANS: E
LA RESPUESTACORRECTA ES E
A.- EL EMBOLISMO DE LÍQUIDO AMNIÓTICO PROVOCA DIFICULTAD RESPIRATORIA EN MUCHOS
DE LOS CASOS FULMINANTE.
B.- VALORES DE TA ELEVADOS NO SE JUSTIFICAN EN MUERTE FETAL INTRA UTERO.
C.- EL DESPRENDIMIENTO PREMATURO DE PLACENTA PUEDE SER UNA CONSECUENCIA DE
PROBLEMAS HIPERTENSIVOS EN EL EMBARAZO.
D.- EL CUADRO CLINICO DE ISOINMUNIZACIÓN ERITROCITARIA MATERNA NO CORRESPONDE.

PTS: 1 DIF: Alta


REF: MSP; (2016). GPC Transtornos Hipertensivos del Embarazo
OBJ: Aplicación conceptual de las principales enfermedades maternas que complican el embarazo y
parto basados en conocimientos actualizados permitiéndole manejar a las pacientes obstétricas con
responsabilidad y ética. TOP: Obstetricia. KEY: Transtornos Hipertensivos del
Embarazo
NOT: CECILIA CASCO MANZANO

2
ID: A

7. ANS: E
LA RESPUESTA CORRECTA ES E.
A.- DETERMINACIÓN DE GLUCOSA EN ORINA NO HACE EL DIAGNOSTICO
B.- DETERMINACIÓN DE GLICEMIA EN AYUNAS POR SI SOLA NO ES CONCLUYENTE.
C.- DETERMINACIÓN DE GLICEMIA EN AYUNAS EN DÍAS DIFERENTE, NO ES CONCLUYENTE.
D.-TEST DE TOLERANCIA ORAL A LA GLUCOSA CON 100G DE SOBRECARGA ES UNA PRUEBA
ALTAMENTE SENSIBLE Y ESPECÍFICA PERO NO ES EL METODO DE SCREENING.

PTS: 1 DIF: Alta REF: MSP, (2015) .GPC Diabetes y Embarazo


OBJ: Aplicación conceptual sobre la clasificaciónde las principales enfermedades maternas que
complican el embarazo basados en conocimientos actualizados permitiéndole manejar a las pacientes
obstétricas con responsabilidad y ética.. TOP: Obstetricia KEY: Diabetes y
Embarazo
NOT: CECILIA CASCO MANZANO
8. ANS: B
LA RESPUESTA CORRECTA ES B
A.- HIPERTENSION ARTERIAL BIEN CONTROLADA NO CONTRAINDICA EL USO DE
ANTICONCEPCION HORMONAL.
C.- DIABETES NO CONTRAINDICA EL USO DE ANTICONCEPCION HORMONAL.
D.- MUJER FUMADORA DE CUALQUIER EDAD ES UN FACTOR DE RIESGO QUE PUEDE
CONTRAINDICAR LA ANTICONCEPCION HORMONAL PERO NODE FORMA ABSOLUTA
E.- INFECCION URINARIA NO TIENE INCONVENIENTES CON SU USO

PTS: 1 DIF: Alto


REF: MSP, (2012). Componente Materno Infantil Guia de Manejo en Planificacion Familiar
OBJ: Comprensión y aplicación conceptual de las principales enfermedades maternas que complican el
embarazo y parto basados en conocimientos actualizados permitiéndole manejar a las pacientes
obstétricas con responsabilidad y ética. TOP: Obstetricia. KEY: Planificacion
Familiar
NOT: CECILIA CASCO MANZANO
9. ANS: A
LA RESPUESTA CORRECTA ES A
B.- LA LEUCORREA EN CANDIDIASIS SUELE SER GRUMOSA Y EN TRICOMONIASIS PRESENTA
BURBUJAS.
C.- LAS CELULAS CLAVE SON CELULAS LLENAS DE COCOBACILOS GRAM NEGATIVOS PROPIOS DE
VAGINOSIS BACTERIANA.
D.- EL ERITEMA VULVAR ES PROPIO DE LA CANDIDIASIS
E.- EL PRURITO VULVAR ES PROPIO DE LA CANDIDIASIS

PTS: 1 DIF: Alto REF: MSP, (2014). Manejo de ETS.


OBJ: Comprensión conceptual, analiza y trata las enfermedades que complican el embarazo y parto
basados en conocimientos actualizados permitiéndole manejar a las pacientes obstétricas con
responsabilidad y ética.
TOP: Obstetricia. KEY: ETS NOT: CECILIA CASCO MANZANO

3
ID: A

10. ANS: A
LA RESPUESTA CORRECTA ES A
B.- TIENE LUGAR SOBRE TODO EN EL PARTO ES CORRECTO.
C.- LA LACTANCIA ESTA CONTRAINDICADA EN ESTAS PACIENTES.
D.- EL INICIO DE LA MEDICACION ANTIRETROVIRAL DEPENDERA DE LA CARGA VIRAL Y DE LA
EDAD GESTACIONAL
E.- EL TRATAMIENTO CON ZIDOVUDINA NO HA DEMOSTRADO EFECTOS ADVERSOS EN EL RECIÉN
NACIDO HASTA LOS 5 AÑOS ES CORRECTO

PTS: 1 DIF: Alto REF: MSP, (2013) Manejo de VIH y Embarazo


OBJ: Aplicación de la terminología, comprende, analiza y trata las enfermedades que complican el
embarazo basados en conocimientos actualizados permitiéndole manejar a las pacientes obstétricas con
responsabilidad y ética. TOP: Obstetricia. KEY: VIH y Embarazo.
NOT: CECILIA CASCO MANZANO
11. ANS: B
LA RESPUESTA CORRECTA ES B
A.- CESÁREA NO PORQUE TIENE CONDICIONES OBSTETRICA PARA PARTO VAGINAL NORMAL
C.- NO ES EMBZ MENOR DE 34.6 SEMANAS EL BISHOP AUN ES DEFABORABLE Y CON DG DE RPM SE
DEBE TERMINAR EL EMBARAZO INMEDIATAMENTE
D.- EN EMBARAZO A TERMINO Y RPM SE DEBE TERMINAR EL EMBARAZO
E.- NO ESPERAR EL COMIENZO ESPONTANEO POR SER EMBZ A TERMINO Y AUN CON SG DE
INFECCION LA MEJOR VIA PARA TERMINACION ES LA VIA BAJA

PTS: 1 DIF: Alta


REF: MSP, (2014). GPC Ruptura Prematura de Membranas Pretermino
OBJ: Comprende, analiza y trata las enfermedades que complican el embarazo basados en
conocimientos actualizados permitiéndole manejar a las pacientes obstétricas con responsabilidad y
ética..
TOP: Obstetricia. KEY: Ruptura Prematura de Membranas NOT: CECILIA CASCO MANZANO
12. ANS: B
LA RESPUESTA CORRECTA ES B.
A. La edad gestacional no justifica la realizacion de ecografia en 48 horas, puesto que observaremos saco
gestacional entre las 4-5 semanas y embrión entre 5-6 semanas,
C.No existe imagen intrauterina que justifique el LIU
D. La BHCG esta acorde a la edad gestacional, en ovario no existe imagen sospechosa por lo que se
descarta el embarazo ectópico
E.No existe imagen extrauterina que justifique la laparotomia o laparoscopia

PTS: 1 DIF: alta REF: Guia de Practica Clinca de Aborto 2013


OBJ: Comprende, analiza y trata las enfermedades que complican el embarazo basados en
conocimientos actualizados permitiéndole manejar a las pacientes obstétricas con responsabilidad y ética.
TOP: Obstetricia. KEY: Aborto NOT: Cecilia Margarita Casco Manzano

4
ID: A

13. ANS: C
LA RESPUESTA CORRECTA ES C
A. la profilaxis esta indicada durante el trabajo de parto inminente
B. en todas las pacientes alergicas a la ampicilana la segunda opcion es la eritromicina
D. el isopado de area perineal y perianal se debe solicitar a partir de las 35 semanas si la prueba es
positva la paciente debe resibir profilaxis durante el trabajo de parto independientemente de su semanas
de gestación
E. la incidencia de infecion neonatal pro estreptoco betahemolitico del grupo B, es mayor es en Rn
prematuros puesto que para ellos constituye mas una causa de inicio de trabajo prematuro que una
consecuencia del mismo

PTS: 1 DIF: Alta


REF: Guia de Practica Clínica de Trabajo de Parto 2015, Guia de Practica Clinica de RPM 2015.
OBJ: Reconoce y trata las principales enfermedades maternas que complican el embarazo basados en
conocimientos actualizados permitiéndole manejar a las pacientes obstétricas con responsabilidad y ética.
TOP: Obstetricia KEY: Estreptococo NOT: Cecilia Casco Manzano
14. ANS: D
LA RESPUESTA CORRECTA ES LA D
A. el diagnostico de preeclampsia supone una EG mayor a 20 semanas a ecepción de 3 casos especiales
que no corresponden a la paciente
B. sin contar sus antecedentes previos, anterioes a las 28 semanas, corresponderia a una preeclampsia
grave
C. al existir TA elevada antes de las 20 semanas, hipertensión cronica evidenciada antes de las 20
semanas se trata de una paciente hipertensa crónica
E. se evidecnia TA alta antes de las 20 semanas lo que descarta un diagnostico de hipertensión
gestacional

PTS: 1 DIF: Alta


REF: Guia de Practica Clinca de Trastornos Hipertensivos del 2017
OBJ: Reconoce y trata las principales enfermedades maternas que complican el embarazo basados en
conocimientos actualizados permitiéndole manejar a las pacientes obstétricas con responsabilidad y ética.
TOP: Obstetricia KEY: Preeclampsia NOT: Cecilia Casco Manzano
15. ANS: C
LA RESPUESTA CORRECTA ES C
A. el diagnostico de Sindrome HELLP se realiza en base a los resultados de laboratorio uno de ellos es la
plaquetopenia
B. el diagnostico de Sindrome HELLP se realiza en base a los resultados de laboratorio en donde la
bilirrubina indirecta se eleva a concecuencia de un proceso de hemolisis.
D. el proceso de hemolisis es marcado lo que se refleja como anemia-
E. el diagnostico de Sindrome HELLP se realiza en base a los resultados de laboratorio uno de ellos son
las transaminasas.

PTS: 1 DIF: Alta REF: Guia de Practica Clínica de Trastornos Hipertensivos


2017
OBJ: Reconoce y trata las principales enfermedades maternas que complican el embarazo basados en
conocimientos actualizados permitiéndole manejar a las pacientes obstétricas con responsabilidad y ética.
TOP: Obstetricia. KEY: HELLP NOT: Cecilia Casco Manzano

5
ID: A

16. ANS: A
LA RESPUESTA CORRECTA ES A
B. toda longitud cevical inferior a 2. 5 cm es un indicador predictivo positivo para amenaza de partoo
pretermino
C. toda paciente con amenaza de parto pretermino entre las 24 y 34.6 semanas debe recibir maduracion
pulmonar fetal
D. la amenaza de parto pretermino incierta presupone AU: irregular sin cambios cervicales, que en
muchas de las ociaciones se resuelven con reposo.

PTS: 1 DIF: Alta REF: Guia de Practica Clinica de Trabajo de Parto 2015
OBJ: Reconoce y trata las principales enfermedades maternas que complican el embarazo basados en
conocimientos actualizados permitiéndole manejar a las pacientes obstétricas con responsabilidad y ética.
TOP: Obstetricia. KEY: Parto pretermino NOT: Cecilia Casco Manzano
17. ANS: A
LA RESPUESTA CORRECTA ES A
B. no se puede suponer parto si existen signos clinicos de infeccion herpetica genital activa
C. todo herpes residivante en fase activa representa un riesgo para el RN de considerarse parto normal
D. si la lesion herpetica esta activa no debe considerarse parto normal el tratemiento puede demorarse
mas alla de 7-15 días
E. no se recomiendo el parto vaginal y no tiene sentido aislar al RN de su madre.

PTS: 1 DIF: Alta REF: Guia de Practica Clinica de Parto por Cesarea 2014
OBJ: Reconoce y trata las principales enfermedades maternas que complican el embarazo basados en
conocimientos actualizados permitiéndole manejar a las pacientes obstétricas con responsabilidad y ética.
TOP: Obstetricia KEY: Herpes NOT: Cecilia Casco Manzano
18. ANS: C
LA RESPUESTA CORRECTA ES LA C
A. duratne los 2 primeros años inciados la menarquia, es común la amenorrea por anovulación y no
constituye un ente patológico
B. el ovario poliquistico amerita tratamiento pero no es la causa de hemorragias uterinas anormales
unicamente en el primer año tras el inicio de la menarquia
D. es una condición bastante infrecuente con una incidencia baja inferior a 0.07%
E. un EPI no justifica hemorragias uterinas en ninguna etapa de la vida fertil.

PTS: 1 DIF: Alta


REF: Guia de Practica Clinica de Sindrome Ovario Poliquistico Española 2016
OBJ: Reconoce y trata las principales enfermedades ginecologicas basados en conocimientos
actualizados permitiéndole manejar a las pacientes con responsabilidad y ética.
TOP: Ginecologia KEY: anovulación NOT: Cecilia Casco Manzano
19. ANS: D
LA RESPUESTA CORRECTA ES D
A. El acido clavulanico durante el embarazo esta contraindicado de enterolitis necrotisante
B. No se trata por una infección micotica.
C. Esta contraindicado en el embarazo

PTS: 1 DIF: Alta


REF: Guia de Practica Clínica de Infecciones Vaginales en el Embarazo 2016
OBJ: Reconoce y trata las principales enfermedades maternas que complican el embarazo basados en
conocimientos actualizados permitiéndole manejar a las pacientes obstétricas con responsabilidad y ética.
TOP: Obstricia KEY: Infecciónes vaginales NOT: Cecilia Casco Manzano

6
ID: A

20. ANS: D
LA RESPUESTA CORRECTA ES D
A. Toda anemia crónica debe ser manejada con aporte de hierro según valores de hemoglobina
B. EL hierro intravenoso esta indicado en anemia moderado
C. Se transfundira a pacientes obstetricas con valores de hemogloibana menores de 7 tomando en cuenta
si existe o no signos de descompensación
E. Se puede indicar sulfato ferroso durante la gestacion pero tomando en cuenta que el aporte de hierro
elemental sea de 100 -200 mg

PTS: 1 DIF: Alta REF: Guia de Practica Clinica de Anemia en el Embazaro


2016
OBJ: Reconoce y trata las principales enfermedades maternas que complican el embarazo basados en
conocimientos actualizados permitiéndole manejar a las pacientes obstétricas con responsabilidad y ética.
TOP: obstetricia KEY: Anemia NOT: Cecilia Casco Manzano

7
Reactivos Ginecología y Obstetricia [Answer Strip] ID: A

C
_____ 3. E
_____ 6. B 11.
_____ A 16.
_____

C
_____ 1.

E
_____ 7.
A 17.
_____
B 12.
_____
E
_____ 4.

D
_____ 2. B
_____ 8.

C 13.
_____ C 18.
_____

A
_____ 5.
A
_____ 9.

D 19.
_____
D 14.
_____

A 10.
_____

D 20.
_____
C 15.
_____
Name: ________________________ Class: ___________________ Date: __________ ID: A

Gineco-obstetricia

Multiple Choice
Identify the choice that best completes the statement or answers the question.

____ 1. Seleccione la patología que presenta incremento en el análisis cuantitativo de Beta-hCG


(human chorionic gonadotropin) valorado cada 48 horas en una embarazada:
a. Embarazo molar c. Amenaza de aborto
b. Retención fetal d. Desprendimiento placentario

____ 2. Del siguiente listado seleccione la opción que contenga las causas de hemorragia
fetomaterna relacionadas con aloinmunización del antígeno eritrocítico:
1) Hiperglucemia.
2) Embarazo ectópico.
3) Aborto electivo.
4) Neutropenia.
5) Amniocentesis.
a. 1, 2, 4 c. 2, 3, 5
b. 1, 2, 3 d. 3, 4, 5

____ 3. ¿Cuál es el medicamento de primera elección y su dosis en el manejo activo de la tercera


etapa del parto para evitar la hemorragia postparto?
a. Oxitocina -10 Ul vía intramuscular c. Metilergonovina - 0.8 mg vía
intramuscular
b. Hidralazina - 50 mg vía d. Misoprostol - 200 mcg vía oral
intavenosa
____ 4. Cuántos grados hay que rotar en un parto céfalo-vaginal de una variedad de
presentación Occipito Ilíaca Izquierda Anterior, en la rotación interna que coloca a la
presentación en directa?
a. 135 grados en contra de las c. 25 grados en sentido de las
manecillas del reloj manecillas del reloj
b. 90 grados en sentido de las d. 45 grados en contra de las
manecillas del reloj manecillas del reloj
____ 5.
Indique la fase que permite un trabajo de parto activo y se divide en tres etapas:
1) borramiento y dilatación del cuello uterino,
2) expulsión fetal y
3) separación y expulsión de la placenta:

a. Fase 1 del parto c. Fase 2 del parto


b. Fase 4 del parto d. Fase 3 del parto
____ 6. Paciente de 54 años con obesidad grado 2, nuligesta que presenta hemorragia
posmenopáusica. La biopsia de endometrio muestra hiperplasia endometrial compleja
atípica ¿Cuál es la conducta más adecuada para esta paciente?
a. Dosis alta de estrógenos c. Conducta expectante
b. Dosis altas de progestágenos d. Histerectomía

1
Name: ________________________ ID: A

____ 7. ¿Cuál es el tratamiento antibiótico ambulatorio de elección de la enfermedad pélvica


inflamatoria leve?
a. Aciclovir 400 mg vía oral cada12 c. Cefoxitína 1g intramuscular dos
horas por 14 días + clindamicina dosis + ciprofloxacino 500 mg vía
900 mg vía oral dosis única + oral cada 24 horas por 7 días +
amoxicilina 500 mg vía oral metronidazol 250 mg cada 12
cada12 horas por 10 días horas por 7 días
b. Ciprofloxacina 400 mg d. Ceftriaxone 250 mg intramuscular
intravenoso unidosis + unidosis + doxiciclina 100 mg vía
eritromicina 500 mg vía oral cada oral cada 12 horas por 14 días +
12 horas por 10 días + metronidazol 500 mg vía oral cada
metronidazol 250 mg cada 12 12 horas por 14 días
horas por 7 días
____ 8. ¿Cuál es la causa más frecuente de mortalidad perinatal en los embarazos gemelares?
a. Preeclampsia c. Parto prematuro
b. Diabetes Gestacional d. Rotura prematura de membranas

____ 9. Determine el parámetro útil, en el primer trimestre de embarazo, para la determinación


más exacta de la edad gestacional:
a. Diámetro biparietal c. La longitud del fémur
b. El perímetro cefálico d. La longitud corona-rabadilla
____ 10. Paciente femenina que acude refiriendo cuadro clínico caracterizado por salida de
secreción vaginal en poca cantidad, sugiere olor a pescado, no prurito, test de pH
vaginal mayor a 4.5. Indique su diagnóstico etiológico.
a. Bacilo de Dóderlein vaginal c. Candidiasis vulvovaginal
b. Vaginosis bacteriana d. Tricomoniasis vaginal

____ 11. Mujer en edad fértil sin deseo de embarazo, con antecedentes de trastornos
tromboembólicos. ¿Cuál es la mejor alternativa anticonceptiva para ella?
a. Dispositivo intrauterino con T de c. Implantes anticonceptivos
cobre
b. b) Sistema intrauterino con d. Anticonceptivos orales
levonorgestrel (LNG - IUS) combinados

____ 12. Con relación al síndrome de Hellp, señale el enunciado correcto:


a. Se recomienda el uso rutinario de c. Según la clasificación de
corticoides para el tratamiento del Míssissipi el síndrome de Hellp, es
síndrome de HELLP. de Clase I, cuando las plaquetas
son menores a 50.000, AST o
ALT mayor a 70 Ul/L, y LDL mayor
a 600 Ul/L
b. Se recomienda el intercambio de d. Cuando el conteo de plaquetas se
plasma o plasmaféresis para el encuentra comprendido entre
tratamiento de síndrome de 20000 y 49000 pL con síndrome
HELLP, en especial durante los de HELLP, no se debe transfundir
primeros 4 días posparto plaquetas previo a la cesárea

2
Name: ________________________ ID: A

____ 13. Identifique la etapa clínica del cáncer cervicouterino de acuerdo a la FIGO (International
Federation of Obstetricians and Gynecologists): El carcinoma se extiende fuera del
cuello uterino, pero no se extiende a la pared pélvica; afecta la vagina, pero no hasta el
tercio inferior y con lesiones clínicas mayores de 4 cm:
a. IIB c. IIA2
b. IA2 d. IIIB

____ 14. Paciente de 23 años con embarazo de 27 semanas de gestación. Hace 18 horas eliminó
un líquido claro trasvaginal en moderada cantidad. Seleccione el tratamiento adecuado:
a. Manejo ambulatorio, vigilar signos c. Inducción con misoprostol y
de alarma y antibioticoterapia amoxicilina más ácido clavulánico
profiláctica
b. Manejo expectante en d. Inicia oxitocina para terminar la
hospitalización y antibioticoterapia gestación por riesgo de infección
con ampicilina y eritromicina
____ 15. La inmunización pasiva contra enfermedad hemolítica del recién nacido, se realiza con
300 mg de un concentrado purificado de anticuerpos contra el antígeno Rho (D).
Seleccione la vía correcta para su administración:
a. Subcutánea c. Intramuscular
b. Intravenosa d. Intraplacentaria

____ 16. RN producto de gestación normal a término, antecedente de labor de parto prolongada,
presentó un APGAR al 1er minuto de 4, a los 5 minutos de 6, se realizó aspiración
nasofaríngea y endotraqueal por presencia de meconio, posteriormente recuperó el
APGAR a 9, no se requirió reanimación cardiopulmonar, la FC se recuperó de 120 Ipm
a 160 Ipm. A las 8 horas de la atención se reporta: respiración periódica, convulsión,
palidez generalizada, succión deficiente, bradicardia FC 120 Ipm ¿Qué ocasiona este
cuadro clínico?
a. Síndrome de sección medular c. Enfermedad de membrana hialina
b. Síndrome aspirativo d. Edema cerebral

____ 17. A qué clasificación BI-RADS corresponden las siguientes características: “Tumor
irregular, calcificaciones lineales y ramificadas finas”
a. BI-RADS 5 c. BI-RADS 2
b. BI-RADS 6 d. BI-RADS 3

____ 18. En una paciente que presenta una úlcera genital indurada no dolorosa, con bordes lisos,
de base regular, con adenopatías bilaterales no dolorosas. Indique el diagnóstico en
este caso:
a. Herpes II c. Sífilis primaria
b. Chancroide d. Granuloma inguinal

____ 19. ¿Cuál es el parámetro normal en el puerperio inmediato?


a. Aumento del volumen uterino c. Lactancia muy dolorosa
b. Tromboflebitis d. Incontinencia urinaria

____ 20. Al examen físico, durante el trabajo de parto, no se detecta el polo fetal en el fondo y la
cabeza móvil se encuentra en una fosa ilíaca y la pelvis en la otra. Seleccione la opción
que indique lo que sucede con el feto:
a. Su posición es oblicua c. Su presentación es de frente
b. Su presentación es compuesta d. Su posición es transversa

3
Name: ________________________ ID: A

____ 21. Señale una condición clínica de la enfermedad de VIH en adultos y adolescentes en el
estadio III.
a. Encefalopatía asociada al VIH c. Queilitis angular
b. Tuberculosis pulmonar actual d. Erupción papular pruriginosa
____ 22. ¿Cuál es el microorganismo causal de la sífilis?
a. Chlamidya trachomatis c. Treponema pallidum
b. Klebsiella granulomatis d. Haemophilus ducreyi

____ 23. ¿Cuál es el tratamiento de la vaginosis bacteriana en el embarazo?


a. Doxiciclina 100 mg VO c /12 h por c. Ampicilina 500 mg VO el 6h por
7 días 10 días
b. Metronidazol 500 mg VO c/12 h d. Penicilina benzatínica 1200.000
durante 7 días Ul una dosis
____ 24. Señale los resultados esperados en el control tiroideo prenatal del segundo trimestre
para una paciente gestante normal:
a. TSH 0.01 ug/100 ml; T3 libre 12 c. TSH 5.4 ug/100 ml; T3 libre 2.4
pg/ml; T4 libre 4 ng/100 ml. pg/ ml; T4 libre 0.8 ng/100 ml
b. TSH 0.8 ug/100 ml; T3 libre 0.1 d. TSH suprimida; T3 libre 4.2 pg/
pg/ml; T4 libre 0.8 ng/100 ml ml; T4 libre: 1.0 ng/100 ml
____ 25. ¿Cuál de estas acciones se realiza en el minuto cero, al activar la Clave Roja obstétrica?
a. Administrar medicamentos c. Iniciar terapia antihipertensiva
uterotónicos
b. Valorar signos clínicos de shock d. Administrar sulfato de magnesio a
hemorrágico dosis de impregnación

4
Name: ________________________ ID: A

____ 26.
Mientras usted está de turno en un hospital llega una mujer de 29 años, embarazada
de 35 semanas de gestación. La paciente está preocupada porque desde la mañana
no se ha movido su niño. El examen de monitoreo fetal presenta el siguiente trazado.
Seleccione la interpretación adecuada de este examen

a. Es un ejemplo de desaceleración c. Este un ejemplo de aceleración y


variable y ocurre por compresión usualmente se produce por acción
del cordón umbilical de los movimientos fetales
b. Es un ejemplo de d. Es un ejemplo de desaceleración
desaceleraciones tempranas y tardía, y ocurre por insuficiencia
ocurre por compresión de la uteroplacentaria
cabeza del feto
____ 27. En el sangrado uterino anormal, que no responde al tratamiento hormonal, se debe
referir a la paciente a un especialista para que realice:
a. Colocación de DIU con liberación c. Histerectomía vaginal
de Levonorgestrel
b. Histeroscopia con biopsia d. Ablación endometrial

____ 28. Paciente en fase activa de labor de parto, con dilatación de 8 cm y 100 % de
borramiento, presentación cefálica en 2o plano. Su monitoreo electrónico demuestra
una frecuencia cardiaca fetal basal de 132 por minuto, con variabilidad de moderados
elementos. Se observan también descensos de la frecuencia cardiaca de 20 latidos por
minuto, que aparecen en coincidencia con el acmé de la contracción. Seleccione el
diagnóstico:
a. Desaceleraciones tardías c. La descripción es normal
b. Desaceleraciones variables d. Desaceleraciones tempranas

____ 29. Según la clasificación anatómica señale una causa de amenorrea adquirida:
a. Tabique vaginal c. Agenesia de los conductos de
Müller
b. Atresia de cuello uterino d. Síndrome de Asherman

5
Name: ________________________ ID: A

____ 30. De acuerdo a la OMS (Organización Mundial de la Salud), indique el número mínimo de
chequeos que debe realizar el médico u obstetriz para un control prenatal óptimo en el
embarazo de bajo riesgo:
a. 4 c. 5
b. 3 d. 6

____ 31. En una paciente que presenta una úlcera genital indurada no dolorosa, con bordes lisos,
de base regular, con adenopatías bilaterales no dolorosas. Indique el diagnóstico en
este caso:
a. Sífilis primaria c. Chancroide
b. Granuloma inguinal d. Herpes II

____ 32. A qué clasificación BI-RADS corresponden las siguientes características: “Tumor
irregular, calcificaciones lineales y ramificadas finas”
a. BI-RADS c. BI-RADS 5
b. BI-RADS 3 d. BI-RADS 2

____ 33. Seleccione la patología que presenta incremento en el análisis cuantitativo de Beta- hCG
(human chorionic gonadotropin) valorado cada 48 horas en una embarazada:
a. Desprendimiento placentario c. Embarazo molar
b. Amenaza de aborto d. Retención fetal

____ 34. Paciente de 23 años con embarazo de 27 semanas de gestación. Hace 18 horas eliminó
un líquido claro trasvaginal en moderada cantidad. Seleccione el tratamiento adecuado:
a. Manejo expectante en c. Inducción con misoprostol y
hospitalización y antibioticoterapia amoxicilina más ácido clavulánico
con ampicilina y eritromicina
b. Manejo ambulatorio, vigilar signos d. Inicia oxitocina para terminar la
de alarma y antibioticoterapia gestación por riesgo de infección
profiláctica
____ 35. Al examen físico, durante el trabajo de parto, no se detecta el polo fetal en el fondo y la
cabeza móvil se encuentra en una fosa ilíaca y la pelvis en la otra. Seleccione la opción
que indique lo que sucede con el feto:
a. Su posición es transversa c. Su posición es oblicua
b. Su presentación es compuesta d. Su presentación es de frente

____ 36. Con relación al síndrome de Hellp, señale el enunciado correcto:


a. Se recomienda el uso rutinario de c. Según la clasificación de
corticoides para el tratamiento del Mississipi el síndrome de Hellp, es
síndrome de HELLP de Clase I, cuando las plaquetas
son menores a 50.000, AST o
ALT mayor a 70 Ul/L, y LDL mayor
a 600 Ul/L
b. Cuando el conteo de plaquetas se d. Se recomienda el intercambio de
encuentra comprendido entre plasma o plasmaféresis para el
20000 y 49000 pL con síndrome tratamiento de síndrome de
de HELLP, no se debe transfundir HELLP, en especial durante los
plaquetas previo a la cesárea primeros 4 días posparto

6
Name: ________________________ ID: A

____ 37. Identifique la etapa clínica del cáncer cervicouterino de acuerdo a la FIGO (International
Federation of Obstetricians and Gynecologists): El carcinoma se extiende fuera del
cuello uterino, pero no se extiende a la pared pélvica; afecta la vagina, pero no hasta el
tercio inferior y con lesiones clínicas mayores de 4 cm:
a. IA2 c. IIIB
b. IIA2 d. IIB

____ 38. Del siguiente listado seleccione la opción que contenga las causas de hemorragia
fetomaterna relacionadas con aloinmunización del antígeno eritrocítico:
1) Hiperglucemia.
2) Embarazo ectópico
3) Aborto electivo.
4) Neutropenia.
5) Amniocentesis.

a. 1, 2, 3 c. 2, 3, 5
b. 3, 4, 5 d. 1, 2, 4
____ 39. ¿Cuál es la causa más frecuente de mortalidad perinatal en los embarazos gemelares?
a. Preeclampsia c. Parto prematuro
b. Diabetes Gestacional d. Rotura prematura de membranas

____ 40. Paciente en fase activa de labor de parto, con dilatación de 8 cm y 100 % de
borramiento, presentación cefálica en 2o plano. Su monitoreo electrónico demuestra
una frecuencia cardiaca fetal basal de 132 por minuto, con variabilidad de moderados
elementos. Se observan también descensos de la frecuencia cardiaca de 20 latidos por
minuto, que aparecen en coincidencia con el acmé de la contracción. Seleccione el
diagnóstico:
a. Desaceleraciones tardías c. La descripción es normal
b. Desaceleraciones tempranas d. Desaceleraciones variables

____ 41. De acuerdo a la OMS (Organización Mundial de la Salud), indique el número mínimo de
chequeos que debe realizar el médico u obstetriz para un control prenatal óptimo en el
embarazo de bajo riesgo:
a. 6 c. 4
b. 5 d. 3

____ 42. Determine el parámetro útil, en el primer trimestre de embarazo, para la determinación
más exacta de la edad gestacional:
a. Diámetro biparietal c. La longitud del fémur
b. El perímetro cefálico d. La longitud corona-rabadilla
____ 43. ¿Cuál es el medicamento de primera elección y su dosis en el manejo activo de la tercera
etapa del parto para evitar la hemorragia postparto?
a. Metilergonovina - 0.8 mg vía c. Oxitocina -10 Ul vía intramuscular
intramuscular
b. Misoprostol - 200 mcg vía oral d. Hidralazina - 50 mg vía
intravenosa

7
Name: ________________________ ID: A

____ 44. Paciente femenina que acude refiriendo cuadro clínico caracterizado por salida de
secreción vaginal en poca cantidad, sugiere olor a pescado, no prurito, test de pH
vaginal mayor a 4.5. Indique su diagnóstico etiológico
a. Bacilo de Dóderlein vaginal c. Trícomoniasis vaginal
b. Candidiasis vulvovaginal d. Vaginosis bacteriana

____ 45. La inmunización pasiva contra enfermedad hemolítica del recién nacido, se realiza con
300 mg de un concentrado purificado de anticuerpos contra el antígeno Rho (D).
Seleccione la vía correcta para su administración:
a. Subcutánea c. Intravenosa
b. Intramuscular d. Intraplacentaria

____ 46. Indique la fase que permite un trabajo de parto activo y se divide en tres etapas:
1) borramiento y dilatación del cuello uterino,
2) expulsión fetal y
3) separación y expulsión de la placenta:
a. Fase 2 del parto c. Fase 4 del parto
b. Fase 1 del parto d. Fase 3 del parto

____ 47. RN producto de gestación normal a término, antecedente de labor de parto prolongada,
presentó un APGAR al 1er minuto de 4, a los 5 minutos de 6, se realizó aspiración
nasofaríngea y endotraqueal por presencia de meconio, posteriormente recuperó el
APGAR a 9, no se requirió reanimación cardiopulmonar, la FC se recuperó de 120 Ipm
a 160 Ipm. A las 8 horas de la atención se reporta: respiración periódica, convulsión,
palidez generalizada, succión deficiente, bradicardia FC 120 Ipm ¿Qué ocasiona este
cuadro clínico?
a. Síndrome de sección medular c. Edema cerebral
b. Síndrome aspirativo d. Enfermedad de membrana hialina

____ 48. ¿Cuál es el parámetro normal en el puerperio inmediato?


a. Aumento del volumen uterino c. Tromboflebitis
b. Incontinencia urinaria d. Lactancia muy dolorosa

____ 49. Señale una condición clínica de la enfermedad de VIH en adultos y adolescentes en el
estadio III.
a. Queilitis angular c. Tuberculosis pulmonar actual
b. Erupción papular pruriginosa d. Encefalopatía asociada al VIH
____ 50. ¿Cuántos grados hay que rotar en un parto céfalo-vaginal de una variedad de
presentación Occipito Ilíaca Izquierda Anterior, en la rotación interna que coloca a la
presentación en directa?
a. 25 grados en sentido de las c. 45 grados en contra de las
manecillas del reloj manecillas del reloj
b. 90 grados en sentido de las d. 135 grados en contra de las
manecillas del reloj manecillas del reloj
____ 51. ¿Cuál es el tratamiento de la vaginosis bacteriana en el embarazo?
a. Penicilina benzatínica 1200.000 c. Ampicilina 500 mg VO c / 6h por
Ul una dosis 10 días
b. Doxiciclina 100 mg VO c /12 h por d. Metronidazol 500 mg VO c /12 h
7 días durante 7 días

8
Name: ________________________ ID: A

____ 52. ¿Cuál es el tratamiento antibiótico ambulatorio de elección de la enfermedad pélvica


inflamatoria leve?
a. Ceftriaxone 250 mg intramuscular c. Cefoxitina 1g intramuscular dos
unidosis + doxiciclina 100 mg vía dosis + ciprofloxacino 500 mg vía
oral cada 12 horas por 14 días + oral cada 24 horas por 7 días +
metronidazol 500 mg vía oral cada metronidazol 250 mg cada 12
12 horas por 14 días horas por 7 días
b. Aciclovir 400 mg vía oral cada12 d. Ciprofloxacina 400 mg
horas por 14 dias + clindamicina intravenoso unidosis +
900 mg vía oral dosis única + eritromicina 500 mg vía oral cada
amoxicilina 500 mg vía oral 12 horas por 10 días +
cada12 horas por 10 días metronidazol 250 mg cada 12
horas por 7 días
____ 53. ¿Cuál de estas acciones se realiza en el minuto cero, al activar la Clave Roja obstétrica?
a. Valorar signos clínicos de shock c. Administrar medicamentos
hemorrágico uterotónicos
b. Iniciar terapia antihipertensiva d. Administrar sulfato de magnesio a
dosis de impregnación
____ 54. ¿Cuál es el microorganismo causal de la sífilis?
a. Haemophilus ducreyi c. Treponema pallidum
b. Chlamidya trachomatis d. Klebsiella granulomatis

____ 55. Señale los resultados esperados en el control tiroideo prenatal del segundo trimestre
para una paciente gestante normal:
a. TSH suprimida; T3 libre 4.2 pg/ c. TSH 5.4 ug/100 ml; T3 libre 2.4
ml; T4 libre: 1.0 ng/100 ml. pg/ ml; T4 libre 0.8 ng/100 ml
b. TSH 0.01 ug/100 ml; T3 libre 12 d. TSH 0.8 ug/100 ml; T3 libre 0.1
pg/ml; T4 libre 4 ng/100 ml pg/ml; T4 libre 0.8 ng/100 ml
____ 56. Según la clasificación anatómica señale una causa de amenorrea adquirida:
a. Atresia de cuello uterino c. Tabique vaginal
b. Agenesia de los conductos de d. Síndrome de Asherman
Müller

9
Name: ________________________ ID: A

____ 57. Mientras usted está de turno en un hospital llega una mujer de 29 años, embarazada
de 35 semanas de gestación. La paciente está preocupada porque desde la mañana no
se ha movido su niño. El examen de monitoreo fetal presenta el siguiente trazado.
Seleccione la interpretación adecuada de este examen.

a. Es un ejemplo de desaceleración c. Es un ejemplo de


variable y ocurre por compresión desaceleraciones tempranas y
del cordón umbilical ocurre por compresión de la
cabeza del feto
b. Este un ejemplo de aceleración y d. Es un ejemplo de desaceleración
usualmente se produce por acción tardía, y ocurre por insuficiencia
de los movimientos fetales uteroplacentaria
____ 58. Paciente de 54 años con obesidad grado 2, nuligesta que presenta hemorragia
posmenopáusica. La biopsia de endometrio muestra hiperplasia endometrial compleja
atípica ¿Cuál es la conducta más adecuada para esta paciente?
a. Conducta expectante c. Dosis alta de estrógenos
b. Dosis altas de progestágenos d. Histerectomía

____ 59. Mujer en edad fértil sin deseo de embarazo, con antecedentes de trastornos
tromboembólicos. ¿Cuál es la mejor alternativa anticonceptiva para ella?
a. Dispositivo intrauterino con T de c. Implantes anticonceptivos
cobre
b. Sistema intrauterino con d. Anticonceptivos orales
Levonorgestrel (LNG - IUS) combinados
____ 60. En el sangrado uterino anormal, que no responde al tratamiento hormonal, se debe
referir a la paciente a un especialista para que realice:
a. Histerectomía vaginal c. Colocación de DIU con liberación
de levonorgestrel
b. Histeroscopia con biopsia d. Ablación endometrial

10
Name: ________________________ ID: A

____ 61. Una mujer de 32 años llega al consultorio porque no ha tenido su menstruación durante
5 meses. Recientemente su marido fue diagnosticado de cáncer por lo que ella se
dedicó a su cuidado. Durante el último año ha perdido 15lbs. El examen físico está
dentro de parámetros normales. BhCG negativa. ¿Cuál es la causa más probable para
la amenorrea secundaria de esta paciente?
a. Hiperprolactinemia c. Aumento de estrógenos y
progesterona
b. Falla ovárica prematura d. Pulsos de GnRH alterados

____ 62. En una paciente en la que se sospecha malignidad en el endometrio, se debe sacar una
muestra para su análisis. Todas las siguientes son indicaciones para este tipo de
procedimiento, EXCEPTO:
a. Presencia de células atípicas c. Cualquier tipo de sangrado
glandulares-endometriales en uterino en una mujer
citología cervical postmenopáusica
b. Presencia de cualquier tipo de d. Monitorización de mujeres con
sangrado uterino anormal en patología endometrial
mujeres menores de 45 años
____ 63. ¿Cuál de los siguientes constituye un dato de certeza de embarazo?
a. Visualización ecográfica del saco c. Falta del ciclo menstrual normal
gestacional
b. Presencia de cloasma a nivel d. Cuantificación de BhCG positiva
facial
____ 64. Paciente femenina de 16 años, acude a consulta por no haber presentado su
menstruación, refiere presencia de dolor pélvico tipo cólico de manera cíclica. Al
examen físico evidencia desarrollo de caracteres sexuales secundarios y sus genitales
externos son de características normales, su diagnóstico es:
a. Amenorrea primaria producida por c. Amenorrea secundaria debido a
un síndrome de Asherman defectos en el desarrollo de los
conductos de Müller
b. Amenorrea secundaría d. Amenorrea primaria a causa de
relacionado a una insensibilidad un himen imperforado
androgénica
____ 65. Un cambio fisiológico del embarazo por liberación de hormonas de la Adenohipófisis es:
a. Aumento de la hormona c. Reducción de hormona de
folicoestlmulante (FSH) crecimiento (GH)
b. Reducción de hormona d. Aumento de la hormona
adrenocorticotropina (ACTH) luteinizante (LH)
____ 66. La opción terapéutica para una paciente con diagnóstico de vaginosis bacteriana es:
a. Doxlcillna 100 mg vía oral dos c. Cotrimazol 100 mg vía oral dosis
veces al día por 14 días única
b. Metronidazol 500 mg vía oral dos d. Albendazol 400 mg vía oral dosis
veces al día por 7 días única

11
Name: ________________________ ID: A

____ 67. Paciente de 20 años llega a la consulta por irreguIaridad en su menstruación. La


anamnesis revela dismenorrea y aumento de peso reciente. A la inspección se nota
hirsutismo en cara, no hay galactorrea ni otras anormalidades. Se realiza un eco
pélvico que encuentra múltiples quistes ováricos. ¿Cómo se llega al diagnóstico?
a. CA 125 positivo c. Únicamente por el Eco pélvico
b. Se debe realizar el índice de d. Clínicamente se utilizan los
HOMA criterios de Rotterdam
____ 68. ¿Cuál de las siguientes indicaciones de manejo es correcta en la Ruptura Prematura de
Membranas en un embarazo a término?
a.Corticoides c. Antibióticos
b.Tocolisis d. Amnioinfusión
____ 69. Una mujer de 16 años llega a la consulta porque no ha menstruado. La anamnesis revela
que no existe crecimiento de vello axilar y púbico, ni desarrollo mamario. Al examen
físico las mamas y el vello púbico están en Tanner 0. ¿Qué es lo siguiente que se debe
hacer?
a. Análisis de cariotipo c. Pedirle que vuelva en un año si
aún no ha menstruado
b. Prueba de BhCG d. Eco pélvico para valorar función
ovárica
____ 70. Los siguientes son criterios de admisión al Centro Obstétrico de una paciente
embarazada en trabajo de parto, EXCEPTO:
a. Contracciones uterinas de 100 c. Contracciones uterinas regulares
mm Hg
b. Borramiento cervical mayor al 50 d. Dilatación de 1 cm
%
____ 71. Mujer de 33 años, gestas 5, para 4, abortos 1. Acude a la consulta a los 4 meses
después del último parto en el que presentó hemorragia importante, con signos de
choque hemodinámico, que precisó transfusión de sangre. Consulta por presentar
sintomatología variada, en la que se incluye: pérdida de peso, caída importante de vello
púbico, disminución significativa del volumen de las mamas, decaimiento, piel seca. Le
preocupa, también, que a pesar de que no pudo dar de lactar, hasta el momento no ha
menstruado. ¿Cuál de los siguientes síndromes es el más probable?
a. Addison c. Sheehan
b. Kallman d. Rokitansky

____ 72. Los siguientes factores son de alto riesgo para diabetes gestacíonal, EXCEPTO:
a. Partos con productos c. IMC mayor a 25kg/m2 antes del
macrosómicos (>4 kilos o embarazo
percentil > a 90)
b. Síndrome de ovario poliquístico d. Glucosuria
____ 73. La utilización de anticonceptivos orales combinados de estrógenos + progestágenos
puede producir los siguientes efectos benéficos, EXCEPTO:
a. Reduce el riesgo de un embarazo c. Reduce el riesgo de cáncer
ectópico endometrial
b. Disminución el riesgo de cáncer d. Mejora de la anemia secundaria a
de cérvix pérdida menstrual abundante

12
Name: ________________________ ID: A

____ 74. Señale lo correcto con respecto a la fase activa del trabajo de parto:
a. El progreso de dilatación es de c. Transcurre entre los 6 y 10 cm de
1.3 cm hora en nulíparas dilatación
b. Se acompaña de dinámica regular d. En las primíparas dura alrededor
de 5 horas
____ 75. Mujer de 22 años, a los 3 meses del nacimiento de su primer hijo, solicita anticoncepción.
¿Cuál método anticonceptivo es el más apropiado para la paciente que quiere
continuar con lactancia materna?
a. Anillo vaginal c. Anticonceptivos orales
combinados
b. Píldora anticonceptiva de d. Anticonceptivos orales de
emergencia progestina sola
____ 76. Mujer de 22 años, nulípara, que deja de menstruar espontáneamente. Ha perdido 9 kg
porque empezó una rigurosa rutina de ejercicios. ¿Cuál es el resultado de laboratorio
esperado?
a. Niveles de prolactína 90ng/ml (N: c. Niveles de LH en 45mlU/ml (N:
menor 20) 6-35)
b. CA 125 de 45 U/ml (N: menor 35) d. Niveles de FSH en 3mlU/ml (N:
5-18)
____ 77. Paciente de 28 años de edad, segunda gestación. Cursa un embarazo de 34 semanas.
Hace 20 horas presenta perdida de líquido por vagina. Al examen físico se aprecia
contracciones uterinas 1 en 10 minutos, frecuencia cardíaca fetal 140 x minuto. Por
examen especular se constata salida de líquido amniótico. Hemoglobina 13g/dl, 10.000
leucocitos (segmentados 60%, linfocitos 40%), VDRL negativo, TP y TTP normales,
200.000 plaquetas. Se realiza tacto vaginal, encontrando: dilatación 4 cm, borramiento
30%, posición central, consistencia firme, cefálico ler plano. Señale la conducta más
adecuada en este caso:
a. Nifedipina 10 mg vía oral cada 6 c. Misoprostol 25 ug sublingual,
horas, Erltromiclna 250 mg vía Ampicilina 2 g intravenoso y luego
oral cada 6 horas 1 g cada 6 horas
b. Oxitocina intravenosa 1-2 d. Nifedipina 10 mg vía oral cada 6
mU/minuto, Eritromicina 250 mg horas, Betametasona 12 mg
vía oral cada 6 horas intramuscular (2 dosis)
____ 78. Indique, ¿Qué método anticonceptivo está contraindicado para una mujer lactante de 25
años en su semana 6ta de posparto?
a. Anticonceptivos orales c. Preservativo masculino
combinados
b. Implante subdérmico de d. Píldoras de progesterona
progesterona
____ 79. En qué momento se debe solicitar urocultivo para tamizaje de bacteriuria asintomática en
una mujer embarazada:
a. Cuando se confirma que la c. Semana 12-16 de embarazo
paciente está embarazada
b. Semana 25-26 de embarazo d. A partir del segundo trimestre

13
Name: ________________________ ID: A

____ 80. Una mujer de 14 años llega a la consulta porque no ha menstruado. La anamnesis
revela que hace un año empezó el crecimiento de vello axilar y púbico y el desarrollo
mamario. Al examen físico las mamas y el vello púbico están en Tanner III. ¿Qué es lo
siguiente que se debe hacer?
a. Eco pélvico para valorar función c. Análisis de cariotipo
ovárica
b. Niveles de FSH y LH para valorar d. Pedirle que vuelva en un año si
función ovárica aún no ha menstruado
____ 81. Mujer de 24 años de edad, embarazada de 15 semanas, acude a la consulta en busca de
tratamiento por presentar secreción vaginal desde hace dos semanas; la paciente
describe la secreción de mal olor como a pescado, de color grisáceo, niega picazón en
esa área. Elija la respuesta más adecuada con relación al diagnóstico y tratamiento:
a. Vaginosis bacteriana y trata con c. Vaginosis bacteriana y trata con
Clindamicina Metronidazol
b. Vaginitis por tricomona y trata con d. Vaginitis por cándida y trata con
Metronidazol Fluconazol
____ 82. Todos los siguientes son factores de riesgo para cáncer de cérvix, EXCEPTO:
a. Inmunosupresión c. Inicio temprano de actividad
sexual
b. Historia de cáncer endometrial u d. Múltiples parejas sexuales
ovárico
____ 83. Mujer de 19 años con vida sexual activa, fecha de última menstruación (FUM) hace 10
días, con ciclos menstruales regulares de 28 días, no usa métodos anticonceptivos;
acude por primera ocasión, por presentar dolor pélvico de moderada intensidad,
acompañado de secreción vaginal purulenta, fiebre y vómito. Señale el diagnóstico de
la paciente:
a. Síndrome de dolor de mitad de c. Enfermedad pélvica inflamatoria
ciclo
b. Endometriosis d. Dismenorrea

____ 84. Una mujer en el tercer trimestre de embarazo refiere tener una secreción vaginal
gris-blanquecina con olor a pescado que no se acompaña de prurito. Se realiza
examen con espéculo y se encuentra una secreción mínima en la vagina y el vestíbulo
sin inflamación valvular. El pH del flujo vaginal es mayor a 4.5 y se evidencia olor a
pescado con KOH al 10%. Posible diagnóstico:
a. Infección por Chlamydia c. Vaginitis por Cándida albicans
trachomatis
b. Vaginosis bacteriana d. Vaginitis por Trichornona vaginalis
____ 85. Paciente da 23 años llega a la consulta preocupada por irregularidad menstrual y
aumento de vello facial para el cual se rasura 2 veces por semana. FUM hace dos días.
Por el momento es sexualmente activa y utiliza condones como método anticonceptivo.
Su IMC es de 26, el resto del examen físico normal. Un eco pélvico revela ovarios con
múltiples quistes. ¿Cuál es el siguiente paso en el manejo?
a. TSH en suero c. Recomendar a la paciente que
intente un método distinto de
depilación
b. Realizar prueba de BhCG d. Iniciar anticonceptivos orales

14
Name: ________________________ ID: A

____ 86. Un hombre de 24 años acude a consulta por presentar una ulcera dolorosa en su pene
desde hace dos semanas; menciona que ha tenido múltiples compañeras sexuales
durante el último año y usa condón ocasionalmente. En el examen físico se puede ver
una ulcera de 1.5 cm, dolorosa a la palpación, de bordes blandos y lisos. Adicional se
acompaña de linfadenopatia en la región inguinal. Se realiza un Gram de la ulcera el
cual muestra cocobacilosgram negativos. Seleccione al microorganismo causante de
esta patología.
a. Haemophilus ducreyi c. Neisseria gonorrhoeae
b. Chlamydia trachomatis d. Treponema pallidum
____ 87. Una mujer embarazada se realiza un urocultivo en la semana 12 donde se aíslan más de
100.000 UFC/ml de un solo uropatógeno. Se puede iniciar tratamiento con uno de los
siguientes antibióticos, EXCEPTO:
a. Cefalosporinas de segunda c. Trimetoprim sulfametoxasol
generación 250-500mg VO cada 6 160mg-800 mg VO cada 12 horas
horas por 7 días por 7 días
b. Fosfomicina 3gr VO dosis única d. Nitrofurantoína 100mg cada 12
por 7 días horas por 7 días
____ 88. Gabriela de 20 años, embarazada de 33 semanas acude por presentar contracciones
uterinas 3 en 10 minutos desde hace 3 horas. Se aprecia dilatación de 2 cm y
herramienta del 30%, con membranas íntegras. A las 30 semanas presentó actividad
uterina por lo que recibió tratamiento tocolítico con nifedipina y 2 dosis de
betametasona 12 mg IM. Utilizando la siguiente lista de tratamientos, escoja el
esquema terapéutico más adecuado en este caso:
1. Nifedipina 10 mg PO cada 20 minutos por 3 dosis y luego cada 6 horas
2. Indometacina 100 mg vía rectal cada 8 horas
3. Betametasona 12 mg intramuscular cada 24 horas (2 dosis)
4. Ampicilina 2 g IV cada 6 horas

a. 1, 3, 4 c. 1, 3
b. 2, 3 d. 1
____ 89. En una mujer con diabetes gestacional, cuando el crecimiento fetal es  al percentil 90 se
recomienda que el control de la glucosa sanguínea en ayuno sea de:
a. Menor a 130mg/dl c. Menor a 90mg/dl
b. Menor a 220mg/dl d. Menor a 80mg/dl

____ 90. Indique, ¿Qué método anticonceptivo está contraindicado para una mujer lactante de 25
años en su semana 6ta de posparto?
a. Implante subdérmico de c. Píldoras de progesterona
progesterona
b. Anticonceptivos orales d. Preservativo masculino
combinados
____ 91. Todos los siguientes son factores de riesgo para cáncer de cérvix. EXEPTO:
a. Historia de cáncer endometrial u c. Inmunosupresión
ovárico
b. Inicio temprano de actividad d. Múltiples parejas sexuales
sexual

15
Name: ________________________ ID: A

____ 92. Los siguientes factores son de alto riesgo para diabetes gestacional, EXCEPTO:
a. Partos con productos c. IMC mayor a 25kg/m2 antes del
macrosómicos (>4 kilos o embarazo
percentil > a 90)
b. Síndrome de ovario poliquístico d. Glucosuria
____ 93. Gabriela de 20 años, embarazada de 33 semanas acude por presentar contracciones
uterinas 3 en 10 minutos desde hace 3 horas. Se aprecia dilatación de 2 cm y
borramiento del 30%, con membranas integras. A las 30 semanas presentó actividad
uterina por lo que recibió tratamiento tocolítico con nifedipina y 2 dosis de
betametasona 12 mg IM. Utilizando la siguiente lista de tratamientos, escoja el
esquema terapéutico más adecuado en este caso:
1. Nifedipina 10 mg PO cada 20 minutos por 3 dosis y luego cada 6 horas
2. Indometacina 100 mg vía rectal cada 8 horas
3. Betametasona 12 mg intramuscular cada 24 horas (2 dosis)
4. Amplclllna 2 g IV cada 6 horas

a. 1 c. 1, 3
b. 2, 3 d. 1, 3, 4
____ 94. Mujer de 22 años, nulípara, que deja de menstruar espontáneamente. Ha perdido 9 kg
porque empezó una rigurosa rutina de ejercicios. ¿Cuál es el resultado de laboratorio
esperado?
a. Niveles de LH en 45mlU/ml (N: c. Niveles de FSH en 3mlU/ml (N:
6-35) 5-18)
b. Niveles de prolactina 90ng/ml (N: d. CA 125 de 45 U/ml (N: menor 35)
menor 20)
____ 95. Señale lo correcto con respecto a la fase activa del trabajo de parto:
a. Transcurre entre los 6 y 10 cm de c. El progreso de dilatación es de
dilatación 1.3 cm hora en nulíparas
b. En las primíparas dura alrededor d. Se acompaña de dinámica regular
de 5 horas
____ 96. ¿Cuál de las siguientes indicaciones de manejo es correcta en la Ruptura Prematura de
Membranas en un embarazo a término?
a. Antibióticos c. Tocolisis
b. Amnioinfusión d. Corticoides

16
Name: ________________________ ID: A

____ 97. Paciente de 28 años de edad, segunda gestación. Cursa un embarazo de 34 semanas.
Hace 20 horas presenta perdida de líquido por vagina. Al examen físico se aprecia
contracciones uterinas 1 en 10 minutos, frecuencia cardíaca fetal 140 x minuto. Por
examen especular se constata salida de líquido amniótico. Hemoglobina 13g/dl, 10.000
leucocitos (segmentados 60%, linfocitos 40%), VDRL negativo, TP y TTP normales,
200.000 plaquetas. Se realiza tacto vaginal, encontrando: dilatación 4 cm, borramiento
30%, posición central, consistencia firme, cefálico ler plano. Señale la conducta más
adecuada en este caso:
a. Misoprostol 25 ug sublingual, c. Oxitocina intravenosa 1-2
Ampicilina 2 g intravenoso y luego mU/mínuto, Eritromicina 250 mg
1 g cada 6 horas vía oral cada 6 horas
b. Nifedipina 10 mg vía oral cada 6 d. Nifedipina 10 mg vía oral cada 6
horas, Eritromicina 250 mg vía horas, Betametasona 12 mg
oral cada 6 horas intramuscular (2 dosis)
____ 98. La opción terapéutica para una paciente con diagnóstico de vaginosis bacteriana es:
a. Metronidazol 500 mg vía oral dos c. Cotrimazol 100 mg vía oral dosis
veces al día por 7 días única
b. Doxicilina 100 mg vía oral dos d. Albendazol 400 mg vía oral dosis
veces al día por 14 días única
____ 99. En una mujer con diabetes gestacional, cuando el crecimiento fetal es  al percentil 90 se
recomienda que el control de la glucosa sanguínea en ayuno sea de:
a. Menor a 130mg/dl c. Menor a 90mg/dl
b. Menor a 120mg/dl d. Menor a 80mg/dl

____ 100. Mujer de 33 años, gestas 5, para 4, abortos 1. Acude a la consulta a los 4 meses
después del último parto en el que presentó hemorragia importante, con signos de
choque hemodinámico, que precisó transfusión de sangre. Consulta por presentar
sintomatología variada, en la que se incluye: pérdida de peso, caída importante de vello
púbico, disminución significativa del volumen de las mamas, decaimiento, piel seca. Le
preocupa, también, que a pesar de que no pudo dar de lactar, hasta el momento no ha
menstruado. ¿Cuál de los siguientes síndromes es el más probable?
a. Sheehan c. Addison
b. Kallman d. Rokltansky

____ 101. La utilización de anticonceptivos orales combinados de estrógenos + progestágenos


puede producir los siguientes efectos benéficos, EXCEPTO:
a. Reduce el riesgo de cáncer c. Disminución el riesgo de cáncer
endometrial de cérvix
b. Mejora de la anemia secundaria a d. Reduce el riesgo de un embarazo
pérdida menstrual abundante ectópico
____ 102. Los siguientes son criterios de admisión al Centro Obstétrico de una paciente
embarazada en trabajo de parto, EXCEPTO:
a. Dilatación de 1 cm c. Contracciones uterinas regulares
b. Borramiento cervical mayor al 50 d. Contracciones uterinas de 100
% mm Hg

17
Name: ________________________ ID: A

____ 103. Paciente de 20 años liega a la consulta por irregularidad en su menstruación. La


anamnesis revela dismenorrea y aumento de peso reciente. A la inspección se nota
hirsutismo en cara, no hay galactorrea ni otras anormalidades. Se realiza un eco
pélvico que encuentra múltiples quistes ováricos. ¿Cómo se llega al diagnóstico?
a. Únicamente por el Eco pélvico c. Clínicamente se utilizan los
criterios de Rotterdam
b. CA 125 positivo d. Se debe realizar el índice de
HOMA
____ 104. Un cambio fisiológico del embarazo por liberación de hormonas de la Adenohipófisis es:
a. Reducción de hormona de c. Aumento de la hormona
crecimiento (GH) folicoestimulante (FSH)
b. Reducción de hormona d. Aumento de la hormona
adrenocorticotropina (ACTH) luteinizante (LH)
____ 105. Mujer de 22 años, a los 3 meses del nacimiento de su primer hijo, solicita anticoncepción.
¿Cuál método anticonceptivo es el más apropiado para la paciente que quiere
continuar con lactancia materna?
a. Anillo vaginal c. Píldora anticonceptiva de
emergencia
b. Anticonceptivos orales de d. Anticonceptivos orales
progestina sola combinados
____ 106. Mujer de 24 años de edad, embarazada de 15 semanas, acude a la consulta en busca de
tratamiento por presentar secreción vaginal desde hace dos semanas; la paciente
describe la secreción de mal olor como a pescado, de color grisáceo, niega picazón en
esa área. Elija la respuesta más adecuada con relación al diagnóstico y tratamiento.
a. Vaginitis por cándida y trata con c. Vaginosis bacteriana y trata con
Fluconazol Clindamicina
b. Vaginosis bacteriana y trata con d. Vaginitis por tricomona y trata con
Metronidazol Metronidazol
____ 107. Mujer de 19 años con vida sexual activa, fecha de última menstruación (FUM) hace 10
días, con ciclos menstruales regulares de 28 días, no usa métodos anticonceptivos;
acude por primera ocasión, por presentar dolor pélvico de moderada intensidad,
acompañado de secreción vaginal purulenta, fiebre y vómito. Señale el diagnóstico de
la paciente:
a. Dismenorrea c. Síndrome de dolor de mitad de
ciclo
b. Enfermedad pélvica inflamatoria d. Endometriosis

____ 108. ¿Cuál de los siguientes constituye un dato de certeza de embarazo?


a. Presencia de cloasma a nivel c. Cuantificación de BhCG positiva
facial
b. Falta del ciclo menstrual normal d. Visualización ecográfica del saco
gestacional

18
Name: ________________________ ID: A

____ 109. Una mujer embarazada se realiza un urocultivo en la semana 12 donde se aíslan más
de 100.000 UFC/ml de un solo uropatógeno. Se puede iniciar tratamiento con uno de
los siguientes antibióticos, EXCEPTO:
a. Trimetoprim sulfametoxasol c. Cefalosporinas de segunda
160mg-800mg VO cada 12 horas generación 250-500mg VO cada 6
por 7 días horas por 7 días
b. Nitrofurantoína 100mg cada 12 d. Fosfomicina 3gr VO dosis única
horas por 7 días por 7 días
____ 110. En qué momento se debe solicitar urocultivo para tamizaje de bacteriuria asintomática en
una mujer embarazada:
a. A partir del segundo trimestre c. Semana 25-26 de embarazo
b. Semana 12-16 de embarazo d. Cuando se confirma que la
paciente está embarazada
____ 111. Paciente de 23 años llega a la consulta preocupada por irregularidad menstrual y
aumento de vello facial para el cual se rasura 2 veces por semana. FUM hace dos días.
Por el momento es sexualmente activa y utiliza condones como método anticonceptivo.
Su IMC es de 26, el resto del examen físico normal. Un eco pélvico revela ovarios con
múltiples quistes. ¿Cuál es el siguiente paso en el manejo?
a. TSH en suero c. Iniciar anticonceptivos orales
b. Realizar prueba de BhCG d. Recomendar a la paciente que
intente un método distinto de
depilación
____ 112. Una mujer en el tercer trimestre de embarazo refiere tener una secreción vaginal
gris-blanquecina con olor a pescado que no se acompaña de prurito. Se realiza
examen con espéculo y se encuentra una secreción mínima en la vagina y el vestíbulo
sin inflamación valvular. El pH del flujo vaginal es mayor a 4.5 y se evidencia olor a
pescado con KOH al 10%. Posible diagnóstico:
a. Vaginosis bacteriana c. Infección por Chlamydia
trachomatis
b. Vaginitis por Cándida albicans d. Vaginitis por Trichomona vaginalis

____ 113. Una mujer de 32 años llega al consultorio porque no ha tenido su menstruación durante 5
meses. Recientemente su marido fue diagnosticado de cáncer por lo que ella se dedicó
a su cuidado. Durante el último año ha perdido 15lbs. El examen físico está dentro de
parámetros normales. BhCG negativa. ¿Cuál es la causa más probable para la
amenorrea secundaria de esta paciente?
a. Aumento de estrógenos y c. Pulsos de GnRH alterados
progesterona
b. Hiperprolactinemia d. Falla ovárica prematura

____ 114. En una paciente en la que se sospecha malignidad en el endometrio se debe sacar una
muestra para su análisis. Todas las siguientes son indicaciones para este tipo de
procedimiento, EXCEPTO:
a. Cualquier tipo de sangrado c. Presencia de células atípicas
uterino en una mujer glandulares-endometriales en
postmenopáusica citología cervical
b. Presencia de cualquier tipo de d. Monitorización de mujeres con
sangrado uterino anormal en patología endometrial
mujeres menores de 45 años

19
Name: ________________________ ID: A

____ 115. Paciente femenina de 16 años, acude a consulta por no haber presentado su
menstruación, refiere presencia de dolor pélvico tipo cólico de manera cíclica. Al
examen físico evidencia desarrollo de caracteres sexuales secundarios y sus genitales
externos son de características normales, su diagnóstico es:
a. Amenorrea secundaria debido a c. Amenorrea secundaría
defectos en el desarrollo de los relacionado a una insensibilidad
conductos de Müller androgénica
b. Amenorrea primaria producida por d. Amenorrea primaria a causa de
un síndrome de Asherman un himen imperforado
____ 116. Una mujer de 16 años llega a la consulta porque no ha menstruado. La anamnesis revela
que no existe crecimiento de vello axilar y púbico, ni desarrollo mamario. Al examen
físico las mamas y el vello púbico están en Tanner 0. ¿Qué es lo siguiente que se debe
hacer?
a. Eco pélvico para valorar función c. Pedirle que vuelva en un año si
ovárica aún no ha menstruado
b. Prueba de BhCG d. Análisis de cariotipo

____ 117. Una mujer de 14 años llega a la consulta porque no ha menstruado. La anamnesis revela
que hace un año empezó el crecimiento de vello axilar y púnico y el desarrollo
mamario. Al examen físico las mamas y el vello púbico están en Tanner III. ¿Qué es lo
siguiente que se debe hacer?
a. Niveles de FSH y LH para valorar c. Eco pélvico para valorar función
función ovárica ovárica
b. Pedirle que vuelva en un año si d. Análisis de cariotipo
aún no ha menstruado
____ 118. Un hombre de 24 años acude a consulta por presentar una ulcera dolorosa en su pene
desde hace dos semanas; menciona que ha tenido múltiples compañeras sexuales
durante el último año y usa condón ocasionalmente. En el examen físico se puede ver
una ulcera de 1.5 cm, dolorosa a la palpación, de bordes blandos y lisos. Adicional se
acompaña de linfadenopatia en la región inguinal. Se realiza un Gram de la ulcera el
cual muestra cocobacilosgram negativos. Seleccione al microorganismo causante de
esta patología.
a. Chlamydia trachomatis c. Neisseria gonorrhoeae
b. Haemophilus ducreyi d. Treponema pallidum

____ 119. En el embarazo los cambios fisiológicos como sensación de ardor estomacal y la pirosis
se atribuyen a:
a. Presiones intraesofágicas más c. Mayor velocidad de vaciamiento
altas e intragástricas más bajas gástrico
b. Hipotonía del esfínter esofágico d. Cambio de posición del estómago
inferior en el primer trimestre
____ 120. Una paciente nuligesta presenta ciclos menstruales regulares con FUM hace 7 días, al
examen físico tiene dolor anexial bilateral, la movilización del cuello uterino y palpación
directa de abdomen son dolorosas, adicionalmente tiene 38.5 grados C y leucocitosis
de 12 000 / ml. ¿Cuál es el diagnóstico?
a. Infección de vías urinarias c. Endometriosis pélvica
b. Ruptura de quiste del cuerpo lúteo d. Enfermedad pélvica inflamatoria

20
Name: ________________________ ID: A

____ 121. Respecto a las claves obstétricas, la toma de muestra de lactato sérico corresponde a:
a. CLAVE NARANJA para manejo c. CLAVE AMARILLA para manejo
de traumatismo obstétrico de choque séptico obstétrico
b. CLAVE ROJA para manejo de d. CLAVE AZUL para manejo
hemorragia obstétrica trastornos hipertensivos
obstétricos
____ 122. Paciente de 32 años de edad, sin antecedentes personales de importancia, G4 P3,
acude al Hospital Gíneco-Obstétrico Isidro Ayora con un embarazo de 32 semanas,
cefalea, TA de 150/120, edema en piernas, cara y manos, ROTS 3/5. Sus indicaciones
de ingreso a la paciente incluyen los siguientes medicamentos. EXCEPTO:
a. Betametasona 12 mg IM 2 dosis c. Sulfato de magnesio IV 6 g de
cada 24 horas carga y 1 g/hora mantenimiento
b. Nifedipina 10 mg VO cada 20 d. Hidralazina IV bolo de 5 mg stat
minutos
____ 123. ¿Qué mecanismo fisiológico en el embarazo produce incremento de la excreción urinaria
de proteínas?
a. Disminución de la tasa de c. Disminución de la reabsorción
filtración glomerular tubular de las proteínas filtradas
b. Disfunción endotelial glomerular d. Alteración del sistema
renina-angiotensina-aldosterona
____ 124. Respecto al protocolo de manejo en la Clave Roja obstétrica, todas las acciones son
correctas. EXCEPTO:
a. Asegurar vía aérea y oxígeno c. Asegurar dos accesos venosos
suplementario para conseguir con catéter No. 16 e infusión
saturación mayor a 85% rápida de líquidos calientes
b. Realizar examen físico completo d. Vaciar vejiga y colocar sonda
que incluya signos vitales vesical a drenaje con bolsa de
completos y nivel de conciencia recolección
____ 125. Señale la afirmación correcta en relación con el cáncer de cuello uterino:
a. El sangrado uterino anormal se c. El piometra está presente en más
observa en la mayoría de las de la mitad de las pacientes
pacientes
b. La secreción vaginal d. En etapas iniciales suele producir
serosanguinolienta es infrecuente sintomatología urinaria y rectal
____ 126. Se recomienda un manejo expectante en embarazos con ruptura prematura de
membranas entre las siguientes semanas de gestación:
a.Menor de 20 semanas c. 20-24 semanas
b.24-34.6 semanas d. Mayor 35 semanas

____ 127. La velocidad de ganancia de peso, promedio, por semana (kg/sem), en una embarazada
durante el segundo trimestre con un IMC pregestacional normal es de:
a. 0.51 Kg/sem c. 0.22 Kg /sem
b. 0.42 Kg/sem d. 0.28 Kg /sem

21
Name: ________________________ ID: A

____ 128. Mujer de 18 años llega a la consulta por taita de menstruación. La anamnesis revela un
desarrollo de mamas normal per© poco vello púbico y axilar. Al examen físico las
mamas están en Tanner 4 y el vello púbico están en Tanner 1. ¿Cuál es el diagnóstico
más probable?
a. Malformación de conductos de c. Síndrome de Kallman
Müller
b. Retraso fisiológico d. Insensibilidad completa a los
andrógenos
____ 129. La infección vaginal que se manifiesta clínicamente con prurito, eritema y signos
inflamatorios de la vulva es:
a. Tricomoniasis c. Candidiasis
b. Herpes genital d. Vaginosis bacteriana
____ 130. El incremento total de peso recomendado (Kg) al final del embarazo gemelar y con un
IIV3C normal pregestacional es de:
a. 17-25 Kg c. 14-23 Kg
b. 11-19 Kg d. por lo menos 23 Kg
____ 131. En la Clave Roja obstétrica en relación a la administración de uterotónicos. ¿Cuál es eí
criterio correcto?
a. Oxitocina 10 Ul/ml IM y 20-40 Ul c. Ergonovina 0.5 mg IM, se puede
en 1 000 cc de solución cristaloide repetir cada 2 a 4 horas, máximo
en infusión IV a 250 rnl/h 5 dosis (2.5 mg) en un período de
24 horas
b. Oxitocina 10 Ul/ml IM y 60-80 Ul d. Ergonovina 0.7 mg IM, se puede
en 1 000 cc de solución cristaloide repetir cada 2 a 4 horas, máximo
en infusión IV a 250 ml/h 5 dosis (3.5 mg) en un período de
24 horas
____ 132. Una paciente tiene una tensión arterial de 160/95 mmHg. al momento del parto y a las 12
semanas posteriores 150/90 mmHg. ¿Cuál es el diagnóstico?
a. Hipertensión crónica c. Eclampsia
b. Hipertensión gestacional d. Preeclampsia

____ 133. El mecanismo correcto de acción de los anticonceptivos a base de progesterona es:
a. Provocan un espesamiento del c. Aumentan la motilidad de las
moco cervical trompas de Falopio
b. Generan un endometrio secretor d. Evitan la ovulación al suprimir
liberación de FSI-I
____ 134. Los siguientes son factores de riesgo para presentar parto prematuro. EXCEPTO:
a. Malformaciones congénitas en el c. Hipertensión gestacional
feto
b. Antecedente de parto pretérmino d. Rotura prematura de membranas
____ 135. Paciente de 25 años llega a la consulta porque después de un legrado por aborto
espontáneo hace 4 meses no ha menstruado. ¿Cuál es el diagnóstico?
a. Endometriosis c. Síndrome de Asherman
b. Estenosis cervical d. Síndrome de Sheehan

22
Name: ________________________ ID: A

____ 136. Primigesta de 16 años, con 38 semanas de gestación, sin antecedentes patológicos,
acude a urgencias de maternidad por referir escotomas y dolor a nivel de epigastrio.
Examen físico presenta cifras tensionales TAS 170mmHg, TAD HOOmmHg. Exámenes
con plaquetas 75 000 pl, ereatinina 1.8mg/dl, AST 38 Ul/L, ALT 112 Ul/L. ¿Cuál es el
diagnóstico de la paciente?
a. Hipertensión gestacional c. Hipertensión crónica más
preeclampsia sobreañadida
b. Preeclampsia sin signos de d. Preeclampsia con signos de
gravedad (leve) gravedad (grave)
____ 137. Una mujer de 17 años llega a la consulta preocupada ya que todas sus amigas han
menstruado menos ella. Durante la anamnesis se queja también de no percibir olores.
El examen físico demuestra una ciara faifa de desarrollo de caracteres sexuales
secundarios. ¿Cuál es el diagnóstico?
a. Retraso fisiológico c. Síndrome de Kallman
b. Síndrome de Turner d. Disgenesia gonadal pura

____ 138. En las maniobras de Leopold, la situación del feto hace referencia a:
a. La relación del dorso del feto con c. La relación entre el polo fetal que
la parte derecha o izquierda de la se pone en contacto con el
madre estrecho superior de la pelvis
b. La relación del eje longitudinal del d. La relación de la fontanela
feto con el eje longitudinal de la posterior del feto con la pelvis
madre
____ 139. ¿Cuál es una contraindicación absoluta para el uso de anticonceptivos orales
combinados?
a. Historia de trombosis c. Historia de cáncer de cérvix
b. Historia de dismenorrea d. Historia de cefalea tensional
____ 140. Indique. ¿Cuál de los siguientes anticonceptivos está contraindicado en una paciente con
enfermedad pélvica inflamatoria aguda?
a. Preservativo masculino c. Anticonceptivos orales derivados
de progesterona
b. Anticonceptivos orales d. Dispositivo intrauterino de cobre
combinados
____ 141. En el manejo de una Clave Roja obstétrica en una paciente con sangrado genital que
presenta choque severo. Señale el criterio correcto:
a. Transfundir 1 concentrado de c. Transfundir 1 concentrado de
glóbulos rojos previa realización glóbulos rojos previa realización
de pruebas cruzadas, y en caso de pruebas cruzadas, y en caso
de no disponer pruebas cruzadas de no disponer pruebas cruzadas
se colocará 1 unidad de ORH se colocará 1 unidad de plasma
Negativo ORH Positivo
b. Transfundir 2 concentrados de d. Transfundir 2 concentrados de
glóbulos rojos previa realización glóbulos rojos previa realización
de pruebas cruzadas, y en caso de pruebas cruzadas, y en caso
de no disponer pruebas cruzadas de no disponer pruebas cruzadas
se colocará 2 unidades de plasma se colocará 2 unidades de ORH
ORH Positivo Negativo

23
Name: ________________________ ID: A

____ 142. Llega a la consulta una mujer de 27 años con un retraso en su menstruación de 2
meses. La anamnesis revela un aumento de 5 kg durante 6 meses anteriores (IMC 27),
sequedad de piel, intolerancia al frío e irritabilidad. Durante el examen físico no se
evidencias signos alarmantes. Una prueba de BhCG es negativa. Con su posible
diagnóstico según la clínica. ¿Cuál es el tratamiento?
a. Bromocriptina c. Pastillas anticonceptivas orales
b. Levotiroxina d. Metformina

____ 143. Los siguientes son cambios hemodinámicos fisiológicos del embarazo. EXCEPTO:
a. Reducción de la resistencia c. Aumento del gasto cardiaco
vascular sistémica
b. Disminución del volumen d. Reducción de la presión arterial
sanguíneo
____ 144. En la Clave Azul obstétrica el manejo correcto de impregnación con sulfato de magnesio
para la prevención de las convulsiones es:
a. Administrar 2 ampollas 20 % IM c. Administrar 2 ampollas 20 % IV de
de SO4Mg con un contenido total SO4Mg + 80 mi solución isotónica
de 4 gramos. a 101 gotas/minuto
b. Administrar 1 gramo/hora IM de d. Administrar 5 ampollas al 20 % +
SO4MG en ampollas al 20 % 400 ml de solución isotónica
pasar a 50 ml/hora
____ 145. Paciente de 21 años con embarazo de 28 semanas por FUMl, refiere fiebre, dolor
costovertebral, náuseas y vómito de 48 horas de evolución. El EMO presenta piuría y
hematuria. ¿Qué diagnóstico tiene la paciente y qué estudio debe solicitar?
a. Bacteriuria asintomática, solicitar c. Pielonefritis, solicitar cultivo y
hemocultivo antibiograma de orina
b. Cistitis aguda, solicitar d. Litiasis renal, solicitar urografía
ultrasonografía renal con contraste
____ 146. Mujer de 17 años consulta por falta de menstruaciones normales. La anamnesis revela
menarquia a los 14 años, la paciente realiza 3 horas de ejercicio diario. Tiene un IMC
de 17.5 y caracteres sexuales secundarios normales. El eco pélvico no revela ninguna
anormalidad. ¿Cuál es el siguiente paso a seguir?
a. La paciente debe dejar de hacer c. Medir niveles de TSH
tanto deporte y/o aumentar su
ingesta calórica
b. Dosificación de CA 125 d. Aplicar los criterios de Rotterdam

____ 147. Una mujer de 16 años llega a la consulta por un chequeo rutinario. La anamnesis revela
amenorrea primaria, ella dice que ha aumentado la cantidad de ejercicio que realiza.
Durante el examen físico se encuentra un IMC de 21, desarrollo mamario Tanner
estadio 1 y FSH elevado. La prueba de BhCG fue negativa. ¿Cuál es el siguiente paso?
a. Realizar un cariotipo c. Pedirle que disminuya las horas
de ejercicio que realiza y que
regrese en 2 meses
b. Medir niveles de TSH d. Prescribir anticonceptivos orales

24
Name: ________________________ ID: A

____ 148. Usted se encuentra en un hospital de tercer nivel de atención y recibe una paciente de
24 años con un embarazo de 33 semanas más ruptura prematura de membranas
confirmada, sin dinámica uterina. ¿Cuál es el protocolo a seguir?
a. Hospitalización, reposo en cama, c. Profilaxis antibiótica, reposo en
uso de tocolíticos y uso de cama, maduración pulmonar y
antibióticos enviar a su casa
b. Hospitalización, reposo en cama, d. Uso de antibiótico, maduración
profilaxis antibiótica y maduración pulmonar, uso de tocolíticos y
pulmonar referir a otro centro asistencial
____ 149. En una paciente con Clave Azul obstétrica. ¿Cuál es un signo inminente de intoxicación
por sulfato de magnesio y su manejo médico?
a. La desaparición de reflejos c. El aumento de las contracciones
rotulianos, gluconato de calcio al uterinas, administrar gluconato de
10% calcio al 20%
b. El aumento de la presión arterial. d. El aumento del reflejo rotuliano,
Administrar nifedipina vía oral administrar hidralazina vía
parenteral
____ 150. ¿Cuál es el examen de laboratorio más útil y de fácil acceso para evaluar deficiencia de
hierro en el embarazo?
a. Hemoglobina c. Ferritina sérica
b. Transferrina sérica d. Hematocrito
____ 151. ¿Cuál es el enunciado correcto en la infección urinaria en el embarazo?
a. Es de elección el Trimetroprim c. Los cambios anatómicos y
Sulfametoxazol por sus mínimos fisiológicos en la gestación
efectos adversos predisponen a infecciones
urinarias
b. Las infecciones urinarias en el d. Embarazada sin síntomas de
embarazo se clasifican en: cistitis infección urinaria, no es necesario
y pielonefritis la realización de urocultivo
____ 152. En un embarazo pretérmino, que cursa con acortamiento de la longitud cervical y
contracciones uterinas, el fármaco de primera elección para maduración fetal es:
a. Betametasona 12 mg IM cada 12 c. Dexametasona 6 mg IM cada 12
horas, 2 dosis horas, 2 dosis
b. Dexametasona 6 mg IM cada 24 d. Betametasona 12 mg IM cada 24
horas, 2 dosis horas, 2 dosis
____ 153. Paciente de 28 años, primigesta, con embarazo de 39 semanas, al tacto vaginal presenta
cuello uterino de consistencia blanda, posición anterior, con dilatación de 3 cm,
borramiento del 60% y la presentación del feto OIIA se encuentra en el III plano de
Hodge (localización -1). ¿A qué índice de Bishop corresponde e indique si es favorable
o desfavorable para inducción de trabajo de parto?
a. 3 puntos, desfavorable c. 10 puntos, favorable
b. 2 puntos, favorable d. 8 puntos, desfavorable

25
Name: ________________________ ID: A

____ 154. Paciente femenino diagnosticada con verrugas genitales externas. ¿Cuál es el
tratamiento aplicado por el médico?
a. Podofilox 5 % una vez al mes c. Imiquimod 5 % cada noche por 5
semanas
b. Resina de podofilina al 10-25 % d. Sinecatequina, al 15% 3 veces a
cada semana la semana
____ 155. Complete el siguiente enunciado: Una de las contraindicaciones para el uso del
dispositivo intrauterino de liberación de levonorgestrel (LNG-IUS) es presentando una
tasa de expulsión de
a. Metrorragia de origen c. La existencia de leiomiomas que
desconocido, 20 % no deformen la cavidad uterina,
30 %
b. Mujeres infectadas por el virus de d. La existencia de leiomiomas que
inmunodeficiencia humana (VIH), deformen la cavidad del útero, 10
50% %
____ 156. ¿Cuál es el criterio clínico más importante para establecer el diagnóstico de metritis
puerperal?
a. Fiebre c. Dolor abdominal a la palpación
b. Escalofrío d. Loquios fétidos
____ 157. Mujer de 24 años, primigesta con 28 semanas de gestación, acude con una presión
arterial de 160/120 mm Hg y proteinuria 3+, con visión borrosa, la exploración a las 6
horas arroja los mismos resultados. Indique cuál es su diagnóstico:
a. Eclampsia c. Hipertensión Gestacional
b. Preeclampsia leve d. Preeclampsia grave

____ 158. La forma de prescripción de dosis de impregnación de Sulfato de Magnesio, como


tratamiento para prevenir la eclampsia es:
a. 6 gramos de Sulfato de c. 3 gramos de Sulfato de
Magnesio, vía intravenosa Magnesio, vía intravenosa
a pasar en 20 minutos. a pasar en 20 minutos
b. 4 gramos de Sulfato de d. 8 gramos de Sulfato de
Magnesio, vía intravenosa Magnesio, vía intravenosa
a pasar en 20 minutos. a pasar en 10 minutos
____ 159. Con respecto al uso del dispositivo intrauterino T de cobre como método de planificación
familiar, señale cuál es su contraindicación de uso
a. Tromboflebitis o trastornos c. Mujer fumadora mayor de 35 años
tromboembólicos
b. Metrorragia de origen d. Hepatopatía
desconocido
____ 160. El orden sincrónico de los movimientos cardinales del feto en el momento del parto es:
1. Extensión y rotación externa.
2. Flexión y rotación interna.
3. Expulsión y extracción.
4. Encajamiento y descenso.

a. 3, 2, 1, 4 c. 1, 2, 3, 4
b. 4, 2, 1, 3 d. 4, 3, 2, 1

26
Name: ________________________ ID: A

____ 161. Seleccione todos los parámetros que evalúa el índice de BISHOP:
1. Dilatación cervical.
2. Variabilidad de la frecuencia cardíaca fetal.
3. Altura uterina
4. Altura de la presentación fetal.
5. intensidad de las contracciones uterinas.
6. Consistencia cervical.
7. Posición de la presentación fetal.
8. Borramiento cervical.

a. 3, 4, 5, 6, 7 c. 1, 4, 6, 7, 8
b. 2, 3, 4, 5, 7 d. 1, 2, 3, 6, 8
____ 162. Una vez que la hemorragia postparto inmediata ha sido identificada, indique el
tratamiento farmacológico de primera opción y la dosis correcta.
a. Misoprostol, 200 mcg VO, 1er c. Acido tranexámico 500mg
minuto posparto, 300 mcg cada intravenoso a pasar diluido en 200
hora por 6 horas ml de solución fisiológica y luego
500 mg intramuscular cada 8
horas
b. Oxitocina 10UI/mL IM (o 5 Ul IV d. Ergonovina 0,2 mg IM, 1er minuto,
lento), o 20-40 Ul en 1000 mL de Ergonovina repetir cada 2 a 4
solución cristaloide en infusión IV horas, máximo 5 dosis (1 mg) en
a 250 ml/h un período de 24 horas
____ 163. Indique la familia a la cual pertenece el virus que produce el molusco contagioso y su
tiempo de incubación
a. C. trachomatis, 1 a 2 semanas c. Virus del papiloma humano, 8 a
12 semanas
b. Herpesviridae, de 3 a 5 semanas d. Poxviridae, 2 a 7 semanas
____ 164. Gimnasta de 17 años, nuligesta, acude a la consulta por presentar falta de periodo
menstrual. Con prueba de embarazo negativa. En la anamnesis revela prácticas físicas
extenuantes de 6 horas diarias, al examen físico IMC (índice de masa corporal) 17
¿Cuál es la causa de la amenorrea secundaria?
a. Hiperplasia suprarrenal congénita c. Defecto de los conductos de
Mullen
b. Función de la FSH (hormona d. Síndrome de Asherman
folículo estimulante) reducida
____ 165. Paciente femenino de 18 años, sin antecedentes patológicos de importancia,
sexualmente activa acude a la consulta por presentar secreción vaginal, al examen
físico se detecta cervicitis con secreción vaginal abundante que es inodora, no irritante
y de color amarillento. ¿Cuál es el diagnóstico y tratamiento médico adecuado?
a. Vaginosis bacteriana - Clotrimazol c. Infección por herpes - Valaciclovir
b. Infección por hongos - d. Infección por gonorrea -
Azitromicina Doxiciclina

27
Name: ________________________ ID: A

____ 166. En una embarazada a término. ¿Con qué maniobra de Leopold es posible determinar la
orientación del feto?
a. Quinta c. Segunda
b. Tercera d. Primera
____ 167. Complete el siguiente enunciado: Los diámetros oblicuos del plano de entrada de la
pelvis se extienden desde las____________a ______________
a. Sincondrosis sacroiliacas, borde c. Sindesmosis sacroiliacas,
lateral de sínfisis del pubis eminencia iliopectínea
contralateral
b. Sindesmosis sacroiliacas, espina d. Sincondrosis sacroiliacas,
ciática contralateral eminencia iliopectínea ipsilateral
____ 168. El enunciado correcto en referencia al parto pretérmino es:
a. La proteína C reactiva en frotis c. El incremento del pH vaginal
vaginal previene el riesgo de parto previene el riesgo de parto
pretérmino pretérmino
b. Las modificaciones cervicales d. El uso de progesterona aumenta
valoradas por ecografía, son un la incidencia de parto pretérmino
marcador predictor
____ 169. En la pelvis materna los límites del estrecho superior incluyen:
a. Atrás limitado por el promontorio c. A los lados limitado por las alas
del sacro y a los lados por la del sacro y adelante por la cresta
cresta pectínea pectínea
b. Atrás limitado por la cresta d. A los lados limitado por
pectínea y adelante por la sínfisis promontorio del sacro y adelante
del pubis por la sínfisis del pubis
____ 170. ¿Cuál es el esquema de tratamiento del ácido fólico en una mujer con epilepsia tratada
con anticonvulsivantes que este planificando un embarazo?
a. 5 microgramos vía oral, al día, c. 5 miligramos vía oral, al día, hasta
hasta la semana 5 de gestación las 12 semanas de gestación
b. 2 miligramos vía oral, tres veces al d. 2 miligramos vía parenteral, a la
día, hasta la 40 semanas de semana, hasta 20 semanas de
gestación gestación
____ 171. Paciente gestante a término con reporte ecográfico de placenta adyacente al orificio
cervical interno sin sobrepasarlo. ¿Qué tipo de anomalía de inserción placentaria
presenta esta paciente?
a. Placenta previa parcial c. Placenta previa marginal
b. Implantación baja de la placenta d. Placenta previa oclusiva total

28
Name: ________________________ ID: A

____ 172. En la mayoría de las pacientes con embarazos avanzados por encima de las 32
semanas, se presenta edema blando a nivel de tobillos y piernas, sobre todo al final del
día. Esto se debe a:
a. Aumento de la presión venosa por c. Disminución de la volemia en el
debajo del nivel del útero debido a embarazo, ocasionando edema
oclusión parcial de la vena cava pretibial
b. Aumento de la presión d. Disminución de la presión venosa
coloidosmótica, ocasionando por debajo del nivel del útero
edema pretibial debido a oclusión parcial de la
vena cava
____ 173. ¿Cuál es el agente etiológico de mayor frecuencia en la mastitis puerperal?
a. Ureaplasma urealyticum c. Escherichia coli
b. Staphylococcus aureus d. Clostridium sordellii

____ 174. En un sangrado uterino anormal de tipo anatómico, usamos la ecografía con infusión de
solución salina como método diagnóstico. ¿En qué caso está contraindicado su uso?
a. Pólipos endometriales c. Infección pélvica
b. Diabetes d. Leiomiomas submucosos

____ 175. Paciente puerperal posterior a cesárea con diagnóstico de infección pélvica. ¿Cuál es el
esquema de tratamiento estándar de oro?
a. Secnidazol y Ciprofloxacina c. Vancomicina y Ciprofloxacina
b. Vancomicina y Metronidazol d. Clindamicina y Gentamicina

____ 176. En el segundo trimestre del embarazo. ¿Qué valores de hemoglobina y hematocrito se
consideran anemia?
a. Hb mayor a 11 g/dL y Hoto 33% c. Hb mayor a 12 g/dL y Hoto 36%
b. Hb menor a 10.5 g/dL y Hoto 32% d. Hb menor a 10 g/dL y Hoto 30%
____ 177. El medicamento de primera elección para el tratamiento preventivo de la eclampsia es:
a. Sulfato de Magnesio c. Fenitoína
b. Hidralazina d. Nifedipina

____ 178. Los anticonceptivos tienen como beneficio principal en la salud de la mujer la eficaz
prevención del embarazo no planificado. Pero además existen una serie de beneficios
adicionales, que en algunas ocasiones justifican su indicación independientemente de
la necesidad de anticoncepción. Refiera usted otro beneficio de la ACO relacionados
con el ciclo menstrual.
a. Prevención de la pérdida de masa c. Prevención de la anemia
ósea ferropénica
b. Disminución del riesgo del cáncer d. Tratamiento de los síntomas de
de ovario hirsutismo y acné en el
hiperandrogenismo

29
ID: A

Gineco-obstetricia
Answer Section

MULTIPLE CHOICE

1. ANS: A PTS: 1
2. ANS: C PTS: 1
3. ANS: A PTS: 1
4. ANS: D PTS: 1
5. ANS: D PTS: 1
6. ANS: D PTS: 1
7. ANS: D PTS: 1
8. ANS: C PTS: 1
9. ANS: D PTS: 1
10. ANS: B PTS: 1
11. ANS: A PTS: 1
12. ANS: C PTS: 1
13. ANS: C PTS: 1
14. ANS: B PTS: 1
15. ANS: C PTS: 1
16. ANS: D PTS: 1
17. ANS: A PTS: 1
18. ANS: C PTS: 1
19. ANS: D PTS: 1
20. ANS: D PTS: 1
21. ANS: B PTS: 1
22. ANS: C PTS: 1
23. ANS: B PTS: 1
24. ANS: D PTS: 1
25. ANS: B PTS: 1
26. ANS: D PTS: 1
27. ANS: B PTS: 1
28. ANS: D PTS: 1
29. ANS: D PTS: 1
30. ANS: C PTS: 1
31. ANS: A PTS: 1
32. ANS: C PTS: 1
33. ANS: C PTS: 1
34. ANS: A PTS: 1
35. ANS: A PTS: 1
36. ANS: C PTS: 1
37. ANS: B PTS: 1
38. ANS: C PTS: 1
39. ANS: C PTS: 1
40. ANS: B PTS: 1
41. ANS: B PTS: 1
42. ANS: D PTS: 1
43. ANS: C PTS: 1

1
ID: A

44. ANS: D PTS: 1


45. ANS: B PTS: 1
46. ANS: D PTS: 1
47. ANS: C PTS: 1
48. ANS: B PTS: 1
49. ANS: C PTS: 1
50. ANS: C PTS: 1
51. ANS: D PTS: 1
52. ANS: A PTS: 1
53. ANS: A PTS: 1
54. ANS: C PTS: 1
55. ANS: A PTS: 1
56. ANS: D PTS: 1
57. ANS: D PTS: 1
58. ANS: D PTS: 1
59. ANS: A PTS: 1
60. ANS: B PTS: 1
61. ANS: D PTS: 1
62. ANS: B PTS: 1
63. ANS: A PTS: 1
64. ANS: D PTS: 1
65. ANS: C PTS: 1
66. ANS: B PTS: 1
67. ANS: D PTS: 1
68. ANS: C PTS: 1
69. ANS: A PTS: 1
70. ANS: D PTS: 1
71. ANS: C PTS: 1
72. ANS: A PTS: 1
73. ANS: B PTS: 1
74. ANS: B PTS: 1
75. ANS: D PTS: 1
76. ANS: D PTS: 1
77. ANS: C PTS: 1
78. ANS: A PTS: 1
79. ANS: C PTS: 1
80. ANS: D PTS: 1
81. ANS: C PTS: 1
82. ANS: B PTS: 1
83. ANS: C PTS: 1
84. ANS: B PTS: 1
85. ANS: D PTS: 1
86. ANS: A PTS: 1
87. ANS: C PTS: 1
88. ANS: D PTS: 1
89. ANS: D PTS: 1
90. ANS: B PTS: 1
91. ANS: A PTS: 1

2
ID: A

92. ANS: A PTS: 1


93. ANS: A PTS: 1
94. ANS: C PTS: 1
95. ANS: D PTS: 1
96. ANS: A PTS: 1
97. ANS: A PTS: 1
98. ANS: A PTS: 1
99. ANS: D PTS: 1
100. ANS: A PTS: 1
101. ANS: C PTS: 1
102. ANS: A PTS: 1
103. ANS: C PTS: 1
104. ANS: A PTS: 1
105. ANS: B PTS: 1
106. ANS: B PTS: 1
107. ANS: B PTS: 1
108. ANS: D PTS: 1
109. ANS: A PTS: 1
110. ANS: B PTS: 1
111. ANS: C PTS: 1
112. ANS: A PTS: 1
113. ANS: C PTS: 1
114. ANS: B PTS: 1
115. ANS: D PTS: 1
116. ANS: D PTS: 1
117. ANS: B PTS: 1
118. ANS: B PTS: 1
119. ANS: B PTS: 1
120. ANS: D PTS: 1
121. ANS: C PTS: 1
122. ANS: C PTS: 1
123. ANS: C PTS: 1
124. ANS: A PTS: 1
125. ANS: A PTS: 1
126. ANS: B PTS: 1
127. ANS: B PTS: 1
128. ANS: D PTS: 1
129. ANS: C PTS: 1
130. ANS: A PTS: 1
131. ANS: A PTS: 1
132. ANS: A PTS: 1
133. ANS: A PTS: 1
134. ANS: C PTS: 1
135. ANS: C PTS: 1
136. ANS: D PTS: 1
137. ANS: C PTS: 1
138. ANS: B PTS: 1
139. ANS: A PTS: 1

3
ID: A

140. ANS: D PTS: 1


141. ANS: D PTS: 1
142. ANS: B PTS: 1
143. ANS: B PTS: 1
144. ANS: C PTS: 1
145. ANS: C PTS: 1
146. ANS: A PTS: 1
147. ANS: A PTS: 1
148. ANS: B PTS: 1
149. ANS: A PTS: 1
150. ANS: C PTS: 1
151. ANS: C PTS: 1
152. ANS: D PTS: 1
153. ANS: C PTS: 1
154. ANS: B PTS: 1
155. ANS: D PTS: 1
156. ANS: A PTS: 1
157. ANS: D PTS: 1
158. ANS: B PTS: 1
159. ANS: B PTS: 1
160. ANS: B PTS: 1
161. ANS: C PTS: 1
162. ANS: B PTS: 1
163. ANS: D PTS: 1
164. ANS: B PTS: 1
165. ANS: D PTS: 1
166. ANS: C PTS: 1
167. ANS: C PTS: 1
168. ANS: B PTS: 1
169. ANS: A PTS: 1
170. ANS: C PTS: 1
171. ANS: C PTS: 1
172. ANS: A PTS: 1
173. ANS: B PTS: 1
174. ANS: C PTS: 1
175. ANS: D PTS: 1
176. ANS: B PTS: 1
177. ANS: A PTS: 1
178. ANS: C PTS: 1

4
Gineco-obstetricia [Answer Strip] ID: A

D
_____ 7. C 13.
_____ B 21.
_____ D 26.
_____

C 22.
_____
A
_____ 1.
B 14.
_____

B 23.
_____

C
_____ 2.

C
_____ 8.
D 24.
_____
C 15.
_____

D
_____ 9.

A
_____ 3.
D 16.
_____ B 25.
_____
B 10.
_____

D
_____ 4.
A 11.
_____ B 27.
_____

A 17.
_____

D 28.
_____
D
_____ 5.
C 12.
_____ C 18.
_____

D 19.
_____
D 29.
_____
D
_____ 6.

D 20.
_____
Gineco-obstetricia [Answer Strip] ID: A

C 30.
_____ B 37.
_____ D 44.
_____ A 52.
_____ D 57.
_____

A 31.
_____ B 45.
_____
C 38.
_____

C 32.
_____ D 46.
_____

A 53.
_____

C 33.
_____
C 39.
_____
C 47.
_____
C 54.
_____
A 34.
_____ B 40.
_____

A 55.
_____

B 48.
_____
D 58.
_____
A 35.
_____ B 41.
_____ D 56.
_____

C 49.
_____

A 59.
_____
C 36.
_____ D 42.
_____
C 50.
_____

C 43.
_____
B 60.
_____

D 51.
_____
Gineco-obstetricia [Answer Strip] ID: A

D 61.
_____ D 67.
_____ B 74.
_____ D 80.
_____ A 86.
_____

D 75.
_____

C 68.
_____
B 62.
_____ C 81.
_____
C 87.
_____

A 69.
_____
D 76.
_____

B 82.
_____
A 63.
_____ D 88.
_____

D 70.
_____ C 77.
_____

C 83.
_____
D 64.
_____

C 71.
_____

B 84.
_____ D 89.
_____

C 65.
_____
A 78.
_____
B 90.
_____
A 72.
_____

B 66.
_____ D 85.
_____

C 79.
_____
B 73.
_____ A 91.
_____
Gineco-obstetricia [Answer Strip] ID: A

A 92.
_____ A 97.
_____ C
_____103. A
_____109. D
_____115.

A 93.
_____

A
_____104. B
_____110.
D
_____116.

A 98.
_____ B
_____105. C
_____111.

B
_____117.

C 94.
_____
D 99.
_____

B
_____106.

A
_____112.
A
_____100.
B
_____118.
D 95.
_____

B
_____107.

A 96.
_____ C
_____113.

C
_____101.
B
_____119.

D
_____108.

A
_____102. B
_____114.
D
_____120.
Gineco-obstetricia [Answer Strip] ID: A

C
_____121. D
_____128. D
_____136. B
_____142. B
_____148.

C
_____122.
B
_____143.
C
_____129.
C
_____137. A
_____149.

A
_____130. C
_____144.

C
_____123.
B
_____138.

A
_____131. C
_____150.

A
_____124. C
_____145.
C
_____151.
A
_____139.

A
_____132. D
_____140.
A
_____146.
A
_____125.
D
_____152.

A
_____133.

D
_____141.

B
_____126.
A
_____147. C
_____153.
C
_____134.

B
_____127.

C
_____135.
Gineco-obstetricia [Answer Strip] ID: A

B
_____154. C
_____161. C
_____166. A
_____172.

C
_____167.

D
_____155.

B
_____173.

B
_____162. B
_____168.

C
_____174.

A
_____156.

D
_____175.
A
_____169.
D
_____157.

B
_____176.
D
_____163.

B
_____158.
C
_____170.
A
_____177.
B
_____164.

C
_____178.

B
_____159. C
_____171.

D
_____165.

B
_____160.
Name: ________________________ Class: ___________________ Date: __________ ID: A

HIDRATACIÓN Y NUTRICIÓN EN CIRUGÍA GENERAL

Multiple Choice
Identify the choice that best completes the statement or answers the question.

____ 1. En un paciente varón de 30 años con 70Kg de ppeso sin antecedente patológicos, inicar la proporción
correcta de líquido que posee:
a. Plasma 3500cc, Líquido intersticial 10500cc, volumen intracelular 28000cc
b. Plasma 10500cc, líquido instersticial 3500cc, volumen intracelular 28000cc
c. Plasma 3500cc, líquido intesrticial 28000cc, líquido intracelular 10500cc
d. Plasma 28000, líquido intersticia 10500, volumen intracelular 3500cc

____ 2. En una paciene femenina de 30 años de edad, la composición de su líquido plasmático es:
a. Na 10; K 150; Mg 40; HCO3 10
b. Na 128; K 7; Ca 5; Cl 103
c. Na 142; K 4; Ca 5; Cl 103
d. Na 135; K 6; Ca 10; Cl 103

____ 3. Un varon de 70K (promedio entre 60-80Kg), consume en promedi al día 2000 a 2500cc, divididos:
a. 1500cc como agua de oxidación, 700 en alimentos sólicos, 250cc vía oral
b. 1500cc vía oral, 700cc en alimentos sólidos, 250cc como agua de oxidación
c. 1500cc como alimento sólido, 700cc como agua de oxidación, 250ccvía oral
d. 1500cc vía oral, 700cc como agua de oxidación, 250cc como alimentos sólidos

____ 4. En un paciente varon de 60 años que presenta producción de líquido por sonda nasogástrica en volumen de
1500 en 24 horas, con la siguiente composición: Na 60meq/l, K10meq/l; Cl 130meq/l, favor indicar
posiblemente de qué organo proviene el líquido.
a. Salival
b. Gástrico
c. Duodenal
d. Colon

____ 5. En un paciente varon de 60 años, con cuadro de ileo paralítico, sin antecedente quirúrgico, debido a una
falta de contractilidad normal del músculo liso. Señale a que tipo de anomalía electrolítica puede
deberse:
a. Hiponatremia
b. Hipopotasemia
c. Hipocalcemia
d. Hipomgnisemia

____ 6. En un paciente varon de 30 años con TCE, en Glasgow 3; se contraindica la alimentación por sonda
nasogástrica debido a:
a. Presencia previa de aspiración pulmonar
b. Pérdida de reflejos laríngeos protectores
c. No existe contraindicación en el uso de sonda nasogastrica
d. Nunca se debe usar sonda nasográstrica

1
Name: ________________________ ID: A

____ 7. En cual de los diguientes pacientes está indicada la nitrición parentel:


a. Enfermos con necesidades metabólicas bajas secundarias a traumatismo grave, como
quemadura extensas de espesor total, fracturas fracura mayores o lesiones de tejidos
blados
b. Pacientes con descerebración irrebersible
c. Pacientes adultos con trastornos digestivos funcionales, como discinesia esofágica
consecutiva a accidentes vascular cerebral, diarrea ideopática, vómito psocógeno o
anorexia nerviosa
d. Período de inestabilidad cardiovascular o alteración metabólica grave

____ 8. En caso de requerir nutrición enteral para un paciente varon de 30 años, de 70Kg, al utilzar una fórmula que
contega 0,86KCAl/ml, 5,92 gr/L de N; 450mol/Kg; Na32; K32, a qué fórmula nos referimos:
a. Criti-carHN
b. Ensure
c. Ensure plus
d. Isocal

____ 9. En un paciente varón de 70Kg, 30 años de edad 1 gr de nitrógeno corresponde a que valor de proteina
muscular
a. 6,20
b. 6,25
c. 6,30
d. 6,35

____ 10. Dentro de las reservas corporales de combustible, señale lo correcto:


a. La masa total de proteinas es relativamente fija y el exceso o deficiencia calórica se
satisfacen por un aumento o una disminución de la masa de carbohidratos del cuerpo
b. La masa total de carbohidratos es relativamente fija y el exceso o deficiencia calórica se
satisfacen por un aumento o una disminución de la masa de grasa del cuerpo
c. La masa total de grasa es relativamente fija y el exceso o deficiencia calórica se
satisfacen por un aumento o una disminución de la masa de proteinas del cuerpo
d. La masa total de proteinas es relativamente fija y el exceso o deficiencia calórica se
satisfacen por un aumento o una disminución de la masa de grasa del cuerpo

2
ID: A

HIDRATACIÓN Y NUTRICIÓN EN CIRUGÍA GENERAL


Answer Section

MULTIPLE CHOICE

1. ANS: A
Del totl de volumen extracelular el plasma corresponde al 5%, líquido intesrticial 15%, volumen
intracelular 40%

PTS: 1 DIF: alta


REF: Shire, GT; Atención hidroelectrolítica y nutricional del paciente quirúrgico; en: Principios de
Cirugía, Schwartz, 6ta edición,vol 1, Cap 1, pag 62
OBJ: Conocer las bases de la hidratación en el paciente quirúrgico
TOP: Manejo de líquidos y electrolicots en el paciente quirúrgico
KEY: Plama, líquido intersticial, volumen intracelular NOT: Dr. Vinicio Moreno Rueda
2. ANS: C
Los valeres normales, que podemos dosificarlos en plasma son los que constan en el literal c, con esto
nos guiamos en alteracines en su concentración asociadas a la hidratación y administración de electrolitos
en un paciente quirúrgico

PTS: 1 DIF: alta


REF: Shire, GT; Atención hidroelectrolítica y nutricional del paciente quirúrgico; en: Principios de
Cirugía, Schwartz, 6ta edición,vol 1, Cap 1, pag 63 OBJ: Conocer valores normales de los
electrolitoa plasmáticos
TOP: Manejo de líquido y electrolitos en un paciente quirúrgico
KEY: Socio, postadio, calcio NOT: Dr. Vinicio Moreno Rueda
3. ANS: B
El requerimiento diario en promedio en un hobre de 70Kg es lo que se indica en el literal b; una
alteración en la administración en forma intravenosa puede llevar a desequilbrio hidroelectrolítico

PTS: 1 DIF: alta


REF: Shire, GT; Atención hidroelectrolítica y nutricional del paciente quirúrgico; en: Principios de
Cirugía, Schwartz, 6ta edición,vol 1, Cap 1,ág 64
OBJ: Conocmiento del aporte diario de líquido en una persona promedio
TOP: Líquidos y electrolitos en el paciente quirúrgico KEY: requerimiento diario de líquido
NOT: Dr. Vinicio Moreno Rueda
4. ANS: B
En el estómago al día se producen unos 1500 cc de íiquido con la composición indicada en el literal b

PTS: 1 DIF: alta


REF: Shire, GT; Atención hidroelectrolítica y nutricional del paciente quirúrgico; en: Principios de
Cirugía, Schwartz, 6ta edición,vol 1, Cap 1, pag 65
OBJ: Conocimiento de la composición de cada uno de los líquidos que se producen en el tubo digestivo
TOP: Hidratación en el paciente quirúrgico KEY: secreción gástrica
NOT: Dr. Vinicio Moreno Rueda

1
ID: A

5. ANS: B
Los signos de déficit de de potasio se relacionan con falta de contractilidad normal de músculos
esqueléticos, lisos, cardíaco; en nuestro caso puede evolucionar a íleo paralítico

PTS: 1 DIF: alta


REF: Shire, GT; Atención hidroelectrolítica y nutricional del paciente quirúrgico; en: Principios de
Cirugía, Schwartz, 6ta edición,vol 1, Cap 1, pag 72
OBJ: Identificar clinicamene las ateraciones electrolíticas en la sintomatología abdominal
TOP: Hidratación y manejo de electrolitos en el paciente quirúrgico
KEY: Hopopitasemia, hipocalemia, ileo paralítico NOT: Dr. Vinicio Moreno Rueda
6. ANS: B
La alimentación con soda nasofaringea o nasogástrica solo se la debe usar en pacientes despiertos, la
principal contraindicación para este mpetodo es la falta de conocimiento o la pérdida de los reflejso
laríngeos protectores, que pueden oriinar complicaciones pulmonares por aspiración que ponen en peligro
la vida

PTS: 1 DIF: alta


REF: Shire, GT; Atención hidroelectrolítica y nutricional del paciente quirúrgico; en: Principios de
Cirugía, Schwartz, 6ta edición,vol 1, Cap 1
OBJ: Conocimiento de complicaciones por el uso de nutrición enteral
TOP: Nutrición en el paciente quirúrgico KEY: Nutrición con sonda nasogástrica
NOT: Dr. Vinicio Moreno Rueda
7. ANS: C
La NPT, se utiliza cuando el tubo digestivo no tiene una funcionalidad normal, com lo indicado en el
literal c

PTS: 1 DIF: alta


REF: Shires, GT; Atención hidroelectrolítica y nutricional del paciente quirúrgico, en Principios de
Cirugía Scheartz, 6ta edición, vol. 1, cap.2, pag 89 OBJ: Conocimiento de indicaciones de NPT
TOP: Nutrición en el paciente quirúrgico KEY: NPT, nutrición parenteral
NOT: Dr. Vinicio Moreno Rueda
8. ANS: B
El ensure es la fórmula más comunmente utilizada, tiene la composición descrita en el enunciado y el
estudiante debe conocerla

PTS: 1 DIF: alta


REF: Shires, GT; Atención hidroelectrolítica y nutricional del paciente quirúrgico, en Principios de
Cirugía, Schwartz, 6ta ed, vol 1, cap 2, pag 85 OBJ: conocimiento de principios de
nutrición enteral
TOP: Nutrición en el paciente quirúrgico KEY: ensure NOT: Dr. Vinicio Moreno Rueda
9. ANS: B
Esta proteina en casos de ininaición proviene en su mayoría del músculo esquelético y es la que se
transformará en glucosa mediante la glucloneogénesis hepática

PTS: 1 DIF: alta


REF: Shires, GT; Atención hidroelectrolítica y nutricional del paciente quirúrgico, en Principios de
Cirugía, Schwartz, 6ta ed, vol 1, cap 2,
OBJ: Conocimeinto de principios de nutrición en un paciente quirúrgico
TOP: Nutrición en el paciente quirúrgico KEY: proteina muscular, nitrógeno, inanición,
gluconeogénesis
MSC: Dr. Vinicio Moreno Rueda

2
ID: A

10. ANS: D
Las proteinas y las grasas son las únicas fuentes importantes de combustible

PTS: 1 DIF: alta


REF: shires, GT; Atención hidroelectrolítica y nutricional del paciente quirúrgico, en Principios de
Cirugía Schwartz, ed 6ta, vol 1, cap. 2, pag 61
OBJ: Conocimiento de los principios de nutrición en un paciente quirúrgico
TOP: Nutrición en un paciente quirúrgico KEY: fuente de energía
NOT: Dr. Vinicio Moreno Rueda

3
HIDRATACIÓN Y NUTRICIÓN EN CIRUGÍA GENERAL [Answer Strip] ID: A

C
_____ 7.

A
_____ 1.

B
_____ 8.

C
_____ 2.

B
_____ 9.

B
_____ 3.

D 10.
_____
B
_____ 4.

B
_____ 5.

B
_____ 6.
GINECOLOGIA Y OBSTETRICIA
INFECCIÓN DE VIAS URINARIAS EN EL EMBARAZO

TRUE/FALSE

1. Luego de completar el tratamiento antibiótico tanto de BA como de cistitis, se debe realizar


urocultivo de control para documentar el éxito de la erradicación ?

ANS: T
Luego de completar el tratamiento antibiótico tanto de BA como de cistitis, se debe realizar
urocultivo de control para documentar el éxito de la erradicación

PTS: 1 DIF: BAJA


REF: INFECCIÓN DE VIAS URINARIAS EN EL EMBARZO GPC/MSP 2013 PAG: 15
OBJ: DIAGNOSTICAR Y TRATAR IVU EN EL EMBARZO PARA PREVENIR
COMPLICACIONES TOP: IVU KEY: INFECCIÓN

2. La cistitis aguda se define como: Infección bacteriana del tracto urinario bajo que se acompaña de
los siguientes signos y síntomas: urgencia, frecuencia, disuria, piuria y hematuria: sin evidencia de
afección sistémica

ANS: T
La cistitis aguda es la Infección bacteriana del tracto urinario bajo que se acompaña de los
siguientes signos y síntomas: urgencia, frecuencia, disuria, piuria y hematuria: sin evidencia de
afección sistémica.

PTS: 1 DIF: MEDIA


REF: GPC / MSP: IVU EN EL EMBARAZO 2013. PAG: 15
OBJ: DIAGNOSTICAR Y TRATAR LAS IVU EN EL EMBARAZO PARA PREVENIR
COMPLICACIONES TOP: U KEY: INFECIÓN

3.
La pielonefritis aguda se define como: Es la infección de la vía excretora urinaria alta y del
parénquima renal de uno o ambos riñones que se acompaña de fiebre, escalofrió, malestar
general, dolor costovertebral y en ocasiones, náusea, vómito y deshidratación

ANS: T
La pielonefritis aguda es la infección de la vía excretora urinaria alta y del parénquima renal de
uno o ambos riñones que se acompaña de fiebre, escalofrió, malestar general, dolor costovertebral
y en ocasiones, náusea, vómito y deshidratación

PTS: 1 DIF: MEDIA


REF: GPC / MSP: IVU EN EL EMBARAZO 2013. PAG: 15
OBJ: DIAGNOSTICAR Y TRATAR LAS IVU EN EL EMBARAZO PARA PREVENIR
COMPLICACIONES LOC: IVU TOP: INFECCÓN

1
MULTIPLE CHOICE

1. Las infecciones de vías urinarias constituyen una de las complicaciones infecciosas más
habituales del embarazo, señale los factores de riesgo que intervienen para esta patología:
a. cambios funcionales, hormonales y c. localización del meato uretral expuesta a
anatómicos bacterias uropatógenas
b. bacterias de la vagina que acceden al d. todo es correcto
tracto urinario bajo
ANS: D
Las infecciones de vías urinarias constituyen una de las complicaciones infecciosas más habituales
del embarazo y son responsables de un importante porcentaje de morbimortalidad tanto materna
cuanto perinatal, especialmente en los lugares de escasos recursos.1 Las mujeres embarazadas
desarrollan de manera fácil infecciones de vías urinarias (IVU) debido a cambios funcionales,
hormonales y anatómicos, además de la localización del meato uretral expuesta a bacterias
uropatógenas y de vagina que acceden al tracto urinario bajo.
RESPALDO BIBLIOGRAFICO: INFECCIÓN DE VIAS URINARIAS EN EL EMBARAZO
GPC/MSP 2013, PAG 10

PTS: 1 DIF: Media REF: Infección de vias urinarias en el embarazo


OBJ: Diagnosticar y trata las IVU durante el embarazo para prevenir complicaciones
TOP: IVU KEY: Infección

2. La elección del antimicrobiano en las IVU, debe estar dirigido para los agentes etiológicos más
frecuentes y debe ser seguro para la madre y el feto, de acuerdo a las GPC /MSP
(INFECCIÓN DE VIAS URINARIAS EN EL EMBARZO). La droga de elección para
nuestro país, por sus bajos niveles de resistencia es:
a. fosfomicina c. Nitrofurantoína
b. cefalosporinas d. Ampicilina e inhibidores de
betalactamasas
ANS: C
La elección del antimicrobiano debe estar dirigido para los agentes etiológicos más frecuentes y
debe ser seguro para la madre y el feto. La droga de elección para nuestro país es la
Nitrofurantoína por sus bajos niveles de resistencia, fosfomicina y cefalosporinas son alternativas
a la nitrofurantoína. Ampicilina e inhibidores de betalactamasas son desaconsejados por sus altas
tasas de resistencia local presentadas en reunión de consenso para toma de decisiones respecto a la
terapéutica
RESPALDO BIBLIOGRAFICO: INFECCIÓN DE VIAS URINARIAS EN EL EMBARAZO
GPC/MSP 2013, PAG 15

PTS: 1 DIF: MEDIA


REF: INFECCIÓN DE VIAS URINARIAS EN EL EMBARAZO GPC/MSP
OBJ: DIAGNOSTICAR Y TRATAR LAS IVU EN EL EMBARAZO PARA PREVENIR
COMPLICACIONES TOP: IVU KEY: INFECCIÓN

2
3. Paciente acude a control preconcepcional, refiere que ha presentado desde hace 3 años frecuentes
infecciones de vías urinarias, usted le explica que durante el embarazo las probabilidades que
presente esta patología son altas por los cambios fisiológicos que se dan en el embarazo, siendo
estos, factores que propician la bacteriuria asintomática y su progresión a pielonefritis. EXCEPTO

a. Compresión mecánica por el crecimiento c. Cambios en el pH urinario


del útero

b. Relajación del músculo liso inducido por d. Ausencia de glucosuria y aminoaciduria


la progesterona

ANS: D
Los cambios fisiológicos en el embarazo (Compresión mecánica por el crecimiento del útero,
relajación del músculo liso inducido por la progesterona, cambios en el pH urinario, presencia de
glucosuria y aminoaciduria) son factores que propician la bacteriuria asintomática (BA) y su
progresión a pielonefritis).

PTS: 1 DIF: ALTA


REF: GPC / MSP: IVU EN EL EMBARAZO 2013. PAG: 14
OBJ: DIAGNOSTICAR Y TRATAR LAS IVU EN EL EMBARAZO PARA PREVENIR
COMPLICACIONES TOP: IVU KEY: INFECCIÓN

4. La bacteriuria asintomática (BA) se define como:

a. Presencia de bacterias en orina c. Es la infección de la vía excretora


detectadas por urocultivo (más de urinaria alta y del parénquima renal de
100.000 unidades formadoras de uno o ambos riñones que se acompaña de
colonias/ml sin síntomas típicos de fiebre, escalofrió, malestar general, dolor
infección aguda del tracto urinario costovertebral y en ocasiones, náusea,
vómito y deshidratación

b. Infección bacteriana del tracto urinario


bajo que se acompaña de los siguientes
signos y síntomas: urgencia, frecuencia,
disuria, piuria y hematuria: sin evidencia
de afección sistémica

ANS: A
De acuerdo a la GPC del MSP define a la BA como: la presencia de bacterias en orina detectadas
por urocultivo (más de 100.000 unidades formadoras de colonias/ml sin síntomas típicos de
infección aguda del tracto urinario.

PTS: 1 DIF: MEDIA


REF: GPC / MSP: IVU EN EL EMBARAZO 2013. PAG: 15
OBJ: DIAGNOSTICAR Y TRATAR LAS IVU EN EL EMBARAZO PARA PREVENIR
COMPLICACIONES TOP: IVU KEY: INFECCIÓN

3
5. Paciente que acude a control prenatal, al momento se encuentra de 20 semanas de gestación,
refiere urgencia y frecuencia urinaria, además disuria, piuria y hematuria: sin evidencia de
afección sistémica, por clínica usted diagnóstica de:

a. Bacteriuria asintomática c. Pielonefritis aguda

b. Cistitis aguda

ANS: B
La cistitis aguda es la Infección bacteriana del tracto urinario bajo que se acompaña de los
siguientes signos y síntomas: urgencia, frecuencia, disuria, piuria y hematuria: sin evidencia de
afección sistémica

PTS: 1 DIF: MEDIA


REF: GPC / MSP: IVU EN EL EMBARAZO 2013. PAG: 15
OBJ: DIAGNOSTICAR Y TRATAR LAS IVU EN EL EMBARAZO PARA PREVENIR
COMPLICACIONES TOP: IVU KEY: INFECCIÓN

4
Name: ________________________ Class: ___________________ Date: __________ ID: A

Medicina interna dos

Multiple Choice
Identify the choice that best completes the statement or answers the question.

____ 1. Paciente de 29 años sin antecedentes de interés, refiere que desde hace 3 días presenta
dolor en zona vulvar y vaginal. No síntomas miccionales. En el examen físico se
objetivan en pubis y vagina lesiones vesiculares múltiples de 1 a 2 mm de diámetro de
bordes eritematosos y adenopatías inguinales bilaterales dolorosas a la palpación. No
se observa secreción vaginal ni uretral ¿Cuál es el tratamiento indicado?
a. Penicilina benzatínica 1.2 millones c. Aciclovir 400 mg vía oral tres
de unidades intramuscular veces por 10 a 14 días
unidosis
b. Azitromicina 1 gramo vía oral por d. Doxiciclina 100 mg vía oral dos
una sola dosis veces al día por 21 días
____ 2. Señale la dosificación ideal (objetivo terapéutico) de HbA1c en pacientes ancianos con
múltiples enfermedades crónicas y dificultad para las actividades de la vida diaria:
a. HbA1c7.0% c. HbA1c 10.0%
b. HbA1c6.5 % d. HbA1c8.0 %

____ 3. Juan tiene 72 años y es llevado al servicio de emergencia, porque presentó tos con flema
hace 1 semana, dificultad para respirar desde hace 4 días y alteración del estado de
conciencia hace 24 horas. Su médico de cabecera recetó arinoxicilina y envío de
exámenes de laboratorio con Urea de 89 mg/dl, glucosa 110 mg/dl y EMO normal. Al
examen físico FC 64, FR 33, TA 110/50. Pulmones con crepitantes bilaterales en bases
y escasas sibilancias en base de pulmón derecho. ¿Cómo manejaría a este paciente,
según la escala de gravedad CURB65?
a. El paciente debe ser manejado c. El paciente deberá ser ingresado
ambulatoriamente con un solo al servicio de neumología para
antibiótico recibir antibiótico intravenoso
b. El paciente tiene una neumonía d. El paciente es de bajo riesgo por
grave y debe ser ingresado a la lo que se envía al domicilio con
unidad de cuidado intensivo para familiares para que continúe con
su manejo el tratamiento de amoxicilina
indicado
____ 4. Señale la patología que ocasiona anemia megaloblástica:
a. Gastrectomía total c. Tumor esofágico
b. Síndrome de Mallory-Weiss d. Hernia del hiato

1
Name: ________________________ ID: A

____ 5. Paciente de 30 años, refiere disuria y secreción uretral purulenta de 3 días de


evolución. En el examen físico se objetiva abundante secreción uretral purulenta e
inflamación del meato uretral. La tinción de Gram muestra abundantes leucocitos
polimorfonucleares y diplococos Gram-negativos. Señale el tratamiento empírico de
elección para este paciente:
a. Gentamicina 160 mg c. Ceftriaxona 250 mg intramuscular
intramuscular una sola dosis + + Azitromicina 1 g vía oral, ambas
Azitromicina 500 mg vía oral por en dosis única
tres días
b. Ceftriaxona 250 mg intramuscular d. Ceftriaxona 250 mg intramuscular
en una sola dosis + Clindamicina una sola dosis + Metronidazol 500
300 mg vía oral por 7 días mg vía oral por 14 días
____ 6. ¿Cuál es la prueba de radiodiagnóstico que detecta lesiones tempranas de cáncer
gástrico?
a. Ecografía abdominal c. Estudio radiológico baritado
b. Endoscopia digestiva alta d. Radiografía simple de abdomen
____ 7. Señale el esquema de tratamiento de la crisis asmática:
a. Agonistas B2 de larga duración + c. Agonista B2 de larga duración +
dosis altas de corticoide inhalado dosis bajas de corticoide inhalado
+ corticoide oral
b. Agonistas B2 de corta duración + d. Dosis bajas de corticoide inhalado
corticoideo nebulizado +
corticoide intravenoso en bolo
____ 8. El uso de los bloqueadores b cardioselectivos en el tratamiento de hipertensión arterial
están contraindicados en caso de:
a. Taquiarritmias ventriculares c. Taquicardia sinusal
b. Infarto al Miocardio d. Bloqueo cardíaco de segundo o
tercer grado
____ 9. En los pacientes con diabetes mellitus tipo 2 (DM2), se recomienda la siguiente
distribución de los tipos de grasa dietaria considerando el total de calorías diarias.
EXCEPTO:
a. Ácidos grasos trans <1 % c. Ácidos grasos saturados 30-45 %
b. Ácidos grasos poli insaturados d. Ácidos grasos mono insaturados
<10 % 12-20 %
____ 10. Indique el tratamiento que debe darse a una paciente que presenta en la primera
consulta, fiebre de 38 °C, dolor abdominal bajo, que se acompaña de disuria,
polaquiuria y dispareunia, al examen físico se encuentra: secreción vaginal amarillenta
abundante; al examen especular se evidencia cérvix eritematoso que sangra fácilmente
a la manipulación, no se encontraron adenomegalias inguinales, vesículas, o úlceras
genitales
a. Ciprofloxacina 500 mg (DU) + c. Ceftriaxona 250 mg IM (DU)
Azitromicina 1g (DU) +Aciclov¡r 200 mg 5 /día (5 a 7
días)
b. Azitromicina 1g (DU) + d. P Benzatina 2.400.000 Ul (IM) DU
eritromicina 500 mg c/6h (15-21 + eritromicina 500 mg VO c/6 h
días) (7días)

2
Name: ________________________ ID: A

____ 11. Los siguientes son fármacos que inducen diabetes, EXCEPTO:
a. Adrenalina c. Glucocorticoides
b. Inhibidores de Proteasas d. Bromocriptina

____ 12. Seleccione los microorganismos relacionados con las diarreas agudas inflamatorias:
1. E. coli enterotoxigénica.
2. E. coli enteroinvasiva.
3. Shigella sp.
4. Clostridium perfringens.
5. Campylobacter sp.

a. 1, 2, 4 c. 1, 3, 4
b. 2, 3, 5 d. 2, 4, 5
____ 13. Indique la respuesta correcta. Una ama de casa de 57 años de edad, sin antecedentes
patológicos previos, con un IMC de 29, acude a su consulta porque tiene el deseo de
dejar de fumar. Ella refiere que ha tratado por todos los métodos, ha recibido ayuda
psicológica en reiteradas ocasiones, pero su dependencia física al cigarrillo le han
hecho imposible dejarlo definitivamente. Usted dispone de algunas estrategias
terapéuticas. ¿Cuál de las siguientes sería la más indicada en este tipo de pacientes?
a. Terapia de reemplazo de nicotina c. Cigarrillo electrónico
más soporte de grupos de apoyo
b. Chicle de nicotina d. Vareniclina oral

____ 14. Señale qué tasa de filtración glomerular indica falla renal en el paciente diabético tipo 2:
a. 89-90 ml/min c. 15-29 ml/min
b. 14-15 ml/min d. 30-44 ml/min

____ 15. Señale cuál es la manifestación clínica de la escabiosis reincidente:


a. Sarpullido local c. Sarpullido generalizado
b. Pseudoliendres d. Máculas azuladas

____ 16. ¿Cuál es el medicamento de elección para el tratamiento de la tiña capitis en adultos?
a. Itraconazol c. Miconazol
b. Griseofulvina d. Clotrimazol

____ 17. El medicamento de primera línea para realizar el tratamiento de tiña corporis es:
a. Itraconazol tópico c. Fluconazol oral
b. Ketoconazol tópico d. Terbinafina tópica

____ 18. Señale el medicamento que se usa como alternativa en el tratamiento sistémico del acné:
a. Metformina c. Fenitoína
b. Dexametasona d. Azatioprina

____ 19. ¿Cuál es el principal pilar de tratamiento en la infección por Cólera?


a. Acortar el período de la excreción c. Restablecer los fluidos y
bacteriana electrolitos
b. Administrar antibióticos como d. Asegurar una nutrición adecuada
terapéutica inicial

3
Name: ________________________ ID: A

____ 20. Las siguientes son factores relacionados al desarrollo de dermatitis seborreica,
EXCEPTO:
a. Infección por virus de c. Infección por Staphylococcus
inmunodeficiencia humana (VIH) aureus
b. Infección por Malassezia furfur d. Enfermedades neurológicas como
Parkinson
____ 21. Señale la prueba habitual de valoración de control glucémico para un paciente diabético
en el que se requiere conocer la glucemia de los últimos 3 meses:
a. Índice glucémico c. Glucosa plasmática capilar
b. Fructosamina d. Hemoglobina glicocilada

____ 22. Paciente de 21 años, sexualmente activa con varias parejas sexuales, que en ocasiones
utiliza preservativo, refiere que desde hace 3 días presenta secreción vaginal y prurito
vulvar. En el examen físico se observa eritema vulvar y secreción vaginal amarilla,
espesa, espumosa; fétida y con un pH de 7. No hay dolor pélvico a la palpación ni
adenopatías inguinales. Al tacto vaginal no se palpan masas anexiales y no hay dolor a
la movilización del cérvix. ¿Cuál es el diagnóstico de la paciente?
a. Tricomoniasis vaginal c. Enfermedad pélvica inflamatoria
b. Vaginosis bacteriana d. Candidiasis vulvovaginal

____ 23. ¿Cuál es la combinación de insulina cuya duración efectiva es de 15 -18 horas?
a. 70/30 (70 % de aspart protamina y c. 75/25 (75 % de lispro protamina y
30 % de aspart) 25 % de lispro)
b. 50/50 (50 % de lispro protamina y d. 70/30 (70 % de NPHy 30 % de
50 % de lispro) insulina simple)
____ 24. Señale la complicación transfusional retardada:
a. Anafilaxia c. Hepatitis
b. Edema pulmonar d. Hemolisis

4
Name: ________________________ ID: A

____ 25. Paciente de 65 años que acude a consulta médica de primer nivel, sexo masculino,
fumador, no tiene antecedentes de diabetes mellitus; se le toma la presión arterial
sistólica marca 160 mm Hg. Utilizando la siguiente tabla de estimación de riesgo
cardiovascular (OMS/ISH), determine cual es el RCV que presenta este paciente:

a. RCV de 30% a menor a 40% c. RCV de 20% a menor a 30%


b. RCV de 10% a menor a 20% d. RCV mayor o igual a 40%
____ 26. ¿Cuáles son complicaciones de la hemorragia subaracnoidea?
a. Hidrocefalia, vasoespasmo y c. Hidrocefalia, vasoespasmo y
resangrado meningitis
b. Hematoma intraparenquimatoso, d. Vasoespasmo, encefalopatía
hipernatremia y ceguera hipertensiva e isquemia
secundaria
____ 27. Señale los exámenes normales de control glucémico (objetivo terapéutico) en un
paciente diabético:
a. Hb A1c <10.0% + glucosa c. Hb A1c <7.0 % + glucosa
postprandial <10 mmol/ L postprandial < 10 mmol/L
b. Hb A1 c < 11.0% + glucosa d. Hb A1c < 2.0% + glucosa
postprandial >10 mmol/ L postprandial >15 mmol/ L
____ 28. ¿Cuál es el diagnóstico de un paciente que presenta eritema en forma de parches en
cuero cabelludo, la frente, pliegue nasolabial y conducto auditivo externo con
descamación y prurito?
a. Psoriasis c. Dermatitis seborreica
b. Dermatitis de contacto alérgica d. Dermatitis atópica

5
ID: A

Medicina interna dos


Answer Section

MULTIPLE CHOICE

1. ANS: C PTS: 1
2. ANS: D PTS: 1
3. ANS: B PTS: 1
4. ANS: A PTS: 1
5. ANS: C PTS: 1
6. ANS: B PTS: 1
7. ANS: B PTS: 1
8. ANS: C PTS: 1
9. ANS: C PTS: 1
10. ANS: A PTS: 1
11. ANS: D PTS: 1
12. ANS: B PTS: 1
13. ANS: A PTS: 1
14. ANS: B PTS: 1
15. ANS: C PTS: 1
16. ANS: B PTS: 1
17. ANS: D PTS: 1
18. ANS: A PTS: 1
19. ANS: C PTS: 1
20. ANS: C PTS: 1
21. ANS: D PTS: 1
22. ANS: A PTS: 1
23. ANS: A PTS: 1
24. ANS: C PTS: 1
25. ANS: B PTS: 1
26. ANS: A PTS: 1
27. ANS: C PTS: 1
28. ANS: C PTS: 1

1
Medicina interna dos [Answer Strip] ID: A

C
_____ 5. D 11.
_____ C 20.
_____ B 25.
_____

B 12.
_____

C
_____ 1.
D 21.
_____

A 22.
_____
B
_____ 6.
A 13.
_____

D
_____ 2.
B
_____ 7.

A 23.
_____
B
_____ 3.

A 26.
_____
B 14.
_____

C
_____ 8.
C 24.
_____
C 15.
_____

C 27.
_____
C
_____ 9. B 16.
_____

D 17.
_____

C 28.
_____
A
_____ 4.
A 10.
_____
A 18.
_____

C 19.
_____
Name: ________________________ Class: ___________________ Date: __________ ID: A

Medicina Interna

Multiple Choice
Identify the choice that best completes the statement or answers the question.

____ 1. ¿Qué diurético utilizado en el tratamiento de la hipertensión arterial puede originar


impotencia sexual?
a. Espironolactona c. Triamtereno
b. Amilorida d. Clortalidona
____ 2. Paciente de 57 años, con antecedentes de hepatopatía crónica de origen idiopático, es
llevado a emergencias por un cuadro de aparición insidiosa caracterizado por deterioro
del estado de conciencia, dificultad para el habla, desorientación mínima y amnesia de
los eventos recientes ¿Cuál es el diagnóstico?
a. Hipoglicemia c. Encefalopatía hepática
b. Hiponatremia d. Enfermedad cerebrovascular

____ 3. La característica más común de la disfunción autonómica de la diabetes:


a. Bradicardia en reposo c. Aumento del Vaciamiento
Gástrico
b. Anhidrosis de piernas d. Hiperglicemia
____ 4. Señale qué medicamento de los usados en el tratamiento del acné (enfocándose en la
seguridad de su uso durante el embarazo) corresponde a Categoría X:
a. Cotrimoxazol c. Doxiciclina
b. Isotrenitoína d. Adapaleno

____ 5. Son factores que producen macrosomía fetal. EXCEPTO:


a. Tolerancia anormal a la glucosa c. Hiperglucemia materna
b. Mayor resistencia a la insulina d. Insulina materna elevada

____ 6. El uso de isotretinoína produce:


a. Aumenta la producción de sebo c. Aumenta la queratinización de la
piel
b. Sequedad extrema de la piel y d. Inflamación de la piel
queilitis
____ 7. ¿Qué medicamento se utiliza en el tratamiento del acné vulgar leve?
a. Tetraciclina c. Isotretinoína
b. Clobetasol d. Metronidazol

____ 8. ¿Cuál es el medicamento recomendado para el tratamiento de pacientes adultos con


diabetes y nefropatía?
a. Valsartán c. Espironolactona
b. Hidralazina d. Metildopa
____ 9. La principal indicación para el tratamiento del Helicobacter pylori es:
a. Antecedente familiar de cáncer c. Complicaciones de reflujo
gástrico gastroesofágico
b. Portadores de Helicobacter pylori d. Úlcera gástrica o duodenal activa

1
Name: ________________________ ID: A

____ 10. Paciente de 45 años, obeso, diabético y con hiperuricemia. Refiere que desde hace 24
horas presenta dolor en el miembro inferior derecho e impotencia funcional. Al examen
físico presenta sensación distérmica no cuantificada, pulsos distales presentes, edema
de la pierna derecha, aumento de la temperatura local en la región pretibial, en donde
se observa una placa eritematosa, brillante, caliente y dolorosa de 10 x 8 cm de
diámetro. ¿Cuál es el diagnóstico?
a. Tromboflebitis c. Erisipela
b. Crisis Gotosa d. Celulitis

____ 11. Paciente sin cardiopatía, refiere episodios anteriores al presente de palpitaciones,
ansiedad, dolor torácico, disnea, que ceden espontáneamente. Presenta el siguiente
trazo electrocardiográfico indique el algoritmo de tratamiento a seguir.

a. Verapamilo + amiodarona + c. Verapamilo + maniobras vagales


adenosina + procainamida
b. Maniobras vagales + adenosina+ d. Adenosina + amiodarona +
procainamida maniobras vagales
____ 12. ¿Cuál es el diagnóstico más probable, de un paciente de 70 años que presenta marcado
malestar general y ortostatismo, que inició hace 1 hora? Al examen físico se objetiva
pulso regular, FC: 31 pulsaciones por minuto y PA: 80/50 mm Hg.
a. Síndrome de c. Enfermedad del nodo sinusal
Wolf-Parkinson-White
b. Bradicardia sinusal d. Bloqueo AV completo

____ 13. Señale el algoritmo para el tratamiento de taquicardia ventricular con < 150/lmin (estable)
que ha producido un paro cardíaco completo, luego de que el paciente ha recibido tres
choques no sincronizados en secuencia (200J, 300J, 360J) y de que ha sido intubado y
se ha establecido una línea endovenosa.
a. Atropina - dopamina c. Maniobras vagales - adenosina
b. Lidocaína - procainamida - bretilio d. Epinefrina - Atropina

____ 14. ¿Cuál es el tratamiento de la ascitis en pacientes con cirrosis?


a. Administrar diuréticos si hay c. Vigilar el índice de masa corporal
respuesta a la restricción de sal y sodio urinario
b. Restricción estricta de la sal a 400 d. Restricción de líquidos en todos
mg de sodio al día los pacientes con cirrosis
____ 15. ¿Qué medicamento se utiliza en el tratamiento de la diabetes gestacional y durante la
lactancia?
a. Metformina c. Glicazida
b. Glimepirida d. Simvastatina

2
Name: ________________________ ID: A

____ 16. Paciente de 15 años, que acude en compañía de un familiar, quien refiere que la
paciente presenta de forma brusca caída de la cabeza sobre el pecho y en ocasiones
caída al suelo de forma súbita ¿Cuál es el tipo de convulsión que presenta la paciente?
a. Mioclónica c. Crisis focal motora
b. De ausencia d. Atónica

____ 17. Según la Guía de diabetes del MSP, los siguientes son criterios de diagnóstico para
Diabetes Mellitus tipo 2, EXCEPTO:
a. Glucosa en ayunas mayor a 126 c. Hemoglobina glicosilada mayor o
mg/dl por dos ocasiones igual a 6.5%
b. Hiperglucemia con síntomas d. Glucosa mayor o igual a 200
clásicos y glucosa al azar mayor o mg/dl, 2 horas después de
igual a 200 mg/dl sobrecarga de glucosa, por dos
ocasiones
____ 18. Señale el marcador de gravedad de la Pancreatitis aguda en las primeras 24 horas de
hospitalización:
a. BUN < 15 mg /100 ml c. Recuento leucocitario > 12000
células/uL
b. Creatinina sérica <1 mg % d. PCO2 > 32mg/100 ml
____ 19. Los siguientes son factores epidemiológicos relacionados al desarrollo de neumonía
extra hospitalaria. EXCEPTO:
a. Demencia c. Tabaquismo
b. Sedentarismo d. Alcoholismo
____ 20. Son factores de riesgo para infección de vías urinarias, EXCEPTO:
a. Diabetes mellitus c. Estado de gravidez
b. Ingesta de alcohol d. Obstrucción urinaria

____ 21. ¿Cuál es el diagnóstico de un paciente adulto con obesidad central, circunferencia de la
cintura mayor a 94 cm, niveles de triglicéridos 170 mg/dl, niveles de colesterol HDL de
60 mg/dL, presión arterial elevada 140/90 y con diabetes tipo 2 previamente
diagnosticada?
a. Síndrome metabólico c. Hiperprolactinemia
b. Síndrome de Cushing d. Diabetes Mellitus 1

____ 22. Paciente de 25 años con antecedente de parto reciente presenta palpitaciones, disnea,
dolor pleurítico, hemoptisis; al examen físico se evidencia fiebre de 38° C, cianosis,
taquipnea, taquicardia, estertores pulmonares. En exámenes solicitados: gasometría
arterial normal, EKG reporta taquicardia sinusal, Rx de tórax reporta atelectasias
básales con elevación del hemidiafragma, ensanchamiento de los hilios pulmonares
bilateralmente, infiltrado alveolar basal borde inferior convexo (joroba de Hampton),
afilamiento de imágenes vasculares. ¿Cuál es el tratamiento anticoagulante que debe
administrar al ingreso, antes de referir a UCI?
a. Heparina c. Warfarina
b. Estreptocinasa d. Clopidrogrel

____ 23. Señale la contraindicación al uso de enalapril en la diabetes con hipertensión arterial:
a. Hipertensa con embarazo c. Hipertensa con nefropatía
b. Normotensa con microalbumlnuria d. Hipertensa con albuminuria

3
Name: ________________________ ID: A

____ 24. ¿Cuál es la conducta terapéutica inicial en un paciente de 45 años, sin antecedentes de
interés, que en un chequeo médico, se objetivan los siguientes resultados: Colesterol
total 260 mg/dl, HDL:34 mg/dl; LDL:140 mg/dl; Triglicéridos:550 mg/dl?
a. Prescribir un fibrato c. Prescribir una estatina
b. Prescribir una estatina y un fibrato d. Dieta, ejercicio y control en 3
meses
____ 25. Señale el enunciado correcto con relación al síndrome nefrítico agudo:
a. Proteinuria > 3 g/ 24h + c. Proteinuria > 4 g/ 24h +
Hipercolesterolemia Hipercoagulabilidad
b. Proteinuria < 2 g /24 h + d. Proteinuria < 4 g/ 24h + Edema
Hematuria intrarrenal
____ 26. Paciente de 60 años, que presenta tos persistente, en ocasiones con hemoptisis,
sudoración nocturna, disnea y dolor torácico tipo punzada de costado. En la radiografía
de tórax se muestra un derrame pleural de tamaño moderado. ¿Cuál es el diagnóstico?
a. Absceso pulmonar c. Neumonía por Pneumocystis
jirovecii
b. Tuberculosis Pulmonar d. Histoplasmosis
____ 27. Señale lo correcto con respecto al tratamiento de los trastornos hipertensivos en el
embarazo:
a. Nifedipina se administra por vía c. Labetalol se administra con
sublingual seguridad durante la lactancia
b. Alfa Metildopa es segura para el d. Nifedipina se administra con
feto y el recién nacido precaución durante la lactancia
____ 28. Las siguientes situaciones son causa de neumonía vinculada con la atención de la salud
debido a Staphylococcus aureus resistente a meticilina. EXCEPTO:
a. Antibioticoterapia en los 90 días c. Cuidado de heridas en el hogar
anteriores
b. Hospitalización mayor a 48 horas d. Venoclisis en el hogar

____ 29. ¿Qué tratamiento se debe utilizar en el síndrome SAHA (seborrea-alopecia- hirsutismo-
acné)?
a. Macrólidos c. Tetraciclinas
b. Dapsona d. Hormonal
____ 30. La forma eritematosa de la candidiasis bucal puede darse con más frecuencia en
pacientes con:
a. Antibioticoterapia c. HIV negativos
b. Terapia antimicótica d. Uso de corticoides sistémicos
____ 31. ¿Cuál es el diagnóstico de un paciente con piel seca y áspera, extremidades frías,
alopecia difusa, bradicardia, cansancio y aumento de peso con escaso apetito?
a. Hipotiroidismo c. Hipertiroidismo
b. Enfermedad de Graves d. Tiroiditis subaguda

____ 32. ¿Qué medicamento controla los factores fisiopatológicos que producen acné?
a. Peróxido de benzoilo c. Isotretinoína
b. Ácido azelaico d. Retinoides tópicos

4
Name: ________________________ ID: A

____ 33. ¿Qué causa de hipertensión arterial origina presión diferencial del pulso amplia?
a. Reflujo aórtico c. Feocromocitoma
b. Hipotiroidismo d. Polineuritis

____ 34. De las siguientes causas de insuficiencia renal aguda ¿Cuál es de origen postrenal?
a. Necrosis tubular aguda c. Litiasis renal
b. Ascitis grave d. Glomerulonefritis

____ 35. Señale la complicación de Pancreatitis Aguda que se asocia a fallo orgánico:
a. Colección necrótica aguda c. Pseudoquiste
b. Colección pancreática tabicada d. Colección retroperitoneal

____ 36. Paciente femenina de 30 años de edad, ha sido diagnosticada con diabetes 1 desde su
infancia, ingresa por presentar cetoacidosis diabética, presentando al examen físico:
TA 95/70, FC: 140 x min, fiebre, delirio, convulsiones, coma, vómito, diarrea e ictericia.
La palpación anterior del cuello demuestra un moderado aumento glandular pero no es
posible obtener otros datos por tener la paciente cuello corto y obesidad; los exámenes
de laboratorio reportan TSH 0.01 uUZ L, T4 6 ng/ mL, T3 de 18 pg/100 mL. ¿Qué tipo
de bocio ocasiona el cuadro clínico descrito?
a. Difuso no tóxico c. Coloide Juvenil
b. Difuso tóxico d. Multinodular no tóxico

____ 37. ¿Cuál es el período más crucial del embarazo y la meta de control glucémico para evitar
malformaciones fetales en pacientes diabéticas?
a. Durante el parto - HbA1c 6.1% c. Segundo trimestre - HbA1c > 8 %
b. Primer trimestre HbA1c < 6.4 % d. Tercer trimestre - HbA 1c < 5.6 %

____ 38. Indique causas de pancreatitis aguda, EXCEPTO:


a. Hipertrigliceridemia c. Alcoholismo
b. Colelitiasis d. Helicobacter Pylori

____ 39. Señale qué medicamento ocasiona exantema acneiforme o agravar un acné previo:
a. Alquitrán de Hulla líquido cutáneo c. Betametasona líquido cutáneo 0.1
5% %
b. Urea semisólido cutáneo 5 % d. Acido fusídico semisólido cutáneo
2%
____ 40. Signo de Prehn positivo (disminución del dolor del testículo al elevar). Indique en que
patología se encuentra presente:
a. Orquiepididimitis c. Hidrocele
b. Torsión testicular d. Gangrena de Fournier
____ 41. ¿Qué hallazgo caracteriza a la reacción transfusional retardada en sangre periférica?
a. Hemolisis con elevación de c. Hemolisis osmótica intravascular
Hemoglobina libre
b. Eritroblastos en sangre periférica d. Hemolisis con Hcto bajo
____ 42. ¿Qué medicamento de primera elección se utiliza para el tratamiento de la onicomicosis?
a. Itraconazol c. Anfotericina B
b. Flucitosina d. Fluconazol

5
Name: ________________________ ID: A

____ 43. Paciente de 50 años en tratamiento anticoagulante con dicumarínicos. De las


siguientes pruebas cual mide la actividad del anticoagulante:
a. Tiempo de sangría c. Tiempo de protrombina
b. Fibrinógeno d. Tiempo de trombina

____ 44. ¿Qué dato de espirometría se interpreta como obstrucción muy intensa (del flujo de aire)
Etapa (IV) de GOLD en la EPOC?
a. FEV1 > 30 % pero < 50 % de la c. FEV1 < 30 % de la cifra prevista
cifra prevista
b. FEV1 > 50 % pero < 80 % de la d. FEV1 > 80 % de la cifra prevista
cifra prevista
____ 45. ¿Cuál es el tratamiento ambulatorio empírico e inicial de la neumonía adquirida en la
comunidad, para un paciente previamente sano que no ha usado antimicrobianos los
últimos tres meses?
a. Un carbapenémico c. Un macrólido
b. Una flouroquinolona respiratoria d. Betalactámico y macrólido

____ 46. ¿Cuál es el diagnóstico de un paciente de 65 años asintomático, que repetidamente


presenta cifras tensionales de 140/90 mm Hg?
a. Hipertensión arterial estadio 2 c. Presión arterial normal
b. Pre hipertensión d. Hipertensión arterial estadio 1

____ 47. El uso de isotretinoína produce:


a. Aumenta la producción de sebo c. Aumenta la queratinización de la
piel
b. Inflamación de la piel d. Sequedad extrema de la piel y
queilitis
____ 48. Paciente de 15 años, que acude en compañía de un familiar, quien refiere que la
paciente presenta de forma brusca caída de la cabeza sobre el pecho y en ocasiones
caída al suelo de forma súbita ¿Cuál es el tipo de convulsión que presenta la paciente?
a. De ausencia c. Crisis focal motora
b. Mioclónica d. Atónica

____ 49. Según la Guía de diabetes del MSP, los siguientes son criterios de diagnóstico para
Diabetes Mellitus tipo 2, EXCEPTO:
a. Glucosa en ayunas mayor a 126 c. Glucosa mayor o igual a 200
mg/dl por dos ocasiones mg/dl, 2 horas después de
sobrecarga de glucosa, por dos
ocasiones
b. Hemoglobina glicosilada mayor o d. Hiperglucemia con síntomas
igual a 6.5%. clásicos y glucosa al azar mayor o
igual a 200 mg/dl
____ 50. Son factores de riesgo para infección de vías urinarias, EXCEPTO:
a. Obstrucción urinaria c. Diabetes mellitus
b. Ingesta de alcohol d. Estado de gravidez

____ 51. Señale la complicación de Pancreatitis Aguda que se asocia a fallo orgánico:
a. Colección retroperitoneal c. Colección pancreática tabicada
b. Pseudoquiste d. Colección necrótica aguda

6
Name: ________________________ ID: A

____ 52. La forma eritematosa de la candidiasis bucal puede darse con más frecuencia en
pacientes con:
a. Antibioticoterapia c. Terapia antimicótica
b. HIV negativos d. Uso de corticoides sistémicos
____ 53. Paciente de 25 años con antecedente de parto reciente presenta palpitaciones, disnea,
dolor pleurítico, hemoptisis; al examen físico se evidencia fiebre de 38°C, cianosis,
taquipnea, taquicardia, estertores pulmonares. En exámenes solicitados: gasometría
arterial normal, EKG reporta taquicardia sinusal, Rx de tórax reporta atelectasias
básales con elevación del hemidiafragma, ensanchamiento de los hilios pulmonares
bilateralmente, infiltrado alveolar basal borde inferior convexo (joroba de Hampton),
afilamiento de imágenes vasculares. ¿Cuál es el tratamiento anticoagulante que debe
administrar al ingreso, antes de referir a UCI?
a. Clopidrogrel c. Warfarina
b. Heparina d. Estreptocinasa

____ 54. ¿Cuál es el diagnóstico más probable, de un paciente de 70 años que presenta marcado
malestar general y ortostatismo, que inició hace 1 hora? Al examen físico se objetiva
pulso regular, FC: 31 pulsaciones por minuto y PA: 80/50 mm Hg
a. Bloqueo AV completo c. Enfermedad del nodo sinusal
b. Bradicardia sinusal d. Síndrome de
Wolf-Parkinson-White
____ 55. ¿Qué medicamento se utiliza en el tratamiento del acné vulgar leve?
a. Metronidazol c. Tetraciclina
b. Isotretinoína d. Clobetasol

____ 56. Indique causas de pancreatitis aguda, EXCEPTO:


a. Helicobacter Pylorí c. Alcoholismo
b. Colelitiasis d. Hipertrigliceridemia

____ 57. Señale qué medicamento de los usados en el tratamiento del acné (enfocándose en la
seguridad de su uso durante el embarazo) corresponde a Categoría X:
a. Isotrenitoína c. Doxiciclina
b. Adapaleno d. Cotrimoxazol

____ 58. ¿Cuál es la conducta terapéutica inicial en un paciente de 45 años, sin antecedentes de
interés, que en un chequeo médico, se objetivan los siguientes resultados: Colesterol
total 260 mg/dl, HDL:34 mg/dl; LDL:140 mg/dl; Trigl¡céridos:550 mg/dl?
a. Prescribir una estatina y un fibrato c. Prescribir una estatina
b. Dieta, ejercicio y control en 3 d. Prescribir un fibrato
meses
____ 59. Paciente de 45 años, obeso, diabético y con hiperuricemia. Refiere que desde hace 24
horas presenta dolor en el miembro inferior derecho e impotencia funcional. Al examen
físico presenta sensación distérmica no cuantificada, pulsos distales presentes, edema
de la pierna derecha, aumento de la temperatura local en la región pretibial, en donde
se observa una placa eritematosa, brillante, caliente y dolorosa de 10 x 8 cm de
diámetro. ¿Cuál es el diagnóstico?
a. Celulitis c. Tromboflebitis
b. Erisipela d. Crisis Gotosa

7
Name: ________________________ ID: A

____ 60. ¿Qué hallazgo caracteriza a la reacción transfusional retardada en sangre periférica?
a. Eritroblastos en sangre periférica c. Hemolisis con elevación de
Hemoglobina libre
b. Hemolisis con Hoto bajo d. Hemolisis osmótica intravascular
____ 61. ¿Cuál es el tratamiento ambulatorio empírico e inicial de la neumonía adquirida en la
comunidad, para un paciente previamente sano que no ha usado antimicrobianos los
últimos tres meses?
a. Un macrólido c. Un carbapenémico
b. Una flouroquinolona respiratoria d. Betalactámico y macrólido

____ 62. Las siguientes situaciones son causa de neumonía vinculada con la atención de la salud
debido a Staphylococcus aureus resitente a meticílina. EXCEPTO:
a. Antibioticoterapia en los 90 días c. Cuidado de heridas en el hogar
anteriores
b. Venoclisis en el hogar d. Hospitalización mayor a 48 horas

____ 63. ¿Qué medicamento se utiliza en el tratamiento de la diabetes gestacional y durante la


lactancia?
a. Metformina c. Glimepirída
b. Simvastatina d. Glicazida
____ 64. Señale el marcador de gravedad de la Pancreatitis aguda en las primeras 24 horas de
hospitalización:
a. Creatinina sérica <1 mg % c. BUN < 15 mg /100 ml
b. Recuento leucocitario > 12000 d. PCO2 > 32mg /100 ml
células/ul
____ 65. Signo de Prehn positivo (disminución del dolor del testículo al elevar). Indique en que
patología se encuentra presente:
a. Gangrena de Fournier c. Torsión testicular
b. Orquiepididimitís d. Hidrocele
____ 66. Paciente de 60 años, que presenta tos persistente, en ocasiones con hemoptisis,
sudoración nocturna, disnea y dolor torácico tipo punzada de costado. En la radiografía
de tórax se muestra un derrame pleural de tamaño moderado. ¿Cuál es el diagnóstico?
a. Histoplasmosis c. Neumonía por Pneumocystis
jirovecii
b. Tuberculosis Pulmonar d. Absceso pulmonar
____ 67. ¿Cuál es el medicamento recomendado para el tratamiento de pacientes adultos con
diabetes y nefropatía?
a. Espironolactona c. Hidralazina
b. Metildopa d. Valsarían
____ 68. ¿Qué diurético utilizado en el tratamiento de la hipertensión arterial puede originar
impotencia sexual?
a. Triamtereno c. Espironolactona
b. Clortalidona d. Amilorida
____ 69. ¿Qué medicamento de primera elección se utiliza para el tratamiento de la onicomicosis?
a. Itraconazol c. Anfotericina B
b. Flucitosina d. Fluconazol

8
Name: ________________________ ID: A

____ 70. Señale el enunciado correcto con relación al síndrome nefrítico agudo:
a. Proteinuria > 3 g/ 24h + c. Proteinuria < 4 g/ 24h + Edema
Hipercolesterolemia intrarrenal
b. Proteinuria > 4 g/ 24h + d. Proteinuria < 2 g /24 h +
Hipercoagulabilidad Hematuria
____ 71. ¿Cuál es el diagnóstico de un paciente adulto con obesidad central, circunferencia de la
cintura mayor a 94 cm, niveles de triglicéridos 170 mg/dl, niveles de colesterol HDL de
60 mg/dL, presión arterial elevada 140/90 y con diabetes tipo 2 previamente
diagnosticada?
a. Diabetes Mellitus 1 c. Síndrome metabólico
b. Síndrome de Cushing d. Hiperprolactinemia

____ 72. La principal indicación para el tratamiento del Helicobacter pylori es:
a. Portadores de Helicobacter pylori c. Úlcera gástrica o duodenal activa
b. Complicaciones de reflujo d. Antecedente familiar de cáncer
gastroesofágico gástrico
____ 73. ¿Cuál es el tratamiento de la ascitis en pacientes con cirrosis?
a. Restricción estricta de la sal a 400 c. Administrar diuréticos si hay
mg de sodio al día respuesta a la restricción de sal
b. Vigilar el índice de masa corporal d. Restricción de líquidos en todos
y sodio urinario los pacientes con cirrosis
____ 74. Los siguientes son factores epidemiológicos relacionados al desarrollo de neumonía
extra hospitalaria. EXCEPTO:
a. Demencia c. Alcoholismo
b. Sedentarismo d. Tabaquismo
____ 75. Son factores que producen macrosomía fetal. EXCEPTO:
a. Mayor resistencia a la insulina c. Insulina materna
b. Hiperglucemia materna d. Tolerancia anormal a la glucosa

____ 76. ¿Cuál es el período más crucial del embarazo y la meta de control glucémico para evitar
malformaciones fetales en pacientes diabéticas?
a. Primer trimestre HbA1c < 6.4 % c. Durante el parto - HbA1c 6.1%
b. Segundo trimestre - HbA1c > 8 % d. Tercer trimestre - HbA 1c < 5.6 %
____ 77. ¿Cuál es el diagnóstico de un paciente con piel seca y áspera, extremidades frías,
alopecia difusa, bradicardia, cansancio y aumento de peso con escaso apetito?
a. Enfermedad de Graves c. Tiroiditis subaguda
b. Hipotiroidismo d. Hipertiroidismo

____ 78. ¿Qué dato de espirometría se interpreta como obstrucción muy intensa (del flujo de aire)
Etapa (IV) de GOLD en la EPOC?
a. FEV1 < 30 % de la cifra c. FEV1 > 30 % pero < 50 %
prevista de la cifra prevista
b. FEV1 > 80 % de la cifra d. FEV1 > 50 % pero < 80 %
prevista de la cifra prevista
____ 79. ¿Qué medicamento controla los factores fisiopatológicos que producen acné?
a. Retinoides tópicos c. Ácido azelaico
b. Isotretinoína d. Peróxido de benzoilo

9
Name: ________________________ ID: A

____ 80. Señale lo correcto con respecto al tratamiento de los trastornos hipertensivos en el
embarazo:
a. Labetalol se administra con c. Alfa Metildopa es segura para el
seguridad durante la lactancia feto y el recién nacido
b. Nifedipina se administra con d. Nifedipina se administra por vía
precaución durante la lactancia sublingual
____ 81. ¿Qué causa de hipertensión arterial origina presión diferencial del pulso amplia?
a. Reflujo aórtico c. Hipotiroidismo
b. Feocromocitoma d. Polineuritis

____ 82. De las siguientes causas de insuficiencia renal aguda ¿Cuál es de origen posrenal?
a. Ascitis grave c. Glomerulonefritis
b. Necrosis tubular aguda d. Litiasis renal

____ 83. Señale la contraindicación al uso de enalapril en la diabetes con hipertensión arterial:
a. Hipertensa con embarazo c. Normotensa con microalbuminuria
b. Hipertensa con nefropatía d. Hipertensa con albuminuria

____ 84. Paciente de 57 años, con antecedentes de hepatopatía crónica de origen idiopático, es
llevado a emergencias por un cuadro de aparición insidiosa caracterizado por deterioro
del estado de conciencia, dificultad para el habla, desorientación mínima y amnesia de
los eventos recientes ¿Cuál es el diagnóstico?
a. Hipoglicemia c. Hiponatremia
b. Enfermedad cerebrovascular d. Encefalopatía hepática

____ 85. La característica más común de la disfunción autonómica de la diabetes:


a. Anhidrosis de piernas c. Bradicardia en reposo
b. Hiperglicemia d. Aumento del Vaciamiento
Gástrico
____ 86. Paciente de 50 años en tratamiento anticoagulante con dicumarínicos. De las siguientes
pruebas cual mide la actividad del anticoagulante:
a. Tiempo de protrombina c. Fibrinógeno
b. Tiempo de trombina d. Tiempo de sangría

____ 87. Paciente sin cardiopatía, refiere episodios anteriores al presente de palpitaciones,
ansiedad, dolor torácico, disnea, que ceden espontáneamente. Presenta el siguiente
trazo electrocardiográfico indique el algoritmo de tratamiento a seguir.

a. Verapamilo + maniobras vagales c. Verapamilo + amiodarona +


+ procainamida adenosina
b. Maniobras vagales + adenosina+ d. Adenosina + amiodarona +
procainamida maniobras vagales

10
Name: ________________________ ID: A

____ 88. Paciente femenina de 30 años de edad, ha sido diagnosticada con diabetes 1 desde su
infancia, ingresa por presentar cetoacidosis diabética, presentando ai examen físico:
TA 95/70, FC: 140 x min, fiebre, delirio, convulsiones, coma, vómito, diarrea e ictericia.
La palpación anterior del cuello demuestra un moderado aumento glandular pero no es
posible obtener otros datos por tener la paciente cuello corto y obesidad; los exámenes
de laboratorio reportan TSH 0.01 uU/ L, T4 6 ng/ mL, T3 de 18 pg/100 mL. ¿Qué tipo de
bocio ocasiona el cuadro clínico descrito?
a. Difuso no tóxico c. Multinodular no tóxico
b. Difuso tóxico d. Coloide Juvenil

____ 89. ¿Cuál es el diagnóstico de un paciente de 65 años asintomático, que repetidamente


presenta cifras tensionales de 140/90 mm Hg?
a. Presión arterial normal c. Hipertensión arterial estadio 2
b. Hipertensión arterial estadio 1 d. Pre hipertensión

____ 90. Señale el algoritmo para el tratamiento de taquicardia ventricular con < 150/lmin (estable)
que ha producido un paro cardíaco completo, luego de que el paciente ha recibido tres
choques no sincronizados en secuencia (200J, 300J, 360J) y de que ha sido intubado y
se ha establecido una línea endovenosa
a. Atropina - dopamina c. Lidocaína - procainamida - bretilio
b. Epinefrina - Atropina d. Maniobras vagales - adenosina

____ 91. ¿Qué tratamiento se debe utilizar en el síndrome SAHA (seborrea-alopecia-


hirsutismo-acné)?
a. Tetraciclinas c. Hormonal
b. Macrólidos d. Dapsona
____ 92. Señale qué medicamento ocasiona exantema acneiforme o agravar un acné previo:
a. Acido fusídico semisólido cutáneo c. Alquitrán de Hulla líquido cutáneo
2% 5%
b. Urea semisólido cutáneo 5 % d. Betametasona líquido cutáneo 0.1
%
____ 93. Todos los siguientes son síntomas de la enfermedad celiaca, EXCEPTO:
a. Ictericia c. Esteatorrea
b. Pérdida de peso d. Anemia

____ 94. De los siguientes fármacos antituberculosos. ¿Cuál pertenece a los aminoglucósidos?
a. Rifampicina c. Pirazinamida
b. Isoniacida d. Estreptomicina

____ 95. Un paciente varón de 33 años de edad, al conducir una motocicleta a 30 km/h, se
impacta con un vehículo que estaba estacionado. Es llevado al servido de urgencias en
un hospital de segundo nivel. Tiene dolor torácico anterior, por el traumatismo.
Presenta tensión arterial de 87/50 mm de Hg; frecuencia cardíaca de 110 x 1 min;
frecuencia respiratoria de 30 x 1 min; saturación de oxígeno de 85 %. ¿Qué examen
específico se debe solicitar para evaluar su trauma de tórax?
a. Ultrasonografía c. Tomografía simple de tórax
b. Rayos X póstero-anterior de tórax d. Resonancia magnética

11
Name: ________________________ ID: A

____ 96. Joven de 16 años de edad es traído a la emergencia por presentar falta de apetito,
dolor abdominal, nausea y vomito desde hace 24 horas, sin causa aparente, llega
somnoliento. Al examen físico tenemos: FC 94/min, FR 21/min, TA 110/70 mmHg, T
37.4 OC. Mucosas secas, disminución de turgencia, respiración rápida y superficial con
un llamativo olor a frutas. Abdomen es blando, depresible, no visceromegalias, ruidos
hidroaereos presentes. Los exámenes de laboratorio arrojan: Hb 14 g/dl; Hct 39%;
Leucocitos 8000 mm3 (Segmentados: 64%, Linfocitos: 30%, Monocítos: 6%}; Na 135
mEq/dl; K 6 mEq/dl; Cl 105 mEq/dl; Glucosa sérica 470 mg/dl; Cr 0,9 mg/dl; BUN 19
mg/dl. ¿Cuál sería el diagnóstico del paciente?
a. Estado hiperosmolar cetósico c. Cetoacidosis diabética
b. Descompensación simple de d. Estado hiperosmolar no cetósico
diabetes
____ 97. La clasificación de Child Pugh sirve para el estudio de, señale lo correcto:
a. Carcinomas de ovario c. Linfomas no Hodgkin
b. Cirrosis d. Leucemias

____ 98. Un paciente de 28 años de edad, sin antecedentes patológicos, presenta súbitamente un
dolor punzante en el hemitórax derecho, que se irradia a la axila del mismo lado. Se
acompaña de una sensación de falta de aire y latidos cardiacos rápidos. En
emergencia del hospital se realiza un examen físico completo y gasometría arterial.
¿Qué examen de imagen se debe solicitar bajo la sospecha diagnóstica de neumotórax
espontáneo?
a. Una TAC simple sin contraste c. Una Rx AP y lateral de tórax en
máxima inspiración
b. El diagnóstico no amerita estudio d. Una Resonancia Magnética
imagenológico para continuar con Nuclear para ubicar la bula
el tratamiento pulmonar
____ 99. Un peatón sufrió un trauma craneal al ser embestido por un automotor y cayó al piso sin
perder la conciencia. Descansó una hora y luego se levantó confuso e irritable. Luego
se tambaleó y cayó nuevamente al piso. Estaba hipersomne y presentó contracciones
de la mitad izquierda inferior de la cara y del brazo izquierdo. Además, presenta una
fractura a nivel temporo-parietal derecho. Indique su diagnóstico clínico:
a. Lesión por contragolpe de la c. Hematoma extradural por sección
arteria cerebral anterior izquierda de la arteria meníngea media
derecha
b. Infarto cerebral secundario al d. Hematoma subdural por ruptura
traumatismo de la artería meníngea media
izquierda

12
Name: ________________________ ID: A

____ 100. Paciente mujer de 55 años nota cansancio, y debilidad desde hace. algunas semanas.
Acude a consulta médica. El facultativo palpa un pequeño bocio que la paciente no ha
notado. En la anamnesis dirigida, el médico se percata que la paciente ha aumentado
de peso y presenta estreñimiento. Decide realizar una biopsia de tiroides, que reporta
infiltración linfocítica marcada. Elija los resultados de laboratorio que tengan relación
con el caso clínico descrito:
1. TSH elevada
2. TSH baja
3. T4 libre elevada
4. T4 libre baja

a. 2, 3 c. 2, 4
b. 1, 3 d. 1, 4
____ 101. ¿Cuál de los siguientes pacientes con cefalea necesita ser referido con urgencia a un
centro de mayor complejidad?
a. Cefalea de reciente aparición en c. Cefalea tenslonal leve con 3 a 4
trueno que cambia con los episodios mensuales que ceden
movimientos posturales con medicación
b. Cefalea localizada en región d. Cefalea con antecedentes de
retro-orbicular de inicio reciente y migraña, con signos de aura
con lagrimeo inicial
____ 102. Elija ¿Cuál de las siguientes medidas farmacológicas, sería la adecuada en el siguiente
caso? Paciente varón de 38 años de edad acude a un control médico de rutina. Se
encuentran niveles de glicemia en ayunas de 100 mg/dl; glucosa a las 2 horas de 160
mg/dl: hemoglobina A1C 6.6%. El índice de masa corporal del paciente es 35 lcg/m2.
Reporta que su padre es diabético.
a. Glipizida c. Metformina
b. Clorpropamida d. Ploglitazona

____ 103. Un varón de 28 años de edad, nacido y residente en Quito. Desde hace 2 meses se
traslada a vivir en Manabí como trabajador en una granja productora de leche. No
presenta antecedentes patológicos significativos. Refiere que desde hace 2 días tiene
alza térmica, acompañada de malestar general, cefalea pulsátil, sudoración nocturna,
anorexia y dolores articulares. Al momento tiene fiebre de 38°C, astenia, deshidratación
leve y pequeños granulomas de tejido blando en la región inguinal, bilateralmente. El
signo del torniquete es negativo. ¿Cuál es el diagnóstico clínico de este paciente?
a. Brucelosis c. Dengue clásico
b. Dengue hemorrágico d. Fiebre amarilla

13
Name: ________________________ ID: A

____ 104. El día sábado, en urgencias de un hospital rural de Esmeraldas y en temporada


invernal, llega un paciente con síndrome febril y sopor. El laboratorio es incapaz de
hacer con garantías una prueba para diagnóstico de paludismo. La actitud a seguir más
lógica sería:
a. Usar una fluoroqulnolona y c. Hacer una gota gruesa para
esperar la evolución enviar a centro especializado y
tratar como si fuera una infección
por P. falciparum
b. Administrar una cefalosporina de d. Dar un tratamiento sintomático y
generación y esperar la evolución esperar el lunes a que lo revise un
especialista
____ 105. Paciente de 24 años de edad, sin antecedentes de importancia que sufre un traumatismo
de miembros inferiores con fractura de fémur, tibia y peroné del mismo lado. Al
evaluarlo en la emergencia, usted debe decidir si presenta signos de choque "shock".
En este contexto, de entre los siguientes signos clínicos del estado de choque ¿Cuál es
el que con mayor frecuencia ocurre más temprano?
a. Hipotensión c. Taquicardia
b. Confusión d. Bradipnea

____ 106. El siguiente enunciado corresponde al mecanismo de acción de una sustancia: Cubre las
fimbrias de Escherichía coli y evita que esta bacteria se una a los receptores
glucosídicos de las células del urotelio, con lo cual disminuye la colonización de las vías
urinarias por este microorganismo. Seleccione el agente profiláctico para infección de
vías urinarias que realiza la función descrita en el enunciado:
a. Extractos liofillzados de E. coli c. Ácido ascórbico
b. Jugo de arándano d. Nitrofurantoina

____ 107. Un varón de 29 años de edad con antecedentes de tabaquismo desde los 17 años,
presenta bronquitis con tos seca y dolor torácico de intensidad moderada, dolor que
nunca antes lo había sentido. El paciente se encuentra estable. En la radiografía de
tórax póstero-anterior se observa un neumotorax apical derecho. Los siguientes
tratamientos son correctos, EXCEPTO:
a. Oxígeno a alto flujo 10 l/min para c. Aspiración simple de la cavidad
eliminar los otros gases de la pleural con tubo torácico
cavidad pleural
b. Cirugía torácica urgente para d. Observación dependiendo del
manejo inmediato tamaño del neumotorax
____ 108. Una mujer de 58 años de edad fue diagnosticada de hipertensión arterial hace un mes y
ha recibido tratamiento con un antagonista de los canales del calcio. Acude a su control
médico, ¿Cuál de las siguientes alteraciones podría encontrar de manera frecuente, en
pacientes tratados con este tipo de fármacos?
a. Hiperkalemia c. Bronco espasmo
b. Tos seca nocturna d. Edema pretibial

14
Name: ________________________ ID: A

____ 109. Mujer de 43 años tiene desde hace 2 días dolores de labios, encías, lado derecho de la
cara y mentón, por varias ocasiones durante el día y la noche con pocos segundos de
duración; ha notado que se produce al cepillarse los dientes y cuando se expone a
corrientes de aire. Al examen físico presenta signos vitales estables, no hay asimetría
facial, no hay signos de focalización ni signos meníngeos. Según el cuadro clínico, elija
el diagnóstico adecuado para este caso:
a. Neuritis facial c. Parálisis de Bell
b. Neuralgia del V par craneal d. Parálisis facial periférica

____ 110. Niño de un año de edad, con 9 kg de peso, presenta signos de deshidratación grave,
está adormecido y no puede tomar líquidos. En el puesto de salud rural en que se
encuentran, no disponen de un equipo para infusión Intravenosa. Se cuenta con una
sonda nasogástrica (SNG). Indique, ¿Qué cantidad de suero oral, por kg de peso y por
hora, se debe administrar a este niño por la SNG?
a. 30 ml/kg/h = 270 ml/h c. 20 ml/kg/h = 180 ml/h
b. 50 ml/kg/h = 450 ml/h d. 10 ml/kg/h = 90 ml/h

____ 111. Seleccione las dosis de los siguientes fármacos utilizados en la terapia triple contra el
Helicobacter Pylori:
Fármaco Dosis
1. Omeprazol a) 1 g cada 12 horas
2. Amoxlcillna b) 500 mg cada 6 horas
3. Claritromicina c) 500 mg cada 12 horas
d) 20 mgeada 12 horas

a. 1d, 2a, 3c c. 1a, 2b, 3c


b. 1d, 2c, 3a d. 1c, 2a, 3b
____ 112. La inmunodefidencla del paciente adulto con infección por VIH, que ya presenta
evidencia de SIDA, se explica por:
a. Defecto en la quimiotaxis, c. Linfopenia así como
fagocitosis y muerte celular desregulación de Iinfocitos
cooperadores
b. Alteración de sus barreras d. Pérdida de la memoria humoral
inespecíficas
____ 113. Son considerados como factores de riesgo predisponentes del cáncer gástrico,
EXCEPTO:
a. Consumo prolongado de c. Mutación autosómica dominante
inhibidores de la bomba de en el gen de la E-caderina
protones
b. Consumo prolongado de d. Helicobacter Pylori como
alimentos ahumados y salados productor de gastritis crónica
____ 114. ¿Cuál de los siguientes es el fármaco de elección para el tratamiento de la Infección
aguda de Tripanozoma cruzí?
a. Alopurinol c. Itraconazol
b. Benznidazole d. Fluconazol

15
Name: ________________________ ID: A

____ 115. Un niño de 8 años de edad presenta desde hace 5 días dolor abdominal tipo cólico, con
aumento progresivo de intensidad. Además, tiene frecuencia aumentada de sus
deposiciones en 5 - 6 al día, de cantidad variable, tienen consistencia líquida, con
presencia de moco, sangre y sin material purulento o de otro tipo. Últimamente
presenta pujo al deponer. Su estado de conciencia y sus signos vitales son normales;
sus mucosas están semihúmedas, sus conjuntivas son algo pálidas. Presenta dolor
abdominal moderado a la palpación, especialmente en el marco colónico. ¿Cuál de los
siguientes parásitos produce el cuadro clínico descrito?
a. Entamoeba histolytica c. Taenia solium
b. Giardia lamblia d. Ascaris lumbricoides
____ 116. Un paciente masculino, de 28 años de edad, fumador. Desde hace 3 semanas presenta
tos seca, rinorrea cristalina, odinofagia. No registra fiebre. No tiene disnea ni
sibilancias. Se establece el diagnóstico de bronquitis aguda y se decide iniciar manejo
inicial primarlo. Seleccione el enunciado correcto:
a. El tratamiento con agonistas 8-2 c. Los agentes mucolíticos ayudan a
está recomendado en pacientes mejorar la tos
no complicados
b. Se debe recomendar dejar de d. El tratamiento con antibióticos es
fumar, hidratación adecuada y ideal para todos estos pacientes
lavado de manos
____ 117. De las siguientes opciones, Indique el parámetro del hemograma con mayor sensibilidad
para diagnosticar anemia ferropénica:
a. Volumen corpuscular medio c. Ancho de distribución eritrocitaria
mayor a 80 fl mayor a 14.5 %
b. Hemoglobina corpuscular media d. Hematocrito de 45 %
de 36 pg
____ 118. Una mujer de 69 años de edad con alcoholismo crónico, última ingesta hace una
semana. El día de hoy, de manera súbita, presenta decaimiento y deposiciones negras,
pegajosas y fétidas. Al examen físico tiene ascitis, red venosa superficial visible en el
abdomen y su hígado no es palpable. En esta paciente, de entre las siguientes, ¿Cuál
es la localización más probable del sangrado?
a. Curvatura mayor del estómago c. Tercio distal del esófago
b. Segunda porción del duodeno d. En el fundos gástrico

____ 119. Indique la curva de la Organización Mundial de la Salud que se utiliza para valorar la
desnutrición global:
a. Peso/talla c. Peso/edad
b. IMC/edad d. Talla/edad

16
Name: ________________________ ID: A

____ 120. Un paciente presenta los siguientes datos: TSH, alto; 74 total y libre, bajos. Las
siguientes son manifestaciones clínicas que se corresponden con estos datos de
laboratorio, EXCEPTO:
1. Espasticidad.
2. Exoftalmus.
3. Diarrea.
4. Piel áspera y seca.
5. Intolerancia al frío.
6. Mlxedema pretlbial.

a. 3, 4, 6 c. 1, 2, 5
b. 2, 3, 6 d. 1, 4, 5
____ 121. Un adolescente de 15 años de edad, sufrió en el glúteo un traumatismo con un objeto
cortante, contaminado con barro. Presenta edema, eritema y dolor moderado, con una
valoración de EVA 7/10. Se observan lesiones por abrasión en el sitio de la lesión; el
eritema es de 10 cm en su diámetro mayor y tiene márgenes poco diferenciados, y
febril (33°C). No hay signos de compromiso sistémico, manifestaciones de necrosis o
de contenido purulento. Indique, ¿Cuál es el diagnóstico clínico?
a. Miositis c. Fascitis necrotizante
b. Celulitis bacteriana d. Absceso subcutáneo

17
Name: ________________________ ID: A

____ 122. Niña de 1 año 6 meses que durante su control de salud presenta la siguiente curva de
crecimiento:

¿Cómo interpreta la siguiente curva de crecimiento?


a. La tendencia de crecimiento es c. La tendencia de crecimiento es
normal ya que el peso y la talla plana, colocando al niño en riesgo
transcurren por el mismo percentil de sufrir malnutrición
b. El peso y la talla están normales d. La tendencia de crecimiento es
pero la tendencia de crecimiento normal ya que aumenta el
es plana percentil con relación a la edad
____ 123. Un chofer de 60 años de edad, acude a emergencia porque desde hace
aproximadamente 2 horas tiene un dolor leve y sordo a nivel torácico, acompañado de
palpitaciones. Refiere que este dolor es intermitente, punzante y que aumentó al subir
las gradas del hospital. El dolor se irradia hacia la "paleta" izquierda. Al examen físico
el paciente es de constitución pícnica, con una obesidad de predominio central (IMC de
35). Su presión arterial es de 155/95 mmHg y la frecuencia cardíaca de 39 x min. El
paciente no presenta sudores fríos o sensación de entumecimiento. Su auscultación
cardiaca es normal y su SO2 91 %. Hace unos 6 meses y a consecuencia de una
actividad física leve, presentó un dolor de similares características, que había
presentado en algunas oportunidades anteriores y en similares circunstancias. Indique
el diagnóstico clínico del paciente:
a. Infarto agudo de ventrículo c. Angina de pecho estable por un
derecho con edema agudo de ateroma coronario
pulmón
b. Infarto agudo de miocardio del d. Insuficiencia cardiaca aguda
ventrículo izquierdo dependiente del ventrículo
derecho

18
Name: ________________________ ID: A

____ 124. Indique el medicamento adecuado para el tratamiento inicial, hasta disponer del
resultado del urocultivo, en infecciones de vías urinarias no complicadas, considerando
su eficacia y seguridad:
a. Trimetropin/sulfametoxazol c. Amoxicillna
b. Nitrofurantoína d. Ácido nalidíxico

____ 125. Un varón de 45 años de edad, sin antecedentes patológicos significativos, acude a
emergencias con disnea de 4 días de evolución, que se presenta en reposo y se
intensifica en decúbito dorsal, llegando a producir ortopnea, disnea paroxística nocturna
y trepopnea. Se acompaña de astenia y adinamia posterior a cualquier tipo de actividad
física, lo cual impide su ejecución. En los últimos dos meses ha perdido 20 kg de peso.
Presenta temblor fino distal en miembros superiores, pérdida de cabello e hiperhidrosis.
Por el cuadro clínico y su diagnóstico presuntivo, ¿Cuáles son los resultados de las
pruebas de función tiroidea de este paciente?
a. TSH disminuido y T4 libre elevado c. TSH elevado y T4 libre elevado
b. TSH normal y T4 libre disminuido d. TSH disminuido y T4 libre normal

____ 126. Una niña de 7 años de edad presenta desde hace 10 días dolor abdominal, realiza
deposiciones explosivas, luego de comidas, líquidas, abundantes, con restos
alimentarios, amarillentas, con olor fétido intenso, espumosas, grasosas y sin presencia
de moco, sangre ni material purulento. Se repiten hasta 4 o 5 veces al día. Tiene
además flatulencia intensa. Sus signos vitales son normales, sus mucosas están
semi-húmedas y sus conjuntivas son rosadas. A la palpación presenta un dolor
abdominal leve, difuso, algo más significativo en epigastrio y sona duodenal. A la
auscultación se escuchan ruidos hidroaéreos aumentados, en gran cantidad. ¿Cuál de
los siguientes parásitos genera el cuadro clínico descrito?
a. Ancylostoma duodenale c. Balantidium coli
b. Entamoeba histolytica d. Giardia lamblia
____ 127. Mientras usted realiza su rotación de Neumología, se le encarga el cuidado de un
paciente de 72 años que fue internado por presentar: disnea, dolor de tipo pleurítico,
fiebre y baja de peso. Eí examen físico realizado por el residente muestra una FR de
24/min, disminución de entrada de aire en el lóbulo pulmonar inferior izquierdo
acompañado de un sonido mate a la percusión. Se solicita una placa AP y lateral de
tórax que muestra un derrame pleural, se realiza una toracocentesls obteniéndose un
fluido pleural de tipo hemorrágico. El residente antes de terminar su turno ordena los
siguientes análisis: LDH sérico 110 U/L (6M: 45-90), proteínas totales 7 g/dl (6.0-7.8
g/dL), LDM fluido pleural 230 U/L, proteínas en fluido pleural 14 g/dl. Elija el diagnóstico
más probable de este caso:
a. Derrame pleural de tipo exudado c. Derrame pleural de tipo trasudado
por malignidad por falla cardiaca
b. Derrame pleural de tipo trasudado d. Derrame pleural de tipo exudado
por embolia pulmonar por neumonía
____ 128. Joven migrante de 23 años con historia de promiscuidad bisexual y drogadicción,
consulta por fiebre, malestar general y dolor en glande. La exploración física muestra
múltiples tatuajes, "piercings" y la presencia de tres lesiones vesiculosas en glande y
adenopatías inguinales bilaterales. ¿Cuál es el diagnóstico más probable?
a. Gonorrea c. Sífilis
b. Herpes genital d. Condilomas acuminados

19
Name: ________________________ ID: A

____ 129. La tinción de elección para Mycobacteríum es:


a. Cristal violeta c. Ziehl-Nelssen
b. Gram d. Giemsa

____ 130. En un hemograma, ¿Cuál de los siguientes parámetros tiene mayor utilidad para
diagnosticar anemia ferropénica?
a. Volumen corpuscular medio c. Ancho de distribución eritrocitaria
inferior a 80 fl mayor a 14.5 %
b. Hemoglobina corpuscular media d. Hematocrito de 45 %
de 36 pg
____ 131. Un paciente de 22 años, presentó faringoamigdalitis estreptocócíca hace 3 semanas.
Desde hace 10 días tiene fiebre alta, decaimiento, malestar general, irritabilidad,
movimientos involuntarios, alteraciones del carácter, odinofagia y disfagia. Refiere
también dolor e hinchazón en diversas articulaciones. Actualmente presenta congestión
amigdalina, con exudado purulento y petequias en paladar; adenopatías submaxilares y
latero- cervicales marcadas y dolorosas a la palpación; fiebre alta y en ciertas partes,
exantema escarlatiniforme. Presenta leucocitosis y niveles de PCR 4 veces por encima
de lo normal. ¿Cuál de las siguientes alternativas permite establecer el diagnóstico de
fiebre reumática?
a. Poliartritis, fiebre y leucocitosis c. Poliartritis, es suficiente para
establecer el diagnóstico
b. Leucocitosis, aumento de PCR y d. Leucocitosis, irritabilidad y
fiebre movimientos involuntarios
____ 132. Una mujer de 28 años de edad, casada, 2 hijos, presenta infección de vías urinarias
(IVU), dado que las IVU son producidas frecuentemente por Escherchia cotí, el médico
que la atiende decide administrar cefuroxima 500mg cada 12 horas durante 10 días.
¿Qué tipo da cefalosporina usa el médico?
a. De tercera generación, útil c. De segunda generación, útil
especialmente para gérmenes principalmente para gérmenes
gramnegativos y muy poco para gramnegativos y algunos
grampositivos grampositivos
b. De cuarta generación, útil d. De primera generación, útil
principalmente para gérmenes especialmente para gérmenes
gramnegativos y algunos grampositivos y algunos
grampositivos gramnegativos
____ 133. Un paciente de 15 años tiene diagnóstico de neumonía adquirida en ía comunidad por
MycopSasma pneumonías. ¿Cuál de los siguientes antibióticos es el adecuado para
tratarlo en forma ambulatoria?
a. Claritromicina c. Sultamicilina
b. Amoxicilina d. Céfpodoxima
____ 134. Un paciente acude a consulta por presentar una erupción macutopapular y rosada,
asentada en tronco, muslos y palmas, de forma simétrica. Se encuentra afebril, pero
asténico, tiene dolor de garganta y cefalea leve. Tras realizarle varias pruebas, se
diagnostica sífilis. Al respecto, ¿cuál de las siguientes afirmaciones es correcta?
a. El tratamiento de sífilis siempre c. Si el paciente es positivo para
debe incluir probenecid VIH, el cuadro clínico no cambia
b. Para el diagnóstico de sífilis se d. El tratamiento de elección es
requiere punción lumbar penicilina G benzatinlca

20
Name: ________________________ ID: A

____ 135. Si a un niño se le administra una suspensión antialérgica, cuyo principio activo es
prednisolona, es probable que los próximos días el niño muestre:
a. Muguet c. Retención urinaria
b. Visión borrosa d. Coluria

____ 136. Un paciente con trauma torácico, presenta dificultad respiratoria, cianosis, sus venas del
cuello están distendidas, presenta desviación de la tráquea hacia el lado izquierdo, con
asimetría del tórax y ruidos respiratorios hipofonéticos en el lado derecho. Además, se
encuentra taquipnélco y con hipotensión sostenida. Por el cuadro clínico sabemos que
se trata de neumotórax a tensión. Elija el procedimiento adecuado:
a. Toracocentesis en el octavo c. Toracocentesis en el segundo
espacio Intercostal, línea axilar espacio intercostal, línea medio
posterior derecha clavicular izquierda
b. Toracocentesis en el segundo d. Toracocentesis en el octavo
espacio intercostal, línea medio espacio intercostal, línea axilar
clavicular derecha posterior izquierda
____ 137. Los siguientes componentes del síndrome metabólico son correctos, EXCEPTO:
a. HDL mayor de 40 mg/dl en c. Presión arterial mayor de 130/85
varones y 50 mg/dl en mujeres mm Hg
b. Glucosa en ayunas mayor de 100 d. Trlglicérldos mayor o igual a 150
mg/dl mg/dl
____ 138. Paciente de 25 años, con ingesta alcohólica de 4 días presenta náusea y vómito
frecuente, luego de 4 horas de evolución el vómito es con sangre por varias ocasiones.
Al examen físico: PC 100 latidlos por minuto, FR 18 por minuto, TA 90/60, palidez
generalizada. Según el probable diagnóstico, decida la medida más acertada en el
manejo:
a. Se trata del Síndrome de c. Se trata de hemorragia digestiva
Mallory-Weiss y necesita alta por cirrosis hepática
endoscopia para confirmar el alcohólica y necesita endoscopia
diagnóstico para confirmar el diagnóstico
b. Se trata de hemorragia digestiva d. Se trata del Síndrome de
alta evidente por lo que no se Boerhaave y necesita endoscopia
debe realizar endoscopia para para confirmar el diagnóstico
confirmar el diagnóstico
____ 139. Lactante de 3 meses de edad, presenta dermatitis seborreica. Los siguientes
tratamientos son correctos, EXCEPTO:
a. Retirar las costras con compresas c. Lavar con champú de ketoconazol
tibias con aceite de oliva, seguida al 2%
de champú para bebé
b. Administrar prednisona 2 d. Aplicar crema de hidrocortisona al
mg/kg/dla, vía oral, por un mes 1 a 2.5 %
____ 140. De las siguientes opciones, indique el parámetro del hemograma con mayor sensibilidad
para diagnosticar anemia ferropénica:
a. Volumen corpuscular medio c. Ancho de distribución eritrocitaria
mayor a 80 fl mayor a 14.5 %
b. Hemoglobina corpuscular media d. Hematocrito de 45 %
de 36 pg

21
Name: ________________________ ID: A

____ 141. La clasificación de Child Pugh sirve para el estudio de, señale lo correcto:
a. Leucemias c. Carcinomas de ovario
b. LInfomas no Hodgkin d. Cirrosis

____ 142. Una mujer de 28 años de edad, casada, 2 hijos, presenta infección de vías urinarias
(IVU), dado que las IVU son producidas frecuentemente por Escherchia coli, el médico
que la atiende decide administrar cefuroxima SOOmg cada 12 horas durante 10 días.
¿Qué tipo de cefalosporlna usa el médico?
a. De tercera generación, útil c. De primera generación, útil
especialmente para gérmenes especialmente para gérmenes
gramnegativos y muy poco para grampositivos y algunos
grampositivos gramnegativos
b. De segunda generación, útil d. De cuarta generación, útil
principalmente para gérmenes principalmente para gérmenes
gramnegativos y algunos gramnegativos y algunos
grampositivos grampositivos
____ 143. Un varón de 29 años de edad con antecedentes de tabaquismo desde los 17 años,
presenta bronquitis con tos seca y dolor torácico de intensidad moderada, dolor que
nunca antes lo había sentido. El paciente se encuentra estable. En la radiografía de
tórax póstero-anterior se observa un neumotórax apical derecho. Los siguientes
tratamientos son correctos, EXCEPTO:
a. Aspiración simple de la cavidad c. Observación dependiendo del
pleural con tubo torácico tamaño del neumotorax
b. Cirugía torácica urgente para d. Oxígeno a alto flujo 10 l/min para
manejo inmediato eliminar los otros gases de la
cavidad pleural
____ 144. Si a un niño se le administra una suspensión antialérgica, cuyo principio activo es
prednisolona, es probable que los próximos días el niño muestre:
a. Visión borrosa c. Muguet
b. Coluria d. Retención urinaria

____ 145. Un chofer de 60 años de edad, acude a emergencia porque desde hace
aproximadamente 2 horas tiene un dolor leve y sordo a nivel torácico, acompañado de
palpitaciones. Refiere que este dolor es intermitente, punzante y que aumentó al subir
las gradas del hospital. El dolor se irradia hacia la "paleta" izquierda. Al examen físico
el paciente es de constitución pícnica, con una obesidad de predominio central (IMC de
35). Su presión arterial es de 155/95 mmHg y la frecuencia cardiaca de 39 x min. El
paciente no presenta sudores fríos o sensación de entumecimiento. Su auscultación
cardiaca es normal y su SO2 91 %. Hace unos 6 meses y a consecuencia de una
actividad física leve, presentó un dolor de similares características, que había
presentado en algunas oportunidades anteriores y en similares circunstancias. Indique
el diagnóstico clínico del paciente:
a. Infarto agudo de ventrículo c. Insuficiencia cardiaca aguda
derecho con edema agudo de dependiente del ventrículo
pulmón derecho
b. Angina de pecho estable por un d. Infarto agudo de miocardio del
ateroma coronario ventrículo izquierdo

22
Name: ________________________ ID: A

____ 146. Un paciente acude a consulta por presentar una erupción maculopapular y rosada,
asentada en tronco, muslos y palmas, de forma simétrica. Se encuentra afebril, pero
asténico, tiene dolor de garganta y cefalea leve. Tras realizarle varias pruebas, se
diagnostica sífilis. Al respecto, ¿cuál de las siguientes afirmaciones es correcta?
a. Si el paciente es positivo para c. Para el diagnóstico de sífilis se
VIH, el cuadro clínico no cambia requiere punción lumbar
b. El tratamiento de elección es d. El tratamiento de sífilis siempre
penicilina G benzatínica debe incluir probenecid
____ 147. Joven de 16 años de edad es traído a la emergencia por presentar falta de apetito, dolor
abdominal, nausea y vomito desde hace 24 horas, sin causa aparente, llega
somnoliento. Al examen físico tenemos: FC 94/min, FR 21/min, TA 110/70 mmHg, T
37.4 °C. Mucosas secas, disminución de turgencia, respiración rápida y superficial con
un llamativo olor a frutas. Abdomen es blando, depresible, no visceromegallas, ruidos
hidroaereos presentes. Los exámenes de laboratorio arrojan: Hb 14 g/dl; Hct 39%;
Leucocitos 8000 mm3 (Segmentados: 64%, Linfocitos: 30%, Monocitos: 6%); Na 135
mEq/dl; K 6 mEq/dl; Cl 105 mEq/dl; Glucosa sérica 470 mg/dl; Cr 0,9 mg/dl; BUN 19
mg/dl. ¿Cuál sería el diagnóstico del paciente?
a. Estado hiperosmolar no cetósico c. Descompensación simple de
diabetes
b. Estado hiperosmolar cetósico d. Cetoacidosis diabética

____ 148. Lactante de 3 meses de edad, presenta dermatitis seborreica. Los siguientes
tratamientos son correctos, EXCEPTO:
a. Administrar prednisona 2 c. Retirar las costras con compresas
mg/kg/día, vía oral, por un mes tibias con aceite de oliva, seguida
de champú para bebé
b. Lavar con champú de ketoconazol d. Aplicar crema de hldrocortisona al
al 2% 1 a 2.5 %
____ 149. La inmunodeficiencia del paciente adulto con infección por VIH, que ya presenta
evidencia de SIDA, se explica por:
a. Linfopenla así como c. Alteración de sus barreras
desregulación de linfocitos inespecíficas
cooperadores
b. Pérdida de la memoria humoral d. Defecto en la quimiotaxis,
fagocitosis y muerte celular

23
Name: ________________________ ID: A

____ 150. Mientras usted realiza su rotación de Neumología, se le encarga el cuidado de un


paciente de 72 años que fue internado por presentar: disnea, dolor de tipo pleurítico,
fiebre y baja de peso. El examen físico realizado por el residente muestra una FR de
24/min, disminución de entrada de aire en el lóbulo pulmonar inferior izquierdo
acompañado de un sonido mate a la percusión. Se solicita una placa AP y lateral de
tórax que muestra un derrame pleural, se realiza una toracocentesis obteniéndose un
fluido pleural de tipo hemorrágico. El residente antes de terminar su turno ordena los
siguientes análisis; LDH sérico 110 U/L (N: 45-90), proteínas totales 7 g/dl (6.0-7.8
g/dl), LDH fluido pleural 230 U/L, proteínas en fluido pleural 14 g/dl. Elija el diagnóstico
más probable de este caso:
a. Derrame pleural de tipo trasudado c. Derrame pleural de tipo exudado
por falla cardiaca por malignidad
b. Derrame pleural de tipo trasudado d. Derrame pleural de tipo exudado
por embolia pulmonar por neumonía
____ 151. El día sábado, en urgencias de un hospital rural de Esmeraldas y en temporada invernal,
llega un paciente con síndrome febril y sopor. El laboratorio es incapaz de hacer con
garantías una prueba para diagnóstico de paludismo. La actitud a seguir más lógica
sería:
a. Hacer una gota gruesa para c. Administrar una cefalosporina de
enviar a centro especializado y 3- generación y esperar la
tratar como si fuera una infección evolución
por P, Falclparum
b. Dar un tratamiento sintomático y d. Usar una fluoroquinolona y
esperar el lunes a que lo revise un esperar la evolución
especialista
____ 152. ¿Cuál de los siguientes es el fármaco de elección para el tratamiento de la infección
aguda de Tripanozoma cruzi?
a. Benznidazole c. Alopurinol
b. Itraconazol d. Fluconazol
____ 153. Una mujer de 69 años de edad con alcoholismo crónico, última ingesta hace una
semana. El día de hoy, de manera súbita, presenta decaimiento y deposiciones negras,
pegajosas y fétidas. Al examen físico tiene ascitis, red venosa superficial visible en el
abdomen y su hígado no es palpable. En esta paciente, de entre las siguientes, ¿Cuál
es la localización más probable del sangrado?
a. Curvatura mayor del estómago c. Segunda porción del duodeno
b. Tercio distal del esófago d. En el fundus gástrico

____ 154. Un paciente de 15 años tiene diagnóstica de neumonía adquirida en la comunidad por
Mycoplasma pneumonías. ¿Cuál de los siguientes antibióticos es el adecuado para
tratarlo en forma ambulatoria?
a. Sultamicilina c. Cefpodoxima
b. Amoxicilina d. Claritromicina

24
Name: ________________________ ID: A

____ 155. Un varón de 45 años de edad, sin antecedentes patológicos significativos, acude a
emergencias con disnea de 4 días de evolución, que se presenta en reposo y se
intensifica en decúbito dorsal, llegando a producir ortopnea, disnea parodística
nocturna y trepopnea. Se acompaña de astenia y adinamla posterior a cualquier tipo de
actividad física, lo cual impide su ejecución. En los últimos dos meses ha perdido 20 kg
de peso. Presenta temblor fino distal en miembros superiores, pérdida de cabello e
hiperhidrosis. Por el cuadro clínico y su diagnóstico presuntivo, ¿Cuáles son los
resultados de las pruebas de función tiroidea de este paciente?
a. TSH disminuido y T4 libre elevado c. TSH normal y T4 libre disminuido
b. TSH disminuido y T4 libre normal d. TSH elevado y T4 libre elevado

____ 156. Joven migrante de 23 años con historia de promiscuidad bisexual y drogadicción,
consulta por fiebre, malestar general y dolor en glande. La exploración física muestra
múltiples tatuajes, "piercings" y la presencia de tres lesiones vesiculosas en glande y
adenopatías Inguinales bilaterales. ¿Cuál es el diagnóstico más probable?
a. Gonorrea c. Condilomas acuminados
b. Sífilis d. Herpes genital

____ 157. Un peatón sufrió un trauma craneal al ser embestido por un automotor y cayó al piso sin
perder la conciencia. Descansó una hora y luego se levantó confuso e irritable. Luego
se tambaleó y cayó nuevamente al piso. Estaba hipersomne y presentó contracciones
de la mitad izquierda Inferior de la cara y del brazo izquierdo. Además, presenta una
fractura a nivel temporo-parietal derecho. Indique su diagnóstico clínico:
a. Infarto cerebral secundario al c. Hematoma subdural por ruptura
traumatismo de la arteria meníngea media
izquierda
b. Hematoma extradural por sección d. Lesión por contragolpe de la
de la arteria meníngea media arteria cerebral anterior izquierda
derecha
____ 158. Una niña de 7 años de edad presenta desde hace 10 días dolor abdominal, realiza
deposiciones explosivas, luego de comidas, líquidas, abundantes, con restos
alimentarlos, amarillentas, con olor fétido intenso, espumosas, grasosas y sin presencia
de moco, sangre ni material purulento. Se repiten hasta 4 o 5 veces al día. Tiene
además flatulencia intensa. Sus signos vitales son normales, sus mucosas están
semi-húmedas y sus conjuntivas son rosadas. A la palpación presenta un dolor
abdominal leve, difuso, algo más significativo en epigastrio y zona duodenal. A la
auscultación se escuchan ruidos hidroaéreos aumentados, en gran cantidad. ¿Cuál de
los siguientes parásitos genera el cuadro clínico descrito?
a. Ancylostoma duodenale c. Giardia lamblia
b. Balantidium coli d. Entamoeba histolytica

25
Name: ________________________ ID: A

____ 159. Un paciente de 28 años de edad sin antecedentes patológicos, presenta súbitamente
un dolor punzante en el hemitórax derecho, que se irradia a la axila del mismo lado. Se
acompaña de una sensación de falta de aire y latidos cardiacos rápidos. En
emergencia del hospital se realiza un examen físico completo y gasometría arterial.
¿Qué examen de imagen se debe solicitar bajo la sospecha diagnóstica de neumotórax
espontáneo?
a. El diagnóstico no amerita estudio c. Una Resonancia Magnética
imagenológico para continuar con Nuclear para ubicar la bula
el tratamiento pulmonar
b. Una Rx AP y lateral de tórax en d. Una TAC simple sin contraste
máxima inspiración
____ 160. Mujer de 43 años tiene desde hace 2 días dolores de labios, encías, lado derecho de la
cara y mentón, por varias ocasiones durante el día y la noche con pocos segundos de
duración; ha notado que se produce al cepillarse los dientes y cuando se expone a
corrientes de aire. Al examen físico presenta signos vitales estables, no hay asimetría
facial, no hay signos de focalización ni signos meníngeos. Según el cuadro clínico, elija
el diagnóstico adecuado para este caso:
a. Parálisis de Bell c. Neuralgia del V par craneal
b. Parálisis facial periférica d. Neuritis facial

____ 161. Elija ¿Cuál de las siguientes medidas farmacológicas, sería la adecuada en el siguiente
caso? Paciente varón de 38 años de edad acude a un control médico de rutina. Se
encuentran niveles de glicemia en ayunas de 100 mg/dl; glucosa a las 2 horas de 160
mg/dl: hemoglobina A1C 6.6%. El índice de masa corporal del paciente es 35 I(g/m2.
Reporta que su padre es diabético.
a. Pioglitazona c. Clorpropamlda
b. Glipizida d. Metformina

____ 162. Un paciente presenta los siguientes datos: TSH, alto; 74 total y libre, bajos. Las
siguientes son manifestaciones clínicas que se corresponden con estos datos de
laboratorio, EXCEPTO:
1. Espasticidad.
2. Exoftalmus.
3. Diarrea.
4. Piel áspera y seca.
5. Intolerancia al frío.
6. Mlxedema pretlbial.

a. 1, 2, 5 c. 2, 3, 6
b. 1, 4, 5 d. 3, 4, 6

26
Name: ________________________ ID: A

____ 163. Niña de 1 año 6 meses que durante su control de salud presenta la siguiente curva de
crecimiento:

¿Cómo interpreta la siguiente curva de crecimiento?


a. La tendencia de crecimiento es c. La tendencia de crecimiento es
normal ya que el peso y la talla plana, colocando al niño en riesgo
transcurren por el mismo percentil de sufrir malnutrición
b. El peso y la talla están normales d. La tendencia de crecimiento es
pero la tendencia de crecimiento normal ya que aumenta el
es plana percentil con relación a la edad
____ 164. Un adolescente de 15 años de edad, sufrió en el glúteo un traumatismo con un objeto
cortante, contaminado con barro. Presenta edema, eritema y dolor moderado, con una
valoración de EVA 7/10. Se observan lesiones por abrasión en el sitio de la lesión; el
eritema es de 10 cm en su diámetro mayor y tiene márgenes poco diferenciados y febril
(38°C). No hay signos de compromiso sistémico, manifestaciones de necrosis o de
contenido purulento. Indique, ¿Cuál es el diagnóstico clínico?
a. Miositis c. Absceso subcutáneo
b. Celulitis bacteriana d. Fascitis necrotlzante

____ 165. Un niño de 8 años de edad presenta desde hace 5 días dolor abdominal tipo cólico, con
aumento progresivo de intensidad. Además, tiene frecuencia aumentada de sus
deposiciones en 5 - 6 al día, de cantidad variable, tienen consistencia líquida, con
presencia de moco, sangre y sin material purulento o de otro tipo. Últimamente
presenta pujo al deponer. Su estado de conciencia y sus signos vitales son normales;
sus mucosas están semihúmedas, sus conjuntivas son algo pálidas. Presenta dolor
abdominal moderado a la palpación, especialmente en el marco colónico. ¿Cuál de los
siguientes parásitos produce el cuadro clínico descrito?
a. Giardia lamblia c. Ascaris lumbricoides
b. Entamoeba histolytica d. Taenia solium

27
Name: ________________________ ID: A

____ 166. Seleccione las dosis de los siguientes fármacos utilizados en la terapia triple contra el
Helicobacter Pylori:
Fármaco Dosis
1. Omeprazol a) 1 g cada 12 horas
2. Amoxlcillna b) 500 mg cada 6 horas
3. Claritromicina c) 500 mg cada 12 horas
d) 20 mgeada 12 horas

a. 1c, 2a, 3b c. 1d, 2c, 3a


b. 1a, 2b, 3c d. 1d, 2a, 3c
____ 167. El siguiente enunciado corresponde al mecanismo de acción de una sustancia: Cubre las
fimbrias de Escherichía coli y evita que esta bacteria se una a los receptores
glucosídicos de las células del urotelio, con lo cual disminuye la colonización de las vías
urinarias por este microorganismo. Seleccione el agente profiláctico para infección de
vías urinarias que realiza la función descrita en el enunciado:
a. Jugo de arándano c. Nitrofurantoina
b. Ácido ascórbico d. Extractos liofilizados de E. coli

____ 168. Indique la curva de la Organización Mundial de la Salud que se utiliza para valorar la
desnutrición global:
a. IMC/edad c. Talla/edad
b. Peso/edad d. Peso/talla
____ 169. En un hemograma, ¿Cuál de los siguientes parámetros tiene mayor utilidad para
diagnosticar anemia ferropénica?
a. Hemoglobina corpuscular media c. Hematocrito de 45 %.
de 36 pg
b. Volumen corpuscular medio d. Ancho de distribución eritrocitaria
inferior a 80 fl mayor a 14.5 %
____ 170. Un varón de 28 años de edad, nacido y residente en Quito. Desde hace 2 meses se
traslada a vivir a Manabí como trabajador en una granja productora de leche. No
presenta antecedentes patológicos significativos. Refiere que desde hace 2 días tiene
alza térmica, acompañada de malestar general, cefalea pulsátil, sudoración nocturna,
anorexia y dolores articulares. Al momento tiene fiebre de 38°C, astenia, deshidratación
leve y pequeños granulomas de tejido blando en la región inguinal, bilateralmente. El
signo del torniquete es negativo. ¿Cuál es el diagnóstico clínico de este paciente?
a. Brucelosis c. Dengue clásico
b. Fiebre amarilla d. Dengue hemorrágico

28
Name: ________________________ ID: A

____ 171. Un paciente de 22 años, presentó faringoamigdalitis estreptocócica hace 3 semanas.


Desde hace 10 días tiene fiebre alta, decaimiento, malestar general, irritabilidad,
movimientos involuntarios, alteraciones del carácter, odinofagia y disfagia. Refiere
también dolor e hinchazón en diversas articulaciones. Actualmente presenta congestión
amigdalina, con exudado purulento y petequias en paladar; adenopatías submaxilares y
latero- cervicales marcadas y dolorosas a la palpación; fiebre alta y en ciertas partes,
exantema escarlatiniforme. Presenta leucocitosis y niveles de PCR 4 veces por encima
de lo normal. ¿Cuál de las siguientes alternativas permite establecer el diagnóstico de
fiebre reumática?
a. Leucocitosis, irritabilidad y c. Leucocitosis, aumento de PCR y
movimientos involuntarios fiebre
b. Poliartritis, fiebre y leucocitosis d. Poliartritis, es suficiente para
establecer el diagnóstico
____ 172. Todos los siguientes son síntomas de la enfermedad celiaca, EXCEPTO:
a. Esteatorrea c. Ictericia
b. Pérdida de peso d. Anemia

____ 173. Niño de un año de edad, con 9 kg de peso, presenta signos de deshidratación grave,
está adormecido y no puede tomar líquidos. En el puesto de salud rural en que se
encuentran, no disponen de un equipo para Infusión intravenosa. Se cuenta con una
sonda nasogástrica (SNG). Indique, ¿Qué cantidad de suero oral, por kg de peso y por
hora, se debe administrar a este niño por la SNG?
a. 20 ml/kg/h = 180 ml/h c. 30 ml/kg/h = 270 ml/h
b. 50 ml/kg/h =450 ml/h d. 10 ml/kg/h =-90 ml/h

____ 174. ¿Cuál de los siguientes pacientes con cefalea necesita ser referido con urgencia a un
centro de mayor complejidad?
a. Cefalea de reciente aparición en c. Cefalea tensional leve con 3 a 4
trueno que cambia con los episodios mensuales que ceden
movimientos posturales con medicación
b. Cefalea con antecedentes de d. Cefalea localizada en región
migraña, con signos de aura retro-orbicular de inicio reciente y
inicial con lagrimeo
____ 175. Un paciente con trauma torácico, presenta dificultad respiratoria, cianosis, sus venas del
cuello están distendidas, presenta desviación de la tráquea hacia el lado izquierdo, con
asimetría del tórax y ruidos respiratorios hipofonétícos en el lado derecho. Además, se
encuentra taquipnéico y con hipotensión sostenida. Por el cuadro clínico sabemos que
se trata de neumotórax a tensión. Elija el procedimiento adecuado:
a. Toracocentesis en el segundo c. Toracocentesis en el octavo
espacio intercostal, línea medio espacio intercostal, línea axilar
clavicular derecha posterior derecha
b. Toracocentesis en el octavo d. Toracocentesis en el segundo
espacio intercostal, línea axilar espacio intercostal, línea medio
posterior izquierda clavicular izquierda

29
Name: ________________________ ID: A

____ 176. Son considerados como factores de riesgo predisponentes del cáncer gástrico,
EXCEPTO:
a.Helicobacter Pylori como c. Consumo prolongado de
productor de gastritis crónica inhibidores de la bomba de
protones
b. Consumo prolongado de d. Mutación autosómica dominante
alimentos ahumados y salados en el gen de la E-caderina
____ 177. Indique el medicamento adecuado para el tratamiento Inicial, hasta disponer del
resultado del urocultivo, en infecciones de vías urinarias no complicadas, considerando
su eficacia y seguridad:
a. Amoxicilina c. Ácido nalidíxico
b. Nitrofurantoína d. Trimetropin/sulfametoxazol

____ 178. Un paciente masculino, de 28 años de edad, fumador. Desde hace 3 semanas presenta
tos seca, rinorrea cristalina, odinofagia. No registra fiebre. No tiene disnea ni
sibilancias. Se establece el diagnóstico de bronquitis aguda y se decide iniciar manejo
inicial primario. Seleccione el enunciado correcto:
a. El tratamiento con antibióticos es c. Se debe recomendar dejar de
ideal para todos estos pacientes fumar, hidratación adecuada y
lavado de manos
b. El tratamiento con agonistas 13-2 d. Los agentes mucolíticos ayudan a
está recomendado en pacientes mejorar la tos
no complicados
____ 179. Paciente de 24 años de edad, sin antecedentes de importancia que sufre un traumatismo
de miembros inferiores con fractura de fémur, tibia y peroné del mismo lado. Al
evaluarlo en la emergencia, usted debe decidir si presenta signos de choque "shock".
En este contexto, de entre los siguientes signos clínicos del estado de choque ¿Cuál es
el que con mayor frecuencia ocurre más temprano?
a. Confusión c. Hipotensión
b. Taquicardia d. Bradipnea

____ 180. Un paciente varón de 33 años de edad, al conducir una motocicleta a 30 km/h, se
impacta con un vehículo que estaba estacionado. Es llevado al servido de urgencias en
un hospital de segundo nivel. Tiene dolor torácico anterior, por el traumatismo.
Presenta tensión arterial de 87/50 mm de Hg; frecuencia cardíaca de 110 x 1 min;
frecuencia respiratoria de 30 x 1 min; saturación de oxígeno de 85 %. ¿Qué examen
específico se debe solicitar para evaluar su trauma de tórax?
a. Tomografía simple de tórax c. Ultrasonografía
b. Rayos X póstero-anterior de tórax d. Resonancia magnética

30
Name: ________________________ ID: A

____ 181. Paciente de 25 años, con ingesta alcohólica de 4 días presenta náusea y vómito
frecuente, luego de 4 horas de evolución el vómito es con sangre por varias ocasiones.
Al examen físico: PC 100 latidlos por minuto, FR 18 por minuto, TA 90/60, palidez
generalizada. Según el probable diagnóstico, decida la medida más acertada en el
manejo:
a. Se trata de hemorragia digestiva c. Se trata de hemorragia digestiva
alta evidente por lo que no se alta por cirrosis hepática
debe realizar endoscopla para alcohólica y necesita endoscopía
confirmar el diagnóstico para confirmar el diagnóstico
b. Se trata del Síndrome de d. Se trata del Síndrome de
Boerhaave y necesita endoscopía Mallory-Weiss y necesita
para confirmar el diagnóstico endoscopía para confirmar el
diagnóstico
____ 182. Paciente mujer de 55 años nota cansancio, y debilidad desde hace. algunas semanas.
Acude a consulta médica. El facultativo palpa un pequeño bocio que la paciente no ha
notado. En la anamnesis dirigida, el médico se percata que la paciente ha aumentado
de peso y presenta estreñimiento. Decide realizar una biopsia de tiroides, que reporta
infiltración linfocítica marcada. Elija los resultados de laboratorio que tengan relación
con el caso clínico descrito:
1. TSH elevada
2. TSH baja
3. T4 libre elevada
4. T4 libre baja

a. 1, 3 c. 1, 4
b. 2, 3 d. 2, 4
____ 183. Una mujer de 58 años de edad fue diagnosticada de hipertensión arterial hace un mes y
ha recibido tratamiento con un antagonista de los canales del calcio. Acude a su control
médico, ¿Cuál de las siguientes alteraciones podría encontrar de manera frecuente, en
pacientes tratados con este tipo de fármacos?
a. Bronco espasmo c. Edema pretibial
b. Tos seca nocturna d. Hiperkalemia

____ 184. La tinción de elección para Mycobacterium es


a. Cristal violeta c. Giemsa
b. Ziehl-Nelssen d. Gram

____ 185. Los siguientes componentes del síndrome metabólico son correctos, EXCEPTO:
a. Presión arterial mayor de 130/85 c. Glucosa en ayunas mayor de 100
mm Hg mg/dl
b. Triglicéridos mayor o Igual a 150 d. HDL mayor de 40 mg/dl en
mg/dl varones y 50 mg/dl en mujeres
____ 186. De los siguientes fármacos antituberculosos. ¿Cuál pertenece a los aminoglucósidos?
a. Isoniacida c. Pirazinamida
b. Estreptomicina d. Rifampicina

31
Name: ________________________ ID: A

____ 187. El tratamiento de cetoacidosis diabética es:


a. Administración de bicarbonato c. Venoclisis con solución salina
b. Tratar la alcalosis metabólica d. Colocar insulina de acción
prolongada
____ 188. Seleccione la causa pre-renal de insuficiencia renal aguda:
a. Daño glomerular causado por c. Urolitiasis crónica
tumor
b. Deshidratación aguda d. Hipertrofia prostética benigna
____ 189. La presente imagen sobre lesiones de piel se relaciona con el siguiente diagnóstico:

Fuente: (Ingratta, 2017, p. 117)

a. Celuiitis c. Erisipela
b. Dermatitis por contacto d. Angioedema
____ 190. En el diagnóstico de diabetes se considera ¡os siguientes enunciados. EXCEPTO:
a. Glucosa en ayunas mayor a 126 c. Glucosa postprandial de 200
mg/100 ml mg/100 ml
b. Poliuria, polidipsia y pérdida de d. Hbalc menor a 3%
peso
____ 191. Paciente con Diabetes Mellitus Tipo 2 que inicia tratamiento farmacológico con
metformina y que durante los primeros días de tratamiento presenta diarrea, nausea y
ha vomitado por dos ocasiones. Estas manifestaciones clínicas se deben a:
a. Descompensación aguda c. Hipoglucemia por el tratamiento
b. Hipersensibilidad a la metformina d. Enteropatía intestinal

____ 192. El principal síntoma o signo predictor de prescripción antibiótica en Bronquitis aguda es:
a. Disnea c. Esputo blanquecino
b. Fiebre d. Tos

____ 193. Indique que enfermedad se relaciona con la hipertrigliceridemia Grave


a. Diabetes Mellitus 2 c. Hepatitis Aguda
b. Hipertiroidismo d. Glomerulonefritis Aguda

32
Name: ________________________ ID: A

____ 194. Los siguientes parámetros, según las tablas de Framingham, se utilizan para calcular el
riesgo cardiovascular. EXCEPTO:
a. Colesterol LDL c. Tabaquismo
b. Diabetes d. Presión arterial
____ 195. Los siguientes enunciados en relación a Herpes Zóstsr. EXCEPTO:
a. Si la función inmunitaria c. Durante la infección primaria, el
disminuye luego de la infección virus de varicela-zóster infecta los
primaria, el virus se reactiva en ganglios sensitivos y persiste en
los ganglios sensitivos, desciende éstos, en forma latente, durante el
por los nervios y se multiplica resto de la vida
b. Se caracteriza por disestesias d. La administración de la vacuna de
unilaterales, erupción vesicular en virus vivo atenuado contra el virus
la piel, en el trayecto del Herpes zóster (VZV), favorece el
dermatorna o los dermatomas desarrollo de Herpes zóster
¡nervados por el ganglio sensitivo durante la tercera edad
correspondiente
____ 196. Las estrategias terapéuticas más eficaces para el tratamiento del VIH tomadas en
consideración son las siguientes. EXCEPTO:
a. Supresión máxima de la c. Combinaciones de
replicación viral antirretrovirales eficaces con los
no presenten interacciones
b. Combinaciones de d. Combinaciones de
antirretrovirales eficaces con los antirretrovirales eficaces con los
que hayan sido tratados que no hayan sido tratados
previamente previamente
____ 197. Durante el Control prenatal específico en mujeres con DM uno de los exámenes a
solicitarse con su debida frecuencia es la ecografía. De la tabla expuesta a
continuación, relacione el período del embarazo con el objetiva para el cual se solicita
la ecografía.
1. Semanas 11 a 14.
2. Semanas 20 a 22.
3. Semanas 28 a 30.
4. Semana 36.
a. Realizar ecografía para valorar crecimiento fetal, perímetro abdominal fetal y
determinación de percentil para descartar macrosomía fetal incipiente.
b. Realizar ecografía para valorar crecimiento fetal y el volumen del líquido amniótico.
c. Realizar ecografía para determinar marcadores Geográficos de AC d. Realizar una
ecografía morfológica fetal (detalle anatómico)

a. 1b, 2c, 3d, 4a c. 1c, 2d, 3a, 4b


b. 1d, 2a, 3b, 4c d. 1a, 2b, 3c, 4d
____ 198. La retinopatía diabética se caracteriza por:
a. Microaneurismas vasculares c. Hemorragia del humor acuoso
b. Aumento de los pericitos d. Disminución de la vascularización

33
Name: ________________________ ID: A

____ 199. La tos crónica con imagen radiográfica normal, se debe a la administración de:
a. Amitriptilina c. Dextrometorfán
b. Captopril d. Hidrocodona

____ 200. La celulitis en diabéticos ancianos es ocasionada por.


a. Estafilococo c. Pasteurella multocida
b. Haemophilus influenzae d. Streptococcus agalactiae

____ 201. Los siguientes son síntomas asociados a hipoíiroidismo en edad escolar. EXCEPTO:
a. Disminución del volumen tiroideo c. Aumento del colesterol y
triglicéridos
b. Anemia por deficiencia de hierro d. Déficit sutil del coeficiente
intelectual
____ 202. ¿Cuál de los siguientes fármacos usados para el tratamiento de diabetes disminuye la
producción hepática de glucosa?
a. Nateglidina c. Sulfanilureas
b. Rosiglítazona d. Biguanidas
____ 203. La evolución natural de la neuropatía diabética se caracteriza por los siguientes eventos.
EXCEPTO:
a. Prostatitis c. Acidosis tubular
b. Hiperperfusión glomerular d. Albuminuria
____ 204. El tratamiento de infección extensa por tiña es:
a. Caspofungina c. Micafungina
b. Amfotericina d. Terbinafina

____ 205. Indique el enunciado perjudicial en pacientes con diabetes:


a. Consumo de edulcorantes sin c. Consumo de antioxidantes u
calorías oligoelementos
b. Dieta hipocalórica baja en d. Consumo mínimo de grasas trans
carbohidratos
____ 206. Relacione las causas de bocio endémico con sus características:
Causas
1. Alteraciones del metabolismo del yodo.
2. Bociógenos.
Características
a) Interfieren en la captación del yodo.
b) Aclaramiento renal de yodo aumentado.
c) Déficit en el aporte de yodo.
d) Interfieren en la producción hormonal.

a. 1bc, 2ad c. 1bd, 2ac


b. 1ab, 2cd d. 1ac, 2bd
____ 207. Un síntoma de Ha anemia por deficiencia de complejo B es:
a. Síndrome de mano-pie c. Abombamiento craneal
b. Priapismo d. Glositis

34
Name: ________________________ ID: A

____ 208. La deficiencia de folato en la anemia megaloblásfica causa:


a. Necrosis aséptica c. Síndrome torácico agudo
b. Abortos recidivantes d. Anemia microcítica

____ 209. La madre lleva a la consulta a su niño de 5 años de edad por cuanto presenta lesiones
de piel que aparentemente se exacerban con el consumo de alimentos. ¿Cuál
diagnóstico está relacionado con la dieta?
a. Dermatitis atópica c. Dermatitis de contacto
b. Dermatitis herpetiforme d. Acné vulgaris

____ 210. Paciente masculino de 25 años de edad, con antecedentes familiares de acné quístico
grave, presenta lesiones que iniciaron con comedones abiertos y cerrados, que se
transformaron en lesiones pápulo-pustulosas, con aumento de tamaño y difusión, lo
cual es característico del acné noduloquístico. Las lesiones son muy doiorosas y
presentan una distribución simétrica en la cara. ¿Cuál es la gravedad del acné y su
tratamiento?
a. Acné leve. Manejo con c. Acné grave. Manejar con
antibióticos vía oral, minociclina, tratamiento tópico y tratamiento
50 a 100 mg/día o doxiciclina, 50 sistémico con isotretinoina
a 100 mg cada 12h
b. Acné moderado. Manejo con d. Acné leve. Manejo con
retinoides tópicos (ácido retinoico, antibióticos tópicos (clindamicina
adapaleno, tazaroteno) y eritrornicina) y geles con
peróxido benzoico
____ 211. El tratamiento de la Esporotricosis Dinfocutánea es:
a. Fluocitosina c. Itraconazol
b. La Terbinafina d. Anfotericina B

____ 212. ¿En qué grupo poblacional se recomienda tratar siempre una bacteriana asintomática?
a. Pacientes pediátricos c. Tercera edad
b. En todos los grupos poblacionales d. Embarazadas
sin excepción
____ 213. Los exámenes indicados determinan el diagnóstico etiológico de la neumonía.
EXCEPTO:
a. Cultivo de esputo c. Proteína C reactiva
b. Hemocultivo d. Prueba urinaria con antígeno
____ 214. Niño de 9 años, acude a consulta con tos leve con expectoración mucosa, disnea
espiratoria, el cuadro se ha presentado por algunas ocasiones. Al examen físico se
encuentra algo cianótico, se auscultara sibilancias e hipertimpanismo. Una de las
sospechas diagnósticas es el asma, con la finalidad de confirmar o descartar el
diagnóstico usted solicita algunos estudios funcionales pulmonares. Los siguientes son
resultados de pruebas funcionales pulmonares normales. EXCEPTO:
a. FEV 1/FVC = 90% c. FVC menor a 68%
b. FEF 25-75% = 86% d. FEV 1.89%

35
Name: ________________________ ID: A

____ 215. Uno de los principales objetivos del manejo nutricional y el soporte de ejercicio
moderado en pacientes con DM gestacional es:
a. Controlar la hemoglobina c. Optimizar el control glicémico
glicosilada
b. Controlar la cetonuria d. Reducir la glucemia postprandial

____ 216. Paciente adulto con antecedente efe traumatismo hace dos semanas que presenta Hb de
16 g/dl, albúmina menor a 2.8 g /100 ml, linfocitopenia de 1500/pl, con deficiente
cicatrización de heridas, desprendimiento fácil del cabello y edema, estatura normal,
¿Cuál es el diagnóstico?
a. Kwashiorkor c. Cretinismo
b. Marasmo d. Anemia

____ 217. Niña de 1 año 6 meses que durante su control de salud presenta la siguiente curva ote
crecimiento:

¿Cómo se interpreta la siguiente curva del gráfico anterior?

a. La niña presenta una tendencia c. La niña no ha subido mucho de


de crecimiento normal baja para peso, pero a su edad este
la edad comportamiento es normal
b. La niña no está subiendo de peso d. La niña ha subido poco de peso y
y tiene una señal de peligro esto nos alerta de algún proceso
patológico
____ 218. Un paciente varón, de 55 años de edad, presenta los siguientes datos clínicos: presión
arterial, 150/90 mm Hg; perímetro abdominal, 90 cm; glucemia en ayunas, 150 mg/dl;
íriglicéridos, 179 mg/dl; colesterol HDL, 25 mg/dl. De acuerdo con los datos expuestos.
¿Cuál de las siguientes opciones corresponde al diagnóstico de síndrome metabólico?
a. Presenta 4 de los c. Presenta 4 de los
5 criterios diagnósticos, 5 criterios diagnósticos,
por tanto, tiene síndrome por tanto, no tiene síndrome
metabólico metabólico
b. Presenta 2 de los d. Presenta 3 de los
5 criterios diagnósticos, 5 criterios diagnósticos,
por tanto, tiene síndrome por tanto, no tiene síndrome
metabólico metabólico

36
Name: ________________________ ID: A

____ 219. El régimen antibiótico de erradicación de cepas resistentes de Helycobacíer pyloríes:


a. Amoxicilina levofloxacina + c. Rifabutina + Tlnidazol +
tlnidazol claritromicina
b. Furazolidina + claritromicina + d. Metronidazol + rifabutina +
tetraciclina tinidazol
____ 220. Indique en cuál de las siguientes patologías hipertensivas se utiliza feniolamina:
a. Encefalopatía hipertensiva c. Feocromocitoma
b. Hipertensión arterial maligna d. Disección aórtica

____ 221. Los siguientes hongos producen micosis superficiales en la cara. EXCEPTO:
a. Esporotricosis c. Dermatofitos
b. Especies de Malassezia d. Especies de Cándida

____ 222. Los siguientes enunciados sobre prevención o tratamiento de las complicaciones en
pacientes diabéticos, son correctos. EXCEPTO:
a. Aspirina para cadioprotección c. Aspirina está indicada para
está contraindicada en presencia cardioprotección en mayores de
de retinopatía diabética 50 años con riesgo cardiovascular
b. Duloxetina es uno de los d. Pregabalina es uno de los
tratamientos iniciales para el dolor tratamientos iniciales para el dolor
neuropático neuropático
____ 223. En los cuadros de artropatía psoriásica. ¿Cuál de los siguientes parámetros clínicos o de
laboratorio es más común entre Has mujeres en relación a los hombres?
a. Tener artritis de afectación solo c. Ser seropositivo para factor
distal reumatoide
b. Tener artritis deformante d. Tener artritis de afectación
simétrica
____ 224. A su consulta acude un paciente masculino de 67 años de edad por presentar pirosis
desde hace 4 semanas relacionado con la ingesta alimentaria. El paciente no presenta
historia personal o familiar de cáncer gastrointestinal. Niega consumo de cigarrillo y
consume alcohol ocasionalmente. Niega dificultad para deglutir, refiere sensación de
plenitud gástrica, melenas o pérdida de peso. El examen físico del paciente es normal,
excepto por leve dolor a nivel epigástrico a la palpación. ¿Cuál de los siguientes
procedimientos se recomienda en el paciente?
a. Investigación de sangre oculta en c. Estudios de imagen
heces
b. Pruebas funcionales hepáticas d. Endoscopia digestiva alta

____ 225. El siguiente estudio de laboratorio clínico es diagnóstico de desnutrición aguda:


a. Transferrina mayor a 150 mg/100 c. Linfocitos aumentados de 1500/ pl
ml
b. Capacidad de transporte de hierro d. Concentraciones séricas de
mayor a 200 mg/100 ml albúmina menores a 2.8 g/100 ml

37
Name: ________________________ ID: A

____ 226. Los siguientes son factores de riesgo, en mayor o menor grado, para desarrollar
enfermedad pulmonar obstructiva crónica (EPOC). EXCEPTO:
a. Ser fumador pasivo por largo c. Neumonía o bronquitis
tiempo
b. Exposición al polvo de algodón y d. Tabaquismo prolongado
carbón
____ 227. En una evaluación nutricional se reporta que Rosario de 20 años tiene una ingesta diaria
de 50 gramos de proteína, 100 gramos de grasa y 400 gramos de hidratos de carbono.
¿Cuál sería ¡a ingesta calórica total?
a. 4200 calorías/día c. 2450 calorías/día
b. 2700 calorías/día d. 2200 calorías/día

____ 228. En la mujer con diabetes gestacional es importante la identificación de tos factores de
riesgo. Los siguientes son factores de riesgo alto. EXCEPTO:
a. Síndrome de ovario poliquísíico c. Partos con productos
(SOP) macrosómicos de más de 4 kilos
b. Sobrepeso (IMC mayor a 25 d. Trastorno del metabolismo de los
kg/m2) antes del embarazo carbohidratos
____ 229. ¿En qué etapa de la sífilis se observa la perforación del paladar blando?
a. Sífilis primaria c. Sífilis terciaria
b. Sífilis prenatal (congénita) d. Sífilis secundaria

____ 230. La hipertensión portal es una de las primeras manifestaciones de una patología cirrótica,
pero también otras patologías prehepáticas e intrahepáticas pueden causarla. En el
tratamiento profiláctico para evitar hemorragia variceal indica el uso de los
betabloqueadores no selectivos (BBNS). Señale cual es un efecto secundario al uso de
estos medicamentos:
a. Bloqueo auriculo ventricular de II c. Insuficiencia cardiaca
grado
b. Impotencia d. Enfermedad pulmonar obstructiva
grave
____ 231. El medicamento que disminuye la hemorragia en el sangrado digestivo alto del paciente
cirrótico es:
a. Ceftriaxona c. Octeotride
b. Eritromicina d. Prostaciclina
____ 232. En un paciente con cetoacidosis diabética, a más de la corrección de la hiperglucemia,
se deberán realizar las siguientes acciones. EXCEPTO:
a. Medición y reposición de potasio c. Medición y reposición de
bicarbonato
b. Medición y reposición de fosfato d. Medición y reposición de calcio

____ 233. Las siguientes sintomatologías se asocian a la encefalopatía metabólica, EXCEPTO:


a. Trastornos psíquicos c. Hipertensión intracraneal
b. Trastornos de la conciencia d. Convulsiones

38
Name: ________________________ ID: A

____ 234. Paciente de 40 años refiere fiebre, sudoración nocturna, pérdida de peso y disnea de
un tiempo evolución de 2 meses. A la exploración física TA: 120/80, FC:80 Ipm, Sat 02:
96%, T°: 38 °C, dolor torácico en hemitórax anterior derecho, murmullo vesicular
disminuido en base derecha. En la radiografía de tórax se observa radiopacidad
homogénea unilateral derecha, borramiento del ángulo costofrénico derecho de borde
cóncavo sin presencia de condensaciones. ¿Cuál es el diagnóstico?
a. Absceso pulmonar c. Asma persistente
b. Neumonía bacteriana d. Tuberculosis pleural

____ 235. Las siguientes son enfermedades que se asocian con disfagia y se alivian con
tratamientos específicos, EXCEPTO:
a. Los cánceres de cabeza y cuello c. Neuralgia del nervio glosofaríngeo
avanzados
b. Globus hystericus d. El Síndrome de Plummer-Vinson o
de Paterson-Kelly
____ 236. ¿Cuál de las siguientes causas de hipoglucemia se relaciona en el paciente no enfermo?
a. Medicamentos c. Tumores no pancreáticos
b. Deficiencia hormonal d. Hiperinsulinismo endógeno

____ 237. Indique qué medicamento está contraindicado para el manejo del dolor en el sangrado
digestivo:
a. Tramadol c. Morfina
b. Ketorolaco d. Fentanilo
____ 238. ¿Cuál de las siguientes es una característica de la ataxia cerebelosa?
a. Déficit propioceptivo c. Signo de Romberg positivo
b. Rigidez d. Dismetría

____ 239. Paciente asintomático, que al examen físico presenta un aumento del tiroides grado II,
sin nodulos palpables. Los exámenes reportan: TSH y T3 normales, T4 total baja,
colesterol alto, anticuerpos contra TPO negativos, la gammagrafía tiroidea pone de
manifiesto un aumento en la captación. Señale el diagnóstico:
a. Bocio multinodular no tóxico c. Bocio difuso no tóxico
b. Bocio multinodular tóxico d. Tiroiditis de Hashimoto

____ 240. Señale el patógeno que causa neumonía extrahospitalaria (adquirida en la comunidad)
en pacientes con EPOC y tabaquismo:
a. Chlamydia psittaci c. Haemophilus influenzas
b. Stafilococo pneumonías d. Coxiella burnetii
____ 241. Todos son cambios en el estilo de vida de un paciente con Diabetes Mellitus tipo 2.
EXCEPTO:
a. Reducción de peso entre el 5 y 10 c. Mantener una dieta fraccionada
% en pacientes con DM2 con con tres ingestas diarias
sobrepeso u obesidad
b. Fraccionar el total de la d. Se sugiere una disminución del 7
alimentación habitual del día en 5 % de grasas en la dieta
o 6 porciones

39
Name: ________________________ ID: A

____ 242. Señale el resultado de laboratorio que se encuentra con relación a la desnutrición
aguda:
a. Albúmina sérica 2.5 g /100 mL c. Nitrógeno de úrea 28 mg /100 mL
b. Capacidad total transporte de d. Creatinina sérica 1.8 mg/100 mL
hierro sérico 480 ug/100 mL
____ 243. Señale el objetivo terapéutico para control de hiperlipidemia que debe tener un paciente
adulto con diabetes:
a. Triglicéridos < 150mg/dl c. Triglicéridos < 100mg/dL (2.6
(1.7mmol/L) mmol/L)
b. Lípidos HDL < 180mg/dL d. Lípidos LDL < 100mg/dL (5.6
(10mmol/L) mmol/L)
____ 244. La dermatitis seborreica se caracteriza por:
a. Aparece por antipalúdicos c. Escamas grasientas sobre zonas
eritematosas
b. Aparece en el ombligo d. Aparece en codos, rodillas y
planta de los pies
____ 245. ¿Cuál es la alteración del equilibrio ácido-base presente en los siguientes gases
arteriales: PaO2: 84 mmltg; PaCO2: 22 mmHg; HCO3: 18 mmol/l; pH: 7.48?
a. Acidosis Metabólica c. Alcalosis metabólica
b. Alcalosis Respiratoria d. Acidosis respiratoria

____ 246. Señale el signo que se presenta en el cáncer pulmonar:


a. Hipocratismo digital c. Resoplador rosado
b. Tórax en tonel d. Signo de Hoover

____ 247. ¿Qué medicamento de uso tópico (de primera elección) se emplea en la terapia de
mantenimiento del acné leve a moderado?
a. Eritromicina en gel 2 % c. Peróxido de benzoilo en gel 5 %
b. Adapaleno en gel 0.1 % d. Ácido azelaico en gel 20 %

____ 248. ¿Qué se observa en la obesidad central?


a. Resistencia a la insulina c. Colesterol HDL, elevado
b. Colesterol LDL disminuido d. Triglicéridos disminuidos
____ 249. Paciente de 38 años, sin antecedentes de interés, refiere que desde hace 3 días
presenta sensación distérmica, tos con expectoración blanquecina y malestar general.
El paciente no ha utilizado antibióticos durante el último año por ninguna causa. Al
examen físico se encuentra alerta, febril (38.9°C), frecuencia cardíaca de 90 Ipm,
frecuencia respiratoria de 25 por minuto, tensión arterial 120/80 mmHg, no cianosis ni
signos de dificultad respiratoria. En la base pulmonar derecha se ausculta estertores
básales. La radiografía de tórax muestra una opacidad no segmentaria en el lóbulo
inferior del pulmón derecho con presencia de broncograma aéreo. ¿Cuál es el
tratamiento antibiótico empírico inicial para este paciente?
a. Moxifloxacino 400 mg por vía oral c. Levofloxacino 750 mg por vía oral
una vez al día durante 5 días una vez al día durante 5 días
b. Amoxicilina 1 gr más d. Doxiciclina 100 mg cada 12 horas
claritromicina 500 mg por vía oral por vía oral durante 5 días
cada 12 horas durante 10 días

40
Name: ________________________ ID: A

____ 250. ¿Cuál de las siguientes situaciones determina un resultado falso positivo en las
pruebas de tamizaje para VIH?
a. Trasplante de médula ósea c. Interferencia de factores
reumatoideos
b. Vacunación contra la gripe d. Disfunciones de las células B

41
ID: A

Medicina Interna
Answer Section

MULTIPLE CHOICE

1. ANS: A PTS: 1
2. ANS: C PTS: 1
3. ANS: B PTS: 1
4. ANS: B PTS: 1
5. ANS: D PTS: 1
6. ANS: B PTS: 1
7. ANS: D PTS: 1
8. ANS: A PTS: 1
9. ANS: D PTS: 1
10. ANS: C PTS: 1
11. ANS: B PTS: 1
12. ANS: D PTS: 1
13. ANS: B PTS: 1
14. ANS: B PTS: 1
15. ANS: A PTS: 1
16. ANS: D PTS: 1
17. ANS: C PTS: 1
18. ANS: C PTS: 1
19. ANS: B PTS: 1
20. ANS: B PTS: 1
21. ANS: A PTS: 1
22. ANS: A PTS: 1
23. ANS: A PTS: 1
24. ANS: A PTS: 1
25. ANS: B PTS: 1
26. ANS: B PTS: 1
27. ANS: B PTS: 1
28. ANS: A PTS: 1
29. ANS: D PTS: 1
30. ANS: A PTS: 1
31. ANS: A PTS: 1
32. ANS: C PTS: 1
33. ANS: A PTS: 1
34. ANS: C PTS: 1
35. ANS: D PTS: 1
36. ANS: B PTS: 1
37. ANS: B PTS: 1
38. ANS: D PTS: 1
39. ANS: C PTS: 1
40. ANS: A PTS: 1
41. ANS: D PTS: 1
42. ANS: A PTS: 1
43. ANS: C PTS: 1

1
ID: A

44. ANS: C PTS: 1


45. ANS: C PTS: 1
46. ANS: D PTS: 1
47. ANS: D PTS: 1
48. ANS: D PTS: 1
49. ANS: B PTS: 1
50. ANS: B PTS: 1
51. ANS: A PTS: 1
52. ANS: A PTS: 1
53. ANS: B PTS: 1
54. ANS: A PTS: 1
55. ANS: A PTS: 1
56. ANS: A PTS: 1
57. ANS: A PTS: 1
58. ANS: D PTS: 1
59. ANS: B PTS: 1
60. ANS: B PTS: 1
61. ANS: A PTS: 1
62. ANS: A PTS: 1
63. ANS: A PTS: 1
64. ANS: B PTS: 1
65. ANS: B PTS: 1
66. ANS: B PTS: 1
67. ANS: D PTS: 1
68. ANS: C PTS: 1
69. ANS: A PTS: 1
70. ANS: D PTS: 1
71. ANS: C PTS: 1
72. ANS: C PTS: 1
73. ANS: A PTS: 1
74. ANS: B PTS: 1
75. ANS: C PTS: 1
76. ANS: A PTS: 1
77. ANS: B PTS: 1
78. ANS: A PTS: 1
79. ANS: B PTS: 1
80. ANS: C PTS: 1
81. ANS: A PTS: 1
82. ANS: D PTS: 1
83. ANS: A PTS: 1
84. ANS: D PTS: 1
85. ANS: A PTS: 1
86. ANS: A PTS: 1
87. ANS: B PTS: 1
88. ANS: B PTS: 1
89. ANS: B PTS: 1
90. ANS: C PTS: 1
91. ANS: C PTS: 1

2
ID: A

92. ANS: D PTS: 1


93. ANS: A PTS: 1
94. ANS: D PTS: 1
95. ANS: A PTS: 1
96. ANS: C PTS: 1
97. ANS: B PTS: 1
98. ANS: C PTS: 1
99. ANS: C PTS: 1
100. ANS: D PTS: 1
101. ANS: A PTS: 1
102. ANS: C PTS: 1
103. ANS: A PTS: 1
104. ANS: C PTS: 1
105. ANS: C PTS: 1
106. ANS: B PTS: 1
107. ANS: B PTS: 1
108. ANS: D PTS: 1
109. ANS: B PTS: 1
110. ANS: C PTS: 1
111. ANS: A PTS: 1
112. ANS: C PTS: 1
113. ANS: A PTS: 1
114. ANS: B PTS: 1
115. ANS: A PTS: 1
116. ANS: B PTS: 1
117. ANS: C PTS: 1
118. ANS: C PTS: 1
119. ANS: C PTS: 1
120. ANS: B PTS: 1
121. ANS: B PTS: 1
122. ANS: B PTS: 1
123. ANS: C PTS: 1
124. ANS: B PTS: 1
125. ANS: A PTS: 1
126. ANS: D PTS: 1
127. ANS: A PTS: 1
128. ANS: B PTS: 1
129. ANS: C PTS: 1
130. ANS: A PTS: 1
131. ANS: A PTS: 1
132. ANS: C PTS: 1
133. ANS: A PTS: 1
134. ANS: D PTS: 1
135. ANS: A PTS: 1
136. ANS: B PTS: 1
137. ANS: A PTS: 1
138. ANS: A PTS: 1
139. ANS: B PTS: 1

3
ID: A

140. ANS: C PTS: 1


141. ANS: D PTS: 1
142. ANS: B PTS: 1
143. ANS: B PTS: 1
144. ANS: C PTS: 1
145. ANS: B PTS: 1
146. ANS: B PTS: 1
147. ANS: D PTS: 1
148. ANS: A PTS: 1
149. ANS: A PTS: 1
150. ANS: C PTS: 1
151. ANS: A PTS: 1
152. ANS: A PTS: 1
153. ANS: B PTS: 1
154. ANS: D PTS: 1
155. ANS: A PTS: 1
156. ANS: D PTS: 1
157. ANS: B PTS: 1
158. ANS: C PTS: 1
159. ANS: B PTS: 1
160. ANS: C PTS: 1
161. ANS: D PTS: 1
162. ANS: C PTS: 1
163. ANS: B PTS: 1
164. ANS: B PTS: 1
165. ANS: B PTS: 1
166. ANS: D PTS: 1
167. ANS: A PTS: 1
168. ANS: B PTS: 1
169. ANS: B PTS: 1
170. ANS: A PTS: 1
171. ANS: B PTS: 1
172. ANS: C PTS: 1
173. ANS: A PTS: 1
174. ANS: A PTS: 1
175. ANS: A PTS: 1
176. ANS: C PTS: 1
177. ANS: B PTS: 1
178. ANS: C PTS: 1
179. ANS: B PTS: 1
180. ANS: C PTS: 1
181. ANS: D PTS: 1
182. ANS: C PTS: 1
183. ANS: C PTS: 1
184. ANS: B PTS: 1
185. ANS: D PTS: 1
186. ANS: B PTS: 1
187. ANS: C PTS: 1

4
ID: A

188. ANS: B PTS: 1


189. ANS: A PTS: 1
190. ANS: D PTS: 1
191. ANS: B PTS: 1
192. ANS: B PTS: 1
193. ANS: A PTS: 1
194. ANS: A PTS: 1
195. ANS: D PTS: 1
196. ANS: B PTS: 1
197. ANS: C PTS: 1
198. ANS: A PTS: 1
199. ANS: B PTS: 1
200. ANS: D PTS: 1
201. ANS: A PTS: 1
202. ANS: D PTS: 1
203. ANS: A PTS: 1
204. ANS: D PTS: 1
205. ANS: C PTS: 1
206. ANS: A PTS: 1
207. ANS: D PTS: 1
208. ANS: B PTS: 1
209. ANS: B PTS: 1
210. ANS: C PTS: 1
211. ANS: C PTS: 1
212. ANS: D PTS: 1
213. ANS: C PTS: 1
214. ANS: C PTS: 1
215. ANS: C PTS: 1
216. ANS: A PTS: 1
217. ANS: B PTS: 1
218. ANS: A PTS: 1
219. ANS: A PTS: 1
220. ANS: C PTS: 1
221. ANS: A PTS: 1
222. ANS: A PTS: 1
223. ANS: C PTS: 1
224. ANS: D PTS: 1
225. ANS: D PTS: 1
226. ANS: C PTS: 1
227. ANS: B PTS: 1
228. ANS: B PTS: 1
229. ANS: C PTS: 1
230. ANS: B PTS: 1
231. ANS: C PTS: 1
232. ANS: C PTS: 1
233. ANS: C PTS: 1
234. ANS: D PTS: 1
235. ANS: B PTS: 1

5
ID: A

236. ANS: D PTS: 1


237. ANS: B PTS: 1
238. ANS: D PTS: 1
239. ANS: C PTS: 1
240. ANS: C PTS: 1
241. ANS: C PTS: 1
242. ANS: A PTS: 1
243. ANS: A PTS: 1
244. ANS: C PTS: 1
245. ANS: B PTS: 1
246. ANS: A PTS: 1
247. ANS: B PTS: 1
248. ANS: A PTS: 1
249. ANS: D PTS: 1
250. ANS: B PTS: 1

6
Medicina Interna [Answer Strip] ID: A

C 10.
_____ D 16.
_____ A 24.
_____ A 33.
_____

C 34.
_____

A
_____ 1. C 17.
_____
B 25.
_____
D 35.
_____
B 11.
_____
C
_____ 2.
B 36.
_____
B 26.
_____

C 18.
_____

B
_____ 3.

B 27.
_____
B 19.
_____
B 37.
_____
B
_____ 4.

D 12.
_____ B 20.
_____
A 28.
_____ D 38.
_____
D
_____ 5.

A 21.
_____
C 39.
_____
B
_____ 6.
B 13.
_____ D 29.
_____

A 22.
_____ A 40.
_____
D
_____ 7. A 30.
_____

B 14.
_____
A
_____ 8. D 41.
_____
A 31.
_____

A 15.
_____
D
_____ 9. A 42.
_____
A 23.
_____ C 32.
_____
Medicina Interna [Answer Strip] ID: A

C 43.
_____ A 52.
_____ B 60.
_____ D 70.
_____ C 80.
_____

C 44.
_____ B 53.
_____ A 61.
_____
C 71.
_____
A 81.
_____

A 62.
_____ D 82.
_____
C 45.
_____
C 72.
_____

A 83.
_____
A 54.
_____
A 63.
_____
D 46.
_____ A 73.
_____
D 84.
_____

B 64.
_____
D 47.
_____ A 55.
_____
B 74.
_____

A 85.
_____
A 56.
_____
B 65.
_____
D 48.
_____ C 75.
_____

A 57.
_____ A 86.
_____

B 66.
_____ A 76.
_____

B 49.
_____
D 58.
_____ B 87.
_____

B 77.
_____

D 67.
_____

B 59.
_____ A 78.
_____

B 50.
_____ C 68.
_____

A 51.
_____
A 69.
_____ B 79.
_____
Medicina Interna [Answer Strip] ID: A

B 88.
_____ C 96.
_____ D
_____100. C
_____104. B
_____109.

C
_____110.
B 89.
_____

C
_____105.
B 97.
_____
A
_____101.
C 90.
_____

C 98.
_____ A
_____111.

B
_____106.

C 91.
_____
C
_____102.

D 92.
_____
B
_____107. C
_____112.
C 99.
_____

A
_____103.
A 93.
_____

A
_____113.
D 94.
_____

D
_____108.
A 95.
_____

B
_____114.
Medicina Interna [Answer Strip] ID: A

A
_____115. B
_____120. B
_____122. B
_____124. C
_____129.

A
_____130.

A
_____125.

B
_____116. A
_____131.

B
_____121.

D
_____126.

C
_____117.

C
_____132.

C
_____118. A
_____127.

C
_____123.

C
_____119.
A
_____133.

B
_____128. D
_____134.
Medicina Interna [Answer Strip] ID: A

A
_____135. D
_____141. B
_____146. C
_____150. A
_____155.

B
_____142.
B
_____136.

D
_____147.

D
_____156.

A
_____151.

A
_____137. B
_____143.
B
_____157.

A
_____148.
A
_____138.

A
_____152.
C
_____144.

A
_____149. C
_____158.

B
_____153.
B
_____145.

B
_____139.

D
_____154.

C
_____140.
Medicina Interna [Answer Strip] ID: A

B
_____159. B
_____163. D
_____166. B
_____171. C
_____176.

B
_____177.

A
_____167.
C
_____160.

C
_____172. C
_____178.

A
_____173.
B
_____168.

D
_____161.

B
_____169.
B
_____179.
A
_____174.

C
_____162.

B
_____164. A
_____170.

C
_____180.

A
_____175.

B
_____165.
Medicina Interna [Answer Strip] ID: A

D
_____181. C
_____187. A
_____194. B
_____199. B
_____208.

D
_____200. B
_____209.
B
_____188. D
_____195.

A
_____201.

A
_____189. C
_____210.

D
_____202.
C
_____182.

B
_____196.
A
_____203.

D
_____204.
C
_____211.

C
_____205.
C
_____183. C
_____197.
D
_____212.

D
_____190.

A
_____206.
C
_____213.
B
_____184.

B
_____191.
C
_____214.
D
_____185.

B
_____192.

B
_____186. A
_____198.
D
_____207.
A
_____193.
Medicina Interna [Answer Strip] ID: A

C
_____215. A
_____219. C
_____226. D
_____234. A
_____242.

A
_____216. C
_____220. A
_____243.
B
_____227.

B
_____235.
A
_____221.

B
_____217. B
_____228. C
_____244.
A
_____222.

D
_____236.

B
_____245.
C
_____229. B
_____237.

C
_____223.
B
_____230. A
_____246.
D
_____238.

B
_____247.
C
_____239.
D
_____224.

A
_____248.
C
_____231.

C
_____240.
D
_____249.
A
_____218.
C
_____232.

D
_____225. C
_____241.

C
_____233.
Medicina Interna [Answer Strip] ID: A

B
_____250.
Name: ________________________ Class: ___________________ Date: __________ ID: A

Sepsis Neonatal

Multiple Choice
Identify the choice that best completes the statement or answers the question.

____ 1.
1. Marque el enunciado correcto

a. La sepsis de inicio temprano c. Solo A es correcta


se debe a microorganismos
adquiridos intraparto.

b. El comienso de la sespsis neonatal d. A y B son correctas


temprana puede ser menor o igual a 3
dias

____ 2.
1. Marque el enunciado correcto

a. El estreptococo grupo B c. Solo B es correcta


(EGB) y los microorganismos
entéricos gramnegativos
(predominantemente, Escheric
hia coli) son responsables de
la mayoría de los casos de
sepsis de inicio temprano.

b. El estreptococo grupo B d. A y B son correctas


(EGB) y los microorganismos
entéricos gramnegativos
(predominantemente, Escheric
hia coli) no son responsables
de la mayoría de los casos de
sepsis de inicio temprano.

1
Name: ________________________ ID: A

____ 3.
1. Marque el enunciado correcto

a. La transmisión de ciertos c. La transmisión de muchos


patógenos virales, protozoos y patógenos virales, protozoos y
treponemas puede producirse por treponemas puede producirse por
diseminación hemática y diseminación hemática y
transplacentaria de la infección transplacentaria de la infección
materna. materna.

b. La transmisión de ciertos d. Ninguna es verdadera


patógenos virales, protozoos y
treponemas no puede producirse
por diseminación hemática y
transplacentaria de la infección
materna.

2
Name: ________________________ ID: A

____ 4.
1. Marque el enunciado correcto

a. El factor de riesgo más c. Los focos iniciales de


importante de la sepsis infección pueden ser
de inicio tardío es el las vías urinarias, los
parto pretérmino. senos paranasales, el
oído medio, los
pulmones o el aparato
digestivo, desde donde
más tarde pueden
diseminarse a las
meninges, los riñones,
los huesos, las
articulaciones, el
peritoneo y la piel.

b. Los microorganismos d. Todas son verdaderas


grampositivos (p. ej.,
estafilococos
coagulasa-negativos
y Staphylococcus
aureus) pueden
provenir del ambiente
o de la piel del niño.

3
Name: ________________________ ID: A

____ 5.
1. Marque el enunciado correcto

a. La sepsis neonatal es una c. A y B son correctas


infección invasiva, en general
bacteriana, que se produce
durante el período neonatal.

b. Los signos son múltiples e d. Todas son verdaderas


incluyen disminución de la
actividad espontánea, succión
menos enérgica, apnea,
bradicardia, inestabilidad
térmica, dificultad respiratoria,
vómitos, diarrea, distensión
abdominal, inquietud,
convulsiones e ictericia.

4
ID: A

Sepsis Neonatal
Answer Section

MULTIPLE CHOICE

1. ANS: D
La sepsis de inicio temprano se debe a microorganismos adquiridos intraparto y el comienso de la
sespsis neonatal temprana puede ser menor o igual a 3 dias.

PTS: 1
2. ANS: A
El estreptococo grupo B (EGB) y los microorganismos entéricos gramnegativos
(predominantemente, Escherichia coli) son responsables de la mayoría de los casos de sepsis de
inicio temprano.

PTS: 1
3. ANS: A
La transmisión de ciertos patógenos virales, protozoos y treponemas puede producirse por
diseminación hemática y transplacentaria de la infección materna.

PTS: 1
4. ANS: D
En la sepsis neonatal el factor de riesgo más importante de la sepsis de inicio tardío es el parto
pretérmino, los microorganismos grampositivos (p. ej., estafilococos coagulasa-negativos
y Staphylococcus aureus) pueden provenir del ambiente o de la piel del niño, lLos focos iniciales
de infección pueden ser las vías urinarias, los senos paranasales, el oído medio, los pulmones o el
aparato digestivo, desde donde más tarde pueden diseminarse a las meninges, los riñones, los
huesos, las articulaciones, el peritoneo y la piel.

PTS: 1
5. ANS: D
La sepsis neonatal es una infección invasiva, en general bacteriana, que se produce durante el
período neonatal, los signos son múltiples e incluyen disminución de la actividad espontánea,
succión menos enérgica, apnea, bradicardia, inestabilidad térmica, dificultad respiratoria,
vómitos, diarrea, distensión abdominal, inquietud, convulsiones e ictericia.

PTS: 1

1
Sepsis Neonatal [Answer Strip] ID: A

A
_____ 3. D
_____ 4. D
_____ 5.

D
_____ 1.

A
_____ 2.
Name: ________________________ Class: ___________________ Date: __________ ID: A

Pediatría

Multiple Choice
Identify the choice that best completes the statement or answers the question.

____ 1. Seleccione el tratamiento inicial de asma en niños mayores de 5 años:


a. Agonista Beta 2 de larga c. Corticoide inhalado como la
duración, como el salmeterol, fluticasona
inhalado
b. Agonista Beta 2 de corta d. Antagonista de receptores de los
duración, como el salbutamol, leucotrienos, como montelukast
inhalado
____ 2. Paciente de 4 años, presenta desde hace 24 horas evacuaciones líquidas, abundantes y
frecuentes; tiene vómitos continuos e intolerancia oral; presenta signo del pliegue bien
definido y la piel regresa muy lentamente a su lugar. Indique el diagnóstico y
tratamiento correcto para este paciente:
a. Deshidratación leve. Administrar c. Deshidratación grave. Administrar
por vía oral sales de rehidratación por vía oral: sales de rehidratación
oral 1000 ml en las primeras 4 oral 1000 ml en las primeras 4
horas horas
b. Deshidratación grave. Administrar d. Deshidratación moderada.
por vía intravenosa: Ringer lactato Administrar por vía intravenosa:
50 ml/kg en la primera hora, luego Ringer lactato 50 ml/kg en la
20ml/kg en la 2da y 3ra horas primera hora, luego 20 ml/kg en la
2da y 3ra hora
____ 3. A continuación, se describen las maniobras para diagnosticar displasia de cadera u otras
patologías. Señale la opción correcta:
a. Maniobra de Barlow: se aduce la c. Prueba de Flexión anterior de
cadera flexionada y se empuja el Adams: al flexionar el muslo y
muslo hacia adelante, sintiéndose hacer maniobra de abducción se
el clank característico de la siente el clic propio de la luxación
luxación
b. Maniobra de Ortolani: se trata de d. Signo de Galealzzi: hay reducción
reducir la cadera lujada, al realizar de la abducción de la cadera y
la abducción de la cadera acortamiento del muslo, sin
asimetría de pliegues glúteos
____ 4. En el servicio de emergencia tiene un paciente pediátrico con una crisis convulsiva tónico
clónica generalizada. Determine el orden de prioridad, de las siguientes acciones, para
su tratamiento inmediato:
1) Canalizar vía venosa.
2) Abrir-Posicionar la vía aérea y evaluar el estado cardiovascular.
3) Administrar diazepam intravenoso 0,3 mg/kg peso.
4) Administrar oxígeno.

a. 1,2,3,4 c. 3,1,2,4
b. 4,1,3,2 d. 2,4,1,3

1
Name: ________________________ ID: A

____ 5. Julia es una lactante de 18 meses de edad que acude a su consulta porque hace 3 días
presenta abundante rinorrea, fiebre, malestar general, hiporexia, tos esporádica y
congestión nasal que no le permite dormir bien en la noche porque le falta el aire. Al
examen físico presenta temperatura de 38°C, congestión nasal, orofaringe ligeramente
eritematosa, el examen de oído y pulmones está normal. Escoja la opción de
tratamiento adecuada para la paciente
a. Acetaminofén, cetirizina más c. Acetaminofén y lavado nasal con
peudoefedrina solución salina
b. Acetaminofén, pseudoefedrina y d. Vitamina C, dextrometorfano y
miel de abeja loratadina
____ 6. Seleccione el plan C de rehidratación para el tratamiento de deshidratación grave:
a. Lactato Ringer = 75 ml/kg: 50 c. Lactato Ringer = 75 ml/kg: 25
ml/kg en la primera hora, y 25 ml/kg en la primera hora, 25 ml/kg
ml/kg la segunda hora, en la segunda hora y 25 ml/kg la
intravenosamente tercera hora
b. Lactato Ringer = 100 ml/kg, IV: 50 d. Lactato Ringer = 100 ml/kg: 25
ml/kg en la primera hora, 25 ml/kg ml/kg cada hora por 4 horas,
en la segunda hora y 25 ml/kg la administrado por vía intravenosa
tercera hora
____ 7. Niña de 7 años, con antecedentes personales de rinitis alérgica y asma bronquial, es
llevada a emergencia con: taquipnea, retracciones intercostales, crepitaciones y
sibilancias bilaterales; saturación de oxígeno 80% con FiO2 0.21. El médico a cargo
utiliza para el tratamiento los siguientes fármacos: metilprednisolona, salbutamol,
aminofilina y montelukast. Usted conoce que la inflamación es la base fisiopatológica
del asma. Seleccione la opción en la que se encuentre el medicamento que resuelve
mejor la crisis asmática, considerando la fisiopatología del asma.
a. Salmeterol c. Metilprednisolona
b. Aminofilina d. Montelukast

____ 8. Un niño presenta una biometría hemática con valores de hemoglobina y hematocrito por
debajo de la normalidad, y un Volumen Corpuscular Medio (VCM) de 108 fL. Con estos
datos seleccione el diagnóstico más probable:
a. Anemia normocítica c. Anemia ferropénica
b. Anemia megaloblástica d. Anemia microcítica

____ 9. Paciente de 5 años, con evacuaciones liquidas con moco y sangre, de 2 horas de
evolución, sediento. Afebril. Está bien hidratado. Seleccione el diagnóstico y el manejo
correcto para este paciente:
a. Sin Deshidratación. Suero oral c. Disentería. Suero oral en casa de
1000 ml. en 4 horas; luego alta 100 a 200 mi después de cada
con zinc 20 mg QD por 14 días e evacuación, zinc 20 mg QD x 14
indicación de signos de alarma días y control en 3 días
b. Sin Deshidratación. Alta con d. Disentería. Alta con información
información de los signos de de los signos de alarma y
alarma, abundantes líquidos y suministro de zinc 10 mg vía oral
regresar a control en 5 días. diario durante 14 días
____ 10. ¿Qué signo al examen físico caracteriza a la anorexia nerviosa?
a. Signo de Russell c. Lanugo en la cara
b. Borborigmos d. Fractura y pérdida dental

2
Name: ________________________ ID: A

____ 11. Una paciente de 5 años ingresa con diagnóstico de desnutrición grave de origen
primario. Seleccione la complicación que está asociada a mortalidad en las primeras
horas de ingreso:
a. Hipotermia e Hipoglicemia c. Insuficiencia cardiaca y renal
b. Anemia y deshidratación d. Insuficiencia hepática y sepsis

____ 12. Según el marco legal de la violencia de género, seleccione la afirmación correcta:
a. El profesional de salud tiene c. El médico debe guardar una copia
obligatoriedad de dar la atención de los informes emitidos, para su
médica, pero la denuncia es archivo personal
decisión de los familiares y la
víctima
b. En el Ecuador, la violencia de d. En violencia de género contra la
género no ha sido declarada niñez, los informes periciales solo
como un problema de salud podrán ser realizados por peritos
pública acreditados
____ 13. Seleccione la opción correcta para el tratamiento de un paciente de 4 años, con 17 kg, en
cuyo coproparasitario se evidencia quistes y trofozoitos de Giardia lamblia y huevos de
Ascaris lumbricoides:
a. Metronidazol de 125 mg / 5 ml en c. Albendazol 400 mg PO el 1 er día;
suspensión: 3.5 mi TID por 7 días; al día siguiente, metronidazol de
al día siguiente Albendazol 400 125 mg / 5 ml en suspensión: 3.5
mg PO una sola toma mi TID por 7 días
b. Metronidazol 3.5 ml de d. Albendazol 400 mg PO el 1er día;
suspensión de 250 mg / 5 ml, TID al día siguiente metronidazol de
durante 7 días; al día siguiente 250 mg / 5 ml en suspensión: 4 mi
albendazol 400 mg PO una vez TID por 5 días
____ 14. Angelita tiene 8 años, hace una semana presentó alza térmica de 38 grados centígrados,
dolor de cabeza, rinorrea y malestar general. Hace 24 horas presentó eritema facial
que tienen el aspecto de “mejillas abofeteadas” y ahora, el exantema se extendió al
tronco y extremidades como eritema macular difuso respetando palmas y plantas.
Escoja la opción correcta con relación al diagnóstico y al tratamiento
a. Parvovirus B19 - control por c. Paramyxoviridae - medidas de
consulta externa soporte
b. Rubivirus - antipiréticos y d. Herpes 6 - tratamiento sintomático
analgésicos
____ 15. Señale la enfermedad aguda autolimitada de la lactancia y la primera infancia que se
caracteriza por la aparición brusca de fiebre alta, que suele resolverse después de 72
horas (crisis), pero puede disminuir de forma gradual durante un día (lisis) coincidiendo
con la aparición en el tronco de un exantema morbiliforme de color rosa o ligeramente
enrojecido y no pruriginoso, de 2-3 mm. El exantema suele durar 1-3 días, pero a
menudo se describe como evanescente y puede ser visible sólo durante horas,
extendiéndose desde el tronco hasta la cara y las extremidades. Los signos asociados
pueden consistir en congestión leve de la faringe, la conjuntiva palpebral o el tímpano,
y en un aumento de tamaño de los ganglios suboccipitales
a. Rubéola c. Mononucleosis infecciosa
b. Roséola d. Sarampión

3
Name: ________________________ ID: A

____ 16. ¿A cuál de los siguientes parásitos corresponde el enunciado?: Parásito que en la
mayoría de los individuos no presenta signos ni síntomas. Los problemas clínicos más
frecuentes se deben a la enfermedad pulmonar y a la obstrucción intestinal o del tracto
biliar. Las larvas que migran por estos tejidos pueden causar síntomas alérgicos, fiebre,
urticaria y granulomatosis
a. Uncinariasis c. Giardiasis
b. Amebíasis d. Ascariasis

____ 17. Luego de cumplir su primer año de vida, un niño que reside en Quito y que tiene todas
las vacunas para la edad recibió la vacuna contra sarampión, rubéola y parotiditis
(SRP) a los 12 meses. Su madre pregunta ¿cuál sería la siguiente vacuna según la
historia inmunológica del niño?
a. La siguiente vacuna es la c. La siguiente vacuna es la
segunda dosis de la SRP pentavalente
b. La siguiente vacuna es la d. La siguiente vacuna es la de la
antineumocócica conjugada fiebre amarilla
____ 18. María es una madre lactante que vive en una parroquia rural de la sierra. Este mes va a
iniciar la introducción de alimentos a su hija. Escoja el alimento menos indicado para
introducir a los 6 meses de edad, de acuerdo con las normas del Ministerio de Salud
Pública del Ecuador
a. Huevo c. Manzana
b. Zanahoria d. Garbanzo

____ 19. Sofía tiene 17 años y es traída a la consulta externa por su madre, debido a que desde
hace 6 meses ha cambiado su comportamiento, ha hecho varias dietas sugeridas por
sus amigas del colegio. Desde entonces ha bajado de peso en un 20% y siempre
busca la aprobación de su familia de cómo se ve antes de salir de la casa y a pesar de
sus comentarios, ella siempre se ve gorda. Se niega a salir con la familia cuando van a
restaurantes porque tiene miedo de subir de peso. Al ser este un trastorno de la
conducta alimentaria, según el DSM V todos son criterios diagnósticos de anorexia
nerviosa, EXCEPTO:
a. Peso marcadamente bajo c. Amenorrea
b. Alteración de la percepción de d. Miedo intenso a ganar peso
peso
____ 20. RN de parto céfalo-vaginal a las 39 semanas. APGAR= 7-9. Su peso es adecuado para
la edad gestacional. Recibe lactancia materna exclusiva. Madre y niño son 0 Rh+. Al
tercer día se observa ictericia con bilirrubinas de 5 mg /dl. Seleccione el diagnóstico
correcto:
a. Ictericia asociada a lactancia c. Kernicterus
materna
b. Ictericia fisiológica d. Ictericia por hemolisis

____ 21. Seleccione la opción en la que todos los microorganismos propuestos producen con
mayor frecuencia otitis media aguda en pacientes pediátricos:
a. Streptococcus pneumoniae, c. Streptococcus pyogenes,
Haemophilus influenzas y Staphylococcus aureus y
Moraxella catarrhalis Enterobius vermicularís
b. Escherichia coli d. Staphylococcus aureus y
enterohemorrágico, Streptococcus [3-hemolítico del
Campylobacterjejuni y Shigella grupo A

4
Name: ________________________ ID: A

____ 22. Seleccione la opción que presenta un conjunto de signos de alarma que la madre debe
reconocer en un niño con disentería:
con disentería:
a) Fiebre alta.
b ) Bebe ávidamente.
c) Cólicos abdominales.
d ) Ojos hundidos.
e ) Dificultad o dolor al orinar.
f) Persistencia de sangre en heces.

a. b, d, c c. a, d, f
b. a, c, e d. b, c, d
____ 23. Un escolar de 10 años presenta desde hace dos semanas prurito anal, cada vez más
intenso y molesto. El coproparasitario reporta la presencia de Enterobius vermicularis.
Seleccione el tratamiento correcto:
a. Albendazol, 400 mg en una toma, c. Albendazol, 400 mg en una sola
que se repite después de dos dosis, junto con alimentos.
semanas. Además, medidas Además, medidas higiénicas para
higiénicas para eliminar el eliminar el parásito en ropa de
parásito en ropa de cama y evitar cama y evitar el contagio familiar
el contagio familiar
b. Albendazol, 200 mg en una toma, d. Albendazol, 400 mg diarios
que se repite después de dos durante dos días seguidos, junto
semanas. Además, medidas con alimentos. Además, medidas
higiénicas para eliminar el higiénicas para eliminar el
parásito en ropa de cama y evitar parásito en la ropa de cama y
el contagio familiar evitar el contagio familiar
____ 24. Para el tratamiento de un paciente con neumonía, se puede utilizar distintos
antimicrobianos, de acuerdo con el germen infectante y a las características
farmacológicas del antibiótico. Seleccione la opción en que el antimicrobiano y la
afirmación sean correctos:
a. Cefuroxima es útil para c. Penicilina G está indicada para
infecciones por gérmenes gram neumonía por Staphylococcus
negativos aureus
b. Gentamicina está indicada para d. Vancomicina es útil para tratar
infecciones por gérmenes neumonía por gérmenes gram
anaerobios gram negativos negativos multirresistentes
____ 25. El coproparasitario de un paciente de 9 años, 28 kg, procedente del oriente ecuatoriano,
reporta huevos de Ancylistoma duodenalis y de Enterobius vermicularís. Seleccione el
tratamiento adecuado para este paciente:
a. Albenzadol 400 mg vía oral por 3 c. Albenzadol 200 mg vía oral.
días + Metronidazol 250 mg tres Repetir igual dosis en 2 semanas.
veces al día por 7 días. Higiene Medidas de higiene personal y
personal y familiar familiar
b. Albenzadol 400 mg vía oral. d. Albenzadol 400 mg vía oral dosis
Repetir igual dosis en 2 semanas. diaria por 5 días. Repetir en 2
Medidas de higiene personal y semanas. Medidas de higiene
familiar personal y familiar

5
Name: ________________________ ID: A

____ 26. Andrés es un niño de 7 años que acude a la consulta externa acompañado por sus
padres por presentar dolor de garganta y fiebre. Al examen físico se evidencia exudado
en las amígdalas y ganglios dolorosos en la región cervical anterior. Todos podrían ser
tratamientos indicados para este cuadro, EXCEPTO:
a. Azitromicina c. Penicilina benzatínica
b. Ceftriaxona d. Amoxicilina

____ 27. Según la guía de Atención Integrada a las Enfermedades Prevalentes en la Infancia
(AIEPI), seleccione la opción que presenta los criterios de referencia para un paciente
pediátrico con sibilancias:
a ) Saturación menor a 90% por oximetría de pulso.
b ) Cuadro gripal de 3 días de evolución.
c ) No toma del seno.
d ) Tos persistente.
e ) Signos de deshidratación.
f ) Antecedentes de episodios previos de sibilancias.

a. b, e, f c. a, b, d
b. a, c, e d. b, c, d
____ 28. Niña de 2 años, tiene evacuaciones líquidas, abundantes e intolerancia a la vía oral.
Presenta signo del pliegue en abdomen y la piel vuelve muy lentamente a su lugar. Su
llanto es débil y sin lágrimas. Se intentó cateterizar una vía endovenosa periférica, sin
éxito. Indique la conducta a seguir:
a. Administrar un antiemético IM, c. Cateterizar vía venosa central
esperar 40 minutos y probar
tolerancia oral
b. Iniciar hidratación con sonda d. Seguir intentando cateterizar vía
nasogástríca venosa periférica
____ 29. Paciente de 3 años, presenta tos desde hace 4 días. T = 38 C°; FC = 85 x 1’; FR = 52 x
1'; Sat. Oxígeno = 93%. No se evidencia estridor y a la auscultación pulmonar no
presenta disminución de murmullo vesicular ni sibilancias. Seleccione el tratamiento
adecuado:
a. Dexametasona 0,5mg/kg/dosis, c. Adrenalina en nebulización
inicialmente 0,5mg/kg dosis
b. Referir a un centro de salud de d. Amoxicilina 80 - 90 mg/kg/día en 2
mayor nivel tomas, 3 días
____ 30. Una niña de 3 años, 14 kg de peso, presenta un urocultivo con >100 000 UfC de
Escherichia Colí, sensible a cefuroxima. Seleccione el tratamiento adecuado:
a. Cefuroxima, suspensión de 250 c. Cefuroxima, suspensión de 250
mg / 5 mi: 3.5 mi cada 12 horas mg / 5ml: 2.5ml cada 12 horas por
por 10 días 10 días
b. Cefuroxima, suspensión de 125 d. Cefuroxima, suspensión de 125
mg / 5ml: 5 mi cada 12 horas por mg / 5ml: 4ml cada 12 horas por
10 días 10 días

6
Name: ________________________ ID: A

____ 31. Un paciente acude a su consulta con síntomas y resultados de un análisis de orina
compatibles con ITU (Infección tracto Urinario). Señale el examen complementario que
le permite tener el diagnóstico definitivo:
a. Parcial de orina c. Urocultivo
b. Gammagrafía renal d. Leucograma

____ 32. Un escolar de 10 años, 40 kg de peso, tiene fiebre de 39,7°C, odinofagia, lengua
enrojecida como frambuesa y con una pasta blanca en los bordes; mejillas rubicundas
con palidez perioral; además, presenta exantema puntiforme que se extiende en todo el
cuerpo y la piel es áspera al tacto. En los pliegues de los codos, inglés y axilas la piel
es de un color rojo intenso. Seleccione el diagnóstico más probable, el germen
productor de esta enfermedad y el tratamiento respectivo.
a. Quinta enfermedad, Parvovirus c. Escarlatina, Streptococcus 
B16. Tratamiento sintomático con hemoliticus y penicilina G
paracetamol benzatínica 1.200.000 Ul IM por
una vez + paracetamol
b. Escarlatina, Staphilococcus d. Erisipela, Streptococcus 
aureus y penicilina G benzatínica hemoliticus y penicilina G
1.200.000 Ul IM por una vez + benzatínica 600.000 Ul IM por una
paracetamol vez + paracetamol
____ 33. Niño de 3 años presenta fiebre de 39 °C y otalgia izquierda; la madre ha administrado
paracetamol. Al examen físico presenta: eritema timpánico bilateral, rinorrea mucoide,
faringe inflamada linfonodular; la otoscopía neumática muestra disminución de la
movilidad de la membrana timpánica. Seleccione uno de los siguientes hallazgos
médicos que tiene mayor correlación con el diagnóstico de otitis media aguda:
a. Disminución de la movilidad de la c. Fiebre 39 grados acompañada de
membrana timpánica rinorrea
b. Otalgia izquierda de aparición d. Eritema timpánico bilateral en la
aguda otoscopía
____ 34. ¿Qué signo al examen físico caracteriza a la anorexia nerviosa?
a. Fractura y pérdida dental c. Borborigmos
b. Signo de Russell d. Lanugo en la cara

____ 35. Seleccione la opción en la que todos los microorganismos propuestos producen con
mayor frecuencia otitis media aguda en pacientes pediátricos:
a. Streptococcus pyogenes, c. Staphylococcus aureus y
Staphylococcus aureus y Streptococcus  -hemolítico del
Enterobius vermicularis. grupo A
b. Escherichia coli d. Streptococcus pneumoniae,
enterohemorrágico, Haemophilus influenzae y
Campylobacter jejuni y Shigella Moraxella catarrhalis

7
Name: ________________________ ID: A

____ 36. Niña de 2 años, tiene evacuaciones líquidas, abundantes e intolerancia a la vía oral.
Presenta signo del pliegue en abdomen y la piel vuelve muy lentamente a su lugar. Su
llanto es débil y sin lágrimas. Se intentó cateterizar una vía endovenosa periférica, sin
éxito. Indique la conducta a seguir:
a. Seguir intentando cateterizar vía c. Cateterizar vía venosa central
venosa periférica
b. Administrar un antiemético IM, d. Iniciar hidratación con sonda
esperar 40 minutos y probar nasogástrica
tolerancia oral
____ 37. A continuación, se describen las maniobras para diagnosticar displasia de cadera u otras
patologías. Señale la opción correcta:
a. Prueba de Flexión anterior de c. Maniobra de Barlow: se aduce la
Adams: al flexionar el muslo y cadera flexionada y se empuja el
hacer maniobra de abducción se muslo hacia adelante, sintiéndose
siente el clic propio de la luxación el clank característico de la
luxación
b. Signo de Galealzzi: hay reducción d. Maniobra de Ortolani: se trata de
de la abducción de la cadera y reducir la cadera lujada, al realizar
acortamiento del muslo, sin la abducción de la cadera
asimetría de pliegues glúteos
____ 38. Sofía tiene 17 años y es traída a la consulta externa por su madre, debido a que desde
hace 6 meses ha cambiado su comportamiento, ha hecho varias dietas sugeridas por
sus amigas del colegio. Desde entonces ha bajado de peso en un 20% y siempre
busca la aprobación de su familia de cómo se ve antes de salir de la casa y a pesar de
sus comentarios, ella siempre se ve gorda. Se niega a salir con la familia cuando van a
restaurantes porque tiene miedo de subir de peso. Al ser este un trastorno de la
conducta alimentaria, según el DSM V todos son criterios diagnósticos de anorexia
nerviosa, EXCEPTO:
a. Amenorrea c. Miedo intenso a ganar peso
b. Peso marcadamente bajo d. Alteración de la percepción de
peso
____ 39. En el servicio de emergencia tiene un paciente pediátrico con una crisis convulsiva tónico
clónica generalizada. Determine el orden de prioridad, de las siguientes acciones, para
su tratamiento inmediato:
1) Canalizar vía venosa.
2) Abrir-Posicionar la vía aérea y evaluar el estado cardiovascular.
3) Administrar diazepam intravenoso 0,3 mg/kg peso.
4) Administrar oxígeno.

a. 1,2,3,4 c. 2,4,1,3
b. 4,1,3,2 d. 3,1,2,4

8
Name: ________________________ ID: A

____ 40. Seleccione la opción que presenta un conjunto de signos de alarma que la madre debe
reconocer en un niño con disentería:
a) Fiebre alta.
b) Bebe ávidamente.
c) Cólicos abdominales.
d) Ojos hundidos.
e) Dificultad o dolor al orinar.
f ) Persistencia de sangre en heces.

a. a, d, f c. b, d, c
b. b, c, d d. a, c, e
____ 41. Un niño presenta una biometría hemática con valores de hemoglobina y hematocrito por
debajo de la normalidad, y un Volumen Corpuscular Medio (VCM) de 108 fL. Con estos
datos seleccione el diagnóstico más probable:
a. Anemia megaloblástica c. Anemia ferropénica
b. Anemia microcítica d. Anemia normocítica

____ 42. Paciente de 5 años, con evacuaciones liquidas con moco y sangre, de 2 horas de
evolución, sediento. Afebril. Está bien hidratado. Seleccione el diagnóstico y el manejo
correcto para este paciente:
a. Disentería. Alta con información c. Disentería. Suero oral en casa de
de los signos de alarma y 100 a 200 mi después de cada
suministro de zinc 10 mg vía oral evacuación, zinc 20 mg QD x 14
diario durante 14 días. días y control en 3 días
b. Sin Deshidratación. Suero oral d. Sin Deshidratación. Alta con
1000 ml. en 4 horas; luego alta información de los signos de
con zinc 20 mg QD por 14 días e alarma, abundantes líquidos y
indicación de signos de alarma regresar a control en 5 días
____ 43. El coproparasitario de un paciente de 9 años, 28 kg, procedente del oriente ecuatoriano,
reporta huevos de Ancylistoma duodenalis y de Enterobius vermicularís. Seleccione el
tratamiento adecuado para este paciente:
a. Albenzadol 400 mg vía oral. c. Albenzadol 400 mg vía oral dosis
Repetir igual dosis en 2 semanas. diaria por 5 días. Repetir en 2
Medidas de higiene personal y semanas. Medidas de higiene
familiar personal y familiar
b. Albenzadol 200 mg vía oral. d. Albenzadol 400 mg vía oral por 3
Repetir igual dosis en 2 semanas. días + Metronidazol 250 mg tres
Medidas de higiene personal y veces al día por 7 días. Higiene
familiar personal y familiar

9
Name: ________________________ ID: A

____ 44. Según el marco legal de la violencia de género, seleccione la afirmación correcta:
a. En violencia de género contra la c. En el Ecuador, la violencia de
niñez, los informes periciales solo género no ha sido declarada
podrán ser realizados por peritos como un problema de salud
acreditados pública
b. El profesional de salud tiene d. El médico debe guardar una copia
obligatoriedad de dar la atención de los informes emitidos, para su
médica, pero la denuncia es archivo personal
decisión de los familiares y la
víctima
____ 45. Andrés es un niño de 7 años que acude a la consulta externa acompañado por sus
padres por presentar dolor de garganta y fiebre. Al examen físico se evidencia exudado
en las amígdalas y ganglios dolorosos en la región cervical anterior. Todos podrían ser
tratamientos indicados para este cuadro, EXCEPTO:
a. Azitromicina c. Penicilina benzatínica
b. Amoxicilina d. Ceftriaxona

____ 46. Un escolar de 10 años presenta desde hace dos semanas prurito anal, cada vez más
intenso y molesto. El coproparasitario reporta la presencia de Enterobius vermicularis.
Seleccione el tratamiento correcto:
a. Albendazol, 200 mg en una toma, c. Albendazol, 400 mg en una toma,
que se repite después de dos que se repite después de dos
semanas. Además, medidas semanas. Además, medidas
higiénicas para eliminar el higiénicas para eliminar el
parásito en ropa de cama y evitar parásito en ropa de cama y evitar
el contagio familiar el contagio familiar
b. Albendazol, 400 mg en una sola d. Albendazol, 400 mg diarios
dosis, junto con alimentos. durante dos días seguidos, junto
Además, medidas higiénicas para con alimentos. Además, medidas
eliminar el parásito en ropa de higiénicas para eliminar el
cama y evitar el contagio familiar parásito en la ropa de cama y
evitar el contagio familiar
____ 47. Paciente de 3 años, presenta tos desde hace 4 días. T° = 38 C°; FC = 85 x 1'; FR = 52 x
1'; Sat. Oxígeno = 93%. No se evidencia estridor y a la auscultación pulmonar no
presenta disminución de murmullo vesicular ni sibilancias. Seleccione el tratamiento
adecuado:
a. Referir a un centro de salud de c. Dexametasona 0,5 mg/kg/dosis,
mayor nivel. inicialmente
b. Adrenalina en nebulización 0,5 d. Amoxicilina 80 - 90 mg/kg/día en 2
mg/kg dosis tomas, 3 días

10
Name: ________________________ ID: A

____ 48. Un escolar de 10 años, 40 kg de peso, tiene fiebre de 39,7°C, odinofagia, lengua
enrojecida como frambuesa y con una pasta blanca en los bordes; mejillas rubicundas
con palidez perioral; además, presenta exantema puntiforme que se extiende en todo el
cuerpo y la piel es áspera al tacto. En los pliegues de los codos, inglés y axilas la piel
es de un color rojo intenso. Seleccione el diagnóstico más probable, el germen
productor de esta enfermedad y el tratamiento respectivo.
a. Escarlatina, Streptococcus  c. Escarlatina, Staphllococcus
hemoliticus y penicilina G aureus y penicilina G benzatínica
benzatínica 1.200.000 Ul IM por 1.200.000 Ul IM por una vez +
una vez + paracetamol. paracetamol.
b. Erisipela, Streptococcus  d. Quinta enfermedad, Parvovirus
hemoliticus y penicilina G B16. Tratamiento sintomático con
benzatínica 600.000 Ul IM por una paracetamol.
vez + paracetamol
____ 49. Paciente de 4 años, presenta desde hace 24 horas evacuaciones líquidas, abundantes y
frecuentes; tiene vómitos continuos e intolerancia oral; presenta signo del pliegue bien
definido y la piel regresa muy lentamente a su lugar. Indique el diagnóstico y
tratamiento correcto para este paciente:
a. Deshidratación grave. Administrar c. Deshidratación moderada.
por vía oral: sales de rehidratación Administrar por vía intravenosa:
oral 1000 ml en las primeras 4 Ringer lactato 50 ml/kg en la
horas primera hora, luego 20ml/kg en la
2da y 3ra hora
b. Deshidratación leve. Administrar d. Deshidratación grave. Administrar
por vía oral sales de rehidratación por vía intravenosa: Ringer lactato
oral 1000 ml en las primeras 4 50 ml/kg en la primera hora, luego
horas 20 ml/kg en la 2da y 3ra horas
____ 50. Según la guía de Atención Integrada a las Enfermedades Prevalentes en la Infancia
(AIEPI), seleccione la opción que presenta los criterios de referencia para un paciente
pediátrico con sibilancias:
a) Saturación menor a 90% por oximetría de pulso.
b) Cuadro gripal de 3 días de evolución.
c) No toma del seno.
d) Tos persistente.
e) Signos de deshidratación.
f) Antecedentes de episodios previos de sibilancias.

a. a, b, d c. a, c, e
b. b, c, d d. b, e, f

11
Name: ________________________ ID: A

____ 51. Señale la enfermedad aguda autolimitada de la lactancia y la primera infancia que se
caracteriza por la aparición brusca de fiebre alta, que suele resolverse después de 72
horas (crisis), pero puede disminuir de forma gradual durante un día (lisis) coincidiendo
con la aparición en el tronco de un exantema morbiliforme de color rosa o ligeramente
enrojecido y no pruriginoso, de 2-3 mm. El exantema suele durar 1-3 días, pero a
menudo se describe como evanescente y puede ser visible sólo durante horas,
extendiéndose desde el tronco hasta la cara y las extremidades. Los signos asociados
pueden consistir en congestión leve de la faringe, la conjuntiva palpebral o el tímpano,
y en un aumento de tamaño de los ganglios suboccipitales
a. Rubéola c. Roséola
b. Sarampión d. Mononucleosis infecciosa

____ 52. Seleccione el plan C de rehidratación para el tratamiento de deshidratación grave:


a. Lactato Ringer = 75 ml/kg: 50 c. Lactato Ringer = 75 ml/kg: 25
ml/kg en la primera hora, y 25 ml/kg en la primera hora, 25 ml/kg
ml/kg la segunda hora, en la segunda hora y 25 ml/kg la
intravenosamente tercera hora
b. Lactato Ringer = 100 ml/kg, IV: 50 d. Lactato Ringer = 100 ml/kg: 25
ml/kg en la primera hora, 25 ml/kg ml/kg cada hora por 4 horas,
en la segunda hora y 25 ml/kg la administrado por vía intravenosa
tercera hora
____ 53. Julia es una lactante de 18 meses de edad que acude a su consulta porque hace 3 días
presenta abundante rinorrea, fiebre, malestar general, hiporexia, tos esporádica y
congestión nasal que no le permite dormir bien en la noche porque le falta el aire. Al
examen físico presenta temperatura de 38° C, congestión nasal, orofaringe ligeramente
eritematosa, el examen de oído y pulmones está normal. Escoja la opción de
tratamiento adecuada para la paciente
a. Acetaminofén, cetirizina más c. Vitamina C, dextrometorfano y
pseudoefedrina loratadina
b. Acetaminofén, pseudoefedrina y d. Acetaminofén y lavado nasal con
miel de abeja solución salina
____ 54. Seleccione el tratamiento inicial de asma en niños mayores de 5 años:
a. Agonista Beta 2 de larga c. Agonista Beta 2 de corta
duración, como el salmeterol, duración, como el salbutamol,
inhalado inhalado
b. Corticoide inhalado como la d. Antagonista de receptores de los
fluticasona leucotrienos, como montelukast
____ 55. Seleccione la opción correcta para el tratamiento de un paciente de 4 años, con 17 kg, en
cuyo coproparasitario se evidencia quistes y trofozoitos de Giardia lamblia y huevos de
Ascaris lumbricoides:
a. Metronidazol de 125 mg / 5 mi en c. Metronidazol 3.5 ml de
suspensión: 3.5 ml TID por 7 días; suspensión de 250 mg / 5 ml, TID
al día siguiente Albendazol 400 durante 7 días; al día siguiente
mg PO una sola toma albendazol 400 mg PO una vez
b. Albendazol 400 mg PO el 1er día; d. Albendazol 400 mg PO el 1er día;
al día siguiente metronidazol de al día siguiente, metronidazol de
250 mg / 5 mi en suspensión: 4 ml 125 mg / 5 mi en suspensión: 3.5
TID por 5 días ml TID por 7 días

12
Name: ________________________ ID: A

____ 56. A cuál de los siguientes parásitos corresponde el enunciado: Parásito que en la
mayoría de los individuos no presenta signos ni síntomas. Los problemas clínicos más
frecuentes se deben a la enfermedad pulmonar y a la obstrucción intestinal o del tracto
biliar. Las larvas que migran por estos tejidos pueden causar síntomas alérgicos, fiebre,
urticaria y granulomatosis
a. Ascariasis c. Amebiasis
b. Giardiasis d. Uncinariasis

____ 57. Angelita tiene 8 años, hace una semana presentó alza térmica de 38 grados centígrados,
dolor de cabeza, rinorrea y malestar general. Hace 24 horas presentó eritema facial
que tienen el aspecto de “mejillas abofeteadas” y ahora, el exantema se extendió al
tronco y extremidades como eritema macular difuso respetando palmas y plantas.
Escoja la opción correcta con relación al diagnóstico y al tratamiento
a. Paramyxoviridae - medidas de c. Rubivirus - antipiréticos y
soporte analgésicos
b. Herpes 6 - tratamiento sintomático d. Parvovirus B19 - control por
consulta externa
____ 58. Un paciente acude a su consulta con síntomas y resultados de un análisis de orina
compatibles con ITU (Infección tracto Urinario). Señale el examen complementario que
le permite tener el diagnóstico definitivo:
a. Parcial de orina c. Gammagrafía renal
b. Urocultivo d. Leucograma

____ 59. RN de parto céfalo-vaginal a las 39 semanas. APGAR= 7-9. Su peso es adecuado para
la edad gestacional. Recibe lactancia materna exclusiva. Madre y niño son 0 Rh+. Al
tercer día se observa ictericia con bilirrubinas de 5 mg /di. Seleccione el diagnóstico
correcto:
a. Ictericia por hemolisis c. Kernicterus
b. Ictericia fisiológica d. Ictericia asociada a lactancia
materna
____ 60. Una paciente de 5 años ingresa con diagnóstico de desnutrición grave de origen
primario. Seleccione la complicación que está asociada a mortalidad en las primeras
horas de ingreso:
a. Insuficiencia hepática y sepsis c. Insuficiencia cardiaca y renal
b. Anemia y deshidratación d. Hipotermia e Hipoglicemia

____ 61. Niña de 7 años, con antecedentes personales de rinitis alérgica y asma bronquial, es
llevada a emergencia con: taquipnea, retracciones intercostales, crepitaciones y
sibilancias bilaterales; saturación de oxígeno 80% con FiO2 0.21. El médico a cargo
utiliza para el tratamiento los siguientes fármacos: metilprednisolona, salbutamol,
aminofilina y montelukast. Usted conoce que la inflamación es la base fisiopatológica
del asma. Seleccione la opción en la que se encuentre el medicamento que resuelve
mejor la crisis asmática, considerando la fisiopatología del asma
a. Aminofilina c. Montelukast
b. Salmeterol d. Metilprednisolona

13
Name: ________________________ ID: A

____ 62. Una niña de 3 años, 14 kg de peso, presenta un urocultivo con >100 000 UfC de
Escherichia Coli, sensible a cefuroxima. Seleccione el tratamiento adecuado:
a. Cefuroxima, suspensión de 250 c. Cefuroxima, suspensión de 250
mg / 5ml: 2.5ml cada 12 horas por mg / 5 mi: 3.5 mi cada 12 horas
10 días por 10 días
b. Cefuroxima, suspensión de 125 d. Cefuroxima, suspensión de 125
mg / 5ml: 5 mi cada 12 horas por mg / 5ml: 4ml cada 12 horas por
10 días 10 días
____ 63. Luego de cumplir su primer año de vida, un niño que reside en Quito y que tiene todas
las vacunas para la edad recibió la vacuna contra sarampión, rubéola y parotiditis
(SRP) a los 12 meses. Su madre pregunta ¿cuál sería la siguiente vacuna según la
historia inmunológica del niño?
a. La siguiente vacuna es la c. La siguiente vacuna es la
antineumococcia conjugada segunda dosis de la SRP
b. La siguiente vacuna es la de la d. La siguiente vacuna es la
fiebre amarilla pentavalente
____ 64. María es una madre lactante que vive en una parroquia rural de la sierra. Este mes va a
iniciar la introducción de alimentos a su hija. Escoja el alimento menos indicado para
introducir a los 6 meses de edad, de acuerdo con las normas del Ministerio de Salud
Pública del Ecuador
a. Garbanzo c. Zanahoria
b. Manzana d. Huevo

____ 65. Niño de 3 años presenta fiebre de 39°C y otalgia izquierda; la madre ha administrado
paracetamol. Al examen físico presenta: eritema timpánico bilateral, rinorrea mucoide,
faringe inflamada linfonodular; la otoscopía neumática muestra disminución de la
movilidad de la membrana timpánica. Seleccione uno de los siguientes hallazgos
médicos que tiene mayor correlación con el diagnóstico de otitis media aguda:
a. Otalgia izquierda de aparición c. Fiebre 39 grados acompañada de
aguda rinorrea
b. Eritema timpánico bilateral en la d. Disminución de la movilidad de la
otoscopía membrana timpánica
____ 66. Para el tratamiento de un paciente con neumonía, se puede utilizar distintos
antimicrobianos, de acuerdo con el germen infectante y a las características
farmacológicas del antibiótico. Seleccione la opción en que el antimicrobiano y la
afirmación sean correctos:
a. Gentamicina está indicada para c. Penicilina G está indicada para
infecciones por gérmenes neumonía por Staphylococcus
anaerobios gram negativos aureus
b. Cefuroxima es útil para d. Vancomicina es útil para tratar
infecciones por gérmenes gram neumonía por gérmenes gram
negativos negativos multirresistentes

14
Name: ________________________ ID: A

____ 67. Un niño de 6 años de edad, pesa 20 kg. Desde hace 7 días presenta dolor abdominal
en mesogastrio, tipo cólico con aparición post prandial. Luego del dolor presenta
diarreas líquidas, abundantes, explosivas, blanquecinas, espumosas, esteatorréicas,
mal olientes, con restos alimentarios. Hay flatulencia intensa. Luego de las
deposiciones calma el dolor. Su estado general es bueno. Se repiten de 3 a 4 veces al
día. El abdomen algo abombado y timpánico, presenta dolor a la palpación profunda,
especialmente en el lado derecho del mesogástrico. Seleccione la posible patología y el
tratamiento correcto:
a. Glardiasis, metronldazol en c. Ascariasis, albendazol 400 mg en
suspensión de 250 mg / 5 ml, una sola toma
tomar 2.5 mi después de
desayuno, almuerzo y merienda, 5
días
b. Enterobiasis, metronizadol en d. Amebiasis, metronidazol en
suspensión de 250 mg / 5 ml, suspensión de 250 mg / 5 ml,
tomar 5 mi después de desayuno, tomar 5 ml después de desayuno,
almuerzo y merienda, 10 días almuerzo y merienda, 10 días
____ 68. Un neonato a término es ingresado a fototerapia debido a hiperbilirrubinemia por
incompatibilidad ABO. Una vez iniciado el tratamiento, seleccione los parámetros que
deben ser valorados periódicamente para evaluar la evolución del paciente
1. Hematocrito
2. Bilirrubinas
3. Ictericia en piel
4. Ictericia en escleras

a. 1, 4 c. 1, 2
b. 2, 4 d. 2, 3
____ 69. Una niña de 3 años de edad presenta fiebre y escalofríos. Además, disuria y polaquiuria.
Acude a consulta. Se solicita examen de orina con cultivo y antibiograma. Se confirma
la existencia de una Infección de vías urinarias producida por Escherichia coli. Es la
primera vez que presenta este tipo de infección. El germen identificado es sensible a
ciprofloxaclno, gentamicina, cefuroxima y ceftriaxona. La niña tiene un peso de 13.5 kg.
Seleccionar la mejor conducta terapéutica para la infección de esta paciente:
a. Cefuroxima 200 mg por vía oral, c. Gentamicina, 30 mg IV o IM cada
cada 12 horas, durante 8 días y 8 horas, durante 7 días, porque
examen de orina de control (EMO) produce una respuesta
a los 3-5 días de iniciado el terapéutica más rápida
tratamiento
b. Ceftriaxona, 1 000 mg IV o IM d. Ciprofloxacino 200 mg cada 12
cada día, durante 7 días y control horas durante 8 días. Próximo
en dos semanas con resultados control a los 3 a 5 días con
de exámenes de orina resultado de un nuevo examen de
orina

15
Name: ________________________ ID: A

____ 70. Laura es una adolescente de 17 años de edad que desde hace 6 meses presenta
cambios en su conducta alimentaria, ha incrementado la actividad física, su vestuario
es de preferencia con ropas de mayor talla a la que le corresponde a ella, se
autocalifica como gorda y ha cambiado de humor, en la valoración del estado
nutricional se encuentra por debajo del percentil 3 de las curvas de peso de la OMS.
Seleccione los signos que corresponden para confirmar el diagnóstico inicial de
anorexia nerviosa
1. Piel grasosa, acné y vellos gruesos en cara y tórax.
2. Esmalte dental erosionado, bradicardia, amenorrea.
3. Hipotermia, acrocianosis en manos y pies.
4. Hipertermía, rubicundez en manos y píes.
5. Rebeldía, ansiedad, tristeza.
6. IMC alto, taquicardia, hipermenorrea.

a. 4, 5, 6 c. 1, 2, 3
b. 2, 3, 5 d. 3, 1, 6
____ 71. Priorice los exámenes a solicitar en un neonato con bajo peso y succión débil:
1. Cortisol.
2. Hemograma completo diferencial.
3. Determinación de glucosa plasmática.
4. Interleuquina 6.

a. 1, 2, 3, 4 c. 3, 1, 2, 4
b. 3, 4, 2,1 d. 3, 2, 4, 1
____ 72. Una niña de 12 meses de edad pesa 7.3 kg, su talla es de 69 cm. Presenta anorexia,
gatea poco, no intenta caminar; dice mamá y papá. Se solicitan exámenes de sangre,
orina y heces. El examen de sangre presenta los siguientes valores: Leucocitos 9 500;
Hematíes 3 850 000; Hemoglobina 10.3 g/dl; Hematocrito 34%; Volumen Corpuscular
Medio 73 fL; Hemoglobina Corpuscular Media 23 pe; Reticulocitos 0.5%. Plaquetas 220
000 /mm3. El frotis de sangre periférica indica la presencia de microcitosis, anisocitosis
y polquilocitosis, hipocromía central de los eritrocitos. Los exámenes de orina y heces
son normales. Seleccionar el diagnóstico más probable:
a. Anemia ferropriva o ferropénica, c. Anemia megaloblástica por la
con posible déficit de ácido fólico presencia de anisocitosis y
poiquilocitosis
b. Anemia aplásica por los niveles d. Anemia fisiológica de los
bajos de eritrocitos, hemoglobina, lactantes, por los valores
hematocrito y la presencia de hematológicos encontrados
reticulocitos
____ 73. Los siguientes son factores de riesgo asociados con sepsis neonatal, EXCEPTO:
a. Depresión inicial c. Peso elevado al nacimiento
b. Prematuridad d. Ruptura prematura de membranas

16
Name: ________________________ ID: A

____ 74. Un lactante de dos meses de edad acude con su madre para control de niño sano.
Recibe lactancia materna exclusiva. Fue un recién nacido a término, nació con 3 kg. Al
mes de edad pesó 3.5 kg. El peso actual es de 4.4 kg. Seleccione la respuesta correcta
con relación al crecimiento de este lactante en el último mes:
a. La ganancia de peso de este c. El incremento de peso es
lactante es excesiva. Su adecuado para su edad. Su
crecimiento es inadecuado crecimiento es adecuado
b. La ganancia de peso no es d. Debería haber duplicado el peso
aceptable, debería haber ganado de nacimiento. Su crecimiento es
600 g más inadecuado
____ 75. Un niño de 6 años de edad sufrió un corte en una de sus manos. El niño fue atendido
caseramente, ocho días después de la herida, aparece una secreción amarillenta
espesa en el lugar de la lesión. Se realisó un cultivo y se identificó a la bacteria
Staphylococcus aureus productor de penicilinasa beta lactamasa. Seleccione el
tratamiento más adecuado para este niño, cuyo peso es de 20 kg
a. Amoxicilina 250 mg, en c. Didoxacilina 125 mg en
suspensión, cada 8 horas, por vía suspensión, cada 6 horas, por vía
oral oral
b. Cotrimoxazol, en suspensión de d. Penicilina Benzatínica, 600 000
400/80 mg por 5 ml, 5ml cada 12 Ul, IM al glúteo, por una vez
horas
____ 76. Paciente masculino de 4 meses de edad, acude con su madre quien indica que desde
hace 4 días presenta morrea hialina, estornudos y sensación de alza térmica por lo que
medica paracetamol 9 gotas cada 8 horas, 48 horas después se suma tos húmeda sin
movilización de secreciones, mal estado general, dificultad respiratoria y tinte cianótico
peribucal por lo que acude. Al examen físico: peso 6.5 kg, FR 75, T 37.6 °C, FC 145,
Sat. 83% con O2 ambiental. Mal estado general, pobre succión, se evidencia cianosis
peribucal, ateto nasal, tiraje intercostal, subxifoideo, diaforesis; tórax: murmullo
vesicular disminuido, sibilancias diseminadas, roncus esporádicos y estertores
crepitantes bibasales; abdomen: se palpa borde hepático a 2 cm del reborde costal
derecho. Examen de sangre: leucocitos 16546, Uto 38.8, Hb 13.5, linfocitos 78%,
neutrófilos 39%, plaquetas en 145000. Sobre el caso clínico anterior indique lo correcto:
a. El cuadro es de etiología viral c. Hay que ingresar al paciente e
iniciar tratamiento con Amoxicilina
más Ácido Clavulánico 100 mg
VO cada 8 horas
b. La hepatomegalia es signo d. En Radiografía de tórax se
inequívoco de sepsis encontraría disminuido de la
trama bronco vascular más
atrapamiento aéreo
____ 77. Estadísticamente, ¿cuántos son los recién nacidos que necesitan reanimación para
sobrevivir?
a. Aproximadamente el 10% de los c. 0% Muy poca o ninguna
recién nacidos asistencia
b. Menos del 1% necesitan d. Al menos 90% de los bebés
reanimación

17
Name: ________________________ ID: A

____ 78. La obesidad en los niños es un problema creciente con las correspondientes
consecuencias sobre la salud y la calidad de vida en general, especialmente por sus
complicaciones cardiovasculares. Se han establecido normas internacionales para su
diagnóstico, factores protectores y factores de riesgo para el desarrollo de obesidad en
niños y niñas. Seleccionar el literal que contenga elementos de diagnóstico, factor
protector y factores de riesgo, en ese orden, relacionados con la obesidad en niños y
niñas. Seleccione el literal que contenga elementos de diagnóstico, factor protector y
factores de riesgo, en ese orden, relacionados con la obesidad en niños y niñas:
a. IMC: 95 percentll; lactancia c. IMC: 95 percentll; sucedáneos de
materna; video juegos, televisión, la leche materna; vivir en área
computador urbana
b. IMC: 95 percentll; lactancia d. IMC: 95 percentll; alimentación
materna; video juegos, televisión, complementaria a los 6 meses;
computador vivir en el campo
____ 79. Una niña de 12 años de edad presenta prurito vulvar, disuria y una secreción blanca, a
manera de requesón. El examen de orina es normal. Se ha realizado lavados con un
jabón líquido de ácido láctico, sin embargo, persiste la secreción. Seleccione el literal
que contenga la posible causa y el tratamiento correspondiente:
a. Gardnerella vaginalis; mejorar c. Candida alblcans; mejorar
higiene, metronidazol + higiene, nistatina o miconazol loca
clindamicina
b. Staphylococcus, Streptococcus; d. Shígella spp; mejorar la higiene
mejorar higiene, cefadroxilo por vulvar, cefpodoxima por vía oral
vía oral
____ 80. Seleccione: ¿Cuál es la relación que define a la EHI (encefalopatía hipóxico isquémica)?
A. EHI- La asfixia.
B. Encefalopatía hipóxica isquémica (EHI):
1. RNs que presentan una encefalopatía aguda con afectación hipóxica isquémica.
2. Manifestación clínica neurológica más importante de la asfixia.
3. Es la causa de las alteraciones sistémicas.
4. Puntuación de Apgar bajo, valorado al minuto y cinco minutos después del
nacimiento diagnostica por sí solo asfixia o EHI.
5. Puntuación de Apgar bajo al 1 minuto indica que el RN requiere de una mayor
observación.
6. Síndrome neurológico secundario a las alteraciones estructurales y bioquímicas.

a. B5, B2 c. A4, A1
b. A3, B3 d. B2, B6

18
Name: ________________________ ID: A

____ 81. La integridad de las membranas amníóticas representan una barrera de protección del
feto contra microorganismos del exterior, cuando esta se rompe, existe el riesgo de que
el feto se contamine con dichos gérmenes. ¿Cuál sería la conducta a tomar en un
recién nacido (RN) cuya madre ha presentado rotura prematura de membranas
amnióticas (RPM)?
a. En RN prematuro con historia c. En RN a término con antecedente
materna de RPM >18 h como de parto potenciafmente
único factor de riesgo y sin contaminado (en domicilio, en
sospecha de infección, se vehículo, en camilla,
recomienda la observación clínica contaminación con heces de la
junto a la madre, sin madre, rotura de membranas,
administración sistemática de etc.) y sin signos clínicos
antibióticos sugestivos de Infección, se
recomienda observación clínica
junto a la madre y realización de
hemocultivos
b. En RN a término con historia d. En RN a término con historia
materna de RPM >18 h como materna de RPM >18 h y otro
único factor de riesgo y sin factor de riesgo asociado
sospecha de infección, se (corioamnionitls clínica, fiebre
recomienda la observación clínica materna) o sospecha clínica de
junto a la madre, sin infección, se recomienda tomar
administración sistemática de cultivos de sangre y no empezar
antibióticos tratamiento antibiótico empírico
hasta obtener el resultado
____ 82. Marque el literal correcto con respecto a la Criptorquidea:
a. Se presenta menos c. Entre las secuelas a largo plazo
frecuentemente en prematuros puede producir esterilidad
b. El testículo se encuentra fuera del d. Es una de las menos frecuente de
trayecto de descenso los trastornos de la diferenciación
sexual masculina
____ 83. Un niño de 5 años de edad presenta súbitamente fiebre alta, cefalea, vómito y
convulsiones tónico, clónico generalizadas. En el examen físico se encuentra rigidez de
nuca. ¿Cuál de los siguientes hallazgos era el líquido cefalorraquídeo sugeriría el origen
bacteriano de este proceso?
a. Hipoglucorraquia c. Hematíes crenados
b. Proteínas disminuidas d. Pleocitosis eosinofílica

____ 84. Un paciente de 7 años de edad, con 20 kg de peso, ha sido diagnosticado de disentería
de origen bacteriano. Se inicia hidratación parenteral y ceftrianona. A las 48 horas se
recibe reporte de sensibilidad del antibiograma, para cotrimoxazol, Para completar el
tratamiento por vía oral se decide administrar el antibiótico que demostró sensibilidad.
Se dispone de cotrimoxazol en suspensión, de 400 mg de sulfametonazol 80 mg de
trimetoprima por cada 5ml. Seleccionar el esquema terapéutico adecuado para este
niño:
a. 5 ml cada 12 horas, durante 5 c. 10 ml cada 12 horas, durante 5
días días
b. 5 ml cada 8 horas, durante 7 días d. 10 ml cada 8 horas, durante 7
días.

19
Name: ________________________ ID: A

____ 85. Un escolar de 6 años de edad con un peso de 20 kg presenta desde hace 48 horas,
diarreas mucosas, con presencia de sangre en pequeña cantidad y con sensación
marcada de tenesmo. Estas deposiciones se repiten de 10 a 12 veces en el día. Se
acompañan de dolor abdominal generalizado, estado general normal, paciente a febril y
bien hidratado. El marco colóníco es palpable, tenso y doloroso. El examen
coproparasitarlo reporta huevos de Ascaris lumbrícoides y trofozoitos de Entamoeba
filstolytica. Determinar el tratamiento adecuado para este niño:
a. Metronidazol en suspensión de c. Metronidazol en suspensión de
250mg/5ml, tomar 5 ml, después 250mg/5 mi, tomar 5 ml, después
de desayuno, almuerzo y de desayuno, almuerzo y
merienda por 7 días seguida de merienda, 7 días. Luego
un curso de Albendazol, 200mg albendazol, 400mg en una sola
en una sola toma toma
b. Albendazol, 400mg en una sola d. Albendazol, 400mg en una sola
toma el primer día. Al siguiente toma el primer día. Al siguiente
día, Metronidazol en suspensión día, Metronidazol en suspensión
de 45mg/5ml, tomar 5ml, después de 250mg/5 ml, tomar 5ml,
de desayuno, almuerzo y después de desayuno, almuerzo y
merienda, 3 días merienda, 7 días
____ 86. Alejandro de 14 meses de edad acude al subcentro para control de salud. En su carné de
vacunación se aprecia que ha recibido vacuna BCG a los 2 días de edad, vacuna
pentavalente (difteria, tosferina, tétanos, hasmophilus influenza B y hepatitis B) a los 2
y 4 meses y antipolio oral 3 dosis a los 2, 4 y 6 meses de edad, vacuna rotavirus 1
dosis a los 2 meses de edad, vacuna neumococo conjugada 1 dosis a los 2 meses de
edad. El esquema de inmunizaciones está incompleto por lo que el personal de salud le
solicita a usted que indique las vacunas que deberían administrarse hoy. Observe el
siguiente listado de vacunas y elija la opción más adecuada:
1) Vacuna pentavalente
2) Vacuna rotavirus
3) Vacuna polio oral
4) Vacuna neumocóccica conjugada
5) Vacuna sarampión, parotiditis y rubéola

a. 1, 4, 5 c. 1, 2, 4
b. 2, 4, 5 d. 1, 3, 4

20
Name: ________________________ ID: A

____ 87. Un escolar de 8 años de edad, presenta desde hace 48 horas, fiebre alta, odinofagia,
astenia, cefalea y añórenla. En las últimas horas aparece un exantema eritematoso,
distribuido en todo el cuerpo, especialmente en la cara, tórax y abdomen. Al examen
físico presenta fiebre de 38,7 °C, FC 110 x 1'. El exantema es intenso en las mejillas y
sin embargo, a nivel peribucal se presenta una zona libre de eritema, más bien pálida y
blanquecina. Se observa una coloración oscura en los pliegues de los codos y rodillas,
así como una mayor pigmentación de los surcos palmares. Presenta también
adenomegalias submaxilares, dolorosas a la palpación. Su lengua es de un rojo
intenso, con aumento de la visibilidad de sus papilas, indicar el diagnóstico, la etiología
del proceso y el tratamiento más adecuado:
a. Escarlatina, producida por el c. Varicela, producida por el
Estreptococo  hemolítico del Varicella-zoster virus, podría
grupo A, se trata con una dosis tratarse con aciclovír,
única de penicilina benzatínica 1 dependiendo de la condición
200 000 Ul IM nutricional del paciente
b. Roséola o Exantema súbito, que d. Rubéola, es producida por un
es una infección producida por el Robivirus de la familia Togaviridae
Herpes virus humano 6, que se y solamente requiere tratamiento
trata solamente con paracetamol sintomático del paciente
____ 88. Un escolar de 11 años de edad pesa 35 kg. Desde hace 24 horas presenta fiebre alta,
odinofagia, malestar general y anorexia. T° 39,3 °C; FC: 96 x 1'; TA: 110/70 mmHg;
facías febril con mejillas enrojecidas y palidez perioral. Adenomegalias submaxilares
dolorosas; hipertrofia y congestión amigdalina con exudado blanquecino, purulento.
Algunas petequias diseminadas en el paladar blando. Determine el diagnóstico y
tratamiento de primera elección:
a. Amigdalitis estreptococia aguda, c. Farlngoamigdalitis aguda,
penicilina G benzatínica 600 000 Amoxicillna 500 mg por vía oral c /
Ul IM, una sola vez + Paracetamol 8 horas, durante 7 días +
500 mg PO cada 6 horas, 3 días Paracetamol 500 mg PO cada 6
horas, 3 días
b. Amigdalitis estreptocócíca aguda, d. Amigdalitis estreptocócíca aguda,
azitromicina 200 mg PO QD, por 3 penicilina G benzatínica 1 200 000
días seguidos + Paracetamol 500 Ul IM, una sola vez + Paracetamol
mg PO cada 6 horas, 3 días 500 mg PO cada 6 horas, 3 días
____ 89. Las infecciones respiratorias y la diarrea aguda en niños son problemas epidemiológicos
prevalentes. La investigación y las normas propuestas por organismos internacionales
como OMS y UNICEF, así corno la implementación de estrategias nacionales para
reducir la morbilidad y mortalidad producida por estas patologías, ha logrado reducir su
impacto en el mundo. Identifique las estrategias que han producido importante impacto
para reducir la morbilidad y mortalidad por estas patologías, en menores de 5 años de
edad:
a. Antibioticoterapia precoz, c. Terapia de Rehldrataclón Oral
lactancia materna, hospitalización (TRO), lactancia materna,
inmediata vitamina A, Zinc
b. Vitamina C, hidratación d. Uso de probióticos,
parenteral, Zinc, lactancia antibioticoterapia precoz,
materna tratamiento hospitalario

21
Name: ________________________ ID: A

____ 90. Relacione los percentiles de peso más utilizados en la valoración del estada nutricional
de los niños/as desde recién nacidos a término hasta los 5 años de edad utilizando el
Patrón de Crecimiento de la OMS - Percentil Diagnóstico Nutricional

Percentiles Patrón de crecimiento


1. Percentil 3 A. Normal en límite superior
2. -3DS desde el Percentil 50 B. Obeso grave
3. Percentil 97 C. Desnutrido leve 4
4. + 4DS desde el Percentil 50 D. Normal en límite inferior

a. 4b, 2c, 1d, 3a c. 2d, 1a, 3b, 4c


b. 3d, 4a, 2c, 1b d. 1b, 2d, 3c, 4a
____ 91. Un escolar de 11 años de edad acude a la consulta por presentar edema palpebral
moderado, cefalea y astenia. Su madre refiere que la orina es de color rojizo. Al
examen físico, el paciente está algo pálido y a nivel regional se observa un discreto
edema bi-maleolar. Los signos vitales al momento son: FC: 72 x 1'; FR: 28 x 1'; TA:
130/90. Al indagar a fondo sobre la historia del paciente, su madre refiere que hace dos
semanas aproximadamente presentó sintomatología compatible con escabiosis,
proceso que le generó algunas lesiones supurativas, que han disminuido luego de
baños con agua de matico. El examen de orina demuestra la presencia de piocitos 5-6
por campo, innumerables hematíes por campe, presencia de cilindros hialinos y una
densidad normal. Los resultados del examen de sangre regresan con la novedad de
tener la fracción C3 del complemento disminuida mientras que fe albúmina plasmática
está en 4,5 g/dl. Con este cuadro, ¿Cuál es el diagnóstico más probable?
a. Glomerulonefritis aguda post c. Glomerulonefritis de púrpura de
estreptococia Schónlein-Henoch
b. La enfermedad por cambios d. Síndrome nefrótico
mínimos (ECM)
____ 92. Un lactante mayor de 18 meses de edad acude al subcentro de salud por presentar
desde hace 24 horas diarreas líquidas, abundantes, mal oliente, sin moco ni sangre, en
número de 10. Al examen físico presenta astenia, sin embargo, se pone irritable al
manejo, tiene los ojos algo hundidos, globos oculares blandos y el llanto sin lágrimas.
Su lengua y mucosas orales están secas. Presenta signo de pliegue únicamente en
abdomen. T: 38,7 °C, FC: 116 x 1', FR: 40 x 1'. Llenado capilar rápido. No ha vomitado
y toma un poco de seno materno. Su peso actual es de 9.2 kg. El último registro de su
peso es de 9.8 kg. La madre del lactante refiere que este ha orinado poco y su orina es
da-un color amarillo intenso. Determinar el grado de deshidratación y la mejor opción
para su tratamiento:
a. Primer grado de deshidratación y c. Tercer grado de deshidratación y
administración de TRO (Terapia reposición de líquidos y
de Rehidratación Oral) electrolitos endovenosos
b. Segundo grado de deshidratación d. Segundo grado de deshidratación
y reposicón de líquidos y y administración de TRO (Terapia
electrolitos endovenosos de Rehidratación Oral)
____ 93. ¿Cuándo es necesario aspirar la vía aérea en un recién nacido?
a. En líquido amniótico claro c. En meconial no vigoroso
b. En meconial vigoroso d. En liquido meconial

22
Name: ________________________ ID: A

____ 94. Un lactante mayor de 22 meses de edad con un peso de 12 kg presenta desde hace
más o menos 24 horas fiebre de 38°C, irritabilidad y rinorrea. Este cuadro se agudiza
desde hace 12 horas en las cuales el paciente no quiere comer alimentos ni tomar
biberón. Al examen físico presenta los siguientes signos vitales: T° 38,7 °C; PC 110 x
1'; FR 32 x min; se le observa decaído, con facies de dolor, astenia y congestión nasal
con descarga sera-purulenta moderada a grave. Al examen visual de la orofaringe
presenta congestión faríngea y timpánica bilateral. Al examen de oído la membrana
timpánica está edematosa y tiene una tonalidad inflamatoria en ambos lados. Se
observa un poco de exudado amarillo sobre la membrana timpánica de los dos oídos y
por ende se diagnostica Otitis Media Aguda. Seleccione el tratamiento más adecuado
para este paciente:
a. Azitromicina, 10 mg/kg/día, el c. Azitromicina, 10 mg/kg/día,
primer día; luego 5 mg/kg/día por administrar PO durante 3 días
4 días más + descongestionante, seguidos + paracetamol para el
antihistamínico y paracetamol dolor y la fiebre
para el dolor y la fiebre
b. Paracetamol en gotas de 100 d. Amoxicílina, 60-80 mg/kg/día,
mg/ml, administrar 1 ml cada 6 dividida en 2 o 3 dosis, por 7 a 10
horas y evaluar evolución del días + paracetamol para el dolor y
paciente en 48 horas la fiebre
____ 95. ¿De acuerdo, al cálculo de las necesidades básales ¿Qué cantidad de líquidos básales
necesita un niño de 26.5 kg, y a qué goteo debe pasar?
a. 2650 ml, pasar a 120 ml/hora c. 1550 ml, pasar a 62 rnl/hora
b. 1825 ml, pasar a 73 ml/hora d. 1630 ml, pasar a 68 ml/hora

____ 96. Un niño de 7 años de edad, procedente de una finca del litoral ecuatoriano, presenta
fiebre de 39.2 ’C, mialgias, dolor articular y óseo, dolor retro ocular y molestias gripales
como odinofagia y tos. Su estado general está deteriorado. Además, presenta
petequias distribuidas en todo el cuerpo, excepto en palmas y plantas. En la biometría
presenta leucopenia y trombocitopenia. Se diagnostica dengue grave. Seleccione la
primera prioridad para su tratamiento:
a. Mantener el volumen de los c. Observación y monitorización del
líquidos corporales con la paciente y administración de
administración de una solución dopamina si disminuye la presión
hidroelectrolítica arterial o presenta colapso
vascular
b. Administrar analgésicos para d. Administrar azitromicina, para
calmar sus dolores y esperar la evitar infecciones pulmonares o
evolución del proceso viral intracraneales secundarias
____ 97. Una niña de 13 años de edad presenta secreción vaginal y enrojecimiento vulvar. La
secreción es homogénea, fluida y tiene olor a pescado; pH 5; al microscopio se
observan células diana en el exudado vaginal. Seleccionar el literal que contenga la
posible causa y el tratamiento correspondiente:
a. Staphylococcus, Streptococcus; c. Gardnerella vaginalis; mejorar
mejorar higiene, cefadroxilo por higiene, metronidazol +
vía oral clindamicina
b. Shigella spp; mejorar la higiene d. Candida albicans; mejorar
vulvar, cefpodoxima por vía oral higiene, nistatina por vía oral y
local

23
Name: ________________________ ID: A

____ 98. Los siguientes son factores de riesgo asociados con sepsis neonatal, EXCEPTO:
a. Prematuridad c. Depresión inicial
b. Ruptura prematura de membranas d. Peso elevado al nacimiento

____ 99. Un escolar de 6 años de edad con un peso de 20 kg presenta desde hace 48 horas,
diarreas mucosas, con presencia de sangre en pequeña cantidad y con sensación
marcada de tenesmo. Estas deposiciones se repiten de 10 a 12 veces en el día. Se
acompañan de dolor abdominal generalizado, estado general normal, paciente a febril y
bien hidratado.El marco colónico es palpable, tenso y doloroso. El examen
coproparasitario reporta huevos de Áscaris lumbricoides y trofozoitos de Entamoeba
histolytica. Determinar el tratamiento adecuado para este niño:
a. Metronidazol en suspensión de c. Albendazol, 400mg en una sola
250mg/5 ml, tomar 5 ml, después toma el primer día. Al siguiente
de desayuno, almuerzo y día, Metronidazol en suspensión
merienda, 7 días. Luego de 250mg/5 ml, tomar 5ml,
albendazol, 400mg en una sola después de desayuno, almuerzo y
toma merienda, 7 días
b. Metronidazol en suspensión de d. Albendazol, 400mg en una sola
250mg/5ml, tomar 5 ml, después toma el primer día. Al siguiente
de desayuno, almuerzo y día, Metronidazol en suspensión
merienda por 7 días seguida de de 45mg/5ml, tomar 5ml, después
un curso de Albendazol, 200mg de desayuno, almuerzo y
en una sola toma merienda, 3 días
____ 100. Un lactante mayor de 18 meses de edad acude al subcentro de salud por presentar
desde hace 24 horas diarreas líquidas, abundantes, mal oliente, sin moco ni sangre, en
número de 10. Al examen físico presenta astenia, sin embargo se pone irritable al
manejo, tiene los ojos algo hundidos, globos oculares blandos y el llanto sin lágrimas.
Su lengua y mucosas orales están secas. Presenta signo de pliegue únicamente en
abdomen. T: 38,7 °C, FC: 13.6 x 1', FR: 40 ü 1'. Llenado capilar rápido. No ha vomitado
y toma un poco de seno materno. Su peso actual es de 9.2 kg. El último registro de su
peso es de 9.8 kg. La madre del lactante refiere que este ha orinado poco y su orina es
de un color amarillo intenso. Determinar el grado de deshidratación y la mejor opción
para su tratamiento:
a. Primer grado de deshidratación y c. Tercer grado de deshidratación y
administración de TRO (Terapia reposición de líquidos y
de Rehidratación Oral) electrolitos endovenosos
b. Segundo grado de deshidratación d. Segundo grado de deshidratación
y administración de TRO (Terapia y reposición de líquidos y
de Rehidratación Oral) electrolitos endovenosos

24
Name: ________________________ ID: A

____ 101. Un escolar de 11 años de edad pesa 36 kg. Desde hace 24 horas presenta fiebre alta,
odinofagia, malestar general y anorexia. T° 39,3 °C; FC: 96 x 1'; TA: 110/70 mmHg;
facies febril con mejillas enrojecidas y palidez perioral. Adenomegalias submaxilares
dolorosas; hipertrofia y congestión amlgdalina con exudado blanquecino, purulento.
Algunas petequias diseminadas en el paladar blando. Determine el diagnóstico y
tratamiento de primera elección:
a. Amigdalitis estreptocócica aguda, c. Amigdalitis estreptocócica aguda,
azitromicina 200 mg PO QD, por 3 penicilina G benzatínica 1 200 000
días seguidos + Paracetamol 500 Ul IM, una sola vez + Paracetamol
mg PO cada 6 horas, 3 días 500 mg PO cada 6 horas, 3 días
b. Faringoamigdalitis aguda, d. Amigdalitis estreptocócica aguda,
Amoxicilina 500 mg por vía oral penicilina G benzatínica 600 000
c/8 horas, durante 7 días + Ul IM, una sola vez + Paracetamol
Paracetamol 500 mg PO cada 6 500 mg PO cada 6 horas, 3 días
horas, 3 días
____ 102. Un escolar de 11 años de edad acude a la consulta por presentar edema palpebral
moderado, cefalea y astenia. Su madre refiere que la orina es de color rojizo. Al
examen físico, el paciente está algo pálido y a nivel regional se observa un discreto
edema bi-maleolar. Los signos vitales al momento son: FC: 72 x 1'; FR: 28 x 1'; TA:
130/90. Al indagar a fondo sobre la historia del paciente, su madre refiere que hace dos
semanas aproximadamente presentó sintomatologfa compatible con escabiosis,
proceso que le generó algunas lesiones supurativas, que han disminuido luego de
baños con agua de matico. El examen de orina demuestra la presencia de piocitos 5-6
por campo, innumerables hematíes por campo, presencia de cilindros hialinos y una
densidad normal. Los resultados del examen de sangre regresan con la novedad de
tener la fracción C3 del complemento disminuida mientras que la albúmina plasmática
esta en 4,5 g/dl. Con este cuadro, ¿Cuál es el diagnóstico más probable?
a. Síndrome nefrótico c. Glomerulonefritis de púrpura de
Schónlein-Henoch
b. Glomerulonefritis aguda post d. La enfermedad por cambios
estreptocócica mínimos (ECM)
____ 103. La obesidad en los niños es un problema creciente con las correspondientes
consecuencias sobre la salud y la calidad de vida en general, especialmente por sus
complicaciones cardiovasculares. Se han establecido normas internacionales para su
diagnóstico, factores protectores y factores de riesgo para el desarrollo de obesidad en
niños y niñas. Seleccionar el literal que contenga elementos de diagnóstico, factor
protector y factores de riesgo, en ese orden, relacionados con la obesidad en niños y
niñas. Seleccione el literal que contenga elementos de diagnóstico, factor protector y
factores de riesgo, en ese orden, relacionados con la obesidad en niños y niñas:
a. IMC: 95 percentil; lactancia c. IMC: 95 percentil; alimentación
materna; video juegos, televisión, complementaria a los 6 meses;
computador vivir en el campo
b. IMC: 95 percentil; sucedáneos de d. IMC: 95 percentil; lactancia
la leche materna; vivir en área materna; video juegos, televisión,
urbana computador

25
Name: ________________________ ID: A

____ 104. Un lactante mayor de 22 meses de edad con un peso de 12 kg presenta desde hace
más o menos 24 horas fiebre de 38°C, irritabilidad y rinorrea. Este cuadro se agudiza
desde hace 12 horas en las cuales el paciente no quiere comer alimentos ni tomar
biberón. Al examen físico presenta los siguientes signos vitales: T° 38,7 °C; PC 110 x
1'; FR 32 x min; se le observa decaído, con facies de dolor, astenia y congestión nasal
con descarga sera-purulenta moderada a grave. Al examen visual de la orofaringe
presenta congestión faríngea y timpánica bilateral. Al examen de oído la membrana
timpánica está edematosa y tiene una tonalidad inflamatoria en ambos lados. Se
observa un poco de exudado amarillo sobre la membrana timpánica de los dos oídos y
por ende se diagnostica Otitis Media Aguda. Seleccione el tratamiento más adecuado
para este paciente:
a. Azitromicína, 10 mg/kg/día, el c. Azitromicína, 10 mg/kg/día,
primer día; luego 5 mg/kg/día por administrar PO durante 3 días
4 días más + descongestionante, seguidos + paracetamol para el
antihistamínico y paracetamol dolor y la fiebre
para el dolor y la fiebre
b. Amoxicilina, 60 - 80 mg/kg/día, d. Paracetamol en gotas de 100
dividida en 2 o 3 dosis, por 7 a 10 mg/ml, administrar 1 mi cada 6
días + paracetamol para el dolor y horas y evaluar evolución del
la fiebre paciente en 48 horas
____ 105. Una niña de 12 meses de edad pesa 7.3 kg, su talla es de 69 cm. Presenta anorexia,
gatea poco, no intenta caminar; dice mamá y papá. Se solicitan exámenes de sangre,
orina y heces. El examen de sangre presenta los siguientes valores: Leucocitos 9 500;
Hematíes 3 850 000; Hemoglobina 10.3 g/dl; Hematocrito 34%; Volumen Corpuscular
Medio 73 fL; Hemoglobina Corpuscular Media 23 pe; Reticulocitos 0.5%. Plaquetas 220
000 /mm3. El frotis de sangre periférica indica la presencia de microcitosis, anisocitosis
y polquilocitosis, hipocromía central de los eritrocitos. Los exámenes de orina y heces
son normales. Seleccionar el diagnóstico más probable:
a. Anemia megaloblástica por la c. Anemia fisiológica de los
presencia de anlsocitosis y lactantes, por los valores
poiquilocitosis hematológicos encontrados
b. Anemia ferropriva o ferropénica, d. Anemia aplásica por los niveles
con posible déficit de ácido fólico bajos de eritrocitos, hemoglobina,
hematocrito y la presencia de
reticulocitos

26
Name: ________________________ ID: A

____ 106. Un escolar de 8 años de edad, presenta desde hace 48 horas, fiebre alta, odinofagia,
astenia, cefalea y añórenla. En las últimas horas aparece un exantema eritematoso,
distribuido en todo el cuerpo, especialmente en la cara, tórax y abdomen. Al examen
físico presenta fiebre de 38,7 °C, FC 110 x 1'. El exantema es intenso en las mejillas y
sin embargo, a nivel peribucal se presenta una zona libre de eritema, más bien pálida y
blanquecina. Se observa una coloración oscura en los pliegues de los codos y rodillas,
así como una mayor pigmentación de los surcos palmares. Presenta también
adenomegalias submaxilares, dolorosas a la palpación. Su lengua es de un rojo
intenso, con aumento de la visibilidad de sus papilas, indicar el diagnóstico, la etiología
del proceso y el tratamiento más adecuado:
a. Roséola o Exantema súbito, que c. Rubéola, es producida por un
es una infección producida por el Robivirus de la familia Togaviridae
Herpes virus humano 6, que se y solamente requiere tratamiento
trata solamente con paracetamol sintomático del paciente
b. Escarlatina, producida por el d. Varicela, producida por el
Estreptococo  hemolítlco del Varicella-zoster virus, podría
grupo A, se trata con una dosis tratarse con acidovir,
única de penicilina benzatínica 1 dependiendo de la condición
200 000 Ul IM nutricional del paciente

____ 107. Las infecciones respiratorias y la diarrea aguda en niños son problemas epidemiológicos
prevalentes. La investigación y las normas propuestas por organismos internacionales
como OMS y UNICEF, así corno la implementación de estrategias nacionales para
reducir la morbilidad y mortalidad producida por estas patologías, ha logrado reducir su
impacto en el mundo. Identifique las estrategias que han producido importante impacto
para reducir la morbilidad y mortalidad por estas patologías, en menores de 5 años de
edad:
a. Antibioticoterapla precoz, c. Vitamina C, hidratación
lactancia materna, hospitalización parenteral, Zinc, lactancia
inmediata materna
b. Uso de probióticos, d. Terapia de Rehidratación Oral
antibioticoterapia precoz, (TRO), lactancia materna,
tratamiento hospitalario vitamina A, Zinc
____ 108. ¿De acuerdo al cálculo de las necesidades básales ¿Qué cantidad de líquidos básales
necesita un niño de 26.5 kg, y a qué goteo debe pasar?
a. 1550 ml, pasar a 62 ml/hora c. 2650 ml, pasar a 120 ml/hora
b. 1825 ml, pasar a 73 ml/hora d. 1630 ml, pasar a 68 ml/hora

____ 109. Un lactante de dos meses de edad acude con su madre para control de niño sano.
Recibe lactancia materna exclusiva. Fue un recién nacido a término, nació con 3 kg. Al
mes de edad pesó 3.5 kg. El peso actual es de 4.4 kg. Seleccione la respuesta correcta
con relación al crecimiento de este lactante en el último mes:
a. El incremento de peso es c. Debería haber duplicado el peso
adecuado para su edad. Su de nacimiento. Su crecimiento es
crecimiento es adecuado inadecuado
b. La ganancia de peso no es d. La ganancia de peso de este
aceptable, debería haber ganado lactante es excesiva. Su
600 g más crecimiento es inadecuado

27
Name: ________________________ ID: A

____ 110. Relacione los percentiles de peso más utilizados en la valoración del estada nutricional
de los niños/as desde recién nacidos a término hasta los 5 años de edad utilizando el
Patrón de Crecimiento de la OMS - Percentil Diagnóstico Nutricional

Percentiles Patrón de crecimiento


1. Percentil 3 A. Normal en límite superior
2. -3DS desde el Percentil 50 B. Obeso grave
3. Percentil 97 C. Desnutrido leve 4
4. + 4DS desde el Percentil 50 D. Normal en límite inferior

a. 2d, 1a, 3b, 4c c. 1b, 2d, 3c, 4a


b. 3d, 4a, 2c, 1b d. 4b, 2c, 1d, 3a
____ 111. Seleccione: ¿Cuál es la relación que define a la EHI (encefalopatía hipóxico isquémica)?
A. EHI- La asfixia.
B. Encefalopatía hipóxica isquémica (EHI):
1. RNs que presentan una encefalopatía aguda con afectación hipóxica isquémica.
2. Manifestación clínica neurológica más importante de la asfixia.
3. Es la causa de las alteraciones sistémicas.
4. Puntuación de Apgar bajo, valorado al minuto y cinco minutos después del
nacimiento diagnostica por sí solo asfixia o EHI.
5. Puntuación de Apgar bajo al 1 minuto indica que el RN requiere de una mayor
observación.
6. Síndrome neurológico secundario a las alteraciones estructurales y bioquímicas.

a. B5, B2 c. B2, B6
b. A3, B3 d. A4, A1
____ 112. Un niño de 6 años de edad, pesa 20 kg. Desde hace 7 días presenta dolor abdominal en
mesogastrio, tipo cólico con aparición post prandial. Luego del dolor presenta diarreas
líquidas, abundantes, explosivas, blanquecinas, espumosas, esteatorréicas, mal
olientes, con restos alimentarios. Hay flatulencia intensa. Luego de las deposiciones
calma el dolor. Su estado general es bueno. Se repiten de 3 a 4 veces al día. El
abdomen algo abombado y timpánico, presenta dolor a la palpación profunda,
especialmente en el lado derecho del mesogástrico. Seleccione la posible patología y el
tratamiento correcto:
a. Enterobiasis, metronizadol en c. Amebiasis, metronidazol en
suspensión de 250 mg / 5 mi, suspensión de 250 mg / 5 mi,
tomar 5 mi después de desayuno, tomar 5 mi después de desayuno,
almuerzo y merienda, 10 días almuerzo y merienda, 10 días
b. Glardlasls, metronidazol en d. Ascariasis, albendazol 400 mg en
suspensión de 250 mg / 5 mi, una sola toma
tomar 2.5 mi después de
desayuno, almuerzo y merienda, 5
días

28
Name: ________________________ ID: A

____ 113. Marque el literal correcto con respecto a la Criptorquidea:


a. Es una de las menos frecuente de c. El testículo se encuentra fuera del
los trastornos de la diferenciación trayecto de descenso
sexual masculina
b. Entre las secuelas a largo plazo d. Se presenta menos
puede producir esterilidad frecuentemente en prematuros
____ 114. Paciente masculino de 4 meses de edad, acude con su madre quien indica que desde
hace 4 días presenta morrea hialina, estornudos y sensación de alza térmica por lo que
medica paracetamol 9 gotas cada 8 horas, 48 horas después se suma tos húmeda sin
movilización de secreciones, mal estado general, dificultad respiratoria y tinte cianótico
peribucal por lo que acude. Al examen físico: peso 6.5 kg, FR 75, T 37.6 °C, FC 145,
Sat. 83% con O2 ambiental. Mal estado general, pobre succión, se evidencia cianosis
peribucal, ateto nasal, tiraje intercostal, subxifoideo, diaforesis; tórax: murmullo
vesicular disminuido, sibilancias diseminadas, roncus esporádicos y estertores
crepitantes bibasales; abdomen: se palpa borde hepático a 2 cm del reborde costal
derecho. Examen de sangre: leucocitos 16546, Uto 38.8, Hb 13.5, linfocitos 78%,
neutrófilos 39%, plaquetas en 145000. Sobre el caso clínico anterior indique lo correcto:
a. En Radiografía de tórax se c. El cuadro es de etiología viral
encontraría disminuido de la
trama bronco vascular más
atrapamiento aéreo
b. Hay que ingresar al paciente e d. La hepatomegalia es signo
iniciar tratamiento con Amoxicilina inequívoco de sepsis
más Ácido Clavulánlco 100 mg
VO cada 8 horas
____ 115. Un neonato a término es ingresado a fototerapia debido a hiperbilirrubinemia por
incompatibilidad ABO. Una vez iniciado el tratamiento, seleccione los parámetros que
deben ser valorados periódicamente para evaluar la evolución del paciente
1. Hematocrito
2. Bilirrubinas
3. Ictericia en piel
4. Ictericia en escleras

a. 2, 3 c. 1, 2
b. 1, 4 d. 2, 4

29
Name: ________________________ ID: A

____ 116. Laura es una adolescente de 17 años de edad que desde hace 6 meses presenta
cambios en su conducta alimentaria, ha incrementado la actividad física, su vestuario
es de preferencia con ropas de mayor talla a la que le corresponde a ella, se
autocalifica como gorda y ha cambiado de humor, en la valoración del estado
nutricional se encuentra por debajo del percentil 3 de las curvas de peso de la OMS.
Seleccione los signos que corresponden para confirmar el diagnóstico inicial de
anorexia nerviosa
1. Piel grasosa, acné y vellos gruesos en cara y tórax.
2. Esmalte dental erosionado, bradicardia, amenorrea.
3. Hipotermia, acrocianosis en manos y pies.
4. Hipertermía, rubicundez en manos y píes.
5. Rebeldía, ansiedad, tristeza.
6. IMC alto, taquicardia, hipermenorrea.

a. 1, 2, 3 c. 2, 3, 5
b. 4, 5, 6 d. 3,1, 6
____ 117. La integridad de las membranas amnióticas representan una barrera de protección dal
feto contra microorganismos del exterior, cuando esta se rompe, existe el riesgo de que
el feto se contamine con dichos gérmenes. ¿Cuál sería la conducta a tomar en un
recién nacido (RN) cuya madre ha presentado rotura prematura de membranas
amnióticas (RPM)?
a. En RN a término con antecedente c. En RN a término con historia
de parto potencialmente materna de RPM >18 h y otro
contaminado (en domicilio, en factor de riesgo asociado
vehículo, en camilla, (corioamnionitis clínica, fiebre
contaminación con heces de la materna) o sospecha clínica de
madre, rotura de membranas, infección, se recomienda tomar
etc.) y sin signos clínicos cultivos de sangre y no empezar
sugestivos de infección, se tratamiento antibiótico empírico
recomienda observación clínica hasta obtener el resultado
junto a la madre y realización de
hemocultivos
b. En RN prematuro con historia d. En RN a término con historia
materna de RPM >18 h como materna de RPM >18 h como
único factor de riesgo y sin único factor de riesgo y sin
sospecha de infección, se sospecha de infección, se
recomienda la observación clínica recomienda la observación clínica
junto a la madre, sin junto a la madre, sin
administración sistemática de administración sistemática de
antibióticos antibióticos

30
Name: ________________________ ID: A

____ 118. Un paciente de 7 años de edad, con 20 kg de peso, ha sido diagnosticado de disentería
de origen bacteriano. Se inicia hidratación parenteral y ceftriaxona. A las 48 horas se
recibe reporte de sensibilidad del antibiograma, para cotrimoxazol. Para completar el
tratamiento por vía oral se decide administrar el antibiótico que demostró sensibilidad.
Se dispone de cotrimoxazol en suspensión, de 400 mg de sulfametotiazol + 80 mg de
trimetoprima por cada 5ml. Seleccionar el esquema terapéutico adecuado para este
niño:
a. 5 ml cada 12 horas, durante 5 c. 10 ml cada 8 horas, durante 7
días días
b. 5 ml cada 8 horas, durante 7 días d. 10 ml cada 12 horas, durante 5
días
____ 119. Un niño de 7 años de edad, procedente de una finca del litoral ecuatoriano, presenta
fiebre de 39.2 °C, mialgias, dolor articular y óseo, dolor retro ocular y molestias gripales
como odinofagia y tos. Su estado general está deteriorado. Además, presenta
petequias distribuidas en todo el cuerpo, excepto en palmas y plantas. En la biometría
presenta leucopenia y trombocitopenia. Se diagnostica dengue grave. Seleccione la
primera prioridad para su tratamiento:
a. Observación y monitorización del c. Administrar azitromicína, para
paciente y administración de evitar infecciones pulmonares o
dopamina si disminuye la presión intracraneales secundarias
arterial o presenta colapso
vascular
b. Administrar analgésicos para d. Mantener el volumen de los
calmar sus dolores y esperar la líquidos corporales con la
evolución del proceso viral administración de una solución
hidroelectrolítica
____ 120. ¿Cuándo es necesario aspirar la vía aérea en un recién nacido?
a. En líquido amniótico claro c. En meconial vigoroso
b. En meconlal no vigoroso d. En líquido meconial

____ 121. Alejandro de 14 meses de edad acude al subcentro para control de salud. En su carné de
vacunación se aprecia que ha recibido vacuna BCG a los 2 días de edad, vacuna
pentavalente (difteria, tosferina, tétanos, hasmophilus influenza B y hepatitis B) a los 2
y 4 meses y antipolio oral 3 dosis a los 2, 4 y 6 meses de edad, vacuna rotavirus 1
dosis a los 2 meses de edad, vacuna neumococo conjugada 1 dosis a los 2 meses de
edad. El esquema de inmunizaciones está incompleto por lo que el personal de salud le
solicita a usted que indique las vacunas que deberían administrarse hoy. Observe el
siguiente listado de vacunas y elija la opción más adecuada:
1) Vacuna pentavalente
2) Vacuna rotavirus
3) Vacuna polio oral
4) Vacuna neumocóccica conjugada
5) Vacuna sarampión, parotiditis y rubéola

a. 1, 4, 5 c. 1, 2, 4
b. 1, 3, 4 d. 2, 4, 5

31
Name: ________________________ ID: A

____ 122. Una niña de 12 años de edad presenta prurito vulvar, disuria y una secreción blanca, a
manera de requesón. El examen de orina es normal. Se ha realizado lavados con un
jabón líquido de ácido láctico, sin embargo, persiste la secreción. Seleccione el literal
que contenga la posible causa y el tratamiento correspondiente:
a. Candida albicans; mejorar c. Staphylococcus, Streptococcus;
higiene, nlstatina o miconazol mejorar higiene, cefadroxilo por
local vía oral
b. Shigella spp; mejorar la higiene d. Gardnerella vaginalis; mejorar
vulvar, cefpodoxima por vía oral higiene, metronidazol +
clindamicina
____ 123. Un niño de 6 años de edad sufrió un corte en una de sus manos. El niño fue atendido
caseramente, ocho días después de la herida, aparece una secreción amarillenta
espesa en el lugar de la lesión. Se realizó un cultivo y se identificó a la bacteria
Staphylococcus aureus productor de penicilinasa beta lactamasa. Seleccione el
tratamiento más adecuado para este niño, cuyo peso es de 20 kg.
a. Amoxicilina 250 mg, en c. Cotrimoxazol, en suspensión de
suspensión, cada 8 horas, por vía 400/80 mg por 5 ml, 5ml cada 12
oral horas
b. Penicilina Benzatínica, 600 000 d. Dicloxacilina 125 mg en
Ul, IM al glúteo, por una vez suspensión, cada 6 horas, por vía
oral
____ 124. Priorice los exámenes a solicitar en un neonato con bajo peso y succión débil:
1. Cortisol.
2. Hemograma completo diferencial.
3. Determinación de glucosa plasmática.
4. Interleuquina 6.

a. 1, 2, 3, 4 c. 3, 2, 4,1
b. 3, 1, 2, 4 d. 3, 4, 2,1
____ 125. Una niña de 3 años de edad presenta fiebre y escalofríos. Además, disuria y polaquiuria.
Acude a consulta. Se solicita examen de orina con cultivo y antibiograma. Se confirma
la existencia de una Infección de vías urinarias producida por Escherichia coli. Es la
primera vez que presenta este tipo de infección. El germen identificado es sensible a
ciprofloxaclno, gentamicina, cefuroxima y ceftriaxona. La niña tiene un peso de 13.5 kg.
Seleccionar la mejor conducta terapéutica para la infección de esta paciente:
a. Ciprofloxadno 200 mg cada 12 c. Ceftriaxona, 1 000 mg IV o IM
horas durante 8 días. Próximo cada día, durante 7 días y control
control a los 3 a 5 días con en dos semanas con resultados
resultado de un nuevo examen de de exámenes de orina
orina
b. Cefuroxima 200 mg por vía oral, d. Gentamicina, 30 mg IV o IM cada
cada 12 horas, durante 8 días y 8 horas, durante 7 días, porque
examen de orina de control (EMO) produce una respuesta
a los 3-5 días de iniciado el terapéutica más rápida
tratamiento

32
Name: ________________________ ID: A

____ 126. Un niño de 5 años de edad presenta súbitamente fiebre alta, cefalea, vómito y
convulsiones tónico, clónico generalizadas. En el examen físico se encuentra rigidez de
nuca. ¿Cuál de los siguientes hallazgos era el líquido cefalorraquídeo sugeriría el origen
bacteriano de este proceso?
a. Pleocitosis eosinofílica c. Hematíes crenados
b. Hipoglucorraquia d. Proteínas disminuidas

____ 127. Una niña de 13 años de edad presenta secreción vaginal y enrojecimiento vulvar. La
secreción es homogénea, fluida y tiene olor a pescado; pH 5; al microscopio se
observan células diana en el exudado vaginal. Seleccionar el literal que contenga la
posible causa y el tratamiento correspondiente:
a. Candida albicans; mejorar c. Gardnerella vaginalis; mejorar
higiene, nistatina por vía oral y higiene, metronidazol +
local clindamicina
b. Staphylococcus, Streptococcus; d. Shigella spp-, mejorar la higiene
mejorar higiene, cefadroxilo por vulvar, cefpodoxima por vía oral
vía oral
____ 128. Estadísticamente, ¿cuántos son los recién nacidos que necesitan reanimación para
sobrevivir?
a. Aproximadamente el 10% de los c. 0% Muy poca o ninguna
recién nacidos asistencia
b. Menos del 1% necesitan d. Al menos 90% de los bebés
reanimación
____ 129. Un niño de 5 años de edad acude por primera vez a su consulta, para control de niño
sano. Es un niño activo, comunicativo. Su peso y talla se encuentran en el percentil 50.
Juega fútbol y es amiguero. Sin embargo, a la auscultación cardíaca presenta soplo
sistólico mesocárdico, grado SIA/I, sin una irradiación definida y que, con los cambios
de posición del niño, disminuye notablemente de intensidad. Seleccione el literal
correspondiente a esta descripción:
a. Comunicación interventricular c. Persistencia del conducto
(CIV) arterioso (PCA)
b. Comunicación interauricular (CIA) d. Soplo funcional o inocente

____ 130. A su consulta es llevado un niño de 4 años por presentar un cuadro diarreico de 2 días
de evolución, sin moco ni sangre. Al examen físico se encuentra afebril, su frecuencia
cardiaca es de 85 latidos por minuto, frecuencia respiratoria de 30 par minuto, y tensión
arterial de 100/60 mmHg. Al examen físico se lo nota irritable e intranquilo, no hay
presencia de ojos hundidos, no presenta signo de pliegue cutáneo. El abdomen es
suave, depresible, ios ruidos hidro-aéreos están aumentados, no parece producirse
dolor a la palpación profunda. Señale el manejo del paciente:
a. Plan A de rehidratación y c. Plan A de rehidratación y
prescribir Zinc prescribir antibiótico
b. Plan B de rehidratación y d. Plan B de rehidratación y
prescribir antibiótico prescribir Zinc

33
Name: ________________________ ID: A

____ 131. En la consulta externa del centro de salud tipo C un paciente masculino de 10 meses
de edad es traído por su madre por presentar un cuadro de tos perruna desde hace 24
horas. Al examen físico el niño se encuentra afebril, no presenta cianosis, se evidencia
retracciones torácicas leves. La tos perruna es evidente, así como un estridor
inspiratorio en reposo. ¿Cuál es el tratamiento?
a. Administrar antibiótico c. Administrar de corticoide oral más
intramuscular y nebulizar con nebulización con adrenalina
adrenalina
b. Nebulización con adrenalina d. Nebulización con salbutamol y
prescribir antibiótico oral
____ 132. Garlitos es un niño de 5 años que hace una semana estuvo con resfriad© común. Hace
24 horas presenta primer evento de dolor en el oído derecho, está muy molesto, no ha
hecho fiebre, come bien, pero en algunos momentos ha llorado por el dolor. Al examen
físico se evidencia la membrana timpánica derecha ligeramente eritematosa y
abombada, la membrana izquierda ligeramente eritematosa. Escoja la opción correcta
para el manejo clínico de Garlitos
a. Dar amoxicilina a altas dosis c. Dar azitromicina a 10 mg/kg/día el
80-90mg/kg/día cada 12 horas por primer día y luego 5mg/kg/día por
10 días más acetaminofén 4 días más
b. Dar ibuprofeno a 10mg/kg/dosis d. Dar amoxicilina más ácido
cada 8 horas y esperar 4 días clavulánico 80-90 mg/9-15 mg
para reevaluar si la molestia no ha /kg/día cada 8 horas más
cedido acetaminofén cada 8 horas
____ 133. Algunas parasitosis intestinales pueden causar la pérdida constante de sangre dentro del
huésped. Seleccione cual parasitosis intestinal causa anemia ferropénica en casos de
infestación moderada a severa:
a. Amibiasis c. Uncinariasis
b. Giardiasis d. Balantidiasis

____ 134. Las siguientes complicaciones de los neonatos con retardo del crecimiento intrauterino
(RCIU) son correctas. EXCEPTO:
a. Hipertensión pulmonar c. Policitemia
b. Asfixia d. Hiponatremia
____ 135. En un niño con diarrea aguda viral y que presenta manifestaciones de deshidratación
moderada, se deberá administrar, en las primeras 4 horas, la solución de rehidratación
oral de la OMS, a la dosis de:
a. 30 ml/kg c. 90 ml/kg
b. 75 ml/kg d. 200 ml/kg

34
Name: ________________________ ID: A

____ 136. Un niño de 4 años de edad, con 16 kg de peso, presenta desde hace 4 días un proceso
respiratorio caracterizado por tos productiva, con esputo café amarillento y fiebre. En
las últimas 24 horas se encuentra con mayor astenia, respiración agitada y anorexia.
Toma muy pocos líquidos. T: 39,2 grado C; FC: 116 x 1; FR: 36 x 1; Sus labios están
secos y respira con la boca entreabierta. Se observa tiraje intercostal y subcostal
moderado. A la auscultación existe una notable disminución del ingreso del aire en la
base derecha. Biometría: leucocitos 18 000/mm3; Polimorfonucleares: 78%; Linfocitos
17%; Eosinófilos: 2%: Monocitos: 3%. La Rx demuestra consolidación del parénquima
pulmonar en la base derecha. Se diagnostica Neumonía. Seleccionar el germen
productor de la neumonía y el antimicrobiano más adecuado para su tratamiento:
a. Streptococcus pyogenes, que c. Streptococcus pneumonías, el
debe ser tratado con cefotaxima tratamiento Indicado es penicilina
IV IV
b. Staphylococcus aureus, debe d. Haemophilus influenzas, debe ser
tratarse con ampicilina IV tratada con penicilina benzatínica
____ 137. Ana es una mujer que tiene una hija de 4 semanas, está cansada y en la noche le da el
seno acostada, pero hace 72 horas presenta fiebre acompañado de eritema de 2 cm de
diámetro en la mama derecha, dolor de moderada intensidad y siente como un bulto de
más o menos 1 cm de diámetro en el seno derecho. ¿Cuál es la opción terapéutica
correcta?
a. Le tranquiliza a la madre, le envía c. Le prescribe dicloxacilina,
con anti-inflamatorios y le da una analgésico y le explica la posición
cita para revaloración en una correcta para dar de lactar,
semana incentivando la lactancia materna
b. Le prescribe dicloxacilina, d. Suspende inmediatamente la
analgésico y le suspende la lactancia porque la pus que sale
lactancia para que los por el seno le puede enfermar al
medicamentos no le hagan daño bebé
al bebé
____ 138. La presencia de los siguientes elementos en un niño de 4 años de edad, sugiere maltrato
infantil. EXCEPTO:
a. Estreñimiento a petición c. Niño o niña con manifiesta timidez
b. Numerosas cicatrices en la piel d. Antecedentes de fracturas
repetidas
____ 139. Matías es un niño que en la evaluación del desarrollo psicomotor en el área social juega
a la pelota, bebe de un vaso, imita tareas domésticas, se quita algunas prendas, dice 3
palabras además de mamá y papá, señala algunas partes de su cuerpo, hace
garabatos y arma una torre con dos cubos. Escoja la opción que corresponde a la edad
de Matías según los resultados de su Denver
a. 9 meses c. 6 meses
b. 15 meses d. 24 meses

35
Name: ________________________ ID: A

____ 140. RN a término, Peso al nacimiento: 2,3 kg; Talla: 49 cm; PC: 35 cm; APGAR: 7 /10; es
reactivo al manejo, su respiración es normal y presenta buen reflejo de succión. Se lo
mantiene en observación. A las 4 horas de edad presenta glucemia ole 50 mg/dl Aún
no se lo ha puesto en contacto con su madre. Determine el diagnóstico y terapéutica:
a. Retardo de crecimiento c. Retardo de crecimiento
intrauterino; lactancia materna a intrauterino; dextrosa al 10% en
libre demanda + controles de AD + controles de glucemia
glucemia
b. Retardo de crecimiento d. Control insuficiente en el
intrauterino; dextrosa al 5% en AD embarazo; dextrosa al 10% en AD
+ controles de glucemia + controles de glucemia
____ 141. A su consulta acude un paciente de 3 años de edad por presentar tos desde hace 4 días.
Al examen físico el paciente se encuentra febril (38 grados C), frecuencia cardiaca de
95 por minuto, frecuencia respiratoria de 52 por minuto y saturación de oxígeno 93%.
No se evidencia estridor y a la auscultación pulmonar no presenta disminución de
murmullo vesicular ni sibilancias. El diagnóstico del paciente es:
a. Bronquitis aguda c. Resfriado común
b. Neumonía d. Crup viral

____ 142. Un niño de 4 años de edad presenta desde hace 24 horas deposiciones diarreicas
líquidas, en moderada cantidad, sin moco ni sangre; ha realizado 6 deposiciones en 24
horas; no presenta vómito ni fiebre. En este tiempo ha comido muy poco y ha recibido
poca cantidad de aguas aromáticas. Se diagnostica deshidratación isotónica leve (1er
grado) y se decide iniciar TRO (Terapia de Rehidratación Oral). En el mercado existen
distintas soluciones comerciales para rehidratación oral.
¿Cuál de las siguientes soluciones corresponde a ia propuesta por la Organización
Mundial de la Salud (OMS) para la TRO?
a. Sodio: 60 mmol/l; Osmolaridad c. Sodio: 30 mmol/l; Osmolaridad
total: 311 mOsm/l total: 311 mOsm/l
b. Sodio: 75 mmol/l; Osmolaridad d. Sodio: 45 mmol/l; Osmolaridad
total: 245 mOsm/l total: 245 mOsm/l
____ 143. Una paciente diagnosticada de anorexia nerviosa tiene en promedio de 8-13 episodios
de conductas compensatorias inapropiadas por semana. Escoja Ha opción que
corresponde al nivel de gravedad de esta paciente
a. Leve c. Moderado
b. Grave d. Extremo

36
Name: ________________________ ID: A

____ 144. Seleccione el literal que describe la disentería en niños en menores de 5 años:
a. Vómito persistente, inicio súbito c. Dolor abdominal tipo cólico,
de deposiciones diarreicas tenesmo, fiebre alia y en
líquidas, abundantes, muy ocasiones convulsiones febriles,
frecuentes y acompañadas de diarrea muco- purulenta y
deshidratación progresiva y sanguinolenta, de cantidad
rápida, con colapso vascular variable. El paciente puede
siendo esta infección producida presentar somnolencia y es
por la bacteria Vibrio Cholerae producida por la bacteria Shigella
spp
b. Vómito, fiebre, cefalea, diarreas d. Dolor abdominal tipo cólico
líquidas, abundantes, mucosas, postprandial, luego diarreas
con mal olor, dolor abdominal y líquidas, abundantes, explosivas,
persistencia de las diarreas blanquecinas, espumosas, mal
durante 5 a 7 días. Puede haber olientes, con restos alimentarios.
somnolencia y convulsiones en Hay flatulencia intensa yes
algunos pacientes y esta infección producida por Giardia lamblia
es producida por la bacteria
Salmonella spp
____ 145. El paludismo se caracteriza por la presencia de crisis febriles que tienen una
determinada periodicidad, la misma que orienta sobre el tipo de Plasmodium infectante.
Los episodios febriles se presentan con intervalos variables y conocidos, que en la
mayoría de ellos son conocidos por su periodicidad. Seleccione el tipo de Plasmodium
que presenta estos episodios clínicos de fiebre con el mayor intervalo de tiempo:
a. P. ovale c. P. vivaz
b. P. malariae d. P. falciparum
____ 146. La radiografía de caderas de una niña de 3 meses de edad presenta signos de luxación
congénita de su cadera izquierda. Identificar el literal que describe en mejor forma las
características radiográficas de esta patología:
a. Hipoplasia del núcleo femoral c. Hipoplasia del núcleo femoral
izquierdo, ángulo acetabular izquierdo, ángulo acetabular
izquierdo de 26 grados, línea de izquierdo de 36 grados, línea de
Perkins alineada desde el Perkins desplazada hacia afuera
extremo lateral del techo del techo acetabular izquierdo
acetabular izquierdo con la epífisis
femoral
b. Presencia del núcleo femoral d. Presencia del núcleo femoral
izquierdo, ángulo acetabular izquierdo, ángulo acetabular
izquierdo de 28 grados, línea de izquierdo inferior de 27 grados,
Perkins desplazada hacia afuera línea de Perkins alineada desde el
del lecho acetabular izquierdo extremo lateral del techo
acetabular izquierdo con la epífisis
femoral

37
Name: ________________________ ID: A

____ 147. Las siguientes afirmaciones son correctas en relación a la rubéola. EXCEPTO:
a. La fiebre de la rubéola es alta, c. En adolescentes y adultos,
dura 3 días y coincide con el presenta con frecuencia
exantema generalizado poliartralgias y poliartritis
transitorias
b. Presenta una erupción máculo d. Es una enfermedad usualmente
papular eritematosa, leve, benigna, excepto para el embrión
acompañada de linfadenopatías durante el embarazo
____ 148. A su consulta es llevado un paciente masculino de 6 meses de edad por sus padres
indicando que no sienten el testículo del lado derecho del niño. Ellos indican que hace
3 meses acudieron a un médico quien Ies indico que el testículo descendería
posteriormente. El examen físico revela que el testículo derecho se encuentra en el
canal inguinal. Señale cual es el manejo en este tipo de paciente
a. Esperar hasta el año de edad c. Indicar que el hallazgo es normal
para que el testículo descienda y que en la infancia se corregirá
b. Realizar una ecografía d. Referirlo a urología pediátrica
diagnóstica
____ 149. Un lactante mayor de 24 meses de edad tiene un peso de 10kg, una talla de 70 cm y un
perímetro cefálico de 43 cm. El peso está bajo el percentil 5, la talla en el percentil 5 y
el perímetro cefálico en el percentil 25. Al examen físico presenta astenia, reducción del
tejido celular subcutáneo y de las masas musculares. Abdomen algo prominente. Piel
seca, peto escaso, descolorido, sin brillo y fácilmente desprendible. No presenta
visceromegalias. Si el percentil 50 del peso para esta edad es de 12 kg, su pérdida de
peso es de 16.6%. Seleccione el diagnóstico más probable para este niño:
a. Desnutrición calórica proteica de c. Desnutrición calórica proteica de
primer grado segundo grado
b. Desnutrición calórica proteica de d. Desnutrición proteica calórica o
tercer grado kwashiorkor
____ 150. Un escotar de 4 años de edad, eufrófico, acude a consulta porque desde hace 10 días
presenta tos intensa, inicialmente y luego de un resfriado, era una tos seca.
Progresivamente se ha transformado en una tos productiva, con esputo claro y espeso.
Ha tomado varios medicamentos para eliminar ¡a flema, sin embargo, la tos persiste y
le impide dormir normalmente. Al examen físico se encuentra afebril, con buen estado
general. Tose en forma repetitiva y a la auscultación presenta estertores roncantes
difusos, especialmente en la espiración. La Rx de tórax es normal, con esta
información. Seleccione el diagnóstico:
a. Adenoiditis c. Bronquitis aguda
b. Laringitis aguda d. Bronquiolitis

____ 151. El tratamiento de una criptorquidia verdadera en un lactante de 9 meses de edad es:
a. Inducción hormonal para c. Orquidopexia
descenso testicular
b. Tratamiento expectante hasta el d. Tratamiento expectante hasta los
año de edad 3 años de edad

38
Name: ________________________ ID: A

____ 152. En relación a neurocisíicercosis, las siguientes afirmaciones son correctas. EXCEPTO:
a. La neurocisticercosis se trata con c. Se produce por ingestión de
antiparasitarios como huevos y proglótides a través de
metronidazol o tinidazol sumado a alimentos contaminados con
AINES para manejo del dolor cisticercos
b. Los cisticercos y sus quistes, d. Los cisticercos, después de morir,
corresponden a la fase larvaria de pueden producir sintomatología
las tenias del cerdo, como es la inflamatoria significativa y
Taenia solium convulsiones
____ 153. Un escolar de 10 años presenta desde hace 3 días diarreas líquidas, abundantes, fétidas,
con moco y un poco de sangre, con una frecuencia de 3 a 4 veces al día. Se
acompañan de dolor abdominal tipo cólico, de moderada intensidad. Se realiza
coproparasitario que reporta la presencia de Balatidium coli. Seleccione el fármaco
correcto para tratar esta infestación:
a. Albendazol c. Cotrimoxazol
b. Mebendazol d. Metronidazol

____ 154. Un lactante de 13 meses de edad. Peso: 8 kg; Talla: 73 cm; PC: 44 cm. Acude a
emergencia por haber presentado desde hace 4 horas, dos crisis convulsivas
generalizadas, una que duró 15 minutos y la otra 20 minutos, aproximadamente. Luego
de las convulsiones permanece aletargado y aparentemente dormido. Su madre refiere
que desde hace 48 horas presenta fiebre, cada vez más alta, astenia, anorexia y siente
algo de rigidez en sus músculos. Se encuentra adormecido, muy poco reactivo a los
estímulos, signos de Brudzinski y Kernig positivo. Babinski bilateral, positivo. Una tía
del lactante tiene epilepsia. Con estos datos. Seleccione el diagnóstico más probable:
a. Convulsiones febriles c. Convulsiones complejas por fiebre
b. Meningitis d. Crisis convulsiva de origen
epiléptico
____ 155. María es una niña que al examen para evaluar el desarrollo psicomoior demuestra que
puede comer galletas sola, imita palabras y dice de forma inespecífica papa, mama,
pasa un cubo de una mano a otra, está sentada con apoyo y es tímida con extraños.
Escoja la opción que corresponde a la edad aproximada de la niña.
a. 6 meses c. 12 meses
b. 9 meses d. 3 meses

____ 156. Una mujer en periodo de lactancia acudió a su consulta porque presentó
faringoamigdalitis estreptocócica, su bebé tiene 4 meses. Escoja la opción que
recomendaría a la madre en relación a la lactancia materna.
a. Explica que reciba el antibiótico y c. Prescribe un antibiótico y le
por ese periodo el bebé recibe explica que su bebé puede iniciar
sucedáneos de la leche materna alimentación complementaria
b. Suspende la lactancia materna de d. Usted explica: Recibir el
forma permanente, porque las antibiótico específico y continuar
penicilinas le van a afectar al con la lactancia materna
bebé normalmente
____ 157. La lactancia materna está contraindicada en las siguientes situaciones. EXCEPTO:
a. VIH y carga viral alta c. Mastitis Aguda
b. Fenilcetonuria d. Galactosemia

39
Name: ________________________ ID: A

____ 158. La administración de zinc a un niño con diarrea aguda, según las recomendaciones de
la OMS, tiene las siguientes normas y ventajas. EXCEPTO:
a. Restituye parte de los electrolitos c. Deberá realizarse desde el
perdidos durante la diarrea comienzo de una diarrea aguda
b. Disminuye el riesgo de diarrea, en d. Reduce la duración, la gravedad y
los 2 o 3 meses siguientes el riesgo de deshidratación
____ 159. El hallazgo clínico más habitual en los casos de neumonía infantil es:
a. La presencia de estridor c. La presencia de auscultación
pulmonar anormal
b. La presencia de taquipnea d. La presencia de tos
____ 160. Un recién nacido, producto de la primera gesta de una madre de 23 años que tuvo una
labor de parto de 12 horas, ruptura de membranas intraparto, líquido amniótico claro y
grumoso. El RN. nació de parto normal en un servicio hospitalario. Al nacer presentó un
Peso de: 2.9 kg; Talla: 49 cm; PC: 34.5 cm; APGAR: 7/9; A las 24 horas de edad
presenta ictericia en cara y tronco. La bilirrubina indirecta es de 7 mg/dl. La bilirrubina
directa es de 0.9 mg/dl. Se encuentra activo, tiene buena reactividad al manejo. Toma
seno materno exclusivo, con succión adecuada, presentó regurgitación por dos
ocasiones. El grupo sanguíneo de su madre es A+; el RN es 0+ y Coombs directo es
negativo. Seleccione el diagnóstico, así como la decisión terapéutica correcta para su
ictericia:
a. Septicemia; hemocultivo y otros c. Ictericia por incompatibilidad ABO;
exámenes relativos + fototerapia y realizar control de
antibioticoterapia bilirrubinas
b. Hepatitis neonatal; solicitar d. Ictericia fisiológica; observar
estudios correlativos y vigilar evolución y realizar control de
bilirrubinas bilirrubinas
____ 161. Según las curvas de crecimiento aceptadas en el Ecuador, para un varón de 7 años de
edad, los percentiles 10, 50 y 95, para su antropometría, son los siguientes: Peso: 16,
22 y 27 kg; Talla: 108,119 y 128 cm; IMC: 13.7; 15 y 18 kg/m2; Un niño de 7 años de
edad presenta los siguientes parámetros antropométricos: Peso: 30 kg; Talla: 118 cm;
IMC: 21.5 kg/m2. De acuerdo con estos datos, seleccione el diagnóstico nutricional:
a. Obesidad grado I c. Obesidad mórbida
b. Obesidad Grado II d. Sobrepeso

____ 162. Paciente de 8 años, que pesa 24 kg, con estatus convulsivo, señale el fármaco y dosis
correcta de anticonvulsivante que debe ser suministrado inmediatamente
a. Diazepam vía intramuscular 2 mg c. Midazolam 5 mg intramuscular
STAT STAT
b. Levetiracetam 4 mg vía oral STAT d. Fenobarbital 480 mg intravenoso
STAT

40
Name: ________________________ ID: A

____ 163. Un niño de 6 años, que vive en una región tropical del país, acude a consulta porque en
los últimos 10 días presenta tos y esputo sanguinolento. Además, dolor abdominal
ocasional, así como deposiciones diarreicas abundantes y con moco, que se producen
por temporadas, desde hace un año. Ocasionalmente presenta también vómitos. En
este año, por tres ocasiones presentó lesiones irregulares, longilíneas, a manera de
trayectos indurados, eritematosos y pruriginosos en tórax, abdomen, glúteos y
miembros inferiores. Estas lesiones desaparecieron en uno o dos días. Su peso está en
el percentil 5 para la edad. Su talla en el percentil 10. La biometría hemática reporta
únicamente una marcada eosinofilia, con 600 eosinófilos/ml de sangre. Se realiza un
coproparasitario que reporta la presencia de larvas parasitarias. Seleccionar el literal
que contenga el parásito más probable de acuerdo con el cuadro clínico de este niño, y
el medicamento con el que debería realizarse el tratamiento:
a. Enterobius vermicularis, c. Strongyloides stercoralis;
mebendazol, 100 mg dos veces ivermectina, una tableta de 3 mg
diarias, por 7 días en ayunas, una vez
b. Ascaris lumbrícoides; albendazol, d. Giardia lamblia; metronidazol, 250
200mg en una sola toma, por una mg, 1 c/día, luego de una comida,
sola vez 10 días
____ 164. El sarampión se caracteriza por lo siguiente:
a. Pródromo durante 3 días con tos, c. Se caracteriza por cursar con
coriza, conjuntivitis y manchas de hepato-esplenomegalia
Koplik
b. En la fase exantemática no hay d. El exantema inicia en abdomen,
fiebre se distribuye a la periferia, y se
desvanece en igual forma
____ 165. Un escolar de 8 años, pesa 21 kg y vive en el subtrópico. Desde hace 1 semana presenta
diarreas que contienen moco y estrías sanguinolentas. La cantidad es muy variable y
se repiten de 4 a 5 veces al día. Se acompañan de dolor abdominal moderado. Hace un
año presentó una sintomatología similar, aunque en aquella ocasión tuvo prolapso
rectal. El coproparasitario reporta presencia de huevos de Trichuris trichiura.
Seleccione el tratamiento adecuado:
a. Albendazol, 400 mg diarios por 1 c. Albendazol, 400 mg diarios por 2
día días
b. Mebendazol, 100 mg diarios por 3 d. Albendazol, 400 mg diarios por 3
días días
____ 166. Elija la reacción adversa asociada a la vacunación por Sarampión, Rubéola y Paperas
(SRP):
a. Ectima c. Estomatitis
b. Escrofuloderma d. Erupción cutánea

41
Name: ________________________ ID: A

____ 167. Un niño de 3 y medio años de edad, cuyo crecimiento y desarrollo psicomotor son
normales, tiene frecuentemente problemas con sus padres porque no desea comer.
Seleccionar la acción adecuada frente a este problema:
a. Tranquilizar y guiar a los padres c. Usar premios y juegos para
distraerlo mientras se le ayuda a
comer
b. Darle estimulantes del apetito d. Ofrecerle alimentos
para que pueda comer mejor constantemente, entre comidas,
para que pique
____ 168. Una niña de 3 años, pesa 15 kg. Desde hace 12 horas presenta una erupción de
aparición abdominal, con vesículas y pápulas eritematosas, con intenso prurito, que se
han extendido hacia el tórax. No ha tenido fiebre. Se mantiene activa y come bien. T°:
37,5 °C; FC: 70 x 1'; FR: 24 x 1'; Al examen físico las máculas, pápulas y vesículas se
han extendido al tronco anterior y posterior. Seleccione el diagnóstico y tratamiento
correctos:
a. Escarlatina, administrar penicilina c. Varicela, administrar aciclovir
benzatínica 600 000 Ul IM por una suspensión de 200 mg / 5 ml, 7.5
sola vez mi cada 6 horas durante 5 días
b. Roséola o exantema súbito, d. Rubéola, administrar paracetamol
paracetamol en gotas de 100 mg / en jarabe de 150 mg / 5 ml, 4 ml
mi, 2 ml cada 6 horas, 3 días cada 6 horas, 3 días
____ 169. Seleccione los criterios que corresponden a un caso de dengue sin signos de alarma:
a. IgM positiva.
b. Exantema.
c. Sangrado de mucosas.
d. Leucopenia.
e. Elevación de AST mayor o igual a 1 000.
f. Prueba de torniquete positivo.

a. a, d, f c. b, d, f
b. c, d, f d. b, d, e
____ 170. Paciente femenina de 5 años, con disuria, polaquiuria y dolor suprapúbico. Presenta
también hematuria visible en ropa interior. ¿Cuál es el diagnóstico clínico?
a. Pieionefritis c. Cólico renal
b. Cistitis d. Bacteriuria asintomática

____ 171. Paciente de 4 años, quien está consciente, afebril, presenta estridor inspiratorio leve y
tos seca "perruna". Indique el diagnóstico correcto:
a. Crup moderado c. Bronquiolitis
b. Neumonía d. Amigdalitis aguda

____ 172. Seleccione las medidas generales que debe recibir un paciente menor de 5 años con
neumonía adquirida en la comunidad:
a. Dar líquidos, fisioterapia c. Dar mucolíticos, antitusígenos,
respiratoria, tratar la fiebre tratar la fiebre
b. Tratar la fiebre, dar líquidos, no d. Dar antitusígenos, tratar la fiebre,
forzar la alimentación sólida no forzar la alimentación sólida

42
Name: ________________________ ID: A

____ 173. Camila tiene 2 años 2 meses, usted realiza el test de Denver y evidencia que dice su
nombre y apellidos, arma torre de 6 cubos, se puede poner y sacar la chompa, salta y
se separa de la madre con facilidad. Escoja el literal correcto relacionado a la
consejería que dará a los padres de Camila
a. Comunica a los padres que c. Elogia a los padres y les
Camila puede tener un retraso del comunica que Camila tiene un
desarrollo pero por falta de desarrollo psicomotor adecuado y
estimulación por lo que la refiere brinda información sobre más
al servicio de estimulación actividades de estimulación
b. Alerta a los padres porque Camila d. Sugiere a los padres que Camila
tiene un evidente retraso del sea evaluada por la psicóloga
desarrollo psicomotor por lo que para un examen completo, ya que
la refiere al servicio de en el primer nivel no se aplica el
rehabilitación física test de Denver II
____ 174. RN de 30 semanas de edad gestacional. Membranas íntegras. Nace por cesárea. Líquido
amniótico claro, sin grumos. APGAR 6-8. Al nacer presenta taquipnea, quejido, cianosis
y retracciones costales y subcostales. Se le suministra oxígeno y una hora después
presenta pausas de apnea. Seleccione el diagnóstico correcto:
a. Enfermedad de membrana hialina c. Neumonía neonatal
b. Síndrome de aspiración de d. Taquipnea transitoria del recién
meconio nacido
____ 175. Un lactante de 1 año que pesa 9 kg y vive en Quito acude con su madre para control de
niño sano. Se encuentra en buen estado general. Trae consigo los resultados de una
biometría: Leucocitos 8 500 /mm3; Eritrocitos: 3 980 000 / mm3; Hematocrito (Hcto):
30%; Hemoglobina (Hb): 10 g/dl; Volumen Corpuscular Medio (VCM): 71 fL;
Concentración de Hemoglobina Corpuscular Media (CHCM): 23%; Morfología
eritrocitaria: hipocromía. Los exámenes de orina y heces son normales. Seleccionar el
diagnóstico y el tratamiento apropiado para este paciente:
a. Anemia normocítica c. Anemia microcítica hipocrómica;
normocrómica; Hierro elemental hierro elemental, 4-6 mg/kg/día,
de 1 - 2 mg/kg/día, dos días a la dividido en 2 o 3 tomas, durante
semana para prevenir una anemia un mes, inicialmente
grave
b. Anemia microcítica hipocrómica; d. Anemia macrocítica hipocrómica;
Sulfato ferroso de 4 - 6 mg/kg/día, Ácido fólico 1 mg, diario durante 1
dividido en 2 o 3 dosis al día, mes, luego evaluación para
inicialmente durante un mes verificar la respuesta al
tratamiento
____ 176. Paciente masculino de 5 años, en consulta se detecta ascenso testicular indoloro. Se
logra bajar el testículo al escroto, sin embargo, el cordón espermático se encuentra a
tensión. Indique el diagnóstico:
a. Criptorquidia c. Hernia inguinal
b. Testículo retráctil d. Torsión testicular

43
Name: ________________________ ID: A

____ 177. Escoja la recomendación correcta en relación a la alimentación de un lactante menor a


6 meses de edad con leche materna según el MSP, UNICEF y OMS
a. Establecer un horario cada 3 c. A libre demanda
horas
b. Lactar 15 minutos de cada seno d. La frecuencia es de 6 a 8 veces al
día
____ 178. A su consulta general acude un paciente de 2 meses, sexo masculino es llevado para
vacunación, el lactante ha sido vacunado con la BCG y con Hepatitis B pediátrica al
nacimiento. Indique el esquema que corresponde a la edad actual del lactante:
a. Vacuna antipoliomielítica, c. Vacuna antipoliomielítica,
Pentavalente, Rotavirus, Pentavalente, Rotavirus, Influenza
Neumococo pediátrica
b. Vacuna antipoliomielítica, d. Vacuna antipoliomielítica,
Pentavalente, Fiebre Amarilla, Pentavalente, Influenza,
Neumococo pediátrica Neumococo pediátrica
____ 179. Las siguientes son las reacciones adversas más comunes asociadas a la vacunación por
rotavirus:
a. Convulsiones, alteración del c. Síncope, hipotensión ortostática
estado de conciencia
b. Irritabilidad, diarrea, vómito, d. Ictericia, elevación de enzimas
flatulencia y dolor abdominal hepáticas y acolia
____ 180. RN de 36.3 semanas por FUM, nació por cesárea sin labor, con membranas íntegras.
Líquido amniótico claro con grumos. Crecimiento adecuado para su edad gestacional.
Al nacimiento presenta taquipnea, quejido, cianosis, retracciones costales y
subcostales. Se le administra oxígeno y una hora después presenta mejoría.
Seleccione el diagnóstico correcto:
a. Enfermedad de membrana hialina c. Taquipnea transitoria del recién
nacido
b. Síndrome de aspiración de d. Neumonía neonatal
meconio
____ 181. Miriam dio a luz hace 3 horas por cesárea, usted le recomienda que alimente al recién
nacido con leche materna, pero la madre le dice que no tiene leche. Escoja la opción
correcta en relación a la lactancia post cesárea
a. Le tranquiliza y le dice que ahora c. Le pide a Miriam que mezcle
tiene calostro, verificando la leche infantilizada con leche del
posición correcta para dar de banco y espera a que la
lactar producción propia aumente
b. Le solicita acudir a un banco de d. Le da leche de fórmula para
leche humana para que el niño complementar hasta que Miriam
reciba leche materna produzca más leche materna

44
Name: ________________________ ID: A

____ 182. Sobre las parasitosis en pediatría, seleccione los enunciados correctos:
1. Entamoeba histolytica - Dolor abomina!, diarrea, tenesmo - metronidazol.
2. Giardia lambía - dolor pélvico, meteorismo y evacuaciones liquidas claras -
nitaxoznida.
3. Tenia soliun - obstrucción intestina! - citrato de piperazina.
4. Trichuris trichiura - prolapso rectal, anemia y retardo del crecimiento - Albendazol.
5. Enterobius vermicularis - prurito anal nocturno, bruxismo y vulvovaginitis -
Albendazol.
6. Estrongyloides stercoralis - esteatorrea y tos - ivermectina.

a. 1, 2, 6 c. 2, 3, 5
b. 1, 2, 5 d. 1, 4, 5
____ 183. Una niña de 5 años presenta dolor intenso de oído derecho de 24 horas de evolución. La
niña se encuentra febril. Al examen otoscópico neumático del oído derecho se
evidencia un eritema intenso de la membrana timpánica con abombamiento moderado
de la misma; además se evidencia líquido y niveles hidro-aéreos detrás de la
membrana timpánica. El diagnóstico correcto de la condición de la paciente es:
a. Otitis externa complicada c. Otitis media con derrame (OMD)
b. Otitis media aguda (OMA) con d. Otitis media aguda (OMA)
otorrea
____ 184. Señale los signos y síntomas que pertenezcan a la enfermedad por reflujo
gastroesofágico en pacientes pediátricos:
1. Disfagia.
2. Artralgia.
3. Regurgitación recurrente con o sin vomito.
4. Perdida o escasa ganancia de peso.

a. 1, 3, 4 c. 2, 3, 4
b. 1, 2, 3 d. 1, 2, 4
____ 185. Paciente de 7 años procedente de Machala. Desde hace 10 días presenta fiebre cada 72
horas, cuantificada en 40° C. Se acompaña de sudoración, cefalea, diarrea y palidez
cutáneo mucosa. Estos episodios alternan con periodos de buen estado general.
Indique el diagnóstico más probable:
a. Malaria c. Dengue
b. Fiebre tifoidea d. Fiebre amarilla

____ 186. Seleccione el enunciado correcto para un paciente pediátrico con fiebre:
a. La temperatura se mide c. Un pico febril aislado se asocia a
inmediatamente después de infección bacteriana
retirar el exceso de ropa
b. La fiebre mayor a 41 0 C es d. La evaluación inicial se centra en
producto de bacterias muy la búsqueda de una causa
agresivas infecciosa

45
Name: ________________________ ID: A

____ 187. Una adolescente de 16 años presenta algunas manifestaciones con relación a su
alimentación. Se diagnostica bulimia nerviosa. ¿Cuál de los siguientes enunciados es
correcto con relación a este diagnóstico?
a. Se presenta con mayor frecuencia c. Episodios de atracón en los que
en hombres, en una relación de se ingiere un cantidad de
10:1 alimentos superior a lo que la
mayoría de personas ingiere en
un periodo similar
b. Se produce exclusivamente como d. Se acompaña de
síntoma acompañante de comportamientos compensatorios
anorexia nerviosa para evitar subir de peso al menos
tres veces al día
____ 188. Con respecto a la urticaria en pacientes pediátricos es correcto que:
a. El habón característico es una c. En la urticaria crónica las lesiones
roncha elevada, pruriginosa y duran más de dos días a la
eritematosa que palidece a la semana durante 3 semanas
presión
b. La urticaria puede ser causada d. En la urticaria aguda las lesiones
por varios factores, siendo lo más suelen durar más de 24 horas,
común los estímulos físicos con tendencia a confluir
____ 189. Paciente masculino de un año acude a la consulta porque sus padres indican que en
ocasiones no sienten el testículo del lado derecho, sobre todo cuando lo bañan. El
examen físico revela que el testículo derecho se encuentra un poco elevado, pero con
facilidad es llevado a la bolsa escrotal y permanece en ella. El manejo apropiado del
paciente es:
a. Inducción hormonal para c. Referirlo a urología pediátrica
descenso testicular
b. Evaluar cada año la evolución del d. Realizar una ecografía
cuadro diagnóstica
____ 190. Seleccione lo correcto en relación a la anemia ferropriva o ferropénica
a. En los hematíes se observa c. Los hematíes son normocíticos y
microcitosis y el recuento de el recuento de reticulocitos es alto
reticulocitos es alto
b. En los hematíes se observa d. Los hematíes son macrocíticos y
microcitosis y el recuento de el recuento de reticulocitos es
reticulocitos es bajo bajo
____ 191. Un preescolar presenta una diarrea aguda viral. Se decide iniciar terapia de rehidratación
oral. Se dispone de soluciones de distintas concentraciones en sodio. Seleccione la
solución más adecuada de acuerdo con las normas de la QMS:
a. Solución de rehidratación oral, c. Sobres de sales para
disponible en el mercado, con 30 rehidratación oral, preparar para
mEq/l obtener 75 mEq/l
b. Sobres de sales para d. Solución de rehidratación oral,
rehidratación oral, preparar para disponible en el mercado, con 45
obtener 90 mEq/l mEq/l

46
Name: ________________________ ID: A

____ 192. A su consulta general acude un paciente de 15 meses de edad masculino con su
madre para vacunación, el paciente ha cumplido con el esquema de vacunación
vigente. ¿Cuál vacuna es la siguiente a administrarse según el Ministerio de Salud
Pública del Ecuador?
a. Vacuna de poliomielitis (OPV) c. Vacuna de difteria y tétano (Td)
b. Vacuna contra virus del papiloma d. Vacuna contra varicela
humano (HPV)
____ 193. En relación al dengue con signos de alarma en niños, seleccione la afirmación correcta:
a. El dolor abdominal en el dengue c. La hepatomegalia debe
se debe a hepatomegalia o considerarse como un factor de
erosiones de la mucosa gástrica riesgo para choque por dengue en
niños
b. La alteración del estado de d. La ictericia debe considerarse
conciencia se explica por la como un factor de riesgo para
significativa alza térmica que choque en niños con dengue
ocurre en el dengue
____ 194. Acude una paciente de 9 años de edad en compañía de su madre en busca de continuar
con su esquema de vacunación, a sabiendas que la paciente ha sido inmunizada
correctamente hasta la fecha. ¿Cuál de las siguientes vacunas está indicada en esta
niña según el Esquema Nacional de Vacunación del Ministerio de Salud Pública del
Ecuador?
a. Vacuna de fiebre amarilla (FA) c. Vacuna contra virus del papiloma
humano (HPV)
b. Vacuna de varicela d. Vacuna de difteria y tétano (Td)

47
ID: A

Pediatría
Answer Section

MULTIPLE CHOICE

1. ANS: B PTS: 1
2. ANS: B PTS: 1
3. ANS: B PTS: 1
4. ANS: D PTS: 1
5. ANS: C PTS: 1
6. ANS: B PTS: 1
7. ANS: C PTS: 1
8. ANS: B PTS: 1
9. ANS: C PTS: 1
10. ANS: C PTS: 1
11. ANS: A PTS: 1
12. ANS: C PTS: 1
13. ANS: C PTS: 1
14. ANS: A PTS: 1
15. ANS: B PTS: 1
16. ANS: D PTS: 1
17. ANS: A PTS: 1
18. ANS: D PTS: 1
19. ANS: C PTS: 1
20. ANS: B PTS: 1
21. ANS: A PTS: 1
22. ANS: B PTS: 1
23. ANS: A PTS: 1
24. ANS: A PTS: 1
25. ANS: B PTS: 1
26. ANS: B PTS: 1
27. ANS: B PTS: 1
28. ANS: B PTS: 1
29. ANS: D PTS: 1
30. ANS: A PTS: 1
31. ANS: C PTS: 1
32. ANS: C PTS: 1
33. ANS: A PTS: 1
34. ANS: D PTS: 1
35. ANS: D PTS: 1
36. ANS: D PTS: 1
37. ANS: D PTS: 1
38. ANS: A PTS: 1
39. ANS: C PTS: 1
40. ANS: D PTS: 1
41. ANS: A PTS: 1
42. ANS: C PTS: 1
43. ANS: A PTS: 1

1
ID: A

44. ANS: D PTS: 1


45. ANS: D PTS: 1
46. ANS: C PTS: 1
47. ANS: D PTS: 1
48. ANS: A PTS: 1
49. ANS: D PTS: 1
50. ANS: C PTS: 1
51. ANS: C PTS: 1
52. ANS: B PTS: 1
53. ANS: D PTS: 1
54. ANS: C PTS: 1
55. ANS: D PTS: 1
56. ANS: A PTS: 1
57. ANS: D PTS: 1
58. ANS: B PTS: 1
59. ANS: B PTS: 1
60. ANS: D PTS: 1
61. ANS: D PTS: 1
62. ANS: C PTS: 1
63. ANS: C PTS: 1
64. ANS: A PTS: 1
65. ANS: D PTS: 1
66. ANS: B PTS: 1
67. ANS: A PTS: 1
68. ANS: C PTS: 1
69. ANS: A PTS: 1
70. ANS: B PTS: 1
71. ANS: D PTS: 1
72. ANS: A PTS: 1
73. ANS: C PTS: 1
74. ANS: C PTS: 1
75. ANS: C PTS: 1
76. ANS: A PTS: 1
77. ANS: A PTS: 1
78. ANS: A PTS: 1
79. ANS: C PTS: 1
80. ANS: D PTS: 1
81. ANS: B PTS: 1
82. ANS: C PTS: 1
83. ANS: A PTS: 1
84. ANS: A PTS: 1
85. ANS: D PTS: 1
86. ANS: A PTS: 1
87. ANS: A PTS: 1
88. ANS: D PTS: 1
89. ANS: C PTS: 1
90. ANS: A PTS: 1
91. ANS: A PTS: 1

2
ID: A

92. ANS: D PTS: 1


93. ANS: C PTS: 1
94. ANS: D PTS: 1
95. ANS: D PTS: 1
96. ANS: A PTS: 1
97. ANS: C PTS: 1
98. ANS: D PTS: 1
99. ANS: C PTS: 1
100. ANS: B PTS: 1
101. ANS: C PTS: 1
102. ANS: B PTS: 1
103. ANS: D PTS: 1
104. ANS: B PTS: 1
105. ANS: B PTS: 1
106. ANS: A PTS: 1
107. ANS: D PTS: 1
108. ANS: D PTS: 1
109. ANS: A PTS: 1
110. ANS: D PTS: 1
111. ANS: C PTS: 1
112. ANS: B PTS: 1
113. ANS: B PTS: 1
114. ANS: C PTS: 1
115. ANS: C PTS: 1
116. ANS: C PTS: 1
117. ANS: B PTS: 1
118. ANS: A PTS: 1
119. ANS: D PTS: 1
120. ANS: B PTS: 1
121. ANS: A PTS: 1
122. ANS: A PTS: 1
123. ANS: D PTS: 1
124. ANS: C PTS: 1
125. ANS: B PTS: 1
126. ANS: B PTS: 1
127. ANS: C PTS: 1
128. ANS: A PTS: 1
129. ANS: D PTS: 1
130. ANS: D PTS: 1
131. ANS: C PTS: 1
132. ANS: B PTS: 1
133. ANS: C PTS: 1
134. ANS: D PTS: 1
135. ANS: B PTS: 1
136. ANS: C PTS: 1
137. ANS: C PTS: 1
138. ANS: A PTS: 1
139. ANS: B PTS: 1

3
ID: A

140. ANS: A PTS: 1


141. ANS: B PTS: 1
142. ANS: B PTS: 1
143. ANS: B PTS: 1
144. ANS: C PTS: 1
145. ANS: B PTS: 1
146. ANS: C PTS: 1
147. ANS: A PTS: 1
148. ANS: D PTS: 1
149. ANS: A PTS: 1
150. ANS: C PTS: 1
151. ANS: C PTS: 1
152. ANS: A PTS: 1
153. ANS: D PTS: 1
154. ANS: B PTS: 1
155. ANS: A PTS: 1
156. ANS: D PTS: 1
157. ANS: C PTS: 1
158. ANS: A PTS: 1
159. ANS: B PTS: 1
160. ANS: D PTS: 1
161. ANS: A PTS: 1
162. ANS: C PTS: 1
163. ANS: C PTS: 1
164. ANS: A PTS: 1
165. ANS: D PTS: 1
166. ANS: D PTS: 1
167. ANS: A PTS: 1
168. ANS: C PTS: 1
169. ANS: C PTS: 1
170. ANS: B PTS: 1
171. ANS: A PTS: 1
172. ANS: B PTS: 1
173. ANS: C PTS: 1
174. ANS: A PTS: 1
175. ANS: C PTS: 1
176. ANS: A PTS: 1
177. ANS: C PTS: 1
178. ANS: A PTS: 1
179. ANS: B PTS: 1
180. ANS: C PTS: 1
181. ANS: A PTS: 1
182. ANS: D PTS: 1
183. ANS: D PTS: 1
184. ANS: A PTS: 1
185. ANS: A PTS: 1
186. ANS: D PTS: 1
187. ANS: C PTS: 1

4
ID: A

188. ANS: A PTS: 1


189. ANS: B PTS: 1
190. ANS: B PTS: 1
191. ANS: C PTS: 1
192. ANS: D PTS: 1
193. ANS: C PTS: 1
194. ANS: C PTS: 1

5
Pediatría [Answer Strip] ID: A

C
_____ 5. A 11.
_____ D 16.
_____ B 22.
_____

B
_____ 1. C 12.
_____

A 17.
_____

B
_____ 6.

A 23.
_____
B
_____ 2.

C 13.
_____ D 18.
_____

C
_____ 7.

C 19.
_____

B
_____ 3.
A 14.
_____ A 24.
_____
B
_____ 8.

B 20.
_____
C
_____ 9.

B 15.
_____
D
_____ 4.
B 25.
_____

A 21.
_____

C 10.
_____
Pediatría [Answer Strip] ID: A

B 26.
_____ C 31.
_____ D 36.
_____ D 40.
_____ D 44.
_____

C 32.
_____
B 27.
_____

D 37.
_____
D 45.
_____
A 41.
_____

C 42.
_____ C 46.
_____

B 28.
_____
A 33.
_____
A 38.
_____

D 29.
_____ A 43.
_____
D 34.
_____

C 39.
_____ D 47.
_____
D 35.
_____

A 30.
_____
Pediatría [Answer Strip] ID: A

A 48.
_____ C 51.
_____ A 56.
_____ C 62.
_____ A 67.
_____

D 57.
_____
C 63.
_____

B 52.
_____

D 49.
_____

B 58.
_____ A 64.
_____ C 68.
_____

D 53.
_____

B 59.
_____
D 65.
_____

C 50.
_____ A 69.
_____

D 60.
_____
C 54.
_____

B 66.
_____

D 61.
_____

D 55.
_____
Pediatría [Answer Strip] ID: A

B 70.
_____ C 74.
_____ A 78.
_____ B 81.
_____ D 85.
_____

C 75.
_____

C 79.
_____

D 71.
_____

A 86.
_____

A 76.
_____

A 72.
_____ D 80.
_____ C 82.
_____

A 83.
_____

A 84.
_____

C 73.
_____
A 77.
_____
Pediatría [Answer Strip] ID: A

A 87.
_____ A 90.
_____ D 94.
_____ D 98.
_____ C
_____101.

C 99.
_____

A 91.
_____

B
_____102.

D 88.
_____
D 95.
_____

B
_____100.

A 96.
_____

D 92.
_____

D
_____103.

C 89.
_____

C 97.
_____

C 93.
_____
Pediatría [Answer Strip] ID: A

B
_____104. A
_____106. D
_____110. B
_____113. C
_____116.

C
_____114.

C
_____111.

B
_____117.

D
_____107.
B
_____105.

C
_____115.
B
_____112.

D
_____108.

A
_____109.
Pediatría [Answer Strip] ID: A

A
_____118. A
_____122. B
_____126. C
_____131. C
_____136.

C
_____127.

D
_____123. B
_____132.
D
_____119.

C
_____137.

A
_____128.

C
_____124.

D
_____129. C
_____133.

B
_____120.

D
_____134. A
_____138.
A
_____121. B
_____125.

D
_____130.
B
_____135.
B
_____139.
Pediatría [Answer Strip] ID: A

A
_____140. C
_____144. A
_____147. A
_____152. A
_____158.

B
_____159.

D
_____148.
D
_____153.

B
_____141. D
_____160.

B
_____154.
A
_____149.
B
_____142. B
_____145.

C
_____146.
A
_____161.
A
_____155.
C
_____150.

B
_____143.

D
_____156. C
_____162.

C
_____151.

C
_____157.
Pediatría [Answer Strip] ID: A

C
_____163. A
_____167. C
_____173. C
_____177. D
_____182.

A
_____178.

C
_____168.

D
_____183.

A
_____174.

B
_____179.

A
_____164.

A
_____184.
C
_____169. C
_____175.
C
_____180.

D
_____165.

A
_____185.

B
_____170. A
_____181.

D
_____166.
A
_____171. D
_____186.

A
_____176.

B
_____172.
Pediatría [Answer Strip] ID: A

C
_____187. D
_____192.

C
_____193.

A
_____188.

C
_____194.

B
_____189.

B
_____190.

C
_____191.
Name: ________________________ Class: ___________________ Date: __________ ID: A

Endocrinología

Multiple Choice
Identify the choice that best completes the statement or answers the question.

____ 1.
Paciente femenina de 38 años de edad, con antecedente de hiperprolactinemia, es
referida a consulta externa de endocrinología por cambios en sus rasgos faciales,
caracterizados por prognatismo, abombamiento frontal y nariz grande, además de
aumento de tamaño de los pies y manos, tras realizarse todos los estudios pertinentes,
se indicó estudio histopatológico que informó adenomas productor de hormona de
crecimiento. Debe plantearse que podría haber otras manifestaciones clínicas, como:
a. Insomnio, irritabilidad, disminución c. Obesidad central, miopatía central,
de peso, taquicardia. estrías amplias y purpúreas.

b. Cardiomegalia, macroglosia y d. Palpitaciones, cefaleas e


agrandamiento de la glándula hiperhidrosis.
tiroides

____ 2.
Paciente masculino de 42 años de edad, acude por bocio, refiere dificultad para
concentrarse en el trabajo, aumento de peso no relacionado con mayor ingesta, ya que
además presentó pérdida de apetito, en la última semana se asocia párpados
edematosos y edema pretibial sin fóvea. De acuerdo a su sospecha diagnóstica se
puede afirmar que:
a. Se puede esperar TSH no c. Una concentración normal de TSH
suprimida y el hallazgo de un excluye el hipotiroidismo
tumor hipofisiario en la primario.
resonancia magnética

b. Existen otras manifestaciones d. En el diagnóstico diferencial hay


como proptosis y aumento de que pensar otros trastornos
tamaño de los músculos como ataques de ansiedad,
extraoculares. consumo de cocaína y
anfetaminas.

1
Name: ________________________ ID: A

____ 3.
Paciente femenina de 52 años de edad, con sobrepeso, acude por presentar poliuria,
polidipsia y pérdida de peso de 6 meses de evolución, en la última semana se asocia
debilidad y visión borrosa, su sospecha diagnostica es diabetes mellitus tipo 2, por lo
que podría esperarse los siguientes resultados de laboratorio:
a. Glicemia en ayunas mayor a c. Glucosa plasmática en ayunas
140mg/dl, con hemoglobina mayor a 126mg/dl con
glicosilada baja. hemoglobina glicosilada
elevada.

b. Glucosa plasmática dos horas d. Intolerancia a la glucosa con


posterior a sobrecarga de glicemias entre 140 a 199mg/dl
glucosa mayor a 180 mg/dl posterior a la prueba de
tolerancia a la glucosa.

____ 4.
Paciente femenina de 46 años de edad, es referida por obesidad, refiere haber seguido
diferentes dietas, además de ejercicios de moderada intensidad en los últimos 6
meses, sin lograr disminución de peso, al examen físico se constata presión arterial
146/92mmHg, piel delgada, rubicundez facial, cara redonda, giba de búfalo, estrías
violáceas, se solicita medir ACTH y se encuentra disminuida; el diagnóstico probable
es:
a. Hiperplasia suprarrenal c. Enfermedad de Cushing
congénita.

b. Feocromocitoma d. Síndrome de Cushing


independiente de ACTH.

____ 5.
Paciente femenina de 49 años de edad, en postoperatorio inmediato de tiroidectomía
total por carcinoma papilar de tiroides, que empieza con espasmos musculares y
gesticulación facial, al examen físico presenta positividad del signo de Trousseau, para
corroborar su diagnóstico se solicitaría con prioridad:
a. Densitometría ósea c. Vitamina D y magnesio.

b. Calcio y parathormona. d. Troponinas

2
Name: ________________________ ID: A

____ 6.
Paciente masculino de 57 años de edad con antecedentes de radiación de cabeza y
cuello, acude por aumento de volumen unilateral de cuello, se realiza ecografía de
tiroides y reporta nódulo tiroideo sospechoso de malignidad, con TSH, T4 libre y T3
libre en rango de la normalidad, su conducta de elección debe ser solicitar:
a. Gammagrafía tiroidea, para c. Tiroglobulina
valorar vascularidad.

b. Anticuerpos antitiroideos d. Punción por aspiración con


aguja fina.

____ 7.
Paciente masculino de 32 años de edad, que se encuentra en terapia intensiva por
Traumatismo Craneoencefálico Severo, Glasgow 8 puntos, presenta poliuria de
6.500ml en las últimas 24 horas, con sodio 145mEq/L, osmolaridad normal, el
diagnóstico más probable sería:
a. Síndrome de secreción c. Polidipsia dipsógena por
inadecuada de hormona disminución en el punto
antidiurética (SIADH). umbral del mecanismo
osmorregulador.

b. Diabetes insípida nefrógena. d. Diabetes insípida hipofisiaria o


central.

____ 8.
Dentro de los criterios de Síndrome metabólico, según ATP III, se incluyen:
a. Perímetro abdominal de menor c. Colesterol mayor a 200mg/dl y
a 94cm e obesidad central
hipercolesterolemia.

b. Triglicéridos mayor a 150mg/dl d. Colesterol HDL alto y


y presión arterial diastólica perímetro abdominal mayor
mayor a igual a 85mmHg a 102 en mujeres.

3
Name: ________________________ ID: A

____ 9.
Paciente masculino de 32 años de edad, que es referido por bocio nodular, refiere
disminución de peso, irritabilidad y deposiciones diarreicas, al examen físico se
constata: frecuencia cardíaca: 108 latidos por minuto, proptosis, piel caliente, temblor
fino en manos, se diagnosticó hipertiroidismo primario. Los exámenes complementarios
podría que sustentaron al diagnóstico son:
a. TSH elevada y T3 libre c. Anticuerpos antitiroideos
elevada, anticuerpos normales y TSH
antitiroideos elevados. elevada.

b. T4 y T3 libres elevadas, d. Anticuerpos antitiroideos


con nódulo elevados y T4 libre baja.
hipercaptante en la
gammagrafía tiroidea.

____ 10.
Las siguientes manifestaciones son parte de las complicaciones microvasculares de la
diabetes mellitus:
a. Coronariopatías, nefropatía y c. Arteriopatía periférica,
retinopatía diabética. neuropatía autonómica y
retinopatía diabética.

b. Nefropatía, retinopatía d. Edema macular, neuropatía


diabética y enfermedad sensorial y motora,
cerebrovascular. nefropatía.

4
ID: A

Endocrinología
Answer Section

MULTIPLE CHOICE

1. ANS: B
Paciente femenina de 38 años de edad, con antecedente de hiperprolactinemia, es
referida a consulta externa de endocrinología por cambios en sus rasgos faciales,
caracterizados por prognatismo, abombamiento frontal y nariz grande, además de
aumento de tamaño de los pies y manos, tras realizarse todos los estudios pertinentes,
se indicó estudio histopatológico que informó adenomas productor de hormona de
crecimiento. Debe plantearse que podría haber otras manifestaciones clínicas, como:
a) Es incorrecta porque son síntomas sugerentes de hipertiroidismo
b) Es correcta porque en la acromegalia surge visceromegalias que pueden
generalizarse.
c) Es incorrecta porque son síntomas sugerentes de síndrome de Cushing.
d) Es incorrecta porque constituye la triada clásica del Feocromocitoma.

PTS: 1 DIF: Baja


REF: HARRISON Tinsley: Principios de medicina interna, 19ª Edición McGraw-Hill Interamericana de
España 2016. Pag. 2269 - 2271
OBJ: Establece un diagnóstico clínico relacionado de trastornos de la adenohipófisis e hipotálamo.
TOP: Medicina Interna - Endocrinología NOT: Autor: Rebeca Silvestre Ramos
2. ANS: C
Paciente masculino de 42 años de edad, acude por bocio, refiere dificultad para
concentrarse en el trabajo, aumento de peso no relacionado con mayor ingesta, ya que
además presentó pérdida de apetito, en la última semana se asocia párpados
edematosos y edema pretibial sin fóvea. De acuerdo a su sospecha diagnóstica se
puede afirmar que:

a) Es incorrecta porque estos pueden sugerir hipertiroidismo secundario.


b) Es incorrecta porque constituyen signos clínicos de oftalmopatía tiroidea.
c) Es correcta porque en el hipotiroidismo primario la TSH se encuentra elevada debido
a un mecanismo de retroalimentación.
d) Es incorrecta porque constituyen el diagnóstico diferencial de Feocromocitoma.

PTS: 1 DIF: Media


REF: HARRISON Tinsley: Principios de medicina interna, 19ª Edición McGraw-Hill Interamericana de
España 2016. Pag. 2289 - 2292.
OBJ: Identifica los trastornos de la glándula tiroidea y correlaciona la sintomatología clínica con el
diagnóstico. TOP: Medicina Interna - Endocrinología NOT: Autor: Rebeca Silvestre Ramos

1
ID: A

3. ANS: C
Paciente femenina de 52 años de edad, con sobrepeso, acude por presentar poliuria,
polidipsia y pérdida de peso de 6 meses de evolución, en la última semana se asocia
debilidad y visión borrosa, su sospecha diagnostica es diabetes mellitus tipo 2, por lo
que podría esperarse los siguientes resultados de laboratorio:
a) Es incorrecta porque hay aumento de hemoglobina glicosilada
b) Es incorrecta porque la glucosa debe ser superior a 200mg/dl.
c) Es correcta porque ambas constituyen los criterios diagnósticos de diabetes mellitus
tipo 2.
d) Es incorrecta porque la intolerancia a la glucosa excluye el diagnóstico de diabetes
mellitus tipo 2.

PTS: 1 DIF: Media


REF: HARRISON Tinsley: Principios de medicina interna, 19ª Edición McGraw-Hill Interamericana de
España 2016. Pag. 2399 - 2407.
OBJ: Identifica la Diabetes Mellitus, correlaciona la sintomatología clínica y sus complicaciones.
TOP: Medicina Interna - Endocrinología NOT: Autor: Rebeca Silvestre Ramos
4. ANS: D
Paciente femenina de 46 años de edad, es referida por obesidad, refiere haber seguido
diferentes dietas, además de ejercicios de moderada intensidad en los últimos 6
meses, sin lograr disminución de peso, al examen físico se constata presión arterial
146/92mmHg, piel delgada, rubicundez facial, cara redonda, giba de búfalo, estrías
violáceas, se solicita medir ACTH y se encuentra disminuida; el diagnóstico probable
es:
a) Es incorrecta porque predominan síntomas asociados a hiperandrogenismo, los
descritos son típicos del síndrome de Cushing.
b) Es incorrecta, porque sólo coincide con la Hipertensión Arterial, sin embargo
esta es paroxística, el resto de las manifestaciones son típicas del Síndrome de
Cushing.
c) Es incorrecta porque en la Enfermedad de Cushing la ACTH se encuentra
normal o elevada.
d) Es correcta, porque son manifestaciones del exceso de glucocorticoides y en
ocasiones de mineralocorticoides.

PTS: 1 DIF: Media


REF: HARRISON Tinsley: Principios de medicina interna, 19ª Edición McGraw-Hill Interamericana de
España 2016. Pag. 2399 - 2407.
OBJ: Establece un diagnostico topográfico, etiológico, patológico y funcional de trastornos de la
Glándula Suprarrenal.TOP: Medicina Interna - Endocrinología NOT: Autor: Rebeca Silvestre Ramos

2
ID: A

5. ANS: B
Paciente femenina de 49 años de edad, en postoperatorio inmediato de tiroidectomía
total por carcinoma papilar de tiroides, que empieza con espasmos musculares y
gesticulación facial, al examen físico presenta positividad del signo de Trousseau, para
corroborar su diagnóstico se solicitaría con prioridad:
a) Es incorrecta porque se solicita cuando de plantea Osteoporosis, no es un
examen que se use con prioridad en estos casos.
b) Es correcta, ya que la medición de calcio será la prueba inicial para corroborar
hipocalcemia y la parathormona confirmará que el mismo es secundario a
hipoparatiroidismo.
c) Es incorrecta porque los mismos no son prioritarios, estos pueden solicitarse con
posterioridad.
d) Es incorrecta, ya que estas son de prioridad cuando se sospecha enfermedades
cardíacas.

PTS: 1 DIF: Baja


REF: HARRISON Tinsley: Principios de medicina interna, 19ª Edición McGraw-Hill Interamericana de
España 2016. Pag. 2482 - 2495.
OBJ: Correlaciona la sintomatología clínica con el diagnostico patológico, etiológico y funcional, para
un tratamiento inicial de trastornos de la tiroides y paratiroides. TOP: Medicina interna -
Endocrinología
NOT: Autor: Rebeca Silvestre Ramos.
6. ANS: D
Paciente masculino de 57 años de edad con antecedentes de radiación de cabeza y
cuello, acude por aumento de volumen unilateral de cuello, se realiza ecografía de
tiroides y reporta nódulo tiroideo sospechoso de malignidad, con TSH, T4 libre y T3
libre en rango de la normalidad, su conducta de elección debe ser solicitar:

a) Es incorrecto, porque la gammagrafía de tiroides se solicita cuando contamos


con perfil tiroideo compatible con nódulo tiroideo hiperduncionante o tóxico, en
ese caso T4 libre y T3 libre se encuentran elevados.
b) Es incorrecto, porque no es de elección para confirmar cáncer de tiroides,
c) Es incorrecto, porque la tiroglobulina se solicita para seguimiento postoperatorio
de cáncer de tiroides.
d) Es correcto, porque describirá las características histopatológicas para confirmar
el diagnóstico.

PTS: 1 DIF: Baja


REF: HARRISON Tinsley: Principios de medicina interna, 19ª Edición McGraw-Hill Interamericana de
España 2016. Pag. 2482 - 2495.
OBJ: Correlaciona la sintomatología clínica con el diagnostico patológico, etiológico y funcional, para
un tratamiento inicial de trastornos de la glándula tiroides y paratiroides.
TOP: Medicina Interna - Endocrinología. NOT: Autor: Rebeca Silvestre Ramos

3
ID: A

7. ANS: D
Paciente masculino de 32 años de edad, que se encuentra en terapia intensiva por
Traumatismo Craneoencefálico Severo, Glasgow 8 puntos, presenta poliuria de
6.500ml en las últimas 24 horas, con sodio 145mEq/L, osmolaridad normal, el
diagnóstico más probable sería:
a) Es incorrecta, porque en el SIADH hay hiponatremia euvolémica con
hipoosmolalidad.
b) Es incorrecta, ya que por el antecedente de traumatismo craneoencefálico es
más planteable la Diabetes insípida hipofisiaria o central.
c) Es incorrecta, porque la polidipsia dipsógena se caracteriza por sed inapropiada,
un paciente con Glasgow de 8 puntos no tolera vía oral.
d) Es correcta, ya que una de las causas de Diabetes insípida hipofisiaria o central
es el traumatismo craneoencefálico y se caracteriza por poliuria importante.

PTS: 1 DIF: Media


REF: HARRISON Tinsley: Principios de medicina interna, 19ª Edición McGraw-Hill Interamericana de
España 2016. Pag. 2275 - 2280.
OBJ: Establece un diagnóstico clínico relacionado con trastornos de la hipófisis e hipotálamo.
TOP: Medicina Interna - Endocrinología. NOT: Autor: Rebeca Silvestre Ramos
8. ANS: B
Dentro de los criterios de Síndrome metabólico, según ATP III, se incluyen:
a) Es incorrecta, porque el perímetros abdominal es mayor a 102cm en mujeres y
88cm en hombres, la hipercolesterolemia no está dentro de los criterios.
b) Es correcta, porque los niveles de colesterol no son parte del Síndrome
Metabólico.
c) Es incorrecta, porque Colesterol HDL alto y perímetro abdominal mayor a 102 en
mujeres.
d) Es incorrecta, porque los valores de HDL están bajos.

PTS: 1 DIF: Media


REF: HARRISON Tinsley: Principios de medicina interna, 19ª Edición McGraw-Hill Interamericana de
España 2016. Pag. 2449 - 2454
OBJ: Identifica la Obesidad y la Diabetes Mellitus, correlaciona la sintomatología clínica y sus
complicasiones. TOP: Medicina Interna - Endocrinología NOT: Autor: Rebec SIlvestre Ramos

4
ID: A

9. ANS: B
Paciente masculino de 32 años de edad, que es referido por bocio nodular, refiere
disminución de peso, irritabilidad y deposiciones diarreicas, al examen físico se
constata: frecuencia cardíaca: 108 latidos por minuto, proptosis, piel caliente, temblor
fino en manos, se diagnosticó hipertiroidismo primario. Los exámenes complementarios
que sustentaron al diagnóstico serían:
a) Es incorrecto, porque en el hipertiroidismo primario la TSH se encuentra baja.
b) Es correcto, ya que el perfil tiroideo se correlaciona con hipertiroidismo
primario y la gammagrafía corrobora nódulo hiperfuncionante o tóxico.
c) Es incorrecto, porque en el hipertiroidismo primario la TSH se encuentra baja.
d) Es incorrecto, porque la T4 se encuentra baja.

PTS: 1 DIF: Media


REF: HARRISON Tinsley: Principios de medicina interna, 19ª Edición McGraw-Hill Interamericana de
España 2016. Pag. 2293 - 2299
OBJ: Correlaciona la sintomatología clínica con el diagnostico patológico, etiológico y funcional para
un tratamiento inicial de trastornos de la Glándula Tiroides y Paratiroides.
TOP: Medicina Interna - Endocrinología. NOT: Autor: Rebeca Silvestre Ramos
10. ANS: D
Las siguientes manifestaciones son parte de las complicaciones microvasculares de la
diabetes mellitus:

a) Es incorrecta, porque las Coronariopatías son complicaciones macrovasculares.


b) Es incorrecta, porque la enfermedad cerebrovascular es una complicación
macrovascular.
c) Es incorrecta, porque la Arteriopatía periférica, es una complicación
macrovascular.
d) Es correcta, porque las tres son complicaciones microvasculares.

PTS: 1 DIF: Media


REF: HARRISON Tinsley: Principios de medicina interna, 19ª Edición McGraw-Hill Interamericana de
España 2016. Pag. 2293 - 2299
OBJ: Identifica la Obesidad y la Diabetes Mellitus, correlaciona la sintomatología clínica y sus
complicaciones. TOP: Medicina Interna - Endocrinología. NOT: Autor: Rebeca Silvestre Ramos

5
Endocrinología [Answer Strip] ID: A

C
_____ 3. D
_____ 6. B
_____ 9.

B
_____ 1.

D
_____ 7.

D
_____ 4. D 10.
_____

C
_____ 2.

B
_____ 8.

B
_____ 5.
Psicologia

MULTIPLE CHOICE

1. Identifique dentro de las alteraciones del contenido del pensamiento el trastorno


psicopatológico que presenta este paciente.
-Paciente AMP , masculino de 38 años , de ocupación ingeniero que acude a
consulta consciente, orientado globalmente que viene remitido del subcentro de
salud y acompañado por la esposa ya que desde hace una semana se encuentra
con gran preocupación después de haber tenido una insignificante discusión de
trabajo con uno de sus compañeros. Su esposa en la entrevista refiere ¨Aunque
todos le hemos señalado que debe olvidar el incidente, desde que discutió con su
compañero de oficina se recrimina constantemente por su comportamiento, ha
llegado a decir que jamás lo perdonaremos y que su solución es trasladarse de
trabajo. Nadie comprende su reacción pues el incidente fue algo muy banal.´

a. Idea Fija c. Idea Fobica


b. Idea Obsesiva d. Idea Sobrevalorada
ANS: D PTS: 1 DIF: Alta
REF: Psicotalogia general.Deshaies Gabriel Editorial. Kapelusz
OBJ: Explica la normalidad y patologia de los trastornos psicopatologicos de los procesos
mentales superiores. TOP: Psicologia- Procesos Mentales Superiores. Normalidad y Patologia

2. - Al realizar la entrevista al paciente OHF masculino de 33 años de edad con


escolaridad superior de profesión profesor este refiere ¨Oigo claramente dentro
de mi cabeza una voz femenina que me ordena ofender a mis vecinos¨.
El objetivo es que usted identifique el trastorno psicopatológico presenta este
paciente.

a. Alucinaciones c. Ilusiones
b. Pseudoalucinaciones d. Alucinosis
ANS: B PTS: 1 DIF: Alta
REF: Psicopatologia General.Deshaies Gabriel. Editorial Kapelusz
OBJ: Explica la normalidad y la patologia de los trastornos psicopatologicos de los procesos
mentales superiores TOP: Psicologia- Procesos Mentales Superiores

1
3. En la entrevista psicológica a un paciente en consulta este recuerda con nitidez el
lugar donde ocurrió el accidente automovilístico e incluso, que participo en la
conducción de un herido grave, pero no recuerda que dicho herido fue su hijo,
que falleció posteriormente.

a. Amnesia lacunar c. Hipomnesia


b. Hipermnesia d. Paramnesia
ANS: A PTS: 1 DIF: Alta
REF: Psicopatologia General. Deshaies Gabriel. Editorial Kapelusz
OBJ: Explica la normalidad y patologia de los trastornos psicopatologicos de los procesos
mentales superiores TOP: Psicologia. Procesos mentales superiores. Normalidad y patologia

4. Paciente MDP de 25 años de edad, femenina, soltera, desocupada que viene


referida del subcentro de salud y que acude acompañada de su madre. En la
entrevista ¨Decía oír voces que la llamaban desde la ventana y que le indicaban
lo que debía hacer¨.
Identifique que trastorno psicopatológico presenta esta paciente.

a. Ilusiones c. Alucinaciones
b. Pseudoalucinaciones d. Despersonalizacion
ANS: C PTS: 1 DIF: Alta
REF: Psicopatologia General.Deshaies Gabriel. Editorial Kapelusz
OBJ: Explica la normalidad y patologia de los trastornos psicopatologicos de los procesos
mentales superiores TOP: Psicologia- Procesos mentales superiores. Normalidad y patologia

5. Paciente OGH de 36 años que viene referido del subcentro de salud. De profesión
albañil, casado con 2 hijos que desde hace algún tiempo su esposa nota que dice
cosas sin sentido . En la entrevista el paciente refiere ¨Yo sé que cuando venga la
mañana del rio, tendrá más frio¨

a. Prolijidad c. Retardo del pensamiento


b. Aceleracion del pensamiento d. Disgregacion
ANS: D PTS: 1 DIF: Alta
REF: Psicopatologia General.Deshaies Gabriel, editorial Kapelusz
OBJ: Explica la normalidad y patologia de los trastornos psicopatologicos de los procesos
mentales superiores TOP: Psicologia-Procesos mentales superiores. Normalidad y Patologia

2
6. A continuación, exponemos una serie de referencias sintomáticas que pueden
obtenerse tanto del propio paciente como de otras fuentes de información. El
objetivo es que usted valore para identificar el trastorno psicopatológico
El paciente refiere ¨Estaba de visita en la casa de Rosa, la esposa de mi nieto
que estudio en Estados Unidos, cuando llego mi vecina con la noticia del
accidente y después de esa impresión empezaron los temblores. ¨

a. Retardo del pensamiento c. Incoherencia


b. Prolijidad d. Disgregacion
ANS: B PTS: 1 DIF: Alta
REF: Psicopatologia General. Deshaiez Gabriel. Editorial Kapelusz
OBJ: Explica la normalidad y patologia de lostrastornos psicopatologiavcos de los procesos
mentales superiores TOP: Psicologia- Procesos Mentales superiores. Normalida y patologia

7. El paciente refiere ¨Doctor no puedo quitarme de la mente sus palabras cuando


ella me dijo que ya no me quería. Esto se ha convertido en una tortura y no
puedo evitar ese recuerdo¨.

a. Idea sobrevalorada c. Idea Fija


b. Idea delirante d. Idea Hipocondriaca
ANS: C PTS: 1 DIF: Alta
REF: Psicopatologia General. Deshaies Gabriel, Editorial Kapelusz
OBJ: Explica la normalida y patologia de los trastornos psicopatologicos de los procesos
mentales superiores TOP: Psicologia. Procesos mentales superipores , Normalidad y patologia

3
UNIVERSIDAD NACIONAL DE CHIMBORAZO
FACULTAD DE CIENCIAS DE LA SALUD
MEDICINA

Name: ________________________ Class: ___________________ Date: __________ ID: ______________

DERMATOLOGÍA

Multiple Choice
Identify the choice that best completes the statement or answers the question.

____ 1. La dermis, hipodermis sus vasos y músculos se derivan del:


a. endodermo c. ectodermo
b. mesodermo d. a y b

____ 2. La epidermis, anexos, melanocitos y nervios se derivan del:


a. Endodermo c. Ectodermo
b. Mesodermo d. a y b

____ 3. El estrato que sólo lo encontramos en las regiones gruesas y de roce como las palmas es:
a. E. basal c. E. lúcido
b. E. espinoso d. E. córneo

____ 4. El proceso por el que pasa la formación de la piel, que dura entre 2-3 semanas y se ve acelerado en ciertos
procesos patológicos como en la psoriasis es:
a. melanogénesis c. queratinización
b. hiperqueratinización d. b y c

____ 5. Los corpúsculos de Meissner sirven para le percepción de:


a. calor c. dolor
b. frío d. tacto

____ 6. Los corpúsculos de Paccini sirven para le percepción de:


a. calor c. dolor
b. frío d. tacto

____ 7. Lesión circunscrita, elevada y sólida de tamaño inferior a 1 cm y que por lo general no deja cicatriz es:
a. Pápula c. Nódulo
b. Pústula d. Tumor

____ 8. Lesión elevada, sólida de tamaño mayor de 1 cm, cuando la lesión tiene carácter inflamatorio puede
utilizarse el término de tubérculo.
a. Pápula c. Nódulo
b. Pútula d. Tumor

____ 9. La clasificación histológica del melasma es:


a. centrofacial, malar y mandibular c. frente, mejillas y labio superior
b. epidermico, dermico y misto d. ayc

____ 10. En un alto porcentaje de pacientes psoriásicos se puede afectar las uñas (40%); se observan erosiones
puntiformes u hoyuelos, este signo se conoce como:
a. signo del dedal c. mancha salmon
b. signo de la mancha de aceite d. leuconiquia

1
ID: A

DERMATOLOGÍA
Answer Section

MULTIPLE CHOICE

1. ANS: B
la piel procede de 2 capas blastodermicas
ectodermo: epidermis, anexos, melanocitos y nervios
mesodermo: dermis, hipodermis, vasos y musculos

PTS: 1 DIF: medio


REF: DERMATOLOGÍA. ATLAS, DIAGNÓSTICO Y TRATAMIENTO, ROBERTO ARENAS
2. ANS: C
La piel procede de 2 capas blastodérmicas
Ectodermo: epidermis, anexos, melanocitos y nervios
Mesodermo: dermis, hipodermis, vasos y musculos

PTS: 1 DIF: medio


REF: DERMATOLOGÍA. ATLAS, DIAGNÓSTICO Y TRATAMIENTO, ROBERTO ARENAS
3. ANS: C
Estrato lúcido: Película traslucida que solo existe en ciertas partes de mayor roce, como palmas y plantas

PTS: 1 DIF: medio


REF: DERMATOLOGÍA. ATLAS, DIAGNÓSTICO Y TRATAMIENTO. Roberto Arenas
4. ANS: C
Proceso por el cual pasa la formación de la piel, generalmente dura de 2 - 3 sem y se va hacelerando en
ciertos procesos patológicos como en la psosiaris.

PTS: 1 DIF: medio


REF: DERMATOLOGÍA. ATLAS, DIAGNÓSTICO Y TRATAMIENTO. Roberto Arenas
5. ANS: D
Corpúsculos de Meissner: para la percepción de tacto, piel y dedos

PTS: 1 DIF: medio


REF: DERMATOLOGÍA. ATLAS, DIAGNÓSTICO Y TRATAMIENTO. Roberto Arenas
6. ANS: C
Corpúsculos de Paccini: sirven para le percepción del dolor y la presión

PTS: 1 DIF: medio


REF: DERMATOLOGÍA. ATLAS, DIAGNÓSTICO Y TRATAMIENTO. Roberto Arenas
7. ANS: A
Pápula es una lesión primaria, circunscrita, elevada y sólida de tamaño inferior a 1 cm y que por lo
general al desaparecer no deja cicatriz.

PTS: 1 DIF: medio


REF: DERMATOLOGÍA. ATLAS, DIAGNÓSTICO Y TRATAMIENTO. Roberto Arenas

1
ID: A

8. ANS: C
El nódulo es una lesión primaria, elevada, sólida de tamaño mayor de 1 cm, cuando la lesión tiene
carácter inflamatorio puede utilizarse el término de tubérculo.

PTS: 1 DIF: medio


REF: DERMATOLOGÍA. ATLAS, DIAGNÓSTICO Y TRATAMIENTO. Roberto Arenas
9. ANS: B
La clasificación histológica de melasma presenta 3 tipos: epidérmico es de color marrón o
café claro; el dérmico, gris o azul cenizo, y el mixto, marrón oscuro, que es la variedad
más frecuente.

PTS: 1 DIF: medio


REF: DERMATOLOGÍA. ATLAS, DIAGNÓSTICO Y TRATAMIENTO. Roberto Arenas
10. ANS: A
En un alto porcentaje de pacientes psoriásicos se puede afectar las uñas (40%); se observan erosiones
puntiformes u hoyuelos, este signo se conoce como signo del dedal ó tambien conocido como peeting
ungueal.

PTS: 1 DIF: medio


REF: DERMATOLOGÍA. ATLAS, DIAGNÓSTICO Y TRATAMIENTO. Roberto Arenas

2
DERMATOLOGÍA [Answer Strip] ID: A

B
_____ 1.

C
_____ 2.

C
_____ 3.

C
_____ 4.

D
_____ 5.

C
_____ 6.

A
_____ 7.

C
_____ 8.

B
_____ 9.

A 10.
_____
Name: ________________________ Class: ___________________ Date: __________ ID: A

REACTIVO - INFECTOLOGIA - DRA RIOS

Multiple Choice
Identify the choice that best completes the statement or answers the question.

____ 1. Mycobacterium tuberculosis se transmite casi siempre desde un paciente con TB pulmonar contagiosa a
otras personas por medio de las gotitas respiratorias que: Señale lo verdadero
a) La tos convierte en un aerosol
b) El estornudo convierte en un aerosol
c) La fonación convierte en un aerosol.

a. a.b.c
b. a.c.
c. b.c
d. a.b

____ 2. En la patogenia de la infección por Herpes I, señale lo falso

a. La exposición al HSV en superficies mucosas o sitios de abrasión cutánea permite la


entrada del virus hacia las células de la epidermis y la dermis
b. Ya sea clínico o subclínico, el contagio por HSV se acompaña de una réplica suficiente
de virus para infectar las terminaciones de nervios sensitivos o autonómicos.
c. En el ser humano, se conoce que el tiempo que transcurre entre la inoculación del virus
en los tejidos periféricos y su propagación hacia los ganglios es de 6 semanas
d. En la infección por HSV-1, los ganglios trigeminales son los más comúnmente
infectados, aunque también pueden ser los ganglios cervicales inferior y superior.

____ 3. En la artritis sifilitica, señale lo falso

a. En la sífilis congénita precoz hay hinchazón periarticular e inmovilidad de los miembros


afectados
b. La articulación de Clutton aparece entre los 2 y 3 años de edad consiste en sinovitis
aguda, dolorosa con derrames en las articulaciones grandes,
c. La articulación de Clutton es una manifestación tardía de la sífilis congénita que
consiste en sinovitis crónica indolora con derrames en las articulaciones grandes,
d. La sífilis secundaria puede asociarse con artralgias, con artritis simétrica de rodillas y
tobillos y en ocasiones de hombros y muñecas, y con sacroileítis
e. En la sífilis terciaria, las articulaciones de Charcot son secundarias a la pérdida de la
sensibilidad producida por la tabes dorsal

____ 4. En la meningitis bacterianan aguda, señale lo falso

a. EL trastorno que agrava el peligro de meningitis neumocócica es la neumonía por


neumococo.
b. La infección puede ser iniciada por la colonización nasofaríngea
c. La mortalidad sigue siendo de alrededor de 90% a pesar del uso de antibióticos.
d. La aparición de petequias en la piel puede orientar hacia el diagnóstico de infección por
meningococos.
e. En algunos pacientes la enfermedad es fulminante y causa la muerte en término de horas

1
Name: ________________________ ID: A

____ 5. Meningitis bacteriana aguda, señale lo verdadero


a) Gran parte de la fisiopatología es consecuencia directa del aumento en el LCR de citocinas y
quimiocinas.
b) La combinación de edemas intersticiales, vasógeno y citotóxico hace que aumente la presión
intracraneal y causa coma.
c) La meningitis puede presentarse como una enfermedad aguda fulminante que avanza
rápidamente en pocas horas
d) La meningitis puede presentarse como una infección subaguda que empeora progresivamente a
lo largo de varios días

a. a,b,c
b. a.c.d
c. b.c.d
d. Todas
e. Ninguna

____ 6.
Meningitis bacteriana aguda, señale el literal falso
a) La tríada clásica de la meningitis incluye fiebre, cefalea y rigidez de la nuca
b) Solo en menos del 5% de los pacientes disminuye el nivel de conciencia y varía desde el letargo
hasta el coma.
c) En casi todos los sujetos con meningitis bacteriana aparecen fiebre y combinaciones de cefalea,
rigidez de cuello o alteración del nivel de conciencia.
d) Otras manifestaciones frecuentes son náusea, vómito y fotofobia.
e) Hasta en 20 a 40% de los pacientes las convulsiones forman parte del cuadro clínico inicial de
una meningitis bacteriana

a. a.b.c
b. a.c.d
c. c,d,e
d. Todas
e. Ninguna

____ 7. Cuál es el tratamiento para una gangrena gaseosa? Señale la respuesta correcta:

a. a- Nafcilina u oxacilina 2gr intravenosa cada 4 a 6 hs


b. b- Clindamicina 660 a 900mg ev cada 6 a 8/hs más penicilina G 4 millones de U
intravenosa cada 4-6hs
c. c- Aciclovir 800 mg PO 5 veces al día durante siete a diez días más eritromicina 500 mg
PO cada 6 hs.
d. d- Vancomicina 1 gr intravenosa cada 12hs

____ 8. Sobre Sífilis


Si se confirma sífilis en una embarazada por las pruebas serológicas pero nunca se evidenció la clínica de
la enfermedad y es de evolución desconocida ¿Cuántas dosis Ud. le indicaría de Penicilina Benzatínica?

a. Una dosis
b. Dos dosis
c. Tres dosis
d. Ninguna pues debe ser un falso positivo.

2
Name: ________________________ ID: A

____ 9. Sobre Sífilis, señale lo correcto

a. El periodo de incubación es de 9 a 120 c. La vía de contagio es exclusivamente


días (promedio 31 días) sexual
b. El periodo de incubación es de 9 a 60 d. La sífilis latente aquella que se
días (promedio 21 días) manifiesta por reacciones serológicas y
por la prueba de campo oscuro

Multiple Response
Identify one or more choices that best complete the statement or answer the question.

____ 10. Paciente de 45 años con antecedentes de VIH bien controlado con antirretrovirales (CD4 550) y carga viral
de VIH indetectable, consulta por tos productiva, dolor pleuriticoen costado derecho y alza termica de 48
horas de evolución, Se examina sat de O2 88%, estertores en base derecha. Rx de torax consolidacion
basal derecha. Que tratamiento empirico iniciaria

a. Ceftriaxona 2 g y Azitromicina 500 mg cada 24 horas


b. Ceftriaxona 2 g, Azitromicina 500 mg cada 24 horas y TMP-Sulfametoxazol 5 mg/Kg/8
horas (basado en el TMP)
c. Meropenem 1 g/8h y Vancomicina 1g/12 horas
d. Metil-prednisolona 40 mg
/d, Ceftriaxone 2 g IV/24 horas y TMP-Sulfametoxazol 5 mg/Kg/8h (basado en dosis de
TMP)

____ 11. Un paciente con paludismo cerebral puede presentarl, escoja el literal incorrecto

1. Se debe sospechar en personas que han viajado recientemente a áreas endémicas de paludismo
2. Tiene un cuadro clínico inicial de fiebre, letargo u otros signos neurológicos.
3. El paludismo fulminante es causado por Plasmodium falciparum
4. La rigidez de nuca y la fotofobia son inusuales.
5. Origina temperatura >40°C, hipotensión, ictericia, síndrome de insuficiencia respiratoria aguda del
adulto y hemorragia.

a. 1,2,3
b. 2,3,4
c. 1,2,5
d. 2,3,5

____ 12. Sobre el Herpes Genital Señale el literal falso

a. El herpes genital primario en su primer c. Las infecciones por reactivación suelen


episodio se caracteriza por fiebre, ser asintomáticas o pueden ocasionar
cefalea, malestar y mialgias. lesiones genitales o uretritis con disuria.

b. Los síntomas locales predominantes son d. Siempre , el antecedente de coito rectal,


dolor, prurito, disuria, secreción vaginal las lesiones perianales pueden
y uretral y linfadenopatía inguinal presentarse como resultado de la latencia
dolorosa. establecida de la infección genital previa
en el dermatoma sacro

3
ID: A

REACTIVO - INFECTOLOGIA - DRA RIOS


Answer Section

MULTIPLE CHOICE

1. ANS: B PTS: 1 DIF: Media


REF: Principios de Medicina Interna de Harrison 18° Edición
OBJ: Realiza correctamente el análisis e interpretación con actitud crítica de las acciones para llegar al
diagnóstico y tratamiento de la Tuberculosis de nuestro país
TOP: Enfermedades Infecciosas respiratorias
2. ANS: C PTS: 1 DIF: Media
REF: Principios de Medicina Interna de Harrison 18° Edición
OBJ: Realiza correctamente el análisis e interpretación con actitud crítica de las acciones para llegar al
diagnóstico y tratamiento de las principales infecciosas de la piel de nuestro país
TOP: Enfermedades Infecciosas de la Piel y tejidos blandos
3. ANS: B PTS: 1 DIF: Alta
REF: Principios de Medicina Interna de Harrison 18° Edición
OBJ: Realiza correctamente el análisis e interpretación con actitud crítica de las acciones para llegar al
diagnóstico y tratamiento de las principales enfermedades infecciosas tropicales de nuestro país
TOP: Enfermedades de Transmisión Sexual
4. ANS: C PTS: 1 DIF: Alta
REF: Principios de Medicina Interna de Harrison 18° Edición
OBJ: Realiza correctamente el análisis e interpretación con actitud crítica de las acciones para llegar al
diagnóstico y tratamiento de las principales enfermedades neuroinfecciosas de nuestro país
TOP: Enfermedades Infecciosas del Sistema Nervisoso Central
5. ANS: D PTS: 1 DIF: Alta
REF: Principios de Medicina Interna de Harrison 18° Edición
OBJ: Realiza correctamente el análisis e interpretación con actitud crítica de las acciones para llegar al
diagnóstico y tratamiento de las principales enfermedades infecciosas del Sistema Nervioso central de
nuestro país TOP: Infecciones del Sistema nervioso Cemtral - Meningitis
6. ANS: A PTS: 1 DIF: Alta
REF: Principios de Medicina Interna de Harrison 18° Edición
OBJ: Realiza correctamente el análisis e interpretación con actitud crítica de las acciones para llegar al
diagnóstico y tratamiento de las principales enfermedades infecciosas del Sistema Nervioso Central de
nuestro país TOP: Infecciones del Sistema Nerviso Central - Meningitis
7. ANS: B PTS: 1 DIF: Media
REF: Principios de Medicina Interna de Harrison 18° Edición
OBJ: Realiza correctamente el análisis e interpretación con actitud crítica de las acciones para llegar al
diagnóstico y tratamiento de las principales enfermedades infecciosas de Piel de nuestro país
TOP: Enfermedades Infecciosas de Piel y Partes Blandas
8. ANS: C PTS: 1 DIF: Media
REF: Principios de Medicina Interna de Harrison 18° Edición
OBJ: Realiza correctamente el análisis e interpretación con actitud crítica de las acciones para llegar al
diagnóstico y tratamiento de las principales enfermedades de Transmision Sexual de nuestro país
TOP: Enfermedades de Transmisión Sexual
9. ANS: B PTS: 1 DIF: Media
REF: Principios de Medicina Interna de Harrison 18° Edición
OBJ: Realiza correctamente el análisis e interpretación con actitud crítica de las acciones para llegar al
diagnóstico y tratamiento de las principales Enfermedades de Transmisión Sexual de nuestro país
TOP: Enfermedades de Transmisión Sexual

1
ID: A

MULTIPLE RESPONSE

10. ANS: B
Cuaderno_MIR2018_v3.pdf: Preg 121, pag 18 Pruebas selectivas 2017

PTS: 1 DIF: Media


REF: Principios de Medicina Interna Harrison 18° Edicion. Dengue : Guia de atencion para enfermos
en la Region de las Americas
OBJ: Realiza correctamente el analisi e interpretacion con actitud critica de acciones para el diagnostico
, tratamiento de las principales infecciones tropicales de nuestro pais
TOP: enfermedades Tropicales - Dengue
11. ANS: B PTS: 1 DIF: Media
REF: Principios de Medicina Interna de Harrison 18° Edición
OBJ: Realiza correctamente el análisis e interpretación con actitud crítica de las acciones para llegar al
diagnóstico y tratamiento de las principales enfermedades infecciosas tropicales de nuestro país
TOP: Enfermedades Infecciosas Tropicales
12. ANS: D PTS: 1 DIF: Media
REF: Principios de Medicina Interna de Harrison 18° Edición
OBJ: Realiza correctamente el análisis e interpretación con actitud crítica de las acciones para llegar al
diagnóstico y tratamiento de las principales enfermedades infecciosas virales de la piel de nuestro país
TOP: Enfermedades Infecciosas de la Piel y pates Blandas

2
REACTIVO - INFECTOLOGIA - DRA RIOS [Answer Strip] ID: A

D
_____ 5. B
_____ 9.

B
_____ 1.

B 10.
_____

A
_____ 6.
C
_____ 2.

B 11.
_____

B
_____ 3.

B
_____ 7.

D 12.
_____
C
_____ 4.
C
_____ 8.
Name: ________________________ Class: ___________________ Date: __________ ID: A

Pediatría - Salem Rosario

Multiple Choice
Identify the choice that best completes the statement or answers the question.

____ 1. Paciente masculino de 6 años de edad, acude por presentar fiebre, cefalea y odinofagia, se observa al
examen físico lengua roja, apariencia de fresa junto con exantema maculopapular en tronco y
extremidades
a. Sarampión c. Fiebre escarlatina
b. Varicela d. Rubeola

____ 2. Paciente masculino de 2 años de acude por presentar diarrea acuosa desde hace 5 días en varias ocasiones
que se acompaña de dolor abdominal y nauseas que llegan al vomito en 10 ocasiones, se realiza
coproparasitario seriado por 3 ocasiones identificando los quistes de Balantidium Coli. ¿Cuál sería el
tratamiento más recomendado para esta parasitosis?
a. Doxiciclina 30 mg/kg/día después de c. Amoxicilina más ácido clavulánico 40
cada comida VO por 7 días. mg/kg/día cada 8 horas VO por 10 días

b. Tetraciclina 6,25 a 12,5 mg/kg de peso d. Albendazol 400 mg dosis única VO


cada 6 horas VO por 7 días

____ 3. La paciente de 14 años de edad, fue atendida en el hospital demandado los días 30 de marzo y 9 de abril de
2011, por presentar crisis convulsivas y fiebre. El 14 de abril de 2011 regresó a Urgencias, estaba
inquieta, pálida, con mucosa oral seca, dolor abdominal franco en epigastrio, Mc. Burney y psoas
negativos. Se diagnosticó epilepsia y gastritis medicamentosa. En esta atención refirió coluria, por lo que
se solicitaron pruebas de funcionamiento hepático que reportaron: bilirrubina total, 8.019; bilirrubina
directa, 4.4; bilirrubina indirecta, 3.5; transaminasa glutámicooxalacética, 16,323; transaminasa
glutámicopirúvica, 8,330; fosfatasa alcalina, 303; deshidrogenasa láctica, 10,570; glucosa, 90; BUN, 20;
urea, 42; creatinina, 0.9. El 16 de abril de 2011, ingresó al Servicio de Pediatría y ese mismo día fue
egresada con diagnóstico de hepatitis probablemente A. Debido a su grave estado de salud, ingresó a otro
hospital donde se le diagnosticó hepatitis B.
a. La paciente no presenta síntomas de c. Estuvo mal guiado el diagnostico
hepatitis
b. La hepatitis es fulminante d. No es fulminante la hepatitis

____ 4. Madre acude a centro de salud refiriendo que su hijo de 2 semanas de edad presenta vómitos de contenido
gástrico de gran volumen alimentario, con aspecto de leche cortada y se da después de un tiempo variable
de alimentación, el niño presenta deshidratación y detención de su curva de crecimiento, al examen
físico se palpa masa en epigastrio.
a. Reflujo gastroesofágico c. Enfermedad del reflujo gastroesofágico
b. Hipertrófia de píloro d. Intolerancia-alergia a proteínas de
vacuno

1
Name: ________________________ ID: A

____ 5. El signo de Rovsing característico en la apendicitis aguda consiste en:


a. Dolor a la presión en el epigastrio al c. Sensibilidad de rebote pasajera en la
aplicar una presión firme y persistente pared abdominal
sobre el punto de McBurney
b. Dolor agudo que aparece al comprimir el d. Dolor en el punto de McBurney al
apéndice entre la pared abdominal y la comprimir el cuadrante inferior izquierdo
cresta iliaca del abdomen.

____ 6. Niño de 3 años masculino de raza mestiza, llega a servicio de emergencia por q refiere q aproximadamente
hace 3 semanas presenta fiebre, tos, brote fino en la piel de 15 días de evolución además de Dolor en el
costado o dolor en la espalda Micción dolorosa, Incremento en la frecuencia/urgencia urinaria, Necesidad
de orinar en la noche, Color de orina anormal o turbia en incluso ha presentado Sangre en la orina, olor
de orina fétido o y sin otros antecedentes patológicos de importancia, fuera de gripas ocasionales.
Ocasionalmente por 3 ocasiones, vómitos, náusea, al momento se encuentra en mismas condiciones
a. Sepsis neonatal tardía c. Infección de vías urinaria en niños

b. Sepsis neonatal temprana d. Síndrome de aspiración meconial

____ 7. Paciente que al examen físico presenta hipopigmentación (color de cabello y piel más claro q sus
progenitores), dermatitis y olor de su orina característico a moho. ¿A que enfermedad nos referimos?
a. Hipotiroidismo congénito c. Fenilcetonuria
b. Galactosemia d. Hiperplasia suprarrenal

____ 8. Paciente masculino, recién nacido a término adecuado para la edad gestacional, quien nace por parto
vaginal, con el antecedente de fiebre en la madre durante el periodo de postparto inmediato. Los padres
consultan a los 2 días de vida pues le notan alza térmica no cuantificada, dificultad respiratoria,
hipoactividad y rechazo a la leche materna, posteriormente presenta: fallo respiratorio que amerita
ventilación mecánica asistida por varios días.
a. Sepsis neonatal tardía c. Taquipnea transitoria del recién nacido
b. Sepsis neonatal temprana d. Síndrome de aspiración meconial

____ 9. Recién nacido de 24 horas presenta ictericia, tipo de sangre de la madre ORh (-), qué conducta tomaría.
a. Realizar Coombs directo más tipificación c. Realizar Coombs indirecto
sanguínea del Recién nacido

b. Realizar exámenes de bilirrubina y d. Ninguna


esperar que ceda el cuadro.

2
Name: ________________________ ID: A

____ 10. Recién nacido femenino nació deprimido Apgar por lo que necesitó reanimación cardiopulmonar. Se
hospitalizó y se inició manejo con líquidos endovenosos, gluconato de calcio y cuidados generales. A las
cuatro horas post natales presentó episodio de cianosis generalizada que mejoró con oxigenoterapia.
Posterior aparecieron chupeteo y convulsión multifocal, se manejó con fenobarbital (20mg/kg). Durante
su evolución clínica persisten las crisis convulsivas. El examen físico: peso 2100 g, talla: 48 cm, PC: 30,5
cm, PT: 29,5 cm, hipotonía generalizada, hiporreflexia, bajo efecto de sedación farmacológica. Glicemia:
18 mg/dl, tratada, con control de 64 mg/dl. Hb 10,4 g %, Hto 31,4%, leucogramo: 24.900 mm3.
Ionograma: Na+: 130 meq/lt, K+: 6,2 meq/lt. Rx de abdomen y líquido cefalorraquídeo normales.
a. Sufrimiento fetal c. Síndrome meconial

b. Asfixia neonatal d. Taquipnea transitoria del recién nacido

____ 11. Paciente de un año de edad que sufre caída de un metro de alto (resbalo de la cama) . el niño al momento
tuvo llanto inconsolable pero calmo con el consuelo de la madre. Con que escala se valora el estado de
conciencia.
a. Escala de Nih c. Índice de Brethel
b. Escala de Glasgow d. Escala de Rankin.

____ 12. Paciente masculino, recién nacido a término adecuado para la edad gestacional, quien nace por parto
vaginal. Madre del neonato regresa a los 15 días preocupada por que su hijo presenta una ulceración en la
región deltoidea del brazo izquierdo en donde le aplicaron una vacuna al momento de nacer. ¿Cuál es la
vacuna y las causas probables de dicha ulceración?
a. BCG-Mala técnica al aplicarla o exceso c. Pentavalente-Sistema inmunológico bajo
en la dosis administrada. del niño.
b. IPV-Mala refrigeración de la vacuna. d. BCG-Sistema inmunológico bajo del
niño.

____ 13. Paciente de 6 meses de edad acude a consulta tras presentar desde hace 4 horas náuseas que llegan al
vomito en tres ocasiones de contenido alimentario teniendo como causa aparente la ingesta de jugo
naranja hace 5 horas, a este cuadro se le acompaña irritabilidad y llanto fácil.
a. Infección por rotavirus c. Intolerancia alimenticia
b. Hipertrofia del píloro d. Síndrome diarreico agudo

____ 14. Niño con anorexia, fatigabilidad e irritable, adoptando una postura con las extremidades semiflexionadas y
abducidas, que llora al movilizarlo y tiene gingivitis hemorrágica e hiperqueratosis folicular, tendrá un
déficit de:
a. Vitamina A c. Vitamina K
b. Triptófano d. Vitamina C

3
Name: ________________________ ID: A

____ 15. Paciente masculino de 18 meses de edad llega a consulta el día 10 de enero del 2017. La madre referir
que presenta diarrea, vómito y fiebre, también menciona que su hijo duerme mucho. Es traído por la
madre quien e la consulta indica que son originarios de una comunidad rural ubicada en Chimborazo.
En la exploración física del paciente se muestra irritable, con pelo seco y quebradizo, aspecto seco de la
boca, presenta hepatomegalia y miembros inferiores con edema maleolar. Además de la diarrea y el
vómito que presenta desde hace 2 días estos posiblemente causados por una infección
a. Desnutrición infantil c. Parasitosis
b. Obesidad d. Amigdalitis

____ 16. Madre acude al subcentro de salud con la preocupación de que su hija de 1 año 7 meses de edad está muy
gordita y al control de peso y talla de la niña se obtiene un IMC de 21, como referencia tomamos la tabla
de IMC para la edad de Niñas y con ella de evidencia que una nia cuyp IMC para la edad está por encima
de 3 está obesa. ¿Cuáles serían las futuras complicaciones o patologías que la niña puede tener a futuro?
a. El síndrome metabólico c. La apnea obstructiva del sueño
b. Las complicaciones ortopédicas incluyen d. Todas
enfermedad de Blount

____ 17. Madre acude a centro de salud por que su hija de 4 año de edad hace 4 días presenta rinorrea, tos leve que
suena como a la de un perro, febrícula, no presenta babeo, mismo síntomas se empeoran por las noches.
a. Amigdalitis crónica. c. Epiglotitis.
b. Laringotraqueobronquitis. d. Traqueitis bacteriana.

____ 18. Un niño de 6 meses presenta tos persistente y fiebre. La exploración física y la radiografía de tórax sugieren
una neumonía. ¿Cuál de los siguientes microorganismos es menos probable que sea el agente causal de
esta infección?
a. Virus sincitial respiratorio c. Virus parainfluenza 1
b. Adenovirus d. Rotavirus

____ 19. Un niño de seis años fue mordido mientras jugaba con el perro de un vecino y presentó después celulitis y
supuración. El antibiótico de elección contra el microorganismo causante de la infección es:
a. Oxacilina c. Cloranfenicol
b. Tetraciclina d. Penicilina

____ 20. Paciente de un año de edad con 8kg presenta, fiebre de 38°C, diarrea, acompañado de vomito hace 3 días,
las deposiciones son liquidas amarillentas abundantes por 8 ocasiones al día. Al examen físico niño
letárgico (se mueve poco, ojos muy hundidos), mucosas muy secas sin saliva, respiración muy rápida.
a. EDA con deshidratación grave. c. EDA con deshidratación leve.
b. EDA con deshidratación moderada. d. Ninguna

4
Name: ________________________ ID: A

____ 21. Madre acude a centro de salud por que su hijo de 1 mes edad hace 4 días presenta vómitos y
regurgitaciones postprandiales el niño tiene peso y talla adecuados para la edad al examen físico no se
palpan masas en el abdomen ni otro signo o síntoma.
a. Reflujo gastroesofágico c. Estenosis hipertrófica de píloro

b. Enfermedad del reflujo gastroesofágico d. Intolerancia-alergia a proteínas de


vacuno

5
ID: A

Pediatría - Salem Rosario


Answer Section

MULTIPLE CHOICE

1. ANS: C
El presente caso se trata de Fiebre escarlatina, pues sus características patognomínicas son la lengua roja
con apariencia de fresa, con exantema macupapular en tronco y extremidades.

PTS: 3 DIF: Media


REF: PALACIOS, C., DURÁN, C., OROZCO, L., GARCÍA, M., & RUIZ, R. (20 de
SEPTIEMBRE-OCTUBRE de 2015). EXANTEMAS EN PEDIATÍA. MEDIGRAPHIC, 36(5), 412-423.
TOP: Medicina NOT: Dra. Salem Rosario
2. ANS: A
El mecanismo de acción de la doxiciclina actúa inhibiendo la síntesis proteica por unión a la subunidad
ribosomal 30S, siendo asi el mejor antiparasitario.

PTS: 3 DIF: Media


REF: Kliegman R, Jenson H, Behrman R, Stanton B. Nelson Tratado de Pediatría. 18th ed. Barcelona :
Elsevier ; 2008 TOP: Medicina NOT: Dra. Salem Rosario
3. ANS: A
Para su estudio, se estiman necesarias las siguientes precisiones: En términos de la literatura
especializada, la hepatitis B es una infección vírica del hígado que puede dar lugar tanto a un cuadro
agudo como a una enfermedad crónica. El virus se transmite entre las personas por contacto directo de
sangre a sangre, a través del semen o secreciones vaginales de una persona infectada. Los modos de
transmisión son los mismos que los del virus de la inmunodeficiencia humana (VIH), pero el virus de la
hepatitis B (VHB) es entre 50 y 100 veces más infeccioso. A diferencia del VIH, el VHB puede
sobrevivir fuera del organismo durante 7 días como mínimo, y en ese lapso puede causar infección si
penetra en el organismo de una persona no protegida por la vacuna.

PTS: 3 DIF: Media REF: AMIR. Pediatria, Marban. Año: 2011. Edición: 8va
TOP: Medicina NOT: Dra. Salem Rosario
4. ANS: B
Estamos ante una Hipertrofia del piloro por que se da en niños de 2 a 4 semanas de vida y se caracteriza
por presentar vómitos de contenido gástrico de gran volumen alimentario, con aspecto de leche cortada y
se da después de un tiempo variable de alimentación, también el niño puede presentar deshidratación y
detención de su curva de crecimiento y al examen físico se palpa masa en epigastrio

PTS: 3 DIF: Media REF: AMIR. Pediatria, Marban. Año : 2011. Edición : 8va
TOP: Medicina NOT: Dra. Salem Rosario
5. ANS: D
El signo de Rovsing es la presencia de dolor en la fosa iliaca derecha al presionar la fosa iliaca izquierda.
Se debe a que al presionar en ese punto el aire del colon se mueve de manera retrograda, distendiendo el
ciego donde está el apéndice inflamado, lo que provoca un aumento del dolor a ese nivel.

PTS: 3 DIF: Media REF: AMIR. Pediatria,Marban. Año : 2011. Edición : 8va.
TOP: Medicina NOT: Dra. Salem Rosario

1
ID: A

6. ANS: C
El presente caso se trata de una infección de las vías urinarias en un niño debido a la presencia de la
sintomatología, q incluye entre los más importantes como: dolor en el costado o dolor en la espalda,
micción dolorosa, Incremento en la frecuencia urinaria incluyendo la fiebre q es la sintomatología propia
de una IVU de niños.

PTS: 3 DIF: Media


REF: Medline Plus. Infección urinaria en niños. [En línea] ADAM, 21 de diciembre de 2017.
https://medlineplus.gov/spanish/ency/article/000505.htm. TOP: Medicina
NOT: Dra. Salem Rosario
7. ANS: C
El presente caso se trata de un paciente con fenilcetonuria que se caracteriza por presentar dermatitis, la
hipopigmentación (color del pelo o de la piel bastante más claros que los hermanos o padres), así como
un olor característico de la piel o de la orina.

PTS: 3 DIF: Media REF: AMIR. Pediatria,Marban. Año : 2011. Edición : 8va
TOP: Medicina NOT: Dra. Salem Rosario
8. ANS: B
El presente caso se trata de una Sepsis Neonatal Temprana ya que se presentó en las primeras 72 horas,
también refiere sintomatología propia de la Sepsis neonatal.

PTS: 3 DIF: Media REF: AMIR. Pediatria,Marban. Año : 2011. Edición : 8va
TOP: Medicina NOT: Dra. Salem Rosario
9. ANS: A
Es necesario saber el tipo de sangre del recién nacido en cuanto a Factor Rh y Grupo ABO en caso de ser
Rh+ se pide prueba de Coombs Directo para ver si hay la formación de Anticuerpos para el Antígeno D

PTS: 3 DIF: Media


REF: Behrman, R. E., Kliegman, R. M., & Jenson, H. B. (2004). Nelson tratado de pediatria. Madrid:
Elsevier. TOP: Medicina NOT: Dra. Salem Rosario
10. ANS: B
Estamos ante un caso de asfixia neonatal debido al que el niño Nació deprimido Apgar por lo que
necesitó reanimación cardiopulmonar. A las cuatro horas post natales presentó episodio de cianosis
generalizada que mejoró con oxigenoterapia. Convulsión multifocal, se manejó con fenobarbital
(20mg/kg). El examen físico: peso 2100 g, talla: 48 cm, PC: 30,5 cm, PT: 29,5 cm, hipotonía
generalizada, hiporreflexia,

PTS: 1 DIF: Media REF: AMIR. Pediatria,Marban. Año : 2011. Edición : 8va
TOP: Medicina NOT: Dra. Rosario Salem

2
ID: A

11. ANS: B
Escala de Glasgow, escala diseñada para evaluar de manera práctica el nivel de Estado de Alerta en los
seres humanos, cuantificación de tres parámetros: la apertura ocular, la respuesta verbal y la respuesta
motora. Dando un puntaje dado a la mejor respuesta obtenida en cada categoría

PTS: 1 DIF: Media


REF: Ignacio Manrique Martinez, Pedro Jesus Alcalá Minagorre. (FEBRERO de 2016). MANEJO DEL
TRAUMATISMO CRANEAL PEDIATRICO (11edicion ed., Vol. 4). (I. V. Valencia., Ed.) VALENCIA,
MADRID, ESPAÑA: Alfaz del Pí, Alicante. TOP: Medicina
NOT: Dra. Salem Rosario
12. ANS: A
El presente caso se trata acerca de analizar la inmunización oportuna además de las complicaciones y
causas de la misma

PTS: 1 DIF: Media


REF: CENTROS PARA EL CONTROL Y LA PREVENCIÓN DE ENFERMEDADES: CDC 2017.
DISPONIBLE EN: https://espanol.cdc.gov/enes/flu/professionals/vaccination/vaccine_safety.htm
TOP: Medicina NOT: Dra. Salem Rosario
13. ANS: C
El presente caso se trata de una intolerancia alimenticia, ya que el paciente presenta el antecedente de
ingesta de alimentos ácidos el cual no consta dentro de los parámetros de inicio de la alimentación o
ablactación en donde se menciona que los alimentos ácidos no deben ser añadidos hasta los 12 meses de
edad.

PTS: 1 DIF: Media


REF: Ministerio de Salud Pública del Ecuador, NORMAS Y PROTOCOLOS DE ALIMENTACIÓN
PARA NIÑOS Y NIÑAS MENORES DE 2 AÑOS, tomado de:
http://instituciones.msp.gob.ec/images/Documentos/nutricion/Alimentacion_nino_menor_2anios.pdf ;
http://www.salud.gob.ec/unidad-de-nutricion-guias-y-manuales/ TOP: Medicina
NOT: Dra. Salem Rosario
14. ANS: D
Es un cuadro de escorbuto por déficit de vitamina C. Es más frecuente entre los 7 meses y 2 años. Se
caracteriza por irritabilidad progresiva, sobre todo, al coger al niño. Presenta hemorragias en piel y
mucosas. Las encías se encuentran edematosas y con tendencia al sangrado

PTS: 1 DIF: Media


REF: López, Joaquín. AMIR TEST. Madrid: Marban Libros. 2012. Pag: 608.
TOP: Medicina NOT: Dra. Salem Rosario
15. ANS: A
Desnutrición infantil porque el paciente presenta peso muy bajo, baja estatura, IMC bajo

PTS: 1 DIF: Media


REF: Nelson, Tratado de pediatria, Elelvier Saunders, Año:2008, Edición:18, Páginas 126-127.
TOP: Medicina NOT: Dra. Salem Rosario

3
ID: A

16. ANS: D
El literal A es correcto ya que el síndrome metabólico se asocia a un riesgo especialmente alto de
enfermedad cardiovascular, con una prevalencia global de 4% en adolescentes y del 30% en
adolescentes con sobrepeso.
El literal B es correcto ya que la enfermedad de Blount, que se caracteriza por un sobrecrecimiento
de la vertiente medial de la metáfisis proximal de la tibia, que condiciona un arqueamiento de las
piernas y el deslizamiento de la epífisis de la cabeza femoral.
El literal C es correcto, porque la apnea obstructiva del sueño es más frecuente en niños con
sobrepeso y puede contribuir a problemas como la hipertensión, la fatiga diurna y la hipertensión
pulmonar.

PTS: 1 DIF: Media


REF: Nelson, Tratado de pediatria, Elelvier Saunders, Año: 2008, Edición:18, Página 237.
TOP: Medicina NOT: Dra. Salem Rosario
17. ANS: B
Estamos ante un caso de Laringotraqueobronquitis, caracterizado por cierto grado de rinorrea, faringitis,
tos leve y febrícula durante 1-3 días antes de la aparición de signos y síntomas claros de obstrucción de la
vía alta y que se presenta a partir de 6 meses a 3 años de edad.

PTS: 1 DIF: Media


REF: Nelson, Tratado de pediatria, Elelvier Saunders, Año:2008, Edición:18, Página 1762.
TOP: Medicina NOT: Dra. Salem Rosario
18. ANS: D
El rotavirus es un virus que tiene tropismo por la mucosa digestiva y es, por tanto, la causa más frecuente
de gastroenteritis aguda en la edad pediátrica, pero sin embargo altísimamente infrecuente en la etiología
de un neumonía.

PTS: 1 DIF: Media


REF: Steven R. Boas Tratado de Pediatría de Nelson 19 Edición Capitulo 397 pág. 1796
TOP: Medicina NOT: Dra. Salem Rosario
19. ANS: D
El antibiótico de elección es la penicilina pues tiene la mayor cobertura de gérmenes encontrados en la
saliva canina, ya que las heridas por mordedura de perro pueden ser de origen polimicrobiano.

PTS: 1 DIF: Media REF: AMIR. Pediatria,Marban. Año : 2011. Edición : 8va
TOP: Medicina NOT: Dra. Salem Rosario
20. ANS: A
Estamos ante un caso de EDA más deshidratación grave, debido que el paciente cursara estuporoso,
letárgico, sudoroso, hipotónico. Mucosas orales: muy secas sin saliva. Ojos: muy hundidos. Piel
turgencia cutánea: ++/+++ . Depresión de fontanela: muy hundida. Respiración muy rápida.

PTS: 1 DIF: Media REF: AMIR. Pediatria,Marban. Año : 2011. Edición : 8va
TOP: Medicina NOT: Dra. Salem Rosario
21. ANS: A
Estamos ante un caso de RGE leve, caracterizado por pequeños vómitos sin fuerza después de las tomas,
por lo demás el niño se encuentra asintomático con peso y talla en percentiles normales.

PTS: 1 DIF: Media REF: AMIR. Pediatria, Marban. Año : 2011. Edición : 8va
TOP: Medicina NOT: Dra. Salem Rosario

4
Pediatría - Salem Rosario [Answer Strip] ID: A

D
_____ 5. B 10.
_____ A 15.
_____ A 21.
_____

C
_____ 1.

C
_____ 6.
D 16.
_____

A
_____ 2.
B 11.
_____

B 17.
_____
A 12.
_____
C
_____ 7.

A
_____ 3.

D 18.
_____
B
_____ 8.

C 13.
_____

D 19.
_____

A
_____ 9.
B
_____ 4. D 14.
_____
A 20.
_____
Name: ________________________ Class: ___________________ Date: __________ ID: A

ABRIL 2020 - REACTIVOS CIRUGIA GENERAL DR. VINICIO MOENO RUEDA

Multiple Choice
Identify the choice that best completes the statement or answers the question.

____ 1. Paciente de 70 años de edad, diabético con 72 horas de evolución de dolor en hipocondrio derecho, sin
ictericia, SV: TA 90/45, FC 130 por minuto, T 39,5 grados centígadros. En el eco se evidencia gas en la
pared y luz vesicular. ¿cuál es su posibilidad diagnóstica?
a. Cólico biliar
b. Colecistitis enfisematosa
c. Colecistitis aguda
d. Hidrocolecisto

____ 2. Paciente mujer de 47 años, diagnosticada de coledocolitiasisi secundaria asintomática. ¿Cuál es la


terapeútica de elección?
a. Exploración laparoscópica de la vía biliar
b. Exploración de la vía biliar con técnica convencional
c. Uso de agentes de dilusión por contacto para cálculos de colesterol
d. CPRE más esfinterotomía

____ 3. Paciente mujer de 55 años de edad, con procesoso repetitivos de colangitis, con dolor en hipocondrio
derecho, ictericia y sépsis, a quien se le palpa una masa en diche región, ¿Cuál es su posibilidad
diagnóstica?
a. Quiste de colédoco
b. Ca de hígado
c. Colecistitis aguda
d. Coledocolitiasis

____ 4. En una paciente mujer de 38 años de edad con cuadro de pancreatitis aguda, con un valor de más de
1000UI/dl de amilasa sérica, le hace sospechar en pancreatitis de qué etiología?:
a. Tumoral
b. Alchólica
c. Biliar
d. Hiperlipidemia

____ 5. Paciente varón de 55 años de edad, con antecedente de cirrosis hepática, acude con dolor y distensión
abdominal, letargo, naúsea que llega al vómito alimentario en 2 ocasiones, alza térmica, leucocitosis en la
biometría hemática, evolución de 36 horas, cuál es la posibilidad diagn´sotica más adecuada?:
a. Sépsis de origen abdominal
b. Peritonitis primaria
c. Peritonitis secundaria
d. Peritonitis terciaria

____ 6. Varón de 0 años, con una masa reductible de 3 cm de diámetro ventral pararectal derecha supraumbilical,
sin antecedentes quirúrgicos, puede estar en relación con qué tipo de hernia?
a. Incisional
b. Crural
c. Spiegel
d. Epigátrica

1
Name: ________________________ ID: A

____ 7. Paciente varón de 60 años, de 160cm de altura, con 80Kg, sometido hace 3 años a laparotomía
exploratoria por peritonitis, presenta desde hace 1 año aproximadamente masa infraumbilical reductible,
que ha incrementado progresivamente de tamaño, llegando en la actualidad a medir 3 cm de diámetro,
cuál es su posibilidad diagnóstica?:
a. Hernia umbilical
b. Hernia de Spiegel
c. Hernia epigástrica
d. Hernia incisional

____ 8. Paciente varón de 28 años con el diagóstico de fístula entero cutanea, secundaria a proceso séptico séptico
por apendicitis complicada, la cual produce 150cc, señale en grupo de gasto se encuentra:
a. Alto gasto
b. Moderado gasto
c. Bajo gasto
d. Es un valor no clasificable

____ 9. Paciente mujer de 30 años con diagnóstico de abdomen obstructivo, quien produce por sonda nasogástrica
un líquido con la siguiente composición Na 60, K10, Cl 90. Sospechamos que el líquido es de qué
origen?:
a. Saliba
b. Gástrico
c. Bilar
d. Pancreático

____ 10. En un paciente varón de 40 años con ICM de 36, diagn´sticado de síndrome metabólico. ¿cuál es la
terapeútica adecuada a seguir?:
a. Cirugía bariátrica
b. Balón intragástrico endocópico
c. Tratamiento farmacológico
d. Dieta y ejercicio

2
ID: A

ABRIL 2020 - REACTIVOS CIRUGIA GENERAL DR. VINICIO MOENO RUEDA


Answer Section

MULTIPLE CHOICE

1. ANS: B
En el paciente diabético con problemas de vesícula biliar, con cuadro séptico y presencia de gas en la
pared vesicular, es posble que en el eco se encuentre gas en la pared, lo que indica infección con
producción de gas

PTS: 3 DIF: Alta


REF: roslyn, J. vesícula biliar y sistema biliar extrehepático, cap 29, en Principios de Cirugía Schwartz,
pag: 1422-1423 OBJ: Conocer la evolución de la colecistitis aguda y sus complicaciones
TOP: Patología de la vesícula y vía bilair KEY: Colecistitis enfisematosa
NOT: Dr. Vinicio Moreno Rueda
2. ANS: D
Diversos estudios muestran la eficacia y seguridad de la esfinterotomía endoscópica para tratar cálculos
de colesterol, ésto más la colangiografía e instrumentación de la vía biliar.

PTS: 3 DIF: Alta


REF: Roslyn, J; Vesícula biliar y sistema biliar extrehepático, cap 29, en Schwartz. Principios de
Cirugía, pag 1425 TOP: Patología de la vesícula y vía biliar
KEY: Coledocolitiais, cpre, esfinterotomía endosc+opica NOT: Dr. Vinicio Moreno Rueda
3. ANS: A
Un tericio de los pacientes con quistes de colédoco cpngénito son asintomáticos hasta la vida adulta,
luego pueden presentar colangitis recurrentes, dolor en hipocondrio derecho, sépsis, ictericia y en
ocasioines se palpa una masa a dicho nivel

PTS: 3 DIF: Alta


REF: Roslyn, J; Vesícula biiiar y sistema biliar extrehepático, capítulo 29, en Shwartz, Principios de
Cirugóa, 6ta edición, pag 1430 OBJ: Conocimiento de la patotología biliar
TOP: Patología biliar y páncreas KEY: Quiste colédoco, colangitis
NOT: Dr. Vinicio Moreno Rueda
4. ANS: C
Es frecuente en casos de pancreatitis biliar valores mayores de 1000UI/dl, los valores inferiores son
típicos de pancreatitis alcoholica aguda

PTS: 3 DIF: Alta


REF: Reber, H; Páncreas, cap 30, en en Schwrtz: Principios de Cirugía, 6t ed, pag 1451
OBJ: Conocimiento de la etiología y la relación con los niveles de amilasa de la pancreatitis aguda
TOP: Patología biliar y páncreas KEY: Pancreatitis aguda, amilasa
NOT: Dr. Vinicio Moreno Rueda
5. ANS: B
En la peritonitis primaria los síntomas clínicos cuelen ser insidiosos, se lo ve en pacientes con asceitis,
con dolor y distensión abdominal, en niños sobre todo vómitos, letargo y fiebre, leucocitosis; es un
cuadro clínico muy variable.

PTS: 3 DIF: Alta


REF: Wittmamm, D; Peritonitis en infección intraabdominal, cap. 32, en Schwartz, Principios de
Cirugía, 6ta edición, pag 1512 TOP: Infección intraabdominal KEY: Peritonitis
NOT: Dr. Vinicio Moreno Rueda

1
ID: A

6. ANS: C
La hernia de Spiegel es ventral en la porción supraumbilical de la línea semilunar de Spiegel (pararrectal)

PTS: 3 DIF: Alta


REF: Wntz, G; Hernias de la pared abdominal, cap 34, en Schwartz, Principios de Cirugía, 6ta edición,
Pag 1581 OBJ: Conocimiento de hernias abdominales ventrales
TOP: Hernias de la pared abdominal NOT: Dr. Vinicio Moreno rueda
7. ANS: D
Por lo general son pacientes obesos, que pasaron por un proceso infeccioso quirúrgico anterior, entre las
causas puede ser una mala cicatrización de la herida quirúrgica

PTS: 3 DIF: Alta


REF: Wantz, G, Hernias de la Pared abdomnal, Cap 34, en Schwartz, Principios de Cirugía, 6ta ed, pag
1582
OBJ: Conocimiento diagnóstico y tratamiento de la hernia incisional
TOP: Hernias de la pared abdominal KEY: Hernia incisional
NOT: Dr. Vinicio Moreno Rueda
8. ANS: C
Fístula enterocutanea, clasificación según el gasto<.
Alto: mayor a 500cc/24 horas
Moderado: 200 a 500 cc/24 horas
Bajo: menos 200cc/24 horas

PTS: 3 DIF: Alta


REF: Evenson, A; Absceso abdominal y fístula entérica; en Maingot, Operaciones Abdominales, 11era
edición, pag 187 OBJ: Conocimiento de la clasificación por la producción de las fístulas
enterocutaneas
TOP: Infección intraabdominal NOT: Dr. Vinicio Moreno Rueda
9. ANS: B
La composición del líquido gástrico es: pH menor a 4, Na 60, K10, no tiene HCO3 y Cl 90

PTS: 3 DIF: Alta


REF: Evenson, A; Absceso intrabadominal y fístula entérica, cap 7; en Maingot, Operaciones
abdominales; 11era edición, pag 188
OBJ: Conocimiento de la composición de los líquidos del tubo digestivo
TOP: Infección intraabdominal KEY: Fístula enterocutanea
NOT: Dr. Vinicio Moreno Rueda
10. ANS: A
Se le debe ofecer realizar una intervención quirúrgica de tipo bariátrico, en pacientes con BMI entre 35 y
40 con comorbilidades esa es la indicación.

PTS: 3 DIF: Alta


REF: Jacobs, D; Obesidad mórbida y sus operaciones, cap.16, en Maingot, Operaciones abdominales,
11era edición, pag 463 OBJ: Manejo de la obesidad TOP: Obesidad
KEY: Obesidad mórbida, comorbilidad, BMI NOT: Dr. Vinicio Moreno Rueda

2
ABRIL 2020 - REACTIVOS CIRUGIA GENERAL DR. VINICIO MOENO ID: A
RUEDA [Answer Strip]
D
_____ 7.

B
_____ 1.

C
_____ 8.

D
_____ 2.

B
_____ 9.

A
_____ 3.

A 10.
_____

C
_____ 4.

B
_____ 5.

C
_____ 6.
Name: ________________________ Class: ___________________ Date: __________ ID: A

CIRUGIA

Multiple Choice
Identify the choice that best completes the statement or answers the question.

____ 1. Paciente de 40 años varón, con ictericia, fiebre y dolor abdominal cólico localizado en hipocondrio derecho
de 72 horas de evolución, que en la ecosonografía presenta una vía biliar no dilatada en las porciones
intra ni esxtrahepáticas, con antecedentes de problemas de colitis; su diagnóstico será:

a. Ca periampular
b. Colangitis esclerosante
c. Coledocolitiasis
d. Cirroisis hepática

____ 2. Paciente mujer de 35 años, con dolor cólico en hipocondrio derecho de 4 días de evolución, ha sido
sometida a colecistectomía y en los hallazgos se reporta vesícula grande con paredes infamadas, cálculo
impactado en el cístico, llena de material mucoide; el posible diagnóstico es:
a. Empiema vesicular
b. Colecistitis crónica
c. Coledocolitiasis
d. Hidropesía vesicular

____ 3. Paciente varón de 40 años con diagnóstico de pancreatitis aguda, a quien se le somete a TAC dinámica y se
reporta lo siguiente: Agrandamiento difuso del páncreas, aspecto heterogéneo de la glándula y de la
grasa peripancreática, con dilatación del conducto de Wirsung y pequeñas colecciones extrapancreáticas.
Señalar que tipo de pancreatitis según la escala tomográfica de Balthazar presenta:
a. A
b. B
c. C
d. D

____ 4. Paciente varón 35 años con diagnóstico de pancreatitis aguda severa de etiología biliar, con 20 días de
evolución, cuyo APACHE ll al momento es de 14, con una TAC que reporta según la escala de Balthazar
una pancreatitis tipo E, con un índice de severidad tomográfico de 7 y cuyo estado general se deteriora
progresivamente, en su opinión el tratamiento a seguir es
a. ERCP
b. Rotación del esquema antibiótico
c. Colecistectomía urgente
d. Necrosectomía

____ 5. Paciente varón de 42 años de edad, quien acude con un sangrado digestivo alto de instauración rápida, se le
somete a una endoscopia digestiva alta que reporta un sangrado activo tipo Forrest B, se señala que al
parecer tiene un sangrado mayor a 1000cc, con un hematocrito real de 28%, más criterios de shock
hipovolémico. Como se debe proceder
a. Incrementar volumen de líquido y sangre
b. Incrementar la dosis de los bloqueadores de bomba de protones
c. Cirugía
d. Nueva endoscopía

1
Name: ________________________ ID: A

____ 6. Paciente de 80 años con episodios de diarrea alternados con estreñimiento, dolor en cuadrante inferior
izquierdo abdominal, fiebre, vómito, eventual proctorragia y que al examen físico se palpa una masa
dolorosa en fosa ilíaca izquierda, nos hace pensar en cuál de las siguientes posibilidades diagnósticas:

a. Enfermedad hemorroidal
b. Colon irritable
c. Diverticulitis colónica
d. Vólvulo de sigma

____ 7. Paciente mujer de 40 años de edad, con antecedente de ascitis secundario a insuficiencia cardíaca
congestiva; con un cuadro de dolor abdominal de 3 días de evolución, nausea, alza térmica, al examen
abdomen doloroso con signos de irritación peritoneal; la posibilidad diagnóstica y tratamiento serán,
señale lo correcto:
a. Peritonitis primaria y resolución quirúrgica urgente
b. Peritonitis primaria y tratamiento antibiótico empírico de inicio
c. Peritonitis secundaria y tratamiento quirúrgico urgente
d. Peritonitis terciaria y tratamiento antibiótico empírico de inicio

____ 8. Paciente varón de 35 años, policía sufre varios impactos de proyectil a nivel abdominal, llegando a
emergencias con 1 hora de evolución, shocado, hipotérmico. Ingresa a cirugía en forma emergente
encontrando, hemoperitoneo de 1000cc, trauma esplénico, trauma hepático, múltiples perforaciones
intestinales y colónicas. Cuál sería la cirugía a realizar
a. Reparación hepática, esplénica, resección intestinal y rafia colónica
b. Drenaje de hemoperitoneo, reparación hepática, rafia esplénica, rafia intestinal y
colostomía temporal
c. Drenaje de hemoperitoneo, sutura de todas las lesiones
d. Cirugía de control de daños

____ 9. Varón de 55 años, campesino, sin antecedentes quirúrgicos, con informe de distensión abdominal,
constipación; que desde hace 24 horas presenta dolor abdominal cólico intenso, distensión abdominal
muy marcada, sin tránsito intestinal, ni expulsión de flatos; señala que en ocasiones anteriores ha
presentado episodios similares, pero que se resolvían solos y en menos tiempo, hace 1 hora un vómito
tipo fecaloide; al examen físico se tiene intenso timpanismo y mayor dolor con resistencia voluntaria en
fosa ilíaca izquierda., además signos de irritación peritoneal. Que posibilidad diagnóstica a su criterio
sería la correcta:

a. Obstrucción intestinal por brida adherencial


b. Obstrucción intestinal por vólvulo de sigma
c. Diverticulits colónica complicada
d. Síndrome pilórico

____ 10. Paciente varón de 23 años de edad, que presenta un peritonitis secundario a apendicitis aguda complicada,
ha sido intervenido en dos ocasiones, siendo sometido a dos laparotomías, al momento el abdomen
distendido, está con una faja para evitar dehiscencia de sutura, dificultad respiratoria, y signos de
hipoperfusión; se le ha midiendo la presión intrabadominal, obteniendo un valor de 26 mmHg, con este
valor favor indicar que tipo de hipertensión abdominal tiene y cuál sería el tratamiento a seguir
a. grado 1 y observación
b. Grado ll y descompresión quirúrgica
c. Grado lll y posible descompresión abdominal quirúrgica
d. Grado lV y descompresión quirúrgica

2
Name: ________________________ ID: A

____ 11. Paciente mujer de 32 años de edad, quien sufre una agresión con arma blanca hace 4 horas, es sometida a
laparotomía encontrando biliperitoneo de 300cc, secundario a lesión de la pared de la vesícula biliar.
Según los datos proporcionados la intervención quirúrgica realizada es una:
a. Cirugía limpia
b. Cirugía límpia contaminada
c. Cirugía contaminada
d. Cirugía sucia

____ 12. Paciente varón, que sufre un trauma abdominal abierto y es sometido a lavado peritoneal diagnóstico,
presentando el siguiente resultado: Contaje de glóbulos rojos de 11.000 por mm cúbico, señale lo
correcto:

a. Es un resultado positivo
b. Es un resultado negativo
c. Es un resultado intermedio
d. No se debe realizar lavado peritoneal diagnóstico en trauma abdominal abierto

____ 13. Paciente mujer de 44 años, quien presenta dolor cólico en HD de 7 horas de evolución, iniciado luego de la
ingesta de comida grasa, cuadro similares en otras ocasiones, Murphy positivo; se le solicita un eco de
hígado y vía biliar el cual reporta: pared de vesícula de 2mm, diámetro transverso de 2cm, presencia de
múltiples cálculos pequeños. Indicar cuál es el diagnóstico ecosonográfico:

a. Colelitasis
b. Colecistitis aguda
c. Hidrocolecisto
d. Piocolecisto

____ 14. Paciente de 28 años con un cuadro de evolución de 72 horas de evolución de dolor abdominal, localizado
en FID, ha sido diagnosticado de peritonitis secundario a apendicitis complicada. Al ser peritonitis
secundaria se debe iniciar con una antibióticoterapia empírica, que esque será el más adecuada:

a. ampicilina, gentamicina, tetraciclina


b. ciprofloxacina, cloranfenicol
c. ampicilina más sulbactam
d. ceftriaxone y metronidazol

____ 15. Paciente varón de 31 años que acude por presentar masa reductible en ingle derecha, de 4 meses de
evolución, no dolorosa, al examen fisco se palpa masa superior que aparece en el orificio profundo y baja
aparentemente por el interior del conducto inguinal, su diagnóstico será:
a. Hernia inguinal directa
b. Hernia inguinal indirecta
c. Hernia crural
d. Hernia mixta

3
ID: A

CIRUGIA
Answer Section

MULTIPLE CHOICE

1. ANS: B
La colangitis esclerosante produce una obliteración inflamatoria y fibrótica progresiva de las vías biliares
intra y extrahepática, se asocia a colitis ulcerativa. La respuesta es B

PTS: 3 DIF: alta


REF: Brunicardi, F; Capítulo 31: Vesícula biliar y sistema biliar extrahepático, Schwartz.: Principios de
Cirugía, 8va edición, McGraw-Hill ISBN 97897010533737, 2006.
OBJ: Tratar al paciente quirúrgico con giagnóstico de coledocolitiasis y colangitis
TOP: Vesícula, vía biliar, páncreas KEY: colangitis, esclerosante
NOT: Dr. Vinicio Moreno Rueda
2. ANS: D
En casos de colecistitis aguda no resuelta en forma inmediata la vesícula se llena de material mucoide y
se distiende hasta alcanzar gran tamaño, este material proviene del epitelio de la vesícula y se denomina
hidropesía vesicular o hidrocolecisto. La respuesta es D.

PTS: 3 DIF: alta


REF: Cazorla, G; Colelitiasis y Colecistitis aguda: Urgencias quirúrgica 2da edición, Indugraf, ISBN
9789942035042, 2011.
OBJ: Tratar al paciente quirúrgico con diagnóstico de patología de vesícula biliar
TOP: Vesícula, vía biliar, páncreas KEY: colecistitis, empiema
NOT: Dr. Vinicio Moreno Rueda
3. ANS: C
Pancreatitis tipo C según la escala de Balthazar está constituido por el grado B de la misma escala más el
aspecto heterogéneo de la grasa peripancreatica lo que implica compromiso de la misma. Representa 2
puntos para el índice de severidad tomográfica, entonces las respuesta es C

PTS: 3 DIF: alta


REF: Camacho, I; Pancreatitis aguda, en Urgencias Quirúrgicas, 2da edición, Indugraf, ISBN
9789942035042, 2011.
OBJ: Tratar al paciente quirúrgico con diagnóstico de pancreatitis
TOP: Vesícula, v{ia biliar y p{ancreas KEY: TAC, Balthazar
NOT: Dr. Vinicio Moreno Rueda
4. ANS: D
El manejo adecuado de una pancreatitis se basa en los criterios de los consensos de Atlanta, en el
APACHE ll, dentro de los cuales se recomienda la cirugía en forma tardía, siendo una de sus indicaciones
el deterioro progresivo de paciente a pesar del buen tratamiento instaurado; recomendándose la
necrosectomía laparoscópica o por técnica abierta de ULM. La respuesta es D

PTS: 3 DIF: alta


REF: Uretsky, G; Chaildhood Pancreatitis, Am Fam Physican, 1999; 59: 2507-12
OBJ: Tratar al paciente quirúrgico con diagnóstico de pancreatitits
TOP: Vesícula, v{ia biliar, páncreas KEY: pancreatitis, APACHE II
NOT: Dr. Vinicio Moreno Rueda

1
ID: A

5. ANS: C
Al tener un sangrado activo, un paciente en shock hipovolémico, un diagnóstico endoscópico que señala
sangrado activo en sábana, se someter a cirugía, manteniendo como es lógico el resto de componentes del
tratamiento como coadyuvantes, son indicaciones quirúrgicas en este caso hemorragia de más de 1000 cc,
hcto inferior a 28, necesidad de haber recibido 1500cc de paquetes globulares, shock persistente,
ancianos, grupo sanguíneo raro y hemorragia recurrente. La respuesta es C

PTS: 3 DIF: alta


REF: Hoskins, S; duodenal ulcer healing by eradication of H pylori withaut antiacid treatment:
randomized controlled trial. Lancet 2004. 343, 508-510.
OBJ: Tratar al paciente quirúrgico con diagnóstico de abdomen agudo
TOP: Abdomen agudo KEY: sangrado digestivo, endoscopia, Forrest
NOT: Dr. Vinicio Moreno Rueda
6. ANS: C
Dentro de la presentación clínica de la enfermedad diverticular del colon sintomática complicada se
encuentra la diverticulitis que puede cursar con plastrón, absceso, o complicarse con perforación,
hemorragia digestiva baja, fístulas u obstrucción colónica; todo lo anterior en un paciente anciano. La
respuesta es C

PTS: 3 DIF: alta


REF: Belén, E; Laparoscopyc surgery in colorectal disease anales, vol 28, suplemento 3; 2005.
OBJ: Tratar al paciente quirúrgico con diagnóstico de abdomen agudo
TOP: Abdomen agudo KEY: diverticulitis, abdomen agudo
NOT: Dr. Vinicio Moreno Rueda
7. ANS: B
Luego de hacer el diagnóstico de peritonitis primaria en base a parámetros clínico, estudio de líquido
ascítico es mandataria una rápida y adecuada antibioticoterapia endovenosa, recomendado ampicilina y
aminoglucósidos. La respuesta es B

PTS: 3 DIF: alta


REF: Palacios, J; Peritonitis; Urgencias Quirúrgicas 2da edición, Indugraf, ISBN 9789942035042, 2011
OBJ: Tratar al paciente quirúrgico con diagnóstico de infección intra abdominal y sus complicaciones
TOP: Infección intra-abdominal KEY: peritonitis, antibiótico
NOT: Dr. Vinicio Moreno Rueda
8. ANS: D
Para prevenir la hipotermia, acidosis y coagulopatía en el paciente grave con trauma abdominal; se debe
realizar la cirugía de control de daños, la cual consta de tres tiempos: de los cuales el primero y el último
son quirúrgicos y el intermedio se lo realiza en UCI. La respuesta es D.

PTS: 3 DIF: alta


REF: Rotondo, M; Damage control: an approach for improved survival in exsanguinating abdominal
injury. Journal of trauma. 1993; 35. 375-383.
OBJ: Tratar al paciente quirúrgico con diagnóstico de abdomen agudo
TOP: Abdomen agudo KEY: Trauma abdominal, control de daños
NOT: Dr. Vinicio Moreno Rueda

2
ID: A

9. ANS: B
Patología en países en vías de desarrollo, más frecuente sobre los 50 años y en hombres, presenta un
cuadro de oclusión intestinal baja de ahí la gran distensión y el vómito tardío. Frecuente en nuestro
medio. La respuesta es B

PTS: 3 DIF: alta


REF: Moreno V; Vólvulo de Sigma: Urgencias Quirúrgicas 2da edición, Indugraf, ISBN
9789942035042, 2011. OBJ: Tratar al paciente con diagnóstico de abdomen agudo
TOP: Abdomen agudo KEY: obstrucción intestinal, vólvulo de sigma
NOT: Dr. Vinicio Moreno Rueda
10. ANS: D
Los grados de presión intrabadominal medido a través de la vejiga son de 4 grados, el grado lll va de 26 a
35mmHg, siendo indicado reanimar en forma agresiva al paciente y una descompresión quirúrgica del
abdomen. La respuesta es D

PTS: 3 DIF: alta


REF: Platell, C; The omentum. Wordl J Gastroenterology 2000; 6: 169-176.
OBJ: Tratar al paciente quirúrgico con diagn´sotico de abdomen agudo
TOP: Abdomen agudo KEY: PIA, hipertensión abdominal
NOT: Dr. Vinicio Moreno Rueda
11. ANS: C
La cirugía contaminada se ve en los casos de heridas traumáticas menores a 6 horas de evolución,
apertura del tracto biliar o genitourinario, en presencia de bilis u orina, contaminación importante por el
contenido del tubo digestivo, ruptura de la asepsia importante o intervención en presencia de inflamación
aguda sin pus. La respuesta es C

PTS: 3 DIF: alta


REF: Bather, GR: Direct observations of surgical wound infections al a comprehensive center. Arch
Surg 2003; 130:1042-1047.
OBJ: Tratar al paciente quirúrgico con diagnóstico con inpección del sitio operatorio
TOP: infección intra-abdominal KEY: herida quirúrgica
NOT: Dr. Vinicio Moreno Rueda
12. ANS: A
El Lavado peritoneal es un examen muy útil para el diagnóstico del trauma abdominal, incluso en el de
tipo abierto, en el que un contaje superior a los 10.000 glóbulos rojos por mm cúbico es indicativo de
positividad e indica que posiblemente se debe someter a cirugía al paciente. La respuesta es A

PTS: 3 DIF: alta


REF: Puylaert, JB; us of acute GI tract conditions. Eur. Radiol. 2001; 11:1167-77.
OBJ: Tratar al paciente quirúrgico con diagnóstico de abdomen agudo
TOP: Andomen agudo KEY: lavado peritoneal diagnóstico
NOT: Dr. Vinicio Moreno Rueda
13. ANS: A
En el eco no se evidencia engrosamiento de la pared vesicular, el diámetro transverso está en el tamaño
considerado normal, lo que se evidencia es la presencia de cálculos. Es una paciente no emergente que
debe ser aliviada su molestia y programada por consulta externa la colecistectomía. La respuesta es A

PTS: 3 DIF: alta


REF: Pérez, B; Abdomen Agudo: Urgencias Quirúrgicas 2da edición, Indugraf, ISBN 9789942035042,
2011. OBJ: Tratar al paciente quirúrgico con diagnóstico de patología de vesícula biliar
TOP: Vesícula, vía biliar y páncreas KEY: ecosonografía de vesícula
NOT: Dr. Vinicio Moreno Rueda

3
ID: A

14. ANS: D
El mejor esquema seguro y sin resistencia alta es el dado por las cefalosporinas de tercera generación por
su efecto contra bacteria gram negativas tipo E. coli más metronidazol con su cobertura para anaerobios
con el B frágilis. L respuesta es D

PTS: 3 DIF: alta


REF: Molly, A; Amebic liver absces infectous disease clinics of North america Vol 14 Num 3 set 2000.
OBJ: Tratar al paciente quirúrgico con diagnóstico de infección intra-abdominal y sus complicaciones
TOP: Infección intra-abdominal KEY: peritonitis, antibiótico
NOT: Dr. Vinicio Moreno Rueda
15. ANS: B
La hernia inguinal indirecta aparece por el orificio inguinal profundo, a menudo aumentado de tamaño y
baja junto con los elementos del cordón espermático dentro de su saco herniario dentro del conducto
inguinal. La respuesta es B

PTS: 3 DIF: alta


REF: Wantz, G; Hernias de la Pared Abdominal: Princios de Cirugía, 6ta edición, cap. 34, Ed.
Interamericana - McGraw-Hill.
OBJ: Tratar al pacente quirúrgico diagnosticado con hernia de la pared abdominal
TOP: Hernias de la pared abdominal KEY: hernia inguinal
NOT: Dr. Vnicio MOreno Rueda

4
CIRUGIA [Answer Strip] ID: A

C
_____ 6. C 11.
_____

B
_____ 1.

A 12.
_____
B
_____ 7.

D
_____ 2.

A 13.
_____
D
_____ 8.

C
_____ 3.

B
_____ 9. D 14.
_____

D
_____ 4.

B 15.
_____

C
_____ 5.
D 10.
_____
Name: ________________________ Class: ___________________ Date: __________ ID: A

Reactivos Ginecología y Obstetricia- Criollo A.

Multiple Choice
Identify the choice that best completes the statement or answers the question.

____ 1. Seleccione la respuesta correcta entre los planteamientos que se relacionan a continuación
sobre Preeclampsia.

a. El tratamiento con sulfato de magnesio c. El sulfato de magnesio es el


se administra solamente en la medicamento anticonvulsivante de
preeclampsia leve. elección en la preeclampsia.

b. La preeclampsia severa se caracteriza


solamente por presentar TA: mayor o
igual a 160/110 mmHg.

____ 2. Seleccione el orden correcto en que se realiza el manejo activo del tercer periodo del parto
(MATEP). a) Masaje uterino, b) Tracción y contratracción controlada del cordón,
c)Adminidtración de 10 UI de oxitocina IM dentro del primer minuto, , d)Verificar que no
haya otro bebé, .

a. d, c, b, a, c. a,c,d,b

b. d,a,c,b d. d,c,a,b

____ 3. Seleccione la opción correcta acerca del tratamiento farmcológico en los trastornos
hipertensivos del embarazo en nuestro medio .

a. Nimodipino c. Nifedipino

b. Labetalol d. Hidralazina

1
Name: ________________________ ID: A

____ 4. Seleccione la opción en la que se refleja el tratamiento correcto de la Ruptura prematura de


membranas.

1. Ampicilina 2 gramos IM dosis inicial, seguido de 1 gramo cada 6 horas + Eritromicina 250
miligramos vía oral cada 6 horas por 48 horas.

2. Ampicilina 2 gramos IM dosis inicial, seguido de 1 gramo cada 6 horas + Eritromicina 250
miligramos vía oral cada 8 horas por 48 horas.

3. Ampicilina 2 gramos IV dosis inicial, seguido de 1 gramo IV cada 6 horas + Eritromicina


250 miligramos vía oral cada 6 horas por 48 horas.

4. Ampicilina 2 gramos IV dosis inicial, seguido de 1 gramo IV cada 6 horas + Eritromicina


250 miligramos vía oral cada 8 horas por 48 horas.

5. Amoxacilina 500 miligramos vía oral cada 6 horas + Eritromicina 250 miligramos vía oral
cada 8 por 5 días.

6. Amoxacilina 500 miligramos vía oral cada 8 horas + Eritromicina 250 miligramos vía oral
cada 6 horas por 5 días.

a. 1,6 c. 3,6

b. 3,5 d. 4,5

____ 5. Seleccione la opción que muestra los planteamientos verdaderos Con relación a la clínica del
parto. Responda V o F, según corresponda.
A___ Una gestante esta en trabajo de parto cuando se constata dilatación cervical de 5cm

B___ La duración de la contracción debe ser de 20-25 segundos.

C___ El cuarto tiempo o movimiento cardinal del parto es aquel donde se produce el
desprendimiento de la cabeza y encajamiento de los hombros.

D___ En el segundo tiempo del mecanismo de parto en cefálico es cuando ocurre el


encajamiento de la cabeza en la excavación pélvica.

E___ El periodo de alumbramiento comienza con la salida de la cabeza del feto

a. B,C,E c. A,C,E

b. A,C,D d. C,D,E

2
Name: ________________________ ID: A

____ 6. Seleccione el planteamiento que considere correcto acerca de la hemorragia posparto.

a. La HPP es la pérdida de sangre que c. La HPP primaria es aquella que se


supera los 500 ml en un parto vaginal y produce entre las 24 horas y seis
que supera los 1.000 ml en un parto por semanas posparto.
cesárea.

b. La HPP secundaria es aquella que se


produce entre las 12 y 24 horas
posparto.

____ 7. Seleccione la opción que muestra los planteamientos correctos acerca de la clasificación de los
trastornos hipertensivos..

1. En la Hipertensión crónica encontramos una TAS = 140 mmHg y/o TAD = 90 mmHg
presente antes del embarazo, o después de las 20 semanas de gestación, o que persiste
después de las 12 semanas del posparto y ausencia de proteinuria (excepto si hay lesión
renal).

2. En la Eclampsia se produce el desarrollo de convulsiones tónico - clónicas generalizadas


y/o coma en mujeres con preeclampsia durante el embarazo, parto o puerperio, no
atribuible a otras patologías o condiciones neurológicas.

3. La Hipertensión gestacional se caracteriza por presentar TAS = 140 mmHg y/o TAD = 90
mmHg, presente antes de las 20 semanas de gestación y ausencia de proteinuria.

4. La Preeclampsia con signos de gravedad (grave), se caracteriza por presentar TAS = 160
mmHg y/o TAD = 110 mmHg y/o uno o más criterios de gravedad y/o afectación de órgano
blanco.

5. La Preeclampsia sin signos de gravedad (leve) se caracteriza por presentar TAS = 140 mm
Hg y menor (<) 160 mmHg y/o TAD = 90 mmHg y < 110 mmHg más proteinuria y sin
criterios de gravedad ni afectación de órgano blanco.

a. 1, 2, 5 c. 2, 3, 4

b. 2, 4,5

____ 8. Seleccione el orden de frecuencia correcto de las causas de HPP.

a- Tono: atonía uterina.

b- Tejido: retención de placenta o coágulos.

c- Trauma: lesión cervical o vaginal, ruptura uterina.

d- Trombina: coagulopatía preexistente.

a. a, c, b,d c. b, a,c,d

b. a, b, c, d

3
Name: ________________________ ID: A

____ 9. Cuáles son los puntos de corte máximos de la Prueba de tolerancia oral a la glucosa (PTGO)
con 75 gramos de glucosa.

a. Basal: menor a 92 mg/dL c. Basal: menor a 92 mg/dL


Primera hora: menor a 183 mg/dL Primera hora: menor a 153 mg/dL
Segunda hora: menor a 150 Segunda hora: menor a 180
mg/dL mg/dL
b. Basal: menor a 92 mg/dL
Primera hora: menor a 180 mg/dL
Segunda hora: menor a 153
mg/dL

____ 10. A) Las etiología de la ruptura prematura de membranas es ------------------------------, el tratamiento con
antibióticos se inicia a ---------------------------------- y cuando la paciente acude con ruptura prematura
de membranas ovulares está
---------------------- realizarle un tacto vaginal, si niega la existencia de dolores y comprobamos la
ausencia de contracciones.

a. unifactorial – 24 horas - c. multifactorial - 12 horas - indicado -


contraindicado

b. multifactorial - 24 horas - d. multifactorial - 12 horas -


contraindicado - contraindicado -

____ 11. ELIJA LA OPCIÓN CON LAS PALABRAS QUE COMPLETAN CORRECTAMENTE LA SIGUIENTE
PROPOSICIÓN: La ------------------------------- ,es la presencia de bacterias en orina detectada
por -------------------------------------- sin síntomas típicos de infección aguda del TU, su
tratamiento de elección es con -----------------------------.

a. cistitis- elemental y microscópico de c. bacteriuria asintomática - urocultivo -


orina - cefalexina nitrofurantoína.

b. pielonefritis - urocultivo - d. cistitis- urocultivo - nitrofurantoína


nitrofurantoína

4
Name: ________________________ ID: A

la respuesta correcta es la c, ya que el sangrado que se presenta en esta patología es característico,


aparece de forma súbita, generalmente no hay afectación de la frecuencia cardiaca fetal, lo que lo
diferencia de la rotura uterina y rotura de vasa previa es que aunque el sangrado es similar, generalmente
en la rotura uterina hay antecedente de cicatriz uterina o se constata una hiperdinamia uterina.

____ 12. 12.


Paciente de 36 años, secundigesta, antecedentes de salud aparente, cursa embarazo de 34, 1
semanas por FUM. Acude a su SCS refiriendo que de forma sintió salida abundante de sangre
roja rutilante de sus “partes” mientras estaba acostada sin otros síntomas acompañantes,
movimientos fetales presentes. Al examen físico se constata signos vitales dentro de
parámetros normales, FCF: 136 lat/mit, actividad uterina negativa. NST reactivo.

a. Rotura uterina c. Placenta previa


b. Desprendimientoplacentario

____ 13. 13.Paciente de 41 años de edad, antecedentes de salud aparente, último parto hace 12 años; cursa embarazo
de 14,6 semas por ECo, FUM imprecisa. Usted la valora por primera vez y realiza el primer control
prenatal en su área de salud. Analice cuál sería la conducta adecuadacon esta paciente?.
1. Anamnesis, examen físico completo y solicitar exámenes.
2. Anamnesis, examen físico completo y transferencia.
3. Iniciar tratamiento con 1.5 gde calcio elemental dividido en 3 tomas despúes de las comidas y
Ácido aceltil salicílico 75-100 mg diario.
4. Transferencia a hospital de tercer nivel.

a. 1,4 c. 2
b. 1, 3 d. 3,4

____ 14. En una gestante que cursa embarazo de 32 semanas, se constata actividad uterina: 2/10/30 segundos que
además refiere dolor abdominal en hipogastrio de 6 horas de evolución, se le realiza tacto vaginal
encontrándose un cérvix posterior, borrado 30%, dilatado 2 cm, cefálico móvil, membranas íntegras;
FCF: 128.145 lat/mit. Señale cuál sería el tratamiento indicado en esta paciente.

a. 1. Betametasona 12 mg IM cada 24 c.
horas, dos dosis 3.Dexametasona: 6 mg IM cada 12 horas
Nifedipino: 10 mg SL cada 20 minutos por dos dosis.
por 3 dosis, luego: 20 mg SL cada 8 Nifedipino 10 mg VO cada 20 minutos
horas. por 3 dosis, luego 20 mg VO cada 6
horas.
b. 2. Betametasona 12 mg IM cada 24 d. 4. Dexametasona: 6 mg IM cada 12 horas
horas, dos dosis. por 4 dosis.
Nifedipino 10 mg VO cada 20 minutos Nifedipino: 10 mg Vo si TA mayor o
por 3 dosis, luego 20 mg VO cada 6 igual a 160 y/o 110 mmHg.
horas.

5
Name: ________________________ ID: A

____ 15. Usted valora a paciente con un embarazo de 42.3semanas calculadas por FUM, que presenta una altura
uterina de 31 cm, para decidir una conducta solicita realizar un ECO más perfil biofísico. De las
opciones siguientes seleccione la que considere correcta respecto a los parámetros que mide en perfil
bioísiíco.

a. 1. Movimientos corporales, movimientos c. 3. Movimientos fetales, movimientos


respiratorios,, reactividad cardiaca, respiratorios, tono fetal, reactividad
líquido amniótico, tono fetal. cardiaca, líquido amniótico.
b. 2. Movimientos fetales, movimientos d.
corporales, tono fetal, frecuencia 4. Movimientos corporales, movimientos
cardiaca fetal. Indice de líquido respiratorios, tono fetal, reactividad
amniótico. cardiaca. Indice de líquido amniótico.

6
ID: A

Reactivos Ginecología y Obstetricia- Criollo A.


Answer Section

MULTIPLE CHOICE

1. ANS: C
La respuesta correcta es la c. puesto que el sulfato de magnesio es el medicamento propuesto como de
primera elección en el tratamiento de las convulsiones en el embarazo.
La pregunta puede ser respondida empleando entre 1 y 3 minutos.

PTS: 1 DIF: Alto


REF: MSP (2016) Transtornos hipertensivos del embarazo. Guía de práctica clínica, segunda edición.
OBJ: Reconoce y trata las principales enfermedades maternas que complican el embarazo basados en
conocimientos actualizados permitiéndole manejar a las pacientes obstétricas con responsabilidad y ética.
TOP: Obstetricia. KEY: Preeclampsia MSC: Anabela Criollo Criollo
2. ANS: A
La respuesta correcta es la a. ya que esos son los pasos subsecuentes y ordenados a seguir para evitar
complicaciones en la tercera etapa del trabajo de parto, para su solución se deben emplear entre 1 y 3
minutos.

PTS: 1 DIF: Alta REF: MSP (2012) Componente Normativo Materno


Neonatal.
OBJ: Comprende, aplica y trata las etapas del parto que complican el embarazo basados en
conocimientos actualizados permitiéndole manejar a las pacientes obstétricas con responsabilidad y ética.
TOP: Obstetricia KEY: Trabajo de parto NOT: Anabela Criollo Criollo
3. ANS: C
La respuesta corecta es la c. ya tanto el labetalol como la nifedipina se recomiendan como primera línea
por ser igual de eficaces; pero, el labetalol no consta en la lista de medicamentos escenciales, no hay en el
país. Se considera que se requerirán de 1 a 3 minutos para su respuesta.

PTS: 0 DIF: Alta


REF: MSP (2016) Transtornos hipertensivos del embarazo. Guía de práctica clínica, segunda edición.
OBJ: Reconoce y trata las principales enfermedades maternas que complican el embarazo basados en
conocimientos actualizados permitiéndole manejar a las pacientes obstétricas con responsabilidad y ética.
TOP: Obstetricia KEY: Trastornos hipertensivos del embarazo.
NOT: Anabela Criollo Criollo.
4. ANS: C
La respuesta correcta es la c. porque estos planteamientos son los recomendados como esquema de
profilaxis antibiótica de primera línea por 7 días si no aparecen complicaciones que indiquen interrupción
inmediata del embarazo.
Se consideran de 1 a 3 minutos suficientes para responder la pregunta.

PTS: 1 DIF: Alta


REF: MSP, (2015). Ruptura prematura de membranas pretérmino. Guía de práctica clínica
OBJ: Comprende, analiza y trata las enfermedades que complican el embarazo basados en
conocimientos actualizados permitiéndole manejar a las pacientes obstétricas con responsabilidad y ética.
TOP: Obstetricia KEY: Ruptura prematura de membranas NOT: Anabela Criollo criollo.

1
ID: A

5. ANS: B
La respuesta correcta es la opción b. porque estas son los movimientos cardinales o mecanismos del parto
realizados por el bebé y descritos en la literatura.
Se considera que la respuesta puede ser ofrecida entre 1 y 3 minutos.

PTS: 1 DIF: Alta REF: MSP, (2012) Componente Normativo Materno


Neonatal.
OBJ: Comprensión conceptual, analiza y trata las enfermedades que complican el embarazo y parto
basados en conocimientos actualizados permitiéndole manejar a las pacientes obstétricas con
responsabilidad y ética.
TOP: Obstetricia KEY: Trabajo departo NOT: Anabela criollo Criollo
6. ANS: A
La respuesta correcta es la a. Es la definición correcta de la pérdida sanguinea esperada y considerada
como normal.
Se considera que la respuesta puede ser dada en un tiempo entre 1 y 3 minutos.

PTS: 1 DIF: Alta


REF: MSP; (2013). Prevención, diagnóstico y tratamiento de la hemorragia posparto. Guía de práctica
clínica.
OBJ: Aplicación conceptual de las principales enfermedades maternas que complican el embarazo y
parto basados en conocimientos actualizados permitiéndole manejar a las pacientes obstétricas con
responsabilidad y ética. TOP: Obstetricia. KEY: Hemorragia posparto.
NOT: Anabela Criollo Criollo.
7. ANS: B
La respuesta correcta es la b. porque es la que recoge las características más aceptadas por la comunidad
científica acerca de la clasificación actual de los trastornos hipertensivos.
El estudiante contará con entre 1 y 3 minutos para contestarla.

PTS: 1 DIF: Alta


REF: MSP, (2016) Transtornos hipertensivos del embarazo. Guía de práctica clínica, segunda edición.
OBJ: Aplicación conceptual sobre la clasificaciónde las principales enfermedades maternas que
complican el embarazo basados en conocimientos actualizados permitiéndole manejar a las pacientes
obstétricas con responsabilidad y ética.. TOP: Obstetricia KEY: Trastornos
hipertensivos
NOT: Anabela Criollo criollo.
8. ANS: A
La opción correcta es la a. Ese es el orden de frecuencia descrito en la literatura médica y evidenciado en
la práctica.Se contará con enter 1 y 3 minutos para responder la pregunta.

PTS: 1 DIF: Alto


REF: MSP, (2013). Prevención, diagnóstico y tratamiento de la hemorragia posparto. Guía de práctica
clínica.
OBJ: Comprensión y aplicación conceptual de las principales enfermedades maternas que complican el
embarazo y parto basados en conocimientos actualizados permitiéndole manejar a las pacientes
obstétricas con responsabilidad y ética. TOP: Obstetricia. KEY: Hemorragia
posparto.
NOT: Anabela Criollo Criollo

2
ID: A

9. ANS: B
La respuesta correcta es la b. La GPC y el ADA 2016 describen esos valores como parámetros de corte
máximo de la PTGO, uno solo de ellos alterado nos orienta al diagnóstico de Diabetes Gestacional,
Se contará con entre 1 y 3 minutos para ofrecer la respuesta

PTS: 1 DIF: Alto


REF: MSP, (2014). Diagnóstico y tratamiento de la diabetes en el embarazo (pregestacional y
gestacional). Guía de práctica clínica.
OBJ: Comprensión conceptual, analiza y trata las enfermedades que complican el embarazo y parto
basados en conocimientos actualizados permitiéndole manejar a las pacientes obstétricas con
responsabilidad y ética.
TOP: Obstetricia. KEY: Diabetes y embarazo. NOT: Anabela Criollo Criollo.
10. ANS: D
La opción correcta es la d. porque es la terminología más aceptada, y el tratamiento profiláctico se inicia
a las 12 horas posteriores a la RPM confirmada. Se contará con entre 1 y 3 minutos para responder la
pregunta.

PTS: 1 DIF: Alto


REF: MSP, (2015). Ruptura prematura de membranas pretérmino. Guía de práctica clínica
OBJ: Aplicación de la terminología, comprende, analiza y trata las enfermedades que complican el
embarazo basados en conocimientos actualizados permitiéndole manejar a las pacientes obstétricas con
responsabilidad y ética. TOP: Obstetricia. KEY: Ruptura prematura de
membranas.
NOT: Anabela Criollo Criollo.
11. ANS: C
La respuesta correcta es la c. porque esta patología no tiene síntomas clínicos y el diagnóstico se hace a
traves de un urocultivo positivo que demuestre la presencia de más de 100.000 col de bacterias en orina y
el tratamiento de primera línea es con nitrofurantoina. Se contará con entre 1 y 3 minutos para dar
respuesta a la pregunta

PTS: 1 DIF: Alta


REF: MSP, (2013). Infección de vías urinarias en el embarazo. Guía de práctica clínica.
OBJ: Comprende, analiza y trata las enfermedades que complican el embarazo basados en
conocimientos actualizados permitiéndole manejar a las pacientes obstétricas con responsabilidad y
ética..
TOP: Obstetricia. KEY: Infección de vías urinarias. NOT: Anabela Criollo Criollo.
12. ANS: C
La respuesta correcta es la c, ya que el sangrado que se presenta en esta patología es característico,
aparece de forma insidiosa, generalmente no hay dolor ni afectación de la frecuencia cardiaca fetal y el
útero esta relajado, lo que lo diferencia de la rotura uterina y del desprendimiento placentario.

PTS: 1 DIF: Alta REF: MSP, (2012) Componente Normaivo materno


neonatal
OBJ: Analiza los signos y síntomas, utilizando el método clínico y realiza diagnóstico diferencial entre
las patologías que comprenden las hemorragias de la segunda mitad de la gestación.
TOP: obstetricia KEY: Hemorragias de la segunda mitad de la gestación
NOT: Anabela Criollo Criollo

3
ID: A

13. ANS: B
La respuesta correcta es la b, ya que en la primera consulta prenatal se valora y examina a la paciente de
forma integral, se solicitan todos los exámenes y se clasifica a paciente tomando en cuenta los factores de
riesgo que presente. En este caso tiene dos factores de riesgo moderado: tener más de 40 años y un
periodo intergenésico prolongado por lo que se inicia con calcio y aspirina a las dosis indicadas desde las
12 semanas de embarazo hasta el final de este como prevención primaria de la preeclampsia.

PTS: 1 DIF: Alta


REF: MSP (2016) Trastornos hipertensivos del embarazo. Guía de práctica clínica, segunda edición.
MSP (2016) Control Prenatal. Guía de Práctica clínica.
OBJ: Analiza y a aplica el método clínico que le permite valorar de forma integral y clasificar según el
riesgo a la paciente y realizar acciones de promoción y prevención de futuras complicaciones en el curso
del embarazo. TOP: Obstetricia KEY: Control prenatal y Trastornos hipertensivos del
emabarzo
NOT: Anabela Criollo Criollo
14. ANS: B
La respuesta correcta es la b, ya que la paciente tiene una amenaza de parto pretérmino y hay que iniciar
maduración pulmonar con corticoides, de elección la betametasona: 12 mg IM cada 24 horas por dos
dosis y, realizar tocólisis con nifedipina: 10 mg VO cada 20 minutos por 3 dosis, seguido de 20 mg VO
cada 6 horas.

PTS: 1 DIF: Alta REF: MSP (2012) Componente Normativo Materno


Neonatal.
OBJ: Analizar y llegar a un diagnóstico para saber cuál es la conducta adecuada a seguir.
TOP: Obstetricia. KEY: Amenaza de parto pretérmino NOT: Anabela criollo criollo.
15. ANS: D
La respuesta correcta es la d, ya que los cinco parámetros que mide el perfil biofísico son: movimiento
corporales, movimientos respiratorios, reactividad cardiaca, índice de líquido amniótico y tono fetal, y
esta indicado cuando sospechamos una restricción del crecimiento intrauterino.

PTS: 1 DIF: Alta REF: MSP (2012) Componente Normativo Materno


Neonatal.
OBJ: Conocer cuáles son los parámetros que mide el perfil biofísico para saberlo interpretar y tomar una
conducta acertada según su resultado. TOP: Obstetricia.
KEY: Restricción del crecimiento intrauterino. NOT: Anabela Criollo Criollo

4
Reactivos Ginecología y Obstetricia- Criollo A. [Answer Strip] ID: A

C
_____ 4. A
_____ 6. B
_____ 9.

C 12.
_____
C
_____ 1.

B
_____ 7.

A
_____ 2. D 10.
_____
B 13.
_____

B
_____ 5.
C
_____ 3.

C 11.
_____
B 14.
_____

A
_____ 8.
Reactivos Ginecología y Obstetricia- Criollo A. [Answer Strip] ID: A

D 15.
_____
Name: ________________________ Class: ___________________ Date: __________ ID: A

REACTIVOS HEMATOLOGIA 2020

Multiple Choice
Identify the choice that best completes the statement or answers the question.

____ 1. Paciente de 55 años, fumador con astenia reciente, incremento de la tos crónica y pérdida de peso, con estos
parámetros en sangre periférica: Hb 10.5 g/dl, leucocitos 11000/ul con 40% segmentados, 10% cayados,
5% metamielocitos, 4% mielocitos, 1% eosinófilos y 1% basófilos. Normoblastos 5%. Plaquetas
300000/il. Morfología de serie roja en sangre aniso-poiquilocitosis y dacriocitos. En exploración se palpa
una adenopatía supraclavicular derecha. ¿Qué diagnostico le sugieren estos datos?
a. Mielofibrosis primaria c. Metástasis de cáncer de pulmón
b. Linfoma no Hodgkin d. Leucemia mieloide crónica

____ 2. Mujer de 40 años que refiere astenia progresiva de meses de evolución, acompañada ocasionalmente de
mareos y palpitaciones. En la exploración física, presenta palidez como único hallazgo patológico. En el
hemograma aparece hemoglobina de 7 g/dl. La sideremia es de 20 ug/dl, concentración de transferrina
480 ug/dl, saturación de transferrina 6%. ¿Cuál es el diagnostico?
a. Anemia sideroblástica c. Anemia ferropénica
b. Talasemia intermedia d. Anemia de enfermedad crónica

____ 3. Mujer de 28 años, diagnosticada de artritis reumatoide, consultada por los siguientes hallazgos analíticos:
hemoglobina 8.5 g/dl, VCM 85 fl, bilirrubina normal, hierro sérico 10 ug/dl, capacidad de fijación total
de hierro 200 ug/dl, índice de saturación de transferrina 15% y ferritina 150 ug/dl. ¿Qué tipo de anemia
pensaría que tiene la paciente?
a. Anemia ferropénica c. Anemia hemolítica autoinmunitaria
b. Anemia inflamatoria (de proceso d. Rasgo talasémico
crónico)

____ 4.
Niño de 4 años, con antecedentes de leucemia aguda linfoblástica B hace un año. En su última
revisión, estaba en remisión completa y sigue tratamiento de mantenimiento con metotrexato y
mercaptopurina. Actualmente, presenta intensa astenia y palidez cutaneomucosa. En la bioquímica,
destaca LDH 2.730 UI/I y bilirrubina 1,7 mg/dl. En el hemograma, muestra 2,9 x 109 leucocitos/l con
recuento diferencial normal, hemoglobina 7,5 g/dl, hematocrito 22%, VCM 135 fL y 100 x 109
plaquetas/l. El recuento de reticulocitos es 1% y en el frotis de sangre periférica se observan
abundantes neutrófilos hipersegmentados. ¿Cuál sería su juicio diagnóstico más probable y su actitud
terapéutica inicial?
a. Recaída leucémica-alotransplante de c. Anemia megaloblástica por déficit de
progenitores hematopoyéticos cobalamina – vitamina B12
b. Anemia megaloblástica por déficit de d. Síndrome mielodisplásico secundario a
folato – ácido folínico oral. quimioterapia – alotransplante de
progenitores hematopoyéticos

1
Name: ________________________ ID: A

____ 5. Un hombre de 55 años, asintomatico, realiza su revisión rutinaria por diabetes mellitus. En la exploración
física de detecta esplenomegalia. El Hemograma muestra 45x10 9 leucitos /l, con neutrofilia, basofilia,
eosinofilia y presencia de formas inmaduras mieloides, hb 14 g/dl y 480x10 9 plaquetas /l. ¿Cuál sería el
estudio adecuado y el tratamiento si se confirma el resultado?

a. Reordenamiento BCR/ABL y vigilancia c. Reordenamiento BCR/ABL e Imatinib


periódica
b. Reordenamiento BCR/ABL y d. Mutaciones JAK 2 e Hidroxiurea
alotrasplante de progenitores
hematopoyéticos

____ 6. Paciente de 62 años, que acude al hospital remitido por hallazgo en una revisión rutinaria de un hemograma
con leucocitos 45000/ul, con 10% segmentados y 90% linfocitos de pequeño tamaño, núcleo redondeado
y cromatina grumosa, sin nucléolos. Hemoglobina 12 g/dl, plaquetas 190000/ul, reticulocitos 1.5%. El
paciente no relataba sintomatología alguna, salvo migrañas habituales. La exploración física no revelo
anomalías. Las pruebas bioquímicas básicas (incluyendo LDH) se encontraban en límites normales. Una
radiografía de tórax y una ecografía abdominal no mostraban alteraciones significativas. En la citometria
de flujo sanguínea, el fonotipo inmunológico delos linfocitos de sangre periférica eran Inmunoglobulinas
débilmente + (cadenas µ y lambda) CD19+, CD20+, CD5+, FMC7-, CD2-. El careotipo no
mostroanomalias citogenéticas en las fases estudiadas. ¿Cuál es el diagnostico?

a. Leucemia prolinfocitica fenotipo B c. Leucemia linfoide crónica


b. Tricoleucemia d. Linfoma centrofolicular leucemizado

____ 7. Paciente de 55 años que ingresa por deterioro reciente del estado general, acompañado de distensión y
molestias abdominales inespecíficas, en la exploración física, se objetivan múltiples adenopatías y
abdomen distendido y con molestia difusa a la palpación, con dudosa ascitis. El Hemograma es el
siguiente: leucocitos 16000/ul, hemoglobina 8 g/dl, plaquetas 25000 /ul, VSG 100 mm/h. La LDH sérica
es de 1500 UI/l. La biopsia ganglionar muestra una proliferación difusa de células de mediano tamaño no
hendidas, con un citoplasma intensamente basófilo con vacuolas, CD19 y CD20 positivas, y
reordenamiento de protooncogen C-MYC. De entre las siguientes entidades, ¿Cuál es el diagnóstico más
probable?

a. Linfoma Folicular c. Linfoma de Burkitt


b. Linfoma MATL gástrico d. Linfoma de células de manto

____ 8. Un paciente de 25 años, inmigrante de la región del Caribe, ingresa por deterioro general acompañado de la
aparición reciente de lesiones cutáneas y dolores óseos. En la exploración se aprecian además múltiples
adenopatías y esplenomegalias. En el hemograma: Leucocitos 35000/ul, hb 8 g/dl, plaquetas 80000 /ul,
con frotis con células de núcleo polilobulado. LDH sérica 850 UI/l, calcemia 12.5 mg/dl. ¿Cuál es el
diagnostico mas probable?

a. Leucemia/Linfoma T del adulto c. Linfoma de Hodgkin


b. Mieloma múltiple d. Linfoma de Burkitt

2
Name: ________________________ ID: A

____ 9. Una mujer de 32 años consulta porque tiene menstruaciones muy abundantes, de duración normal, y
porque las heridas le sangran mas de lo habitual. Como antecedente refiere que, 2 años antes, tuvo una
hemorragia posparto copiosa que retrasó el alta hospitalaria. Como antecedente familiar, destaca que su
abuela materna también tuvo hemorragias posparto inusuales. El hemograma fue completamente normal y
el tiempo de hemorragia se prolongó hasta 10 minutos, estando también ligeramente prolongado el
tiempo de tromboplastina (TTPA) ¿Cuál es el trastorno de coagulación que padece?

a. Disfibrinogenemia c. Enfermedad de von Willebrand tipo II


b. Enfermedad de von Willebrand tipo I d. Hemofilia A

____ 10. Una mujer de 42 años, sin antecedentes patológicos significativos, acude a Urgencias con cuadro de
cefaleas y disminución del nivel de consciencia, asociado a un cuadro de petequias y equimosis. En el
hemograma se objetiva una Hb de 8 g/dl y plaquetas de 30000/mm3, reticulocitos 200000/ul, con
abundantes esquistocitos en el frotis, junto con datos de insuficiencia renal. ¿Cuál sería el tratamiento
más adecuado con la sospecha diagnostica?

a. Esteroides y gammaglobulina c. Recambio plasmático y transfusión de


intravenosa plaquetas
b. Esteroides, gammaglobulina intravenosa d. Recambio plasmático y esteroides
y transfusión de plaquetas

____ 11. Una paciente de 60 años, en tratamiento con heparina no fraccionada en perfusión intravenosa por
tromboembolismo pulmonar múltiple, con buena evolución hemodinámica y respiratoria, presenta al
octavo dia de tratamiento una disminución de la cifra de plaquetas. Se confirma además la presencia de
anticuerpos IgG contra el complejo heparina-factor4 plaquetario. ¿Cuál es el tratamiento más adecuado?

a. Cambiar a heparina de bajo peso c. Cambiar a dabigatran


molecular
b. Cambiar a dicumarínicos d. Cambiar a fondaparinux

3
ID: A

REACTIVOS HEMATOLOGIA 2020


Answer Section

MULTIPLE CHOICE

1. ANS: C PTS: 1 DIF: ALTA


REF: FARRERAS ROZMAN MEDICINA INTERNA EDICION 18
OBJ: ANALIZA LOS SINTOMAS Y SIGNOS PARA SU DIAGNOSTICO Y TRATAMIENTO
TOP: MEDICINA INTERNA - HEMATOLOGIA NOT: DR. HIPOLITO PAULA
2. ANS: C PTS: 1 DIF: ALTA
REF: FARRERAS ROZMAN MEDICINA INTERNA EDICION 18
OBJ: ANALIZA LOS SINTOMAS Y SIGNOS PARA SU DIAGNOSTICO Y TRATAMIENTO
TOP: MEDICINA INTERNA - HEMATOLOGIA NOT: DR. HIPOLITO PAULA
3. ANS: B PTS: 1 DIF: ALTA
REF: FARRERAS ROZMAN MEDICINA INTERNA EDICION 18
OBJ: ANALIZA LOS SINTOMAS Y SIGNOS PARA SU DIAGNOSTICO Y TRATAMIENTO
TOP: MEDICINA INTERNA - HEMATOLOGIA NOT: DR. HIPOLITO PAULA
4. ANS: B PTS: 1 DIF: ALTA
REF: FARRERAS ROZMAN MEDICINA INTERNA EDICION 18
OBJ: ANALIZA LOS SINTOMAS Y SIGNOS PARA SU DIAGNOSTICO Y TRATAMIENTO
TOP: MEDICINA INTERNA - HEMATOLOGIA NOT: DR. HIPOLITO PAULA
5. ANS: C PTS: 1 DIF: ALTA
REF: FARRERAS ROZMAN MEDICINA INTERNA EDICION 18
OBJ: ANALIZA LOS SINTOMAS Y SIGNOS PARA SU DIAGNOSTICO Y TRATAMIENTO
TOP: MEDICINA INTERNA - HEMATOLOGIA NOT: DR. HIPOLITO PAULA
6. ANS: C PTS: 1 DIF: ALTA
REF: FARRERAS ROZMAN MEDICINA INTERNA EDICION 18
OBJ: ANALIZA LOS SINTOMAS Y SIGNOS PARA SU DIAGNOSTICO Y TRATAMIENTO
TOP: MEDICINA INTERNA - HEMATOLOGIA NOT: DR. HIPOLITO PAULA
7. ANS: C PTS: 1 DIF: ALTA
REF: FARRERAS ROZMAN MEDICINA INTERNA EDICION 18
OBJ: ANALIZA LOS SINTOMAS Y SIGNOS PARA SU DIAGNOSTICO Y TRATAMIENTO
TOP: MEDICINA INTERNA - HEMATOLOGIA NOT: DR. HIPOLITO PAULA
8. ANS: A PTS: 1 DIF: ALTA
REF: FARRERAS ROZMAN MEDICINA INTERNA EDICION 18
OBJ: ANALIZA LOS SINTOMAS Y SIGNOS PARA SU DIAGNOSTICO Y TRATAMIENTO
TOP: MEDICINA INTERNA - HEMATOLOGIA NOT: DR. HIPOLITO PAULA
9. ANS: B PTS: 1 DIF: ALTA
REF: FARRERAS ROZMAN MEDICINA INTERNA EDICION 18
OBJ: ANALIZA LOS SINTOMAS Y SIGNOS PARA SU DIAGNOSTICO Y TRATAMIENTO
TOP: MEDICINA INTERNA - HEMATOLOGIA NOT: DR. HIPOLITO PAULA
10. ANS: D PTS: 1 DIF: ALTA
REF: FARRERAS ROZMAN MEDICINA INTERNA EDICION 18
OBJ: ANALIZA LOS SINTOMAS Y SIGNOS PARA SU DIAGNOSTICO Y TRATAMIENTO
TOP: MEDICINA INTERNA - HEMATOLOGIA NOT: DR. HIPOLITO PAULA
11. ANS: D PTS: 1 DIF: ALTA
REF: FARRERAS ROZMAN MEDICINA INTERNA EDICION 18
OBJ: ANALIZA LOS SINTOMAS Y SIGNOS PARA SU DIAGNOSTICO Y TRATAMIENTO
TOP: MEDICINA INTERNA - HEMATOLOGIA NOT: DR. HIPOLITO PAULA

1
REACTIVOS HEMATOLOGIA 2020 [Answer Strip] ID: A

C
_____ 5. B
_____ 9.

C
_____ 1.

C
_____ 6. D 10.
_____

C
_____ 2.

B
_____ 3.
D 11.
_____
C
_____ 7.

B
_____ 4.

A
_____ 8.
Name: ________________________ Class: ___________________ Date: __________ ID: A

Psiquiatria II

Multiple Choice
Identify the choice that best completes the statement or answers the question.

____ 1. En el caso clínico que exponemos a continuación diga qué tipo de esquizofrenia presenta
este paciente.
Paciente RMG, 30 años, masculino, leptosómico, casado, arquitecto con antecedentes
familiares psiquiátricos dados por un hermano y la madre con ingresos hospitalarios y
tratamiento de terapia electroconvulsiva. Desde hace unos dos meses se ha visto
distraído en su trabajo, prácticamente no habla con nadie y ha descuidado su aseo y
vestuario; parece como si nada le interesara. En días pasados le comunico a su jefe en
forma descompuesta que él tenía derecho a vivir su vida en privado y que no permitía
que hicieran públicos sus pensamientos a través de la televisión. La víspera del ingreso
todos se alarmaron cuando fue al trabajo con un casco protector de motociclista
refiriendo que lo tenía forrado de plomo por dentro. En la consulta vemos un paciente
con barba de varios días, que rechaza la entrevista con el argumento que no está
enfermo. Dice que todo cuanto le pasa es fruto de los aparatos que controlan sus ideas
y le imponen pensamientos extraños ¨que escucha dentro de su cabeza en forma de
ecos¨. Sabe que todo cuanto hace es ajeno a su voluntad y no puede pensar cosas
privadas porque después las difunden por la televisión. Esto lo expresa sonriendo
inadecuadamente. Los familiares refieren que antes de enfermarse era un joven
sociable, amistoso y de magnifico carácter. No se evidencian factores ambientales
significativos.

a. Esquizofrenia Paranoide c. Esquizofrenia Residual


b. Esquizofrenia Hebefrenica d. Esquizofrenia Indiferenciada

1
Name: ________________________ ID: A

____ 2. - Paciente AMJ femenina, 17 años, leptosómica soltera, secretaria, segundo curso de
bachillerato, su madre falleció en un hospital psiquiátrico y tiene dos ríos con historia de
ingresos por trastornos mentales. Ella siempre fue una joven seria, estudiosa y
apegada a la familia. Sin embargo, desde los 17 años se presentó un cambio
importante en su comportamiento, comenzó a no interesarse por la escuela y, por
último, dejo de asistir, refiriendo que iba a trabajar. Por esos días comenzó a llegar
tarde al hogar contrariamente a su costumbre, en varias ocasiones durmió fuera y
expresaba que estaba con unas amigas. La familia la notaba descuidada en su aseo,
aunque muy interesada por maquillarse y vestirse en forma algo provocativa. Comenzó
a tener muchos enamorados y hablaba de ellos en forma despectiva, a veces con
palabras obscenas.
Hace unos meses ha estado muy rara, se ríe por gusto, a veces habla cosas sin
sentido, abandono el trabajo porque sus compañeros eran¨ muy pesados¨.
Últimamente hace bromas de mal gusto, y la manifestación que decidió a su familia a
traerla al psiquiatra es que insinuó sexualmente a su hermano. En la entrevista vemos
a una paciente exageradamente maquillada, en actitud seductora y mantiene una risa
inadecuada y pueril que intensifica después de bromas de mala gusto. Doctor, su nariz
es más grande que la de un elefante¨Su comunicación es muy desorganizada, a veces
difícilmente comprensible.

a. Esquizofrenia Paranoide c. Esquizofrenia residual


b. esquizofrenia Hebefrenica d. Esquizofrenia Indiferenciada

____ 3. Paciente ABD, de 40 años, leptosómico, 17 años, soltero, estudiante de bachillerato, con
referencia de varios familiares con ingresos psiquiátricos. Desde hace unos 3 meses se
le comenzó a notar muy pensativo y alejado de sus amistades y familiares por
momentos parecía que estaba en otro mundo ¨dejó de preocuparse por su aseo y
vestuario y frecuentemente se le observaba mirando por las persianas de la casa. Se
armó de un machete que guardaba debajo de la almohada y decía que había que
dormir con un ojo abierto. Los familiares lo han visto hablando solo, como si contestara
a personas imaginarias y ha dicho a su familia que por momentos se nota los brazos
más gruesos. Se ha negado a comer en la casa porque piensa que su madre lo quiere
envenenar, la familia aprecia que nada le interesa ni lo hace reaccionar.
En días pasados falleció su abuela y durante el sepelio se le veía sonriendo y
conversando solo. Siempre fue un joven solitario y reservado, pero ahora estas
características se han exagerado. No se evidencias contingencias ambientales
actuales.

a. Esquizofernia Residual c. Esquizofrenia Paranoide


b. Esquizofernia Hebefrenica d. Esquizofrenia Indiferenciada

2
Name: ________________________ ID: A

____ 4. Paciente RGB de 46 años, desocupada, con historia familiar de cuadros de excitación
repetidos en el padre, por lo que ha estado ingresado, 2 tíos maternos con intentos
suicidas y 1 abuelo con cuadros depresivos a repetición. Acude a consulta
acompañada de su hija, quien nos refiere que en el último mes ha notado a su madre
muy triste, alejada de todas sus amistades, desinteresada en las tareas del hogar, y
descuidada en su aseo y arreglo personal. Se le ha visto frecuentemente llorando y
refiere que tiene una mala enfermedad, ha dicho que preferiría estar muerta; en la
noche se duerme muy temprano, pero a las 3:00 am. Se despierta y ya no puede
conciliar el sueño.
Dice que ha sido una mujer mala y que merece lo que está pasando. Señala que es un
estorbo para su familia y reporta que escucha a sus vecinas en conversaciones en las
que se alegran de su padecimiento. Refiere escuchar voces desconocidas que le dicen
¨matate¨, prácticamente no come y ha pedido 20 libras de peso. En la entrevista vemos
a una paciente con notable tristeza que habla y se mueve muy pausadamente,
comunica que ha perdido la capacidad para disfrutar y ve su futuro con pesimismo.

a. Trastorno bipolar episodio actual c. Trastorno bipolar episodio actual


hipomaniaco maniaco sin sintomas psicoticos
b. Trastorno bipolar episodio depresivo d. Trastorno Bipolar episodio actual
grave con sintomas psicoticos depresivo leve o moderado

____ 5. Paciente ADU de 32 años, soltero, trabaja con su padre en la agricultura, segundo grado
de escolaridad, repitió primero y segundo grado de escolaridad. Nació de un parto
institucional eutócico, a los 6 meses presento un cuadro de hipertermia severa y
convulsiones que requirió tratamiento institucional por 2 meses y fue diagnosticada una
meningitis bacteriana. Luego de su alta se apreció importante retraso para caminar y
hablar, presentó incontinencia vesical hasta los 20 años. Su vocabulario es de
aproximadamente un niño de 6 años. Desempeña su trabajo adecuadamente, cuando
se le orienta que hacer.

a. Retraso Mental Severo c. Retraso Mental Moderado


b. Retraso Mental Leve d. Retraso Mental Profundo

3
ID: A

Psiquiatria II
Answer Section

MULTIPLE CHOICE

1. ANS: A PTS: 1 DIF: Alta


REF: Kaplan y Sadock. Manual de bolsillo de Psiquiatria Clinica. 5ta Edicion España.,Lippincott
Willians and Wilkins, Wolters Kluwer Health 2015
OBJ: Diagnostica temapranamente da tratamiento oportuno y adecuado para su nivel de complejidad,
tanto farmacologico y psicologico basico
TOP: Psiaquiatria II- Psicosis organicas o somaticas y funcionales o psicogenas
2. ANS: B PTS: 1 DIF: Alta
REF: Kaplan y sadock. Manual de bolsillo de Psiquiatria . 5ta edicion España, Lippincott Willians and
Wilkinins, woltwers Kluwer Health 2015
OBJ: Diagnostica tempranamente, da tratamiento oportuno y adecuado para su nivel de complejidad,
tanto farmacologico psicologico basico
TOP: PsiquiatriaII- Psicosis organicas o somaticas funcionales o psicogenas
3. ANS: C PTS: 1 DIF: Alta
REF: Kaplan y Sadock. Masnual de bolsillo de Psiquiatria Clinica. 5ta edicion España Lippincott
Willians and Wilkins, Wolters Kluwer Health 2015
OBJ: Diagnosticar tempranamente, da tratamiento oportuno y adecuado para su nivel de complejidad,
tanto faramacologico y psicologico basico
TOP: Psiquiatria II - Psicosis organicas o somaticas y funcionales o psicogenas
4. ANS: B PTS: 1 DIF: Alta
REF: Clasificacion Internacional de Enfermedades 10 version CIE-10
OBJ: Diagnostica tempranamente da tratamiento oportuno y adecuado para su nivel de complejidad,
tanto farmacologico y psicologico basico
TOP: Psiquiatria II Psicosis organicas o somaticas y funcionales o psicogenas
5. ANS: C PTS: 1 DIF: Alta
REF: Clasificacion Internacional de Enfermedades
OBJ: Diagnostica tempranamente , da tratamiento oportuno y adecuado para su nivel de complejiodad,
tanto farmacologico y psicologico basico TOP: Psiquiatria II - Trastoornos del desarrollo

1
Psiquiatria II [Answer Strip] ID: A

B
_____ 2. B
_____ 4.

A
_____ 1.

C
_____ 5.

C
_____ 3.
Name: ________________________ Class: ___________________ Date: __________ ID: A

Salud Mental

Multiple Choice
Identify the choice that best completes the statement or answers the question.

____ 1. Son causas de delirium, EXCEPTO:


a. Intoxicación por sustancias c. Abstinencia de sustancias
b. Exposición a eventos traumáticos d. Trastornos metabólicos
____ 2. La angustia patológica se distingue por una de las siguientes manifestaciones:
a. Ser fantasmagórica, c. Es adaptativa para el sujeto,
representación imaginaria de un especialmente en las crisis de
conflicto real y consciente angustia
b. Ser estereotipada, repetitiva, sin d. Ser anacrónica, pues lleva a
estar afincada en el carácter del revivir situaciones pasadas
sujeto
____ 3. Indique, ¿Cuál es la característica del delirium?
a. Síntomas disociativos c. Alteración de las funciones
ejecutivas
b. Presencia de alucinaciones d. Alteración de la atención y la
conciencia
____ 4. Seleccione la proposición que corresponde a un comportamiento suicida:
a. Padecer un trastorno mental c. Ausencia de planificación de
suicidio
b. Ausencia de intentos de suicidio d. Tener ideas o pensamientos de
como provocarse la muerte.
____ 5. Señale lo correcto con relación a los ataques de pánico:
a. Son de naturaleza exógena, c. La crisis de pánico se presenta
ocasionados por circunstancias con una situación determinada y
adversas o enfermedades estresante para el paciente
preexistentes
b. La desregulación neurovegetativa d. Son de naturaleza endógena, las
y neuroendocrina son circunstancias y el entorno son
responsables de los síntomas elementos secundarios
psíquicos
____ 6. Indique un síntoma que constituye parte de un ataque de pánico:
a. Broncoconstricción c. Diplopía
b. Sialorrea d. Opresión o malestar torácico

1
Name: ________________________ ID: A

____ 7. Los siguientes son criterios diagnósticos para delirium según DSM-IV, EXCEPTO:
a. La alteración se presenta en un c. La historia clínica y de las pruebas
largo período de tiempo (días o de laboratorio, demuestran que la
semanas) y tiende a fluctuar a lo alteración es efecto fisiológico
largo del día directo de una enfermedad
médica
b. Alteración de la conciencia, con d. Cambio en las funciones
disminución de la capacidad para cognoscitivas o una alteración
centrar, mantener o dirigir la perceptiva, no explicada por una
atención demencia previa o en desarrollo
____ 8. Seleccione la característica diferencial más significativa entre delirium y demencia:
a. Solamente el delirium cursa con c. Únicamente la demencia
síntomas psicóticos y de agitación incrementa la prevalencia con la
edad
b. Delirium se diagnostica solo en d. Delirium es un proceso agudo y
pacientes hospitalizados cursa con alteración de la
conciencia
____ 9. ¿Cuáles son los síntomas fundamentales de la esquizofrenia según el DSM-5?
a. Catatonía, verborrea e ilusiones c. Delirios, alucinaciones y habla
desorganizada
b. Acumulación, delirium bizarro y d. Ilusiones, pensamientos erróneos
atracones y tristeza
____ 10. Escoja la opción que define la conducta suicida.
a. Conducta autoinflingida y c. Familiares que amenazan sobre
potencialmente lesiva donde lesiones potencialmente lesivas
existe evidencia de que la debido a intentos anteriores de
finalidad es la terminación de la suicidio
propia vida
b. Comunicación verbal y no verbal d. Pensamientos sobre el suicidio e
sobre un plan suicida que se ideas potencialmente lesivas en
presenta con otra persona presencia de desórdenes
diagnosticada con depresión familiares actuales o antiguos
____ 11. Un paciente masculino de 34 años se encontraba jugando fútbol. En forma súbita
presenta palpitaciones intensas, ahogo, opresión torácica, mareos, náuseas, diaforesis
y sensación de muerte inminente. Se diagnostica ataque de pánico. Seleccione el
tratamiento que debe recibir inicialmente:
a. Zoplicona y Betahistina por vía c. Benzodiacepina y Pregabalina por
oral vía oral
b. Bolo endovenoso de 100 mi de d. Oxígeno por bigotera a 3 litros por
Dextrosa al 10 % minuto
____ 12. Indique un síntoma que constituye parte de un ataque de pánico:
a. Diplopía c. Opresión o malestar torácico
b. Broncoconstricclón d. Sialorrea

____ 13. Son causas de delirium, EXCEPTO:


a. Trastornos metabólicos c. Intoxicación por sustancias
b. Exposición a eventos traumáticos d. Abstinencia de sustancias

2
Name: ________________________ ID: A

____ 14. Indique, ¿Cuál es la característica del delirium?


a. Presencia de alucinaciones c. Alteración de la atención y la
conciencia
b. Síntomas disociativos d. Alteración de las funciones
ejecutivas
____ 15. La angustia patológica se distingue por una de las siguientes manifestaciones:
a. Ser anacrónica, pues lleva a c. Ser estereotipada, repetitiva, sin
revivir situaciones pasadas estar afincada en el carácter del
sujeto
b. Es adaptativa para el sujeto, d. Ser fantasmagórica,
especialmente en las crisis de representación imaginaria de un
angustia conflicto real y consciente
____ 16. Seleccione la proposición que corresponde a un comportamiento suicida:
a. Ausencia de intentos de suicidio c. Padecer un trastorno mental
b. Ausencia de planificación de d. Tener ideas o pensamientos de
suicidio como provocarse la muerte
____ 17. Los siguientes son criterios diagnósticos para delirium según DSM-IV, EXCEPTO:
a. La historia clínica y de las pruebas c. Alteración de la conciencia, con
de laboratorio, demuestran que la disminución de la capacidad para
alteración es efecto fisiológico centrar, mantener o dirigir la
directo de una enfermedad atención
médica
b. La alteración se presenta en un d. Cambio en las funciones
largo período de tiempo (días o cognoscitivas o una alteración
semanas) y tiende a fluctuar a lo perceptiva, no explicada por una
largo del día demencia previa o en desarrollo
____ 18. ¿Cuáles son los síntomas fundamentales de la esquizofrenia según el DSM-5?
a. Acumulación, delirium bizarro y c. Delirios, alucinaciones y habla
atracones desorganizada
b. Catatonía, verborrea e ilusiones d. Ilusiones, pensamientos erróneos
y tristeza
____ 19. Seleccione la característica diferencial más significativa entre delirium y demencia:
a. Únicamente la demencia c. Solamente el delirium cursa con
incrementa la prevalencia con la síntomas psicóticos y de agitación
edad
b. Delirium es un proceso agudo y d. Delirium se diagnostica solo en
cursa con alteración de la pacientes hospitalizados
conciencia
____ 20. Señale lo correcto en relación a los ataques de pánico:
a. Son de naturaleza endógena, las c. La crisis de pánico se presenta
circunstancias y el entorno son con una situación determinada y
elementos secundarios estresante para el paciente
b. La desregulación neurovegetativa d. Son de naturaleza exógena,
y neuroendocrina son ocasionados por circunstancias
responsables de los síntomas adversas o enfermedades
psíquicos preexistentes

3
Name: ________________________ ID: A

____ 21. Escoja la opción que define la conducta suicida


a. Conducta autoinflingida y c. Pensamientos sobre el suicidio e
potencialmente lesiva donde ideas potencialmente lesivas en
existe evidencia de que la presencia de desórdenes
finalidad es la terminación de la familiares actuales o antiguos
propia vida
b. Familiares que amenazan sobre d. Comunicación verbal y no verbal
lesiones potencialmente lesivas sobre un plan suicida que se
debido a intentos anteriores de presenta con otra persona
suicidio diagnosticada con depresión
____ 22. Un paciente masculino de 34 años se encontraba jugando fútbol. En forma súbita
presenta palpitaciones intensas, ahogo, opresión torácica, mareos, náuseas, diaforesis
y sensación de muerte inminente. Se diagnostica ataque de pánico. Seleccione el
tratamiento que debe recibir inicialmente:
a. Benzodiacepina y Pregabalina por c. Oxígeno por bigotera a 3 litros por
vía oral minuto
b. Zoplicona y Betahistina por vía d. Bolo endovenoso de 100 ml de
oral Dextrosa al 10 %
____ 23. Una mujer de 50 años de edad, está en proceso de separación conyugal desde hace 3
meses. Desde hace un mes presenta anhedonía, pérdida de memoria, llanto frecuente
y cansancio fácil, sensación de inutilidad y culpa, e insomnio. Además, registra
anorexia y una pérdida de peso marcada, así como incapacidad para realizar sus
tareas habituales. Señale el diagnóstico apropiado:
a. Distimia c. Depresión mayor
b. Seudodemencia d. Depresión menor

____ 24. Las siguientes patologías se incluyen en el diagnóstico diferencial de demencia,


EXCEPTO:
a. Déficits cognoscitivos con c. Delirium, conocido también como
deterioro de la memoria, afasia, síndrome confusional agudo
apraxia o agnosia
b. Deterioro cognitivo leve, llamado d. Deterioro cognitivo moderado sin
también deterioro cognitivo asociación a afasia, apraxia,
amnésico agnosia o una alteración de la
capacidad de ejecución
____ 25. Los siguientes son factores de riesgo sociodemográficos para el suicidio consumado,
EXCEPTO:
a.Divorcio c. Sexo masculino
b.Desempleo d. Sexo femenino

4
Name: ________________________ ID: A

____ 26. Una mujer de 80 años, viuda, vive sola, tiene diabetes. Ingresó traumatología por una
fractura de cadera. Se le practicó osteosíntesis. Luego de la cirugía la paciente estaba
confusa e inquieta, merodeaba por la sala durante la noche, buscaba a su madre
difunta, conversaba con personas inexistentes y tenía otras alteraciones de su
comportamiento. Psiquiatría estableció el diagnóstico de delirium. Presenta, además,
una moderada desviación de los valores de los electrolitos en suero el primer día
después de la operación, pero al momento de la evaluación psiquiátrica, se habían
normalizado. En el hemograma tiene una leve leucocitosis. El electrocardiograma,
presentó un infarto menor antiguo y su presión arterial era normal. En este caso, las
causas de este trastorno son las siguientes, EXCEPTO:
a. Desequilibrio electrolítico c. Primer episodio psicótico
b. Proceso infeccioso d. Reacción post anestésica

____ 27. Una mujer de 62 años de edad, con antecedentes de hipertensión leve bien controlada,
presenta súbitamente en las últimas horas, confusión mental, desorientación temporal,
preguntas reiterativas e incapacidad para recordar lo dicho o sucedido recientemente.
No presenta trastornos motores, sensitivos ni de la conciencia. Cuatro horas después
del Inicio, se encuentra notoriamente mejor. Su desempeño cognitivo es normal. Su
evaluación neurológica no presenta alteraciones. Sin embargo, no recuerda lo
sucedido. ¿Cuál es el diagnóstico más probable?
a. Cuadro psicótico c. Demencia aguda transitoria
b. Intoxicación por monóxido de d. Amnesia global transitoria
carbono
____ 28. El ataque o crisis de pánico presenta las siguientes características, EXCEPTO:
a. Temblores y sacudidas c. Miedo a morir
b. Desrealización y d. Ausencia de agorafobia
despersonalización
____ 29. Para los siguientes trastornos se considera a los antipsicóticos como el tratamiento de
primera línea, EXCEPTO:
a. Fase aguda en episodios c. Delirium
maniacos
b. Síndrome depresivo d. Esquizofrenia
____ 30. Las siguientes son recomendaciones de acción de la OMS ante una situación de
ideación suicida, EXCEPTO:
a. Si el riesgo es alto, permanecer c. Preguntar acerca de los intentos
con la persona previos
b. Decirle al paciente, "todo estará d. Preguntarle acerca del plan de
bien" suicidio
____ 31. Los siguientes son medicamentos utilizados para controlar los trastornos del
comportamiento en la demencia, EXCEPTO:
a. Haloperidol c. Donepezil
b. Risperidona d. Aripiprazol

____ 32. Las características del estado mental en la agitación psicomotriz de origen orgánico son
las siguientes, EXCEPTO:
a. La orientación témporo-espacial c. Existe confusión mental
no está afectada
b. El humor es fluctuante y lábil d. El discurso es incoherente

5
Name: ________________________ ID: A

____ 33. Mujer de 64 años acude a la consulta porque no puede dormir en las noches, tiene falta
de apetito, cansancio y menciona que llora fácilmente durante el día. Hace un mes y
medio su espeso murió en un accidente de tránsito; desde entonces piensa
constantemente en él y a veces cree mirar a su esposo llegar por las tardes a la casa
luego del trabajo. Niega pensamientos de suicidio, no tiene antecedentes patológicos
personales, familiares y tampoco toma medicamento alguno. ¿Cuál de los siguientes
diagnósticos corresponde al caso clínico?
a. Reacción normal de duelo c. Desorden de esquizofrenia
b. Desorden de distimia d. Desorden depresivo mayor

____ 34. Los siguientes son factores de riesgo para una conducta suicida, EXCEPTO:
a. Acoso por parte de personas c. Presentar flexibilidad cognitiva
socialmente semejantes
b. Apoyo socio-familiar ausente d. Situación laboral estresante
____ 35. Para los siguientes trastornos se considera a los antipsicóticos como el tratamiento de
primera línea, EXCEPTO:
a. Esquizofrenia c. Delirium
b. Fase aguda en episodios d. Síndrome depresivo
maníacos
____ 36. Los siguientes son medicamentos utilizados para controlar los trastornos del
comportamiento en la demencia, EXCEPTO:
a. Donepezil c. Aripiprazol
b. Haloperidol d. Risperidona

____ 37. El ataque o crisis de pánico presenta las siguientes características, EXCEPTO:
a. Temblores y sacudidas c. Desrealización y
despersonalización
b. Miedo a morir d. Ausencia de agorafobia
____ 38. Una mujer de 80 años, viuda, vive sola, tiene diabetes. Ingresó traumatología por una
fractura de cadera. Se le practicó osteosíntesis. Luego de la cirugía la paciente estaba
confusa e inquieta, merodeaba por la sala durante la noche, buscaba a su madre
difunta, conversaba con personas inexistentes y tenía otras alteraciones de su
comportamiento. Psiquiatría estableció el diagnóstico de delirium. Presenta además,
una moderada desviación de los valores de los electrolitos en suero el primer día
después de la operación, pero al momento de la evaluación psiquiátrica, se habían
normalizado. En el hemograma tiene una leve leucocitosis. El electrocardiograma,
presentó un infarto menor antiguo y su presión arterial era normal. En este caso, las
causas de este trastorno son las siguientes, EXCEPTO:
a. Proceso infeccioso c. Desequilibrio electrolítico
b. Reacción post anestésica d. Primer episodio psicótico

____ 39. Los siguientes son factores de riesgo sociodemográficos para el suicidio consumado,
EXCEPTO:
a. Sexo femenino c. Sexo masculino
b. Divorcio d. Desempleo

6
Name: ________________________ ID: A

____ 40. Una mujer de 50 años de edad, está en proceso de separación conyugal desde hace 3
meses. Desde hace un mes presenta anhedonia, pérdida de memoria, llanto frecuente
y cansancio fácil, sensación de inutilidad y culpa, e insomnio. Además registra anorexia
y una pérdida de peso marcada, así como incapacidad para realizar sus tareas
habituales. Señale el diagnóstico apropiado:
a. Depresión mayor c. Seudodemencia
b. Depresión menor d. Distimia

____ 41. Los siguientes son factores de riesgo para una conducta suicida, EXCEPTO:
a. Situación laboral estresante c. Apoyo socio-familiar ausente
b. Presentar flexibilidad cognitlva d. Acoso por parte de personas
socialmente semejantes
____ 42. Mujer de 64 años acude a la consulta porque no puede dormir en las noches, tiene falta
de apetito, cansancio y menciona que llora fácilmente durante el día. Hace un mes y
medio su esposo murió en un accidente de tránsito; desde entonces piensa
constantemente en él y a veces cree mirar a su esposo llegar por las tardes a la casa
luego del trabajo. Niega pensamientos de suicidio, no tiene antecedentes patológicos
personales, familiares y tampoco toma medicamento alguno. ¿Cuál de los siguientes
diagnósticos corresponde al caso clínico?
a. Desorden depresivo mayor c. Reacción normal de duelo
b. Desorden de distimia d. Desorden de esquizofrenia

____ 43. Las siguientes son recomendaciones de acción de la OMS ante una situación de
ideación suicida, EXCEPTO:
a. Decirle al paciente, "todo estará c. Si el riesgo es alto, permanecer
bien" con la persona
b. Preguntarle acerca del plan de d. Preguntar acerca de los intentos
suicidio previos
____ 44. Las siguientes patologías se incluyen en el diagnóstico diferencial de demencia,
EXCEPTO:
a. Deterioro cognitivo moderado sin c. Deterioro cognitivo leve, llamado
asociación a afasia, apraxia, también deterioro cognitlvo
agnosia o una alteración de la amnésico
capacidad de ejecución
b. Déficits cognoscitivos con d. Delirium, conocido también como
deterioro de la memoria, afasia, síndrome confusional agudo
apraxia o agnosia
____ 45. Una mujer de 62 años de edad, con antecedentes de hipertensión leve bien controlada,
presenta súbitamente en las últimas horas, confusión mental, desorientación temporal,
preguntas reiterativas e incapacidad para recordar lo dicho o sucedido recientemente.
No presenta trastornos motores, sensitivos ni de la conciencia. Cuatro horas después
del inicio, se encuentra notoriamente mejor. Su desempeño cognitivo es normal. Su
evaluación neurológica no presenta alteraciones. Sin embargo, no recuerda lo
sucedido. ¿Cuál es el diagnóstico más probable?
a. Intoxicación por monóxido de c. Cuadro psicótico
carbono
b. Amnesia global transitoria d. Demencia aguda transitoria

7
Name: ________________________ ID: A

____ 46. Las características del estado mental en la agitación psicomotriz de origen orgánico
son las siguientes, EXCEPTO:
a. La orientación témporo-espacial c. El humor es fluctuante y lábil
no está afectada
b. Existe confusión mental d. El discurso es incoherente
____ 47. Las siguientes patologías son parte del diagnóstico diferencial de las demencias.
EXCEPTO:
a.Cognición normal c. Delirium
b.Trastorno delirante d. Trastorno depresivo mayor
____ 48. Un hombre de 44 años de edad Mega por primera vez a su consulta. En la sala de
espera se le solicita que llene varios formularios, con datos de su historial médico
familiar y personal, ante lo cual el paciente asiente amablemente. En la consulta, el
médico constata que los documentos están vacíos, a pesar de que si paciente ha
tenido el tiempo necesario para llenarlos. Al preguntarle ¿por qué no llenó los
documentos? el paciente dice en tono sarcástico: '"estoy seguro que un médico con su
fama y educación puede hacer una historia clínica'”. El médico le pregunta al paciente
si todo está bien, con lo que el paciente responde: “Si, iodo está de maravilla, no hay
ningún problema”; Indique. ¿Cuál de las siguientes opciones describe la actitud del
paciente?
a. Proyección c. Desplazamiento
b. Agresión pasiva d. Comportamiento impulsivo

____ 49. Una mujer de 25 años, sin antecedentes psiquiátricos, es ingresada al servicio de
emergencias. Informa pérdida de memoria frecuentemente, explicando que a menudo
encuentra evidencia de realizar acciones que no recuerda. Además, explica que
durante estas pérdidas de memoria, familiares y amigos han notado que su
personalidad cambia claramente. La madre de la paciente afirma su familia tiene una
sólida base religiosa y que los síntomas de la paciente son de “posesión diabólica”. En
la evaluación se demuestra que la paciente no tiene memoria de ninguno de los
incidentes y sus funciones mentales superiores están normales salvo una leve
hípertimia displacentera. El examen toxicológico es negativo al igual que la TAC. ¿Cuál
es la patología de la paciente?
a. Trastorno de identidad disociativo c. Trastorno de
despersonalización/desrealización
b. Trastorno de conversión d. Amnesia disociativa

8
Name: ________________________ ID: A

____ 50. Paciente diagnosticado con depresión severa, ha recibido tratamiento de forma
intermitente por dos años, es traído por sus familiares al primer nivel de atención por
emergencia, en contra de su voluntad, presenta un corte profundo en la región anterior
de la muñeca, está muy agitado, ha golpeado a su hermano e insulta a todos. Escoja la
opción terapéutica adecuada para este paciente:
a. Valoración y referir para su c. Valoración e ingreso a
ingreso a hospitalización, cierre emergencia, seguimiento
de la herida y terapia electro conductual semanal e inicio de
convulsiva terapia sistémica familiar de 30 a
40 sesiones
b. Valoración e ingreso a d. Ingreso a hospitalización, cierre
emergencia, tratamiento con de la herida y olanzapina oral
ansiolíticos e inicio de terapia
cognitivo conductual de 6 a 8
sesiones
____ 51. Las benzodiacepinas son utilizadas en el tratamiento sintomático de los trastornos de
pánico. ¿Cuál es el mecanismo farmacológico por el que logra su efecto ansiolítico?
a. La acción del fármaco sobre el c. La acción del fármaco sobre el
receptor NMDA receptor 5HT1
b. La acción del fármaco sobre el d. La acción del fármaco sobre el
receptor GABA-A receptor D2
____ 52. A un paciente con un primer episodio psicótico. ¿Qué se aplica una vez instaurado el
tratamiento farmacológico y de apoyo terapéutico?
a. Tratamiento de desintoxicación c. Internación psiquiátrica forzosa
b. Plan de actividad física y d. Tratamiento en hospital del día
nutricional
____ 53. Para el tratamiento de un primer episodio de depresión mayor, moderada o severa, los
diversos grupos de antidepresivos han demostrado tener niveles similares de
efectividad. ¿Cuál de ellos tiene mayor tolerabilidad y mejor relación riesgo-beneficio?
a. Inhibidores selectivos de la c. Antidepresivos duales
recaptación de serotonina (ISRS)
b. Inhibidores de la mono amino d. Antidepresivos tricíclicos
oxidasa
____ 54. Una mujer de 36 años, casada, empleada pública, presenta desde hace 7 meses
episodios de taquicardia, falta de aire, temblor y mareo, que se repiten dos veces al
mes. Define a estos episodios como que “siente que va a morir”. Los estudios de ECG
y de hormonas tiroideas, son normales. Adicionalmente, tiene miedo a salir de casa y
evita ciertas situaciones específica. Cuando está en compañía o se convence de que
no pasará nada supera ese miedo. A! examen psiquiátrico la paciente se manifiesta
muy intranquila, pide ayuda al médico constantemente, está orientada, no se objetivan
alteraciones de la sensopercepción, presenta ideas fijas con respecto a su
padecimiento somático, tiene una leve hipertimia displacentera y se encuentra
eubúlica. Le administraron una benzodiacepina y el episodio cede, aunque persiste el
miedo a que se repita. ¿Cuál es el diagnóstico de esta paciente?
a. Trastorno de ansiedad social c. Trastorno de pánico
b. Trastorno de ansiedad d. Fobia específica
generalizada

9
Name: ________________________ ID: A

____ 55. Wiliiam tiene 44 años, está desempleado, divorciado, consume bebidas alcohólicas
todos los fines de semana, se siente triste, duerme mucho, ha bajado de peso, no
puede concentrarse, siente que piensa lento, por dos ocasiones se ha cortado las
venas y ha decidido lanzarse de un puente. Escoja el plan de tratamiento a seguir en el
manejo clínico de Wiliiam.
a. Manejo y apoyo familiar para la c. Hospitalización en el servicio de
guía conductual e inicio de psiquiatría e inicio de
ansiolíticos psicofármacos
b. Manejo por consulta externa de d. Se respeta la autonomía del
psiquiatría semanal paciente porque es una voluntad
anticipada
____ 56. Son factores de riesgo para la demencia tipo Alzheimer. EXCEPTO:
a. Edad avanzada c. Sexo masculino
b. Tabaquismo d. Hipertensión arterial

____ 57. José tiene 39 años, se encuentra desempleado, su hijo nació con una malformación de la
oreja y es motivo de discusiones en su hogar, ha estado triste por más de 4 meses; ya
no sale con sus amigos que era algo que le gustaba mucho, acude a la consulta
solicitando ayuda porque ha pensado que sería mejor si estuviera muerto, sabe que su
pensamiento es malo, pero esa idea es recurrente. Seleccione la severidad de riesgo
suicida en este caso:
a. Moderado c. Leve
b. Grave d. Extremo

____ 58. En un paciente con riesgo suicida. ¿Cuál de los siguientes enunciados es un criterio para
hospitalización inmediata?
a. Tener una sólida red de apoyo c. No tener ninguna comorbilidad
psicosocial
b. Sexo femenino d. El alto grado de letalidad del
intento suicida
____ 59. Paciente masculino de 40 años, con antecedente de alcoholismo, el cual presenta fiebre,
tinte ictérico, taquipnea, con eritema palmar, temblor en manos, con manos en puño y
lenguaje agresivo y soez. ¿Qué fármaco se debe suministrar?
a. Midazolam 5 mg intravenosos, c. Paracetamol 1 gr Intravenoso
repitiendo la dosis a los 30 STAT, luego cada 8 horas
minutos hasta suministrar 15
mg/día
b. Haloperidol 5mg, vía d. Quetiapina 25 mg, vía oral,
Intramuscular, repitiendo cada 30 repitiendo la dosis a los 30
minutos, dosis máxima 20 mg/día minutos, dosis máxima 50 mg/día

10
Name: ________________________ ID: A

____ 60. Un hombre de 29 años presenta alteración de la conducta sin causa aparente. Su
familiar refiere que lo encuentran con marcada inquietud, con elevado estado de
nerviosismo, muy irritable con su entorno. Increpa a transeúntes sin motivo aparente.
Además, no concilia el sueño, deambula de forma errática por el domicilio, presenta
soliloquios y conductas extravagantes (anota mensajes en papeles que coloca en las
paredes). Expresa miedos respecto a su seguridad. A la exploración se muestra lábil,
con afecto incongruente, con risas inmotivadas y suspicaz. No refieren antecedentes de
consumo de sustancias psicoactivas. Indique el diagnóstico más probable:
a. Delirio c. Agitación Psicomotriz
b. Esquizofrenia d. Pseudodemencia depresiva

____ 61. Elija el término correcto para la siguiente definición: Los síntomas somáticos pueden
manifestarse de forma brusca por una descarga súbita y aislada del sistema nervioso
vegetativo junto a un miedo intenso a morir o a perder el control. La duración de los
síntomas es de aproximadamente entre 15 y 30 minutos y su expresión máxima es en
los primeros 10 minutos.
a. Manía c. Ansiedad
b. Euforia d. Angustia

____ 62. Uno de los tipos de demencia senil es un trastorno neurocognitivo mayor, con carga
genética, que evoluciona progresivamente en un período de 20 años. Es más frecuente
en mujeres después de los 60 años. Entre sus síntomas se incluye alteraciones en la
memoria de largo plazo, afasia, apraxia, desorientación, depresión, cambios en la
conducta y comportamiento. Señale el diagnóstico correspondiente:
a. Demencia frontotemporal c. Demencia de Alzheimer
b. Demencia de cuerpos de Lewy d. Demencia Vascular

____ 63. Relacione las siguientes sustancias que pueden ser empleadas con fines suicidas, con el
antídoto correcto a suministrar en cada caso:
1. Benzodiacepinas
2. Monóxido de carbono
3. Órganos fosforados /carbamatos
4. Paracetamol
5. Opiáceos
6. Antidepresivos
a. Oxígeno
b. N-acetil cisteína
c. Bicarbonato de sodio
d. Flumazenil
e. Atropina
f. Naloxona
a. 1c, 2a, 3e, 4d, 5c, 6f c. 1d, 2a, 3e, 4b, 5f, 6c
b. 1f, 2a, 3e, 4b, 5d, 6c d. 1b, 2d, 3a, 4e, 5f, 6c

____ 64. ¿Cuál grupo farmacológico se usa como primera opción para el manejo sintomático de
un episodio de agitación psicomotriz de origen orgánico, una vez identificada la causa
de base?
a. Antipsicóticos c. Benzodiacepinas
b. Anticonvulsivantes d. Antidepresivos

11
Name: ________________________ ID: A

____ 65. Relacione los tipos de demencias con sus características:


Características
a. Amnesia, afasia, apraxia, agnosia
b. Apatía, trastornos ejecutivos
c. Daño en Sustancia Blanca profunda
d. Déficit en la neurotransmisión colinérgica

Tipos de Demencias
1. Demencias Corticales
2. Demencias Subcorticales
a. 1ad, 2bc c. 1bd, 2ac
b. 1cd,2ab d. 1ac, 2bd
____ 66. ¿En qué circunstancias se debe optar por el tratamiento farmacológico en un paciente
con delirium?
a. Si el paciente se encuentra c. Siempre que exista angustia en
desorientado una vez que sale de un paciente de la tercera edad
la sala de recuperación hospitalizado
postquirúrgica
b. Siempre que se diagnostica d. Agitación o agresividad que
delirium pongan en riesgo la integridad del
paciente o terceros
____ 67. ¿Qué grupo farmacológico se recomienda en pacientes adultos con depresión mayor que
presentan ideación suicida?
a. Benzodiazepinas c. Inhibidores selectivos de la
recaptación de serotonina
b. Antipsicóticos atípicos d. Anticonvulsivantes
____ 68. ¿Cuál de las siguientes situaciones representa una complicación grave en la agitación
psicomotriz?
a. Somnolencia c. Ataque de pánico
b. Hiperpatía d. Hiperventilación

12
ID: A

Salud Mental
Answer Section

MULTIPLE CHOICE

1. ANS: B PTS: 1
2. ANS: D PTS: 1
3. ANS: D PTS: 1
4. ANS: D PTS: 1
5. ANS: D PTS: 1
6. ANS: D PTS: 1
7. ANS: A PTS: 1
8. ANS: D PTS: 1
9. ANS: C PTS: 1
10. ANS: A PTS: 1
11. ANS: C PTS: 1
12. ANS: C PTS: 1
13. ANS: B PTS: 1
14. ANS: C PTS: 1
15. ANS: A PTS: 1
16. ANS: D PTS: 1
17. ANS: B PTS: 1
18. ANS: C PTS: 1
19. ANS: B PTS: 1
20. ANS: A PTS: 1
21. ANS: A PTS: 1
22. ANS: A PTS: 1
23. ANS: C PTS: 1
24. ANS: A PTS: 1
25. ANS: D PTS: 1
26. ANS: C PTS: 1
27. ANS: D PTS: 1
28. ANS: D PTS: 1
29. ANS: B PTS: 1
30. ANS: B PTS: 1
31. ANS: C PTS: 1
32. ANS: A PTS: 1
33. ANS: A PTS: 1
34. ANS: C PTS: 1
35. ANS: D PTS: 1
36. ANS: A PTS: 1
37. ANS: D PTS: 1
38. ANS: D PTS: 1
39. ANS: A PTS: 1
40. ANS: A PTS: 1
41. ANS: B PTS: 1
42. ANS: C PTS: 1
43. ANS: A PTS: 1

1
ID: A

44. ANS: B PTS: 1


45. ANS: B PTS: 1
46. ANS: A PTS: 1
47. ANS: B PTS: 1
48. ANS: B PTS: 1
49. ANS: A PTS: 1
50. ANS: A PTS: 1
51. ANS: B PTS: 1
52. ANS: B PTS: 1
53. ANS: A PTS: 1
54. ANS: C PTS: 1
55. ANS: C PTS: 1
56. ANS: C PTS: 1
57. ANS: C PTS: 1
58. ANS: D PTS: 1
59. ANS: B PTS: 1
60. ANS: C PTS: 1
61. ANS: D PTS: 1
62. ANS: C PTS: 1
63. ANS: C PTS: 1
64. ANS: A PTS: 1
65. ANS: A PTS: 1
66. ANS: D PTS: 1
67. ANS: C PTS: 1
68. ANS: A PTS: 1

2
Salud Mental [Answer Strip] ID: A

A
_____ 7. C 14.
_____ A 21.
_____ C 26.
_____

B
_____ 1. A 15.
_____

D
_____ 2.
D
_____ 8. A 22.
_____

D 27.
_____
D 16.
_____

D
_____ 3.
C
_____ 9. B 17.
_____
C 23.
_____

D
_____ 4.
D 28.
_____
A 10.
_____

A 24.
_____
D
_____ 5. B 29.
_____
C 18.
_____

B 30.
_____
C 11.
_____ B 19.
_____

D 25.
_____
D
_____ 6.

C 31.
_____

A 20.
_____
C 12.
_____

A 32.
_____

B 13.
_____
Salud Mental [Answer Strip] ID: A

A 33.
_____ A 40.
_____ A 46.
_____ A 50.
_____ C 55.
_____

B 47.
_____

B 41.
_____

C 34.
_____
B 48.
_____
C 56.
_____
C 42.
_____

D 35.
_____
B 51.
_____ C 57.
_____

A 36.
_____

A 43.
_____ B 52.
_____
A 49.
_____
D 58.
_____
D 37.
_____

A 53.
_____
B 44.
_____
D 38.
_____
B 59.
_____

C 54.
_____

B 45.
_____

A 39.
_____
Salud Mental [Answer Strip] ID: A

C 60.
_____ A 65.
_____

D 61.
_____

D 66.
_____

C 62.
_____

C 67.
_____

C 63.
_____

A 68.
_____

A 64.
_____
GINECOLOGIA Y OBSTETRICIA
SCORE MAMA

MULTIPLE CHOICE

1.
El síndrome de respuesta inflamatoria sistémica (SIRS), es un síndrome que se caracteriza por
la presencia de 2 o más de las siguientes variables:

a. Temperatura > 38oC o < 36oC, frecuencia c. Glóbulos blancos > 12.000/ul o <
cardiaca > de 90 lmp 4.000/ul o > 10 % de formas inmaduras

b. Frecuencia respiratoria > 20 por minuto o d. Todas son correctas


presión parcial de dióxido de carbono
(PaCO2) < 32 mmHg en la costa y < 28
mmHg en la sierra

ANS: D
El síndrome de respuesta inflamatoria sistémica (SIRS), es un síndrome que se caracteriza por la
presencia de 2 o más de las siguientes variables: Temperatura > 38oC o < 36oC, frecuencia
cardiaca > de 90 lmp, Frecuencia respiratoria > 20 por minuto o presión parcial de dióxido de
carbono (PaCO2) < 32 mmHg en la costa y < 28 mmHg en la sierra, Glóbulos blancos > 12.000/ul
o < 4.000/ul o > 10 % de formas inmaduras

PTS: 1 DIF: ALTA


REF: GPC / MSP Score mamá y claves obstétricas. PAG: 8
OBJ: VALORAR COMPLICACIONES MATERNAS TOP: SCORE MAMA

1
UNIVERSIDAD NACIONAL DE CHIMBORAZO
FACULTAD DE CIENCIAS DE LA SALUD
CARRERA DE MEDICINA

REACTIVOS CÁTEDRA DE OTRORRINOLARINGOLOGIA-Perez Rolando

Multiple Choice
Identify the choice that best completes the statement or answers the question.

____ 1. De la anatomofisiologia de las fosas nasales y senos perinasales, seleccione la respuesta correcta.

a. En la pared externa de las fosas nasales esta constituida por los cornetes y los meatos

b. La pared superior de las fosas nasales esta constituidas por los huesos propios de la
nariz, la espina nasal del frontal, la lamina cribosa y el cuerpo del esfenoides

c. La pared inferior o suelo se separa de las fosas nasales de la cavidad oral

d. Los senos etmoidales posteriores drenan en el meato medio

____ 2. Identifique las complicaciones extracraneales de la otitis media

a. Mastoiditis aguda

b. Meningitis bacteranea

c. Tromboflebitis septica del seno lateral y del golfo de la yugular

d. Laberintitis purulenta

____ 3. El tumor maligno más frecuente de la glándula submaxilar es:

a. Carcinoma mucoepidermoide

b. Crcinoma ex-adenoma pleomorfo

c. Cilindroma o carcinoma adenoide quistico

d. Citoadenolinfoma papilomatoso

____ 4. Ante un paciente con otitis externa disfusa, cual es el tratamiento inicial

a. Cirugía

b. Antibioticos mas analgésicos, ambos vía endovenosa

c. Antibióticos tópicos

d. Limpieza del conducto auditivo externo, antibioticoterapia tópica más analgésicos orales

1
Name: ________________________ ID: A

____ 5. Dentro de los carcinomas de cabeza y cuello, que tipo de tumor se relaciona de forma más evidente
con el virus del papiloma humano.

a. Carcinoma epidermoide de Laringe

b. Carcinoma epidermoide de orofaringe

c. Carcinoma epidermoide de hipofaringe

d. Adenocarcinoma nasosinusal

____ 6. Niño de 5 años de edad que no pasó las pruebas de cribado auditivo al nacimiento y que presenta
potenciales evocados auditivos del tronco cerebral que determinan una hipoacusia bilateral leve
moderada - grave en el oído externo, cual es las actitud mas correcta a seguir en el paciente.

a. Realizar audiometría en el plazo de 6 meses para confirmar diagnóstico

b. Cirugía para adaptar implante coclear en el oído derecho

c. Cirugía para adaptar un implante coclear en el oído izquierdo

d. Adaptación de audoprótesis bilateral y rehabilitación logopédica

____ 7. Paciente de 45 años de edad en el 15avo día postransplante de progenitores hematopoyéticos con
neutropenioa absoluta, plaquetas de 15000/ul y hemoglobina de 7g/dl que presenta un cuadro
clínico de dolor ocular con edema preorbitario con discreta secresión nasal sanguinolienta, cual es
el diagnóstico de presunción.

a. Hematoma preorbitario

b. Mucormicosis

c. Sinusitis aguda bacteriana

d. Reacción alércica posiblemente a la medicación

____ 8. Colocando un diapasón que está vibrando frente al conducto auditivo del oído que queremos
explorar y apoyandolo después en la apófisis masotides podemos de modo sencillo y en la cosnulta
distinguir entre sordera nerviosa alteración en la cóclea o nervio auditivo y sordera de conducción
transtorno en el sistema de transmisión tímpano oscilar, cual de estas afirmaciones es correcta para
un paciente que presenta sordera de conducción.

a. La percepción del sonido es igual por vía aérea que por vía ósea

b. La percepción del sonido es mejor por vía ósea que por vía aérea

c. La percepción del sonido es mejor por vía aérea que por vía ósea

d. La percepción del sonido es inditinguible tanto por vía aérea como por vía ósea

2
Name: ________________________ ID: A

____ 9. La otitis media aguda no suele complicarse, no obstante cuando lo hace es con más frecuencia con
una:

a. Petrositis

b. Meningitis

c. Mastoiditis

d. Osteomielitis

____ 10. El síndrome de Ramsay Hunt se caracteriza por parálisi facial acompañada de:

a. Otitis media colesteatomatosa

b. Parálisis Yatrogénica

c. Parálisis por traumatismo craneal

d. Herpes zóster

3
ID: A

REACTIVOS CÁTEDRA DE OTRORRINOLARINGOLOGIA-Perez Rolando


Answer Section

MULTIPLE CHOICE

1. ANS: B
La pared superior de las fosas nasales esta constituidas por los huesos propios de la nariz, la espina nasal
del frontal, la lamina cribosa y el cuerpo del esfenoides forman la anatomofisiologia de las fosas nasales
y senos perinasales

PTS: 1 DIF: Alta REF: Otorrinolaringología. secretos Jafek Bruce W Elsevier


OBJ: Diagnostica las enfermedades de la nariz y los senos paranasales. - Operaciones Cognitivas -
Resolución de errores - Planificacion de acciones y correccion de errores
LOC: Dr. Rolando Perez TOP: NARIZ Y SENOS PARANASALES
2. ANS: A
La mastoiditis aguda es una de las principales complicaciones de la otitis media

PTS: 1 DIF: Alta REF: Otorrinolaringología. secretos Jafek Bruce W Elsevier


OBJ: Diagnostica las enfermedades de oído- | Operaciones Cognitivas - Resolución de errores -
Planificacion de acciones y correccion de errores LOC: Dr. Rolando Perez
TOP: OIDO
3. ANS: C
El carcinoma mucoepidermoide es el tumor maligno mas frecuente en la glandula parotida, sin embargo
cuando se trata de la glandula submaxilar el carcinoma adenoide quistico presenta uno en cada dos
tumores malignos

PTS: 1 DIF: Alta


REF: Escajadillo JR. Oídos, Nariz, Garganta y Cirugía de Cabeza y Cuello. Editorial: El Manual
Moderno. 2da ed; 2002
OBJ: Diagnostica las enfermedades de las glándulas salivales y las que se originan en el cuello -
Operaciones Cognitivas - Resolución de errores - Planificacion de acciones y correccion de errores
LOC: Dr. Rolando Perez TOP: GLANDULAS SALIVALES Y CUELLO
4. ANS: D
Según La guía clínica de la academia americana de Otorrinolaringología hay una fuerte recomendación
para el tratamiento del dolor y la inflamación, tambein hacen referencia el tratamiento inicial de la otitis
externas no complicadas que debe ser con atibióticos tópicos así como inciar por la limpieza del conduto
auditivo externo

PTS: 1 DIF: Alta


REF: Escajadillo JR. Oídos, Nariz, Garganta y Cirugía de Cabeza y Cuello. Editorial: El Manual
Moderno. 2da ed; 2002
OBJ: Diagnostica las enfermedades de oído - Operaciones Cognitivas - Resolución de errores -
Planificacion de acciones y correccion de errores LOC: Dr. Rolando Perez
TOP: OIDO

1
ID: A

5. ANS: B
El VPH se contagia muchas veces a través de secreciones por lo que el lugar donde las secreciones
tienden hacer mas contacto es la orofaringe, se debe añadir que los tumores de orofaringe con positividad
para VPH se presentan en pacientes jovenes con ausencia de otros factores de reisgo y tienden mejor
pronóstico

PTS: 1 DIF: Alta


REF: Escajadillo JR. Oídos, Nariz, Garganta y Cirugía de Cabeza y Cuello. Editorial: El Manual
Moderno. 2da ed; 2002
OBJ: Diagnostica las enfermedades de las glándulas salivales y las que se originan en el cuello -
Operaciones Cognitivas - Resolución de errores - Planificacion de acciones y correccion de errores
LOC: Dr. Rolando Perez TOP: GLANDULAS SALIVALES Y CUELLO
6. ANS: D
El objetivo de cribado universal es el diagnóstico y tratamiento lo más pronto posible trartando de
conservar tanto antómica como funcionalmente la mayor cantidad de estructuras y adaptar al paciente
para que este se desemvuelva con la mayor normalidad posible en la sociedad.

PTS: 1 DIF: Alta


REF: Escajadillo JR. Oídos, Nariz, Garganta y Cirugía de Cabeza y Cuello. Editorial: El Manual
Moderno. 2da ed; 2002
OBJ: Diagnostica las enfermedades de oído - Operaciones Cognitivas - Resolución de errores -
Planificacion de acciones y correccion de errores LOC: Dr. Rolando Perez
TOP: OIDO
7. ANS: B
Se describen tres tipos de risnosinusitis invasivas, la invasiva crónica por Aspergillus Fumigatus, propia
de pacientes inmunodeprimidos que se desarrolla en más de tres meses, la invasiva subaguda
granulomatosa, por Aspergillus Flavus propia de pacientes inmunocompetentes, y la invasiva aguda por
Mucor muy rápida que es la que orienta a la pregunta.

PTS: 1 DIF: ALta


REF: Escajadillo JR. Oídos, Nariz, Garganta y Cirugía de Cabeza y Cuello. Editorial: El Manual
Moderno. 2da ed; 2002
OBJ: Diagnostica las enfermedades de la nariz y los senos paranasales- Operaciones Cognitivas -
Resolución de errores - Planificacion de acciones y correccion de errores
LOC: Dr. Rolando Perez TOP: NARIZ Y SENOS PARANASALES
8. ANS: B
Para un paciente que presenta sordera de conducción. la afirmación correcta es que La percepción del
sonido es mejor por vía ósea que por vía aérea

PTS: 1 DIF: Alta


REF: Escajadillo JR. Oídos, Nariz, Garganta y Cirugía de Cabeza y Cuello. Editorial: El Manual
Moderno. 2da ed; 2002 | Otorrinolaringología. secretos Jafek Bruce W Elsevier
OBJ: Diagnostica las enfermedades de oído - Operaciones Cognitivas - Resolución de errores -
Planificacion de acciones y correccion de errores LOC: Dr. Roalndo Perez
TOP: OIDO

2
ID: A

9. ANS: C
La complicación mas frecuente en caso de otitis media es la mastoiditis

PTS: 1 DIF: Alta


REF: Otorrinolaringología. secretos Jafek Bruce W Elsevier | Escajadillo JR. Oídos, Nariz, Garganta y
Cirugía de Cabeza y Cuello. Editorial: El Manual Moderno. 2da ed; 2002
OBJ: Diagnostica las enfermedades de oído - Operaciones Cognitivas - Resolución de errores -
Planificacion de acciones y correccion de errores LOC: Dr. Rolando Perez
TOP: OIDO
10. ANS: D
El virus varicela zóster que cusa el síndromde Ramsay Hunt es el mismo que causa la varicela y el herpes
zóster, en las personas con este virus infecta al nervio facial cercano al oído interno lo cual lleva a que se
presente irritación e inflamación del mismo causando parálisis facial

PTS: 1 DIF: Alta


REF: Otorrinolaringología. secretos Jafek Bruce W Elsevier | Escajadillo JR. Oídos, Nariz, Garganta y
Cirugía de Cabeza y Cuello. Editorial: El Manual Moderno. 2da ed; 2002
OBJ: Diagnostica las enfermedades de oído - Operaciones Cognitivas - Resolución de errores -
Planificacion de acciones y correccion de errores LOC: Dr. Rolando Perez
TOP: OIDO

3
REACTIVOS CÁTEDRA DE OTRORRINOLARINGOLOGIA-Perez Rolando ID: A
[Answer Strip]
B
_____ 5. C
_____ 9.

D
_____ 6. D 10.
_____

B
_____ 1.

B
_____ 7.
A
_____ 2.

C
_____ 3.
B
_____ 8.

D
_____ 4.
Name: ________________________ Class: ___________________ Date: __________ ID: A

GASTROENTEROLOGIA

Multiple Choice
Identify the choice that best completes the statement or answers the question.

____ 1.
¿Cuál es el tratamiento quirúrgico indicado para el reflujo gastroesofágico grave?

a. Gastrectomía manga c. Gastrectomía subtotal

b. Banda gástrica d. Fundoplicatura de Nissen

____ 2.
Las bacterias implicadas en la colangitis con más frecuencia son, excepto?

a. Escherichia coli c. Staphylococcus

b. Klebsiella d. Clostridium

____ 3.
Dentro de los métodos invasivos para la detección de Helicobacter pylori, cual estudio es
superior o tiene mayor sensibilidad y especificidad?

a. Test rápido de la ureasa c. Cultivo

b. Estudio histopatológico d. Todas

____ 4. ¿Cuándo es aconsejable realizar la endoscopia digestiva alta, en los casos de hemorragia digestiva
alta?

a. Primeras 8 horas de inicio de la c. Instaurado el tratamiento, no se debe


hemorragia realizar

b. Después de las 24 a 48 horas de inicio d. Ninguna de las anteriores


de la hemorragia

____ 5.
Porque se recomienda anticoagulación profiláctica con heparina de bajo peso molecular en los
pacientes con pancreatitis aguda?
a. Prevenir infecciones asociadas a la c. Prevenir enfermedad tromboembólica
pancreatitis pulmonar

b. Prevenir sangrados abdominales. d. Ninguna

1
Name: ________________________ ID: A

____ 6.
¿Cuál de las siguientes complicaciones locales de la pancreatitis aguda, aparece mayor a las 4
semanas?

a. Colección necrótica aguda c. Trombosis de vena esplénica

b. Seudoquiste d. Necrosis colónica

____ 7.
En general un paciente sin factores de riesgo, con diagnóstico de hemorragia digestiva, ¿cuál
sería el valor de laboratorio que me justifique el uso de transfusión sanguínea?
a. Hematocrito inferior al 20% y c. Hematocrito inferior al 45% y
Hemoglobina inferior a 7 g/dl Hemoglobina inferior a 7 g/dl

b. Hematocrito inferior al 25% y d. Hematocrito inferior al 30% y


Hemoglobina inferior a 8 g/dl Hemoglobina inferior a 8 g/dl

____ 8.
En relación a los signos endoscópicos de hemorragia reciente, señale lo correcto respecto al
riesgo de recidiva?
a. Presencia de un Coágulo adherido c. Hemorragia activa la incidencia de
sobre la úlcera indica que no se trata recidiva es muy alta de 30%
de una lesión de alto riesgo de
recidiva, con una media del 43 %
b. Ausencia de signos de hemorragia d. Ninguna
reciente, el riesgo de recidiva es
mínimo, situado alrededor del 2%

____ 9.
¿Cuál de los siguientes estados clínicos o complicaciones en pancreatitis aguda tiene indicación
quirúrgica, cuando ha fallado al tratamiento inicial?
a. Colección aguda de fluido peri c. Colección necrótica sintomática
pancreático
b. Presencia de tres o más criterios de d. Balthazar B
Ranson

____ 10.
¿Cuáles son las causas etiológicas de la diarrea aguda?

a. Bacterias c. Viral

b. Fármacos d. Todas

2
Name: ________________________ ID: A

____ 11. La cirrosis hepática ocasiona las siguientes complicaciones, excepto:


a. Trastornos de la coagulación c. Elevación de albumina en sangre

b. Encefalopatía d. Ascitis.

____ 12. 12. El parámetro que no pertenece a la clasificación de Child Pugh


a. Abumina c. Tiempo de Protrombina
b. TGO - TGP d. Bilirrubinas total

____ 13. 13.¿Cuál es el periodo de incubación promedio de la Hepatitis B?


a. 14-120 días c. 30-140 días
b. 15-160 días d. 40-180 días

____ 14. 14 La hepatitis B suele diagnosticarse, por lapositividad de cuál marcador?


a. HBeAg c. anti-HBc IgM
b. HBsAg d. b y c son corectas

____ 15. 15. Cuál es la causa más frecuente de hipertensión portal pre hepática?
a. Trombosis esplenoportal c. Síndrome de Budd-Chiari
b. Cirrosis Hepática d. Trombosis de lavena cava inferior

3
ID: A

GASTROENTEROLOGIA
Answer Section

MULTIPLE CHOICE

1. ANS: D
La funduplicatura quirúrgica es un tratamiento con una eficacia comparable a los IBP. Está indicada en
pacientes que no desean tomar IBP de manera crónica, en los que desarrollan efectos secundarios al
tratamiento médico y en aquellos con síntomas nocturnos derivados de un alto volumen de regurgitación
que no se contrala ni responde adecuadamente al tratamiento

PTS: 1 DIF: MEDIA


REF: Farreras-Rozman: Medicina Interna, XVIII Edición Elsevier España. 2016
TOP: ESOFAGO NOT: DRA. MERCEDES MACAS
2. ANS: C
Las bacterias implicadas en la colangitis son las mismas que producen colecististis aguda son:
Escherichia coli, Clostridium, Klebsiella, Streptococcus.

PTS: 1 DIF: BAJA


REF: Farreras-Rozman: Medicina Interna, XVIII Edición. Ediciones Elsevier- España
TOP: COLANGITIS VIA BILIAR NOT: Dra. MERCEDES MACAS
3. ANS: B
El estudio histopatológico tiene sensibilidad y especificidad muy elevadas superiores a los otros métodos.

PTS: 1 DIF: BAJA


REF: Farreras-Rozman: Medicina Interna, XVIII Edición. Ediciones Elsevier- España
TOP: ESTOMAGO NOT: Dra. MERCEDES MACAS
4. ANS: A
En la mayoría de los casos es aconsejable realizar la endoscopia en la primeras 8 hora de inicio de la
hemorragia, un retraso superior supone na reducción notable de su renatbilidad diagnostica.

PTS: 1 DIF: MEDIA


REF: Farreras-Rozman: Medicina Interna, XVIII Edición. Ediciones Elsevier- España
TOP: ESTOMAGO NOT: DRA. MERCEDES MACAS
5. ANS: C
Se debe valorar la anticoulación profiláctica con heparina de bajo peso molecular en los pacientes con
pancreatitis aguda, para eviar la enfermedad tromboembólica pulmonar y quizál trombosis de venas
peripancreaticas.

PTS: 1 DIF: Media


REF: Farreras-Rozman: Medicina Interna, XVIII Edición. Ediciones Elsevier- España
TOP: Páncreaa NOT: Dra. Mercedes Macas
6. ANS: B
Colección de líquido peripancreatico sin presencia de necrosis con pared fina demayor a 4 semanas de
evolución

PTS: 1 DIF: Media


REF: Farreras-Rozman: Medicina Interna, XVIII Edición. Ediciones Elsevier- España
TOP: Páncreas NOT: Dra. Mercedes Macas

1
ID: A

7. ANS: B
Aunque no existe un crterio absoluto, en general la transfusión sanguinea está indicada en la hemorragia
digestiva alta, cuando el hematocrito inferior al 25% y hemoglobina inferior a 8 g/dl

PTS: 1 DIF: Media


REF: Farreras-Rozman: Medicina Interna, XVIII Edición. Ediciones Elsevier- España
TOP: Estomago Hemorragia Digestiva Alta NOT: Dra. Mercedes Macas
8. ANS: B
La identificación de ls signos de hemorragia reciente , predice el risgo de recidiva del episodio
hhemorraágico, por ello en ausencia de signos de hemorragia reciente, el riesgo de recidiva es
mínimo, situado alrededor del 2%

PTS: 1 DIF: Media


REF: Farreras-Rozman: Medicina Interna, XVIII Edición. Ediciones Elsevier- España
TOP: Estómago Hemorragia Digestiva Alta NOT: Dra Mercedes Macas
9. ANS: C
Ante una colección necrótica sintomática, rebelde al tratamiento médico o con alta sopecha de infección
y existe persitencia de sintomas, se procederá a la necreoectomía endoscópica o quirurgica.

PTS: 1 DIF: Alta


REF: Farreras-Rozman: Medicina Interna, XVIII Edición. Ediciones Elsevier- España
TOP: Páncreas NOT: Dra. Mercedes Macas
10. ANS: D
La diarrea aguda dentro de las principales causas están las infecciones causadas por bacterias y virus; por
fármacos, entre otras causas.

PTS: 1 DIF: Baja


REF: Farreras-Rozman: Medicina Interna, XVIII Edición. Ediciones Elsevier- España
TOP: Intestino NOT: Dra. Mercedes Macas
11. ANS: C
En un paciente con cirrosis se suele encontrar descenso de protrombina como trastorno de lacoagulaión,
hipoalbuminemia, ascitis y encefalopatía hepatica entre otras.

PTS: 1 DIF: Baja


REF: Farreras-Rozman: Medicina Interna, XVIII Edición. Ediciones Elsevier- España
TOP: Hígado NOT: Dra. Mercedes Macas
12. ANS: B
La escala pronostica de Child Pugh valora los items de Enefalopatia Heptica, ascitis, tiempo de
protrombina, albumina y bilirrubina

PTS: 1 DIF: Baja


REF: Farreras-Rozman: Medicina Interna, XVIII Edición. Ediciones Elsevier- España
TOP: Hígado NOT: Dra. Mercedes Macas
13. ANS: D
El periodo de incubación promedio es de 40 a 180 dias

PTS: 1 DIF: Baja


REF: Farreras-Rozman: Medicina Interna, XVIII Edición. Ediciones Elsevier- España
TOP: Higado NOT: Dra. Mercedes Macas

2
ID: A

14. ANS: D
Para el diagnostico de la hepatitis B se requiere la positivvidad de HBsAg y anti-HBc IgM

PTS: 1 DIF: Media


REF: Farreras-Rozman: Medicina Interna, XVIII Edición. Ediciones Elsevier- España
TOP: Higado NOT: Dra. Mercedes Macas
15. ANS: A
La hipertensión portal pre hepática la causa más frecuente es la trombosis espleno portal

PTS: 1 DIF: Media


REF: Farreras-Rozman: Medicina Interna, XVIII Edición. Ediciones Elsevier- España
NOT: Dra. Mercedes Macas

3
GASTROENTEROLOGIA [Answer Strip] ID: A

B
_____ 6. C 11.
_____

D
_____ 1.
B 12.
_____

B
_____ 7.
D 13.
_____

C
_____ 2.
D 14.
_____

A 15.
_____
B
_____ 8.

B
_____ 3.

A
_____ 4. C
_____ 9.

C
_____ 5. D 10.
_____
Name: ________________________ Class: ___________________ Date: __________ ID: A

LECCION

Multiple Choice
Identify the choice that best completes the statement or answers the question.

____ 1. Paciente de 65 años, varón, hipertenso, quien se someterá a una cirugía de bypass coronario. ¿El riesgo de
infección de su herida es?:

a. 5%
b. 10%
c. 15%
d. 20%

____ 2. En una paciente mujer de 45 años, con 72 horas PO de una histerectomía abdominal, presenta alza térmica
de 39 grados centígrados. Usted sospecha en:
a. Infección por catéteres
b. Infección inherente a la herida quirúrgica
c. Infección del sitio quirúrgico propiamente dicho
d. Infección de vías urinarias

____ 3. Paciente varón de 40 años chofer, quien sufre un accidente de tránsito producto del cual ha tenido trauma
hepático y colónico, sometido a laparotomía. En el PO se lo maneja con criterios de sépsis, uno de los
cuales es la medición de la PIA Presión intraabdominal), la cual se encuentra en 26. ¿Cuál es la conducta
a seguir?
a. Ciurgía electiva
b. Observación durante 24 horas y posible cirugía
c. Solamente observación
d. Descompresión abdominal

____ 4. En un paciente varón de 30 años, con diagnóstico de apendicitis complicada, quien ingresa con choque,
¿Cuál de los siguiente valores de procalcitonina indica la posibilidad de que la etiología no sea séptica?
a. 0,5ng/mi
b. 2ng/ml
c. 4ng/mi
d. 5ng/ml

____ 5. Paciente mujer de 50 años de edad, diagnosticada de sépsis abdominal, con un SOFA de 13, luego de la
valoración ventilatoria, de coagulación, función hepática, cardio vascular, del SNC y aparato renal;
señale ¿cuál es el porcentaje de mortalidad que tiene?:
a. <10%
b. 15 al 20%
c. 50 al 60%
d. >80%

____ 6. Se opera a una paciente mujer de 40 años de una hernia umbilical, encontrando en el saco herniario un
divertículo de Meckel, se trata de un ahernia de:
a. Ritcher
b. Litre
c. Amyand
d. Incisional

1
Name: ________________________ ID: A

____ 7. En un paciente varón de 50 años, con diagnóstico de hernia inguinal bilateral: Gilberth IV derecha y
Gilberth V en el lado izquierdo, el procedimiento recomendado es:
a. Herniorrafia con técnica de Shuldice bilateral
b. Hernioplastia con técnica libre de tensión por vía anterior bilateral
c. Hernioplastia con técnica de Lichtenstein izquierda y herniorrafia con técnica de Bassini
derecha
d. TAP

____ 8. En una paciente mujer de 70 años, con una masa herniaria tipo Nyhius IIIC, los límites de dicho defecto
son:
a. cintilla ileo-pectínea, ligamento lacunar y ligamento inguinal
b. Borde lateral del recto anterior del abdomen, vasos epigástricos, lilgamento inguinal
c. dorsal ancho, oblicuo externo, cresta ilíaca
d. Arco de Dpuglas a nivel de la línea semilunar de Spiegel

____ 9. En un niño de 5 años, con un ahernia umbilical de 2 cm de diámetro. ¿Cuál es la conducta a seguir?:
a. Observación hasta los 7 años de edad
b. Cirugía para reparar el defecto herniario
c. Esperar a que tenga 3 cm de diámtero y usar una malla para operarlo
d. No hace falta cirugía

____ 10. Se opera a un varón de 18 añlos de edad, de una hernia inguinal izquierda, los hallazgos son: saco herniario
localizado en la región antero externa del cordón espermpatico, ingresa al conducto por el orificio
profundo, se acompaña de un lipoma del cordón, llega al lado del escroto, es de deslizamiento. Según la
clasificación de Nyhus, corresponde a:
a. Tipo IIIB
b. Tipo IIIA
c. Tipo IIIC
d. Tipo II

2
ID: A

LECCION
Answer Section

MULTIPLE CHOICE

1. ANS: A
Al ser una cirugía limpia, el porcentaje de infección permisible llega al 5%

PTS: 1 DIF: Alta


REF: Ochoa, G; Infección de heridas quirúrgicas, en Urgencias quirúrgicas, pag 80.
OBJ: conocimiento de los tipos de heridas quirúrgicas TOP: Herida quirúrgica
KEY: herida límpia NOT: Dr. Vinicio Moreno Rueda
2. ANS: D
Saber las causas de alza térmica en el PO y relacionarlas con el tiempo al que aparecen

PTS: 1 DIF: Alta


REF: Darryl, T; Complicaciones quirúrgicas, cap 11, Principios de cirugía, Schwartz, pag 473
OBJ: Causas de fiebre PO TOP: Fiebre perioperatoria
KEY: Fiebre, infección vía urinaria NOT: Dr. Vinicio Moreno Rueda
3. ANS: D
Al paciente séptico quirúrgico se lo debe manejar con escalas como APACHE II, SOCA, QSOFA, PIA,
etc. Manjo conjunto entre el Cirujano y el Intensivista

PTS: 1 DIF: Alta


REF: Dietmar, H; Peritonitis e infección intraabdomina; en Schwarta, Principios de Cirugía, capítulo 32,
pag 1499 OBJ: conocimiento del manejo de la infección intrabadominal
TOP: peritonitis, sépsis de origen abdominal KEY: Sépsis, peritonitis, PIA
NOT: Dr. Vinicio Moreno Rueda
4. ANS: A
conocer el valor de la procalcitonina como un indicador de infección bacteriana grave

PTS: 1 DIF: Alta


REF: Dietmar,H; Peritonitis e infección intraabdominal; en Schwartz: Principios de Cirugía, capítulo
32, pag 1504 OBJ: Conocer la utilidad de la procalcitonina como un marcador séptico
TOP: Sépsisi abdominal KEY: Procalcitonina, sépsis abdominal
NOT: Dr. Vinicio Moreno Rueda
5. ANS: C
Conocimiento de las escalas de manejo de la sépsis

PTS: 1 DIF: Alta


REF: Dietmar, H; Peritonitis e infección intraabdominal; en Schwartz, Principios de Cirugía, capítulo
32, pag 1523 OBJ: Conocimiento de la utilidad del uso de escales de sépsis
TOP: Sépsis abdominal KEY: SOFA NOT: Dr. Vinicio Monreo Rueda
6. ANS: B
Conocimiento de los tipos especiales de hernias de la pared abdominal

PTS: 1 DIF: Alta


REF: Courtey, M; Intestino delgado; en Schwartz, Principios de Cirugía, capítulo 25, pag: 1218
OBJ: Conocimiento de los tipos especiales de herinias de la pared abdominal
TOP: Hernias de la pared abdominal KEY: Hernia de litre
NOT: Dr. Vinicio Moreno Rueda

1
ID: A

7. ANS: D
Las principales indicaciones de herniplastia con técnica tap por vía laparoscópica son:
Hernia inguinal bilateral
Hernia inguinal recidivante

PTS: 1 DIF: Alta


REF: Wantz, G; Hernias de la pared abdominal, en Schwart, Principios de Cirugía; capítulo 34, pag
1576
OBJ: Hernias de la pared abdominal, hernia inguinal TOP: Hernia inguinal
KEY: Hernia inguinal, laparoscopía NOT: Dr. Vinicio Moreno Rueda
8. ANS: A
El conocimiento de las clasificaciones de la hernia inguinal, nos permite saber que se trata de una hernia
femoral, los límites del anillo femoral son cintilla ileo-pectínea, ligamento lacunar y ligamento inguinal

PTS: 1 DIF: Alta


REF: Wantz, G, Hernias de la pared abdominal; en Schwartz Principios de Cirugía, capítulo 34, pag.
1571
OBJ: Conocimiento de las clasificaciones de hernia inguinal, y la anatomía de la región
inguinal-femoral
TOP: Hernia inguinal KEY: Hernia femoral, clasificación Nyhus
NOT: Dr. Vinicio Moreno Rueda
9. ANS: B
Se puede esperar hasta los 3 años en espera de un cierre expontáneo, de no ocurrir se debe realizar la
herniorrafia umbilical

PTS: 1 DIF: Alta


REF: Wantz, G, Hernias de la pared abdominal; en Schwartz: Principios de Cirugía, cap. 34, pag 1580
OBJ: Hernia umbiilical al en niños TOP: Hernias de la pared abdominal
KEY: Hernia umbilical NOT: Dr. C¿Vinicio Moreno Rueda
10. ANS: A
Se describe a una hernia ingunal indirecta grande, tipo inguino-escrotal

PTS: 1 DIF: Alta


REF: Wantz, G; Hernias de la pared abdomina; en: Schwartz, Principios de Cirugía; cap 34, pág. 1567
OBJ: Hernia inguino escrotal TOP: Hernia inguinal
KEY: hernia inguinoescrotal, Nyhus NOT: Dr. Vinicio Moreno Rueda

2
LECCION [Answer Strip] ID: A

D
_____ 7.

A
_____ 1.

A
_____ 8.

D
_____ 2.
B
_____ 9.

D
_____ 3. A 10.
_____

A
_____ 4.

C
_____ 5.

B
_____ 6.
shirley
shirley

MULTIPLE CHOICE

1. a que corresponde el siguiente enunciado. Infección dermoepidérmica de rápido avance,


potencialmente transmisible y reviste mayor gravedad en niños

a. Impetigo c. Varicela
b. Celulitis d. Erisipela
ANS: A
E literal correcto es el A ya que el impetigo es una dermitis estreptocócica, la cual es una
infección dermoepidérmica de rápido avance, potencialmente transmisible y reviste
mayor gravedad en niños. Su máxima incidencia ocurre entre los 2-5 años de edad

PTS: 1 DIF: Media


REF: Comité de Enfermedades Infecciosas. Red Book 2012. Academia Americana de Pediatria
OBJ: Conocimiento KEY: Impetigo NOT: Dr, Angel Mayacela

2. Impétigo ampolloso predomina en niños pequeños. Siempre causado por


a. Streptococo c. Anaerobio
b. SBHGA d. S. Aureus
ANS: D
El literal correcto es la D ya que el impétigo ampolloso predomina en niños pequeños. Siempre es
causado por S. aureus y es el resultado de la acción de una toxina epidermolítica.

PTS: 1 DIF: Media


REF: Fonseca Capdevila, Eduardo. Dermatología 1era Edicion 2010
OBJ: Conocimiento KEY: Etiologia Impetigo
NOT: Dr, Angel Mayacela

3. Como se confirma el diagnostioc de Impetigo


a. Mediante una Biometria c. Mediante gram y cultivo del contenido
líquido o de la superficie de la lesión.

b. Mediante un Emo d. Nada de lo anterior


ANS: C
El literal correcto es C ya que se basa en el aspecto clínico de las lesiones o ampollas flácidas con
superficie erosiva. La confirmación se realizará mediante gram y cultivo del contenido líquido o
de la superficie de la lesión

PTS: 1 DIF: Media


REF: Richard E. Behrman; Robert M. Kliegman; Hal | B. Jenson. Nelson Tratado de Pediatría
Decimoséptima 2012 OBJ: Conocimiento
KEY: Diagnostico Impetigo NOT: Dr, Angel Mayacela

1
4. A que literal corresponde el siguiente enunciado. Inflamación de la dermis y TCS que
se caracteriza por edema, eritema y dolor de la zona afecta
a. Erisipela c. Impetigo
b. Celulitis d. Foliculitis
ANS: B
El literal correcto es B ya que es una inflamación de la dermis y TCS que se caracteriza por
edema, eritema y dolor de la zona afecta. Los márgenes laterales tienden a ser poco diferenciados
dada la profundidad de la infección

PTS: 1 DIF: Media


REF: Breuer K, Haussler S, Kapp A, Werfel T. | Staphylococcus aureus: colonizing features.
Br J Dermatol. 2014 OBJ: Conocimiento KEY: Definicion Celulitis
NOT: Dr, Angel Mayacela

5. Cuales son los microorganismo mas frecuentes de celulitis


a. S. pyogenes c. S. aureus
b. Candida d. A y C son correctas
ANS: D
El literal correcto es D ya que los microorganismos más frecuentemente implicados son S.
pyogenes y S. aureus, aunque en ciertas circunstancias neumococo.

PTS: 1 DIF: Media


REF: Burillo A, Moreno A, Salas C. Microbiological 2010 OBJ: Conocimiento
KEY: Etiologia Celulitis NOT: Dr. Angel Mayacela

6. Que porcentaje puede alcanzar el diagnóstico etiológico de la celulitis


a. 40% c. 50%
b. 20% d. 25%
ANS: D
el literal correcto es D ya que el diagnóstico etiológico puede conseguirse en un 25% de los casos
con hemocultivo (5%) o cultivo de aspirado de la zona de máxima inflamación

PTS: 1 DIF: Media REF: Enfermedades Infecciosas en Pediattria 2012


OBJ: Conocimiento KEY: Diagnostico Etiologico Celulitis
NOT: Dr. Angel Mayacela

7. A que literal corresponde el siguiente enunciado. Infección aguda de la piel, no necrosante, que
afecta la dermis superficial, con marcado compromiso de los vasos linfáticos subyacentes
a. Celulitis c. Impetigo
b. Erisipela d. Costra
ANS: B
El literal correcto es B ya que la erisipela es una infección aguda de la piel, no necrosante, que
afecta la dermis superficial, con marcado compromiso de los vasos linfáticos subyacentes y, en
algunos casos, rápidamente progresiva.

PTS: 1 DIF: Media


REF: Ackson MA. Bacterial Skin Infections. En: Feigin RD, Cherry JD, Demmler GJ, Kaplan
SL, eds. Feigin and Cherry's Textbook of Pediatric Infectious Diseases, 6th ed. Philadelphia: W.B
Saunders; 2014 OBJ: Conocimiento KEY: Definicion Erisipela
NOT: Dr. Angel Mayacela

2
8. Cual es el agente etiologico de la erisipela
a. SBHGA c. S. aeureus
b. Haemophilus influenzae d. Nada de lo anterior
ANS: A
el literal correcto es A ya que el SBHGA es el patógeno predominante

PTS: 1 DIF: Media


REF: Jackson MA. Bacterial Skin Infections. En: Feigin RD, Cherry JD, Demmler GJ, Kaplan
SL, eds. Feigin and Cherry's Textbook of Pediatric Infectious Diseases, 6th ed. Philadelphia: W.B
Saunders; 2014 OBJ: Conocimiento KEY: Etiologia Erisipela
NOT: Dr. Angel Mayacela

9. Señale las manifestaciones clinicas de Erisipela


a. Placa roja brillosa c. Flictenas y bullas
b. Piel de naranja d. Todo lo anterior
ANS: D
El literal correcto es D ya que la lesión típica es una placa roja brillosa, indurada ("piel de
naranja"), de bordes nítidos, dolorosa, con rápida extensión y linfedema regional. Sobre la placa,
pueden visualizarse flictenas y bullas

PTS: 1 DIF: Media


REF: Estudio multicéntrico sobre las infecciones pediátricas por Staphylococcus aureus
meticilino-resistente provenientes de la comunidad en la Argentina. Arch Argent Pediatr 2014
OBJ: Conocimiento KEY: Manifestaciones clinicas Erisipela
NOT: Dr. Angel Mayacela

10. Cuales son las complicaciones de Erisipela


a. Síndrome de shock tóxico c. Meningitis
b. Endocarditis d. Todo es correcto
ANS: D
El literal correcto es D ya que las complicaciones de la erisipela son raras e incluyen sepsis,
síndrome de shock tóxico, endocarditis y meningitis

PTS: 1 DIF: Media


REF: Lawrence H, Nopper A. Skin and Soft-Tissue Infections. En: Long S, Pickering L, Prober
C, eds. Principles and Practice of Pediatric Infectious Disease, 4th ed. Philadelphia: Churchill
Livingstone/Elsevier; 2012 OBJ: Conocimiento
KEY: Complicaciones Erisipela NOT: Dr. Angel Mayacela

3
Name: ________________________ Class: ___________________ Date: __________ ID: A

Endocrinología

Multiple Choice
Identify the choice that best completes the statement or answers the question.

____ 1.
Paciente femenina de 38 años de edad, con antecedente de hiperprolactinemia, es
referida a consulta externa de endocrinología por cambios en sus rasgos faciales,
caracterizados por prognatismo, abombamiento frontal y nariz grande, además de
aumento de tamaño de los pies y manos, tras realizarse todos los estudios pertinentes,
se indicó estudio histopatológico que informó adenomas productor de hormona de
crecimiento. Debe plantearse que podría haber otras manifestaciones clínicas, como:
a. Insomnio, irritabilidad, disminución c. Obesidad central, miopatía central,
de peso, taquicardia. estrías amplias y purpúreas.

b. Cardiomegalia, macroglosia y d. Palpitaciones, cefaleas e


agrandamiento de la glándula hiperhidrosis.
tiroides

____ 2.
Paciente masculino de 42 años de edad, acude por bocio, refiere dificultad para
concentrarse en el trabajo, aumento de peso no relacionado con mayor ingesta, ya que
además presentó pérdida de apetito, en la última semana se asocia párpados
edematosos y edema pretibial sin fóvea. De acuerdo a su sospecha diagnóstica se
puede afirmar que:
a. Se puede esperar TSH no c. Una concentración normal de TSH
suprimida y el hallazgo de un excluye el hipotiroidismo
tumor hipofisiario en la primario.
resonancia magnética

b. Existen otras manifestaciones d. En el diagnóstico diferencial hay


como proptosis y aumento de que pensar otros trastornos
tamaño de los músculos como ataques de ansiedad,
extraoculares. consumo de cocaína y
anfetaminas.

1
Name: ________________________ ID: A

____ 3.
Paciente femenina de 52 años de edad, con sobrepeso, acude por presentar poliuria,
polidipsia y pérdida de peso de 6 meses de evolución, en la última semana se asocia
debilidad y visión borrosa, su sospecha diagnostica es diabetes mellitus tipo 2, por lo
que podría esperarse los siguientes resultados de laboratorio:
a. Glicemia en ayunas mayor a c. Glucosa plasmática en ayunas
140mg/dl, con hemoglobina mayor a 126mg/dl con
glicosilada baja. hemoglobina glicosilada
elevada.

b. Glucosa plasmática dos horas d. Intolerancia a la glucosa con


posterior a sobrecarga de glicemias entre 140 a 199mg/dl
glucosa mayor a 180 mg/dl posterior a la prueba de
tolerancia a la glucosa.

____ 4.
Paciente femenina de 46 años de edad, es referida por obesidad, refiere haber seguido
diferentes dietas, además de ejercicios de moderada intensidad en los últimos 6
meses, sin lograr disminución de peso, al examen físico se constata presión arterial
146/92mmHg, piel delgada, rubicundez facial, cara redonda, giba de búfalo, estrías
violáceas, se solicita medir ACTH y se encuentra disminuida; el diagnóstico probable
es:
a. Hiperplasia suprarrenal c. Enfermedad de Cushing
congénita.

b. Feocromocitoma d. Síndrome de Cushing


independiente de ACTH.

____ 5.
Paciente femenina de 49 años de edad, en postoperatorio inmediato de tiroidectomía
total por carcinoma papilar de tiroides, que empieza con espasmos musculares y
gesticulación facial, al examen físico presenta positividad del signo de Trousseau, para
corroborar su diagnóstico se solicitaría con prioridad:
a. Densitometría ósea c. Vitamina D y magnesio.

b. Calcio y parathormona. d. Troponinas

2
Name: ________________________ ID: A

____ 6.
Paciente masculino de 57 años de edad con antecedentes de radiación de cabeza y
cuello, acude por aumento de volumen unilateral de cuello, se realiza ecografía de
tiroides y reporta nódulo tiroideo sospechoso de malignidad, con TSH, T4 libre y T3
libre en rango de la normalidad, su conducta de elección debe ser solicitar:
a. Gammagrafía tiroidea, para c. Tiroglobulina
valorar vascularidad.

b. Anticuerpos antitiroideos d. Punción por aspiración con


aguja fina.

____ 7.
Paciente masculino de 32 años de edad, que se encuentra en terapia intensiva por
Traumatismo Craneoencefálico Severo, Glasgow 8 puntos, presenta poliuria de
6.500ml en las últimas 24 horas, con sodio 145mEq/L, osmolaridad normal, el
diagnóstico más probable sería:
a. Síndrome de secreción c. Polidipsia dipsógena por
inadecuada de hormona disminución en el punto
antidiurética (SIADH). umbral del mecanismo
osmorregulador.

b. Diabetes insípida nefrógena. d. Diabetes insípida hipofisiaria o


central.

____ 8.
Dentro de los criterios de Síndrome metabólico, según ATP III, se incluyen:
a. Perímetro abdominal de menor c. Colesterol mayor a 200mg/dl y
a 94cm e obesidad central
hipercolesterolemia.

b. Triglicéridos mayor a 150mg/dl d. Colesterol HDL alto y


y presión arterial diastólica perímetro abdominal mayor
mayor a igual a 85mmHg a 102 en mujeres.

3
Name: ________________________ ID: A

____ 9.
Paciente masculino de 32 años de edad, que es referido por bocio nodular, refiere
disminución de peso, irritabilidad y deposiciones diarreicas, al examen físico se
constata: frecuencia cardíaca: 108 latidos por minuto, proptosis, piel caliente, temblor
fino en manos, se diagnosticó hipertiroidismo primario. Los exámenes complementarios
podría que sustentaron al diagnóstico son:
a. TSH elevada y T3 libre c. Anticuerpos antitiroideos
elevada, anticuerpos normales y TSH
antitiroideos elevados. elevada.

b. T4 y T3 libres elevadas, d. Anticuerpos antitiroideos


con nódulo elevados y T4 libre baja.
hipercaptante en la
gammagrafía tiroidea.

____ 10.
Las siguientes manifestaciones son parte de las complicaciones microvasculares de la
diabetes mellitus:
a. Coronariopatías, nefropatía y c. Arteriopatía periférica,
retinopatía diabética. neuropatía autonómica y
retinopatía diabética.

b. Nefropatía, retinopatía d. Edema macular, neuropatía


diabética y enfermedad sensorial y motora,
cerebrovascular. nefropatía.

4
Name: ________________________ ID: A

____ 11.
Acerca del tratamiento farmacológico de la Diabetes Mellitus, la siguiente afirmación se
considera correcta:
a. Las biguanidas disminuyen la c. Los inhibibidores de la enzima
producción hepática de dipeptidil-peptidasa IV,
glucosa y mejoran la reducen la glucemia por
utilización periférica de ese inhibición selectiva de este
azúcar. cotransportador
sodio-glucosa, que se
expresa casi de manera
exclusiva en el túbulo
contorneado proximal de
los riñones.

b. La metformina es un d. Las insulinas glargina y


secretagogo de insulina detemir, son insulinas de
que estimula la secreción acción corta.
de esta a través de la
interacción con el conducto
de potasio sensible a
trifosfato de adenosina
(ATP) de la célula beta.

____ 12.
Acerca de la fisiología de la hipófisis, la siguiente afirmación se considera correcta:
a. La neurohipófisis produce: c. Las hormonas hipofisarias raras
prolactina, vasopresina u veces provocan respuestas
hormona antidiurética y especificas en otros tejidos
oxicitocina. glandulares periféricos.
b. La hipófisisis no tiene regulación d. La adenohipófisis produce
por sistemas neuroendocrinos. hormonas importantes como:
hormona del crecimiento (GH)
hormona adrenocorticotropica
(ACTH), hormona luteinizante
(LH), hormona foliculoestimulante
(FSH) hormona estimulante de la
tiroides (TSH) y prolactina.

5
Name: ________________________ ID: A

____ 13. El hipotiroidismo es una enfermedad con relevante prevalencia en las consultas de
endocrinología, acerca del mismo, ¿cuál planteamiento se considera correcto?
a. El hipotiroidismo congénito no es c. Las manifestaciones clínicas del
frecuente, por lo que no es hipotiroidismo incluyen
necesario realizar la punción del disminución de peso, sudoración
talón como tamizaje a todo recién profusa y bradicardia.
nacido, salvo existan
manifestaciones clínicas de la
enfermedad.
b. En áreas en las que hay suficiente d. Una vez diagnosticado el
yodo, la enfermedad hipotiroidismo, el tratamiento
autoinmunitaria (tiroiditis de consiste en uso de tionamidas.
Hashimoto) es causa frecuente de
hipotiroidismo.
____ 14. La tirotoxicosis se define como el exceso de hormonas tiroideas, acerca de esta
patología, ¿Cuál enunciado se considera correcto?
a. Una de las principales causas de c. Los antitiroideos como la
la tirotoxicosis es la enfermedad levotiroxina si útiles en el
de Graves. tratamiento de los estados de
tirotoxicosis.
b. La manifestación cardiovascular d. No es necesario utilizar beta
más frecuente es la bradicardia bloqueadores para el control de
sinusal. síntomas adrenérgicos.
____ 15. A su consulta acude paciente masculino de 52 años de edad, con diagnóstico de
diabetes mellitus tipo 2 desde hace tres meses, dentro de los objetivos terapeúticos
¿Cuál enunciado es el correcto?
a. Hemoglobina glicosilada menor c. Lipoproteína de baja densidad
7%, glucosa en ayunas menor menor a 100mg/dl, hemoglobina
180mg/dl. glicosilada menor a 7%,
triglicéridos menor a 150mg/dl.
b. Hemoglobina glicosilada menor a d. Lipoproteína de baja densidad
6.5%, glucosa posprandial menor menor a 130mg/dl, hemoglobina
a 200mg/dl. glicosilada menor a 7%,
triglicéridos menor a 150mg/dl.

6
ID: A

Endocrinología
Answer Section

MULTIPLE CHOICE

1. ANS: B
Paciente femenina de 38 años de edad, con antecedente de hiperprolactinemia, es
referida a consulta externa de endocrinología por cambios en sus rasgos faciales,
caracterizados por prognatismo, abombamiento frontal y nariz grande, además de
aumento de tamaño de los pies y manos, tras realizarse todos los estudios pertinentes,
se indicó estudio histopatológico que informó adenomas productor de hormona de
crecimiento. Debe plantearse que podría haber otras manifestaciones clínicas, como:
a) Es incorrecta porque son síntomas sugerentes de hipertiroidismo
b) Es correcta porque en la acromegalia surge visceromegalias que pueden
generalizarse.
c) Es incorrecta porque son síntomas sugerentes de síndrome de Cushing.
d) Es incorrecta porque constituye la triada clásica del Feocromocitoma.

PTS: 1 DIF: Baja


REF: HARRISON Tinsley: Principios de medicina interna, 19ª Edición McGraw-Hill Interamericana de
España 2016. Pag. 2269 - 2271
OBJ: Establece un diagnóstico clínico relacionado con los trastornos de la hipófisis y el hipotálamo.
TOP: Medicina Interna - Endocrinología KEY: Acromegalia, prognatismo.
NOT: Autora: Rebeca Silvestre Ramos
2. ANS: C
Paciente masculino de 42 años de edad, acude por bocio, refiere dificultad para
concentrarse en el trabajo, aumento de peso no relacionado con mayor ingesta, ya que
además presentó pérdida de apetito, en la última semana se asocia párpados
edematosos y edema pretibial sin fóvea. De acuerdo a su sospecha diagnóstica se
puede afirmar que:
a) Es incorrecta porque estos pueden sugerir hipertiroidismo secundario.

b) Es incorrecta porque constituyen signos clínicos de oftalmopatía tiroidea.


c) Es correcta porque en el hipotiroidismo primario la TSH se encuentra elevada debido
a un mecanismo de retroalimentación.
d) Es incorrecta porque constituyen el diagnóstico diferencial de Feocromocitoma.

PTS: 1 DIF: Media


REF: HARRISON Tinsley: Principios de medicina interna, 19ª Edición McGraw-Hill Interamericana de
España 2016. Pag. 2289 - 2292.
OBJ: Identifica los trastornos de la glándula tiroidea y paratiroidea.
TOP: Medicina Interna - Endocrinología KEY: Bocio, proptosis.
NOT: Autora: Rebeca Silvestre Ramos

1
ID: A

3. ANS: C
Paciente femenina de 52 años de edad, con sobrepeso, acude por presentar poliuria,
polidipsia y pérdida de peso de 6 meses de evolución, en la última semana se asocia
debilidad y visión borrosa, su sospecha diagnostica es diabetes mellitus tipo 2, por lo
que podría esperarse los siguientes resultados de laboratorio:
a) Es incorrecta porque hay aumento de hemoglobina glicosilada
b) Es incorrecta porque la glucosa debe ser superior a 200mg/dl.
c) Es correcta porque ambas constituyen los criterios diagnósticos de diabetes mellitus
tipo 2.
d) Es incorrecta porque la intolerancia a la glucosa excluye el diagnóstico de diabetes
mellitus tipo 2.

PTS: 1 DIF: Media


REF: HARRISON Tinsley: Principios de medicina interna, 19ª Edición McGraw-Hill Interamericana de
España 2016. Pag. 2399 - 2407.
OBJ: Establece factores de riesgo para el desarrollo de Síndrome Metabólico y diabetes mellitus, para
favorecer la intervención oportuna mediante la prevención y promoción.
TOP: Medicina Interna - Endocrinología KEY: Diabetes, glucosa
NOT: Autora: Rebeca Silvestre Ramos
4. ANS: D
Paciente femenina de 46 años de edad, es referida por obesidad, refiere haber seguido
diferentes dietas, además de ejercicios de moderada intensidad en los últimos 6
meses, sin lograr disminución de peso, al examen físico se constata presión arterial
146/92mmHg, piel delgada, rubicundez facial, cara redonda, giba de búfalo, estrías
violáceas, se solicita medir ACTH y se encuentra disminuida; el diagnóstico probable
es:

a) Es incorrecta porque predominan síntomas asociados a hiperandrogenismo, los


descritos son típicos del síndrome de Cushing.
b) Es incorrecta, porque sólo coincide con la Hipertensión Arterial, sin embargo
esta es paroxística, el resto de las manifestaciones son típicas del Síndrome de
Cushing.
c) Es incorrecta porque en la Enfermedad de Cushing la ACTH se encuentra
normal o elevada.
d) Es correcta, porque son manifestaciones del exceso de glucocorticoides y en
ocasiones de mineralocorticoides.

PTS: 1 DIF: Media


REF: HARRISON Tinsley: Principios de medicina interna, 19ª Edición McGraw-Hill Interamericana de
España 2016. Pag. 2399 - 2407.
OBJ: Identifica, relaciona y aplica el conocimiento de las enfermedades de las Glándulas Suprarrenales
y gónadas. TOP: Medicina Interna - Endocrinología KEY: Suprarrenal, Cushing
NOT: Autora: Rebeca Silvestre Ramos

2
ID: A

5. ANS: B
Paciente femenina de 49 años de edad, en postoperatorio inmediato de tiroidectomía
total por carcinoma papilar de tiroides, que empieza con espasmos musculares y
gesticulación facial, al examen físico presenta positividad del signo de Trousseau, para
corroborar su diagnóstico se solicitaría con prioridad:
a) Es incorrecta porque se solicita cuando de plantea Osteoporosis, no es un
examen que se use con prioridad en estos casos.
b) Es correcta, ya que la medición de calcio será la prueba inicial para corroborar
hipocalcemia y la parathormona confirmará que el mismo es secundario a
hipoparatiroidismo.
c) Es incorrecta porque los mismos no son prioritarios, estos pueden solicitarse con
posterioridad.
d) Es incorrecta, ya que estas son de prioridad cuando se sospecha enfermedades
cardíacas.

PTS: 1 DIF: Media


REF: HARRISON Tinsley: Principios de medicina interna, 19ª Edición McGraw-Hill Interamericana de
España 2016. Pag. 2482 - 2495.
OBJ: Identifica los trastornos de la glándula tiroidea y paratiroidea.
TOP: Medicina interna - Endocrinología KEY: Calcio, vitamanina D.
NOT: Autora: Rebeca Silvestre Ramos.
6. ANS: D
Paciente masculino de 57 años de edad con antecedentes de radiación de cabeza y
cuello, acude por aumento de volumen unilateral de cuello, se realiza ecografía de
tiroides y reporta nódulo tiroideo sospechoso de malignidad, con TSH, T4 libre y T3
libre en rango de la normalidad, su conducta de elección debe ser solicitar:

a) Es incorrecto, porque la gammagrafía de tiroides se solicita cuando contamos


con perfil tiroideo compatible con nódulo tiroideo hiperduncionante o tóxico, en
ese caso T4 libre y T3 libre se encuentran elevados.
b) Es incorrecto, porque no es de elección para confirmar cáncer de tiroides,
c) Es incorrecto, porque la tiroglobulina se solicita para seguimiento postoperatorio
de cáncer de tiroides.
d) Es correcto, porque describirá las características histopatológicas para confirmar
el diagnóstico.

PTS: 1 DIF: Baja


REF: HARRISON Tinsley: Principios de medicina interna, 19ª Edición McGraw-Hill Interamericana de
España 2016. Pag. 2482 - 2495.
OBJ: Identifica los trastornos de la glándula tiroidea y paratiroidea.
TOP: Medicina Interna - Endocrinología. KEY: Nódulo. NOT: Autora: Rebeca Silvestre
Ramos

3
ID: A

7. ANS: D
Paciente masculino de 32 años de edad, que se encuentra en terapia intensiva por
Traumatismo Craneoencefálico Severo, Glasgow 8 puntos, presenta poliuria de
6.500ml en las últimas 24 horas, con sodio 145mEq/L, osmolaridad normal, el
diagnóstico más probable sería:
a) Es incorrecta, porque en el SIADH hay hiponatremia euvolémica con
hipoosmolalidad.
b) Es incorrecta, ya que por el antecedente de traumatismo craneoencefálico es
más planteable la Diabetes insípida hipofisiaria o central.
c) Es incorrecta, porque la polidipsia dipsógena se caracteriza por sed inapropiada,
un paciente con Glasgow de 8 puntos no tolera vía oral.
d) Es correcta, ya que una de las causas de Diabetes insípida hipofisiaria o central
es el traumatismo craneoencefálico y se caracteriza por poliuria importante.

PTS: 1 DIF: Media


REF: HARRISON Tinsley: Principios de medicina interna, 19ª Edición McGraw-Hill Interamericana de
España 2016. Pag. 2275 - 2280.
OBJ: Establece un diagnóstico clínico relacionado con los trastornos de la hipófisis y el hipotálamo.
TOP: Medicina Interna - Endocrinología. KEY: Diabetes insípida.
NOT: Autora: Rebeca Silvestre Ramos
8. ANS: B
Dentro de los criterios de Síndrome metabólico, según ATP III, se incluyen:

a) Es incorrecta, porque el perímetros abdominal es mayor a 102cm en mujeres y


88cm en hombres, la hipercolesterolemia no está dentro de los criterios.
b) Es correcta, porque los niveles de colesterol no son parte del Síndrome
Metabólico.
c) Es incorrecta, porque Colesterol HDL alto y perímetro abdominal mayor a 102 en
mujeres.
d) Es incorrecta, porque los valores de HDL están bajos.

PTS: 1 DIF: Media


REF: HARRISON Tinsley: Principios de medicina interna, 19ª Edición McGraw-Hill Interamericana de
España 2016. Pag. 2449 - 2454
OBJ: Establece factores de riesgo para el desarrollo de Síndrome Metabólico y diabetes mellitus, para
favorecer la intervención oportuna mediante la prevención y promoción.
TOP: Medicina Interna - Endocrinología KEY: Síndrome metabólico.
NOT: Autora: Rebeca Silvestre Ramos

4
ID: A

9. ANS: B
Paciente masculino de 32 años de edad, que es referido por bocio nodular, refiere
disminución de peso, irritabilidad y deposiciones diarreicas, al examen físico se
constata: frecuencia cardíaca: 108 latidos por minuto, proptosis, piel caliente, temblor
fino en manos, se diagnosticó hipertiroidismo primario. Los exámenes complementarios
que sustentaron al diagnóstico serían:
a) Es incorrecto, porque en el hipertiroidismo primario la TSH se encuentra baja.
b) Es correcto, ya que el perfil tiroideo se correlaciona con hipertiroidismo
primario y la gammagrafía corrobora nódulo hiperfuncionante o tóxico.
c) Es incorrecto, porque en el hipertiroidismo primario la TSH se encuentra baja.
d) Es incorrecto, porque la T4 se encuentra baja.

PTS: 1 DIF: Media


REF: HARRISON Tinsley: Principios de medicina interna, 19ª Edición McGraw-Hill Interamericana de
España 2016. Pag. 2293 - 2299
OBJ: Identifica los trastornos de la glándula tiroidea y paratiroidea.
TOP: Medicina Interna - Endocrinología. KEY: hipertiroidismo
NOT: Autora: Rebeca Silvestre Ramos
10. ANS: D
Las siguientes manifestaciones son parte de las complicaciones microvasculares de la
diabetes mellitus:

a) Es incorrecta, porque las Coronariopatías son complicaciones macrovasculares.


b) Es incorrecta, porque la enfermedad cerebrovascular es una complicación
macrovascular.
c) Es incorrecta, porque la Arteriopatía periférica, es una complicación
macrovascular.
d) Es correcta, porque las tres son complicaciones microvasculares.

PTS: 1 DIF: Media


REF: HARRISON Tinsley: Principios de medicina interna, 19ª Edición McGraw-Hill Interamericana de
España 2016. Pag. 2293 - 2299
OBJ: Establece factores de riesgo para el desarrollo de Síndrome Metabólico y diabetes mellitus, para
favorecer la intervención oportuna mediante la prevención y promoción.
TOP: Medicina Interna - Endocrinología. KEY: Diabetes. NOT: Autora: Rebeca Silvestre
Ramos

5
ID: A

11. ANS: A
Acerca del tratamiento farmacológico de la Diabetes Mellitus, la siguiente afirmación se
considera correcta:
a) Es correcta porque las biguanidas disminuyen producción hepática de glucosa y
mejoran ligeramente la utilización periférica de ese azúcar.
a) Es incorrecta. La metformina no es un secretagogo de insulina, es parte de las
biguanidas.
b) Es incorrecta. Los inhibibidores de la enzima dipeptidil-peptidasa IV, actúan
prolongando la acción de GLP-1 endógena y los inhibidores del cotransportador
sodio-glucosa son los que reducen la glucemia por inhibición selectiva de este
cotransportador sodio-glucosa, que se expresa casi de manera exclusiva en el
túbulo contorneado proximal de los riñones.
c) Las insulinas glargina y detemir, son insulinas de acción prolongada.

PTS: 1 DIF: Alta


REF: HARRISON Tinsley: Principios de medicina interna, 19ª Edición McGraw-Hill Interamericana de
España 2016. Pag. 2407 - 2422
OBJ: Establece factores de riesgo para el desarrollo de Síndrome Metabólico y diabetes mellitus, para
favorecer la intervención oportuna mediante la prevención y promoción.
TOP: Medicina Interna - Endocrinología KEY: Biguanidas NOT: Autor: Rebeca Silvestre Ramos
12. ANS: D
Acerca de la fisiología de la hipófisis, la siguiente afirmación se considera correcta:
a) Es incorrecta. En la neurohipófisis no se sintetizan hormonas, sólo se
almacenan y liberan hormonas como la vasopresina u hormona antidiurética y
oxicitocina, además la prolactina se sintetiza en la adenohipófisis.
b) Es incorrecta. La hipófisisis tiene regulación por sistemas neuroendocrinos como
el hipotálamo.
a) Es incorrecta. Las hormonas hipofisarias raras veces provocan respuestas
específicas en otros tejidos glandulares periféricos.
b) Es correcta. La adenohipófisis produce seis hormonas importantes: hormona del
crecimiento (GH) hormona adrenocorticotropica (ACTH), hormona luteinizante
(LH), hormona foliculoestimulante (FSH) hormona estimulante de la tiroides
(TSH) y prolactina.

PTS: 1 DIF: Baja


REF: HARRISON Tinsley: Principios de medicina interna, 19ª Edición McGraw-Hill Interamericana de
España 2016. Pag. 2255 - 2261.
OBJ: Establece un diagnóstico clínico relacionado con los trastornos de la hipófisis y el hipotálamo.
TOP: Medicina Interna - Endocrinología KEY: Adenohipófisis
NOT: Autor: Rebeca Silvestre Ramos

6
ID: A

13. ANS: B
El hipotiroidismo es una enfermedad con relevante prevalencia en las consultas de
endocrinología, acerca del mismo, ¿cuál planteamiento se considera correcto?:
a) Es incorrecto. La punción del talón como tamizaje se realiza a todo recién
nacido, independientemente si hay o no manifestaciones clínicas de la
enfermedad.
b) Es correcto. En áreas en las que hay suficiente yodo, la enfermedad
autoinmunitaria (tiroiditis de Hashimoto) es causa frecuente de hipotiroidismo.
c) Es incorrecto. Las manifestaciones clínicas del hipotiroidismo no incluyen
disminución de peso y sudoración profusa.
d) Es incorrecto. Una vez diagnosticado el hipotiroidismo, el tratamiento consiste en
levotiroxina, las tionamidas son para tratar casos con hipertiroidismo.

PTS: 1 DIF: Media


REF: HARRISON Tinsley: Principios de medicina interna, 19ª Edición McGraw-Hill Interamericana de
España 2016. Pag. 2289 - 2293.
OBJ: Identifica los trastornos de la glándula tiroidea y paratiroidea.
TOP: Medicina Interna - Endocrinología KEY: Hipotiroidismo
NOT: Autora: Rebeca Silvestre Ramos
14. ANS: A
La tirotoxicosis se define como el exceso de hormonas tiroideas, acerca de esta
patología, ¿Cuál enunciado se considera correcto?
a) Es correcto. Una de las principales causas de la tirotoxicosis es la enfermedad
de Graves.
b) Es incorrecto. La manifestación cardiovascular más frecuente es la taquicardia
sinusal.
c) Es incorrecto. La levotiroxina no es un antitiroideo.
d) Es incorrecto. Los beta bloqueadores son útiles para el control de síntomas
adrenérgicos.

PTS: 1 DIF: Media


REF: HARRISON Tinsley: Principios de medicina interna, 19ª Edición McGraw-Hill Interamericana de
España 2016. Pag. 2293 - 2307.
OBJ: Identifica los trastornos de la glándula tiroidea y paratiroidea.
TOP: Medicina Interna - Endocrinología KEY: Tirotoxicosis
NOT: Autora: Rebeca Silvestre Ramos

7
ID: A

15. ANS: C
A su consulta acude paciente masculino de 52 años de edad, con diagnóstico de
diabetes mellitus tipo 2 desde hace tres meses, dentro de los objetivos terapeúticos
¿Cuál enunciado es el correcto?
a) Es incorrecta. La glucosa en ayunas debe estar entre 80 – 130mg/dl.
b) Es incorrecta. La hemoglobina glicosilada debe ser menor a 7% y la glucosa
posprandial menor a 180mg/dl.
c) Es correcta. La lipoproteína de baja densidad debe ser menor a 100mg/dl, la
hemoglobina glicosilada menor a 7%, los triglicéridos menor a 150mg/dl.
d) Es incorrecta. La lipoproteína de baja densidad debe estar menos a 100mg/dl.

PTS: 1 DIF: Media


REF: HARRISON Tinsley: Principios de medicina interna, 19ª Edición McGraw-Hill Interamericana de
España 2016. Pag. 2407 - 2422.
OBJ: Establece factores de riesgo para el desarrollo de Síndrome Metabólico y diabetes mellitus, para
favorecer la intervención oportuna mediante la prevención y promoción.
TOP: Medicina Interna II - Endocrinología KEY: Diabetes.
NOT: Autora: Rebeca Silvestre Ramos

8
Endocrinología [Answer Strip] ID: A

C
_____ 3. D
_____ 6. B
_____ 9. A 11.
_____

B
_____ 1.

D
_____ 7.

D
_____ 4. D 10.
_____

C
_____ 2.

D 12.
_____

B
_____ 8.

B
_____ 5.
Endocrinología [Answer Strip] ID: A

B 13.
_____

A 14.
_____

C 15.
_____
Name: ________________________ Class: ___________________ Date: __________ ID: A

Ginecologia

Multiple Choice
Identify the choice that best completes the statement or answers the question.

____ 1. Antes de utilizar métodos anticonceptivos en pacientes con Diabetes mellitus, debe recibir una valoración inicial, la
cual debe incluir. Excepto. Seleccione una:

a. Control de peso

b. Síntomas de alerta de hipoglicemia.

c. Control de presión arterial

d. Control de glicemia por HbA1c

e. Control de glucosa capilar y sérica, función hepática y renal

____ 2. Gestante adolescente de 17 años, aqueja de prurito vulvar acompañado de secreción amarillenta de mal olor, al
examen especular se aprecia cérvix con presencia de punteado rojizo que le da la apariencia de “fresa”. Agente
etiológico más probable será:

a. Chlamydia trachomatis.

b. Cándida albicans.

c. Trichomona vaginalis.

d. Gardnerella vaginalis.

____ 3. En el control prenatal la viabilidad del embarazo se determinará entre las semanas. Seleccione la respuesta correcta.

a. 11 a 14

b. 20 a 22

c. 6a8

d. 28 a 30

e. a las 36

1
Name: ________________________ ID: A

____ 4. Paciente de 42 años, cursa embarazo de 35 semanas, las cifras tensionales en el control prenatal han sido variables
y limítrofes, usted sospecha un trastorno hipertensivo. ¿Cuándo diagnostica preeclampsia? Marque la respuesta
correcta:

a. Presión sistólica 135 mmHg más proteinuria.

b. Edad gestacional menor a 20 semanas.

c. Presión sistólica mayor de 140 mmHg más proteinuria.

d. Presión arterial menor de 140/90 mmHg más proteinuria.

____ 5. Gestante de 36 años, antecedentes de salud aparente, cursa embarazo de 32 semanas por FUM. Refiere que de
forma súbita presentó salida abundante de sangre roja rutilante de sus genitales mientras estaba acostada sin
otros síntomas acompañantes. Al examen físico se constata signos vitales dentro de parámetros normales, FCF: 152
lat/mit, actividad uterina negativa. Monitoreo fetal reactivo.
Seleccione la respuesta correcta.

a. Rotura uterina.

b. Placenta previa

c. Desprendimiento placentario.

____ 6. Paciente de 35 años, antecedentes de cesárea anterior hace 4 años por distocia de presentación. Esta en trabajo de
parto en fase activa, de pronto presenta dolor abdominal intenso, ausencia de movimientos fetales y se observa
en el abdomen el “signo de reloj de arena”. Usted sospecha de rotura uterina, según la profundidad, las roturas
uterinas se clasifican en: Seleccione la respuesta correcta.

a. Espontánea y traumática.

b. Longitudinal, oblicua y transversal.

c. Completa e incompleta.

d. Segmentarias y corporales.

____ 7. ¿Cuál es la meta en la glucosa sanguínea durante el embarazo en una mujer con diabetes gestacional?
Seleccione la respuesta correcta

a. Glucosa en ayuno mayor a 100mg/Dl

b. Glucosa dos horas postprandial mayor a 120mg/dL

c. Glucosa una hora postprandial mayor a 130mg/dL

d. Glucosa en ayuno menor a 90mg/dL

2
Name: ________________________ ID: A

____ 8. Paciente de 38 años de edad, antecedentes de salud aparente, último parto hace 10 años; cursa embarazo normal
de 12,6 semas por ECO, asintomática. Usted la valora por primera vez y realiza el primer control prenatal en su área
de salud, constata un IMC de 26. ¿Escoja previo análisis cuál sería la conducta adecuada con esta paciente?
1. Anamnesis, examen físico completo y solicitar exámenes.
2. Anamnesis, examen físico completo y transferencia.
3.Iniciar tratamiento con 1.5 g de calcio elemental dividido en 3 tomas después de las comidas y
Ácido acetil salicílico 75-100 mg diario.
4. Transferencia a hospital de tercer nivel.

a. 1,3
b. 2
c. 3,4
d. 4,1

____ 9. Usted ingresa en hospitalización a paciente de 24 años de edad, primigesta, nulípara, que cursa embarazo de 31
semanas por FUM, constata al examen físico: FCF: 145 latidos/minuto, actividad uterina: 3/10/30 segundos, al
tacto vaginal: cérvix central de consistencia blanda, borrado 50%, dilatado 2- 3 cm, cefálico móvil, membranas
íntegras, pelvis útil. Seleccione la opción que muestra los objetivos terapéuticos a seguir según su criterio
diagnóstico.

a. Reposo absoluto, ecografía obstétrica, maduración pulmonar, terminar el embarazo una


vez completada la maduración pulmonar

b. Tocólisis, maduración pulmonar, exámenes para determinar la causa, transferencia a


unidad de neonatología especializada

c. Reposo absoluto, maduración pulmonar, ecografía transvaginal, terminación inmediata


del embarazo por la vía alta

____ 10. Gestante de 19 años, antecedentes de salud anterior, cursa embarazo de 20 semanas. Acude a subcentro de salud
refiriendo disuria, polaquiuria, tenesmo vesical y dolor lumbar bilateral, afebril. Trae examen de orina elemental
y microscópico que informa: orina turbia, nitritos negativos, leucocitos 80, bacterias +++, bacilos gram negativos
numerosos. Seleccione la opción correcta en cuanto a la opción terapéutica para su paciente.

a. Cefazolina 1 g IV cada 6 horas

b. Ampicilina 1 g IV cada 6 horas

c. Nitrofurantoina 100 mg VO cada 12 horas

d. Cefalexina 500 mg VO cada 6 horas

3
ID: A

Ginecologia
Answer Section

MULTIPLE CHOICE

1. ANS: B
OPCIÓN DE RESPUESTA CORRECTA:

B. Síntomas de alerta de hipoglicemia, estos se recomiendan en la asesoría de cuidado neonatal¸pero, no se incluye


en la valoración incial para decider método de anticoncepción.

RESPUESTA INCORRECTA 1.
A.Control de peso, si es un parámetro a valorar descrito dentro de los cuidados posnatales en pacientes con
Diabetes Mellitus.
RESPUESTA INCORRECTA 2.
C.Control de presión arterial, si es un parámetro a valorar descrito dentro de los cuidados posnatales en pacientes
con Diabetes Mellitus.
RESPUESTA INCORRECTA 3
D. Control de glicemia por HbA1c, si es un parámetro a valorar descrito dentro de los cuidados posnatales en
pacientes con Diabetes Mellitus.
RESPUESTA INCORRECTA 4
E. Control de glucosa capilar y sérica, función hepática y renal, si es un parámetro a valorar descrito dentro de los
cuidados posnatales en pacientes con Diabetes Mellitus.

RESPUESTA INCORRECTA 1.
II: A, D:
A. La HPP primaria o inmediata es aquella que se produce dentro de las primeras 24 horas posteriores al parto.
Aproximadamente, el 70% de los casos de HPP inmediata se producen debido a atonía uterina. Se define como
atonía del útero a la incapacidad del útero de contraerse adecuadamente después del nacimiento del producto.
D. Atonía uterina: es la causa más común e importante de HPP. El mecanismo primario de hemostasia inmediata
luego del parto es la contracción miometrial, que produce la oclusión de los vasos sanguíneos uterinos miometriales
que pasan entre las células musculares del útero, cuando esto no ocurre es por un fallo en este mecanismo.

RESPUESTA INCORRECTA 2.
III. B, C:
B Retención de restos placentarios: sangrado variable: leve o profuso, continuo o irregular que continúa luego de 12
horas después del parto, útero más blando y más grande que lo previsto según el tiempo transcurrido desde el
parto. Cursa con anemia.
C. Hemorragia posparto secundaria o tardía: La HPP secundaria o tardía se produce entre las 24 horas y seis
semanas posparto. La mayoría de los casos de HPP tardía se deben a la retención de productos de la concepción,
infección o ambas.

RESPUESTA INCORRECTA 3.
IV: C, D:
C. Hemorragia posparto secundaria o tardía: La HPP secundaria o tardía se produce entre las 24 horas y seis
semanas posparto. La mayoría de los casos de HPP tardía se deben a la retención de productos de la concepción,
infección o ambas.
D. Atonía uterina: es la causa más común e importante de HPP. El mecanismo primario de hemostasia inmediata
luego del parto es la contracción miometrial, que produce la oclusión de los vasos sanguíneos uterinos miometriales
que pasan entre las células musculares del útero, cuando esto no ocurre es por un fallo en este mecanismo.

RESPUESTA INCORRECTA 4.
V: B, D:
B Retención de restos placentarios: sangrado variable: leve o profuso, continuo o irregular que continúa luego de 12
horas después del parto, útero más blando y más grande que lo previsto según el tiempo transcurrido desde el
parto. Cursa con anemia.
D. Atonía uterina: es la causa más común e importante de HPP. El mecanismo primario de hemostasia inmediata
luego del parto es la contracción miometrial, que produce la oclusión de los vasos sanguíneos uterinos miometriales
que pasan entre las células musculares del útero, cuando esto no ocurre es por un fallo en este mecanismo.

La pregunta puede ser respondida empleando entre 1 y 3 minutos.

1
ID: A

PTS: 3 DIF: Alto


REF: Guía de Práctica Clínica Diagnóstico y tratamiento de la diabetes en el embarazo (pregestacional y
gestacional), 2014. Página 32, 33 OBJ: Comprensión y aplicación de los conceptos
TOP: Ginecología y Obstetricia
KEY: Métodos anticonceptivos en pacientes con Diabetes mellitus
2. ANS: C
RESPUESTA CORRECTA:

C. Trichomona vaginalis, protozoo patógeno flagelado, produce secreción verde amarillenta espumosa, cérvix con
presencia de punteado rojizo que le da la apariencia de “fresa.

RESPUESTA INCORRECTA 1.
A. Chlamydia trachomatis, es una bacteria intracelular obligado, produce generalmente dolor al orinar, dolor
genital, secreción vaginal y sangrado entre periodos menstruales.
RESPUESTA INCORRECTA 2.
B. Cándida albicans, es un hongo diploide asexual, con forma de levadura, produce secreción blanca grumosa
abundante, acompañada de prurito intenso.
RESPUESTA INCORRECTA 3
C. Gardnerella vaginalis, bacteria inmóvil, anaerobia facultativa, suele producir secreción vaginal de color
grisácea con olor a pescado.
Para su solución se deben emplear entre 3 y 4 minutos.

PTS: 3 DIF: Alta


REF: Guía de Práctica Clínica Diagnóstico y tratamiento de la infección vaginal en obstetricia, 2014.
Página 12- 15 OBJ: Aplicación de teoría diagnóstica TOP: Ginecología y Obstetricia
KEY: Agente etiológico durante infección vaginal
3. ANS: C
RESPUESTA CORRECTA:

C. 6 a 8, entre estas semanas se confirma la viabilidad del embarazo y la edad gestacional.

RESPUESTA INCORRECTA 1.
A. 11 a 14, se realiza ecografía para determinar marcadores ecográficos de AC.
RESPUESTA INCORRECTA 2.
B. 20 a 22, se realiza una ecografía morfológica fetal (detalle anatómico).
RESPUESTA INCORRECTA 3
D. 28 a 30, se realiza ecografía para valorar crecimiento fetal, perímetro abdominal fetal y determinación
de percentil para descartar macrosomía fetal incipiente.
RESPUESTA INCORRECTA 4
E. a las 36, se realiza ecografía para valorar crecimiento fetal y el volumen del líquido amniótico.
Tiempo estimado para respuesta 1-3 min.

PTS: 3 DIF: Alto


REF: Guía de Práctica Clínica Diagnóstico y tratamiento de la diabetes en el embarazo (pregestacional y
gestacional), 2014. Página 23 OBJ: Determinar tiempo de inicio de control prenatal
KEY: Control prenatal

2
ID: A

4. ANS: C
RESPUESTA CORRECTA:

C. Presión sistólica mayor de 140 mmHg más proteinuria: si, porque los criterios son: TA: mayor o igual a 140mmHg
y/o 90 mmHg, más proteinuria.

RESPUESTA INCORRECTA 1.
A. Presión sistólica 135 mmHg más proteinuria: los criterios son: TA: mayor o igual a 140mmHg y/o 90 mmHg,
más proteinuria.
RESPUESTA INCORRECTA 2.
B. Edad gestacional menor a 20 semanas: no, la preeclampsia se presenta en embarazos con 20 semanas
cumplidas o más.
RESPUESTA INCORRECTA 3
D. Presión menor de 140/90 mmHg más proteinuria: los criterios son: TA: mayor o igual a 140mmHg y/o 90 mmHg,
más proteinuria.
Tiempo estimado de respuesta: 3 min

PTS: 3 DIF: Alta


REF: Guía de práctica clínica Trastornos hipertensivos del embarazo, 2016. Página 14-15
OBJ: Diagnosticar preeclampsia TOP: Ginecología y Obstetricia
KEY: Preeclampsia
5. ANS: B
RESPUESTA CORRECTA:

B. Placenta previa. Las características del sangrado descrito en el caso clínico corresponden a esta patología,
además que en la misma no hay alteración de la FCF, no hay dolor y tampoco tiene antecedentes de enfermedad
hipertensiva del embarazo, traumatismo abdominal grave o cordón abdominal típicamente corto que nos haga
pensar en otra patología (DPPNI).

RESPUESTA INCORRECTA 1.
A. Rotura uterina. En esta patología generalmente hay antecedentes de cicatriz uterina anterior, uso inapropiado
de uterotónicos para inducción y conducción del parto, frecuentemente el útero no se palpa y hay signos de shock
hipovolémico y no es el caso de esta paciente.
RESPUESTA INCORRECTA 2.
C.Desprendimiento placentario. Esta patología se caracteriza por presentar generalmente un sangrado oscuro,
puede haber alteración de la FCF, cursa con dolor abdominal e hipertonía uterina y frecuentemente hay
antecedentes de enfermedad hipertensiva del embarazo, traumatismo abdominal grave, cordón umbilical
atípicamente corto y, no es el caso de esta paciente.
Tiempo estimado de respuesta: 3 min

PTS: 3 DIF: Alta


REF: Componente Normativo Materno Neonatal 2012. Página 103, 104, 105.
OBJ: Diagnóstico en hemorragias obstétricas TOP: Ginecología y Obstetricia
KEY: Hemorragias obstétricas

3
ID: A

6. ANS: C
RESPUESTA CORRECTA:

3. Completa e incompleta:
ROTURA UTERINA COMPLETA: Involucra todo el espesor de la pared uterina e incluso el peritoneo visceral. El
producto y la placenta pueden exteriorizarse del útero de forma total o parcial.
ROTURA UTERINA INCOMPLETA
Rotura parcial del espesor de la pared uterina, se considera además la dehiscencia de una incisión uterina por
cirugía previa.

RESPUESTA INCORRECTA 1.
1. Espontánea y traumática: es la clasificación según la causa de la rotura uterina, no según la profundidad.
RESPUESTA INCORRECTA 2.
2. Longitudinal, oblicua y transversal: no especifican la profundidad.
RESPUESTA INCORRECTA 3.
4. Segmentarias y corporales: no especifican la profundidad, sino localización (en tipos de cesárea).

Respuesta para 3 min.

PTS: 3 DIF: Alto


REF: Componente Normativo materno neonatal 2012. Página 101.
OBJ: Manejo de claves obstétricas NAT: Diagnóstico de la rotura uterina
TOP: Ginecología y Obstetricia KEY: Rotura uterina
7. ANS: D
RESPUESTA CORRECTA:

D. Glucosa en ayuno menor a 90mg/Dl, es la meta en la glucosa sanguínea durante el embarazo en una mujer con
diabetes gestacional.

RESPUESTA INCORRECTA 1.
A. Glucosa en ayuno mayor a 100mg/Dl, es incorrecta. a meta en la glucosa sanguínea durante el embarazo en una
mujer con diabetes gestacional es ayuno menor a 90 mg/dL.
RESPUESTA INCORRECTA 2.
B. Glucosa dos horas postprandial mayor a 120mg/dL, es incorrecta. a meta en la glucosa sanguínea durante el
embarazo en una mujer con diabetes gestacional dos horas posprandial menor a 120 mg/dL.
RESPUESTA INCORRECTA 3
C. Glucosa una hora postprandial mayor a 130mg/Dl, es incorrecta. a meta en la glucosa sanguínea durante el
embarazo en una mujer con diabetes gestacional se valora en ayuno y dos horas posprandial.

Tiempo estimado para respuesta 3 min

PTS: 3 DIF: Medio


REF: Guía de Práctica Clínica Diagnóstico y tratamiento de la diabetes en el embarazo (pregestacional y
gestacional), 2014. Página 25 OBJ: Realizar diagnóstico
TOP: Ginecología y Obstetricia KEY: Glucosa sanguínea durante el embarazo

4
ID: A

8. ANS: A
RESPUESTA CORRECTA:

A.1,3. En la primera consulta prenatal se valora y examina a la paciente de forma integral, se solicitan todos los
exámenes y se clasifica a la paciente tomando en cuenta los factores de riesgo que presente. En este caso tiene dos
factores de riesgo moderado: un periodo intergenésico prolongado y un IMC de 26 por lo que se inicia con calcio y
aspirina a las dosis indicadas desde las 12 semanas de embarazo hasta el final de este como prevención primaria de
la preeclampsia, no amerita transferencia al momento ya que esta asintomática.

RESPUESTA INCORRECTA 1.
B.2. No amerita transferencia al momento ya que la paciente esta asintomática, se puede manejar en la atención
primaria.
RESPUESTA INCORRECTA 2.
C.3,4. No amerita transferencia al momento ya que la paciente esta asintomática, se puede manejar en la atención
primaria.
RESPUESTA INCORRECTA 3.
D.4,1. No amerita transferencia al momento ya que la paciente esta asintomática, se puede manejar en la atención
primaria.

3 min. para responder.

PTS: 3 DIF: Alto


REF: Guía de Práctica Clínica Control Prenatal 2016. Página 14. | Guía de Práctica Clínica Trastornos
Hipertensivos del Embarazo 2016. Página 18. OBJ: Valoración en consulta prenatal
TOP: Ginecología y Obstetricia KEY: Consulta prenatal
9. ANS: B
RESPUESTA CORRECTA:

2. Tocólisis, maduración pulmonar, exámenes para determinar la causa, transferencia a unidad de neonatología
especializada. Estos son los objetivos terapéuticos de una amenaza de parto pretérmino orientados en las guías del
MSP: tocólisis para evitar que el bebé nazca y, maduración pulmonar y transferencia a unidad de neonatología
especializada para brindarle mayor posibilidad de sobrevida al bebé y los exámenes para determinar a causa y
tratarla.

RESPUESTA INCORRECTA 1.
1. -Reposo absoluto, ecografía obstétrica, maduración pulmonar, terminar el embarazo una vez completada la
maduración pulmonar. Se trata de un embarazo de 31 semanas con una amenaza de parto pretérmino, el objetivo
es lograr con la conducta adecuada que no nazca el bebé, no interrumpirle el embarazo. El reposo y la maduración
pulmonar están indicados, la ecografía que se debe realizar es transvaginal no solamente obstétrica.
RESPUESTA INCORRECTA 2.
3. -Reposo absoluto, maduración pulmonar, ecografía transvaginal, terminación inmediata del embarazo por la vía
alta. Se trata de un embarazo de 31 semanas con una amenaza de parto pretérmino, el objetivo es lograr con la
conducta adecuada que no nazca el bebé, no interrumpirle el embarazo. El reposo y la maduración pulmonar están
indicados, la ecografía que se debe realizar es transvaginal no solamente obstétrica.

Tres min para realizar respuesta.

PTS: 3 DIF: Alto


REF: Guía de Práctica Clínica Prevención, ruptura prematura de membranas pretérmino 2015. Página
22. | Componente Normativo Materno Neonatal 2012. Página 133. OBJ: Realizar diagnóstico
probable
TOP: Ginecología y Obstetricia

5
ID: A

10. ANS: C
RESPUESTA CORRECTA:

3.Nitrofurantoina 100 mg VO cada 12 horas. Ante el diagnóstico de Infección de vías urinarias baja, el tratamiento
de elección es la nitrofurantoína de liberación retardada 100 mg consta como medicamento de primera línea en el
tratamiento tanto de la cistitis como de la bacteriuria asintomática, según evidencia de farmacoresistencia local en
Ecuador.
Se considera que se requerirán de 1 a 3 minutos para su respuesta.

RESPUESTA INCORRECTA 1.
1. cefazolina 1 g IV cada 6 horas. No es un medicamento de primera línea, se puede indicar bajo otras
circunstancias.
RESPUESTA INCORRECTA 2.
2.Ampicilina 1 g IV cada 6 horas. No es un medicamento de primera línea, se puede indicar bajo otras
circunstancias.
RESPUESTA INCORRECTA 3.
4.Cefalexina 500 mg VO cada 6 horas. No es un medicamento de primera línea, se puede indicar bajo otras
circunstancias.
Tres min para responder

PTS: 3 DIF: Alto


REF: Guía de Práctica Clínica Infección de Vías Urinarias en el Embarazo 2013. Página 15, 16, 17, 18,
19.
OBJ: Diagnósticar durante embarazo TOP: Ginecología y Obstetricia

6
Ginecologia [Answer Strip] ID: A

C
_____ 4. A
_____ 8.

B
_____ 1.

B
_____ 5.
B
_____ 9.

C
_____ 2.

C
_____ 6.

C 10.
_____
C
_____ 3.

D
_____ 7.
Name: ________________________ Class: ___________________ Date: __________ ID: A

MEDICINA INTERNA - HEMATOLOGIA

Multiple Choice
Identify the choice that best completes the statement or answers the question.

____ 1. CASO CLINICO


Paciente masculino de 35 años de edad, procedente de La Paz, APP y APF sin importancia. Presentó
cuadro clínico de 6 meses de evolución caracterizado por astenia, adinamia y disnea progresiva. En el
examen físico presentó parálisis de Bell derecha, adenomegalias cervicales, esplenomegalia, ruidos
respiratorios rudos, sibilancias en proyección de lóbulos superiores. El hemograma reportó hemoglobina
9,6 g/ dL, leucocitos 15900/µl, neutrófilos 1000/µl, linfocitos 9700/µl, monocitos 5200/µl y plaquetas
67000/µl. La ecografía abdominal reportó adenopatías mesentéricas, esplenomegalia y derrame pleural
bilateral. Se realizó estudio morfológico de sangre periférica y médula ósea observándose infiltrado de
blastos linfoides en 80%. Aspirado medular presencia de infiltrado de blastos linfoides de 90%,
caracterizados por presencia de nucléolos prominentes, escaso citoplasma agranular. El estudio de
inmunofenotipo mostró población clonal de blastos linfoides del 90% con expresión positiva de
CD19/CD10, CD7, TdT, cCD79á/cCD3, CD5/CD22, siendo MPO, CD13, IgM negativos.
1. QUE SINDROMES ENCONTRAMOS EN ESTE CASO CLINICO
a. SD. ANEMICO, SD ESPLENOMEGALICO, SD. ADENOPATICO, SD. DE
INSUFICIENCIA MEDULAR GLOBAL
b. SD. ANEMICO, SD ESPLENOMEGALICO, SD. ADENOPATICO, SD.
BICITOPENIA, SD. DISGLOBULINEMICO.
c. SD. ANEMICO, SD ESPLENOMEGALICO, SD. ADENOPATICO, SD.
POLICITEMICO.
d. SD. ANEMICO, SD ESPLENOMEGALICO, ADENOPATICO, DE INSUFICIENCIA
MEDULAR GLOBAL. SD GRANULOCITOPENICO

1
Name: ________________________ ID: A

____ 2. CASO CLINICO


Paciente masculino de 35 años de edad, procedente de La Paz, APP y APF sin importancia. Presentó
cuadro clínico de 6 meses de evolución caracterizado por astenia, adinamia y disnea progresiva. En el
examen físico presentó parálisis de Bell derecha, adenomegalias cervicales, esplenomegalia, ruidos
respiratorios rudos, sibilancias en proyección de lóbulos superiores. El hemograma reportó hemoglobina
9,6 g/ dL, leucocitos 15900/µl, neutrófilos 1000/µl, linfocitos 9700/µl, monocitos 5200/µl y plaquetas
67000/µl. La ecografía abdominal reportó adenopatías mesentéricas, esplenomegalia y derrame pleural
bilateral. Se realizó estudio morfológico de sangre periférica y médula ósea observándose infiltrado de
blastos linfoides en 80%. Aspirado medular presencia de infiltrado de blastos linfoides de 90%,
caracterizados por presencia de nucléolos prominentes, escaso citoplasma agranular. El estudio de
inmunofenotipo mostró población clonal de blastos linfoides del 90% con expresión positiva de
CD19/CD10, CD7, TdT, cCD79á/cCD3, CD5/CD22, siendo MPO, CD13, IgM negativos.
Cuadro No. 1
Criterios diagnósticos para Leucemia Aguda de linaje ambiguo del
European Group for the Inmunological Classification of Leukemias
(EGIL)
Puntos Linaje B Linaje T Linaje mieloide
2 CD 79a CD3 (cyt/m) Anti-MPO
cyt IgM Anti-TCR á/â (anti-lisozima)
CD22 Anti-TCR ã/ä
1 CD19 CD2 CD117 (c-kit)
CD10 CD5 CD13
CD20 CD8 CD33
CD10 CD65s
0.5 TdT TdT CD7 CD14
CD24 CD1á CD15
CD64

2. EN EL PRESENTE CASO CLINICO QUE TIPO DE LEUCEMIA ENCONTRAMOS


a. LEUCEMIA AGUDA BIFENOTIPICA (B - T)
b. LEUCEMIA LINFOBLASTICA AGUDA TIPO B
c. LEUCEMIA MIXTA: MIELOBLASTICA Y LINFOBLASTICA
d. LEUCEMIA MIELOBLASTICA AGUDA

____ 3. Interprete el siguiente hermograma que pertenence a un niño de 6 años que llega al hospital con fiebre y
aftas orales: hb 14 g/dl, leucocitos 2.8 x 109/L, (segmentados 13%, linfocitos 82% monocitos 5%)
plaquetas 210000.

a. Tiene una linfocitosis, que es c. Tiene una neutropenia


probablemente virica
b. Tiene un alinfocitosis, lo que es normal d. Este hemograma es normal,
para la edad de este niño independientemente de la edad

____ 4. En un paciente tratado durante durante años con vitamina B12 parenteral por anemia perniciosa, con buena
respuesta, aparece anemia microcitica moderada y progresiva. Señale cùal es la explicacion mas probable
de esta evolucion:
a. La vitamina B12 es inadecuada c. Tiene un deficit asociado de acido fòlico
b. Ha desarrollado anemia ferropenica por d. Es portador de enfermedad celiaca
gastritis atrofica

2
Name: ________________________ ID: A

____ 5. Paciente de 35 años, diagnosticado de artritis reumatoide de 8 años de evolucion ya tratada con AINEs
por via oral, Presenta una anemia Hipocroma con niveles de ferritina serica normales. Se quiere descartar
una carencia de hierro. Cuàl serìa la prueba disgnostica definitiva?
a. Realizar un estudio ferrocinetico c. Estudiar el frotis de sangre periferica
b. Estudiar por microscopia optica los d. Medir niveles de G6P dehidrogenasa en
depositos de hierro de la medula osea hematies

____ 6. Una mujer de 52 años presenta un sindrome anemico sin causa clinicamente aparente. En la analitica se
aprecia HB de 8.5 g/dl, VCM de 75 fl, y CHM de 25 pg, Cuàl de las siguientes determinaciones sericas le
permitira orientar con seguridad el origen ferropenico de la anemia?
a. Sideremia c. Capacidad de fijacion del hierro
b. Saturacion de la transferrina d. Ferritina

____ 7. Paciente de 28 años, sin antecedentes de haber recibido quimiterapia ni radioterapia previamente, con
leucopenia de 1.300/ul, trombopenia de 25.000/ul y anemia de 7g/dl de Hb, sin blastos en sangre
periferica y con aspirado de medula osea muy hipocelular. Señale que prueba de confirmacion esta
indicada y cual es el diagnostico mas problable:
1. Biopsia de medula osea a. Aplasia Medular
2. Estudio citoquimico b. Leucemia Aguda
3. Estudio Citogenetico c. Linfoma con invasion de la medula osea

a. 1a c. 3c
b. 2a d. 2c

____ 8. Un paciente presenta el siguiente hemograma: Hb 9g/dl, VCM 120fl, HCM 34pg, reticulocitos 12%,
leucocitos 9500/mm3, plaquetas 320000/mm3. En el frotis se observa hipersegmentacion de neutrofilos.
El diagnostico es de:
1. Anemia a. Megaloblastica
2. Leucemia b. Hemolitica
3. Trombocitopenia c. Aguda

a. 1a c. 3c
b. 2b d. 1c

____ 9. Enferma de 78 años de edad enviada al hospital por astemia progresiva y anemia. Los datos hematologicos
son VS 90, Hb 9g/dl, Hcto 24% VCM 102 fl, HCM 30 pg, CHCM 34g/fl, reticulocitos 1%, leucocitos
7300. Las cifras de acido folico y votamina B12 son normales. El diagnostico probable sera:
a. Anemia por deficiencia de hierro c. Sindrome mielodisplasico
b. Leucemia linfocitica cronica d. Anemia hemolitica autoinmune

____ 10. Un paciente presenta trombocitopenia y el estudio de medula osea revela una cifra de megacariocitos
normal o elevada. Que diagnostico considera menos probable?

a. Purpura trombocitopènica idiopatica c. Sindrome mielodisplásico


b. Hiperesplenismo d. Purpura trombótica trombocitopénica

3
Name: ________________________ ID: A

____ 11. En un paciente con 60 años, buen estado general, exploracion física normal y un hemograma con
leucocitos 70000, (90% linfocitos) sin anemia, ni trombopenia el disgnostico mas probable y tratamiento
mas adecuado son:

1. Leucemia linfatica Crónica a. Tratamiento no precisa


2. Leucemia linfatica Crónica estadio A b. Tratamiento Poliquimioterapia
3. Leucemia prolinfocítica c. Tratamiento Fludarbina

a. 1b c. 3a
b. 2a d. 2c

____ 12. ¿En cuál de estos casos de enfermedad de Hodgkin, estaría claramente indicada la laparotomía en el
diagnostico de extension?
1. Varon de 40 años a. Sin sintomas B.
con afectacion de Sin estudio de
ganglios medula osea
laterocervicales
bilaterales,
retroperitonales
2. Mujer de 33 años b. Con sintomas B
con afectacion de
ganglios
laterocervicales,
axilares y
mediantínicos (mas
de 1/3 de diametro
anteroposterior
taràcico) con medula
osea norma
3. Varon de 18 Años c. Sin Sintomas B
con afectación
laterocervical y
supracalvicular
ipsilateral, con
medula osea normal

a. 1a c. 2b
b. 3c d. 1c

____ 13. Ante un paciente de 61 años, diagnosticado de Mieloma Multiple y que se mantiene asintomatico:

a. No está justificado comenzar con c. Interesa comenzar precozmente el


tratamiento quimioterapeutico tratamiento poco agresivo con Melfalán y
antitumoral prednisona
b. Tratamiento con pulsos de dexametasona d. Se debe comenzar tratamiento con poli
quimiterapia tipo VBMCP

4
Name: ________________________ ID: A

____ 14. Una mujer de 29 años consulta porque tiene menstruaciones muy abundantes, de duracion normal y
porque la heridas le sangran mas de lo habitual. Como antecedente refiere que 2 años antes tuvo
hemorragia posparto copiosa que retraso el alta hospitalaria. Como antecedente familiar destaca que su
abuela materna tambien tubo hemorragias posparto inusuales. El hemograma fue completamente normal y
el tiempo de hemorragia se prolongó hasta 10 minutos. ¿Cuál es el trastorno de coagulación que padece?

a. Purpura trombocitopénica diopática c. Enfermedad de Von Willebrand tipo II


(PTI)
b. Enfermedad de Von Willebrand tipo I d. Hemofilia A

____ 15. Un paciente de 65 años padece fibrilacion auricular crónica, con historia de embolia cerebral un año antes.
Esta anticoagulado con acenocumarol oral. Qué actitud, entre las siguientes, es la mas adecuada?

a. Mantener el cociente internacional c. Mantener el INR entre 5 y 6


normalizado INR entre 2 y 3
b. Mantener el INR entre 4 y 5 d. Mantener el INR entre 3 y 4 y añadir 200
mg de aspirina al dia

____ 16. Una mujer de 68 años sin aparentes factores de reisgo cardiovascular ingresa en la unidad coronarias del
hospital por un cuadro agudo de cardiopatia isquemica. En la analìtica realizada a su llegada se objetiva
una anemia (Hb 8 g/dl) previamente no conocida. En este caso, la actitud mas adecuada con respecto a la
anemia es:

a. Transfundir hematies c. Instaurar tratamiento con eritropoyetina


b. Transfundir sangre total d. Administrar hierro intravenoso

____ 17. Paciente presenta anemia de 10 g de Hb, hierro serico de 30 mug/dl (valores normales 50-150) y TIBC
(capacidad de transporte de hierro) de 200 mug/dl (valores normales 250 a 370). Con mayor probabilidad
se trata de una anemia:

a. Ferropenica c. Sideroblastica
b. De trastornos cronicos d. Embarazo

____ 18. Un paciente con enfermedad de Hodgkin que recibio multiples lineas de quimioterapia y un autotrasplante
de mèdula òsea, del que se recuperò adecuadamente que permanece en remision, presenta 3 años mas
tarde la siguiente analìtica: Hb 80 g/lm leucocitos 1,2x109/l y plaquetas 30x109/l. El diagnostico mas
problable entre los siguientes es:

a. Sindrome mielodisplasico secundario c. Pancitopenia autoinmune


b. Aplasia medular d. Hepatopatia crònica con
hiperesple-nismo

5
Name: ________________________ ID: A

____ 19. 1. Antecedentes


Niña de 12 años concurre a realizarse un estudio clínico por presentar decaimiento, palidez y leve
ictericia.
2. Examen físico
Ictericia y leve esplenomegalia.
3. Laboratorio
Hemograma
Hematocrito: 32 %
Hemoglobina: 11.4 g/100ml Reticulocitos : 8.5 %
Glóbulos rojos: 3.800.000 /μL Leucocitos : 8.000 /μL
CHCM: 34.8 % Plaquetas : 280.000 /μL
VCM: 84.6 fL
Glóbulos rojos: Anisocitosis leve, poiquilocitosis moderada
Leucocitos: Normales Fórmula leucocitaria: Normal
Prueba de fragilidad osmótica: aumentada.
Bilirrubina total: 16 mg/l
Bilirrubina indirecta: 14 mg/l
Ferremia
Hierro sérico: 120 μg/dL
TIBC: 215 μg/dL
Saturación de transferrina: 51.2 %
Ferritina: 101 μg/dL

Electroforesis de hemoglobina: No se observan bandas anormales.

¿Qué tipo de anemia presenta esta paciente? Analice los parámetros alterados.

1. Anemia Esferosis Hereditaria A. Hematocrito, Hb., Reticulocitos,


Prueba de Fragilidad Osmotica,
Bilirrubina Total, TIBC, Saturacion de
Transferrina

2. Anemia Ferropenica B. CHCM aumentado, Glóbulos Rojos


Aumentados, Reticulositos aumentados

3. Anemia Megaloblastica C.CHCM disminuidos, Glóbulos Rojos


Aumentados, Reticulositos aumentados

4. Anemia Aplasica D. Hematocrito, Hb., TIBC, Saturacion


de Transferrina

a. 1A c. 3C
b. 2A d. 1D

6
Name: ________________________ ID: A

____ 20. 1Paciente de sexo masculino de 35 años de edad consulta al médico por cuadro de dolor lumbar, por
más de una semana.
Tiene el antecedente de haber recibido terapia con hierro en su consultorio de salud desde la infancia y
hasta los 16 años por la presencia de una leve anemia, aparentemente de etiología ferropénica, luego de lo
cual se pierde de control.
2. Examen físico
Buen estado general, conjuntivas rosadas sin cianosis ni ictericia.
3. Laboratorio
Hemograma
Hematocrito: 36 %
Hemoglobina: 11,4 g/dL Leucocitos: 8.300 /ìL
Glóbulos Rojos: 4.900.000 /ìL Plaquetas: 160.000 /ìL
CHCM : 31.7 g/dL
VCM: 73.5 fL ADE 17%
Glóbulos Rojos: Anisocitosis moderada, microcitosis leve a moderada, hipocromía leve a moderada,
poiquilocitosis leve, codocitos escasos,
policromatofilia leve.
Ferremia
Hierro sérico : 82 ìg/dL (V.N: 50-150 ìg/dL)
TIBC : 290 ìg/dL (V.N.: 200-400 ìg/dL)
Saturación de transferrina : 31% (30±15%)
Electroforesis de hemoglobina: Hemoglobina A2: 5,2%.

¿Qué tipo de anemia presenta esta paciente? Analice los parámetros alterados y normales.

1. Anemia Megaloblastica A. Hematocrito, Hb., GR, VCM, ADE

2. Anemia Ferropenica B. Hto levemente aumentado, CHCM


aumentado levemente

3. Anemia Depranocitica C. Hto levemente disminuido, CHCM


aumentado levemente

4. Anemia B -talasemia D. Hematocrito, Hb., GR, VCM, ADE,


Electroforesis de HB: HbA2 elevada

a. 1B c. 3C
b. 2A d. 4D

7
Name: ________________________ ID: A

____ 21. 1. Antecedentes


Varón de 15 años de edad comenzó de forma aguda con un cuadro caracterizado por fiebre de 39ºC,
escalofríos y dolor faríngeo. Consulta en un centro hospitalario donde se le diagnostica faringitis y es
tratado con antibióticos y antiinflamatorios sin mejoría. Dos semanas más tarde acude a un servicio de
urgencia con un cuadro de astenia y anorexia, sin pérdida de peso, faringitis y molestias abdominales, en
hipocondrio derecho. Además refería febrícula vespertina intermitente, sin prurito ni sudoración y la
aparición de adenopatía dolorosas, También refiere la emisión de orinas de color oscuro durante la última
semana.
2. Examen físico
Se observan varias adenopatías latero cervicales y una axilar de pequeño tamaño y leve ictericia.
3. Laboratorio
Hemograma
Hematocrito: 44 %
Hemoglobina: 15.5 g/dL Leucocitos : 15.000 /μL
Glóbulos rojos : 5.100.000 /μL Plaquetas : 200.000 /μL
CHCM : 35.2 %
VCM : 86.2 fL
Glóbulos rojos : Anisocitosis leve.
Leucocitos : 21% de linfocitos de mediano tamaño, con citoplasma abundante e
hiperbasófilo (linfocitos reactivos).
Proteínas totales : 8 g/dL
Transaminasa Glutámico Oxaloacética : 68 UI/L
Transaminasa Glutámico Pirúvica: 120 U I/L
Fosfatasa alcalina: 105 UI/L
Bilirrubina directa: 4 mg/L
Bilirrubina indirecta: 11 mg/L
Bilirrubina total: 15 mg/L
Nota. Generalmente no se solicitan, pero en este caso los anticuerpos anti-Hepatitis (A y B), anti-
Citomegalovirus y anti-HIV, estuvieron negativos, y los anticuerpos específicos anti-virus Epstein Barr,
IgM e IgG IFI (Inmunofluorescencia indirecta) fueron positivos.
¿Qué patología presenta el paciente?

a. Anemia Hemolitica c. Hemofilia


b. Mononucleosis Infecciosa d. Leucemia Aguda

____ 22. Paciente de 63 años que esta en tratamiento con heparina I.V. y desarrolla un cuadro compatible con
apendicitis que requiere intervencion en las proximas 24 horas. Que actitud plantearia?

1. Pasar a heparina de bajo PM A. Operar pasadas al menos 6 horas

2.Administrar plasma fresco congelado B. Operar pasadas al menos 3 horas

3. Suspender la heparina C. Operar pasadas al menos 4 horas

a. 1A c. 2B
b. 3A d. 1C

8
Name: ________________________ ID: A

____ 23. Paciente de 20 años de edad que sufre una hemorragia moderada despues de una extraccion dentaria. Se
le practica une studio de hemostasia y se encuentra un tiempo de sangrìa alargado. ¿En cuàl de estos
procesos pensaria en primer lugar?

a. Hemofilia A c. Enfermedad de von Willebrand


b. Anticoagulante circulante d. Hepatopatia

____ 24. Paciente varon de 65 años, fumador esporadico, con 17.5 g/dl de Hb, Hcto 59%, masa eritrocitaria elevada,
saturacion de oxigeno en sangre arterial nayor o igual a 92%, plaquetas de 512000, eritropoyetina en
suero de 1mU/ml (normal 6-25 mU/ml) y carboxihemoglobina de 0.5% (normal inferior al 1%). El
diagnostico mas probable es:

a. Poliglobulia del fumador c. Policitemia de stress


b. Riñon Poliquistico d. Policitemia Vera

____ 25. En una paciente joven, con buen estado general, que presenta una anemia ferropenica que no responde al
tratamiento con hierro oral, sin ninguna evidencia de sangradom cual de las siguientes pruebas
diagnosticas debe practicarse en primer lugar?

a. Cuantificacion del hierro en orina c. Determinacion de anticuerpos


antiendomisio y antigliadina (IgG e IgA)
b. Estudio de la medula osea d. Arteriografia Abdominal

9
ID: A

MEDICINA INTERNA - HEMATOLOGIA


Answer Section

MULTIPLE CHOICE

1. ANS: B
PRESENTA LOS SINDROMES ANEMICO POR ASTENIA Y DISNEA, SD. ADENOPATICO POR
LOS ADENOMEGALIAS CERVICALES. SD. ESPLENOMEGALICO POR LA ESPLENOMEGALIA,
SD. DISGLOBULINEMICO POR LA ANEMIA.

PTS: 1 DIF: ALTA


OBJ: PLANIFICACION DE ACCIONES: | La planificación es un proceso de toma de decisiones para
alcanzar un futuro deseado, teniendo en cuenta la situación actual y los factores internos y externos que
pueden influir en el logro de los objetivos TOP: MEDICINA INTERNA-HEMATOLOGIA
NOT: DR. HIPOLITO PAULA
2. ANS: A
Este caso es de un de un paciente diagnosticado con leucemia bifenotípica B/T cuya
población clonal presentó positividad para marcadores linfoides, obteniéndose una puntuación de 6,5 para
el linaje B y 4,5 para el linaje T según los criterios de EGIL.

PTS: 1 DIF: ALTA


OBJ: PLANIFICACION DE ACCIONES: | La planificación es un proceso de toma de decisiones para
alcanzar un futuro deseado, teniendo en cuenta la situación actual y los factores internos y externos que
pueden influir en el logro de los objetivos TOP: MEDICINA INTERNA-HEMATOLOGIA
NOT: DR. HIPOLITO PAULA
3. ANS: C PTS: 1 DIF: Alta TOP: Medicina Interna
4. ANS: B PTS: 1 DIF: Alta TOP: Medicina Interna
5. ANS: B PTS: 1 DIF: Alta TOP: Medicina Interna
6. ANS: D PTS: 1 DIF: Alta TOP: Medicina Interna
7. ANS: A PTS: 1 DIF: Alta TOP: Medicina Interna
8. ANS: A PTS: 1 DIF: Alta TOP: Medicina Interna
9. ANS: C PTS: 1 DIF: Alta TOP: Medicina Interna
10. ANS: C PTS: 1 DIF: Alta TOP: Medicina Interna
11. ANS: B PTS: 1 DIF: Alta TOP: Medicina Interna
12. ANS: B PTS: 1 DIF: Alta TOP: Medicina Interna
13. ANS: A PTS: 1 DIF: Alta TOP: Medicina Interna
14. ANS: B PTS: 1 DIF: Alta TOP: Medicina Interna
15. ANS: A PTS: 1 DIF: Alta TOP: Medicina Interna
16. ANS: A PTS: 1 DIF: Alta TOP: Medicina Interna
17. ANS: B PTS: 1 DIF: Alta TOP: Medicina Interna
18. ANS: A PTS: 1 DIF: Alta TOP: Medicina Interna
19. ANS: A PTS: 1 DIF: Alta TOP: Medicina Interna
20. ANS: D PTS: 1 DIF: Alta TOP: Medicina Interna
21. ANS: B PTS: 1 DIF: Alta TOP: Medicina Interna
22. ANS: B PTS: 1 DIF: Alta TOP: Medicina Interna
23. ANS: C PTS: 1 DIF: Alta TOP: Medicina Interna
24. ANS: D PTS: 1 DIF: Alta TOP: Medicina Interna
25. ANS: C PTS: 1 DIF: Alta TOP: Medicina Interna

1
MEDICINA INTERNA - HEMATOLOGIA [Answer Strip] ID: A

A
_____ 2. B
_____ 5. B 11.
_____ B 14.
_____

B
_____ 1.
D
_____ 6.

A 15.
_____

B 12.
_____
A
_____ 7.

A 16.
_____

A
_____ 8.

B 17.
_____

A 18.
_____
C
_____ 9.

C
_____ 3.

C 10.
_____
A 13.
_____

B
_____ 4.
MEDICINA INTERNA - HEMATOLOGIA [Answer Strip] ID: A

A 19.
_____ D 20.
_____ B 21.
_____ C 23.
_____

D 24.
_____

C 25.
_____

B 22.
_____
Name: ________________________ Class: ___________________ Date: __________ ID: A

REACTIVO 2 INFECTOLOGIA . DRA RIOS

Multiple Response
Identify one or more choices that best complete the statement or answer the question.

____ 1. Paciente masculino de 54 años de edad con cuadro de 3 días de evolución consistente en dolor de inicio progresivo en
hemiabdomen superior, acompañado de fiebre, nauseas, vomito, tras haber ingerido alimentos grasos desde hace 48
horas.
AP: Hipertension arteroial tratado con enalapril, Diabetes Mellitus: en tratamiento con insulina
AQ: herniorrafia Inguinal derecha hace 1 año.
Al examen físico: T/A: 90/50 paciente somnoliento, FC: 92 x min, FR: 22 x min, T°: 38.1, conjuntivas ictéricas,
mucosas secas, ruidos cardiacos rítmicos, pulmones ventilados, murmullo vesicular normal, , abdomen distendido, piel
brillante, perístasis ausente, dolor generalizado a la palpación, mayor intensidad en epigastrio e hipocondrio derecho.
Extremidades simétricas sin edema, pulsos distales presentes..

a. Sepsis c. Shock septico


b. Sepsis severa d. Shock septico mas disfuncion
multiorganica

____ 2. Niña de 10 años de edad acude a Centro de Salud de la región Costa, presentando cuadro clínico de 2 días
de evolución con fiebre, hiporexia, decaimiento, mialgias, artralgias y nauseas que han llegado al vómito
en dos ocasiones en las últimas 12 horas. No presenta síntomas gripales. Reporta orina normal, última
micción hace 3 horas. En su barrio se han presentado casos de dengue.

Al examen físico: TA 100/70, T” 38 grados, Peso 30 Kg. Ingresa ambulatoria. No hay ictericia, no
rinorrea. Oídos normales. Orofaringe algo congestiva, amígdalas normales. Cuello: no adenopatías, no
retracción supraclavicular. Tórax: no tirajes, murmullo vesicular conservado. Corazón: FC en 90 por
minuto, no soplos. Abdomen: no distendido, suave, depresible, no refiere dolor a la presión abdominal.
No hay hepatomegalia. En piel no hay petequias ni equimosis, presencia de un rash ligero en tórax.

De acuerdo a las respuestas anteriores, la paciente se encuentra en qué fase de la enfermedad:

a. Perido de incubacion c. Fase febril


b. Fase de defervescencia o critica d. Fase de recuperacion

1
Name: ________________________ ID: A

____ 3. Niña de 12 años de edad acude a Centro de Salud de la región Costa, presentando cuadro clínico de 2
días de evolución con fiebre, hiporexia, decaimiento, cefalea retroorbitaria, mialgias, artralgias y
nauseas que han llegado al vómito en dos ocasiones en las últimas 12 horas. No presenta síntomas
gripales. Reporta orina normal, última micción hace 3 horas. En su barrio se han presentado casos de
dengue.

Al examen físico: TA 100/70, T” 38 grados, Peso 30 Kg. Ingresa ambulatoria. No hay ictericia, no
rinorrea. Oídos normales. Orofaringe algo congestiva, amígdalas normales. Cuello: no adenopatías, no
retracción supraclavicular. Tórax: no tirajes, murmullo vesicular conservado. Corazón: FC en 90 por
minuto, no soplos. Abdomen: no distendido, suave, depresible, no refiere dolor a la presión abdominal.
No hay hepatomegalia. En piel no hay petequias ni equimosis, presencia de un rash ligero en tórax

Dentro deel tratamiento ideal para esta paciente incluye

a. Hidratacion oral, nada nutricion oral, c. Hidrtatacion intravenosa y referencia a


control cada 24 horasa un Centro hospitalario de forma
inmediata
b. Hidratacion oral, acetaminofen, d. Hidratacion intravenosa y control de
informacion sobre signos de alarma, uso ingesta/excreta cada hora
de toldo y control cada 24 hora

____ 4. La paciente vuelve al Centro de Salud a las 48 horas. Niña decaída, afebril, no come, ha vomitado por seis
ocasiones en las últimas 8 horas. Presentó epistaxis por dos ocasiones en las últimas ocho horas. Se
queja de dolor abdominal. Ultima diuresis hace 6 horas, orina normal, no diarreas. Al examen físico:
Temperatura en 37 grados centígrados, TA 92/54, Pulso en 90 por minuto. Ambulatoria, decaída, facies
de dolor, no hay ictericia, orofaringe congestiva, cuello sin adenopatías. Tórax: murmullo vesicular
normal, no retracciones intercostales. Corazón: rítmico, no soplos. Abdomen: hepatomegalia, dolor
intenso a la palpación abdominal, con signos de ascitis, ruidos hidroaéreos normales. Piel con petequias
a nivel de tórax.

La paciente presenta signos de alarma los cuales son:

a. El eneunciado es incorrecto, la paciente c. Vomito, decaimienro, petequias


no tiene signos de alarma
b. No tiene fiebre, esta taquicardica, tiene d. Vomito persistente, hepatomegalia, dolor
petequias abdominal, epistaxis

____ 5. Un paciente con paludismo cerebral puede presentarl, escoja el literal incorrecto

1. Se debe sospechar en personas que han viajado recientemente a áreas endémicas de paludismo
2. Tiene un cuadro clínico inicial de fiebre, letargo u otros signos neurológicos.
3. El paludismo fulminante es causado por Plasmodium falciparum
4. La rigidez de nuca y la fotofobia son inusuales.
5. Origina temperatura >40°C, hipotensión, ictericia, síndrome de insuficiencia respiratoria aguda del adulto y
hemorragia.

a. 1,2,3 c. 1,2,5
b. 2,3,4 d. 2,3,5

2
Name: ________________________ ID: A

____ 6. Niña de 13 años de edad acude a Centro de Salud de la región Costa, presentando cuadro clínico de 2
días de evolución con fiebre, hiporexia, decaimiento, mialgias, artralgias y nauseas que han llegado al
vómito en dos ocasiones en las últimas 12 horas. No presenta síntomas gripales. Reporta orina normal,
última micción hace 3 horas. En su barrio se han presentado casos de dengue.

Al examen físico: TA 100/70, T” 38 grados, Peso 30 Kg. Ingresa ambulatoria. No hay ictericia, no
rinorrea. Oídos normales. Orofaringe algo congestiva, amígdalas normales. Cuello: no adenopatías, no
retracción supraclavicular. Tórax: no tirajes, murmullo vesicular conservado. Corazón: FC en 90 por
minuto, no soplos. Abdomen: no distendido, suave, depresible, no refiere dolor a la presión abdominal.
No hay hepatomegalia. En piel no hay petequias ni equimosis, presencia de un rash ligero en tórax.

El diagnóstico presuntivo inicial

a. Dengue grave c. Dengue sin signos de alambra


b. Dengue con signos de alarma d. No tiene dengue

____ 7. Paciente de 18 años , sin antecedentes patologicos de importancia con cuadro clinico de 3 dias por presentar
fiebre no cuantificada acompañada de escalofrios , mialgias, artralgias y cefalea. El dia de ingreso 2 horas
previas presenta epistaxis. Al EF se encuentra orientado, colaborador, mucosas humedas y rosadas,
pilmones bien ventilados. TA: 120/80 FC 86 FR:18. No dolor abdominal, no vomito y el sangrado ya
cedio en el momento de la evaluacion. Al momento se encuentra febril.
Con los datos de la HCl cual cree Ud es el diagnostico del paciente

a. Fiebre Amarilla c. Dengue


b. Enfermedad de Weil d. Sindrome icterico hemorragico

____ 8. Paciente de 18 años , sin antecedentes patologicos de importancia con cuadro clinico de 3 dias por presentar
fiebre no cuantificada acompañada de escalofrios , mialgias, artralgias y cefalea. El dia de ingreso 2 horas
previas presenta epistaxis. Al EF se encuentra orientado, colaborador, mucosas humedas y rosadas,
pilmones bien ventilados. TA: 120/80 FC 86 FR:18. No dolor abdominal, no vomito y el sangrado ya
paro en el momento de la evaluacion. Al momento se encuentra febril.
Su su diagnostico fue Dengue, en que grupo clinico lo ubicaria

a. Dengue grave c. Dengue sin signos de alarma


b. Dengue con signos de alarma d. Dengue hemorragico

____ 9. Paciente de 18 años , sin antecedentes patologicos de importancia con cuadro clinico de 3 dias por presentar
fiebre no cuantificada acompañada de escalofrios , mialgias, artralgias y cefalea. El dia de ingreso 2 horas
previas presenta epistaxis. Al EF se encuentra orientado, colaborador, mucosas humedas y rosadas,
pilmones bien ventilados. TA: 120/80 FC 86 FR:18. No dolor abdominal, no vomito y el sangrado ya
paro en el momento de la evaluacion. Al momento se encuentra febril.
De acuerdo a su evaluacion que examenes de laboratorio considera importante para el manejo y
evaluacion inicial del paciente

a. TP/TTP/plaquetas, hematocrito,/Na/K/Ca c. TP/TTP/plaquetas/hematocrito/amonio/p


rueba de torniquete
b. TP/TTP/plaquetas/hematocrito/amonio/f d. Plaquetas/hematocrito
osforo

3
Name: ________________________ ID: A

____ 10. Paciente de 23 años, acude al centro de salud con cuadro clinico de 4 dias de evolucion caracterizado por
fiebre, artralgias, mialgias y cefalea, ademas dolor abdominal de un dia de evolucion de gran intensidad.
Presenta vomito por 4 ocaciones. Al EF hay disminucion de murmullo vesicular en ambas bases con rash
en extrenidades , piel sudorosa y fria. TA: 90/60 FC: 110 FR: 22 , T 36°C.
Los examenes de laboratorio indican Hto 52% , Plaquetas 12.000 . Leucocitos 2.500 Linfocitos 60%
Cual es el diagnostico mas apropiado

a. Leptospirosis c. Dengue grave


b. Meningitis bacteriana d. Dengue hemorragico

____ 11. Mujer de 20 años, cursa con embarazo de 9 semanas, con antecedentes de infeccion por Toxoplasma 7
meses antes de la concepcion
a. La incidencia de infeccion transplacentaria sera minima (alrededor del 15%)
b. La enfermedad del recien nacido sera mas grave
c. La incidencia de la infeccion transplacentaria sera maxima (alrededor del 65%)
d. La enfermedad del recien nacido sera menos grave
e. El riesgo de infeccion congenita es nulo
a. La incidencia de infeccion c. La enfermedad del recien nacido sera
transplacentaria sera minima (alrededor menos grave
del 15%)
b. La enfermedad del recien nacido sera d. El riesgo de infeccion congenita es nulo
mas grave

____ 12. Paciente masculino de 54 años de edad con cuadro de 3 días de evolución consistente en dolor de inicio
progresivo en hemi abdomen inferior, acompañado de fiebre, nauseas, vomito, manifestó ausencia de
deposiciones y flatos desde hace 48 horas.
APP: HTA en tto con enalapril, D Mellitus: en tto con insulina NPH 25 UI cada 24 horas
AQ: herniorrafia Inguinal derecha hace 1 año.
AF: Madre con Ca de Mama.
Al examen físico: Peso: 78Kg, Talla: 163cm, T/A: 90/50 paciente consiente alerta de regular condición
general con signos vitales; FC: 122 x min, FR: 25 x min, T°: 37.9, cojuntivas rosaceas, mucosas secas,
ruidos cardiacos rítmicos, pulmones bien ventilados, murmullo vesicular universal, disnea en reposo,
abdomen distendido, piel brillante, peristalsis ausente, dolor generalizado a la palpación, mayor
intensidad en hipogastrio y fosa iliaca derecha, con signo de rovsing positivo, signo de irritación
peritoneal en todo hemiabdomen inferior. Extremidades simétricas sin edema, pulsos distales presentes.
Preclínicos: Hg: 7.1, HTO: 22, Leucocitos: 14,800. Neutrofilos: 89% Linfocitos:10%, PCR 36
Creatinina 1.6
ECOGRAFÍA: Liquido libre en cavidad peritoneal, con área engrosada de intestino delgado de contenido
liquido heterogéneo de 6.5 x 7.5cm. ubicada en fosa iliaca derecha

Cual seria su diagnostico

a. Sepsis por Neumonia d. Sepsis y disfuncion organica


b. SIRS por Neumonia e. Sepsis por infeccion abdominal
c. SIRS por infeccion abdominal

4
Name: ________________________ ID: A

____ 13. Según la clasificación de SEPSIS, a cual etapa correspondería y cuál sería la causa de base probable de
su origen.

Paciente de 65 años, con antecedentes de hipertensión y diabetes mellitus desde hace 9 años, presenta
desde hace 4 horas somnolencia, y alza térmica no cuantificada. Es llevada al hospital y encuentran a
paciente con Glasgow de 11, autonomía respiratoria, ´pero con mala mecánica, TA: 90/50 FC: 120 FR
28 Disminución de murmullo vesicular y rales en ambas bases pulmonares. Corazón normal.
Abdomen normal. Laboratorio:: Leucocitos 13.600 Hto 32 Creatinina 1.8 . Bilirrubina total 3.3 mg/dl
BD: 1.5 mg/dl

a. SIRS c. Shock Séptico


b. Sepsis d. Sepsis y disfuncion orgánica

____ 14.
Según el las manifestaciones del paciente , que valor de escala de SOFAq presenta:
Paciente de 65 años, con antecedentes de hipertensión y diabetes mellitus desde hace 9 años, presenta
desde hace 4 horas somnolencia, y alza térmica no cuantificada. Es llevada al hospital y encuentran a
paciente con Glasgow de 11, autonomía respiratoria, ´pero con mala mecánica, TA: 105/50 FC: 120 FR
28 Disminución de murmullo vesicular y rales en ambas bases pulmonares. Corazón normal.
Abdomen normal. Laboratorio:: Leucocitos 13.600 Hto 32 Creatinina 1.8 . Bilirrubina total 3.3 mg/dl
BD: 1.5 mg/dl

a. SOFAq de 1 = Sepsis c. SOFAq 2 = Sepsis


b. SOFAq de 1= Sin sepsis d. SOFAq 2 = Sin sepsis

____ 15. Paciente masculino, de 34 años de edad, residente del Puyo Trabajó anteriormente en el campo. Como
antecedentes de importancia: alcoholismo y tabaquismo positivos. Actualmente trabaja cortando madera
en Arajuno Previamente laboró. Presenta tatuaje en antebrazo izquierdo, desconoce medidas higiénicas
del lugar donde se lo realizó.
Acudió a consulta al Centro de Salud , por haber presentado en Octubre pápula pequeña en antebrazo
derecho. Ésta inició en mayo del mismo año, acompañada de prurito, la cual fue creciendo
progresivamente hasta “reventar y salirle líquido claro”, y posteriormente ulcerarse. En junio , presentó el
mismo tipo de lesión, en cara (frente), pabellón auricular, en región posterolateral izquierda de cuello,
con el mismo patrón de evolución que la primera lesión. Refiere que todas se acompañan de prurito
intenso, son indoloras. Se aplicó remedios caseros, después tomó trimetoprim-sulfametoxazol e
itraconazol sin presentar mejoría.
A la exploración física presenta dermatosis diseminada que afecta la cara (frente y debajo del labio
inferior), el pabellón auricular izquierdo, la región posterolateral izquierda del cuello y la extremidad
superior derecha, en el nivel del antebrazo. Predominio de las lesiones en cara. Respeta tronco y
extremidades inferiores, así como extremidad superior izquierda. Asimétrica. Las lesiones ulceradas son
de de 3 cm x 3 cm, la mayor (localizada en región frontal), con depresión central y bordes elevados,
indurados y bien delimitados. Base limpia, sin salida de secreción purulenta, con costra melicérica en
región inferior. Todas las lesiones indoloras.
Cual seria la Impresion Diagnostica

a. Herpes tipo I c. Leishmaniasis


b. Varicela Zoster d. Fasceitis Necrotizante

5
Name: ________________________ ID: A

____ 16. Paciente masculino, de 34 años de edad, residente del Puyo Trabajó anteriormente en el campo. Como
antecedentes de importancia: alcoholismo y tabaquismo positivos. Actualmente trabaja cortando madera
en Arajuno Previamente laboró. Presenta tatuaje en antebrazo izquierdo, desconoce medidas higiénicas
del lugar donde se lo realizó.
Acudió a consulta al Centro de Salud , por haber presentado en Octubre pápula pequeña en antebrazo
derecho. Ésta inició en mayo del mismo año, acompañada de prurito, la cual fue creciendo
progresivamente hasta “reventar y salirle líquido claro”, y posteriormente ulcerarse. En junio , presentó el
mismo tipo de lesión, en cara (frente), pabellón auricular, en región posterolateral izquierda de cuello,
con el mismo patrón de evolución que la primera lesión. Refiere que todas se acompañan de prurito
intenso, son indoloras. Se aplicó remedios caseros, después tomó trimetoprim-sulfametoxazol e
itraconazol sin presentar mejoría.
A la exploración física presenta dermatosis diseminada que afecta la cara (frente y debajo del labio
inferior), el pabellón auricular izquierdo, la región posterolateral izquierda del cuello y la extremidad
superior derecha, en el nivel del antebrazo. Predominio de las lesiones en cara. Respeta tronco y
extremidades inferiores, así como extremidad superior izquierda. Asimétrica. Las lesiones ulceradas son
de de 3 cm x 3 cm, la mayor (localizada en región frontal), con depresión central y bordes elevados,
indurados y bien delimitados. Base limpia, sin salida de secreción purulenta, con costra melicérica en
región inferior. Todas las lesiones indoloras.
Cual serian los examenen que solicitaria para el diagnostico

a. Frotis de histopatologico de la lesion d. a,b

b. Elisa, IFI.9 e. b,c

c. Métodos inmunológicos incluyendo f. Todas


intradermorreacción de Montenegro

6
Name: ________________________ ID: A

____ 17. Paciente masculino, de 34 años de edad, residente del Puyo Trabajó anteriormente en el campo. Como
antecedentes de importancia: alcoholismo y tabaquismo positivos. Actualmente trabaja cortando madera
en Arajuno Previamente laboró. Presenta tatuaje en antebrazo izquierdo, desconoce medidas higiénicas
del lugar donde se lo realizó.
Acudió a consulta al Centro de Salud , por haber presentado en Octubre pápula pequeña en antebrazo
derecho. Ésta inició en mayo del mismo año, acompañada de prurito, la cual fue creciendo
progresivamente hasta “reventar y salirle líquido claro”, y posteriormente ulcerarse. En junio , presentó el
mismo tipo de lesión, en cara (frente), pabellón auricular, en región posterolateral izquierda de cuello,
con el mismo patrón de evolución que la primera lesión. Refiere que todas se acompañan de prurito
intenso, son indoloras. Se aplicó remedios caseros, después tomó trimetoprim-sulfametoxazol e
itraconazol sin presentar mejoría.
A la exploración física presenta dermatosis diseminada que afecta la cara (frente y debajo del labio
inferior), el pabellón auricular izquierdo, la región posterolateral izquierda del cuello y la extremidad
superior derecha, en el nivel del antebrazo. Predominio de las lesiones en cara. Respeta tronco y
extremidades inferiores, así como extremidad superior izquierda. Asimétrica. Las lesiones ulceradas son
de de 3 cm x 3 cm, la mayor (localizada en región frontal), con depresión central y bordes elevados,
indurados y bien delimitados. Base limpia, sin salida de secreción purulenta, con costra melicérica en
región inferior. Todas las lesiones indoloras.
La prueba de la intradermorreacción de Montenegro fue positiva.
Que tratamiento administraria

a. Ceftriaxona 1 g intravenoso cada 12 c. Antimonio pentavalente de 20 mg/kg de


horas por 7 días peso/día dosis unica durante 20 días

b. Ciprofloxacino 500 mg via oral cada 12 d. Ampicilina 1 g via intravenosa cada 6


horas por 30 días horas por 7 días

____ 18. Paciente de 33 años, sexo femenino, procedente de Guayaquil. Oficinista quien unos días atrás paso en el
campo donde tuvo contacto con aves, perros y gatos, pero no con vacunos, equinos. Bebió agua
embotellada y todos los alimentos que ingirió fueron cocidos. Ninguno de los que estuvieron con ella
enfermeraron. La paciente frecuentaba desde hacía años la casa de un familiar que tenía un loro.
Acude por presenter fiebre
Enfermedad actual. Sana hasta 15 días antes en que comenzó a tener dolores en cuello y nuca los que
aumentaban con los movimientos. Consultó médico que le indicó una radiografía de columna cervical,
informando el radiólogo la existencia de pinzamientos intervertebrales. Una semana después agregó
fiebre de hasta 39ºC, decaimiento y anorexia leves. Fue internada en el hospital. A los 4 días presentó
diarrea líquida sin sangre ni mucus y no acompañada de dolores abdominales, motivo por el cual se le
inició tratamiento empírico con ciprofloxacina 500 mg intravenoso c/8 horas.
El examen físico fue siempre negativo salvo dolor a la movilización de columna cervical.
Exámenes: Hb 13 g %, leucocitos 4.200/mm3, PMN 70%. L 28%. Plaquetas normales.
Bilirrubinas normales, TGO 88 UK/ml, TGP 102 UK/ml. Glucemia y azoemia normales.
Radiografía de tórax normal. Ecografía abdominal normal.
Ademas se solicitaron estudios serológicos para Chlamydia psitacci, fiebre Q, toxoplasmosis

Cual seria la impresion diagnostica

a. Toxoplasmosis c. Fiebre Tifoidea


b. Tracoma d. Giardiasis

7
Name: ________________________ ID: A

____ 19. Paciente de 33 años, sexo femenino, procedente de Guayaquil. Oficinista quien unos días atrás paso en el
campo donde tuvo contacto con aves, perros y gatos, pero no con vacunos, equinos. Bebió agua
embotellada y todos los alimentos que ingirió fueron cocidos. Ninguno de los que estuvieron con ella
enfermeraron. La paciente frecuentaba desde hacía años la casa de un familiar que tenía un loro.
Acude por presenter fiebre
Enfermedad actual. Sana hasta 15 días antes en que comenzó a tener dolores en cuello y nuca los que
aumentaban con los movimientos. Consultó médico que le indicó una radiografía de columna cervical,
informando el radiólogo la existencia de pinzamientos intervertebrales. Una semana después agregó
fiebre de hasta 39ºC, decaimiento y anorexia leves. Fue internada en el hospital. A los 4 días presentó
diarrea líquida sin sangre ni mucus y no acompañada de dolores abdominales, motivo por el cual se le
inició tratamiento empírico con ciprofloxacina 500 mg intravenoso c/8 horas.
El examen físico fue siempre negativo salvo dolor a la movilización de columna cervical.
Exámenes: Hb 13 g %, leucocitos 4.200/mm3, PMN 70%. L 28%. Plaquetas normales.
Bilirrubinas normales, TGO 88 UK/ml, TGP 102 UK/ml. Glucemia y azoemia normales.
Radiografía de tórax normal. Ecografía abdominal normal.
Ademas se solicitaron estudios serológicos para Chlamydia psitacci, fiebre Q, toxoplasmosis

a. Serológicos para Chlamydia psitacc c. Aglutinacioes febriles


b. Serológico para toxoplasma d. Hemocultivo

____ 20. Paciente de 33 años, sexo femenino, procedente de Guayaquil. Oficinista quien unos días atrás paso en el
campo donde tuvo contacto con aves, perros y gatos, pero no con vacunos, equinos. Bebió agua
embotellada y todos los alimentos que ingirió fueron cocidos. Ninguno de los que estuvieron con ella
enfermeraron. La paciente frecuentaba desde hacía años la casa de un familiar que tenía un loro.
Acude por presenter fiebre
Enfermedad actual. Sana hasta 15 días antes en que comenzó a tener dolores en cuello y nuca los que
aumentaban con los movimientos. Consultó médico que le indicó una radiografía de columna cervical,
informando el radiólogo la existencia de pinzamientos intervertebrales. Una semana después agregó
fiebre de hasta 39ºC, decaimiento y anorexia leves. Fue internada en el hospital. A los 4 días presentó
diarrea líquida sin sangre ni mucus y no acompañada de dolores abdominales, motivo por el cual se le
inició tratamiento empírico con ciprofloxacina 500 mg intravenoso c/8 horas.
El examen físico fue siempre negativo salvo dolor a la movilización de columna cervical.
Exámenes: Hb 13 g %, leucocitos 4.200/mm3, PMN 70%. L 28%. Plaquetas normales.
Bilirrubinas normales, TGO 88 UK/ml, TGP 102 UK/ml. Glucemia y azoemia normales.
Hemocultivo: Salmonela Thipy.
Ud trataria con
a. Ampicilina clavulanico 1 g intravenoso c. Ciprofloxaciono 400 mg via oral cada 6
cada 12 horas por 7 días horas por 7 días
b. Ciprofloxaciono 500 mg via oral cada 12 d. Ampicilina sulbactam 2 gramos
horas por 7 días intravenoso cada 12 horas por 7 días

8
ID: A

REACTIVO 2 INFECTOLOGIA . DRA RIOS


Answer Section

MULTIPLE RESPONSE

1. ANS: D PTS: 1 DIF: Media


REF: Principios de Medicina Interna de Harrison. 18° Edciion.
OBJ: Realiza correctamente el analisis e interpretacion con actitud critica de las acciones para llegar al
diagnostico y tratamiento de pacientes con sepsis y sus complicaciones
TOP: Sepsis
2. ANS: C PTS: 1 DIF: Media
REF: Principios de Medicina Interna de Harrison. 18° Edciion.
OBJ: Realiza correctamente el analisis e interpretacion con actitud critica de las acciones para llegar al
diagnostico y tratamiento de las principales enfermedades infecciosas tropicales de nuestro pais
TOP: Enfermedades tropicales - Dengue
3. ANS: B PTS: 1 DIF: Media
REF: Principios de Medicina Interna de Harrison. 18° Edciion.
OBJ: Realiza correctamente el analisis e interpretacion con actitud critica de las acciones para llegar al
diagnostico y tratamiento de las principales enfermedades infecciosas tropicales de nuestro pais
TOP: Enfermedades Tropicales. Dengue
4. ANS: D PTS: 1 DIF: Media
REF: Principios de Medicina Interna de Harrison. 18° Edciion.
OBJ: Realiza correctamente el analisis e interpretacion con actitud critica de las acciones para llegar al
diagnostico y tratamiento de las principales enfermedades infecciosas tropicales de nuestro pais
TOP: Enfermedades Tropicales - Dengue
5. ANS: B PTS: 1 DIF: Media
REF: Principios de Medicina Interna de Harrison. 18° Edciion.
OBJ: Realiza correctamente el analisis e interpretacion con actitud critica de las acciones para llegar al
diagnostico y tratamiento de las principales enfermedades infecciosas tropicales de nuestro pais
TOP: Enfermedades Infecciosas - Paludismo
6. ANS: C PTS: 1 DIF: Media
REF: Principios de Medicina Interna de Harrison. 18° Edciion.
OBJ: Realiza correctamente el analisis e interpretacion con actitud critica de las acciones para llegar al
diagnostico y tratamiento de las principales enfermedades infecciosas tropicales de nuestro pais
TOP: Enfermedades Infecciosas - Dengue
7. ANS: C PTS: 1 DIF: Media
REF: Principios de Medicina Interna de Harrison. 18° Edciion.
OBJ: Realiza correctamente el analisis e interpretacion con actitud critica de las acciones para llegar al
diagnostico y tratamiento de las principales enfermedades infecciosas tropicales de nuestro pais
TOP: Enfermedades Infecciosas . Dengue
8. ANS: B PTS: 1 DIF: Media
REF: Principios de Medicina Interna de Harrison. 18° Edciion.
OBJ: Realiza correctamente el analisis e interpretacion con actitud critica de las acciones para llegar al
diagnostico y tratamiento de las principales enfermedades infecciosas tropicales de nuestro pais
TOP: Enfermedades Infecciosas . Dengue
9. ANS: D PTS: 1 DIF: Media
REF: Principios de Medicina Interna de Harrison. 18° Edciion.
OBJ: Realiza correctamente el analisis e interpretacion con actitud critica de las acciones para llegar al
diagnostico y tratamiento de las principales enfermedades infecciosas tropicales de nuestro pais
TOP: Enfermedades Infecciosas . Dengue

1
ID: A

10. ANS: C PTS: 1 DIF: Media


REF: Principios de Medicina Interna de Harrison. 18° Edciion
OBJ: Realiza correctamente el analisis e interpretacion con actitud critica de las acciones para llegar al
diagnostico y tratamiento de las principales enfermedades infecciosas tropicales de nuestro pais
TOP: Enfermedades Infecciosas . Dengue
11. ANS: D PTS: 1 DIF: Media
REF: Principios de Medicina Interna de Harrison. 18° Edciion.
OBJ: Realiza correctamente el analisis e interpretacion con actitud critica de las acciones para llegar al
diagnostico y tratamiento de las principales enfermedades infecciosas de nuestro pais
TOP: Enfermedades Infecciosas . Toxoplasmosis
12. ANS: D PTS: 1 DIF: Media
REF: Principios de Medicina Interna de Harrison. 18° Edciion
OBJ: Realiza correctamente el analisis e interpretacion con actitud critica de las acciones para llegar al
diagnostico y tratamiento de pacientes que cursen Sepsis y sus complicaciones
TOP: Enfermedades Infecciosas . Sepsis
13. ANS: D PTS: 1 DIF: Media
REF: Principios de Medicina Interna de Harrison. 18° Edciion
OBJ: Realiza correctamente el analisis e interpretacion con actitud critica de las acciones para llegar al
diagnostico y tratamiento de pacientes que cursen con Sepsis y sus complicaciones
TOP: Enfermedades Infecciosas . Sepsis
14. ANS: C PTS: 1 DIF: Media
REF: Principios de Medicina Interna de Harrison. 18° Edciion.
OBJ: Realiza correctamente el analisis e interpretacion con actitud critica de las acciones para llegar al
diagnostico y tratamiento de pacientes con sepsis y sus complicaciones
TOP: Enfermedades Infecciosas . Sepsis
15. ANS: C PTS: 1 DIF: Media
REF: Principios de Medicina Interna de Harrison. 18° Edciion.
OBJ: Realiza correctamente el analisis e interpretacion con actitud critica de las acciones para llegar al
diagnostico y tratamiento de las principales enfermedades infecciosas de Piel y Tejidos Blandos
TOP: Enfermedades Infecciosas . Infecciones de piel y tejidos blandos
16. ANS: F PTS: 1 DIF: Alta
REF: Principios de Medicina Interna de Harrison. 18° Edciion
OBJ: Realiza correctamente el analisis e interpretacion con actitud critica de las acciones para llegar al
diagnostico y tratamiento de las principales enfermedades infecciosas de piel y tejidos blandos
TOP: Enfermedades Infecciosas . Infecciones de Piel y tejidos blandos
17. ANS: C PTS: 1 DIF: Media
REF: Principios de Medicina Interna de Harrison. 18° Edciion.
OBJ: Realiza correctamente el analisis e interpretacion con actitud critica de las acciones para llegar al
diagnostico y tratamiento de las principales enfermedades infecciosas de Piel y Tejidos Blandos
TOP: Enfermedades Infecciosas . Infecciones de Piel y Tejidos Blandos
18. ANS: C PTS: 1 DIF: Alta
REF: Principios de Medicina Interna de Harrison. 18° Edciion
OBJ: Realiza correctamente el analisis e interpretacion con actitud critica de las acciones para llegar al
diagnostico y tratamiento de las principales enfermedades infecciosas gastrointestinales de nuestro pais
TOP: Enfermedades Infecciosas . Infecciones gastrointestinales
19. ANS: D PTS: 1 DIF: Media
REF: Principios de Medicina Interna de Harrison. 18° Edciion
OBJ: Realiza correctamente el analisis e interpretacion con actitud critica de las acciones para llegar al
diagnostico y tratamiento de las principales infecciones Gastrointestinales
TOP: Enfermedades Infecciosas . Infecciones gastrointestinales

2
ID: A

20. ANS: B PTS: 1 DIF: Media


REF: Principios de Medicina Interna de Harrison. 18° Edciion.
OBJ: Realiza correctamente el analisis e interpretacion con actitud critica de las acciones para llegar al
diagnostico y tratamiento de las principales enfermedades infecciosas gastrointestinales del país
TOP: Enfermedades Infecciosas . Infecciones gastrointestinales

3
REACTIVO 2 INFECTOLOGIA . DRA RIOS [Answer Strip] ID: A

B
_____ 3. C
_____ 6. C 10.
_____ D 13.
_____

D
_____ 1.

D 11.
_____

C 14.
_____

C
_____ 7.

C
_____ 2.

D
_____ 4.

D 12.
_____

B
_____ 8. C 15.
_____

D
_____ 9.
B
_____ 5.
REACTIVO 2 INFECTOLOGIA . DRA RIOS [Answer Strip] ID: A

F 16.
_____ C 17.
_____ D 19.
_____

B 20.
_____

C 18.
_____
Name: ________________________ Class: ___________________ Date: __________ ID: A

REACTIVOS DE MEDICINA INTERNA: NEFROLOGÍA

Multiple Choice
Identify the choice that best completes the statement or answers the question.

____ 1. Paciente femenino 23 años refiere disuria, polaquiuria y tenesmo vesical cual maniobra utiliza para
examinar los riñones

a. Maniobra mcburney
b. Maniobra de guyon
c. Maniobra de murphy
d. Maniobra de blumberg

____ 2. Paciente hombre de 44 años de edad, presenta hipertensión arterial difìcil de controlar, la cual no esta
asociada a medicamentos ni a factor genetico, ademas al examen fisico se escucha un soplo sobre el
riñon, al monento de la auscultaciòn. Se sospecha de una estenosis de la arteria renal. ¿que examen
imagenologico se puede realizar para comfirmar el diagnostico?

a. Rx simple de abdomen
b. Eco Dopler renal
c. Ecografia renal
d. Gammagrafìa renal

____ 3. Paciente femenino de 45 años que presenta proteinuria (1.5g/24h), hematuria con cilindros eritrociticos,
piuria, hipertension arterial, retencion de liquidos e incremento de la concentracion de creatinina serica,
el cuadro se acompaña de disminucion del filtrado glomerular. ¿cual es su diagnostico?

a. Sindrome nefrítico agudo


b. Tumor Renal
c. Sindrome nefrotico
d. Nefrolitiasis

____ 4. Preescolar de 4 años, sexo femenino.Consultó en el Servicio de Urgencia por vómitos intensos,
edema leve, oliguria, se constató hematuria microscópica, creatinina plasmática de 0,33 mg/dl y
ecografía renal que fue informada normal. Se continúa en seguimiento evolucionando
posteriormente con persistencia de la hematuria y aparición de proteinuria sobre los 3.5 gr/24h,
hipertrigliceridemia, hipercolesterolemia, hipoalbuminemia, hipercoagulabilidad, hinchazón en la
cara y alrededor de los ojos y compromiso de la función renal en forma progresiva. ¿Cuales el
diagnostico?

a. Sindrome Nefrotico
b. SindromeNefritico
c. IVU
d. Insuficiencia Renal Aguda

1
Name: ________________________ ID: A

____ 5. Mujer de 68 años de edad, diagnosticada hace 12 años de Diabetes Mellitus, obesa, hipertensa. Al
examen del laboratorio presenta (filtrado glomerular estimado de 38 mL/min/l, albuminuria de 420 mg
en orina de 24 h). Ecografícamente: Riñon disminuido de tamaño. ¿Identifique cual es el Estadio de
Nefropatía Diabetica que se encuentra la paciente?

a. Estadio I
b. Estadio II
c. Estadio III
d. Estadio IV

____ 6. Un paciente de 85 años acude al servicio de salud con debilidad muscular, temblor de brazos y piernas ,
hormigueos y parestesia, insomnio y disminucion de actividad cardiaca. La presión arterial es de 90/60
mmHg Potasio (K+) 6 mEq/L Mg de 2 mg/dL Ca 9 mg/ dl. En el ECG destacan T prominente
prolongaciòn del intervalo PR perdida de la onda P y ensanchamiento del compleho QRS acompañado de
arritmias ventriculares ¿Cual es su diagnostico?
a. Hiperpotasemia c. Hipercalcemia
b. Hipermagnesemia d. Hipopotasemia

____ 7. Un paciente de 80 años con episodios de diarreas abundantes acude al hospital por astenia y debilidad
muscular severa. La presión arterial es de 150/80 mmHg Potasio (K+) 2.5 mEq/L Mg de 2 mg/dL Ca 9
mg/ dl. En el ECG destacan descenso del segmento ST con aplanamiento de la onda T y aumento de
amplitud de la onda U, Cual es su diagnostico?
a. Hiperpotasemia c. Hipercalcemia
b. Hipermagnesemia d. Hipopotasemia

____ 8. Un paciente de 54 años, con antecedentes de enfermedad renal crónica, presenta disminucion de volumen
urinario y elevación de azoados, laboratorio urea 100, creatinina 4; gasometría: pH 7,10 HCO3 10, PCO2
30, ¿Cuál es su diagnóstico?

a. acidocis respiratoria
b. acidocis metabólica
c. alcalosis respiratoria
d. alcalosis metabolica

2
Name: ________________________ ID: A

____ 9. Paciente mujer de 35 años de edad presenta dolor lumbar y es positivio a la puño percusión, ademas,
presenta fiebre de 39°C acompañado de vomitos en varias ocasiones, nausea,escalofrio, disuria,
polaquiuria, dolor suprapúbico y tenesmo vesical, ademas al realizar examenes de laboratorio de un
EMO + gram tenemos los siguientes datos: Proteínas 100mg/dl; Cetonas: ++; Hemoglobina+++;
Células: 8-10/campo; Piocitos: campo lleno/campo; Hematíes: 10-12/campo; Bacterias: +++; Cilíndros
leucocitarios: 5-6/campo; Moco: ++; Gram de Gota fresca: Bacilos gram negativos + cocos gram
positivos +. Urocultivo: Escherichia coli: 100.0000 UFC/ml. Ecografía Renal con hipertrofia de
piramides.
Elija la respuesta correcta:

a. Pielonefritis
b. Cistitis
c. Prostatitis
d. Uretritis

____ 10.
Paciente de 50 años, trabajador de una institución pública, que acude a control de rutina. En el
examen fisico presenta tensión arterial alta: de 156/95 mmHg. 83 Kg de peso y 1,72 de altura
(IMC 28). Antecedentes personales: fumador de 25 paquetes/año y consumo ocasional de
alcohol (<40 gr etanol/día). Analíticas previas con cifras de colesterol de 186 mg/dl, LDL112
mg/dl, HDL 56 mg/dl, triglicéridos 143 mg/dl, glucemia 79 mg/dl, creatinina 0.8, ¿Cùal es el
estadío de hipertensiòn arterial?

a. Normal alta
b. HTA Grado 1
c. HTA Grado 2
d. HTA Grado 3

____ 11. Paciente masculino de 70 años de edad acude al centro médico por presentar, nauseas que llegan a
vómito, sudoración, palidez y dolor agudo que se inicia a nivel de la fosa lumbar irradiado a
testiculos. Al examen físico presenta puño percusion positivo y puntos renoureterales positivos.
En el uroanálisis presenta microhematuria, oxolatos > 50 mg/día, Gram: no bacterias, pH
urinario de 6
¿Cuál de las siguientes pruebas de imagen es el más específico para confirmar el diagnóstico de
litiasis renoureteral?

a. Tomografia Computarizada c. Ecografía Abdominal


Helicoidal sin contraste
b. Radiografía simple de abdomen d. Urografía intravenosa

3
Name: ________________________ ID: A

____ 12. Paciente masculino de 56 años de edad se acerca a consulta por presentar dolor en región
lumbar, disminusion de volumen urinario, fatiga, náuseas y vómitos que han durado ya varios
días, como antecedentes presenta hipertensión arterial desde hace varios años controlada. Al
examen físico presenta puntos ureterales bajos positivos. Se le envía a realizar exámenes de
laboratorio donde se ve alterada la creatinina con un valor de 2.5. Al examen imagenológico
se puede observar un cálculo bilateral de 8,3mm en los uréteres. ¿Qué tipo de insuficiencia
renal es?

a. Insuficiencia Renal Aguda Prerrenal


b. Insuficiencia Renal Aguda Intrínseca
c. Insuficiencia Renal Aguda Postrenal
d. Insuficiencia renal crónica

____ 13. Paciente masculino 78 años pesa 85kg, TA: 120/70, con antecedentes de HTA de varios años de evolucion,
presenta los siguientes datos de laboraorio Na: 145 K: 5, urea 200, Creatinina: 6 mg/dl un aclaramiento
de creatinina CKD-EPI de 12 ml/min ¿cual es el diagnostico del paciente?

a. Enfermedad renal crónica estadio 2


b. Enfermedad renal crónica estadio 5
c. Enfermedad renal crónica estadio 3
d. Enfermedad renal crónica estadio 1

____ 14. Paciente masculino de 48 años de edad con un peso de 70 Kg, que se le acaba de realizar un
trasplante renal hace 1 semana, presenta una temperatura corporal de 39.5 ºC, una tensión arterial
de 160/100, al examen físico presenta sensibilidad y edema sobre el riñón trasplantado. Su
creatinina es de 1.7. ¿Cuál es el diagnostico? Señale la respuesta correcta.

a. Rechazo Hiperagudo
b. Rechazo Agudo Acelerado
c. Rechazo Agudo
d. Rechazo Crónico

Multiple Response
Identify one or more choices that best complete the statement or answer the question.

____ 15. Paciente con diagnóstico de diabetes mellitus, nefropatia diabetica, hipertension arterial, enfermedad renal
cronica estadio 4, al laboratorio urea 140, creatinina 2,5, aclaramiento de creatinina ckd-epi de 18 ml/
minuto, potasio de 5,5, sodio 140, ¿cual es el acceso vascular permanente mas adecuado para planificar
hemodialisis?
a. cateter yugular c. cateter femoral
b. cateter subclavio d. fistula arteriovenosa

4
ID: A

REACTIVOS DE MEDICINA INTERNA: NEFROLOGÍA


Answer Section

MULTIPLE CHOICE

1. ANS: B
A.- Es incorrecto porque la Maniobra de mcburney es para valorar apendice

B.- Es correcta la Maniobra de guyon porque sirve para valorar riñon

C.- Es incorrecta Maniobra Murphy porque sirve para valorar colelitiasis

D.- Es incorrecta Maniobra de Blumberg porque sirve para valorar irritcion peritoneal

PTS: 1 DIF: Media


REF: GUARDERAS, C. SEMIOTECNIA INTEGRADA GENERAL Y ESPECIAL. QUITO:
Guarderas, Peñafiel, Arias, Dávalos.(1995) Pg.473 OBJ: Conocer las maniobras para examinar
el riñon
TOP: Nefrología KEY: Riñon NOT: Dr. Raúl Inca Andino
2. ANS: B
A.- Es incorrecta debido a que el rx simple de abdomen, solo evalua el tamaño del riñon, y posibles
calculos en los riñones o ureteres.
B.- Es correcta. debido a que el eco doppler renal nos permite visualizar el flujo sanguineo de las arterias
a color, permitiendonos dar un posible diagnostico de estenosis de la arteria renal o estrechamiento de la
misma.
C.- Es incorrecta, debido a que la ecografia renal solo nos permite visualizar el tamaño del riñon, posibles
litiasis, quistes corticales y diferenciacion cortico-,medular
D.- Es incorrecta, debido a que la Gammagrafia renal nos sirve para medir el funcionamiento de los
riñones.

PTS: 1 DIF: Alta


REF: Pedrosa. Diagnostico por imagen . España : S.A. MCGRAW-HILL. (2015)
OBJ: Identificar que examen de imagen TOP: Nefrología KEY: Ecografia Renal
NOT: Dr. Raúl Inca Andino
3. ANS: A
A.- Es correcto, porque el cuadro clínico es para síndrome nefrítico agudo
B.- Es incorrecto, porque el cuadro clínico es diferente en tumor renal
C.- Es incorrecto, el paciente con sindrome nefrótico presenta proteinuria mayor de 3.5g/24h,
hipoalbuminemia, edema, hiperlipidemia.
D.- Es incorrecto, el paciente no tiene antecedentes de litiasis renal

PTS: 1 DIF: Alta


REF: Harrison principios de Medicina Interna 18 edicción - Nefropatía crónica año 2012 CAPÍTULO
45 cuadro 45-1 OBJ: Conocer el cuadro clinico del Sindrome Nefritico
TOP: Nefrología KEY: sindrome NOT: Dr. Raúl Inca Andino

1
ID: A

4. ANS: A
A.- Es correcta ya que el sindrome nefrótico se caracteriza por proteinuria mayor de 3.5 gr,
hipoalbuminemia, hiperlipidemia, hipercoagulabilidad.
B.- Es incorrecta ya que el sindrome nefritico se caracterizapor la tríada de hipertensión arterial, edema y
hematuria
C.- Es incorrecta ya que las infecciones urinarias se caracterisa por disuria, polaquiuria, tenesmo vesical,
alza termica.
D.- Es incorrecta porque la insuficiencia renal aguda se caracteriza por el deterioro rápido de la función
renal en horas o días, con disminución de la tasa de filtración e incapacidad para regular líquidos y
electrolitos.

PTS: 1 DIF: Alta


REF: Farreras y Rozman.Medicina interna. Barcelona : Elsevier, 2016. Vol. I. pag 829
OBJ: Conocer el cuadro clinico del Sindrome Nefrotico TOP: Nefrología
KEY: Sindrome NOT: Dr. Raúl Inca Andino
5. ANS: D

Feedback
A.- NO ES CORRECTA: En el Estadio I: existe aumento del Filtrado Glomerular y la presión de
filtración, y los riñones aumentan de tamaño
B.- NO ES CORRECTA: En el Estadio II: existe excreción renal de albumina permanece entre los valores
normales y estructuralmente existe engrosamiento de la menbrana basal glomerular. Se desarrolla entre
2-3 años siguientes del diagnostico de Diabetes Mellitus.
C.- NO ES CORRECTA: En el Estadio III: existe eliminación constante de albumina por orina en
cantidad entre 30 mg/día y 300 mg/dia. Existe un incremento leve de la TA con disminución Filtrado
Glomerular. Estructuralmente los glomérulos presentan esclerosis focal y segmentaria, aumento del
grosor de la Membrana Basal con formación de nódulos.
D.- ES CORRECTA: En el Estadio IV: se inicia entre los 10 - 15 años de evolución de la Diabetes
Mellitus. Se caracteriza con albuminuria >300 mg/dl; descenso progresivo del filtrado glomerular y
aparición de HTA.

PTS: 1 DIF: Alta


REF: Harrison principios de Medicina Interna 18 edicción - Nefropatía crónica año 2012 CAPÍTULO
149 Páginas 960 - 961 OBJ: Identifcar los estadios de la Nefropatia Diabetica
TOP: Nefrología KEY: Diabetes NOT: Dr. Raúl Inca Andino
6. ANS: A
A.- Es correcto ya que en la hiperpotasemia el potasio esta elevado y no se encuentra en los valores
normales que son 3,5 a 5,3 mEq/L , la hiperpotasemia produce trastornos de la repolarización que se
traducen en la instalación de una T prominente, prolongaciòn del intervalo PR, simétrica, de base
estrecha, visible en las derivaciones precordiales(1).
B.- Es incorrecta ya que los valores normales del magnesio es 1,7 a 2,2 mg/dl (1)
C.- Es incorrecta porque los valores de calcio son 8,5 a 10.2 mg/dl y En el ECG destacan T prominente
prolongacion del intervalo PR perdida de la onda P y ensanchamiento del compleho QRS acompañado de
arritmias ventriculares (1)
D.- Es incorrecta ya una de las principales causas de hipopotasemia es trastornos gastrointestinales como
diarrea , el valor normal de potasio en los adultos es de 3,5 a 5,3 mEq/L y en este caso estan disminuido
y las alteraciones en el ECG son descenso del segmento ST con aplanamiento de la onda T y aumento de
la amplitud de la onda U (1)

PTS: 1 DIF: Alta


REF: Harrison. Principios de Medicina Interna, (2012) 18edicion Capítulo 15:
OBJ: Conocer las alteraciones electrocardiograficas de Hiperpotasemia
TOP: Nefrología KEY: Potasio NOT: Dr. Raúl Inca Andino

2
ID: A

7. ANS: D
A.- Es incorrecto porque la hiperpotasemia produce trastornos de la repolarización que se traducen en la
instalación de una T grande, simétrica, de base estrecha, visible en las derivaciones precordiales(1).
B.- Es incorrecta ya que los valores normalesç del magnesio es 1,7 a 2,2 mg/dl (1)
C.- Es incorrecta porque los valores de calcio son 8,5 a 10.2 mg/dl y en el electrocardiograma se
observan el segmento ST acortado o ausente y la duración del intervalo QT está disminuida(1)
D.- Es correcta ya que la hipopotasemia es producido por trastornos gastrointestinales como diarrea , el
valor normal de potasio en los adultos es de 3,5 a 5,3 mEq/L y en este caso el (K+) està disminuido y las
alteraciones en el ECG son descenso del segmento ST con aplanamiento de la onda T y aumento de la
amplitud de la onda U (1)

PTS: 1 DIF: Alta


REF: Harrison. Principios de Medicina Interna, (2012) 18 edicion | .Capítulo 15:
OBJ: Conocer las alteraciones electrocardiograficas de hipopotasemia
TOP: Nefrología KEY: Potasio NOT: Dr. Raúl Inca Andino
8. ANS: B
A.- Es incorrecta porque la acidosis respiratoria se caracteriza por pH disminuido y PCO2 aumentado
B.- Es correcta porque en la acidosis metabolica se caracteriza por pH disminuido y bicarbonato
disminuido.
C.- Es incorrecta porque la alcalosis respiratoria el pH aumentado, y PCO2 disminuido
D.- Es incorrecta porque en una alcalosis metabolica el pH aumentado, bicarbonato aumentado

PTS: 1 DIF: Alta


REF: Harrison Manual de Medicina 18 edicción-Fisiología respiratoria y estudios para el diagnóstico
(2012). CAPÍTULO 137 Pagina 901.
OBJ: Conocer los valores de la gasometría en acidosis metabólica
TOP: Nefrología KEY: Bicarbonato NOT: Dr. Raúl Inca Andino
9. ANS: A
A.- Es correcta porque la pielonefritis suele comenzar de forma súbita con fiebre elevada,
escalofríos y afección del estado general. Dolor en la fosa lumbar, a veces con náuseas y vómitos.
El dolor puede irradiar al flanco, a la fosa iliaca del mismo lado o al epigastrio. La percusión de
la fosa lumbar es positivo, al laboratorio EMO con bacteriuria, piuria, leucocituria.
B.- Es incorrecta porque la cistitis se caracteriza por la aparicion de disuria, polaquiuria y miccion
urgente (sndrome miccional irritativo o sindrome cistico)
C.- Es incorrecta porque las mujeres no tienen prostata.
D.- Es incorrecta porque para el disgnostico de uretritis es una paciente joven con antecedentes
de cambio reciente de pareja sexual o esta sufriendo uretritis.

PTS: 1 DIF: Alta


REF: HARRISON, R. Principios de medicina Interna XIX Edición . Barcelona : Mac-Graw Hill
Interamericana editores. (2016) OBJ: Identificar el cuadro clinico de pielonefritis
TOP: Nefrología KEY: Vias urinarias NOT: Dr. Raúl Inca Andino

3
ID: A

10. ANS: B
a.- Es incorrecta porque la presión arterial normal alta se considera entre Presión arterial sistólica:
120-129mmHg y Presion arterial diastólica: 85-89mmHg.
b.- Es correcta porque la HTA Grado 1 se considera entre Presión arterial sistólica: 140-159mmHg y
Presion arterial diastólica: 90-99mmHg.
c.- Es incorrecta porque la HTA Grado 2 se considera entre Presión arterial sistólica: 160-179mmHg y
Presion arterial diastólica: 100-109mmHg.
d.- Es incorrecta porque la HTA Grado 3 se considera entre Presión arterial sistólica: >180mmHg y
Presion arterial diastólica: >110mmHg

PTS: 1 DIF: Alta


REF: Harrison Principios de Medicina Interna (2016) 19a Ed. Vol. 2, pag 1616
OBJ: Conocer los estadias de la Hipertensión Arterial TOP: Nefrología
KEY: Presión Arterial NOT: Dr. Raúl Inca Andino
11. ANS: A
A.- Es correcta ya que la Tomografia Computarizasa helicoidal sin contraste se considera la
mejor prueba radiológica para el diagnóstico de litiasis urinaria, ya que detecta la mayoría de las
litiasis y según su densidad, su localización y su apariencia, puede sugerir la composición del
cálculo. Además, detecta obstrucciones de la vía urinaria y es capaz de definir otros diagnósticos
diferenciales.
B.- Es incorrecta ya que la radiografía simple de abdomen sólo es útil en casos de litiasis
radiopaca, sin que se visualicen las litiasis úricas y de xantinas.
C.- Es incorrecta ya que con la ecografía abdominal es difícil de ver las litiasis pequeñas (<5mm)
o localizadas en el uréter medio.
D.- Es incorrecta ya que la urografía intravenosa ha sido sustituida por la TC helicoidal por la
mayor seguridad que aporta y por su coste/efectividad.

PTS: 1 DIF: Alta


REF: Harrison. Medicina Interna (18 ed., Vol. II). México D.F.: McGraw-Hill. (2012) Pag 2382
OBJ: Conocer los examenes de imagen para Litiasis Renal TOP: Nefrología
KEY: Imagenologia Renal NOT: Dr. Raúl Inca Andino
12. ANS: C
a) Es inconrrecta por que para ser una Insuficiencia Renal Aguda prerrenal la alteracion deberia ser
causada por un decenso de la perfución renal
b) Es incorrecta por que para ser una Insuficiencia Renal Aguda intrinseca debe haber un daño o
alteracion en las estrucuras renales propiamente dichas.
c) Es correcta por que en la Insuficiencia Renal Aguda Postrenal puede presenta calculo bilateral en los
uréteres causando una obstrucción bilateral del flujo urinario.
d) Es incorrecta por que la insuficiencia renal crónica se da por deterioro de la funcion renal en varios
meses.

PTS: 1 DIF: Alta


REF: Harrison, Principios de Medicina Interna. España: McGraw-Hill Interamericana. (2012) (págs.
268-271) OBJ: Identificar el cuadro clinico de la Insuficiencia Renal Aguda
TOP: Nefrología KEY: Riñon NOT: Dr. Raúl Inca Andino

4
ID: A

13. ANS: B
a) Es incorrecta ya que la enfermedad renal crónica estadio 2 tiene un aclaramento de creatinina que va
desde 60 a 89 ml/min
b) Es correcta ya que la enfermedad renal crónica estadio 5 tiene un aclaramento de creatinina < 15
ml/min
c) Es incorrecta ya que la enfermedad renal crónica estadio 3 tiene un aclaramento de creatinina que va
desde 30 a 59 ml/min
d) Es incorrecta ya que la enfermedad renal crónica estadio 1 tiene un aclaramento de creatinina que va
desde 90 a 100 ml/min

PTS: 1 DIF: Alta


REF: Harrison principio de medicina inerna, edición español, nuevo leon, mexico,, McGrawn Hill. |
,(2012) pagina 895 OBJ: Identificar los estadios de la Enfermedad Renal Crónica
TOP: Nefrología KEY: Riñon, Creatinina NOT: Dr. Raúl Inca Andino
14. ANS: B
A.- Es incorrecta ya que el rechazo hiperagudo se presenta inmediatamente después de la
revascularización del injerto. Es causado por la presencia de anticuerpos preformados en el
receptor y se caracteriza por la trombosis y pérdida irreversible del injerto. (2)
B.- Es correcta porque un rechazo agudo acelerado ocurre en la primera semana,
generalmente en el segundo a cuarto día, y se manifiesta por fiebre, hipertensión arterial y
edema del riñón trasplantado. Además que en la mayoría de los casos presenta una elevación
de la creatinina. (2)
C.- Es incorrecta ya que el rechazo agudo se produce durante el primer año. Muchas veces es
subclínico, pero en el 65% de los casos hay manifestaciones clínicas como es el caso de una
insuficiencia renal aguda, proteinuria, hipertensión arterial, fiebre, dolor, oliguria, anuria, etc.
(2)
D.- Es incorrecta porque un rechazo crónico se caracterizada por esclerosis glomerular,
atrofia tubular y fibrosis intersticial. Se presenta en forma lenta después de meses o años del
trasplante llevando a la pérdida del mismo. (2)

PTS: 1 DIF: Alta


REF: 1. Rozman, Farreras y. Medicina interna. Barcelona : Elsevier, 2016. Vol. I.. Sección VI pag:
685-691
OBJ: Identificarlos tipos de rechazos de trasplante Renal TOP: Nefrología
KEY: Riñon, Trasplante NOT: Dr. Raúl Inca Andino

MULTIPLE RESPONSE

15. ANS: D
a.- Es incorrecta ya que el cateter yugular para hemodoalisis pernamece por tres semanas
b.- Es incorrecta ya que el cateter subclavio para hemodialisis permace por 15 dias
c.- Es incorrecta ya que el cateter femoral para hemodialisis permanece por 7 dias
d.- Es correcta ya que la fistula arteriovenosa para hemodoalisis es de forma permanente

PTS: 1 DIF: Alta


REF: Harrison principios de Medicina Interna 18 edicción - Nefropatía crónica año 2012 CAPÍTULO
281
OBJ: Conocer el acceso vascular para hemodialisis TOP: Nefrología
KEY: Accesos vasculares NOT: Dr. Raúl Inca Andino

5
REACTIVOS DE MEDICINA INTERNA: NEFROLOGÍA [Answer Strip] ID: A

D
_____ 5. A
_____ 9. C 12.
_____

B
_____ 1.

A
_____ 6.

B 13.
_____
B
_____ 2.

D
_____ 7.
B 10.
_____

B 14.
_____

A
_____ 3. B
_____ 8.

A 11.
_____

A
_____ 4.

D 15.
_____
UNIVERSIDAD NACIONAL DE CHIMBORAZO
CATEDRA DE MEDICINA INTERNA
ASIGNATURA; NEUROLOGIA
Dr. Guillermo Gualpa Jaramillo

Name: _______________________________________________ GRUPO: B1 FECHA: __________ ID: A

NEUROLOGIA1

True/False
Indicate whether the statement is true or false.

F
____ 1. la mortalidad con TEC grave esta alrededor del 20 %

F
____ 2. la mortalidad con TEC con escala de Glasgow de 8 T o menos se asocia al 30 %

V
____ 3. la mortalidad en paciente con TEC leve se halla alrededor del 15 %

F
____ 4. la mortalidad con TEC y Glasgow de 3 se halla alrededor del 40 %

V
____ 5. Complete

el hematoma epidural tiene la forma biconvexo

F
____ 6. segun la escala de Hoehn y Yahr, el estado III de Parkinson: habla de un compromiso bilateral, pero sin
anormalidades posturales

Multiple Choice
Identify the choice that best completes the statement or answers the question.

____ 7. cual de los siguientes signos y sintomas estan involucrados en los trastornos de la consciencia en un
paciente con traumatismo craneal.
a. atención d. conservación del ritmo circadiano
b. lenguaje coherente e. alucinaciones
c. orientado en tiempo y espacio f. sueño y vigilia conservado

____ 8. Establezca la relación que tiene el encéfalo y su función especifica

lobulo cerebral función


1.- frontal a.- conductual
2.- parietal b.- motriz
3.- occipital c.- visual
4.- temporal d- sensitivo
5.- limbico e.- analisis
6.- insula f.- integración

a. 1b-2b-3e-4a-5c-6f c. 1b-2a-3c-4d-5e-6f
b. 1b-2c-3c-4d-5e-6f d. 1b-2d-3c-4a-5e-6f

1
Name: ________________________ ID: A

____ 9. Los siguientes enunciados forman parte de la clínica del trastorno cognitivo agudo seleccione la mejor
respuesta
a. metastasis leptomeningea c. hematoma extradural
b. hematoma subdural subagudo d. hidrocefalia obstructiva tumoral del IV
ventrículo

____ 10. En la meninguitis aséptica el germen causal mas frecuente es:


a. neumococo d. estreptococo
b. riketsia e. virus
c. ningun germen

____ 11. LA MENINGITIS RECURRENTE SE CONSIDERA CUANDO


a. el paciente se encuentra en etapa de c. presenta dos o mas casos de episodios de
recuperación de una meningitis aguda meningitis aguda
b. su evolución es de más de cuatro d. se acompaña de estado comatoso o crisis
semanas epilépticas

____ 12. NORMA TECNICA MSP. CIEX: G 43: señale la falsa

a. Tratamiento profiláctico con c. Tratamiento farmacológico de la


sibelium crisis de la migraña

b. Tratamiento con Migradorixina d. Tratamiento psicológico

____ 13. paciente joven 24 años de edad sufrió accidente de moto sin casco, sin fracturas, pérdida de la conciencia
desde el accidente con TAC en las primeras 24 horas normal, y ECG de O1 V2 M3 se cataloga como:

a. TCE moderado con mortalidad del 10 % d. TCE grave con mortalidad del 80 %

b. TCE grave con mortalidad del 20 % e. TCE leve con mortalidad de 4 %

c. TCE grave con mortalidad del 50 %

____ 14. En relación a las apraxias: seleccione la falsa


a. se evalua haciendo chasquear los dedos o c. las apraxias unilaterales se deben sobre
aplaudir todo a lesiones contralaterales de la
corteza frontal premotora
b. apraxia de construcción, desorientación d. las apraxias bilaterales se observa en
derecha e izquierda y negligencia del lesiones cerebelosas
espacio exterior son trastronos de la
ubicación del espacio l

____ 15.
1.- cefalea migraña a.- cefalea en racimos
2.- se despierta a la misma hora con cefalea b.- cefalea tensional
3.- cede con analgesicos comunes c.- tendencia familiar
4.- cronica progresiva d.- hemorragia subaracnoidea
5.- el peor dolor de mi vida e.- cefalea tumoral

a. 1b,2c,3d,4e c. 1e,2a,3b,4c,5d
b. 1a,2b,3c,4d,5e d. 1c,2a,3b,4e,5d

2
Name: ________________________ ID: A

____ 16. El síndrome de schok medular comprende:


a. solo el sitio de la lesión medular c. un nivel por encima de lalesión
b. dos niveles por encima y dos niveles por d. un nivel por abajo de la lesión
debajo de la lesión

____ 17. En el sindrome de Guillean Barre, la sintomatología es.


a. segmentaria c. descendente
b. ascendente d. aberrante

____ 18. El mejor diagnostico de la efermedad de Parkinson se obtiene


a. con la nucleotomia talámica por d. por estudios de Resonancia Magnética
estereotaxia y biopsia para determinar lesion en sustancia gris
mesencefálica
b. por determinación de dopamina en el e. ninguno
LCR
c. Inmuno elisa del LCR

____ 19. los ataques izquemicos transitorios se caracterizan por: seleccione la falsa
a. dura entre 4 dias y tres semanas c. no deja secuelas neurológicas
permanentes
b. dura maximo 24 horas d. el paciente vuelve a ser completamente
normal

____ 20. caracteristicas del temblor senil son: seleccione la mejor respuesta
a. oscilaciones en cuenta monedas c. bilateralidad
b. bradicinesia d. cede con el reposo

____ 21. la escala de Hunt y Hess, nos prmite evaluar


a. grado de afectación tomográfica c. pronostico vital del paciente
b. nivel de conciencia del paciente d. localizar el sitio especifico del area
infartada

____ 22. Seleccione la verdadera en el tratamiento de la Enf. de Parkinson


a. Levodopa Carbidopa Nivel I c. Selgirina Nivel I
b. Bromocriptina Nivel I d. ninguna

____ 23. En el sindrome Silviano Total, encontramos, señale la incorrecta


a. alteración de la conciencia c. hemianopsia heterónima
b. hemiparesia contralateral d. desviacion oculo cefalica al lado de la
lesión

____ 24. en el sindome de la arteria cerebral anterior se puede evidenciar clinicamente


a. hemiparesia e hemihipoestasia de c. afasia de Wernike
predominio crural
b. hipertonía, d. preservación esfinteriana

____ 25. En la meningitis los datos de laboratorio habitualmente se comportan de la siguiente manera seleccione la
verdadera
a. Glucosa en LCR al 70 % de la Gicemia c. Graam, da positividad en 70 a 90 por
ciento de los casos en M. Bacteriana
b. Niveles de proteinas descienden hasta d. Pleocitosis de entre 100 y 500
valores de 20 mg/dl.

3
Name: ________________________ ID: A

Multiple Response
Identify one or more choices that best complete the statement or answer the question.

____ 26.
ORIGEN FILOGENETICO PARTE ENCEFALICA
1.- telencefalo a.- cerebro medio
2.- diencefalo b.- talamo, hipotalamo, pituitaria y pineal
3.- metencefalo c.- protuberancia y zona dorsal del cerebelo
4.- mesencefalo d.- hemisferios cerebrales y ganglios basales

a. 1a,2c,3b,4d c. 1c,2b,3a,4d
b. 1b,2a,3d,4c d. 1d,2b,3c,4a

Completion
Complete each statement.

27. Complete
SEMILUNA
el hematoma subdural tiene la forma de: ……………………………….

28. complete
CON TUBO ENDOTRAQUEAL
Glasgow 8T significa:.....................................................................

29. Complete
CON ASFIXIA
Glasgow 12A significa:.................................

Matching

Correlacione: en las meningitis los germenes causales son por grupo etario
a. adulto d. escolar
b. neonato e. cirugia
c. lactante
B
____ 30. Escherichia Coli y Streptococo del grupo B
A
____ 31. neumococo, meningococo, estafilococo
D
____ 32. hemophilus influenza
C
____ 33. hemophilus influenza
E
____ 34. estafilococo y flora mixta

4
Name: ________________________ ID: A

Problem

35. Medico de 26 años de edad, trabaja en un sistemas de ambulancia medicas de emergencia en la ciudad en
turnos rotatorios, son llamados para auxiliar a una paciente mujer de 67 años de edad,158 libras de peso
antecedentes de artritis reumatoide en tratamiento con metotrexate y otros fármacos que no especifica
tiempo ni dosis, fue hallada por los familiares en la ducha de la casa, en el piso, inconsciente, con
heridas en la rostro, respiración rítmica ptosis palpebral izquierda, con cefalea intensa holocraneal
colaborador con el interrogatorio y limitación de los movimientos de su miembro superior izquierdo y
miembro inferior izquierdo que podía moverlos únicamente hacia los lados.

En el examen físico se encuentra:

Escoriaciones sangrantes en hombro izquierdo, el paciente presenta al momento de llegar la ambulancia,


TA 140/90 pulso rítmico, regular, autonomía respiratoria, de 16 por minuto, anisocoria con derecha de
3mm e izquierda de 2 mm, reflejo consensual presente, desorientada auto alopsiquicamente, disartria , el
examen demuestra que en su miembro superior izquierdo al igual que su pierna izquierda, la fuerza
estaba limitada a moverlos únicamente hacia los lados, no podía levantarlos, sin embargo abría los ojos
espontáneamente, movía su hemicuerpo derecho normalmente, se encontraba algo irritable.
Concomitantemente presento, escoriaciones abrasivas en varias partes de su economía

determinar:

1.- Escala de Coma de Glasgow O4 V4 M6 : 14


2.- Nivel encefálico CORTICO- SUBCORTICAL
3.- diagnostico topográfico SNC SOBRE AM, ST, HD,FRONTAL B4

5
Name: ________________________ ID: A

36. Varón de 65 años de edad, agricultor residente en la Moya, educación secundaria incompleta, que en
estado etílico es arrollado por automotor con pérdida de conciencia por tiempo indeterminado,
politraumatismos, es transportado por bomberos a unidad médica de alta especialidad, al momento de
ingreso en urgencias, el paciente se halla inconsciente, Glasgow O2 V1 M2 respiración estertorosa,
apneustica, anisocoria con dilatación de 4 mm izquierda y 3 mm derecha, oculocefalicos de recorrido
completo, no abre los ojos, y en respuesta verbal, responde con sonidos incompresibles, hiperreflexia
generalizada con Babinski espontáneo bilateral, presencia de salida de sangre por oído izquierdo,
escoriaciones abrasivas en rostro y en tórax posterior se observa hemiparesia braquiocrural de 1/5 en
hemicuerpo derecho

Dterminar.
1.- riesgo de mortalidad porcentaje en estos casos CON GLASGOW 5/15 DEL 90%
2.- nivel encefálico MESENCEFALICO
3.- diagnostico funcional MALO
4.- tratamiento inmediato. UCI
3.- 2 datos de la imagen HEMATOMA SUBDURAL IZQUIERDO E INFARTO CEREBRAL BASAL

6
ID: A

NEUROLOGIA1
Answer Section

TRUE/FALSE

1. ANS: F PTS: 1 DIF: BAJA NOT: Gualpa,G.


2. ANS: F PTS: 1 DIF: BAJA
3. ANS: T PTS: 1 DIF: BAJA
4. ANS: F PTS: 1 DIF: BAJA
5. ANS: T PTS: 1 DIF: baja
6. ANS: F PTS: 1 DIF: baja

MULTIPLE CHOICE

7. ANS: E PTS: 1 DIF: baja REF: silabo


OBJ: resultado de aprendizaje: Conoce los principales signos y síntomas de los trastornos de la
consciencia
TOP: NEUROLOGIA NOT: Gualpa,G.
8. ANS: D PTS: 1 DIF: media NOT: Gualpa,G.
9. ANS: C PTS: 1 DIF: alta
OBJ: Aplica el conocimiento de los principales signos y síntomas de los trastornos de la consciencia que
se asocian con patologías generales y específicas del SNC TOP: G,Gualpa.
KEY: trauma craneoencefalico
10. ANS: E PTS: 1 DIF: media
OBJ: Analiza los principales componentes de la pedida de consciencia y su correlación clínica con
trastornos funcionales encefálicos infecciosos, traumáticos y tumorales. MSC: G,Gualpa.
11. ANS: C PTS: 1 DIF: alta
OBJ: Analiza los principales componentes de la pedida de consciencia y su correlación clínica con
trastornos funcionales encefálicos infecciosos, traumáticos y tumorales TOP: G,Gualpa,
12. ANS: B PTS: 1 DIF: baja
13. ANS: D PTS: 1 DIF: baja
14. ANS: D PTS: 1 DIF: alta REF: Aminoff. Michael6ta ed. pg. 7
NOT: G, Gualpa.
15. ANS: D PTS: 1 DIF: alta NOT: Gualpa,G.
16. ANS: B PTS: 1 DIF: baja
17. ANS: B PTS: 1 DIF: baja
18. ANS: E PTS: 1 DIF: baja
19. ANS: B PTS: 1 DIF: baja
20. ANS: C PTS: 1 DIF: baja
21. ANS: C PTS: 1 DIF: baja
22. ANS: D PTS: 1 DIF: baja
23. ANS: C PTS: 1 DIF: baja
24. ANS: A PTS: 1 DIF: baja
25. ANS: C PTS: 1 DIF: baja

1
ID: A

MULTIPLE RESPONSE

26. ANS: D PTS: 1 DIF: alta NOT: Gualpa,G.

COMPLETION

27. ANS: semiluna

PTS: 1 DIF: baja


28. ANS: Con tubo endotraqueal

PTS: 1 DIF: baja NOT: Gualpa,G.


29. ANS: con afasia.

PTS: 1 DIF: baja NOT: Gualpa,G.

MATCHING

30. ANS: B PTS: 1 DIF: media


31. ANS: A PTS: 1
32. ANS: D PTS: 1
33. ANS: C PTS: 1
34. ANS: E PTS: 1

PROBLEM

35. ANS:
1.- Glasgow O4 V4M6 = 14
2.- Cortico - Subcortical
3.- SNC sobre AM, ST, HD, Frontal B4

PTS: 1 DIF: Alta NOT: Gualpa,G.


36. ANS:
1.- rissgo de mortalidad con glasgow 5/15, del 90%
2.- nivel encefálico: mesencefálico
3.- malo
4.- UCI.
5.- hematoma subdural izquirdo e infarto cerebral basal.

PTS: 1 DIF: alta NOT: Gualpa, G.

2
NEUROLOGIA1 [Answer Strip] ID: A

C
_____ 9. B 16.
_____

D 26.
_____

B 17.
_____
E 10.
_____

E 18.
_____

C 11.
_____
F
_____ 1.

F
_____ 2.

B 12.
_____ B 19.
_____
T
_____ 3.

F
_____ 4.

C 20.
_____
T
_____ 5.
D 13.
_____

C 21.
_____
F
_____ 6.

D 22.
_____

D 14.
_____
E
_____ 7. C 23.
_____
B 30.
_____
A 31.
_____
D 32.
_____
A 24.
_____
D
_____ 8. C 33.
_____
D 15.
_____ E 34.
_____

C 25.
_____
Name: ________________________ Class: ___________________ Date: __________ ID: A

OFTALMOLOGÍA

Multiple Choice
Identify the choice that best completes the statement or answers the question.

____ 1.
Mujer de 30 años que acude a urgencias por pér­ dida de visión, dolor, ojo rojo y fotofobia de
01 de 5 días de evolución. En el diagnóstico diferencial debes incluir todas estas
patologías EXCEPTO una:
a. Uveítis anterior c. Esclerouveítis.
b. Úlcera corneal d. Coriorretinopatía central
serosa.

____ 2.
Hombre de 75 años que refiere disminución de agudeza visual central en su ojo derecho de
dos semanas de evolución. En el examen de fondo de ojo se aprecian drusas blandas y
desprendimiento seroso a nivel de la mácula. ¿Qué tratamiento es actualmente el más
indicado para esta enfermedad?
a. Fotocoagulación focal con láser. c. Inyecciones intravítreas de
fármacos antiangiogénicos.
b. Terapia fotodinámica. d. Vitrectomía posterior.

____ 3.
La causa más frecuente de pérdida visual irreversible en el mundo occidental en
personas de más de 50 años es:
a. Retinopatía diabética. c. Desprendimiento de retina.
b. Degeneración macular asociada a d. Cataratas.
la edad.

____ 4. ¿Cuál es el tratamiento de primera elección de la degeneración macular húmeda asociada a la


edad?
a. Triamcinolona intravítrea. c. Somatostatina intravítrea.
b. Plasmina intravítrea. d. Ranibizumab intravítreo

____ 5. Mujer de 75 años diagnosticada hace tres años de drusas blandas en el fondo del ojo. Refiere
presentar, desde hace dos semanas, metamorfopsia y pérdida visual importante en su ojo
derecho que le impide leer. Señale el diagnóstico más probable:
a. Coriorretinopatía serosa central. c. Trombosis de la vena central de la
retina.
b. Degeneración macular senil. d. Agujero macular.

1
Name: ________________________ ID: A

____ 6.
Paciente de 50 años diagnosticado de Diabetes Mellitus tipo 2 hace 10 años con mal control
de su glucemia, que acude a urgencias por disminución grave de agudeza visual en
ojo derecho de varios días de evolución. ¿Cuál es la causa más frecuente de disminución
de agudeza visual en pacientes con estas características clínicas?:
a. Aparición de Hipermetropía c. Aparición de una Queratitis
secundaria a aumento de los Estromal secundaria a los
niveles de glucemia. niveles de glucemia.
b. Desarrollo de un Edema d. Estrabismo Secundario.
Macular de reciente
instauración, secundario a su
diabetes.

____ 7.
Paciente de 57 años de edad que acude a revisión anual rutinaria al Centro de Salud,
pesa 84 kg y mide 1.75 m. Se encuentra según sus propias manifestaciones bien de
salud, hace poco ejercicio y come normalmente. Su presión arterial es 155/90. Al
explorar el fondo de ojo previa dilatación pupilar con tropicamida, observamos en el
polo posterior de ambos ojos, rodeando el área macular, una serie de puntos rojos y
blancos junto con pequeñas manchas blancas y rojas. Esto nos hace pensar que:
a. Es una retinopatía c. Es una retinopatía diabética,
hipertensiva, controlaremos la indicaremos una curva de
presión arterial y/o glucemia.
recomendamos un Holter.
b. Se trata de una degeneración d. Podría ser una Retinitis por VIH,
macular asociada a la edad, solicitaremos las pruebas
remitimos de urgencia al correspondientes.
oftalmólogo.

____ 8.
Mujer de 65 años, hipertensa y diagnosticada de diabetes no insulinodependiente, con
buen control metabólico, acude a consulta por pérdida importante de agudeza visual en
su ojo derecho de 4 días de evolución. En la exploración del fondo de ojo destaca la
presencia de venas dilatadas y tortuosas, hemorragias en llama, edema retiniano difuso
y algunas manchas algodonosas; no se observan alteraciones significativas en cabeza
de nervio óptico (papila). Con los datos indicados, señale el diagnóstico más probable:

a. Oclusión de la arteria central de c. Retinopatía diabética proliferativa.


la retina.
b. Obstrucción de la vena central d. Retinopatía hipertensiva grado IV
de la retina. de Keith­Wegener.

____ 9.
Paciente de 61 años con antecedentes de HTA y Diabetes Mellitus que consulta por diplopía
de inicio brusco. En la exploración oftalmológica presenta ptosis ojo derecho, limitación de
la aducción, supra e infraducción y movimientos pupilares conservados. El diagnóstico
más probable es:
a. Aneurisma de la arteria comunicante c. Isquemia microvascular.
posterior.
b. Parálisis postraumática. d. Migraña oftalmopléjica.

2
Name: ________________________ ID: A

____ 10.
Una lesión periférica del 111 par craneal, nervio Oculomotor (Motor Ocular Común) suele
llevar consigo alteración de sus fibras motoras viscerales (parasimpáticas) al que están
asociadas. Si estas fibras se lesionan, el paciente manifiesta alteraciones que afectan a
los reflejos oculares. De las respuestas dadas a continuación sólo una es verdadera.
¿Cuál es?:
a. Presenta pupila contraída (en miosis) c. Su pupila no está fija (a veces
y conservado el reflejo de dilatada y a veces contraída) y el
acomodación. reflejo de la acomodación es normal.
b. Su pupila está contraída (en miosis) y d. Presenta pupila dilatada (en
abolido el reflejo de acomodación. midriasis) fija y abolido el reflejo de
acomodación.

3
ID: A

OFTALMOLOGÍA
Answer Section

MULTIPLE CHOICE

1. ANS: D PTS: 1
2. ANS: C
Pregunta de dificultad baja. El caso clínico es un paciente con una degeneración macular
asociada a la edad (DMAE). La clave para contestar esta pregunta es reconocer como signo
típico de la DMAE las drusas. Al tener una clínica de corta evolución y un desprendimiento
seroso, presuponemos que se trata de una DMAE húmeda o exudativa, en la que el
tratamiento es con antiangiogénicos. Recuerda que en el mercado nacional tenemos
actualmente tres opciones: ranibizumab, bevacizumad y aflibercept.

PTS: 1
3. ANS: B
Pregunta de dificultad baja, pero que puede conllevar a error por el punto de corte
de la edad que ha escrito el autor. La DMAE (degeneración macular asociada a la
edad) es la causa más frecuente de pérdida visual irreversible en el mundo
occidental en personas mayores. Dependiendo de los libros hablan de más de 50
años, más de 60 años o más de 65 años. La otra gran causa de ceguera legal, la
retinopatía diabética, es la causa más frecuente en las personas entre los 20­65
años de edad, es decir, en personas en edad laboral.

PTS: 1
4. ANS: D
Pregunta de dificultad media­baja. Estaba claro desde los últimos años, que esta pregunta
iba a salir en el MIR. La han formulado de manera clara y directa, sin trucos de despiste ni
opciones que llevasen a error. La opción correcta es la inyección intravítrea de ranibizumab.
Recuerde que es un anticuerpo monoclonal que actúa inhibiendo el factor de crecimiento
endotelial vascular. Dicho factor es el principal responsable conocido en el crecimiento de
neovasos en la DMAE húmeda. Existe otro antiVEGF, llamado bevacizumab, también eficaz
en el tratamiento de este tipo de DMAE. En oftalmología sólo está permitido su empleo como
"uso compasivo", pero debido a su bajo coste y a la situación económica actual. cada vez son
más los hospitales que lo utilizan.

PTS: 1
5. ANS: B
Pregunta de dificultad media. La clave del enunciado son las drusas blandas,
características de la degeneración macular asociada a la edad. El resto de opciones pueden
producir metamorfopsias (visión distorsionada) y pérdida de visión central (problemas para
la lectura), pero no presentan drusas. Otro dato que nos ayuda a descartar una opción es la
edad: la coriorretinopatía serosa central es típica en pacientes jóvenes (esta enfermedad
consiste en un desprendimiento de la retina neurosensorial a nivel de la mácula).
En la trombosis de la vena central de la retina aparecen hemorragias en llama y exudados
algodonosos en el fondo de ojo, que no hay que confundir con drusas blandas. El agujero
macular de espesor completo deja la zona central de la retina sin fotorreceptores, lo que
conlleva un escotoma central en la visión. La membrana epirretiniana macular consiste en la
proliferación de células gliales entre la retina y el vítreo, que al contraerse provoca pliegues
maculares y por consiguiente metamorfopsias.

PTS: 1

1
ID: A

6. ANS: B
La pérdida de la agudeza visual en los pacientes diabéticos puede ser de dos tipos:
1. Progresiva: siendo la causa más frecuente el edema macular (que sobreviene por la
exudación debida al aumento de la permeabilidad vascular) (respuesta 2 correcta) y que
se trata con fármacos antiVEGF (en concreto, ranibizumab). Los ensayos clínicos
publicados recientemente demuestran que el ranibizumab es superior a la fotocoagulación
focal con láser argón en términos de mejoría de agudeza visual.
2. Brusca: siendo la causa más frecuente la hemorragia vítrea (ocasionada por la formación
de neovasos en la retinopatía diabética proliferativa) y que se trata mediante la vitrectomía
(siendo necesario a posteriori la panfotocoagulación con láser argón para eliminar los
neovasos). De forma global, la causa más frecuente de pérdida de la agudeza visual en
los diabéticos es el edema macular.

PTS: 1
7. ANS: C
Esta descripción de puntos rojos y blancos con pequeñas manchas blancas y rojas, es decir,
hemorragias y exudados, es bastante inespecífica y de hecho puede aparecer en cualquiera
de las opciones que nos plantean. La clave para responder esta pregunta es que nos dicen
"esto nos hace pensar en". En la práctica clínica habitual, es bastante frecuente diagnosticar
una retinopatía diabética mediante un fondo de ojo en un paciente previamente sano y que
desconocía que era diabético (respuesta 4 correcta).
La retinopatía hipertensiva suele aparecer con cifras de tensión arterial superiores a las de
este paciente y no es lo primero que "debemos pensar". La DMAE tiene una clínica y una
exploración diferentes, con síndrome macular (disminución de la agudeza visual, escotoma
central, metamorfopsias y discromatopsias) y un fondo de ojo con drusas y neovasos. Por
último, la retinopatía por VIH sí podría ajustarse a la descripción de la pregunta, pues existen
hemorragias y exudados, pero no es lo primero que "debemos pensar"

PTS: 1
8. ANS: B
Pregunta de dificultad media­alta. Este enunciado puede ser engañoso, ya que los
antecedentes personales pueden hacernos creer que la DM es la causante de la clínica. Pero
al decir que tiene buen control de sus glucemias, el autor quiere que la descartemos como
causa (opciones 3 y 5 falsas). Debe recordarse que la hemorragia vítrea (o hemovítreo), se
caracteriza en el MIR por disminución brusca e indolora de la agudeza visual en los pacientes
diabéticos, pero no se puede ver el fondo de ojo por la turbidez vítrea. La retinopatía
hipertensiva no produce disminución brusca de la agudeza visual, sino que se caracteriza por
signos de esclerosis vascular (opción 5 falsa). En la oclusión de la arteria central de la retina, la
disminución de la agudeza visual es brusca y la retina está pálida y edematosa en el área
isquémica. No habrá hemorragias, sino una mancha rojo cereza (opción 1 falsa). Por lo tanto,
la opción correcta es la oclusión de la vena central de la retina, que se caracteriza por:
disminución brusca e indolora de la agudeza visual, hemorragias en llamarada, dilatación
de venas, edema de retina y exudados algodonosos.

PTS: 1

2
ID: A

9. ANS: C
Pregunta de dificultad media. El paciente tiene como antece­ dentes HTA y DM, por lo que la
causa más probable de esta parálisis oculomotora sea una isquemia en la vasavasorum del
111 par. Sabemos que es una parálisis del 111 par craneal porque el paciente tiene afectados
varios músculos inervados por este par: elevador del párpado superior (ptosis), recto medio
(limitación de la aducción), recto superior (limitación de la supraducción) y recto inferior
(limitación de la infra­ ducción). Se puede descartar que el paciente presente una lesión
compresiva del nervio, porque la pupila está intacta (movimientos pupilares conservados).
Recuerde que las fibras parasimpáticas que llevan la inervación de la pupila se localizan en la
parte más periférica del nervio, por lo que se afectan si éste se comprime, mientras que las
ramas motoras viajan en el interior.

PTS: 1
10. ANS: D
El tercer par craneal o motor ocular común vehiculiza la inervación parasimpática del ojo,
que se encarga de la miosis de la pupila, por lo que su lesión dejará el ojo en midriasis
(respuestas 1, 2 y 5 falsas). Por otro lado, la acomodación ocular depende de tres
fenómenos: la miosis, la convergencia y la contracción del cuerpo ciliar. Teniendo en
cuenta que el tercer par craneal se encarga de la miosis, de gran parte de la inervación de
los músculos extraoculares del ojo y del funcionamiento del cuerpo ciliar, es de esperar que
su afectación altere el proceso de acomodación (respuesta 3 falsa y 4 correcta). Resulta
interesante recordar que las fibras parasimpáticas del tercer par craneal viajan por la
periferia del nervio, por lo que una compresión del mismo (p. ej., por un tumor o masa)
afectará primero estas fibras y posteriormente las centrales, por las que viaja el
componente motor del nervio. En cambio, si se afecta la parte central del nervio (p. ej.,
debido a una patología vascular) las manifestaciones iniciales serán motoras y
posteriormente parasimpáticas.

PTS: 1

3
OFTALMOLOGÍA [Answer Strip] ID: A

B
_____ 6. D 10.
_____

D
_____ 1.

C
_____ 7.
C
_____ 2.

B
_____ 3.

B
_____ 8.

D
_____ 4.

B
_____ 5.

C
_____ 9.
Name: ________________________ Class: ___________________ Date: __________ ID: A

SALUD MENTAL

Multiple Choice
Identify the choice that best completes the statement or answers the question.

____ 1. Paciente femenina de 45 años sin antecedentes psiquiátricos que es hospitalizada para estudio de
metrorragia y a los 5 días de su ingreso es informada del diagnóstico de neoplasia uterina con metástasis.
Se consulta a psiquiatría porque 24 horas después muestra tristeza y llanto frecuente, refiere ideas de
muerte y presenta insomnio. En la evaluación psiquiátrica no se recoge ningún antecedente psiquiátrico y
la exploración detecta elevada ansiedad y desesperanza en relación con las consecuencias de su
enfermedad neoplásica. El diagnóstico más probable es:

a. Distimia c. Trastorno de estrés


postraumático.

b. Reaccion normal frente al estres d. Trastorno de ansiedad generalizada

____ 2. Paciente femenina de 22 años que acude por tercera vez en la semana a la urgencia por cortes
superficiales, autoinflingidos, en ambos brazos. En la entrevista clínica destacan datos como importante
impulsividad, consumo de diferentes tóxicos de manera abusiva, inestabilidad en las relaciones y
numerosos intentos autolíticos, señale el diagnóstico más probable.
a. Fase maniaca de un trastorno afectivo c. Trastorno de personalidad esquizotipico
bipolar
b. Trastorno de personalidad limite d. Sindrome de Cotard

____ 3. Paciente masculino de 60 años, empleado de comercio, acude a urgencias acompañado de su familia; estos
cuentan que el día anterior, de forma bastante brusca, comenzó a decir cosas raras, a no responder a lo
que le preguntaban y a mostrarse confuso incluso respecto a su propio nombre. Efectivamente en la
exploración parece no entender lo que se le pregunta, no recuerda nada de lo que le ha pasado y no sabe
ni el día, ni el lugar en el que está. El diagnostico seria:

a. Psicosis breve. c. Esquizofrenia

b. Delirium d. Alhzeimer

____ 4. Mientras usted realiza una entrevista a un paciente, no entiende lo que el paciente le dice. Decide centrar
su atención en el discurso y se da cuenta que este no tiene idea directriz a pesar de que fragmentos del
mismo resultan comprensibles. Esta alteración del lenguaje-pensamiento, típica `por otro lado de la
esquizofrenia, es lo que en psicopatología se conoce como:

a. Disociación del pensamiento. c. Desorganizacion del pensamiento

b. Fuga de ideas. d. Bloqueo del pensamiento.

1
Name: ________________________ ID: A

____ 5. Un paciente de 78 años previamente sano presenta una clínica de varias horas de evolución de alteración
de conciencia y de las funciones mentales superiores con tendencia a la apatía y a la somnolencia. Tiene
trastornos de la percepción con algunas alucinaciones. A su familia lo que más le extraña es que el cuadro
sea muy fluctuante, pues se pasa de estar casi dormido a agitarse y vociferar, y a ratos parece estar lucido.
Pensaría:

a. Trastorno histérico de la c. Inicio de demencia


personalidad.

b. Sindrome confusional agudo d. Ictus en territorio de la arteria


cerebral media derecha.

____ 6. Paciente masculino de 25 años de edad que refiere que le genera mucha ansiedad tocar objetos que otras
personas hayan podido tocar antes (como picaportes de puertas) o que le rocen en transportes públicos
por miedo a contaminarse. Sabe que es absurdo, pero mantiene una actitud continuada de vigilancia,
realiza conductas de lavado de manos repetidamente y progresivamente ha ido restringiendo sus salidas
para evitar las numerosas situaciones que considera de potencial riesgo de contaminación. ¿Cuál es el
fenómeno psicopatológico subyacente básico?
a. Idea delirante de contaminación. c. .
Fobias especificas
b. Obsesiones de contaminación. d. Ideas sobrevaloradas de
contaminación.

____ 7. Paciente femenina de 18 años estudiante sin antecedentes personales somáticos ni psiquiátricos relevantes
que es llevada a urgencias hospitalarias por su familia por ingesta masiva de pastillas. El intento
autolitico no había tenido providencia de rescate, siendo la paciente encontrada por su madre casualmente
al volver a su casa antes de lo que tenía previsto. Una vez estabilizada orgánicamente la paciente refería
presentar desde de muerte, inhibición psicomotriz y enlentecimiento del pensamiento, dificultad en la
concentración y rendimiento en los estudios, aislamiento social, hipersomnia diurna y empeoramiento
matutino de los síntomas. Asimismo, la paciente refería tener la sensación de que sus vecinos la espiaban,
hablaban de ella cuando salía y se reían de ella lo que había incrementado su angustia. No sabía el motivo
porque el que la gente se fijaba en ella, pero estaba convencida de que no eran imaginaciones suyas.
Señale el diagnóstico:

a. Trastorno de ideas delirantes c. Trastorno de ansiedad.


persistentes.

b. Esquizofrenia. d. Trastorno depresivo con síntomas


psicóticos

2
Name: ________________________ ID: A

____ 8. A la exploración de un paciente psicótico, en tratamiento con medicación neuroléptica, refiere sensación
subjetiva de inquietud., se ve que es incapaz de relajarse, se va y viene por la consulta alterna entre
sentarse y levantarse, y cuando está de pie, se balancea de pierna a pierna, el cuadro descrito sería
compatible con el siguiente trastorno inducido por fármacos:

a. Parkinsonismo. c. Sindrome de discontinuacion

b. Acatisia d. Distonía Aguda.

____ 9. Paciente masculino de 40 años, casado con tres hijos menores de edad. Sin antecedentes psiquiátricos.En el
banco en que trabajaba, es finalmente despedido y lleva cinco meses de paro. No encuentra otra actividad
laboral. Acude a consulta con el siguiente cuadro clínico: desanimo general, inapetencia, nerviosismo,
insomnio, preocupaciones recurrentes sobre su futuro y evitación de actividades sociofamiliares. ¿Qué
opción diagnostica de las siguientes es la más adecuada?

a. Trastorno adaptativo con síntomas c. Trastorno obsesivo.


ansioso-depresivos
b. Distimia. d. Fobia social

____ 10. Un adolescente de 16 años acude a la consulta contando que lleva de cerca de un año sintiéndose más
cansada, con poco apetito y dificultades para concentrarse en los estudios, cuando se le pregunta comenta
también que sale menos con las amigas y se muestra pesimista respecto de su futuro. El diagnóstico más
probable seria:

a. c. Distimia
Anorexia

b. Agarofobia d. Trastorno de ansiedad

____ 11. Paciente BMD de 42 años sexo femenino que acude a consulta con una documentación
medica copiosa donde guarda el resultado de varios análisis y estudios
electrocardiográficos. Pese a que diferentes médicos le han informado que su corazón
está sano, él tiene la preocupación constante de que no sea así y que cuando le dicen
es para tranquilizarlo. En la entrevista describe, con lujo de detalles y vocabulario
médico, diferentes manifestaciones que ha apreciado en zonas del cuerpo donde
radican órganos vitales. Se muestra angustiado.

a. Trastorno hipocondriaco c. Ansiedad generalizada


b. trastorno ansioso depresivo d. Trastorno hipocondriaco

3
Name: ________________________ ID: A

____ 12.  Hombre de 28 años que es traído a urgencias un sábado en la madrugada , en


estado de agitación psicomotriz y con ideas delirantes. Su acompañante refiere
abuso previo de cocaína por parte de el . Cuál síntoma no pertenece a cuadro
clínico

a. Hipotermia c. Midriasis
b. Taquicardia d. Diaforesis

____ 13. Un paciente varón de unos 40 años es traído a urgencias por haber sido hallado caído
en la vía pública en una zona de ocio nocturno. El paciente está poco reactivo,
presenta una frecuencia respiratoria de 7 por minuto, una presión arterial de 90/60
y frecuencia cardiaca de 58 por minuto. Pupilas mióticas. Después de asegurar la
permeabilidad de la vía aérea, ¿Cuál sería el paso inmediato más apropiado a
seguir?
a. Administrar 0,4 mg de naloxona c. Solicitar un examen toxicológico
im o iv de orina

b. Obtener en hemograma y una d. Solicitar una TAC craneal


bioquímica básica

____ 14.  Una mujer viuda de 75 años que vive sola acude a consulta. Padece de hipertensión
arterial. Preguntaba sobre la ingesta de alcohol, dice tomar unas 4 copas al día
.La Tensión arterial está más elevada que hace 1 año. Afirma que desea disminuir
la ingesta de alcohol. Tres meses más tarde dice haber mejorado algo pero
anhela el alcohol. Solicita alguna medicación que la ayude. ¿Cuál de los
siguientes es el tratamiento más apropiado?
a. Buspirona c. Litio
b. Disulfiram d. Naltrexona

____ 15. Paciente de 36 años refiere presentar desde hace 10 meses, sintomatologia ansiosa y humor depresivo. Este
cuadro interfiere moderadamente en su actividad cotidiana, No teien antecedentes psiquiatricos previos.
Dicha situacion se produce a raiz del fallecimiento en un accidente automobilistico de su hermano mayor
, con quien se encontraba muy unido.
a. Depresion mayor c. Distimia
b. Trastorno de adaptacion d. Ciclotimia

4
ID: A

SALUD MENTAL
Answer Section

MULTIPLE CHOICE

1. ANS: B
b) Se trata de una reacción normal frente al estrés, en concreto frente a un agravio plausible. Lo normal es
aceptar esta noticia con tristeza, propia de la condición humana, y no con frialdad. Si la sintomatología
depresiva se prolonga o aparecieran síntomas somáticos tendríamos que intervenir.

PTS: 3 DIF: ALTA REF: Marban libros texbook AMIR. 2016


OBJ: Conocer y dianosticar los trastornos neuroticos TOP: Clasificacion Sindromologica
KEY: Estres, ansiedad NOT: Dra. Zilma Diago Alfes
2. ANS: B
b)El trastorno límite pertenece a los trastornos de personalidad, conductas arraigadas e inflexibles que
son desviaciones extremas de la personalidad de los sujetos sanos. Se caracteriza por inestabilidad en
todos los ámbitos (principalmente afectivo) , con sentimientos de vacío, gestos impulsivos y autolíticos,
cambios rápidos entre el amor y el odio…Es frecuente la comorbilidad con el abuso de sustancias

PTS: 3 DIF: ALTA REF: MARBAN libros Texbook AMIR 5. 2016


OBJ: Conocer los factores predictores del suicidio TOP: Autolesion y suicidio
KEY: Personalidad,Suicidio, adicciones. NOT: Dra Zilma Diago Alfes
3. ANS: B
b)Lo más característico del síndrome Confusional agudo es la fluctuación del nivel de conciencia con un
comienzo brusco y fundamentalmente lo que debemos hacer es tratar la causa orgánica que lo ha
producido.

PTS: 3 DIF: ALTA REF: MARBAN libros Texbook AMIR 5.2016


OBJ: Comprender la etiologia de los trastornos psicoticos organicos y funcionales
TOP: Psicosis organicas o somaticas y funcionales o psicogenas
KEY: Conciencia,psicosis, esquizofrenia NOT: Dra Zilma Diago Alfes
4. ANS: C
c) La clave de esta pregunta está en que es típico de la esquizofrenia y en que no existe un hilo conductor.
Es lo característico del pensamiento disgregado.

PTS: 3 DIF: ALTA REF: MARBAN libros texbook AMIR 5. 2016


OBJ: Conocer el conecpto de psicosis TOP: Psicosis organicas o somaticas y funcionales o
psicogenas
KEY: Pensamiento,esquizofrenia NOT: Dra Zilma Diago Alfes

1
ID: A

5. ANS: B
b) Importante diferenciar entre síndrome Confusional agudo(delirium), curso fluctuante y agudo, y nivel
de conciencia disminuido. Estas características las diferencian del inicio de una demencia (inicio
insidioso, curso progresivo y nivel de conciencia conservado). La inversión del ciclo sueño- vigilia
orientan también hacia el delirium. Recordar que ante un delirium no se emplean benzodiacepinas (la
única excepción seria el delirium tremens por abstinencia alcohólica

PTS: 3 DIF: ALTA REF: MARBAN libros texbook AMIR 5.2016


OBJ: Conocer la etiologia de los trastornos psicoticos organicos o somaticos y funcionales o psicogenos
TOP: Psicosis organicas o somaticas y funcionales o psicogenas
KEY: Delirium, psicosis, alucinaciones NOT: Dra. Zilma Diago Alfes
6. ANS: B
b)En este trastorno aparecen ideas obsesivas, por ejemplo, de contaminación (como en este caso), que son
intrusivas, reiterativas y que, a pesar de entender lo absurdo de las mismas, el enfermo es incapaz de
evitarlas; estas ideas se continúan de compulsiones, actos motores voluntarios que neutralizan la angustia
generada por la idea obsesiva, en el caso de las ideas de contaminación, las compulsiones de limpieza.
Al entender lo absurdo de las ideas, no llegan a conformar una estructura delirante o una idea
sobrevalorada.

PTS: 3 DIF: ALTA REF: MARBAN libros texbook AMIR 5. 2016


OBJ: Diagnostica los trastornos neuroticos mediante una adecuada historia clinica , basada en una
apropiada relacion medico paciente TOP: Clasificacion sindromologica
KEY: Trastorno Obsesivo, ideas, compulsiones NOT: Dra. Zilma Diago Alfes
7. ANS: D
d)Parece claro que la primera parte de la pregunta habla acerca de una chica que cumple criterios de
trastorno depresivo mayor. Nos orienta a ello la, merma en el funcionamiento global, la intensidad de los
síntomas y sobre todo la ideación suicida. El resto del cuadro clínico narra sintomatología delirante; si
bien lo habitual en una depresión es que el delirio sea congruente con el estado de ánimo (culpa, ruina,
enfermedad…), en este caso es una ideación delirante de perjuicio, que también puede darse en algunos
casos.

PTS: 3 DIF: ALTA


REF: MARBAN libros texbook. Academia de estudios MIR,S:L(AMIR) 2016
OBJ: Conocer el convcepto , criterios y diagnostico de la depresion moderada grave
TOP: Depresion Moderada Grave KEY: Depresion, psicosis, angustia
NOT: Dra Zilma Diago alfes
8. ANS: B
b)Se presenta un caso clínico sobre efectos secundarios de los antipsicóticos.
La respuesta correcta es la Acatisia, que se define como la incapacidad de mantenerse quieto, con
movimientos de piernas característicos(balanceo) que se alivian con la deambulación y empeoran con el
reposo. Se asocia además inquietud psíquica y ansiedad.

PTS: 3 DIF: ALTA REF: MARBAN Libros texbook AMIR 5.2016


OBJ: Conocer el tratamiento farmacologico y no farmacologico de las psicosis
TOP: Trastornos psicoticos organicos o somaticos y funcionales o psicogenos
KEY: Antipsicoticos, esquizofrenia NOT: Dra Zilma Diago Alfes

2
ID: A

9. ANS: A
a)Trastorno adaptativo implica un ánimo bajo secundario a algo que no llega a cumplir criterios clínicos
de trastorno afectivo mayor ni temporales de Distimia.

PTS: 3 DIF: ALTA REF: MARBAN Libros texbook AMIR 5.2016


OBJ: Clasificacion sindromologica TOP: Clsificacion sindromologica
KEY: Trastorno adaptativo,ansiedad, NOT: Dra Zilma Diago alfes
10. ANS: C
c)En este es fácil descartar las opciones 1, 4 y 5. La duda quedaría entre la depresión mayor y la Distimia.
Si bien para adultos hace falta que el ánimo depresivo se mantenga durante 2 años, en niños y
adolescentes es suficiente 1 año. Además, respecto a la intensidad de los síntomas leves, que encajan
mejor con el diagnóstico de distimia.

PTS: 3 DIF: alta REF: MARBAN libros texbook AMIR 5. 2016


OBJ: Depresion Moderada grave TOP: Depresion moderada-grave
KEY: Distimia, ansiedad NOT: Dra Zilma Diago Alfes
11. ANS: A
a) En este trastorno las personas hipocondríacas están tan preocupadas por su salud que continuamente
se observan e interpretan cualquier señal o cambio en el cuerpo que les confirma una enfermedad grave.
Están constantemente preocupadas y convencidas de que están enfermas.

PTS: 3 DIF: ALTA REF: MARBAN libros texbook AMIR. 2016


OBJ: Diagnostica los trastornos neuroticos mediante de una adecuada historia clinica basada en una
apropiada historia clinica TOP: Clasificacion sindromologica
KEY: hipocondria , ansiedad , enfermedad NOT: Dra Zilma Diago Alfes
12. ANS: A
a) En una intoxicacion por cocaina, es frecuente encontrar taquicardia, HTA, sudoracion, hipertermia y
sintomas psicoticos, dado que es una sustancia dopaminergica. El tratamiento debe ser sintomatico

PTS: 3 DIF: ALTA REF: MARBAN Texbook AMIR . 2016


OBJ: Conocer eel concepto de uso , abuso y dependencia de drogas
TOP: Uso de drogas y trastornos po0r su uso KEY: drogas , abstinencia, cocaina
NOT: Dra Zilma Diago Alfes
13. ANS: A
a)Nos encontramos ante una intoxicacion por opiaceos( probablemente heroina) debido a las pupilas
mioticas , la depresion respiartoria, la alteracion del estado de conciencia y la hipotension arterial. el
antidoto para ello es la naloxona que debe ser pñrioritaria tras asegurar la via aerea a cualquier
procedimiento diagnostico

PTS: 3 DIF: ALTA REF: MARBAN Texbook AMIR 2016


OBJ: Conocer la importancia del manejo y tratamiento de los diferentes cuadros de abstinencia a drogas
y de sus complicaciones. TOP: Uso de drogas y tarstornos por uso.
KEY: opiaceos, heroina, droga NOT: Dra Zilma Diago Alfes
14. ANS: D
d) En esta pregunta la naltrexona es es un medicamento anticraving, busca disminuir el anhelo de
consumo y puede ser usado en pacientes con HTA

PTS: 3 DIF: ALTA REF: MARABAN texbook AMIR .2016


NAT: Conocer el concepto de uso, abuso y dependencia a drogas.
TOP: Uso de drogas y trastornos por su uso KEY: craving, alcohol, dependencia
NOT: Dra Zilma Diago alfes

3
ID: A

15. ANS: B
b) El trastorno adaptativo se define como la reaacion emocional negativa(sintomas depresivos y ansiosos
habitualmente) a un suceso biografico pero que no cumple los criterios de otro trastorno mental. En este
caso hay una relacion causa efecto muerte de un pariente de referencia.

PTS: 3 DIF: ALTA REF: MARABAN texbook AMIR 2016


OBJ: Diagnostica los trastornos neuroticos, mediante una adecuada historia clínica basada en una
apropiada relación TOP: Trastornos neuroticos KEY: adaptacion ,ansiedad, depresion
NOT: Dra Zilma Diago Alfes

4
SALUD MENTAL [Answer Strip] ID: A

B
_____ 5. B
_____ 8. A 12.
_____

B
_____ 1.

A 13.
_____

A
_____ 9.

B
_____ 6.

B
_____ 2.

D 14.
_____

C 10.
_____

B
_____ 3. D
_____ 7.

B 15.
_____

A 11.
_____

C
_____ 4.
Name: ________________________ Class: ___________________ Date: __________ ID: A

TRAUMATO

Multiple Choice
Identify the choice that best completes the statement or answers the question.

____ 1. ¿Cuál de las siguientes lesiones traumáticas precisa, para evitar complicaciones locales. un
tratamiento más precoz?
a. Luxación traumática posterior de la c. Fractura trocantérea del anciano
cadera.
b. Fractura desplazada del cuello d. Fractura de cotilo
femoral del anciano.

____ 2. La complicación más frecuente de las fracturas del cuello del astrágalo es:
a. Algodistrofia refleja c. Pseudoartrosis.
b. Consolidación viciosa. d. Osteonecrosis.

____ 3. Joven que acude a urgencias por quemadura por llama de segundo grado del 10% de la
superficiecorporal, afectando al brazo derecho de forma extensa y circular. No se halla pulso
arterial en la mano medido por doppler. ¿Cuál es el tratamiento de elección?
a. Curas con sulfadiacina argéntica c. Escarotomía o incisiones de
oclusivas y evaluación de la descompresión de urgencia.
profundidad a la semana.
b. Drenajes linfáticos y valorar un d. Conducta expectante
by-pass vascular.

____ 4. Joven de 14 años de edad, 158 cm de altura y 76 kg de peso, que acude a nuestra consulta por
presentar desde hace 4 meses, sin antecente traumático previo, dolor mecánico en región
inguinal derecha, presentando en la exploración clínica actual, cojera con marcha con ligera
actitud en rotación externa de dicha extremidad inferior y limitación de la flexión de cadera en
rotación neutra. El diagnóstico de sospecha será:
a. Fractura del cuello femoral. c. Enfermedad de Perthes.
b. Epifisiólisis femoral proximal. d. Enfermedad de Still.

____ 5. Paciente de 28 años, que tras sufrir un accidente de tráfico frontal de gran energía, presenta
deformidad del miembro inferior derecho en aducción y rotación interna de la cadera. Llama la
atención que cualquier intento de manipulación le produce intenso dolor. Debemos pensar que
el paciente tiene:
a. Fractura subtrocantérea de fémur. c. Luxación coxofemoral anterior.
b. Fractura subcapital de fémur. d. Luxación coxofemoral posterior.

____ 6. Mujer de 53 años que presenta una fractura cerrada del radio distal tipo "Colles" y ha sido tratada
mediante reducción y colocación de yeso antebraquial. Como antecedentes relevantes no es
fumadora y no tiene antecedentes de osteoporosis. Se obtuvo una reducción correcta y se ha
optado por un tratamiento conservador. ¿Qué tiempo de inmovilización es el recomendado para
una fractura de estas características?:
a. Un tiempo máximo de 2 semanas. c. Un tiempo máximo de 1O semanas.
b. Un tiempo máximo de 6 semanas. d. Un tiempo máximo de 14 semanas.

____ 7. Un niño de 7 años sufre una caída sobre el brazo izquierdo produciéndose una fractura en tercio
medio de clavícula desplazada. ¿Cuál sería el tratamiento a seguir?:
a. VendaJe de yeso. c. Férula de abducción.
b. No precisa tratamiento. d. Vendaje en 8.

1
Name: ________________________ ID: A

____ 8. Paciente de 32 años que sufre caída casual sobre la palma de la mano con la muñeca en
híperextensión. En la exploración clínica destaca dolor selectivo a nivel de la tabaquera
anatómica. la sospecha clínica es de fractura de escafoides carpiano, pero las radiografías
practicadas son normales. ¿Cuál debe ser nuestra actitud?:
a. Realizar una tomografía axial c. Realizar tratamiento médico
computarizada.
b. Inmovilizar la muñeca con un yeso de d. Pedir una gammagrafía ósea
escafoides y repetir la radiografía a
las dos semanas.

____ 9. ¿Cuál de estas afirmaciones es cierta sobre la frac- tura de la extremidad distal del radio?:
a. La mayoría se producen como c. La rotura del extensor largo del
consecuencia pulgar no es una complicación de
estas fracturas.
b. La mayoría responden d. El tratamiento quirúrgico está
favorablemente al tratamiento contraindicado.
conservador.

____ 10. Un mecánico de 40 años se produce una herida al escapársele un destornillador con el que hacía
fuerza y clavársele en la palma de la mano izquierda. En la exploración se aprecia déficit de
flexión de las art culaciones interfalángica proximal y distal del tercer dedo y sangrado continuo.
Se debe sospechar:
a. Lesión de ambos tendones c. Lesión de ambos tendones
flexores y arcada arterial. flexores y arcada venosa.
b. Lesión del tendón flexor d. Lesión del tendón flexor
profundo y arcada venosa. superficial y arcada arterial.

2
ID: A

TRAUMATO
Answer Section

MULTIPLE CHOICE

1. ANS: A
En esta pregunta nos plantean una serie de lesiones traumática y nos preguntan acerca de
cuál de ellas requiere un tratamiento más precoz para evitar complicaciones. De las 5 opciones
tenemos 4 fracturas y una luxación. La luxación coxofemoral posterior requiere reducción
urgente para evitar complicaciones, la más frecuente es la lesión del nervio ciáti- co. El resto de
las opciones requieren tratamiento quirúrgico, pero desde luego no tan urgente como la
luxación posterior de cadera.

PTS: 1
2. ANS: D
La complicación más frecuente de las fracturas del cuello del astrágalo es la osteonecrosis del
cuerpo del astrágalo. En fracturas no desplazadas el riesgo es más bajo pero en fracturas
desplazadas puede llegar al100%. Recordad que el cuerpo del astrágalo tiene una
vascularización precaria a través del cuello, por lo que fracturas que afectan al cuello pueden
producir necrosis avascular. La algodistrofia refleja es poco frecuente. La consolidación viciosa,
o consolidación en mala posición oscila entre un 25-30% en caso de fracturas desplazadas. La
pseudoartrosis es rara en este tipo de fracturas. Dentro de las alteraciones de la consolidación
sería más frecuente el retardo de consolidación, hasta un 15%. La lesión neurológica del tibial
posterior no es una complicación frecuente de este tipo de fracturas.

PTS: 1
3. ANS: C PTS: 1
4. ANS: B
Pregunta sencilla. En el enunciado nos dan los datos t‫ي‬picos de esta patolog‫ي‬a: varَn
adolescente obeso con cojera, rotaciَn externa de la cadera y limitaciَn de la flexiَn. El diagnَstico
de sospecha es la epifisiَlisis femoral proximal. Esta patolog‫ي‬a hab‫ي‬a sido preguntada en el
MIR 2012.

PTS: 1
5. ANS: D
En la pregunta describen la posición de la extremidad de un paciente que ha sufrido un
accidente de tráfico de alta energía. La posición del miembro es en aducción y rotación interna
de la cadera. Lo que describen es la clásica posición de bañista sorprendido de la luxación
coxofemoral posterior. Recuerden que en esta lesión pueden existir fracturas de acetábulo y
fémur asociadas y lesión del nervio ciático.

PTS: 1
6. ANS: B
Pregunta en la que describen el caso de una paciente con una fractura de radio distal tipo
"Calles" = extraarticular metafisaria, en la que se ha obtenido una reducción estable con un
yeso antebraquial. El tratamiento de este tipo de fracturas, con criterios de estabilidad, es la
inmovilización con yeso durante 6 semanas. Inicialmente debemos realizar controles
semanales para detectar desplazamientos precoces de la fractura que puedan ser
subsidiarios de nuevas reducciones o de tratamiento quirúrgico.

PTS: 1

1
ID: A

7. ANS: D
En el niño, hasta los 12 años de edad, podemos aceptar cualquier forma de contacto entre los
fragmentos puesto que el foco remodelará satisfactoriamente en meses. El tratamiento
consistirá en un vendaje en ocho durante 3 o 4 semanas (opción 5 correcta). Más allá de esa
edad se debe ser más cuidadoso con la reducción de la fractura y el control posterior de su
contención colocando un vendaje en ocho durante 4 a 6 semanas.

PTS: 1
8. ANS: B
Fractura del escafoides: Debemos sospechar una fractura del escafoides tras una caída en
hiperextensión de la muñeca y dolor global de la muñeca que se acentúa al presionar nivel de
la tabaquera anatómica. Para el diagnóstico se realizan radiografías en distintas proyecciones.
A veces no se ve el trazo de fractura en un principio pero, debido al riesgo de necrosis, con
clínica sugestiva, se inmoviliza y se repiten las radiografías en 2 semanas (opción 3 correcta).

PTS: 1
9. ANS: B
Cuando en el MIR nos hablen de la extremidad distal del radio, debemos pensar en una
fractura de Colles, preguntada hasta la saciedad. Esta fractura es típica de mujeres
postmenopáusicas , y ocurre al caer sobre la mano en extensión (= dorsiflexión; si nos
dicenflexión significa flexión palmar -respuesta 1 incorrecta-, típica del Calles invertido o
Goyrand-Smith).
Sus complicaciones más típicas son la lesión del extensor largo del pulgar (respuesta 3
incorrecta), y la del nervio mediano Su tratamiento es casi siempre conservador (opción 2
correcta), pero si hay mucha desviación de los fragmentos de frac- tura puede ser quirúrgico
(opción 4 incorrecta).

PTS: 1
10. ANS: C
Para el MIR, hay que saber tres cosas sobre los cortes en la mano:
1. El sangrado venoso es continuo, mientras que el arterial es pulsátil (respuestas 1 y 4
incorrectas).
2. Qué movimientos se afectan cuando se lesionan los tendones extensores y flexores:
• Cuando se lesionan los extensores, se afecta la extensión del dedo en general (si la lesión
ocurre a nivel de la interfalángica distal (IFD), aparece dedo en martillo, y si ocurre a nivel
de la interfalángica proximal (IFP) aparece dedo en ojal).
• Cuando se lesionan los flexores, hay que distinguir entre la lesión del flexor superficial y la
del flexor profundo: la lesión del flexor superficial impide la flexión de la IFP del dedo en
cuestión, y la lesión del flexor profundo impide la flexión de la IFD (¡Ojo! Es al revés de lo
que parece!).
Así, la respuesta correcta es la 3 porque el paciente no flexiona ni la IFP ni la IFD.

PTS: 1

2
TRAUMATO [Answer Strip] ID: A

B
_____ 8.

A
_____ 1.

B
_____ 9.

D
_____ 2.

C
_____ 3.
C 10.
_____

B
_____ 4.

D
_____ 5.

B
_____ 6.

D
_____ 7.
Name: ________________________ Class: ___________________ Date: __________ ID: A

UROLOGÍA

Multiple Choice
Identify the choice that best completes the statement or answers the question.

____ 1. Es aparato urogenital es un derivado del:


a. Ectodermo ventral c. Mesodermo intermedio.
b. Esclerotomo. d. Celoma intraembrionario.

____ 2. Respecto a los somites es cierto que:


a. Son estructuras ectodérmicas. c. Dan lugar al mesodermo cardiaco.
b. Dan lugar a los ganglios raquídeos. d. Constituyen el primordio del
metanefros.

____ 3. Ante un paciente con insuficiencia renal crónica y sospecha de uropatía obstructiva en una
explora­ ción ecográfica. ¿Qué exploración es la idónea para evaluar el nivel de obstrucción y
la causa?:
a. Renograma isotópico. c. Tomografía axial computarizada.
b. Cistoscopia. d. Urografía mediante resonancia
magnética.

____ 4. La primera linea de tratamiento de la vejiga hipe­ ractiva es:


a. Neuromodulación raíces sacras. c. Fármacos antimuscarínicos.
b. Inyección endoscópica de toxina d. Electroestimu\aciones perineales a
botulínica en vejiga urinaria. baja frecuencia.

____ 5. Paciente varón de 46 años, intervenido hace 45 días de dos hernias discales lumbares. Presenta
micciones frecuentes de escasa cantidad, incontinencia urinaria y tumoración hipogástrica.
Se realiza una ecografía, que demuestra que corresponde a un globo vesical, no observando
otra patología de vejiga, además de comprobar que los riñones son normales y que la próstata
es de un volumen de 24 ce en su medición ecográfica. Se pasa por uretra sin dificultad hasta
vejiga una sonda Foley 16F, saliendo unos 950 ce de orina. Se realiza un estu­ dio
urodinámico, apreciando en la cistometría una hipoactividad e hipocontractilidad del detrusor,
con presiones del detrusor bajas durante la fase de vaciado (o miccional) y con residuo
postmiccional elevado. De las posibilidades en su enfoque tera­ péutico posterior, ¿qué NO
estaría indicado?:
a. Cateterismo intermitente c. Electroestimulación vesical.
b. Maniobras de compresión d. Anticolinérgicos.
suprapúbica (Credé)

____ 6. Una maestra jubilada de 74 años con diabetes mellitus presenta frecuentes episodios de
retención urinaria. No presenta incontinencia al toser o cambiar de postura. En la exploración
física se detecta una neuropatía sensitiva en extremidades inferiores. El residuo postmiccional
es de 400 ml. La maniobra de Valsalva realizada en bipedestación no provoca pérdida de orina.
El estudio urodinámico muestra un detrusor acontráctil sin obstrucción al vaciamiento vesical.
Los estudios de laboratorio son normales y el tratamiento farmacológico ha sido inefectivo.
¿Cuál es la opción terapéutica más apropiada?:
a. Sondajes intermitentes. c. Dilatación uretral.
b. Sonda urinaria permanente. d. Uretrólisis.

1
Name: ________________________ ID: A

____ 7. ¿Cuál cree usted que es el trastorno endocrino que provoca mayor prevalencia de disfunción
eréctil?:
a. Diabetes Mellitus. c. Hipogonadismo hipogonadotropo.
b. Síndrome de deficiencia androgénica d. Síndrome de Cushing.
asociada a la edad.

____ 8. Un varón de 68 años de edad, con antecedentes de hipertensión arterial e infarto agudo de
miocardio, en tratamiento con mononitrato de isosorbide; consulta por cuadro de disfunción
eréctil de dos años de evolución. ¿Cuál de las siguientes actitudes terapéuticas estaría
CONTRAINDICADA?:
a. Inyección intracavernosa de PGE 1. c. Citrato de Sildenafilo vía oral.
b. Colocación intrauretral de PGE 1. d. Utilización de dispositivos de vacío.

____ 9. Paciente de 63 años en tratamiento a demanda con Citrato de Sildenafilo por presentar
disfunción eréctil de años de evolución. Señale cuál de los siguientes fármacos NO asociaría
en ningún caso a su tratamiento:
a. Amiodarona. c. lndapamida.
b. Verapamilo. d. Mononitrato de lsosorbide.

____ 10. Varón de 78 años sexualmente activo que acude a nuestra consulta por impotencia de reciente
comienzo. Sus erecciones han sido progresivamen­ te menos intensas hasta impedir la
penetración. Tiene antecedentes de cardiopatía isquémica e hipertensión y ha estado tomando
aspirina y ate­ nolol desde hace años. No es diabético. ¿Cuál de las siguientes es la causa más
probable de la disfunción eréctil de este paciente?:
a. Atenolol. c. Neuropatía.
b. Descenso de los niveles de d. Enfermedad vascular.
testosterona.

2
ID: A

UROLOGÍA
Answer Section

MULTIPLE CHOICE

1. ANS: A
Pregunta con respuesta directa. El aparato genitourinario deriva del mesodermo intermedio,
fundamentalmente del metanefros y del seno urogenital. Sin embargo, la vejiga y la uretra
proceden del seno urogenital (derivado del alantoides). que es de origen endotelial, motivo
por el que el Ministerio decidió anular la pregunta de manera directa en la plantilla provisional
(antes siquiera de las impugnaciones).

PTS: 1
2. ANS: A
Pregunta controvertida que finalmente fue anulada. El Ministerio dio por correcta la respuesta
4. Si bien, según algunos libros de urología, parece cierto que los somitas podrían constituir el
primordio del metanefros, la función fundamental de los somitas y la que aparece en los libros
de embriología es que dan lugar a los músculos del tronco.

PTS: 1
3. ANS: D
El paciente presenta una insuficiencia renal crónica, lo que nos hace descartar los medios de
contraste por su nefrotoxicidad; así se excluyen claramente las opciones 5 y 3 puesto que la
TC sin contraste sólo sirve para diagnóstico de certeza de la litiasis, necesitando el contraste
para la visualización de otras causas de UPO como los tumores.
La cistoscopia sólo sirve para evaluar la vejiga, no el tracto urinario superior.
El renograma isotópico es una prueba funcional, no de localización.
La RM permite la visualización del árbol urinario sin necesidad de contraste, con una
sensibilidad y especificidad mayor a la de la TC con contraste.

PTS: 1
4. ANS: C
El trastorno denominado "vejiga hiperactiva" consiste en contracciones involuntarias no
inhibidas del músculo detrusor de la vejiga, el cual está inervado por el sistema nervioso
parasimpático, cuyo mediador es la acetilcolina que actúa sobre receptores muscarínicos. Por
esta razón, la primera línea de tratamiento son los fármacos antimuscarínicos como
oxibutinina, la propantelina, la tolterodina o la solifenacina (opción 3 correcta). Este concepto ya
fue preguntado en el MIR 2001.

PTS: 1

1
ID: A

5. ANS: D
La vejiga neurógena es un concepto preguntado en varias convocatorias.
Por el cuadro clínico, el antecedente de cirugía en el raquis, y sobre todo la
hipercontractilidad del detrusor en el estudio urodinámico, nuestro paciente tiene una vejiga
neurógena hipotónica, cuyo tratamiento es el de la retención urinaria (cateterismo
intermitente, concepto preguntado en 2008). Usando conceptos que estudiamos en otras
enfermedades, sabemos que los alfabloqueantes (simpsaticolíticos) se usan en la HBP
porque relajan el esfínter vesical y favorecen la micción, por lo que si se los damos a nuestro
paciente le irán bien. En cambio, los anticolinérgicos (antimuscarícos; parasimpaticolíticos)
inhiben el detrusor (se utilizan en la incontinencia de urgencia); si a nuestro paciente no le
funciona el detrusor, no se los podremos dar (respuesta 5 correcta).

PTS: 1
6. ANS: A
Esta paciente presenta una vejiga neurógena como consecuencia de una neuropatía por su
diabetes. Ésto determina un detrusor arrefléxico, lo que limita el vaciado vesical con residuo
postmiccional elevado; por lo tanto, en estos enfermos la mejor medida es la evacuación de
esa orina retenida siendo necesario un sondaje, que siempre que sea posible es mejor de
forma intermitente (cateterizaciones periódicas diarias) por el menor riesgo de complicaciones
asociadas como infecciones.

PTS: 1
7. ANS: A
La principal causa de disfunción eréctil es la patología orgánica (90% frente al 10% que
representa la causa psicógena). Dentro de este grupo, la principal patología asociada es la
vascular; siendo la enfermedad subyacente más habitual la diabetes, seguida de HTA.
hipercolesterolemia, cardiopatía isquémica y tabaquismo.

PTS: 1
8. ANS: C
El citrato de sildenafilo es un inhibidor de la 5 fosfodiesterasa. Está contraindicado en:
­ Individuos en los que se desaconseja el ejercicio físico moderado.
­ Combinación con donadores de óxido nítrico como el mononitrato y dinitrato de isosorbide o
el nitroprusTito sódico.
Además, es necesario ajustar su dosis en:
­ Pacientes con hepatopatía activa.
­ Pacientes en tratamiento con fármacos que modifiquen el metabolismo hepático.
­ Pacientes con retinitis pigmentaria.
El efecto adverso más frecuente de estos fármacos es la cefalea.

PTS: 1
9. ANS: D
En un paciente tratado con sildenafilo (Viagra) puede ser peligrosa la asociación de fármacos
vasodilatadores. Este hecho es de particular importancia en pacientes afectos de
cardiopatía isquémica, en los que la asociación de dos fármacos vasodilatadores venosos
(nitratos) potentes podría inducir un estatus de bajo gasto y la aparición de angor
hemodinámico.

PTS: 1

2
ID: A

10. ANS: D
Las causas de disfunción eréctil organiCa son múltiples, contándose entre las más frecuentes
las de causa vascular (70%), farmacológicas (1 0%), iatrogénicas (10%), neurológicas (5%),
endocrinológicas (3%) y traumáticas (2%).

PTS: 1

3
UROLOGÍA [Answer Strip] ID: A

A
_____ 7.

A
_____ 1. C
_____ 8.

A
_____ 2.

D
_____ 9.

D
_____ 3.

D 10.
_____

C
_____ 4.

D
_____ 5.

A
_____ 6.
Name: ________________________ Class: ___________________ Date: __________ ID: A

CIRUGIA 2

Multiple Choice
Identify the choice that best completes the statement or answers the question.

____ 1. ¿A qué tipo de abdomen Agudo pertenece la peritonitis?


a. Abdomen Traumatico
b. Abdomen Inflamatorio
c. Abdomen Obstructivo
d. Abdomen Hemorragico

____ 2.
¿Cuáles son las causas que producen una perforación tífica?

a. Colecistitis

b. Colitis ulcerativa

c. Apendicitis

d. Todas las anteriores


____ 3.
Mujer de 75 años de edad con HTA controlada, hipercolesterolemia, que 2 años antes el
inicio del cuadro actual fue estudiada por presentar un cuadro de diarreas y se
realizó. Enema Opaco, que no mostró alteraciones y posteriormente, colonoscopia:
hasta ciego no se ven alteraciones de la mucosa. En la actualidad acude al Servicio
de Urgencias con un cuadro de dolor abdominal, más intenso en hemiabdomen
izqdo. Malestar general, sudoración y emisión de sanare roía tranca gar ano. En la
exploración física destaca el abdomen doloroso, con leve distensión y el tacto rectal
muestra restos hemáticos. Presenta Hto. 36%, Hb 11,7 g/dl, VCM 78 fl, urea 55,
creatinina 1,1 mg/dl. Se realiza colonoscopía. ¿Cuál considera que es el diagnóstico
más probable?
a. Cancer de Colon
b. Colitis Ulcerosa
c. Colitis Actìnica
d. Colitis Isquemica

1
Name: ________________________ ID: A

____ 4. Un varón de 50 años, cardiópata conocido, en fibrilación auricular crónica acude a servicio
de emergencia refiriendo dolor centro abdominal intenso y continuo, irradiado a epigastrio,
y de comienzo brusco hace unas 2 horas. En la exploración, el paciente está estable y con
sensación de mucho dolor abdominal, aunque el abdomen aparece blando y depresible sin
signos de irritación peritoneal. La exploración radiológica simple de abdomen es normal.
Señale, entre las siguientes, la afirmación correcta
a. La localización y características del dolor permiten descartar una isquemia
miocárdica.

b. La exploración abdominal normal permite descartar un abdomen agudo quirúrgico


c. Se debe realizar arteriografía mesentérica urgente para descartar una embolia
mesentérica

d. Lo más probable es que se trate de un dolor abdominal inespecífico y sin


consecuencias adversas.

____ 5.
Un paciente de 72 años, con fibrilación auricular crónica, acude al servicio de Urgencia
por un cuadro brusco de intenso dolor abdominal en región periumbilical progresiva. En
la arteriográfica selectica se observa una obstrucción redondeada de la arteria mesentérica
superior distal a la salida de la arteria cólica media. El tratamiento fundamental será:

a. Quirúrgico: embolectomía y/o resección del intestino no variable.

b. Quirúrgico: derivación mesentérica safena sin resección intestinal.

c. Medico: bolos de vasopresina por vía sistémica.

d. Medico: perfusión continua de glucagón por vía arterial.

____ 6.
Una mujer de 85 años ingreso con insuficiencia ingreso con insuficiencia cardiaca y
diarrea sanguinolenta. La exploración abdominal no mostraba signos de irritación
peritoneal y había ruidos. Un enema opaco mostro imágenes “en huella digital” y la
colonoscopia, colitis en colon izquierdo respetando la mucosa rectal. ¿Cuál sería, entre
las siguientes, la actuación correcta?
a. Realizar artereografia

b. Practicar hemicolectomía izquierda


c. Prescribir dieta absoluta, tratamiento de insuficiencia cardiaca y observación
d. Instaurar anticoagulación con heparina

2
Name: ________________________ ID: A

____ 7.
Varón de 70 años, con enfermedad pulmonar obstructiva crónica con restricción modera,
insuficiencia renal crónica, con creatinina sérica de 3ml/dl, infarto de miocardio antiguo y
aneurisma de aorta abdominal de 7cm de diámetro, asintomático. ¿Cuál es la actitud
correcta?

a. Resección quirúrgica del aneurisma e interposición de injerto aórtico.

b. Controles con Tomografía Axial Computarizada anuales.


c. Mediación antiagregante plaquetaria por alto riesgo quirúrgico.

d. Anticoagulación con dicumarinicos y controles con Resonancia Magnética


Nuclear
____ 8. A sala de urgencia llega un politraumatizado múltiple con costillas derechas fracturadas, que
se presenta en coma medialmente reactivo con discreta anisocoria pupilar, hipotensión
arterial muy grave, grave compromiso respiratorio con murmullo vesicular inaulidable en
hemitórax derecho y abdomen contracturado a la palpación. Indique, entre los siguientes,
cual es los procedimientos asistenciales menos prioritarios
a. Radiografia de torax

b. Intubación traqueal
c. Puncion – lavado intraperitoneal
d. Tomografía computalizada craneal
____ 9. Discreta distensión abdominal y matidez en flancos y el Hto, que era prácticamente normal al
ingreso, disminuye a 30 % a las tres horas. En la Rx de tórax se objetiva fractura de las
costillas 10-11 izquierdas. La causa más probable de la anodización es este paciente es
a. Traumatismo renal con hemorragia retroperitoneal

b. Rotura de hígado con hemoperitoneo


c. Rotura de bazo con hemoperitoneo
d. Traumatismo pancreático con pancreatitis traumática
____ 10.
Indique en que tipos de heridas se utiliza el Hilo de Poliamida y en que cirugía es
recomendable.

a. Heridas complicadas, cirugía.


b. Heridas Simples y complicadas.
c. Heridas Simples y en cirugía plástica.
d. Cirugia

3
Name: ________________________ ID: A

____ 11. El ácido poliglicolico cuales son sus propiedades e indicaciones


a. Monofilamento, Sintético, gastroenterología.
b. Heridas Simples y en cirugía plástica.
c. Negro Alta Fuerza tensil, Inerte, dura de 14 a 21 días.
d. Multifilamento trenzado, fuerza tensil dura de 14 a 21 días, cirugía general,
gastroenterología, urología, etc.

____ 12.
Como se realiza la Desinfección del contorno de la herida.

a. Antiinflamatorios, analgésicos.
b. Tocando los bordes de la herida
c. Antisépticos de uso tópico, de preferencia no irritantes, Iodopovidona o alcohol.
d. Zona menos sensible al dolor.

____ 13. Que líneas se utiliza para realizar la diéresis de la piel para realizar las suturas.
a. Líneas de klefla.
b. Líneas de langer
c. Puntos anatómicos
d. Tejido celular subcutáneo.

4
Name: ________________________ ID: A

____ 14.
Con respecto a la profilaxis antibiótica en cirugía responda cual de los siguientes
enunciados es falso.

a. Debe utilizarse un fármaco que actúe sobre los microorganismos mas


comunes en el sitio quirúrgico.
b. Debe administrarse 12 horas antes de la intervención quirúrgica
c. El antibiótico debe repetirse durante operaciones prolongadas según la
semivida del fármaco para asegurar las concentraciones adecuadas en los
tejidos.

d. en la profilaxis habitual, el régimen antibiótico no debe continuarse más de 24


h después de la intervención quirúrgica.

____ 15.
Son signos de infección en el sitio quirúrgico, EXCEPTO.

a. Edema.
b. Eritema.
c. Hipotermia.
d. Dolor.

____ 16.
Dentro de los factores del paciente que promueven una infección en el sitio quirúrgico
cual de los siguientes no corresponde.

a. Edad, obesidad, tabaquismo


b. Preparación del sitio quirúrgico deficiente.
c. Anemia, desnutrición.
d. Patología de base.

5
Name: ________________________ ID: A

____ 17.
Las infecciones del sitio quirúrgico se clasifican en. EXCEPTO:

a. aInfecciones insicionales.
b. Infecciones de órgano/ espacio.
c. Limpias, limpias-contaminadas, contaminadas y sucias.
d. Infecciones de piel y tejido subcutáneo e infecciones profundas.

____ 18.
La peritonitis primaria o espontánea se produce por invasión bacteria via:

a. Hematogena o linfática
b. Por perforación de víscera hueca
c. Perforación por úlcera
d. Ninguna

____ 19. Con respecto a la herida limpia/contaminada señale el enunciado falso.


a. Comprenden aquellas en las cuales se abre una víscera hueca, como las vías
respiratorias, digestivas o genitourinarias.
b. Ejemplos de estas son: Diverticulitis perforada, infecciones necrosantes de tejido
blando.
c. El cierre de estas heridas es por primera intención
d. Ejemplos de estas heridas son: Colecistectomía, cirugía electiva de tubo digestivo.

____ 20.
Sobre cicatrización patológica: señale lo correcto
a. Entre las principales encontramos: Cicatriz Hipertrófica, cicatriz dolorosa,
Ulceras cicatriciales rebeldes.
b. Cicatriz Hipertrófica: surge por un desarrollo excesivo de miofibroblastos,
que conlleva a una tendencia a la retracción y tiene importante
repercusiones funcionales. Suele diferenciarse del queloide en que si
sobrepasa los límites de la cicatriz.
c. Queloide: cicatriz exuberante por exceso de colágeno, que rebasa los límites
de la piel sana. Se considera un tumor benigno. Son más frecuente en la
región pree esternal y espalda.
d. Ulceras cicatriciales rebeldes: se ocasionan cuando la proliferación del colágeno
estrangula la formación de yemas vasculares, produciendo isquemia.

6
Name: ________________________ ID: A

____ 21.
Fisiología de la cicatrización: señale lo incorrecto
a. Fase inflamatoria
b. Epitelización. Fase celular o de neo formación vascular
c. Fase proliferativa y de síntesis de ácido hialuronico
d. Fase de remodelado
____ 22.
Señale lo correcto sobre los aspectos que afectan a la cicatrización de heridas
a. Existen dos aspectos: Sistémicos y Locales
b. Sistémicos: edad, nutrición, traumatismos, enf. Metabólicas,
inmunodepresión, trastornos del tejido conjuntivos, tabaquismo.
c. Locales: lesión mecánica, infección, edema, isquemia/necrosis de tejido,
agentes tópicos, radiación ionizante, tensión de oxigeno baja, cuerpos extraños.
d. Todas son correctas.
____ 23.
Características de Queloides y Cicatrices Hipertróficas: señale lo correcto
a. Queloide: incidencia rara, predisposición genética, no hay regresión
b. Cicatriz hipertrófica: incidencia frecuente, sin predilección por grupos
étnicos,
c. Cicatriz hipertrófica: sin predisposición genética, regresión frecuente y
espontanea
d. Todo es correcto

____ 24. Sobre la clasificación de los materiales de sutura según su origen ponga la respuesta correcta
a. Simples y compuestos
b. Inorgánicos, orgánicos, sintéticos

c. acero inoxidable, agrafes, catgut.

d. polipropileno, polietileno, poligaláctico


____ 25. Señale las complicaciones de una hemorroidectomía.
a. Retención urinaria
b. Retención fecal
c. Hemorragia masiva
d. Todas
e. Ninguna

7
Name: ________________________ ID: A

____ 26. Referente al tratamiento del absceso perianal, señale lo falso:


a. Drenaje bajo anestesia local en el consultorio, clínica o departamento de
urgencias.
b. En abscesos grandes y complicados es indispensable el drenaje en el quirófano
c. Hacer una incisión cutánea y se extirpa un disco de piel para prevenir el cierre
prematuro.
d. Los abscesos simples se drenan a través de una incisión de la piel suprayacente.
____ 27.
Señale lo falso referente al quiste pilonidal
a. Consiste en un seno o absceso que contiene pelo y aparece en la hendidura
interglútea.
b. La hendidura interglútea ocasiona aspiración que introduce pelos al interior de
fosos en la línea media cuando el sujeto se sienta.
c. Estos abscesos suelen ser muy profundos, por lo que el procedimiento se
realiza en el quirófano.
d. Debe cortarse y drenarse el absceso agudo.
____ 28.
Referente al procedimiento correcto para el quiste pilonidal señale lo verdadero
a. Consiste en destechar el trayecto, raspar la base y marsupializar la herida.
b. Mantener limpia la herida y sin pelo hasta que cicatrice por completo.
c. Practicar una incisión lateral pequeña y extirpar el foso.
d. Todas
e. Ninguna
____ 29.
Referente a una fistula anal, señale lo incorrecto:
a. Casi todas las fístulas son de origen criptoglandular
b. Las fístulas se clasifican de acuerdo con su relación con el complejo del
esfínter anal y las opciones terapéuticas se basan en estas clasificaciones:
fístula interesfinteriana, fístula Transesfinteriana, fístula
supraesfinteriana, fístula extraesfinteriana.
c. El objetivo del tratamiento de la fístula anal es erradicar la infección sin
sacrificar la continencia.
d. Los trayectos fistulosos circundan tramos variables del complejo esfinteriano,
por lo tanto no es considerable un tratamiento quirúrgico.

8
Name: ________________________ ID: A

____ 30.
Sobre el tema: tratamiento de las heridas: señale lo correcto
a. Cierre secundario o por primera intención, cierre por segunda intención y,
cierre por tercera intención o sutura primaria diferida.
b. Por primera intención: sutura inmediata de la herida. En heridas con
mínima contaminación bacteriana, hemorragia controlable, y sin tejido necrótico
ni cuerpos extraños.
c. Cierre por segunda intención: se sutura la herida y no se espera que
cicatrice espontáneamente. Indicada en: heridas muy contaminadas,
cuando el tratamiento se ha demorado más de 6 -8 horas, cuando hay
trayectos muy irregulares, mordeduras humanas y de animales con
tratamiento antibiótico

d. Ninguna es correcta.

____ 31.
Señale la opción correcta acerca de la pancreatitis aguda y la frecuencia de
presentación:

a. Ileo reflejo 90%


b. Dolor Abdominal 50%
c. Hemorragia 12%
d. Vomitos 50%

____ 32. Aun paciente de 70 años, colecistectomizado, con ictericia de 48 horas, de evolución, con bilirrubina total
de 8 mg/dl y bilirrubina directa de 6 mg/dl. fosfatasa alcanila 620 U/L, fiebre de 39 C y leucocitosis
mayor de 20000 con desviación a la izquierda. Se le relaiza eco abdominal que da como resultado
coledocolitiasis. El tratamiento inicial debe ser:
a. Reposición hidroelectrolítica y antibioticoterapia unicamente, posponiendo cualquier
otro proceder a la desaparición de los síntomas y signos de la infección
b. Reposición hidroelectrolítica y antibioticoterapia y laparotomía urgente
c. Reposición hidroelectrolítica y antibioticoterapia y corticoides
d. Reposición hidroelectrolítica y antibioticoterapia y esfinterotomía más drenaje bililar
más colangiografía retrógrada endoscópica

9
Name: ________________________ ID: A

____ 33.
Hombre de 69 años de edad con antecedentes de diabetes mellitus y consumo
moderado de alcohol desde hacía años, que acude a Urgencias por dolor abdominal de
tipo cólico de 7 días de evolución en hemiabdomen superior, más localizado en
hipocondrio derecho. Asocia prurito generalizado que le dificulta el sueño y orinas
oscuras en los últimos dos días. En la exploración física destacaba TA 130/61, FC 102
lpm, Tª 36,8ºC. El abdomen estaba blando, depresible, doloroso a la palpación
profunda y sensación de masa en el hipocondrio derecho. Los análisis de sangre
muestran proteína C reactiva 12.0 mg/dL, glucosa 115 mg/dL, urea 45 mg/dL,
creatinina 0.72 mg/dL, bilirrubina total 8.45 mg/dL, electrolitos normales, ALT (GPT
plasma) 45 U/L, GGT 112 U/L, LDH 110 U/L, Lipasa 16 U/L. Leucocitos 9.3 10
E3/μl, hematíes 4.08 10 E6/μl, hemoglobina 12.3 g/dl, hematocrito 35,9%, VCM 87.9
fl, recuento de plaquetas 217 10 E3/μl ( N 69.3%, L 26.2%, M 4.05%, Eo 0.0%, Bo
0.0%). Se efectuó ecografía abdominal que se muestra en la imagen dilatación del
colédoco, con positividad de coledocolitiasis.
¿Cuál es la actitud más recomendable en este momento?

a. Colecistectomía, Antibioterapia y medidas sintomáticas


b. Colangiografía retrógrada endoscópica con papilotomía
c. Colecistectomía diferida
d. Ecoendoscopia para valorar el páncreas
____ 34. Paciente colecistectomizada hace 6 años por colelitiasis. desde hace 6 meses presenta dolor cólico en flanco
derecho a temporadas. En la analítica hay valores de colestacis moderada. La ecografía abdominal
suguiere coledocolitiasis. ¿Cuál entre las siguientes, es la prueba indicada para realizar, en primer lugar,
de este caso?
a. Tomografía computarizada con contraste abdominal IV
b. Colangiografía I.V
c. Colangiografía retrógrada endoscópica
d. Colangiografía isotópica

____ 35. Los càlculos en el colédoco pueden ser asintomáticos o presentar:


a. Còlico biliar
b. Ictericia Obstructiva
c. Colangitis ascendente y pancreatitis
d. Todas las anteriores

____ 36.
Que maniobra se puede realizar al paciente con una hernia de la pared abdominal al
momento de explorar señale la correcta

a. Valsalva
b. Mac Burney
c. Murphy
d. Ninguna de las anteriores

10
Name: ________________________ ID: A

____ 37.
Las hernias umbilicales adquiridas se cierran de forma espontánea alrededor de los.
Señale la correcta

a. 5 años de edad
b. 10 años de edad
c. 12 años de edad
d. Ninguna de las anteriores

____ 38.
Señale cuales son los factores que pueden contribuir a una eventración o hernia
postoperatoria que dé lugar a su formación

a. Obesidad + cicatrización de heridas


b. Diabetes + malos hábitos de alimentación
c. Esfuerzo físico + mala técnica quirúrgica
d. Ninguna de las anteriores

____ 39.
Con respecto a las hernias de la Pared Abdominal señale la opción correcta.

a. En las Hernias ventrales no siempre están afectadas la aponeurosis y


músculos de la pared abdominal
b. Estas pueden ser congénitas y no adquiridas
c. El hallazgo más común es una masa o un abultamiento
d. Todas son correctas

____ 40.
Con respecto a la utilización de las mallas en cirugía señale la opción correcta

a. No son absorbibles
b. Son absorbibles y se degradan
c. Se eliminan desde los tejidos
d. Pierden su integridad estructural más rápido en los tejidos
e. A y B son correctas

11
Name: ________________________ ID: A

____ 41.
Las infecciones del sitio quirúrgico se clasifican en. EXCEPTO:

a. Infecciones insicionales.
b. Infecciones de órgano/ espacio.
c. Limpias, limpias-contaminadas, contaminadas y sucias.
d. Infecciones de piel y tejido subcutáneo e infecciones profundas.

____ 42. La peritonitis primaria o espontánea se produce por invasión bacteria vía:
a. Hematogena o linfática
b. Por perforación de víscera hueca
c. Perforación por úlcera
d. Ninguna
____ 43.
Peritonitis tuberculosa (tuberculosis peritoneal), Etiología:

a. Reactivación de un foco peritoneal secundario a una diseminación hematógena


b. Reactivación de un foco peritoneal secundario a una rotura de una adenopatía
mesentérica afecta
c. Más frecuente en mujeres
d. Todas las anteriores
____ 44.
Síntomas subdiafragmaticos por vecindad en la peritonitis secundaria:

a. Disnea
b. Tos
c. Dolor en hombro izquierdo o derecho e hipo por irritación diafragmática.
d. Todas las anteriores
____ 45. En la peritonitis secundaria extrahospitalara cuál es el tratamiento de elección.

a. Ceftriaxona o cefotaxima + metronidazol.


b. Imipenen o meropenem
c. Piperzacilia + tazobactam
d. El tratamiento de elección en peritonitis secundaria extrahospitalaria es
Ceftriaxona o cefotaxima + metronidazol.

12
Name: ________________________ ID: A

____ 46. Se define fístula enterocutánea a:


a. Una cavidad de líquido infectado y pus localizada dentro de la cavidad abdominal, puede
haber más de una
b. La comunicación anormal entre el aparato gastrointestinal y la piel, con salida del
contenido intestinal a través de la misma
c. La protrusión de cualquier órgano o tejido fuera de la cavidad del cuerpo en que está
alojado normalmente
d. Una respuesta sistémica a un proceso infeccioso localizado

____ 47. Las complicaciones principales de los pacientes con fístulas son:

a. Desequilibrio hidroelectrolítico
b. Desnutrición
c. Sepsis
d. Todas son correctas

____ 48. Según la etiología de las fistulas enterocutáneas las fistulas postoperatorias son causales de:
a. Isquemia intestinal
b. Enfermedad de Crohn
c. Operaciones por cáncer
d. Apendicitis
____ 49. Paciente varón de 69 años intervenido quirúrgicamente hace 13 meses por hernia inguinal derecha
estrangulada, con compromiso de asas intestinales, como consecuencia portador de ileostomía, hace 13
días acude al servicio para intervención quirúrgica de restitución de tránsito intestinal, donde le realizan
anastomosis latero lateral ileontransversa isoperistaltica, afrontada en dos planos. Actualmente ingresa al
servicio por presentar malestar general, asociado a secreción de material fecaloide por sitio operatorio.
Cúal es su diagnóstico?

a. Hernia estrangulada
b. Fístula biliocutánea
c. Fascitis necrosante
d. Fístula enterocutánea

____ 50. Cuál es la complicación más temida en los pacientes con fistulas enterocutáneas.
a. Formación de adherencias
b. Deshidratación
c. Sepsis
d. Todas las anteriores

13
Name: ________________________ ID: A

____ 51.
Respecto a los canales de calcio: señale lo incorrecto
a. A los canales de calcio también se les denomina canales lento y necesitan 10 a 20
veces más tiempo para su activación que los canales de sodio.
b. Cuando hay un déficit de iones calcio la fibra e hace muy excitable
c. Para que se produzca tetania muscular es necesario una disminución del 30% del
ion calcio
d. Los iones Ca+ parecen unirse a la superficie externa de la molécula de la proteína
del canal sodio

____ 52.
Todos los signos y síntoma de hiponatremia excepto:
a. Taquicardia
b. Hipertensión arterial
c. Convulsiones
d. Diarrea acuosa

____ 53.
Todo son signos y síntoma de hipernatremia excepto:
a. Taquicardia
b. Hipotensión
c. Bradicardia
d. Delirio
____ 54.
Los síntomas predominantes de la hiperpotasemia son excepto:

a. Gastrointestinales
b. Neuromusculares
c. Cardiovasculares
d. Respiratorios
____ 55. ¿Cuál no es una causa de hipopotasemia?
a. Ingestión insuficiente
b. Excreción renal disminuida
c. Diuréticos
d. Diarreas

14
Name: ________________________ ID: A

____ 56.
La nutrición parenteral total determina acciones como:

a. Mayor expresión del interferón gamma en el epitelio intestinal


b. Perdida de la función de barrera epitelial intestinal
c. Presenta complicaciones sépticas con alimentación entera
d. Solo a y b
____ 57. Las causas de la desnutrición en un paciente hospitalizado son, señale la respuesta incorrecta
a. infecciones agudas o cronicas
b. ayunos repetidos (paciente bien investigados pero ma nutridos )
c. cicatrizacion deficiente de heridas
d. uno de infusiones glucosadas (hipocaloricas)

____ 58. En qué período de tiempo se da el ayuno prolongado?


a. de 14 a 16 horas sin ingerir alimentos.
b. de 24 a 48 horas sin ingerir alimentos
c. más de 72 horas.
d. más de 96 horas

____ 59. ¿La nutrición enteral es?


a. Modalidad de terapia nutricional en la cual se administran soluciones de elementos
nutritivos en el torrente circulatorio
b. Es una técnica de soporte nutricional mediante la cual se aportan nutrientes de forma
directa al aparato digestivo, por vía oral mediante fórmulas líquidas químicamente
definidas o en los diversos tramos del tubo digestivo con sondas específicas.
c. Todas las anteriores
d. Ninguna de las anteriores

____ 60. ¿La necesidad basal de potasio es de?


a. 90mEg
b. 5 mEg
c. 20mEg
d. Ninguna de las anteriores

15
Name: ________________________ ID: A

____ 61. Paciente de 11 años llega al servicio de emergencia por presentar, hematuria macroscópica y dolor en
epigastrio que se irradia hacia fosa iliaca derecha, con síntomas acompañantes: disuria, fiebre máxima de
38,5°C en las últimas 24 horas, presenta 3 vómitos desde el inicio del cuadro, no presenta diarreas y la
diuresis se encuentra conservada. ¿Al examen físico que signo especifico desea encontrar?
a. Signo de Mc Burney
b. Signo de Murphy
c. A y B son correctas
d. Ninguna de las anteriores.

____ 62. Paciente de 33 años a emergencia por presentar dolor en hipocondrio derecho con intensidad 8/10, refiere
haber ingerido comida copiosa, acompañado de nausea, vomito e hipertermia. Al examen físico: dolor a
la descompresión y presencia de signo de Murphy. ¿Qué espera encontrar en los exámenes de
laboratorio?
a. Leucocitos y neutrofilia
b. Leucocitosis entre 12000 y 15000
c. Bilirrubinas hasta niveles de 4mg/dl
d. Todas las anteriores son correctas.

____ 63. ¿Cuál es el examen Gold estándar para identificar la pancreatitis crónica?
a. Ecografía abdominal
b. Rx de abdomen
c. Resonancia magnética de abdomen
d. CPRE o colangiopancreatografia retrograda endoscópica

____ 64.
Cuando se presenta peritonitis terciaria señale la correcta.
a. En pacientes postoperados con una peritonitis secundaria que no
responden a tratamientoy que presentan fallo multiorganico o sepsis.
b. Aparece tras una complicación intraabdominal o tras contaminación
quirúrgica o trauma.
c. Esta relaciona con un foco abdominal o perforación del tubo digestivo.
d. Ninguna
____ 65.
El absceso intrabdominal está vinculado con ciertas patologías señale la correcta.
a. Infección de sitio quirúrgico
b. Sigmoitis aguda
c. Ulcera perforada
d. Todas las anteriores

16
Name: ________________________ ID: A

____ 66.
Cuáles son los factores predisponentes para un absceso intraabdominal señale la
incorrecta.
a. Liberación de microorganismos en la cavidad peritoneal
b. Sinergismo bacteriano
c. Ausencia de hemoglobina y pigmentos biliares
d. Obstrucción
____ 67. Los abscesos abdominales según su clasificación por la localización intraperitoneal tenemos.
Señale la incorrecta.

a. Visceral.
b. De los espacios anatómicos existentes.
c. Del espacio de un órgano extirpado.
d. Diafragmaticos
____ 68.
Entre las complicaciones de un absceso intraabdominal tenemos las siguientes . Señale
la incorrecta.

a. Reaparición del absceso


b. Ruptura del absceso
c. Diseminación de la infección
d. Cáncer
____ 69. La hernia inguinal incarcerada se manifiesta:
a. Bulto doloroso en la región inguinal y cuadro obstructivo
b. Bulto doloroso, caliente, eritematoso o azulado
c. Se acompaña de leucocitosis, fiebre y signos de sepsis
d. Cuadro obstructivo, eritematoso o azulado
____ 70. La hernia inguinal estrangulada se debe:
a. Intentar reducir bajo sedación suave
b. Debe ser intervenida sin intentar reducirla
c. No se debe intentar reducir por el riesgo que conlleva reintroducir un segmento
intestinal con compromiso vascular
d. B y C son correctas

17
Name: ________________________ ID: A

____ 71. La hernia directa:


a. Protruye a través del suelo del canal inguinal a nivel del triángulo de Hesselbach,
no pasan a través del orificio profundo y no se localizan por dentro de las fibras
del cremáster, sino por detrás.
b. No tiene relación con el conducto inguinal, su riesgo de incarceración y
estrangulación es más elevado que en cualquier otra hernia
c. Sale de la cavidad abdominal por el anillo profundo, surge lateralmente a la arteria
epigástrica y al ligamento de Hesselbach
d. Acompaña a las estructuras del cordón inguinal por dentro de las fibras del
músculo cremáster, pudiendo salir por el orificio externo hasta el escroto.
____ 72. En la hernioplastia como procedimiento quirúrgico en hernias inguinales se utiliza con
mayor frecuencia las técnicas:
a. Técnica de Lichtenstein
b. Técnica de Rutkow
c. Técnica original de Bassini
d. A y B son correctas
____ 73.
Paciente de 22 años refiere que hace 3 horas recibe puñalada con arma blanca en
hipocondrio derecho, al cuadro clínico se acompaña de hemorragia que no sede,
hipotensión, bradicardia, bradipnea y con Glasgow 12, cuál sería el primer examen que se
le realizaría al paciente.

a. EMO

b. Tomografía computarizada

c. Biometría hemática

d. Rx de tórax

____ 74. Que es el traumatismo abierto y cuáles son sus características:

a. Se caracteriza por no presentar solución de continuidad en la pared abdominal. El


agente que lo produce es de superficie roma o plana, tipo barra de timón, puño,
etc.
b. Es cuando existe solución de continuidad en la pared abdominal, producida por
elementos cortantes o transfixiones, como en las heridas por arma blanca o heridas por
arma de fuego

c. Se caracteriza por no presentar una cicatriz a nivel abdominal

d. Se caracteriza por no presentar solución una herniación a nivel abdominal.

18
Name: ________________________ ID: A

____ 75.
Mujer de 54 años que, desde hace 6 horas, presenta vómitos intensos y dolor abdominal.
En la exploración clínica se aprecia tumoración umbilical dolorosa y en la radiología
simple, dilatación de asas de intestino delgado. Entre los siguientes, el diagnóstico más
probable es:

a. Tumor de intestino delgado

b. Invaginación intestinal

c. Hernia estrangulada

d. Estenosis Pilórica

____ 76.
Paciente de 75 años, sin antecedentes clínicos de interés, que consulta por un episodio de
hematoquecia reciente. La colonoscopia total demuestra un pólipo único o pediculado, de
unos 25 mm de diámetro, en sigma con una ulceración en su cúspide. ¿Qué actitud de las
siguientes es más adecuada?

a. Seguimiento cada 6 meses e intervención quirúrgica, si aumenta de tamaño

b. Indicar intervención quirúrgica.

c. Biopsia endoscópica del pólipo y decidir según resultado.

d. Realizar polipectomía endoscópica y estudio anatomopatológico del pólipo


____ 77. Ante un paciente que presenta dolor abdominal de tipo cólico, vómitos, distención abdominal
e incapacidad para evacuar gases y heces, hay que pensar como primer diagnóstico en:
a. Diverticulitis aguda c. Colecistitis
b. Obstrucción intestinal d. Apendicitis aguda

19
Name: ________________________ ID: A

____ 78.
Enfermo de 50 años que acude al servicio de urgencias por dolor abdominal difuso y
progresivo, distensión, borborigmos y vómitos ocasionales en las últimas 48 horas.
Apendicetomía a los 14 años. En la exploración hay fiebre (38.5°C) distensión abdominal
con ruidos intestinales aumentados y ocasionalmente en “espita”, sensibilidad a la
palpitación abdominal sin defensa ni signo de rebote. En la analítica hay leucocitosis con
neutrofilia; NA 133 mEq/L; CO#H de 14 mEq/L. En la placa simple de abdomen hecha en
bipedestación hay dilatación de las asas del delgado con niveles y edema de la pared, sin
prácticamente gas en el colon, ¿Cuál sería la conducta a seguir?

a. Aspiración nasogastrica más antibioticos c. Laparotomía urgente


de amplio espectro
b. Enema opaco para descartar volvulo de d. Colonoscopia descompresiva
sigma

____ 79.
Paciente masculino de 74 años de edad que presenta dolor abdominal de tipo cólico,
diarrea, distensión abdominal e incapacidad para evacuar gases y heces, hay que pensar
como primer diagnóstico en la radiología simple se observa un dilatación colónica con
una imagen de “asa en omega” o en “grano de café”

a. Volvulo del sigma c. Síndrome de Colon Irritable

b. Pseudoobstrucciónn del colon d. Obstrucción intestinal

____ 80.
Una mujer de 54 años inicia estudio por molestias abdominales. El escáner descubre una
lesión quística de 8 cm en el ovario derecho y dos nódulos abdominales que parecen
depender del peritoneo. El Ca-125 es normal. ¿Cuál de estas actitudes le parece más
acertada?

a. Laparotomía exploradora para c. Se trata de un quiste diseminado, la


diagnosticar y tratar un probable paciente debe recibir quimioterapia
cáncer de ovario sistémica.

b. Puede tratarse de un quiste lúteo, d. Vigilancia, repitiendo la exploración


convendría realizar un tratamiento de escáner en tres meses
prueba con anovulatorios y reevaluar
en tres meses.

20
Name: ________________________ ID: A

____ 81.
Paciente que presenta amenorrea de 7 semanas, asintomática, sin evidencia, mediante
ecografía vaginal, de útero ocupado. Se determina la fracción b de la gonadotropina
coriónica humana, obteniéndose una cifra de 2500 mU/ml. Ante estos datos, en primer
lugar, habrá que pensar en:

a. Gestación de evolución normal c. Gestación ectópica de evolución


correspondiente a la amenorrea asintomática.
b. Gestación normal con menor tiempo d. Aborto precoz completo, con
de evolución del correspondiente a su expulsión total de restos ovulares
amenorrea. intrauterinos.
____ 82.
Paciente de 30 años, nuligesta, con última regla hace 20 días, que acude a la consulta por
presentar esterilidad y dismenorrea importante. En las exploraciones que se realizan,
destaca tumoración anexial en la ecografía y elevación moderada de Ca-125 sérico. ¿Cuál
es el diagnóstico de presunción?

a. Embarazo ectópico c. Mioma uterino


b. Cáncer de ovario d. Endometriosis
____ 83. Paciente de 20 años que acude a consulta por dolor abdominal localizado a nivel de fosa iliaca
izquierda, de intensidad 6/10, de tipo sordo, desde hace aproximadamente 2 semanas, el
dolor se acompaña además nauseas que llegan al vómito por dos ocasiones en el último día.
Paciente es diagnosticado con enfermedad inflamatoria pélvica leve. ¿Cuál es el tratamiento
antibiótico ambulatorio de elección que usted prescribiría?

a. Clindamicina y gentamicina. c. Ceftriaxona y doxiciclina.


b. Metronidazol. d. Azitromicina.

21
Name: ________________________ ID: A

____ 84.

6. Adolescente de 15 años de edad, nulípara, acude con dolor abdominal bajo, de inicio
súbito, de 60 minutos de evolución, sin vómito, evacuaciones normales, sin molestias
urinarias ni flujo genital. Al examen físico: paciente muy quejumbrosa, afebril, con
palidez de piel y mucosas, diaforesis, sin agitación psicomotora, abdomen blando,
depresible, sensible en la fosa iliaca derecha, signo de Blumberg ausente, sin masas
palpables. Con un espéculo se observó el cuello uterino sano, sin metrorragia. Al
tacto vaginal se apreció el útero en retroversoflexión, sin dolor a la movilización
cervical, anexos sin masas palpables. La ecografía transvaginal mostró el útero en
retroversoflexión. En relación con el fondo uterino y hacia la izquierda, se observó
una imagen bien delimitada, ecogénica, de 8 x 9 cm de diámetro. Se observó una
representación ecográfica del pedículo vascular rotado, un vaso arterial central
rodeado por un vaso venoso, lo que daba la imagen de remolino, se perdía el sentido
paralelo en que corrían los vasos arteriales y venosos, compatible con el signo del
remolino. ¿En qué diagnóstico pensaría usted?

a. Torsión ovárica c. Enfermedad inflamatoria pélvica


b. Embarazo ectópico d. Ovario poliquístico
____ 85.
Paciente de 87 años con antecedentes de bronquitis crónica e insuficiencia cardiaca ha sido
diagnosticado de colecistitis aguda litiásica tras 4 días de hospitalización en tratamiento con
dieta absoluta, sueroterapia, piperacilina/tazobactam, el paciente continua con fiebre, dolor
abdominal persistente y leucocitosis. La actitud más adecuada en estos momentos sería:

a. Tratamiento quirúrgico c. Sustituir la piperacilina/tazobactam


colecistectomía urgente. por metronidazol + cefotaxima.
b. Drenaje biliar mediante d. Añadir al tratamiento un
colecistostomia percutánea. aminoglucósido como gentamicina.
____ 86.
Paciente de 52 años con datos clínico- biológicos de ictericia obstructiva de evolución
intermitente y sangre oculta en heces positiva, con dieta idónea. De los diagnósticos que se
reseñan. ¿Cuál es el más probable?:

a. Carcinoma pancreático c. Colecistitis crónica.


b. Litiasis vesicular. d. Ampuloma.

22
Name: ________________________ ID: A

____ 87.
El procedimiento inicial de elección para investigar una posible obstrucción de las vías
biliares es:
a. Ecografía hepatobiliar. c. Colangio resonancia magnética
b. Colangiopancreatografía retrógrada d. Tomografía computarizada.
endoscópica(CPRE).
____ 88.
¿Cuales son los factores que predisponen para el desarrollo de colelitiasis?

a. Obesidad, embarazo, factores c. Esferocitosis hereditaria, talasemia,


dietéticos, cirrosis. diabetes.
b. Enfermedad de Crohn, resección ileal d. Todas las anteriores
terminal
____ 89.
Un tumor de la bifurcación de los conductos biliares principales (t. de Klatskin), se
asocia con los siguientes hechos, EXCEPTO:
a. Ictericia indolora c. Acolia
b. Prurito d. Distensión vesicular
____ 90.
La colecistectomía laparoscópica tiene como ventaja sobre la colecistectomía por
laparotomía:
a. La reducción de la estancia c. No se asocia a lesiones de la vía
hospitalaria. biliar.
b. Permite conservar la vesícula. d. Es una técnica exenta de mortalidad.
____ 91.
¿Cuál de las siguientes sustancias forma parte de la secreción biliar?:
a. Carboxipeptidasa. c. Elastasa
b. Tripsina d. Lecitina

23
Name: ________________________ ID: A

____ 92.
Al leer el resultado de una ecografía abdominal, que usted ordenó por infecciones
urinarias de repetición, comprueba que la mujer de 67 años que tiene sentada en su
consulta tiene cálculos en la vesícula biliar, sin otros hallazgos significativos. La paciente
niega haber presentado cólicos o ninguna otra complicación relacionada con la colelitiasis.
Su única queja digestiva es la distensión abdominal postprandial. ¿Cuál de las siguientes
recomendaciones le daría usted?

a. La distención postprandial indica que c. Debe ser sometida a una


debe ser sometida a una colecistectomía abierta y no
colecistectomía de urgencia. laparoscópica para prevenir un cáncer
de vesícula.

b. No está indicado el tratamiento d. La colecistectomía profiláctica esta


quirúrgico porque se trata de una indicada por tratarse de una mujer
colelitiasis asintomática.

____ 93.
De las cuatro afirmaciones que se hacen sobre la colangitis aguda, una de ellas es
FALSA:

a. La presentación clásica se caracteriza c. Es obligatoria la hospitalización


por fiebre alta, ictericia y dolor reposición hidroelectrolítica, soporte
abdominal. general y terapéutica antibiótica

b. Una causa frecuente de colangitis es d. La endoscopia precoz permite el


la estenosis maligna del colédoco diagnóstico por colangiografía y el
drenaje de la vía biliar

24
Name: ________________________ ID: A

____ 94. Paciente de29 años de edad, tratado por una colitis ulcerosa desde hace dos años, y sin
episodios de descompensación en los últimos 6 meses. Refiere un dolor sordo en
hipocondrio derecho y astenia en el último mes. La exploración física no demuestra
ninguna alteración significativa, y en los estudios complementarios se observa una
bilirrubinemia total de 1.2 mg/ dl (bilirrubina directa 0.7 mg/dl), aspartato
aminotransferasa 89 UI/I (N=40 UI/I), fosfatasa alcalina 1 124 UI/I, (N= 320 UI/I), gamma
glutamiltransferasa 345 UI/I, albuminemia 38g/l y tasa de protrombina 100%. Negatividad
de los anticuerpos mitocondriales. Señale cuál de las siguientes enfermedades padece con
mayor probabilidad.

a. Fase inicial de una cirrosis biliar c. Metástasis hepática de un


primaria adenocarcinoma de colon

b. Hepatitis autoinmune d. Colangitis esclerosante primaria

____ 95.
Señale la respuesta correcta con respecto al diagnóstico de colangitis aguda.

a. Puede ser silenciosos y con frecuencia c. Cursa con falla orgánica, riesgo de
se descubren de manera incidental. progresión a una colangitis grave sin
Pueden provocar obstrucción, drenaje biliar temprano
completa o incompleta

b. Fiebre, dolor en el epigastrio o el d. Hipersensibilidad epigástrica o en el


cuadrante superior derecho e ictericia. cuadrante superior derecho e ictericia
se conocen como la tríada de Charcot ligeras.

25
Name: ________________________ ID: A

____ 96.
En la colangitis aguda es común encontrar en los exámenes de laboratorio lo siguiente:

a. Leucocitosis, hiperbilirrubinemia y c. En el 30% de los casos hay


aumento de la fosfatasa alcalina y de hipergammaglobulinemia y en el
las transaminasas 40-50% hay un aumento de la IgM.

b. Presencia de cálculos en la vesícula d. Fosfatasa alcalina e IgM elevadas.


biliar y demostrará conductos
dilatados y tal vez señala el sitio de
obstrucción.

____ 97.
Señale lo correcto con respecto al tratamiento de colangitis:

a. Se recomienda colecistectomía c. El tratamiento inicial de la colangitis


laparoscópica electiva incluye antibióticos y reanimación
con líquidos por vía intravenosa, es
necesario drenar el conducto biliar
obstruido tan pronto se estabilice el
paciente.
b. Utilización de quelantes de las sales d. El tratamiento específico de la
biliares, administración de vitaminas enfermedad se han ensayado diversos
liposolubles. fármacos como: ursodiol, colchicina,
metotrexate, azatioprina o
ciclosporina. Los corticoides y la
D-penicilina no son eficaces.

26
Name: ________________________ ID: A

____ 98.
Señale lo correcto:

a. La pancreatitis aguda es una c. Muy pocos episodios de pancreatitis


enfermedad inflamatoria del aguda se inician con dolor intenso,
páncreas que no se acompaña de por lo general después de una comida
fibrosis de la glándula, o muy poca. abundante

b. Un 50% de los casos depende de dos d. El dolor característico de la


factores: cálculos de vías biliares y pancreatitis aguda se describe como
alcoholismo, mientras que el restante “puñalada” o “transfictivo” hacia la
50% lo constituyen una enfermedad espalda y se incrementa cuando se
idiopática o una diversidad de causas, inclina el paciente hacia adelante.
entre ellas traumatismo, operación,
fármacos, herencia, infección y
toxinas.

27
ID: A

CIRUGIA 2
Answer Section

MULTIPLE CHOICE

1. ANS: B
Es una inflamación o irritación del peritoneo, el tejido delgado que recubre la pared interna del
abdomen y la mayor parte de los órganos abdominales

PTS: 3 DIF: ALTA


REF: SINCHINARO. (2017). CIRUGIA GENERAL. Argentina: surgeryforum.
OBJ: Reconocer los principales conceptos y patologias relacionados a la cirugia
TOP: CIRUGIA KEY: Abdomen Agudo NOT: DR. VICTOR QUISIGUIÑA
2. ANS: D
Por qué todas estas enfermedades pueden ser una causa de una perforación tífica debido a su
etiología y su forma de presentación.

PTS: 3 DIF: ALTA


REF: Gasman, O. A. (2014). Urgencias Quirurgicas. Quito: Biblioteca de salud publica.
OBJ: Reconocer los principales conceptos y patologias relacionadas a la cirugia
TOP: CIRUGIA KEY: Abdomen Agudo NOT: DR. VICTOR QUISIGUIÑA
3. ANS: D
La forma de presentación de la CI es heterogénea y varía en función de la causa y la extensión de
la oclusión vascular, el tiempo que el colon permanece privado del flujo vascular, el grado de
circulación colateral y las comorbilidades asociadas. En su forma más típica, el paciente presenta
un cuadro de dolor abdominal cólico, a menudo referido al hemiabdomen izquierdo, de
intensidad moderada, seguido de urgencia por la defecación y por la emisión de heces mezcladas
con sangre de color rojo brillante u oscura en las 12-24 h siguientes a la presentación del dolor

PTS: 3 DIF: ALTA REF: Dr. Carlos Téllez. (2014). Aparato digestivo. AMIR,
186.
OBJ: Reconocer los principales conceptos y patologias relacionadas a la cirugia
TOP: CIRUGIA KEY: ABDOMEN AGUDO INFLAMATORIO
NOT: DR.VICTOR QUISIGUIÑA
4. ANS: C
La sospecha de IMA es indicación de arteriografía mesenterio, que es el estudio de diagnóstico
definitivo, y debe realizarse proyecciones anteroposterior y lateral (para visualizar el origen del
tronco celiaco de la AMS)

PTS: 3 DIF: ALTA


REF: Soto S. (2004 - 2005). Isquemi mesenterica cronica o angina intestinal. Manual CTO digestivo,
69.
OBJ: Reconocer los principales conceptos y patologías relacionadas a la cirugía
TOP: CIRUGIA KEY: Isquemia Mesenterica NOT: DR. VICTOR QUISIGUIÑA

1
ID: A

5. ANS: A
El único tratamiento definitivo es la revascularización quirúrgica mediante by-pass con injerto de
dacrón o vena safena o endarterectomía

PTS: 3 DIF: ALTA


REF: Soto S. (2004 - 2005). Enfermedades vasculares intestinales . Manual CTO Digestivo, 289.
OBJ: Reconocer los principales conceptos y patologías relacionadas a la cirugía
TOP: CIRUGIA GENERAL KEY: ENFERMEDADES VASCULARES
NOT: DR. VICTOR QUISIGUIÑA
6. ANS: C
En la colitis isquémica transitoria la más frecuente, los síntomas son leves y se controlan bien con
medicas conservadoras

PTS: 3 DIF: ALTA


REF: Soto S. (2004 - 2005). Enfermedades vasculares intestinales . Manual CTO Digestivo, 289.
OBJ: Reconocer los principales conceptos y patologías relacionadas a la cirugía
TOP: CIRUGIA KEY: ABDOMEN VASCULAR NOT: DR. VICTOR QUISIGUIÑA
7. ANS: A
Los aneurismas son dilataciones localizadas y permanentes de un vaso de por lo menos 1.5 veces
el diámetro normal del mismo. La aorta es el vaso más frecuente implicado, siendo el 80% de los
aneurismas aórticos de localización infrarrenal. La evolución natural es hacia el crecimiento,
siendo la ruptura la complicación más grave y muchas veces mortal. La cirugía es la única actitud
terapéutica definitiva

PTS: 3 DIF: ALTA REF: Cirugia Vascular . (2005). MIR , 94.


OBJ: Reconocer los principales conceptos y patologías relacionadas a la cirugía
TOP: CIRUGIA KEY: ABDOMEN VASCULAR NOT: DR. VICTOR QUISIGUIÑA
8. ANS: D
La tomografía computarizada craneal es la menos indicada porque las lesiones del paciente son
más a nivel de tórax y abdomen, otro dato importante es que este no tiene ningún trauma a nivel
craneal

PTS: 3 DIF: ALTA REF: SOTO. S. (2005). HERNIAS. CTO.


OBJ: Reconocer los principales conceptos y patologías relacionadas a la cirugía.
TOP: CIRUGIA KEY: ABDOMEN TRAUMATICO NOT: DR.VICTOR QUISIGUIÑA
9. ANS: C
La rotura espontánea es rara en el bazo normal. Ésta suele afectar a bazos patológicos,
habiéndose descrito por traumatismos mínimos e incluso espontáneos en distintas entidades,
sobre todo aquellas que cursan con esplenomegalia

PTS: 3 DIF: ALTA REF: SOTO.S. (2005). OBSTRUCCIÓN INTESTINAL.


CTO.
OBJ: Reconocer los principales conceptos y patologías relacionadas a la cirugía
TOP: CIRUGIA KEY: ABDOMEN OBSTRUCTIVO NOT: DR. VICTOR QUISIGUIÑA

2
ID: A

10. ANS: C
Para suturar las Heridas Simples de la Piel, se puede utilizar Hilo de Poliamida. (NYLON /
DAFILON M.R.). Es recomendable en cirugía plástica y reparadora como en oftalmológica. Es
IRREABSORBIBLE, Monofilamento, Sintético, Azul / Negro Alta Fuerza tensil, Inerte.

PTS: 3 DIF: ALTA REF: (MARIO, MANUAL DE CIRUGIA, 2004)


OBJ: Reconocer los principales conceptos y principales patologías relacionados a la cirugia
TOP: Cirugia NOT: Dr. Victor Quisiguiña
11. ANS: D
ACIDO POLIGLICOLICO. (DEXON / SAFIL M. R.) PROPIEDADES: Multifilamento
trenzado, fuerza tensil dura de 14 a 21 días. REABSORBIBLE: degradación por hidrólisis entre
60 y 90 días. INDICACIONES: Cirugía General, gastroenterología. Urología, ginecología,
oftalmología cirugía plástica y pediatría. Se esteriliza en Óxido de etileno.

PTS: 3 DIF: ALTA REF: (MARIO, MANUAL DE CIRUGIA, 2004)


OBJ: Reconocer los principales conceptos y patologias relacionadas ala cirugia
TOP: Cirugia NOT: Dr. Victor Quisiguiña
12. ANS: C
La desinfeccion se realiza con antisépticos de uso tópico, de preferencia no irritantes, por
ejemplo, Iodopovidona, DG6 o alcohol aplicándolo con una gasita estéril suavemente sobre piel
sana, en el contorno de la herida (que el antiséptico no haga contacto con los bordes de la lesión).
En esta zona se hará la primera punción. No tocar los bordes, es la zona más sensible al dolor.

PTS: 3 DIF: ALTA REF: (MARIO, MANUAL DE CIRUGIA, 2004)


OBJ: Reconocer los principales conceptos y patologias relacionadas a la cirugia
TOP: Cirugia NOT: Dr. Victor Quisiguiña
13. ANS: B
LINEAS DE LANGER: Estas líneas fueron diseñadas por el autor en 1861 con el objetivo de
mejorar la estética de la cicatriz. Los estudios lo realizó en cadáveres, obviamente sin tener en
cuenta la dinámica muscular como lo hizo Kraissl en 1951. Estas líneas aseguran una buena
aposición de los bordes de la herida cuando éstas son superficiales (dérmicas) no cuando llegan la
tics.

PTS: 3 DIF: ALTA REF: (MARIO, MANUAL DE CIRUGIA, 2004)


OBJ: Reconocer los principales conceptos y patologia relacionadas a la cirugia
TOP: Cirugia NOT: Dr Victor Quisiguiña
14. ANS: B
En la profilaxis antibiotica se debe administrar de 30 minutos a 1 hora previa a la insicion quirurgica,

PTS: 3 DIF: ALTA


REF: Principios de Cirugía de Schwartz 10ma edición capitulo 6 (Infecciones
OBJ: Reconocer los principales conceptos y patologias relacionadas a la cirugia
TOP: Cirugia NOT: Dr. Victor Quisiguiña
15. ANS: C PTS: 3 DIF: ALTA
REF: Digestivo y Cirugía General AMIR tercera edición, pagina 88.
OBJ: Reconocerlos principales conceptos y patologias relacionadas a la cirugia
TOP: CIRUGIA KEY: Infeccion sitio quirurgico NOT: Dr. Victor Quisiguiña

3
ID: A

16. ANS: B PTS: 3 DIF: ALTA


REF: Principios de Cirugía de Schwartz 10ma edición capitulo 6 (Infecciones
OBJ: Reconocerlos principales conceptos y patologias relacionadas a la cirugia
TOP: CIRUGIA KEY: Infeccion sitio quirurgico NOT: Dr. Victor Quisiguiña
17. ANS: C PTS: 3 DIF: ALTA
REF: Principios de Cirugía de Schwartz 10ma edición capítulo 6 (Infecciones
OBJ: Reconocerlos principales conceptos y patologias relacionadas a la cirugia
TOP: CIRUGIA KEY: Infecciones sitio quirurgico NOT: Dr. Victor Quisiguiña
18. ANS: A

La peritonitis primaria o espontánea se produce por invasión bacteria via Hematogena y linfática

PTS: 3 DIF: ALTA REF: CTO Séptima edición /Medicina y cirugía / Pag 291
OBJ: Reconocerlos principales conceptos y patologias relacionadas a la cirugia
TOP: CIRUGIA KEY: Peritonitis NOT: Dr. Victor Quisiguiña
19. ANS: B PTS: 3 DIF: ALTA
REF: Principios de Cirugía de Schwartz 10ma edición capitulo 6 (Infecciones
OBJ: Reconocerlos principales conceptos y patologias relacionadas a la cirugia
TOP: CIRUGIA KEY: Heridas limpias NOT: Dr. Victor Quisiguiña
20. ANS: C
Queloide: cicatriz exuberante por exceso de colágeno, que rebasa los límites de la piel sana. Se
considera un tumor benigno. Son más frecuente en la región pree esternal y espalda, así como en
sujetos de raza negra.

PTS: 3 DIF: Alta REF: (Maria Rodriguez, 2014)


OBJ: Reconcer los principales conceptos y patologias relacionadas a la cirugia
TOP: Cirugia KEY: Cicatrización NOT: Dr. Victor Quisiguiña
21. ANS: C
La reparación de las heridas presenta una serie de cambios químicos, morfológicos y físicos que
dan como resultado la formación del tejido cicatricial: Fase inflamatoria, Epitelización, Fase
celular o de neo formación vascular, Fase proliferativa y de síntesis de colágeno, Fase de
remodelado

PTS: 3 DIF: Alta REF: (Maria Rodriguez, 2014)


OBJ: Reconocer los principales conceptos y patologias relacionadas con la cirugia
TOP: Cirugia KEY: cicatrización NOT: Dr. Victor Quisiguiña
22. ANS: D
Factores que afectan a la cicatrización de heridas:
Sistémicos
 Edad, nutrición, traumatismos, enf. Metabólicas, inmunodepresión, trastornos del
tejido conjuntivos, tabaquismo.
Locales
 lesión mecánica, infección, edema, isquemia/necrosis de tejido, agentes tópicos,
radiación ionizante, tensión de oxigeno baja, cuerpos extraños.

PTS: 3 DIF: Alta REF: (Charles Brunicardi, 2015)


OBJ: Reconocer los principales conceptos y patologias relacionadas a la cirugia
TOP: Cirugia KEY: cicatrización NOT: Dr. Victor Quisiguiña

4
ID: A

23. ANS: D
Queloide: incidencia rara, predisposición genética, no hay regresión, contractura rara.
Cicatriz hipertrófica: incidencia frecuente, sin predilección por grupos étnicos, sin predisposición
genética, regresión frecuente y espontánea, contractura frecuente.

PTS: 3 DIF: Alto REF: (Charles Brunicardi, 2015)


OBJ: Reconocer los principales conceptos y palabras relacionadas a la cirugia
TOP: Cirugia KEY: Queloides y Cicatrices NOT: Dr. Victor Quisiguiña
24. ANS: B
Desde el punto de vista didáctico los materiales de sutura se clasifican atendiendo su origen o
algunas propiedades específicas.
Así tenemos materiales
- Inorgánicos; acero inoxidable, agrafes.
- Orgánicos; sedas, catgut, lino, algodón.
- Sintéticos; nylon, polipropileno, polietileno, poligaláctico, lino.

PTS: 3 DIF: Alta REF: (MARIO, MANUAL DE CIRUGIA, 2004)


OBJ: reconocer los principales conceptos y palabras relacionadas a la cirugia
TOP: Cirugia KEY: materiales de sutura NOT: Dr. Victor Quisiguiña
25. ANS: D
Una complicación frecuente después de la hemorroidectomía es la retención urinaria y ocurre en
10 a 50% de los casos. El dolor también puede ocasionar retención fecal. También es posible que
después de la hemorroidectomía suceda un sangrado masivo.

PTS: 3 DIF: alta


REF: SCHWARTZ. (2015). Principios de Cirugía. Mexico: Mc Graw Hill .
OBJ: reconocer los principales conceptos y palabras relacionadas a la cirugia
TOP: cirugia KEY: hemorroidectomía NOT: Dr. Victor Quisiguiña
26. ANS: D
Los abscesos isquiorrectales simples son los que se drenan a través de una incisión de la piel
suprayacente

PTS: 3 DIF: alto


REF: SCHWARTZ. (2015). Principios de Cirugía. Mexico: Mc Graw Hill .
OBJ: reconocer los principales conceptos y patologias relacionados a la cirugia
TOP: cirugia KEY: absceso perianal NOT: Dr. Victor Quisiguiña
27. ANS: C
Estos abscesos suelen ser muy superficiales, por lo que con frecuencia es posible practicar este
procedimiento en el consultorio, la clínica o el departamento de urgencias bajo anestesia local.

PTS: 3 DIF: alta


REF: SCHWARTZ. (2015). Principios de Cirugía. Mexico: Mc Graw Hill .
OBJ: reconocer los principales conceptos y patologias relacionados a la cirigia
TOP: cirugia KEY: quiste pilonidal NOT: Dr. Victor Quisiguiña.

5
ID: A

28. ANS: D
Se han propuesto varios procedimientos para tratar un seno pilonidal crónico. El método más
simple consiste en destechar el trayecto, raspar la base y marsupializar la herida. A continuación,
debe conservarse limpia la herida y sin pelo hasta que cicatrice por completo. Otra opción
consiste en practicar una incisión lateral pequeña y extirpar
el foso.

PTS: 3 DIF: alta


REF: SCHWARTZ. (2015). Principios de Cirugía. Mexico: Mc Graw Hill .
OBJ: reconocer los principales conceptos y patologias relacionasos a la cirugia
TOP: cirugia KEY: quiste pilonidal NOT: Dr. Victor Quisiguiña
29. ANS: D
Los trayectos fistulosos circundan tramos variables del complejo esfinteriano, por lo tanto el
tratamiento quirúrgico depende de la localización de las aberturas interna y externa y el trayecto
de la fístula.

PTS: 3 DIF: alta


REF: SCHWARTZ. (2015). Principios de Cirugía. Mexico: Mc Graw Hill .
OBJ: reconocer los principales conceptos y patologias relacionadas a la cirugia
TOP: cirugia KEY: fistula anal NOT: Dr. Victor Quisiguiña
30. ANS: B
Por primera intención: sutura inmediata de la herida. En heridas con mínima contaminación
bacteriana, hemorragia controlable, y sin tejido necrótico ni cuerpos extraños

PTS: 3 DIF: Alta REF: (Maria Rodriguez, 2014)


OBJ: reconocer los principales conceptos y patologias relacionadas a la cirugia
TOP: Cirugia KEY: tratamiento de las heridas NOT: Dr. Victor Quisiguiña
31. ANS: D
Principales manifestaciones clínicas de la pancreatitis aguda y su frecuencia: dolor abdominal
90%, Vómitos 50%, Ileo reflejo 30%, Estado de shock 30%, Disnea 10%, Oliguria o anuria
10-15%, Signos neurológicos 5%, signos de infección 4% y Hemorragia 3%.

PTS: 3 DIF: alto REF: (LEVY, 2015)


OBJ: Reconocer los principales conceptos y patologías relacionadas a la cirugía
TOP: CIRUGIA KEY: vómito NOT: DR. Victor Quisiguiña
32. ANS: D
La coledocolitiasis debe tratarse con CPRE, reservándose la cirugía a los fallos de esta La cpre permitirá
realizar una conlangiografía retrógada endoscópica que permitirá realizar una colangiografía, la
extracción de cálculos y esfinterotomía endoscópica

PTS: 3 DIF: Alto


REF: ORTEGA, M. R. (2014). CTO 9na Edición manuela de Medicina y Cirugía. Mexico.
OBJ: Reconocer los principales conceptos y patologías relacionadas a la cirugía
TOP: Cirugía KEY: coledocolitiasis NOT: DR. Victor Quisiguiña

6
ID: A

33. ANS: A
El tratamiento de la coledocolitiasis consiste en la extracción de cálculos y colecistectomía. Si el
paciente tiene una historia de ictericia obstructiva o pancreatitis, alteración de las pruebas
hepáticas o un cálculo coledociano con dilatación de la vía biliar, es preferible realizar una
CPRE, con esfinterotomía y extracción de cálculos y posteriormente una colecistectomía
laparoscópica. También puede explorarse la vía biliar durante la colecistectomía y realizar la
CPRE tras la colecistectomía, si es necesario. En caso de que no se hayan extraído todos los
cálculos (empedramientos coledocianos o litiasis intrahepáticas) puede ser preciso realizar una
derivación biliodigestiva.

PTS: 3 DIF: Alta


REF: FARRERAS, V., & ROZMAN, C. (2016). MEDICINA INTERNA. BARCELONA: ELSEVIER.
OBJ: Reconocer los principales conceptos y patologías relacionadas con cirugía
TOP: Cirugía KEY: Colicistectomía NOT: DR. Victor Quisiguiña
34. ANS: C
En el caso de coledocolitiasis, tras iniciar un tratamiento médico con antibióticos y fluidoterapia, si la
evolución es favorable se debe realizar una colangiografía retrógrada endoscópica retrógrada con
esfinterotomía para extraer la litiasos con una cestilla endoscópica

PTS: 3 DIF: Alta REF: ARREOS, M. (2012). AMIR. MADRID: MARBAN.


OBJ: Reconocer los principales conceptos y patologías relacionadas con cirugía
TOP: Cirugía KEY: litiasis NOT: DR. Victor Quisiguiña
35. ANS: D
Los cálculos del colédoco pueden cursar asintomáticos o presentar Còlico biliar, ictericia bstructiva,
colangitis ascendente y pancreatitis. Otras complicaciones menos común es la cirrosis biliar secundaria.
Ante un cuadro de colangitis o ictericia en un paciente colecistectomizado, hay que pensar siempre en
coledocolitiasis residual.

PTS: 3 DIF: Alto


REF: ORTEGA, M. R. (2014). CTO 9na Edición manuela de Medicina y Cirugía. Mexico.
OBJ: Reconocer los principales conceptos y patologías relacionadas con cirugía
TOP: Cirugía KEY: Coledocolitiasis NOT: DR. Victor Quisiguiña
36. ANS: A
El hallazgo más común es una masa o abultamiento en la pared anterior del abdomen que
aumenta de tamaño con la maniobra de Valsalva.

PTS: 3 DIF: ALTA REF: (SCHAWARTZ, 2010)


OBJ: Reconocer los principales conceptos y patologias relacionadas a la cirugia
TOP: CIRUGIA KEY: Hernia abdominal NOT: Dr. Victor Quisiguiña
37. ANS: D
Casi todas las hernias umbilicales congénitas se cierran de manera espontánea alrededor de los
cinco años de edad.

PTS: 3 DIF: ALTA REF: (SCHAWARTZ, 2010)


OBJ: Reconocer los principales conceptos y patologias relacionadas a la cirugia
TOP: CIRUGIA KEY: Hernias umbilicales NOT: Dr. Victor Quisiguiña

7
ID: A

38. ANS: A
Factores que pueden contribuir son obesidad, defectos primarios de cicatrización de heridas,
práctica previa de múltiples métodos, hernias posquirúrgicas previas y errores técnicos durante la
reparación

PTS: 3 DIF: ALTA REF: (SCHAWARTZ, 2010)


OBJ: Reconocer los principales conceptos y patologias relacionadas a la cirugia
TOP: CIRUGIA KEY: Hernia postoperatoria NOT: Dr. Victor Quisiguiña
39. ANS: C
Las hernias de la pared anterior del abdomen, o hernias ventrales, representan defectos de la
aponeurosis y los músculos de la pared abdominal a través de los cuales puede salir contenido
intraabdominal o preperitoneal. Las hernias ventrales pueden ser congénitas o adquiridas. Estas
últimas se forman por un deterioro lento de la estructura de las aponeurosis musculares o se
desarrollan por fracaso de la cicatrización de una incisión en la pared anterior del abdomen
(hernia incisional). El hallazgo más común es una masa o abultamiento en la pared anterior del
abdomen que aumenta de tamaño con la maniobra de Valsalva

PTS: 3 DIF: ALTA REF: (SCHAWARTZ, 2010)


OBJ: Reconocer los principales conceptos y patologias relacionadas a la cirugia
TOP: CIRUGIA KEY: Hernias abdominales NOT: Dr. Victor Quisiguiña
40. ANS: E
Las mallas absorbibles se degradan y eliminan desde los tejidos y pierden poco a poco la
integridad estructural necesaria para el soporte de los tejidos.

PTS: 3 DIF: ALTA REF: (SCHAWARTZ, 2010)


OBJ: Reconocer los principales conceptos y patologias relacionadas a la cirugia
TOP: CIRUGIA KEY: Mallas NOT: Dr. Victor Quisiguiña
41. ANS: C
Las infecciones del sitio quirúrgico se clasifican en infecciones insicionales (Infecciones de piel y
tejido subcutáneo e infecciones profundas) e infecciones de órgano/ espacio.

PTS: 3 DIF: ALTA


REF: Principios de Cirugía de Schwartz 10ma edición capítulo 6 (Infecciones
OBJ: RECONOCER LOS PRINCIPALES CONCEPTOS Y PALABRAS RELACIONADAS A ESTA
PATOLOGÍA TOP: CIRUGÍA KEY: INFECCIÓN
NOT: DR. VICTOR QUISIGUIÑA
42. ANS: A
La peritonitis primaria o espontánea se produce por invasión bacteria via Hematogena y linfática

PTS: 3 DIF: ALTA REF: CTO Séptima edición /Medicina y cirugía / Pag 291
OBJ: RECONOCER LOS PRINCIPALES CONCEPTOS Y PALABRAS RELACIONADAS A ESTA
PATOLOGÍA TOP: CIRUGÍA KEY: PERITONITIS
NOT: DR. VICTOR QUISIGUIÑA

8
ID: A

43. ANS: D
La causa más frecuente es la reactivación de un foco peritoneal (secundario a una diseminación
hematógena o por rotura de una adenopatía mesentérica afecta). Más frecuente en mujeres. Es la
tercera causa de ascitis en nuestro medio, tras la cirrótica y la neoplásica.

PTS: 3 DIF: ALTA REF: (AMIR, digestivo y cirugía general, 3 edición)


OBJ: RECONOCER LOS PRINCIPALES CONCEPTOS Y PALABRAS RELACIONADAS A ESTA
PATOLOGÍA TOP: CIRUGÍA KEY: PERITONITIS TUBERCULOSA
NOT: DR. VICTOR QUISIGUIÑA
44. ANS: D
Sintomas Subdiafragmáticos: disnea, tos, dolor en hombro izquierdo o derecho e hipo por
irritación diafragmática.

PTS: 3 DIF: ALTA REF: (AMIR, digestivo y cirugía general, 3 edición)


OBJ: RECONOCER LOS PRINCIPALES CONCEPTOS Y PALABRAS RELACIONADAS A ESTA
PATOLOGÍA TOP: CIRUGÍA KEY: PERITONITIS
NOT: DR. VICTOR QUISIGUIÑA
45. ANS: A
Debido a que la flora bacteriana que afecta es mixta anaerobia y aerobia; los grampositivos son
caracteristicos de la perforación gástrica, gramnegativos y anaerobios de perforación apendicular
y del colon.

PTS: 3 DIF: ALTA


REF: Manual CTO de Medicina y Cirugía 8.a edición Digestivo y cirugía pag 97
OBJ: RECONOCER LOS PRINCIPALES CONCEPTOS Y PALABRAS RELACIONADAS A ESTA
PATOLOGÍA TOP: CIRUGIA KEY: PERITONITIS SECUNDARIA
NOT: DR. VICTOR QUISIGUIÑA
46. ANS: B
Se define fístula enterocutánea a la comunicación anormal entre el aparato gastrointestinal y la
piel, con salida del contenido intestinal a través de la misma por un periodo mayor de 24 horas.

PTS: 3 DIF: Alta REF: (Martínez-Ordaz, 2002)


OBJ: Reconocer los principales conceptos y patologías relacionados a la cirugía
TOP: CIRUGÍA KEY: Fístula Enterocutánea NOT: Dr. Victor Quisiguiña
47. ANS: D
Las tres complicaciones principales de los pacientes con fistulas son desequilibrio
hidroelectrolítico, desnutrición y sepsis, las cuales están en relación con la
localización de la fistula, el gasto, las características bioquímicas y electrolíticas de la
descarga y la condición patológica subyacente.

PTS: 3 DIF: Alta REF: (Martínez-Ordaz, 2002)


OBJ: Reconocer los principales conceptos y palabras relacionadas a la cirugía
TOP: CIRUGÍA KEY: complicaciones de fístulas NOT: Dr. Victor Quisiguiña

9
ID: A

48. ANS: B
En la tabla 39.1 se reflejan las principales etiologías implicadas en la génesis de las fistulas
enterocutáneas.

PTS: 3 DIF: Alta REF: (PARICIO)


OBJ: Reconocer los principales conceptos y palabras relacionadas a la cirugía
TOP: CIRUGÍA KEY: fistulas enterocutaneas, fistulas postoperatorias
NOT: Dr. Victor Quisiguiña
49. ANS: D PTS: 3 DIF: Alta REF: (Joseph, 2012)
OBJ: Reconocer los principales conceptos y palabras relacionadas a la cirugía
TOP: CIRUGÍA KEY: hernia inguinal estrangulada NOT: Dr. Victor Quisiguiña
50. ANS: C
La sepsis es la complicación más temida en los pacientes con fistulas enterocutáneas.

PTS: 3 DIF: Alta REF: (Martínez-Ordaz, 2002)


OBJ: Reconocer los principales conceptos y palabras relacionadas a la cirugía
TOP: CIRUGÍA KEY: fistulas enterocutáneas NOT: Dr. Victor Quisiguiña
51. ANS: C
Hay abundantes canales de calcio tanto en el músculo cardíaco como el músculo liso. De hecho,
en algunos tipos de músculo liso apenas hay canales rápidos de sodio, de modo que los
potenciales de acción están producidos casi totalmente por la activación de los canales lentos de
calcio.

PTS: 3 DIF: Alto


REF: GUYRON, Arthur; Hall, Jhon: Tratado de fisiología medica 11° edición 2006; Elsevier España
OBJ: reconocer los principales conceptos y patologias relacionadas con la cirugia
TOP: cirugia KEY: canales de calcio NOT: Dr. Victor Quisiguiña
52. ANS: A
El nivel bajo de sodio sérico se produce cuando hay exceso de agua extracelular respecto
del sodio, por lo que puede producir aumento del gasto cardíaco.

PTS: 3 DIF: Alto


REF: SCHWARTZ. Principios de cirugía Mc Graw-Hill. Interamericana. Editores S.A. Séptima edición.
México 2003 OBJ: reconocer las principales conceptos y patologias relacionadas a la cirugia
TOP: Cirugia KEY: Hiponatremia NOT: Dr. Victor Quisiguiña
53. ANS: C
Los síntomas de hipernatremia son raros, a menos que la concentración sérica de sodio sea mayor
de 160 meq/L. Como los síntomas se deben a la hiperosmolaridad, predominan los efectos en el
sistema nervioso central.

PTS: 3 DIF: Alto


REF: SCHWARTZ. Principios de cirugía Mc Graw-Hill. Interamericana. Editores S.A. Séptima edición.
México 2003. pág. 35
OBJ: reconocer los principales conceptos y patologias relacionadas con la cirugia
TOP: Cirugia KEY: Hipernatremia NOT: Dr. Victor Quisiguiña

10
ID: A

54. ANS: D
Los síntomas predominantes de la hiperpotasemia son gastrointestinales, neuromusculares y
cardiovasculares

PTS: 3 DIF: Alto


REF: SCHWARTZ. Principios de cirugía Mc Graw-Hill. Interamericana. Editores S.A. Séptima edición.
México 2003. pág. 36
OBJ: Reconocer las principales conceptos y patologias relacionadas con la cirugia
TOP: Cirugia KEY: hiperpotasemia NOT: Dr. Victor Quisiguiña
55. ANS: B
Las causas de hipopotasemia incluyen ingestión insuficiente, excreción renal excesiva
(hiperaldosteronismo, medicamentos como diuréticos que incrementan la excreción de potasio, o
fármacos como la penicilina que promueven la pérdida tubular renal de potasio), pérdida en
secreciones gastrointestinales (pérdida directa de potasio en heces o pérdida renal de potasio por
vómito o gasto nasogástrico alto) o desviaciones intracelulares (como en la alcalosis metabólica o
el tratamiento con insulina).

PTS: 3 DIF: Alto


REF: SCHWARTZ. Principios de cirugía Mc Graw-Hill. Interamericana. Editores S.A. Séptima edición.
México 2003. pág. 36
OBJ: reconocer los principales conceptos y patologias relacionadas con la cirugia
TOP: Cirugia KEY: hipopotasemia NOT: Dr. Victor Quisiguiña
56. ANS: D
Es la que intenta satisfacer todas las necesidades calórico-proteicas, además de las
hidroelectrolíticas, de todo tipo de pacientes, incluidos los que padecen una agresión grave y una
desnutrición severa.

PTS: 3 DIF: Alto REF: Bosco Mera.2006. AFEME. Ecuador


OBJ: Reconocer los principales conceptos y patologias relacionadas a la cirugia
TOP: Cirugia KEY: Nutricion Parenteral NOT: Dr. Victor Quisiguiña
57. ANS: C
en las actualidad concoermos que la desnutricion hospitalaria nos lleva a mala cicaatrizacion de las
heridas quirurgicas, alteracion de los mecanismos inmunologicos de defensa, aumento de infecciones,
aumento de estancia hospitalaria, mayor mortalidad, lo que incide en aumento en costos hospitalatios

PTS: 3 DIF: Alto REF: ROCABRUNA, R. (13 de marzo de 2009). Scielo.


OBJ: renococer las principales conceptos y patologias relacionadas en la cirugia
TOP: Cirugia KEY: desnutricion NOT: Dr. Victor Quisiguiña

11
ID: A

58. ANS: C
Ayuno: Acto de abstenerse total o parcialmente de comer o beber, a veces por un período de tiempo. Este
puede dividirse en:
• Simple: de 14 a 16 horas sin ingerir alimentos.
• Basal: de 24 a 48 horas sin ingerir alimentos.
• Prolongado: más de 72 horas.
• Inanición: más de 96 horas

PTS: 3 DIF: Alto REF: ROCABRUNA, R. (13 de marzo de 2009). Scielo.


OBJ: reconocer los principales conceptos y patologias relacionadas con la cirugia
TOP: Cirugia KEY: tiempo de ayuno NOT: Dr.Victor Quisiguiña
59. ANS: B
La nutrición parenteral es la modalidad de terapia nutricional en la cual se administran soluciones de
elementos nutritivos en el torrente circulatorio.

PTS: 3 DIF: Alto


REF: MARTINEZ, S. (05 de noviembre de 2012). Nutricion en cirugia.
OBJ: reconocer los principales conceptos y patologias relacionadas en la cirugia
TOP: Cirugia KEY: la nutricion enteral NOT: Dr. Victor Quisiguiña
60. ANS: D
La necesidad basal de potasio es de 80mEg

PTS: 3 DIF: Alto REF: ROCABRUNA, R. (13 de marzo de 2009). Scielo.


OBJ: reconocer los principales conceptos y patologias relacionadas en cirugia
TOP: Cirugia KEY: necesidad basal NOT: Dr. Victor Quisiguiña
61. ANS: A
En las apendicitis el signo de Mc Burney es uno de los signos específicos e importante para poder
diagnosticar en la mediante el examen físico que es una apendicitis

PTS: 3 DIF: Alto REF: (Gasman, 2014)


OBJ: reconocer los principales conceptos y patologias relacionadas en la cirugia
TOP: Cirugia KEY: caso clinico NOT: Dr. Victor Quisiguiña
62. ANS: B
Se va a encontrar los leucocitos mayores a 12000 por presentar la infección y esta hace que sistema de
defensa se altere

PTS: 3 DIF: Alto REF: (Gasman, 2014)


OBJ: reconocer los principales coneptos y patolgogias relaciondas con la cirugia
TOP: Cirugia KEY: Caso clinico NOT: Dr. Victor Quisiguiña

12
ID: A

63. ANS: D
Es un procedimiento para examinar los conductos biliares y se realiza a través de un endoscopio. Los
conductos biliares son las vías que llevan la bilis desde el hígado hasta la vesícula y el intestino delgado.
La CPRE se usa para tratar cálculos, tumores o áreas estrechas de los conductos biliares

PTS: 3 DIF: Alto REF: (SINCHINARO, 2017)


OBJ: reconocer los principales conceptos y patologias relacionadas con la cirugia
TOP: Cirugia KEY: Examen Gold estandar de pancreatitis
NOT: Dr. Victor Quisiguiña
64. ANS: A
En pacientes postoperados con una peritonitis secundaria que no responden a tratamientoy que
presentan fallo multiorganico o sepsis. Por perforación de una visera hueca.

PTS: 3 DIF: alta


REF: Manual CTO de Medicina y Cirugía 8.a edición Digestivo y cirugía pag 97
OBJ: reconocer los principales conceptos y patologias relacionados a la cirugia
TOP: cirugia KEY: peritonitis terciaria NOT: Dr. Victor Quisiguiña
65. ANS: D
Los abscesos intrabdominales son una forma de peritonitis localizada que en su evolución pueden
ser locales vinculados a sigmoiditis aguda, apendicitis, colecistitis ulcera perforada, infeccion del
sitio quirurgico etc

PTS: 3 DIF: alta REF: (Galindo, 2009)


OBJ: reconoser los principales conseptos y patologias relacionadas a la cirugia
TOP: cirugia KEY: absceso intrabdominal NOT: Dr.Victor Quisiguiña
66. ANS: C
Hemoglobina y pigmentos biliares en cavidad peritoneal aumenta la proliferación bacteriana y los
pigmentos bacterianos disminuyen la tensión superficial del peritoneo.

PTS: 3 DIF: alta REF: (Suarez, 2004)


OBJ: reconoser los principales conseptos y patologias relacionadas a la cirugia
TOP: cirugia KEY: absceso intraabdominal NOT: Dr.Victor Quisiguiña
67. ANS: D
Según su localización
a) Intraperitoneal
- Visceral: hepático, esplénico, vesícula biliar, ovario, trompas y útero.
- De los espacios anatómicos existentes (supramesocólicos: subfrénico, subhepático y trascavidad
de los epiplones e inframesocólicos: interasas
- Del espacio de un órgano extirpado.
b) Retroperitoneal
- Visceral: riñón, páncreas.
- De los espacios anatómicos preexistentes
- Del espacio de un órgano extirpado.

PTS: 3 DIF: alta REF: (Garcia, 2014)


OBJ: reconoser los proncipales conseptos y patologias relacionadas a la cirugia
TOP: cirugia KEY: intraperitoneal NOT: Dr.Victor Quisiguiña

13
ID: A

68. ANS: D
Las complicaciones comprenden:

 Reaparición del absceso

 Ruptura de un absceso

 Diseminación de la infección al torrente sanguíneo

 Infección generalizada en el abdomen

PTS: 3 DIF: alta REF: (Sanchez, 2012)


OBJ: reconoser los proncipales conseptos y patologias relacionadas a la cirugia
TOP: cirugia KEY: absceso intraabdominal NOT: Dr. Victor Quisiguiña
69. ANS: A
La hernia inguinal incarcerada se manifiesta con bulto doloroso en la región inguinal y cuadro
obstructivo; se puede intentar la reducción de una hernia inguinal incarcerada bajo sedación
suave (diazepam IM o IV), pero nunca de una hernia crural, pues puede poner en riesgo el asa
intestinal.

PTS: 3 DIF: ALTO REF: (Rodríguez & Merino, 2014)


OBJ: RECONOCER LOS PRINCIPALES CONCEPTOS Y PALABRAS RELACIONADAS A ESTA
PATOLOGÍA TOP: CIRUGÍA KEY: HERNIA INGUINAL
NOT: DR. VICTOR QUISUGUIÑA
70. ANS: D
La hernia estrangulada se manifiesta con bulto doloroso, caliente, eritematoso o azulado, que
puede crepitar, en la región inguinal y obstrucción intestinal; puede acompañarse de fiebre,
leucocitosis y signos de sepsis.
Toda hernia estrangulada por el riesgo que conlleva reintroducir un segmento intestinal con
compromiso vascular, debe ser intervenida sin intentar reducirla.

PTS: 3 DIF: ALTO REF: (Rodríguez & Merino, 2014)


OBJ: RECONOCER LOS PRINCIPALES CONCEPTOS Y PALABRAS RELACIONADAS A ESTA
PATOLOGÍA TOP: CIRUGÍA KEY: HERNIA ESTRANGULADA
NOT: DR. VICTOR QUISIGUIÑA
71. ANS: A
La hernia directa protruye a través del suelo del canal inguinal a nivel del triángulo de
Hesselbach, que está formado por la fascia transversalis reforzada por fibras aponeuróticas del
músculo transverso del abdomen. Así pues, estas hernias no pasan a través del orificio profundo y
no se localizan por dentro de las fibras del cremáster, sino por detrás.

PTS: 3 DIF: ALTA REF: (Rodríguez & Merino, 2014)


OBJ: RECONOCER LOS PRINCIPALES CONCEPTOS Y PALABRAS RELACIONADAS A ESTA
PATOLOGÍA TOP: CIRUGÍA KEY: HERNIA DIRECTA
NOT: DR, VICTOR QUISIGUIÑA

14
ID: A

72. ANS: D
Hernioplastia (reparación protésica): reparación de la hernia con material sintético. Actualmente
se realizan con mayor frecuencia, dados los excelentes resultados obtenidos (técnicas de
Lichtenstein, Rutkow, entre otros).

PTS: 3 DIF: ALTA REF: (Rodríguez & Merino, 2014)


OBJ: RECONOCER LOS PRINCIPALES CONCEPTOS Y PALABRAS RELACIONADAS A ESTA
PATOLOGÍA TOP: CIRUGÍA KEY: HERNIOPLASTIA
NOT: DR. VICTOR QUISIGUIÑA
73. ANS: C
La biometría hemática y química sanguínea nos sirve para saber la cantidad tanto de hemoglobina
que tiene el paciente y el caso de esta estar baja reponer para compensar al paciente al igual que
electrolitos

PTS: 3 DIF: ALTA REF: SOTO. S. (2005). HERNIAS. CTO.


OBJ: Reconocer los principales conceptos y patologías relacionadas a la cirugia
TOP: CIRUGIA GENERAL KEY: Hernia NOT: DR. VICTOR QUISIGUIÑA
74. ANS: B
En general el traumatismo abierto es producido por elementos cortantes que dañan la continuidad
del tejido, a diferencia del cerrado que es lo contrario

PTS: 3 DIF: ALTA REF: SOTO. S. (2005). HERNIAS. CTO


OBJ: Reconocer los principales conceptos y patologias relacionados con la cirugía
TOP: CIRUGIA KEY: Hernias NOT: DR. VICTOR QUISIGUIÑA
75. ANS: C
Son aquellas que presentan un riesgo de necrosis por compromiso vascular e isquemia de la
víscera herniada. Normalmente el trastorno del riego acompaña a una incarceración pero no
siempre es así, como ocurre en la hernia de Richter (sólo se estrangula un sector limitado de la
porción antimesentérica) y en la hernia de Littre (la luz intestinal está permeable porque existe
una estrangulación del divertículo de Meckel). Es la complicación más grave de una hernia
pudiendo llegar a acaba con la vida del paciente

PTS: 3 DIF: ALTA REF: SOTO. S. (2005). HERNIAS. CTO.


OBJ: Reconocer los principales conceptos y patologías relacionadas con cirugia
TOP: CIRUGIA KEY: Hernias NOT: DR. VICTOR QUISIGUIÑA

15
ID: A

76. ANS: D
Un pólipo es una protrusión macroscópica de la pared intestinal.

Polipectomía y seguimiento endoscópica: Benignos y malignízados intramucosos. Los pólipos


adenomatosos necesitan de más de cinco años de crecimiento antes de alcanzar expresión clínica
por lo que no es necesario repetir las colonoscopias antes de tres años después de la polipectomía.
Hay lesiones sincrónicas y metacronicas lo que obliga a una primera colonoscopia total tras
extirpación de un pólipo.

Si se ha extraído un adenoma avanzado se recomienda efectuar una colonoscopia a los 3 años,


por el contrario, si tan sola se detectaron l ó2 adenomas no avanzados el intervalo entre
exploraciones puede ser entre 5- 10 años.

PTS: 3 DIF: ALTA REF: SOTO.S. (2004). POLIPOS . CTO.


OBJ: Reconocer los conceptos y patologías relacionadas a la cirugia
TOP: CIRUGIA KEY: Polipos NOT: DR. VICTOR QUISIGUIÑA
77. ANS: B
Ocurre obstrucción intestinal cuando hay interferencia en la progresión normal del contenido
intestinal. Se denomina obstrucción mecánica cuando existe una verdadera barrera física que
obstruye la luz intestinal. La localización más frecuente es el intestino delgado. Se caracteriza por
olor abdominal, vómitos (fecaloideos si es distal), distensión abdominal (mayor cuanto más distal
sea la obstrucción), hiperperistaltismo con ruidos metálicos.

PTS: 3 DIF: Alta REF: (SOTO.S, 2005)


OBJ: Reconocer los principales conceptos y patologías relacionadas a la cirugía
TOP: Abdomen agudo obstructivo KEY: Obstrucción intestinal
NOT: Dr. Victor Quisiguiña
78. ANS: C
El 90% de las obstrucciones de ID se resuelven con aNG y reposición hidroelectrolítica. Es
necesaria la intervención quirúrgica cuando sospechamos estrangulación, el dolor y la fiebre
aumentan, y si no se resuelve en un plazo de 3 – 5 días.

Laparotomía indica sección o incisión del flanco, vacío o de las partes blandas colocadas por
debajo de las costillas. Laparotomía exploradora es pues la incisión pura y simple de las paredes
del abdomen con el objeto de llegar a formular un diagnóstico

PTS: 3 DIF: Alta REF: (SOTO.S, 2005)


OBJ: Reconocer los principales conceptos y patologías relacionados a la cirugía
TOP: Abdomen Agudo obstructivo KEY: Laparotomía NOT: Dr. Víctor Quisiguiña

16
ID: A

79. ANS: A
El vólvulo de sigma es la localización más frecuente. La torsión se produce en sentido
antihorario. Se manifiesta por dolor abdominal, ausencia de flatos y distensión abdominal
asimétrica. En la radiografía de abdomen se aprecia una importante dilatación colónica con una
imagen “asa en omega” o en “grano de café

PTS: 3 DIF: ALTA REF: (SOTO; 2005)


OBJ: Reconocer los principales conceptos y patologías relacionados a la cirugía
TOP: Abdomen Agudo Obstructivo KEY: Dilatación Colonica
NOT: VICTOR QUISIGUIÑA
80. ANS: A
La clasificación del cáncer de ovario se hace mediante laparotomía exploradora, por lo que la
cirugía se realizará a todas las mujeres. En la mayoría de los casos, se encontrarán tumores en
estadios avanzados, ya que la vía de diseminación es la implantación directa por siembra
peritoneal de células tumorales

PTS: 3 DIF: Alta REF: (SOTO,2005)


OBJ: Reconocer los principales conceptos y patologías relacionados a la cirugía
TOP: ABDOMEN GINECOLOGICO KEY: Lesión quística
NOT: Víctor Quisiguiña
81. ANS: C
El diagnóstico de embarazo ectópico se realiza fundamentalmente mediante ecografía, al
visualizar una cavidad uterina vacía y una imagen anexial sugestiva/diagnóstica de gestación. En
caso de duda, es posible ayudarse con la determinación de HCG. En el embarazo ectópico
presentará niveles más bajos, y además aumentará más lentamente que en el embarazo normal

PTS: 3 DIF: ALTA REF: (SOTO;S, 2005)


OBJ: Reconocer los principales conceptos y patologías relacionados a la cirugía
TOP: ABDOMEN GINECOLOGICO KEY: QUISTE NOT: VICTOR QUISIGUIÑA
82. ANS: D
El dolor es el síntoma más frecuente y característico. Generalmente se manifiesta en forma de
dismenorrea progresiva que no cede con la toma de antiinflamatorios ni con anticonceptivos
hormonales, otros síntomas que pueden aparecer son alteraciones menstruales rectorragia, disuria

PTS: 3 DIF: ALTA REF: (SOTO.S, 2005)


OBJ: Reconocer los principales conceptos y patologías relacionados a la cirugía
TOP: ABDOMEN GINECOLOGICO KEY: Dismenorrea importante
NOT: VICTOR QUISIGUIÑA
83. ANS: C
En la enfermedad inflamatoria pélvica, el tratamiento de elección en fases precoces para prevenir
secuelas, combina de 2 ó 3 antibióticos de amplio espectro, así pues, ceftriaxona y doxiciclina.

PTS: 3 DIF: ALTA REF: (SOTO.S, 2005).


OBJ: Reconocer los principales conceptos y patologías relacionados a la cirugía
TOP: ABDOMEN GINECOLOGICO KEY: ENFERMEDAD PELVICA INFLAMATORIA
NOT: VICTOR QUISIGUIÑA

17
ID: A

84. ANS: A
Las manifestaciones clínicas de la torsión ovárica son dolor intenso de inicio súbito localizado en
el hemiabdomen inferior, generalmente lateralizado con signos de irritación peritoneal, con o sin
masa palpable. Náusea y vómito, junto con alza térmica leve, a la ecografía se visualiza el signo
de remolino que se describe inicialmente en la rotación del mesenterio en el vólvulo intestinal y
luego se adapta a la torsión anexial, representa del pedículo vascular rotado, un vaso arterial
central rodeado por un vaso venoso.

PTS: 3 DIF: ALTA REF: (SOTO.S, 2005)


OBJ: Reconocer los principales conceptos y patologías relacionados a la cirugía
TOP: ABDOMEN GINECOLOGICO KEY: Sensible en la fosa iliaca derecha
NOT: VICTOR QUISIGUIÑA
85. ANS: B
Tenemos un paciente de edad avanzada y comorbilidades importantes que pueden limitar desde
un punto de vista anestésico la cirugía abdominal por la alta mortalidad durante la cirugía. Tras la
ausencia de mejoría con medidas de resucitación y antibioticoterapia, la medida mas segura de
lograr una posible mejoría del paciente es descomprimir la vesícula mediante una colecistostomia
guiada radiológicamente por ecografía o fluoroscopia

PTS: 3 DIF: ALTA REF: (AMIR, 2011)


OBJ: Reconocer los principales conceptos y patologías relacionados a la cirugía
TOP: Colecistitis KEY: Colecistitis NOT: VICTOR QUISIGUIÑA
86. ANS: D
El cuadro descrito es el clásico de un Ampuloma. Tenga en cuenta que el dato clave para el
diagnostico diferencial es la sangre en heces, que en un principio no estaría presente en el resto de
las opciones. También se habla de heces plateadas, debido a las melenas acolicas que puede
provocar el Ampuloma.

PTS: 3 DIF: ALTA REF: (AMIR, 2010)


OBJ: Reconocer los principales conceptos y patologías relacionados a la cirugía
TOP: ABDOMEN OBSTRUCTIVO KEY: SANGRE OCULTA EN HECES
NOT: VICTOR QUISIGUIÑA
87. ANS: A
La ecografía es la primera prueba diagnostica ante un cuadro de posible obstrucción de las vías
biliares, y según los resultados de la misma se valoraría CPRE (cuando se puede hacer
terapéutica, ya que es invasiva) o colangio RMN.

PTS: 3 DIF: ALTA REF: (MIR, 2014)


OBJ: Reconocer los principales conceptos y patologías relacionados a la cirugía
TOP: CIRUGIA KEY: OBSTRUCCION NOT: DR. VICTOR QUISIGUIÑA
88. ANS: D
Los cálculos de colesterol se producen por aumento del colesterol en la bilis que forman grumos
en la vesícula, algunos factores predisponen a la formación del cálculo: el mal vaciamiento de la
vesícula, obesidad, sexo femenino, edad avanzada, embarazo, algunas enfermedades hepáticas,
rápida pérdida de peso, dietas ricas en grasa.

PTS: 3 DIF: ALTA REF: (HospitalEugenioEspejo, 2014)


TOP: CIRUGIA KEY: COLELITIASIS NOT: DR. VICTOR QUISIGUIÑA

18
ID: A

89. ANS: D
Se trata de una pregunta de fácil contestación si se piensa en causas anatómicas para explicar la
respuesta. El conducto cístico desemboca en el colédoco una vez que los conductos hepáticos
principales se han fusionado por lo que un tumor que se origina por encima de la desembocadura
del conducto cístico no puede ocasionar distensión vesical por colestasis obstructiva.

PTS: 3 DIF: ALTA REF: (AMIR, 2011)


OBJ: Reconocer los principales conceptos y patologías relacionados a la cirugía
TOP: CIRUGIA KEY: VIA BILIAR NOT: DR. VICTOR QUISIGUIÑA
90. ANS: A
Como se conoce, las principales ventajas del abordaje laparoscópico son la reducción de la
estancia postoperatoria, la reducción del dolor postoperatorio, y una mejor recuperación funcional
postquirúrgica.

PTS: 3 DIF: ALTO REF: (AMIR, 2011)


OBJ: Reconocer los principales conceptos y patologías relacionados a la cirugía
TOP: CIRUGIA KEY: COLECISTECTOMIA NOT: DR. VICTOR QUISIGUIÑA
91. ANS: D
La lecitina es un fosfolípido presente en la secreción biliar que sirve para solubilizar los ácidos
biliares. El resto de opciones son enzimas presentes en la secreción pancreática.

PTS: 3 DIF: ALTO REF: (AMIR, 2011)


OBJ: Reconocer los principales conceptos y patologías relacionados a la cirugía
TOP: CIRUGIA KEY: BILIS NOT: DR. VICTOR QUISIGUIÑA
92. ANS: B
Si bien la colelitiasis sintomática es siempre tributaria de tratamiento, la conducta que ha de
seguir cuando es asintomática debe ser todo lo conservadora posible. Numerosos estudios han
demostrado que pacientes con colelitiasis asintomática siguen en general un curso benigno sin
tratamiento y que el riesgo de desarrollar complicaciones es pequeña y no supera el de la cirugía
de modo que en principio no se debería intervenir al paciente de la pregunta. Debemos recordar
ciertas excepciones en donde aun siendo asintomática debemos intervenir al paciente calculo
grandes(>2,5cm) anomalías congénitas, diabéticos, y los pacientes con calcificación de la pared
de la vesícula biliar porque se asocia con mayor riesgo de carcinoma.

PTS: 3 DIF: ALTA REF: (AMIR, AMIR TEST, 2016)


OBJ: Reconocer los principales conceptos y patologías relacionados a la cirugía
TOP: CIRUGÌA KEY: CASO NOT: DR. VICTOR QUISIGUIÑA
93. ANS: B
La colangitis presenta la denominada triada de Charcot (fiebre, ictericia y dolor abdominal). Cada
vez se utiliza más la vía endoscópica para el diagnóstico y tratamiento de la patología biliar. La
opción dos es la Falsa, el cáncer de colédoco no se considera una causa frecuente de colangitis

PTS: 3 DIF: ALTA REF: (ARREOS.M; AMIR.ESPAÑA: MARBAN, 2012)


OBJ: Reconocer los principales conceptos y patologías relacionados a la cirugía
TOP: CIRUGÌA KEY: COLANGITIS NOT: DR. VICTOR QUISIGUIÑA

19
ID: A

94. ANS: D
Los hallazgos de laboratorio revelan un aumento de la fosfatasa alcalina, en general con valores
que superan 3 veces a los normales. Una elevación de la fosfatasa alcalina en un paciente con
enfermedad inflamatoria intestinal puede sugerir el diagnóstico de colangitis esclerosante, pero
no es un requisito indispensable, ya que un 8.5% de los pacientes la fosfatasa alcalina es normal
en el momento del diagnóstico de la enfermedad hepática. También se observa un aumento
moderado de las transaminasas. En el 60% de los casos, la bilirrubinemia es normal en el
momento del diagnóstico, así como la albuminemia y la tasa de protrombina. Puede observarse
un aumento de las inmunoglobulinas en el 61% de los casos, preferen- temente de la IgG. La IgM
está aumentada en menos ocasiones (20-45%). Los anticuerpos antimitocondriales son negativos
y, desde el punto de vista inmunológico, el dato más característico es la detección

PTS: 3 DIF: ALTA REF: (DIGESTIVO-MANUAL CTO 7a EDICIÓN, 2010)


OBJ: Reconocer los principales conceptos y patologías relacionados a la cirugía
TOP: CIRUGIA KEY: CASO NOT: DR. VICTOR QUISIGUIÑA
95. ANS: B
La presentación habitual incluye fiebre, dolor en el epigastrio o el cuadrante superior derecho e
ictericia. Estos síntomas comunes, que se conocen como la tríada de Charcot, se identifican en
casi dos tercios de los enfermos.

PTS: 3 DIF: ALTA


REF: (SCHWARTZ PRINCIPIOS DE CIRUGIIA 10A EDICION, 2010)
OBJ: Reconocer los principales conceptos y patologías relacionados a la cirugía
TOP: CIRUGÌA KEY: COLÀNGITIS AGUDA NOT: DR. VICTOR QUISIGUIÑA
96. ANS: A
Son comunes leucocitosis, hiperbilirrubinemia y aumento de la fosfatasa alcalina y de las
transaminasas; cuando se presentan sugieren el diagnóstico clínico de colangitis. Es útil una
ecografía, ya que revela la presencia de cálculos en la vesícula biliar y demostrará conductos
dilatados y tal vez señala el sitio de obstrucción; sin embargo, rara vez dilucida exactamente la
causa.

PTS: 3 DIF: ALTA


REF: (SCHWARTZ PRINCIPIOS DE CIRUGIIA 10A EDICION, 2010)
OBJ: Reconocer los principales conceptos y patologías relacionados a la cirugía
TOP: CIRUGÌA KEY: COLANGITIS NOT: DR. VICTOR QUISIGUIÑA
97. ANS: C
El tratamiento inicial de la colangitis incluye antibióticos y reanimación con líquidos por vía
intravenosa. Estos enfermos tal vez ameriten vigilancia en la unidad de cuidados intensivos y
apoyo vasopresor. Casi todos los enfermos responden a estas medidas. No obstante, es necesario
drenar el conducto biliar obstruido tan pronto se estabilice el paciente.

PTS: 3 DIF: ALTA


REF: (SCHWARTZ PRINCIPIOS DE CIRUGIIA 10A EDICION, 2010)
OBJ: Reconocer los principales conceptos y patologías relacionados a la cirugía
TOP: CIRUGIA KEY: CCOLANGITIS NOT: DR. VICTOR QUISIGUIÑA

20
ID: A

98. ANS: A
La pancreatitis aguda es una enfermedad inflamatoria del páncreas que no se acompaña de
fibrosis de la glándula, o muy poca.

PTS: 3 DIF: ALTA REF: (SCWARTZ, XXX)


OBJ: Reconocer los principales conceptos y patologías relacionados a la cirugía
TOP: CIRUGÌA KEY: APENDICITIS NOT: DR. VICTOR QUISIGUIÑA

21
CIRUGIA 2 [Answer Strip] ID: A

C
_____ 4. A
_____ 7. D 11.
_____ B 14.
_____

B
_____ 1.

C 12.
_____

D
_____ 2. D
_____ 8.

A
_____ 5.
C 15.
_____

B 13.
_____
D
_____ 3.
C
_____ 9.

B 16.
_____

C
_____ 6.

C 10.
_____
CIRUGIA 2 [Answer Strip] ID: A

C 17.
_____ C 21.
_____ D 26.
_____ B 30.
_____ A 33.
_____

D 22.
_____
C 27.
_____
A 18.
_____

D 31.
_____

D 23.
_____
D 28.
_____
B 19.
_____
C 34.
_____

D 29.
_____
D 32.
_____
D 35.
_____
B 24.
_____
C 20.
_____

A 36.
_____

D 25.
_____
CIRUGIA 2 [Answer Strip] ID: A

D 37.
_____ C 41.
_____ B 46.
_____ C 51.
_____ D 56.
_____

C 57.
_____
D 47.
_____

A 42.
_____
A 38.
_____
A 52.
_____
C 58.
_____
B 48.
_____

D 43.
_____

D 49.
_____
C 53.
_____ B 59.
_____

C 39.
_____

D 44.
_____

D 54.
_____

C 50.
_____
D 60.
_____

A 45.
_____
E 40.
_____
B 55.
_____
CIRUGIA 2 [Answer Strip] ID: A

A 61.
_____ C 66.
_____ A 71.
_____ C 75.
_____ C 78.
_____

D 67.
_____

B 62.
_____
D 72.
_____

D 76.
_____

D 68.
_____ A 79.
_____

C 73.
_____

D 63.
_____

A 69.
_____

B 77.
_____
A 80.
_____
A 64.
_____

D 70.
_____

B 74.
_____

D 65.
_____
CIRUGIA 2 [Answer Strip] ID: A

C 81.
_____ A 84.
_____ A 87.
_____ B 92.
_____ D 94.
_____

D 88.
_____

D 82.
_____

D 89.
_____

B 95.
_____

A 90.
_____ B 93.
_____
C 83.
_____ B 85.
_____

D 91.
_____

D 86.
_____
CIRUGIA 2 [Answer Strip] ID: A

A 96.
_____ A 98.
_____

C 97.
_____

También podría gustarte